52
February 2013 MEE Questions and Analyses National Conference of Bar Examiners 302 South Bedford Street | Madison, WI 53703-3622 Phone: 608-280-8550 | Fax: 608-280-8552 | TDD: 608-661-1275 www.ncbex.org e-mail: [email protected]

February 2013 MEE Questions and Analyses

  • Upload
    others

  • View
    1

  • Download
    0

Embed Size (px)

Citation preview

Page 1: February 2013 MEE Questions and Analyses

February 2013 MEE Questions and Analyses

National Conference of Bar Examiners 302 South Bedford Street | Madison WI 53703-3622 Phone 608-280-8550 | Fax 608-280-8552 | TDD 608-661-1275

wwwncbexorg e-mail contactncbexorg

Copyright copy 2013 by the National Conference of Bar Examiners All rights reserved

Contents

Preface ii

Description of the MEE ii

Instructions iii

February 2013 Questions

Real Property Question 3

Contracts Question 4

Constitutional Law Question 5

Secured Transactions Question 6

Federal Civil Procedure Question 7

Agency Question 8

Evidence Question 10

Trusts and Future Interests Question 11

Negotiable Instruments Question 12

February 2013 Analyses

Real Property Analysis 15

Contracts Analysis 19

Constitutional Law Analysis 21

Secured Transactions Analysis 24

Federal Civil Procedure Analysis 27

Agency Analysis 31

Evidence Analysis 35

Trusts and Future Interests Analysis 38

Negotiable Instruments Analysis 41

Used as one of the six questions on the February 2013 Uniform Bar Examination in Alabama Arizona Colorado Idaho Missouri Nebraska North Dakota and Utah

i

Preface

The Multistate Essay Examination (MEE) is developed by the National Conference of Bar Examiners (NCBE) This publication includes the questions and analyses from the February 2013 MEE Each test includes nine 30-minute questions user jurisdictions may elect which of the nine questions they wish to use (Jurisdictions that administer the Uniform Bar Examination [UBE] use a common set of six MEE questions as part of their bar examinations) In the actual test the questions are simply numbered rather than being identified by area of law The instructions for the test appear on page iii For more information see the MEE Information Booklet available on the NCBE website at wwwncbexorg

The model analyses for the MEE are illustrative of the discussions that might appear in excellent answers to the questions They are provided to the user jurisdictions to assist graders in grading the examination They address all the legal and factual issues the drafters intended to raise in the questions

The subjects covered by each question are listed on the first page of its accompanying analysis followed by roman numerals that refer to the MEE subject matter outline for that subject For example the Federal Civil Procedure question on the February 2013 MEE tested the following area from the Federal Civil Procedure outline VIE Verdicts and judgmentsmdashEffect claim and issue preclusion Subject matter outlines are included in the MEE Information Booklet

Description of the MEE

The MEE consists of nine 30-minute essay questions any of which a jurisdiction may select to include as part of its bar examination (UBE jurisdictions use a common set of six MEE questions as part of their bar examinations) It is administered by participating jurisdictions on the Tuesday before the last Wednesday in February and July of each year The areas of law that may be covered by the questions on any MEE are Business Associations (Agency and Partnership Corporations and Limited Liability Companies) Conflict of Laws Constitutional Law Contracts Criminal Law and Procedure Evidence Family Law Federal Civil Procedure Real Property Torts Trusts and Estates (Decedentsrsquo Estates Trusts and Future Interests) and Uniform Commercial Code (Negotiable Instruments and Bank Deposits and Collections Secured Transactions) Some questions may include issues in more than one area of law The particular areas covered vary from exam to exam

The purpose of the MEE is to test the examineersquos ability to (1) identify legal issues raised by a hypothetical factual situation (2) separate material which is relevant from that which is not (3) present a reasoned analysis of the relevant issues in a clear concise and well-organized composition and (4) demonstrate an understanding of the fundamental legal principles relevant to the probable solution of the issues raised by the factual situation The primary distinction between the MEE and the Multistate Bar Examination (MBE) is that the MEE requires the examinee to demonstrate an ability to communicate effectively in writing

ii

_____

Instructions

The back cover of each test booklet contains the following instructions

You will be instructed when to begin and when to stop this test Do not break the seal on this booklet until you are told to begin

You may answer the questions in any order you wish Do not answer more than one question in each answer booklet If you make a mistake or wish to revise your answer simply draw a line through the material you wish to delete

If you are using a laptop computer to answer the questions your jurisdiction will provide you with specific instructions

Read each fact situation very carefully and do not assume facts that are not given in the question Do not assume that each question covers only a single area of the law some of the questions may cover more than one of the areas you are responsible for knowing

Demonstrate your ability to reason and analyze Each of your answers should show an understanding of the facts a recognition of the issues included a knowledge of the applicable principles of law and the reasoning by which you arrive at your conclusion The value of your answer depends not as much upon your conclusions as upon the presence and quality of the elements mentioned above

Clarity and conciseness are important but make your answer complete Do not volunteer irrelevant or immaterial information

Answer all questions according to generally accepted fundamental legal principles unless your jurisdiction has instructed you to answer according to local case or statutory law (UBE instructions Answer all questions according to generally accepted fundamental legal principles rather than local case or statutory law)

iii

February 2013 MEE

QUESTIONS

Real Property Contracts

Constitutional Law Secured Transactions

Federal Civil Procedure Agency

Evidence Trusts and Future Interests

Negotiable Instruments

REAL PROPERTY QUESTION _______________

In 2008 a landlord and a tenant entered into a 10-year written lease commencing September 1 2008 for the exclusive use of a commercial building at a monthly rent of $2500 The lease contained a covenant of quiet enjoyment but no other covenants or promises on the part of the landlord

When the landlord and tenant negotiated the lease the tenant asked the landlord if the building had an air-conditioning system The landlord answered ldquoYes it doesrdquo The tenant responded ldquoGreat I will be using the building to manufacture a product that will be irreparably damaged if the temperature during manufacture exceeds 81 degrees for more than six consecutive hoursrdquo

On April 15 2012 the buildingrsquos air-conditioning system malfunctioned causing the building temperature to rise above 81 degrees for three hours The tenant immediately telephoned the landlord about this malfunction The tenant left a message in which he explained what had happened and asked the landlord ldquoWhat are you going to do about itrdquo The landlord did not respond to the tenantrsquos message

On May 15 2012 the air-conditioning system again malfunctioned This time the malfunction caused the building temperature to rise above 81 degrees for six hours The tenant telephoned the landlord and left a message describing the malfunction As before the landlord did not respond

On August 24 2012 the air-conditioning system malfunctioned again causing the temperature to rise above 81 degrees for 10 hours Again the tenant promptly telephoned the landlord The landlord answered the phone and the tenant begged her to fix the system The landlord refused The tenant then attempted to fix the system himself but he failed As a result of the air-conditioning malfunction products worth $150000 were destroyed

The next day the tenant wrote the following letter to the landlord

Irsquove had enough I told you about the air-conditioning problem twice before yesterdayrsquos disaster and you failed to correct it I will vacate the building by the end of the month and will bring you the keys when I leave

The tenant vacated the building on August 31 2012 and returned the keys to the landlord that day At that time there were six years remaining on the lease

On September 1 2012 the landlord returned the keys to the tenant with a note that said ldquoI repeat the air-conditioning is not my problem You have leased the building and you should fix itrdquo The tenant promptly sent the keys back to the landlord with a letter that said ldquoI have terminated the lease and I will not be returning to the building or making further rent paymentsrdquo After receiving the keys and letter the landlord put the keys into her desk To date she has neither responded to the tenantrsquos letter nor taken steps to lease the building to another tenant

On November 1 2012 two months after the tenant vacated the property the landlord sued the tenant claiming that she is entitled to the remaining unpaid rent ($180000) from September 1 for the balance of the lease term (reduced to present value) or if not that then damages for the tenantrsquos wrongful termination

Is the landlord correct Explain

3

CONTRACTS QUESTION _______________

On January 2 a boat builder and a sailor entered into a contract pursuant to which the builder was to sell to the sailor a boat to be specially manufactured for the sailor by the builder The contract price was $100000 The written contract signed by both parties stated that the builder would tender the boat to the sailor on December 15 at which time payment in full would be due

On October 15 the builderrsquos workers went on strike and there were no available replacements

On October 31 the builderrsquos workers were still on strike and no work was being done on the boat The sailor read a news report about the strike and immediately sent a letter to the builder stating ldquoI am very concerned that my boat will not be completed by December 15 I insist that you provide me with assurance that you will perform in accordance with the contractrdquo The builder received the letter on the next day November 1

On November 25 the builder responded to the letter stating ldquoIrsquom sorry about the strike but it is really out of my hands I hope we settle it soon so that we can get back to workrdquo

Nothing further happened until December 3 when the builder called the sailor and said ldquoMy workers are back and I have two crews working overtime to finish your boat Your boat is task one Donrsquot worry wersquoll deliver your boat by December 15thrdquo The sailor immediately replied ldquoI donrsquot trust you As far as Irsquom concerned our contract is over I am going to buy my boat from a shipyardrdquo Two days later the sailor entered into a contract with a competing manufacturer to buy a boat similar to the boat that was the subject of the contract with the builder

The builder finished the boat on time and tendered it to the sailor on December 15 The sailor reminded the builder about the December 3 conversation in which the sailor had announced that ldquoour contract is overrdquo and refused to take the boat and pay for it

The builder has sued the sailor for breach of contract

1 What was the legal effect of the sailorrsquos October 31 letter to the builder Explain

2 What was the legal effect of the builderrsquos November 25 response to the sailorrsquos October 31 letter Explain

3 What was the legal effect of the sailorrsquos refusal to take and pay for the boat on December 15 Explain

4

CONSTITUTIONAL LAW QUESTION

AutoCo is a privately owned corporation that manufactures automobiles Ten years ago AutoCo purchased a five-square-mile parcel of unincorporated land in a remote region of the state and built a large automobile assembly plant on the land To attract workers to the remote location of the plant AutoCo built apartment buildings and houses on the land and leased them to its employees AutoCo owns and operates a commercial district with shops and streets open to the general public AutoCo named the area Oakwood and provides security fire protection and sanitation services for Oakwoodrsquos residents AutoCo also built operates and fully funds the only school in the region which it makes available free of charge to the children of its employees

A family recently moved to Oakwood The father and mother work in AutoCorsquos plant rent an apartment from AutoCo and have enrolled their 10-year-old son in Oakwoodrsquos school Every morning the students are required to recite the Pledge of Allegiance while standing and saluting an American flag With the approval of his parents the son has politely but insistently refused to recite the Pledge and salute the flag at the school on the grounds that doing so violates his own political beliefs and the political beliefs of his family As a result of his refusal to say the Pledge the son has been expelled from the school

To protest the schoolrsquos actions the father walked into the commercial district of Oakwood While standing on a street corner he handed out leaflets that contained a short essay critical of the schoolrsquos Pledge of Allegiance policy Some of the passersby who took the leaflets dropped them to the ground An AutoCo security guard saw the litter told the father that Oakwoodrsquos anti-litter rule prohibits leaflet distribution that results in littering and directed him to cease distribution of the leaflets and leave the commercial district When the father did not leave and continued to distribute the leaflets the security guard called the state police which sent officers who arrested the father for trespass

1 Did the sonrsquos expulsion from the school violate the First Amendment as applied through the Fourteenth Amendment Explain

2 Did the fatherrsquos arrest violate the First Amendment as applied through the Fourteenth Amendment Explain

5

SECURED TRANSACTIONS QUESTION

On June 1 a bicycle retailer sold two bicycles to a man for a total purchase price of $1500 The man made a $200 down payment and agreed to pay the balance in one year The man also signed a security agreement that identified the bicycles as collateral for the unpaid purchase price and provided that the man ldquoshall not sell or dispose of the collateral until the balance owed is paid in fullrdquo The retailer never filed a financing statement reflecting this security interest

The man had bought the bicycles for him and his girlfriend to use on vacation However shortly after he bought the bicycles the man and his girlfriend broke up The man has never used the bicycles

On August 1 the man sold one of the bicycles at a garage sale to a buyer who paid the man $400 for the bicycle The buyer bought the bicycle to ride for weekend recreation

On October 1 the man gave the other bicycle to his friend as a birthday present The friend began using the bicycle for morning exercise

Neither the buyer nor the friend had any knowledge of the manrsquos dealings with the retailer

1 Does the buyer own the bicycle free of the retailerrsquos security interest Explain

2 Does the friend own the bicycle free of the retailerrsquos security interest Explain

6

FEDERAL CIVIL PROCEDURE QUESTION _____

Mother and Son who are both adults are citizens and residents of State A Mother owned an expensive luxury car valued in excess of $100000 Son borrowed Motherrsquos car to drive to a store in State A As Son approached a traffic light that had just turned yellow he carefully braked and brought the car to a complete stop Driver who was following immediately behind him failed to stop and rear-ended Motherrsquos car which was damaged beyond repair Son was seriously injured Driver is a citizen of State B

Son sued Driver in the United States District Court for the District of State A alleging that she was negligent in the operation of her vehicle Son sought damages in excess of $75000 for his personal injuries exclusive of costs and interest In her answer Driver alleged that Son was contributorily negligent in the operation of Motherrsquos car She further alleged that the brake lights on Motherrsquos car were burned out and that Motherrsquos negligent failure to properly maintain the car was a contributing cause of the accident

Following a trial on the merits in Sonrsquos case against Driver the jury answered the following special interrogatories

Do you find that Driver was negligent in the operation of her vehicle Yes

Do you find that Son was negligent in the operation of Motherrsquos car No

Do you find that Mother negligently failed to ensure that the brake lights on her car were in proper working order Yes

The judge then entered a judgment in favor of Son against Driver Driver did not appeal

Two months later Mother sued Driver in the United States District Court for the District of State A alleging that Driverrsquos negligence in the operation of her vehicle destroyed Motherrsquos luxury car Mother sought damages in excess of $75000 exclusive of costs and interest

State A follows the same preclusion principles that federal courts follow in federal-question cases

1 Is Motherrsquos claim against Driver barred by the judgment in Son v Driver Explain

2 Does the juryrsquos conclusion in Son v Driver that Mother had negligently failed to maintain the brake lights on her car preclude Mother from litigating that issue in her subsequent suit against Driver Explain

3 Does the juryrsquos conclusion in Son v Driver that Driver was negligent preclude Driver from litigating that issue in the Mother v Driver lawsuit Explain

7

_____

AGENCY QUESTION

Over 5000 individuals in the United States operate hot-air balloon businesses A hot-air balloon has four key components the balloon that holds the heated air the basket that houses the riders the propane burner that heats the air in the balloon and the propane storage tanks

The owner of a hot-air balloon business recently notified several basket and burner manufacturers that she or her agent might be contacting them to purchase baskets or burners The owner did not specifically name any person as her agent Basket and burner manufacturers regularly receive such notices from hot-air balloon operators Such notices typically include no restrictions on the types of baskets or burners agents might purchase for their principals

The owner then retained an agent to acquire baskets burners and fuel tanks from various manufacturers The owner authorized the agent to buy only (a) baskets made of woven wicker (not aluminum) (b) burners that use a unique ldquowhisper technologyrdquo (so as not to scare livestock when the balloon sails over farmland) and (c) propane fuel tanks

The agent then entered into three transactions with manufacturers all of whom had no prior dealings with either the owner or the agent

(1) The agent and a large manufacturer of both wicker and aluminum baskets signed a contract for the purchase of four aluminum baskets for a total cost of $60000 The agent never told the manufacturer that he represented the owner or any other principal The contract listed the agent as the buyer and listed the ownerrsquos address as the delivery address but did not indicate that the address was that of the owner rather than the agent When the baskets were delivered to the owner she learned for the first time that the agent had contracted to buy aluminum not wicker baskets The owner immediately rejected the baskets and returned them to the manufacturer Neither the owner nor the agent has paid the basket manufacturer for them

(2) The agent contacted a burner manufacturer and told him that the agent represented a well-known hot-air balloon operator who wanted to purchase burners The agent did not disclose the ownerrsquos name The agent and the burner manufacturer signed a contract for the purchase of four burners that did not have ldquowhisper technologyrdquo for a total price of $70000 The burner contract like the basket contract listed the ownerrsquos address for delivery but did not disclose whose address it was The burners were delivered to the ownerrsquos business and the owner discovered that the agent had ordered the wrong kind of burners The owner rejected the burners and returned them to the manufacturer Neither the owner nor the agent has paid the burner manufacturer for the burners

(3) The agent contracted with a solar cell manufacturer to make three cells advertised as ldquostrong enough to power all your ballooning needsrdquo The agent did not tell the manufacturer that he was acting on behalf of any other person One week after the cells were delivered to the agent he took them to the owner who installed them and discovered that she could save a lot of money using solar cells instead of propane to power her balloons The owner decided to keep the solar cells but she has not paid the manufacturer for them

8

Agency Question

Assume that the rejection of the baskets and the burners and the failure to pay for the solar cells constitute breach of the relevant contracts

1 Is the owner liable to the basket manufacturer for breach of the contract for the aluminum baskets Is the agent liable Explain

2 Is the owner liable to the burner manufacturer for breach of the contract for the burners Is the agent liable Explain

3 Is the owner liable to the solar cell manufacturer for breach of the contract for the solar cells Is the agent liable Explain (Do not address liability based upon restitution or unjust enrichment)

9

EVIDENCE QUESTION _____

A woman who owns a motorized scooter brought her scooter to a mechanic for routine maintenance service As part of the maintenance service the mechanic inspected the braking system on the scooter As soon as the mechanic finished inspecting and servicing the scooter he sent the woman a text message to her cell phone that read ldquoJust finished your service When you pick up your scooter you need to schedule a follow-up brake repair Wersquoll order the partsrdquo

The woman read the mechanicrsquos text message and returned the next day to pick up her scooter As the woman was wheeling her scooter out of the shop she saw the mechanic working nearby and asked ldquoIs my scooter safe to ride for a whilerdquo The mechanic responded by giving her a thumbs-up The woman waved and rode away on the scooter

One week later while the woman was riding her scooter a pedestrian stepped off the curb into a crosswalk and the woman collided with him causing the pedestrian severe injuries The woman had not had the scooterrsquos brakes repaired before the accident

The pedestrian has sued the woman for damages for his injuries resulting from the accident The pedestrian has alleged that (1) the woman lost control of the scooter due to its defective brakes (2) the woman knew that the brakes needed repair and (3) it was negligent for the woman to ride the scooter knowing that its brakes needed to be repaired

The woman claims that the brakes on the scooter worked perfectly and that the accident happened because the pedestrian stepped into the crosswalk without looking and the woman had no time to stop The woman the pedestrian and the mechanic will testify at the upcoming trial

The pedestrian has proffered an authenticated copy of the mechanicrsquos text message to the woman

The woman plans to testify that she asked the mechanic ldquoIs my scooter safe to ride for a whilerdquo and that he gave her a thumbs-up in response

The evidence rules in this jurisdiction are identical to the Federal Rules of Evidence

Analyze whether each of these items of evidence is relevant and admissible at trial

1 The authenticated copy of the mechanicrsquos text message

2 The womanrsquos testimony that she asked the mechanic ldquoIs my scooter safe to ride for a whilerdquo and

3 The womanrsquos testimony describing the mechanicrsquos thumbs-up

10

TRUSTS AND FUTURE INTERESTS QUESTION

Ten years ago Settlor validly created an inter vivos trust and named Bank as trustee The trust instrument provided that Settlor would receive all of the trust income during her lifetime The trust instrument further provided that

Upon Settlorrsquos death the trust income shall be paid in equal shares to Settlorrsquos surviving children for their lives Upon the death of the last surviving child the trust income shall be paid in equal shares to Settlorrsquos then-living grandchildren for their lives Upon the death of the survivor of Settlorrsquos children and grandchildren the trust corpus shall be distributed in equal shares to Settlorrsquos then-living great-grandchildren

The trust instrument expressly specified that the trust was revocable but it was silent regarding whether Settlor could amend the trust instrument

Immediately after creating the trust Settlor validly executed a will leaving her entire estate to Bank as trustee of her inter vivos trust to ldquohold in accordance with the terms of the trustrdquo

Five years ago Settlor signed an amendment to the inter vivos trust The amendment changed the disposition of the remainder interest specifying that all trust assets ldquoshall be paid upon Settlorrsquos death to Universityrdquo Settlorrsquos signature on this amendment was not witnessed

A state statute provides that any trust interest that violates the common law Rule Against Perpetuities ldquois nonetheless valid if the nonvested interest in the trust actually vests or fails to vest either (a) within 21 years of lives in being at the creation of the nonvested interest or (b) within 90 years of its creationrdquo

Recently Settlor died leaving a probate estate of $200000 She was survived by no children one granddaughter (who would be Settlorrsquos only heir) and no great-grandchildren The granddaughter has consulted your law firm and has raised four questions regarding this trust

1 Was Settlorrsquos amendment of the inter vivos trust valid Explain

2 Assuming that the trust amendment was valid do its provisions apply to Settlorrsquos probate assets Explain

3 Assuming that the trust amendment was valid how should trust assets be distributed Explain

4 Assuming that the trust amendment was invalid how should trust assets be distributed Explain

11

NEGOTIABLE INSTRUMENTS QUESTION

A chef entered into a contract with a repairman pursuant to which the repairman agreed to repair the chefrsquos commercial oven for $10000 The repairman agreed to accept as payment a negotiable promissory note for $10000 payable two months after its issuance

After the repairman worked on the oven the chef gave him a $10000 note as payment for the work As agreed the note was signed by the chef as maker was payable to the order of the repairman was payable in two months and fulfilled all criteria for negotiability

The next day the repairman sold the note to a buyer for $9500 To effectuate the sale the repairman wrote ldquono warrantiesrdquo on the back of the note signed his name immediately below that and handed the note to the buyer The buyer bought the note in good faith and without knowledge of any facts relating to the work that the repairman had performed for the chef

Later the buyer gave the note to his niece as a gift To effectuate the gift the buyer handed the note to the niece but did not indorse it

Shortly thereafter the chef discovered that the repair work had been done improperly and the oven still did not function correctly The chef tried repeatedly to get the repairman to return to correct the repair work but the repairman ignored all the chefrsquos calls

On the notersquos due date the niece contacted the chef and demanded that he pay the amount of the note to her The chef refused and told the niece that he would not pay the note because the repairman did not properly repair the oven

1 What are the niecersquos rights against the chef Explain

2 What are the niecersquos rights against the repairman Explain

3 What are the niecersquos rights against the buyer Explain

12

February 2013 MEE

ANALYSES

Real Property Contracts

Constitutional Law Secured Transactions

Federal Civil Procedure Agency

Evidence Trusts and Future Interests

Negotiable Instruments

REAL PROPERTY ANALYSIS (Real Property ID1a 4 amp 5)

ANALYSIS

Legal Problems

(1) Does the tenant have a defense to the landlordrsquos action for unpaid rent based on constructive eviction

(2) Does the tenant have a defense to the landlordrsquos action for unpaid rent based on the tenantrsquos surrender of the premises

(3) What if anything may the landlord recover from the tenant for the period after the tenant vacated the building

DISCUSSION

Summary

Under the common law the tenant does not have a defense to the landlordrsquos action for unpaid rent based on constructive eviction Constructive eviction is based on the tenant proving that (1) the landlord breached a duty to the tenant (2) the breach caused a loss by the tenant of the substantial use and enjoyment of the premises (3) the tenant gave the landlord adequate notice and opportunity to repair and (4) the tenant vacated the leased premises Here there was no constructive eviction because although the tenant vacated and gave the landlord adequate notice the landlord breached no express or implied duty to the tenant to repair the premises

The tenant does not have a defense based on the landlordrsquos acceptance of his surrender of the premises a landlordrsquos retention of keys does not constitute an acceptance of the tenantrsquos surrender unless the landlord so intended and here the landlordrsquos statements to the tenant at the time of the surrender of the keys do not evidence the intent to accept the tenantrsquos surrender

Under the common law a landlord has no duty to mitigate damages but also cannot sue for rents due in the future Under this approach the landlord can sue only for past-due rents Using this approach on November 1 the landlord could recover all the rent past due (ie rent for September and October) but could not recover for rents due in the future However some courts have authorized recovery for future rent minus the fair market rental value of the premises It is thus possible that the landlord could recover damages equal to the amount of rent due from September 1 to the end of the six-year lease term ($180000) minus the propertyrsquos fair-market rental value over that same period

Point One (45) The tenant was not constructively evicted because the landlord had no duty to repair the commercial premises that were the subject of the lease

The landlord and the tenant entered into a term-of-years lease because the lease specified both a beginning and an ending date HERBERT HOVENKAMP amp SHELDON F KURTZ THE LAW OF

PROPERTY 256 (5th ed 2001) Although a term-of-years lease normally cannot be terminated by the tenant prior to the end of the term a tenant may terminate a term-of-years lease if the tenant

15

Real Property Analysis

is constructively evicted See id at 286ndash88 Typically as here a claim of constructive eviction is made as a defense to a landlordrsquos action for damages or unpaid rent

In order to establish a constructive eviction the tenant must prove that the landlord breached a duty to the tenant such as a duty to repair and that the landlordrsquos breach caused a loss of the substantial use and enjoyment of the premises The tenant must also show that he gave the landlord notice adequate to permit the landlord to meet his duty to the tenant and that the tenant vacated the leased premises Id see also JOHN G SPRANKLING UNDERSTANDING

PROPERTY LAW sect 1704 (2d ed 2007) Under the common law there was no implied duty on the part of a landlord to repair

leased premises such a duty arose only if expressly set forth in the lease SPRANKLING supra sect 1702[B] Here the written lease contained no term requiring the landlord to repair the air-conditioning Even if the conversation created a lease term that the building had air-conditioning that itself should not create a duty for the landlord to repair it

Over the past several decades courts have generally implied a duty to repair in residential leases either as part of a revised constructive eviction doctrine or based on an implied warranty of habitability JOSEPH W SINGER PROPERTY 469ndash70 (3d ed 2010) This shift has been justified based on the economic disparity between the typical landlord and tenant as well as the fact that residential tenants generally lack both the authority to authorize repairs to common areas of a building and the incentive to make repairs that will ultimately benefit the landlord

However courts have been more reluctant to imply a duty to repair in commercial leases a context in which the tenant is often a valuable business and in a better position to assess and make repairs than is the landlord But see eg Davidow v Inwood North Professional Group 747 SW2d 373 (Tex 1988) When courts have implied a duty to repair in a commercial lease it is typically when the repair has been mandated by public authorities and involves work so substantial that it would not ordinarily fall within the tenantrsquos common law repair duty andor the value of the repair would primarily inure to the landlordrsquos reversionary interest See Brown v Green 884 P2d 55 (Cal 1994) Eugene L Grant et al The Tenant as Terminator Constructive Eviction in Modern Commercial Leases 2 THE COMMERCIAL PROPERTY LEASE ch 15 (ABA 1997) Some courts have also permitted constructive eviction claims by commercial tenants of office buildings based on repairs required in common areas of the building See id Echo Consulting Services Inc v North Conway Bank 669 A2d 227 (NH 1995)

Here the tenant is the owner of a valuable manufacturing operation and is the exclusive occupant of the building the repair has not been mandated by public authorities and the repair is not structural To the contrary the repair involves a feature of the building of unusual importance in the tenantrsquos manufacturing operation and the tenant is likely far more knowledgeable than the landlord about the air-conditioning specifications necessary for the manufacture of the tenantrsquos product

Based on these facts it is unlikely that a court will find that the tenant in this case was constructively evicted Although the tenant can show that he gave adequate notice to the landlord of the air-conditioning malfunction and vacated the premises the lease was commercial and it did not contain any promises or covenants by the landlord except a covenant of quiet enjoyment a covenant of quiet enjoyment does not entail any repair obligations

[NOTE An examineersquos conclusion is less important than his or her demonstrated awareness of the elements of constructive eviction and the need to imply a repair duty for such a defense to be viable here Although the implied warranty of habitability is not available to this tenant Texas Minnesota and Massachusetts imply a warranty of suitability in commercial leases in limited circumstances and an examinee might argue that this warranty should apply

16

Real Property Analysis

here If an examinee concludes that this warranty applies he or she should discuss the other requirements for constructive eviction

If the examinee wrongly concludes that the first element for a constructive eviction has been met the examinee will then have to discuss the remaining three elements in order to conclude that the tenant can claim constructive eviction The tenant would have a strong argument that the second elementmdashsubstantial interference with the use and enjoyment of the premisesmdashalso is met As indicated above the landlord was aware that a functioning air-conditioning system was vital to the tenantrsquos manufacturing operations The facts further indicate that the system had failed three times in the past few months The landlord may try to argue that the malfunctions did not substantially interfere with the tenantrsquos use of the premises because the malfunctions caused the temperature to climb above 81 degrees for only a short period of timemdash 3 hours 6 hours and 10 hours respectivelymdashon each occasion The tenant will argue however that the landlord was aware that the tenantrsquos manufacturing operations could tolerate temperatures above 81 degrees for no more than 6 hours The final malfunction exceeded that limit destroying $150000 worth of the tenantrsquos products

The tenant would also have a strong argument that the third element is met notice and opportunity to cure The tenant notified the landlord of the problem immediately upon the systemrsquos first malfunction and did so again when it malfunctioned a second time and then a third time The landlord might argue that there was insufficient time to cure the problem because the system corrected itself within a few hours on the first and second times Although the malfunction lasted more than 10 hours the third time the landlord might argue that the time period was insufficient to get a repair person on the premises A court would be likely to find this argument unpersuasive however because the landlord could have attempted to correct the problem after the first and second malfunctions

Assuming that the landlord was given sufficient notice and opportunity to cure a court would be likely to conclude that the tenant also satisfied the final element of vacating the premises within a reasonable time The landlord might argue that the tenant remained in the premises for almost four months after the air conditioning first failed which would suggest that the problem was not so severe as to have constructively evicted the tenant The tenant will argue however that he gave the landlord three months to cure the problem after the first two malfunctions threatened (but did not actually harm) his operations The tenant then moved out shortly after the final malfunction caused temperatures to exceed the tolerance levels of his manufacturing operations]

Point Two (10) The landlord did not accept the tenantrsquos surrender of the lease

When a tenant wrongfully moves from leased premises with the intent to terminate the lease the landlord may either accept the tenantrsquos surrender of the premises and terminate the lease or hold the tenant to the terms of the lease See HOVENKAMP amp KURTZ supra at 295ndash96 Here the tenantrsquos only basis for the claim that the landlord accepted his surrender is the landlordrsquos retention of the keys Many courts have considered whether a landlordrsquos retention of keys delivered by a tenant constitutes acceptance of surrender The weight of the case law holds that retention of the keys alone does not constitute acceptance of surrender without other evidence showing that the landlord intended to accept the surrender See generally 49 AM JUR 2d Landlord and Tenant sect 213

Here the landlordrsquos note saying ldquoI repeat the air-conditioning is not my problem You have leased the building and you should fix itrdquo strongly suggests that the landlord did not intend

17

Some courts have rejected the no-mitigation-of-damages rule based on efficiency concerns and societyrsquos interest in assuring that resources remain in the stream of commerce rather than lying vacant see id at 464ndash65 and allow landlords to sue tenants who have wrongfully terminated a lease for damages equal to the difference between the unpaid rent due under the lease and the propertyrsquos fair market rental value Other courts have abandoned the no-recovery-for-future-rent rule These courts responding to the fact that a tenant may well disappear or be judgment-proof by the time a lease term is concluded have allowed a landlord to collect damages equal to the value of rent over the entire lease term minus the propertyrsquos fair rental value when a tenant has wrongfully terminated a lease and unequivocally shown an intention not to return to the premises or pay future rent Under this approach a landlord receives approximately the same amount he would have received were there a duty to mitigate damages See Sagamore Corp v Willcutt 180 A 464 (Conn 1935)

Real Property Analysis

to accept the tenantrsquos surrender The tenant might argue that the landlordrsquos failure to make a similar statement when the keys were sent to her a second time and she retained them evidences a change of heart However it is likely that a court would find that the landlordrsquos retention of the keys represented a decision to safeguard the keys not to accept the tenantrsquos surrender

[NOTE An examinee should receive credit for arguing the other way with a well-reasoned argument]

Point Three (45) Under the common law the landlord had no duty to mitigate damages Additionally a landlord was not entitled to recover unpaid rents due in the future but was only entitled to recover rents in arrears at the time of the commencement of the suit Applying the common law here the landlord could recover $5000 the amount of rents due at the commencement of the suit ($2500 for September and the same for October) Today some courts allow the landlord under certain circumstances to sue the tenant for damages (not rent) equal to the difference if any between the unpaid promised rent for the balance of the term (here $175000) and the propertyrsquos fair rental value for the balance of the term

Under the common law because a lease was viewed as a conveyance instead of a contract a landlord had no duty to mitigate damages resulting from a tenantrsquos wrongful termination of a lease A landlord could thus recover the full value of rents that were due and unpaid at the time of the suit However under the common law a landlord could not sue a tenant for rents due in the future because there was always a possibility that the tenant might pay the rent when it was due See SINGER supra at 462 Thus using the common law approach on November 1 the landlord could only recover the full value of the two monthsrsquo rent actually due and unpaid ie $5000 for September and October

Here because the tenant returned the keys to the landlord and said ldquoI will not be returning to the building or making further rent paymentsrdquo the landlord could establish abandonment and an intention not to return It is thus possible that the landlord might recover damages in the amount of $5000 (for the months of September and October) plus the present value of $175000 minus the fair market rental value of the property over the remaining months of the lease

18

CONTRACTS ANALYSIS ____ (Contracts II IVE)

ANALYSIS

Legal Problems

(1) What was the legal effect of the sailorrsquos October 31 letter to the builder

(2)(a) What was the legal effect of the builderrsquos November 25 response to the sailorrsquos October 31 letter

(2)(b) What was the legal effect of the sailorrsquos refusal to take and pay for the boat on December 15

DISCUSSION

Summary

This is a sale of goods governed by the Uniform Commercial Code Because the sailor had reasonable grounds for insecurity about the builderrsquos ability to deliver the boat in a timely manner when the sailor learned about the strike on October 31 the sailor was legally justified in sending the letter to the builder seeking adequate assurance of the builderrsquos performance pursuant to the contract The builderrsquos failure to provide such assurance within a reasonable time operated as a repudiation of the contract However the builder was free to retract the repudiation before the sailor either cancelled the contract or materially changed position in reliance on the builderrsquos repudiation The builder retracted the repudiation when he informed the sailor that the workers were back and that the boat would be delivered by the date stipulated in the partiesrsquo contract Because the sailor had taken no action in response to the original repudiation he no longer had the right to cancel the contract with the builder The sailorrsquos subsequent statement that ldquoour contract is overrdquo may have constituted repudiation by the sailor In any event when the sailor failed to perform on December 15 that constituted breach

Point One (35) Because the sailor had reasonable grounds for insecurity with respect to the builderrsquos performance the sailorrsquos letter to the builder was a justified demand seeking assurance of the builderrsquos performance under the contract failure of the builder to provide such assurance within a reasonable time constituted repudiation of the contract

The sailor was legally justified in sending the letter to the builder on October 31 Contract parties are entitled to expect due performance of contractual obligations and are permitted to take steps to protect that expectation UCC sect 2-609 states that ldquo[w]hen reasonable grounds for insecurity arise with respect to the performance of either party the other may in writing demand adequate assurance of due performance rdquo Here the sailor learned on October 31 that the builderrsquos workers were on strike This gave the sailor reasonable grounds for insecurity about the builderrsquos ability to complete performance on time and thus gave the sailor the right to seek adequate assurance from the builder Because the sailorrsquos demand for assurance was justified the builder was required to provide assurance that was adequate under the circumstances within a reasonable time (not to exceed 30 days) or be held to have repudiated the contract UCC sect 2-609(4)

19

Contracts Analysis

Point Two(a) (30) The builder did not within a reasonable time provide the sailor adequate assurance of due performance this failure to provide assurance constituted a repudiation of the contract

Because the sailor with legal justification (see Point One) demanded from the builder assurance of due performance the builderrsquos failure to provide such assurance within a reasonable time was a repudiation of their contract See UCC sect 2-609(4) (ldquoAfter receipt of a justified demand[] failure to provide within a reasonable time not exceeding thirty days assurance of due performance is a repudiation of the contractrdquo) On October 31 the sailor requested that the builder provide adequate assurance regarding the completion of the boat by December 15 The builder did not respond to the sailorrsquos letter until November 25mdashnearly a month later Even if that response had been given in a reasonable time it nonetheless did not provide assurance of due performance It simply stated ldquoIrsquom sorry about the strike but it is really out of my hands I hope we settle it soon so that we can get back to workrdquo Therefore the builderrsquos November 25 response did not provide adequate assurance in response to the sailorrsquos justified request Thus the builder had repudiated the contract

Point Two(b) (35) Although the builder repudiated the contract with the sailor the builder probably retracted that repudiation on December 3 and the sailor was no longer entitled to cancel their contract Thus the sailorrsquos failure to perform the sailorrsquos obligations under the contract constituted a breach

The builderrsquos failure to provide adequate assurance of performance constituted a repudiation of their contract (see UCC sect 2-609(4)) but the builder was free to retract that repudiation until the sailor cancelled the contract or materially changed his position or indicated by communication or action that the sailor considered the repudiation to be final See UCC sect 2-611(1) (ldquoUntil the repudiating partyrsquos next performance is due he can retract his repudiation unless the aggrieved party has since the repudiation cancelled or materially changed his position or otherwise indicated that he considers the repudiation finalrdquo)

Here the facts state that before the builderrsquos December 3 telephone call to the sailor the sailor did nothing in response to the builderrsquos repudiation such as contracting with a third party for a boat The builderrsquos December 3 call informing the sailor that the boat would be timely delivered probably constituted a retraction of the repudiation because it clearly indicated to the sailor that the builder would be able to perform UCC sect 2-611(2) Thus after being so informed the sailor did not have the right to treat their contract as cancelled UCC sect 2-611(3) Accordingly the sailorrsquos failure to perform the sailorrsquos obligations under the contract by taking the boat and paying for it constituted a breach of the contract

20

CONSTITUTIONAL LAW ANALYSIS (Constitutional Law IVA F2b amp e)

ANALYSIS

Legal Problems

(1) Does AutoCorsquos operation of a ldquocompany townrdquo result in its actions counting as those of the state for purposes of constitutional analysis

(2) Does the expulsion of a schoolchild for failure to recite the Pledge of Allegiance violate the First Amendment as applied through the Fourteenth Amendment

(3) Does the arrest of a pamphleteer in connection with violation of an anti-littering rule where the littering is done by the recipients of leaflets distributed by the pamphleteer violate the First Amendment as applied through the Fourteenth Amendment

DISCUSSION

Summary

The First Amendment as applied through the Fourteenth Amendment applies only to state action It does not typically govern private actors However courts have found state action where the private actor has exercised a ldquopublic functionrdquo such as running a privately owned ldquocompany townrdquo as AutoCo has done here Thus First Amendment protections apply By requiring the son to participate in a mandatory Pledge of Allegiance ceremony AutoCo has compelled the expression of political belief in violation of the First Amendment as applied through the Fourteenth Amendment The fatherrsquos arrest in connection with breaching the anti-litter rule also violated the First Amendment as applied through the Fourteenth Amendment Although state actors can regulate the incidental effects of speech on the public streets on a content-neutral basis this power is limited and cannot extend to punishing a distributor of literature because of littering by third parties

Point One (30) AutoCorsquos operation of a company town (including a school) makes it a state actor under the public function strand of the state action doctrine

The individual rights protections of the Constitution apply only where there is ldquostate actionrdquomdash either direct action by the government or some action by a private party that is fairly attributable to the government As a general rule the actions of a private company like AutoCo or of a private school like the school operated by AutoCo would not constitute state action and the protections of the Constitution (in this case the First Amendment) would not apply

However there are situations in which the actions of a private actor are attributed to the state One such situation is when the private actor undertakes a public function There are not many bright-line rules in the Supreme Courtrsquos state action doctrine but one of them is this Where a private actor undertakes a ldquopublic functionrdquo the Constitution applies to those actions Where a corporation operates a privately owned ldquocompany townrdquo that provides essential services typically provided by a state actor the public function doctrine applies and the Constitution

21

Constitutional Law Analysis

binds agents of the town as if they were agents of the government See eg Marsh v Alabama 326 US 501 (1946) Here AutoCo does more than own the town it provides security services fire protection sanitation services and a school Thus the actions of AutoCo constitute state action and are governed by the Fourteenth Amendment

Point Two (35) The sonrsquos expulsion for failure to recite the Pledge of Allegiance violates the First Amendment as applied through the Fourteenth Amendment as a compelled expression of political belief

As explained in Point One the First Amendment applies to the school as a state actor Although children in public schools (and in schools subject to the First Amendment like

the Oakwood school) have some First Amendment rights Tinker v Des Moines Independent Community School District 393 US 503 506 (1969) schools have greater leeway to regulate the speech of students and teachers than the state would have outside the school context Hazelwood School Dist v Kuhlmeier 484 US 260 (1988) Morse v Frederick 551 US 393 (2007) However the Supreme Court has long held that public schools may not force their students to participate in a flag salute ceremony when it offends the political or religious beliefs of the students or their families West Virginia Board of Educ v Barnette 319 US 624 (1943) (invalidating a mandatory public school flag salute ceremony) see also Wooley v Maynard 430 US 705 (1977) (invalidating compelled expression of political belief on state-issued license plates)

In this case the school requires its students to participate in a flag salute and Pledge of Allegiance ceremony and punishes them when they refuse to participate Pursuant to this policy the school has expelled the son This expulsion violates the First Amendment ban on compelled expression

Point Three (35) Because the father was distributing leaflets in a traditional public forum his trespass arrest violated the First Amendment as applied through the Fourteenth Amendment

As explained in Point One AutoCo is treated as a state actor Thus Oakwoodrsquos commercial district is treated as government-owned property for purposes of the First Amendment Thus the leafleting here is subject to the First Amendment because it is an expressive activity Schneider v State of New Jersey Town of Irvington 308 US 147 (1939) When expression takes place on government-owned property government regulation of the expression is assessed under the public forum doctrine Public streets and sidewalks have long been held to be the classic example of a ldquotraditional public forumrdquo open to the public for expression Hague v CIO 307 US 496 515ndash16 (1939) Because the father was distributing leaflets while standing on a street corner in the commercial district his expressive activity occurred in a traditional public forum

When a state tries to regulate expressive activity in a traditional public forum it is prohibited from doing so based on the expressive activityrsquos content unless its regulation is narrowly tailored to achieve a compelling governmental interest (ldquostrict scrutinyrdquo) In this case however AutoCo is regulating the fatherrsquos expressive activity on the ostensibly neutral ground that his expressive activity has produced litter and made the street unsightly When a state tries to regulate expressive activity without regard to its content intermediate scrutiny applies Under intermediate scrutiny the true purpose of the regulation may not be the suppression of ideas (if so then strict scrutiny applies) the regulation must be narrowly tailored to achieve a significant

22

Constitutional Law Analysis

governmental interest and it must leave open ample alternative channels for expressive activity Ward v Rock Against Racism 491 US 781 791 (1989)

Here the application of the ordinance to the father will fail for two reasons First the Supreme Court has held that the governmentrsquos interest in keeping the streets clean is insufficient to ban leafleting in the public streets as the government power to regulate with incidental effects on public sidewalk speech is very limited See eg Schneider 308 US at 162 (leafletinglittering) Second the regulation (a blanket ban on distribution that results in littering) is not narrowly tailored to protect expression A narrowly tailored alternative would be prosecution only of people who litter Moreover the effect of the littering rule is likely to be a ban on all leafleting thus eliminating an entire class of means of expression This raises the possibility that there are not ldquoample alternative channels of communicationrdquo open to the father as required under the Courtrsquos standard of review for content-neutral regulation of speech

[NOTE Some examinees might argue that this is a ldquotime place and mannerrdquo restriction and that AutoCo might have greater latitude to regulate the public sidewalks under this theory This argument is incorrect for two reasons First the Supreme Court has held that the power to regulate speakers through littering laws is very limited for the reasons given and in the cases cited above But more generally a ldquotime place and mannerrdquo restriction involves the shifting of speech from one time and place to another or to another manner here there is no shifting but a direct punishment for expressive activity (albeit one couched in content-neutral terms) In addition some examinees might read the ordinance to be in effect a total ban on leafleting since most leafleting will produce some litter Those examinees might note that the Court has required total bans on an entire mode of expression to satisfy strict scrutiny and analyze the fatherrsquos prosecution here accordingly See United States v Grace 461 US 171 177 (1983) (invalidating ban on display of signs on public sidewalks surrounding US Supreme Court ldquo[a]dditional restrictions such as an absolute prohibition on a particular type of expression will be upheld only if narrowly drawn to accomplish a compelling governmental interestrdquo)]

23

SECURED TRANSACTIONS ANALYSIS (Secured Transactions IID E IVA B C)

ANALYSIS

Legal Problems

(1) Is a purchase-money security interest in consumer goods perfected even though there has been no filing of a financing statement

(2) Does a person who buys consumer goods for personal use take those goods free of a prior perfected purchase-money security interest in the goods

(3) Does a person who receives consumer goods as a gift take those goods subject to a prior perfected security interest in them

DISCUSSION

Summary

The retailerrsquos security interest in the bicycles was perfected even though no financing statement was filed because it was a purchase-money security interest in consumer goods A purchase-money security interest in consumer goods is automatically perfected upon attachment

The buyer is not subject to the retailerrsquos security interest in the bicycle that the buyer bought from the man Because the bicycle was consumer goods in the hands of the man and the retailer never filed a financing statement covering the bicycle the retailerrsquos security interest is not effective against someone like the buyer who bought the bicycle for value without knowledge of the retailerrsquos security interest and for personal use

On the other hand the retailerrsquos security interest continues in the bicycle given to the friend because the friend did not give value for the bicycle or buy it in the ordinary course of business

Point One (35) The retailerrsquos security interest in the bicycles attached on June 1 Because this interest was a purchase-money security interest in consumer goods it was automatically perfected when it attached

The retailerrsquos security interest in the bicycles attached on June 1 when the man bought the bicycles (acquiring rights in the collateral) signed a security agreement containing a description of the collateral and received value from the retailer (by being given credit with which to purchase the bicycles) UCC sect 9-203(a) amp (b)

Despite the retailerrsquos failure to file a financing statement its security interest was perfected Pursuant to UCC sect 9-309(1) a security interest is automatically perfected upon attachment if the goods are ldquoconsumer goodsrdquo and the security interest is a ldquopurchase-money security interestrdquo

In this case the bicycles sold by the retailer to the man were consumer goods at the time of sale The bicycles were ldquogoodsrdquo because they were ldquomovable when a security interest

24

Secured Transactions Analysis

attachesrdquo UCC sect 9-102(a)(44) They were also consumer goods because they were ldquobought for use primarily for personal family or household purposesrdquo UCC sect 9-102(a)(23) The retailerrsquos security interest in these consumer goods was also a ldquopurchase-money security interestrdquo A purchase-money security interest is an interest that secures a debt that was incurred in order to ldquoenable the debtor to acquire rights in or the use of the collateralrdquo UCC sect 9-103(a) (b)(1) Here the man incurred an obligation to the retailer to purchase the bicycles so the security interest he gave the retailer to secure that obligation was a purchase-money security interest

Because the retailerrsquos security interest was a purchase-money security interest in consumer goods it was automatically perfected on June 1 when the interest attached to the bicycles

Point Two (35) The buyer took the bicycle free of the retailerrsquos security interest because (i) the retailer did not file a financing statement covering the bicycle (ii) the bicycle was ldquoconsumer goodsrdquo and (iii) the buyer bought the bicycle for value without knowledge of the retailerrsquos security interest and for personal use

A security interest continues in collateral even after a sale or other disposition of that collateral unless the creditor authorized the disposition ldquofree of the security interestrdquo or another Article 9 exception applies UCC sectsect 9-201(a) and 9-315(a)(1)

However a buyer of goods like the buyer here can take free of a prior security interest in those goods under certain circumstances See UCC sectsect 9-317(b) (buyers who give value and receive delivery of goods without knowledge of an unperfected security interest in the goods) and 9-320(a) amp (b) (buyer in ordinary course of business buyer of consumer goods in a consumer-to-consumer transaction who gives value) In this case the retailerrsquos security interest was perfected when the buyer purchased the bicycle so UCC sect 9-317(b) does not protect the buyer The buyer also is not a protected ldquobuyer in ordinary course of businessrdquo because he did not purchase from a person who is in the business of selling bicycles See UCC sect 1-201(b)(9)

The buyer can however qualify for the protection of UCC sect 9-320(b) That section provides that a buyer of goods from a person who used them for personal family or household purposes takes free of a perfected security interest in the goods if (1) the buyer had no knowledge of the security interest (2) the buyer gave value for the goods (3) the buyer purchased the goods primarily for personal family or household purposes and (4) the purchase occurred before the filing of a financing statement covering the goods

The buyer met all of these criteria The man used the bicycle for personal purposes The buyer purchased the bicycle from the man and the buyer had no knowledge of the retailerrsquos security interest The buyer gave value ($400) for the bicycle and he bought it ldquoprimarily for personal family or household purposesrdquo as he planned to use it for recreation which is a personal rather than a business use Finally no financing statement had been filed Therefore under UCC sect 9-320(b) the buyer took free of the retailerrsquos security interest

Point Three (30) The retailerrsquos security interest continues in the bicycle that the man gave to the friend Thus the retailer can recover the bicycle from the friend because the friend did not give value for the bicycle or buy it in the ordinary course of business

25

Secured Transactions Analysis

As noted in Point Two the retailer did not authorize the man to dispose of the bicycle Consequently the retailerrsquos security interest continued in the bicycle even after the man transferred ownership of the bicycle to the friend See UCC sectsect 9-201(a) and 9-315(a)(1) The retailerrsquos security interest in the bicycle will be effective against the friend unless some other provision of Article 9 allows the friend to take the bicycle free of that security interest

Unfortunately for the friend there is no Article 9 provision that allows him to take free of the retailerrsquos interest The friendrsquos basic problem is that he is not a buyer of the bicyclemdashhe received the bicycle as a gift and did not give value for it Thus the friend is not protected by any of the applicable exceptions See UCC sectsect 9-317(b) (protecting buyers who give value for goods subject to an unperfected security interest) 9-320(a) (protecting buyers in ordinary course of business) and 9-320(b) (protecting buyers of consumer goods who give value)

In short the retailerrsquos security interest continues in the bicycle that the man gave to the friend The friend took the bicycle subject to that security interest

26

FEDERAL CIVIL PROCEDURE ANALYSIS (Federal Civil Procedure VIE)

ANALYSIS

Legal Problems

(1) Does a judgment in a prior action preclude a nonparty from suing the same defendant on a closely related claim when the nonparty and the original plaintiff are in a family relationship

(2) Does a judgment rendered in an earlier action preclude a nonparty from litigating an issue that was actually decided in the first suit

(3) May a nonparty to an earlier action invoke the judgment in that action to preclude a party to the prior action from relitigating an issue that the party had a full and fair opportunity to litigate in the earlier action

DISCUSSION

Summary

Pursuant to the doctrines of claim preclusion (res judicata) and issue preclusion (collateral estoppel) a judgment is binding on the parties thereto In the absence of privity nonparties to a prior suit cannot be bound by a judgment rendered in their absence Thus in the absence of privity a nonparty to the first suit is not precluded from presenting her claim in a second suit even if it is factually related to the claims and defenses presented in the first suit nor is she bound by determinations of issues made in the first suit A family relationship without more does not support a finding of privity For this reason Mother as a nonparty is not bound by the judgment in the Son-Driver action She may bring her separate claim for damage to her car and she is not precluded from litigating the question of whether she was negligent in the maintenance of her car

Driver on the other hand could be precluded from relitigating the issue of her negligence pursuant to the doctrine of non-mutual issue preclusion (also called non-mutual offensive collateral estoppel) which allows a nonparty to a prior action to invoke issue preclusion to prevent a party to that prior action from relitigating determinations of issues made therein However Mother may be prevented from invoking non-mutual collateral estoppel in this case because she could easily have joined her claim in the prior action but did not do so

[NOTE Federal common law governs the preclusive effect of a judgment rendered by a federal court sitting in diversity See Semtek Intrsquol Inc v Lockheed Martin Corp 531 US 497 508 (2001) But the Semtek Court concluded that federal common law in this context incorporates the preclusion law of the state in which the rendering federal court sits (unless the state law is incompatible with federal interests) id at 508ndash09 Thus State Arsquos preclusion law determines the preclusive effect of the judgment rendered in Sonrsquos suit against Driver The problem says that State A preclusion law is identical to federal preclusion law so the following analysis utilizes general principles of preclusion drawn from Supreme Court case law (announcing federal preclusion rules) and the Restatement (Second) of Judgments]

27

Federal Civil Procedure Analysis

Point One (35) Under the doctrine of claim preclusion the judgment rendered in the first action does not preclude Mother a nonparty from suing Driver for the damage to her car because the judgment binds only parties or those in privity with them and Mother and Son are not in privity

Driver may contend that the doctrine of claim preclusion (res judicata) precludes Mother from presenting a claim arising from the same nucleus of facts that was presented in the first action brought by Son According to the doctrine of claim preclusion ldquowhen a court of competent jurisdiction has entered a final judgment on the merits of a cause of action the parties to the suit and their privies are thereafter bound lsquonot only as to every matter which was offered and received to sustain or defeat the claim or demand but as to any other admissible matter which might have been offered for that purposersquordquo Commissioner of Internal Revenue v Sunnen 333 US 591 597 (1948) (citation omitted)

However the doctrine of claim preclusion does not apply to Mother on the facts of this problem First Mother was not a party to the earlier case ldquoIt is a principle of general application in Anglo-American jurisprudence that one is not bound by a judgment in personam in a litigation in which he is not designated as a party or to which he has not been made a party by service of processrdquo Taylor v Sturgell 553 US 880 884 (2008) (citing Hansberry v Lee 311 US 32 40 (1940)) see also RESTATEMENT (SECOND) OF JUDGMENTS sect 34(3) (1982) This rule reflects our ldquodeep-rooted historic tradition that everyone should have his own day in courtrdquo Martin v Wilks 490 US 755 762 (1989) (citation omitted) (superseded by statute on other grounds) Since Mother was not a party to the first suit she is not bound by the judgment unless an exception to the general rule applies

Mother might be bound by the prior judgment if she were considered to have been sufficiently in privity with Son that Son represented her interests in that action ldquoA person who is not a party to an action but who is represented by a party is bound by and entitled to the benefits of a judgment as though he were a partyrdquo RESTATEMENT (SECOND) OF JUDGMENTS sect 41(1) But there is no suggestion in the facts of the problem that Son who is an adult purported to represent Motherrsquos interests in the first suit ldquo[C]lose family relationships are not sufficient by themselves to establish privity with the original suitrsquos party or to bind a nonparty to that suit by the judgment entered therein rdquo Cuauhtli v Chase Home Finance LLC 308 Fed Appx 772 773 (5th Cir 2009) (citation omitted) accord 18A CHARLES ALAN WRIGHT ET AL FEDERAL

PRACTICE AND PROCEDURE sect 4459 (2d ed 2002) In Taylor v Sturgell supra the Supreme Court identified other special circumstances in

which nonparties may be bound by a prior judgmentmdashwhen a nonparty consents to be bound when a nonparty is in a pre-existing substantive legal relationship with a party (such as preceding and succeeding property owners) when a nonparty assumed control of the prior litigation when a party seeks to relitigate through a proxy or where a special statutory scheme seeks to foreclose successive litigation by nonparties See Taylor 553 US at 893ndash95 None of these circumstances exists here

Because Mother was not a party to the first suit and is not in privity with Son who is an adult the judgment in the first action does not preclude her from bringing her own claim against Driver

Point Two (35) Under the doctrine of issue preclusion the judgment rendered in the first action does not preclude Mother a nonparty from litigating the issue of her negligence in maintaining her carrsquos

28

Federal Civil Procedure Analysis

brake lights because the judgment binds only parties or those in privity with them and Mother and Son are not in privity

By its affirmative response to a special interrogatory the jury in the first action expressly concluded that ldquoMother negligently failed to ensure that the brake lights on her car were in proper working orderrdquo Driver may attempt to invoke the doctrine of issue preclusion to preclude Mother from relitigating this issue in the second action

[I]ssue preclusion arises in a second action on the basis of a prior decision when the same lsquoissuersquo is involved in both actions the issue was lsquoactually litigatedrsquo in the first action after a full and fair opportunity for litigation the issue was lsquoactually decidedrsquo in the first action by a disposition that is sufficiently lsquofinalrsquo lsquoon the meritsrsquo and lsquovalidrsquo it was necessary to decide the issue in disposing of the first action and the later litigation is between the same parties or involves nonparties that are subject to the binding effect or benefit of the first action Once these requirements are met issue preclusion is available not only to defend against a demand for relief but also as offensive support for a demand for relief Issue preclusion moreover is available whether or not the second action involves a new claim or cause of action

18 CHARLES ALAN WRIGHT ET AL FEDERAL PRACTICE AND PROCEDURE sect 4416 at 392ndash93 (2d ed) see also RESTATEMENT (SECOND) OF JUDGMENTS sect 27 (1982)

Here several of the elements necessary for issue preclusion are present The same issue is involved in both actionsmdashthe issue of Motherrsquos negligence in failing to maintain the brake lights on her car That issue was actually litigated in the first action and decided by the jury There is nothing to suggest anything less than a full and fair opportunity to litigate The judgment disposing of the issue was final

Nevertheless the judgment will not preclude Mother from relitigating the issue for two reasons First Mother was not a party to the first action and as explained above Mother and Son are not in privity Therefore she cannot be denied an opportunity to litigate the issue of her negligence Second it does not appear that the juryrsquos decision as to Motherrsquos negligence was necessary to the prior judgment against Driver Nothing suggests that the finding on Motherrsquos negligence had any bearing on the outcome of the first action

Point Three (30) Under the doctrine of non-mutual issue preclusion the judgment rendered in the first action might preclude Driver from relitigating the issue of her negligence However Driver has a strong argument that such a result would be inconsistent with the policy against offensive use of non-mutual estoppel when the non-party plaintiff easily could have joined as a plaintiff in the first action

Because Son already convinced the jury in the first action that ldquoDriver was negligent in the operation of her vehiclerdquo Mother may wish to invoke the doctrine of non-mutual issue preclusion to prevent Driver from relitigating the question of her negligence As noted above ldquoissue preclusion arises in a second action on the basis of a prior decision when the same lsquoissuersquo is involved in both actions the issue was lsquoactually litigatedrsquo in the first action after a full and fair opportunity for litigation the issue was lsquoactually decidedrsquo in the first action by a disposition that is sufficiently lsquofinalrsquo lsquoon the meritsrsquo and lsquovalidrsquo it was necessary to decide the issue in disposing of the first action rdquo 18 CHARLES ALAN WRIGHT ET AL FEDERAL PRACTICE AND

PROCEDURE sect 4416 at 392 (2d ed) see also RESTATEMENT (SECOND) OF JUDGMENTS sect 27

29

Federal Civil Procedure Analysis

Here these basic requirements for issue preclusion are met First the same issue is involved in both suits whether Driver was negligent in the operation of her car Second this issue was actually litigated and decided in the first action the jury answered a special interrogatory raising this very question There is nothing to suggest that Driver lacked a full and fair opportunity to litigate the issue Since a judgment was rendered against Driver for the injuries Son sustained as a result of Driverrsquos negligence resolution of the issue was necessary to dispose of the first action Driver was a party to the first action so she may be bound by the judgment

[NOTE Traditionally issue preclusion required mutualitymdashboth the party asserting issue preclusion and the party against whom issue preclusion was asserted were bound by the prior judgment Under the traditional mutuality rule Mother could not assert issue preclusion against Driver because Mother would not be bound by the judgment if Driver sought to rely on it See Point One There is no mutuality between Mother and Driver with respect to the prior judgment

This traditional mutuality requirement has been abandoned in most jurisdictions The Supreme Court rejected a strict mutuality requirement in Blonder-Tongue Laboratories Inc v University of Illinois Foundation 402 US 313 (1971) (non-mutual defensive collateral estoppel used by a defendant to preclude a plaintiff from relitigating a claim the plaintiff previously litigated) and Parklane Hosiery Co v Shore 439 US 322 (1979) (non-mutual offensive collateral estoppel used by a plaintiff to preclude a defendant from relitigating a claim the defendant previously litigated) In Parklane Hosiery the Court concluded (as a matter of federal preclusion law) that trial courts should have ldquobroad discretionrdquo to determine whether or not to permit a plaintiff to invoke non-mutual issue preclusion ldquoThe general rule should be that in cases where a plaintiff could easily have joined in the earlier action or where the application of offensive estoppel would be unfair to a defendant a trial judge should not allow the use of offensive collateral estoppelrdquo Id at 331

The Parklane Hosiery decision identified a number of circumstances that might make it unfair to allow a plaintiff to invoke non-mutual issue preclusion (non-mutual offensive collateral estoppel in the traditional terminology) against a defendant In particular the Parklane Hosiery court suggested that issue preclusion may not be appropriate if the plaintiff in the second action ldquocould easily have joined in the earlier actionrdquo Id Prohibiting plaintiffs from using non-mutual estoppel under such circumstances would promote judicial efficiency by encouraging plaintiffs to join the prior action It would also discourage plaintiffs from staying out of prior litigation in order to secure in effect two bites at the apple using the prior litigation offensively if the defendant loses and forcing the defendant to litigate a second time if the defendant wins the prior action

An exceptional exam answer might therefore argue that non-mutual issue preclusion should be denied on these facts Son and Mother both reside in State A since they are related they know each other well and Son was driving Motherrsquos car when the accident occurred They could have sued together and Rule 20 of the Federal Rules of Civil Procedure would have authorized joinder of their claims because those claims arose from the same transaction or occurrence and raised a common question of law or fact FED R CIV P 20(a) The facts do not suggest that Mother had any reason not to join Sonrsquos suit other than a desire to see how Sonrsquos action concluded before bringing her own claim Cf Nations v Sun Oil Co (Del) 695 F2d 933 938 (5th Cir 1983) (concluding that plaintiff ldquowas entitled to await the development of his injuries and their predictable consequencesrdquo) Because it appears that Mother may be a ldquowait-and-seerdquo plaintiff who could easily have joined the original action a trial court might disallow as a matter of discretion her use of non-mutual issue preclusion]

30

AGENCY ANALYSIS __________ (Agency I II)

ANALYSIS

Legal Problems

(1) Is the principal or the agent or both liable on contracts with a third party when the principal is an ldquoundisclosed principalrdquo

(2) Is the principal or the agent or both liable on contracts with a third party when the principal is ldquopartially disclosedrdquo or an ldquounidentified principalrdquo

(3) Is the principal or the agent or both liable on contracts with a third party for the purchase of goods when the agent exceeded his authority but the principal nonetheless accepts the goods

DISCUSSION

Summary

The agent but not the owner is liable to the basket manufacturer because the owner is an undisclosed principal and the agent acted without actual or apparent authority Both the agent and the owner however are liable on the burner contract because the owner is an unidentified principal and the agent had apparent authority to enter into that contract With respect to the solar cells contract whether the owner is liable depends upon whether a court would follow the Second or Third Restatement of Agency which take different positions on the effect of the ratification of a contract by an undisclosed principal Under either the agent would also be liable on the contract as he was a party to the contract

[NOTE The contracts that are the subject of this question are contracts for the sale of goods and therefore are governed by Article 2 of the Uniform Commercial Code Article 2 however does not contain agency rules Accordingly common law concepts of agency are applicable UCC sect 1-103(b)]

Point One (35) The agent but not the owner is liable to the basket manufacturer The agent had no actual authority to enter into the contract to buy aluminum baskets and because the owner was an undisclosed principal the manufacturer had no reason to believe that the agent had apparent authority Furthermore the manufacturer had no reason to believe that the agent was not contracting for his own benefit

An agent acting on behalf of a principal can bind the principal to contracts if the agent has either actual or apparent authority An agent has actual authority when contracting on behalf of his principal if he ldquoreasonably believes in accordance with the principalrsquos manifestations to the agent that the principal wishes the agent so to actrdquo RESTATEMENT (THIRD) OF AGENCY sect 201 (2006) Here the agent was told to buy only wicker baskets not aluminum baskets Thus when he contracted with the basket manufacturer to buy aluminum baskets he had no actual authority to do so

31

Agency Analysis

An agent acts with apparent authority ldquowhen a third party [with whom the agent acts] reasonably believes the actor has authority to act on behalf of the principal and that belief is traceable to the principalrsquos manifestationsrdquo Id sect 203 Here the owner notified basket manufacturers that she or her agent might contact them to purchase baskets but that notification did not specifically name the agent or any other person as the ownerrsquos agent Furthermore the basket manufacturer had no prior dealings with the agent or the owner or any reason to think that the agent was acting for the benefit of anyone but himself Thus there is no basis to conclude that the basket manufacturer thought that the agent had apparent authority to act for the owner

Generally when an agent acts on behalf of an undisclosed principal and the agent lacks authority to enter into the contract the agent is liable on the contract as a party to the contract but the principal is not liable This rule is consistent with the third partyrsquos expectations ldquoThe third party expected the agent to be a party to the contract because the agent presented the deal as if he were acting for himself Moreover if the third party is unaware of the principalrsquos existence the third party must be relying on the agentrsquos solvency and reliability when entering into the contractrdquo See ROBERT W HAMILTON JONATHAN R MACEY amp DOUGLAS K MOLL CORPORATIONS INCLUDING PARTNERSHIPS AND LIMITED LIABILITY COMPANIES 34 (11th ed 2010) See also RESTATEMENT (THIRD) OF AGENCY sect 603 cmt c Furthermore because the third party has no idea that the agent is acting or is seemingly acting on behalf of another there is no reason to believe that the third party would be expecting an undisclosed principal to be liable on the contract Id

Point Two (35) Because the owner is an unidentified (as opposed to undisclosed) principal both she and the agent (as a party to the contract) probably are liable on the contract with the burner manufacturer

When the agent contracted with the burner manufacturer he did not have actual authority to do so as the owner had expressly restricted the agentrsquos authority to purchase only burners with ldquowhisper technologyrdquo See Point One However the agent may have had apparent authority to buy burners without whisper technology

An agent acts with apparent authority ldquowhen a third party [with whom the agent acts] reasonably believes the actor has authority to act on behalf of the principal and that belief is traceable to the principalrsquos manifestationsrdquo RESTATEMENT (THIRD) OF AGENCY sect 203 (2006) The owner indicated that an agent might contact the burner manufacturer The notice contained no restriction regarding the type of burners that the agent was authorized to purchase The facts indicate that burner manufacturers regularly receive such notices

Although the agent told the burner manufacturer that he represented a well-known hot-air balloon operator he did not disclose the ownerrsquos name Thus the owner was a partially disclosed or unidentified principal See RESTATEMENT (SECOND) OF AGENCY sect 4(2) (1958) (using term ldquopartially disclosed principalrdquo) RESTATEMENT (THIRD) OF AGENCY sect 104(2)(c) (2006) (using term ldquounidentified principalrdquo) An agent for a partially disclosed principal may have apparent authority RESTATEMENT (SECOND) OF AGENCY sect 159 cmt e (1958) Based upon (1) the notice sent by the owner (2) the agentrsquos revelation that he was acting as an agent and (3) the fact that burner manufacturers regularly receive such notices and sell to agents the manufacturer may argue that it reasonably and actually believed that the agent was authorized to purchase burners without whisper technology The manufacturer may also argue that because the agent revealed that he was an agent his listing of the ownerrsquos address as the delivery address connects the agent to the notice given by the owner Arguably this distinguishes the burner contract from the basket

32

Agency Analysis

contract Here there is a strong case to support the conclusion that the agent had apparent authority if he did then the owner is liable to the burner manufacturer

The agent also is liable as a party to the contract because he did not fully disclose his agency relationship Although he told the burner manufacturer that he represented a well-known hot-air balloon operator he did not disclose the ownerrsquos name Generally even an authorized agent of a partially disclosed or unidentified principal is liable as a party to a contract with a third person RESTATEMENT (SECOND) OF AGENCY sect 321 (1958) (ldquounless otherwise agreedrdquo) RESTATEMENT (THIRD) OF AGENCY sect 602(2) (2006) (ldquounless the agent and the third party agree otherwiserdquo)

Point Three (30) Under the Second Restatement of Agency the owner is not liable on the contract for solar cells because the agent did not have actual or apparent authority and the owner as an undisclosed principal cannot ratify the contract Under the Third Restatement the owner could be liable as she ratified the contract Under either Restatement the agent is liable as a party to the contract

The owner is not liable to the solar cell manufacturer for breach of the contract for the solar cells because the agent had no actual or apparent authority to purchase solar cells on the ownerrsquos behalf and the owner under the Second Restatement of Agency did not ratify the contract with knowledge of the material facts Thus she is not liable as a ratifier of the contract

The facts state that the agent had authority to purchase only propane fuel tanks In addition he had no apparent authority to purchase solar cells The owner made no manifestations to the solar cell manufacturer that would lead a reasonable person in the manufacturerrsquos position to believe that the agent had the authority to bind the owner to a contract to purchase solar cells In fact the agent made no manifestations at all to the solar cell manufacturer Unlike with the basket manufacturer and the burner manufacturer the owner did not notify the manufacturer of solar cells that an agent might contact it to purchase solar cells In addition the solar cells were delivered to the agent and not to the ownerrsquos address In sum the manufacturer was unaware of any relationship between the owner and the agent As to the solar cell manufacturer the owner is an undisclosed principal There can be no apparent authority in the case of an undisclosed principal because there are no manifestations from the principal to the third person See RESTATEMENT (SECOND) OF AGENCY sect 8 cmt a (1958) (ldquothere can be no apparent authority created by an undisclosed principalrdquo) RESTATEMENT (THIRD) OF AGENCY sect 203 cmt f (2006) (ldquoapparent authority is not present when a third party believes that an interaction is with an actor who is a principalrdquo)

The owner also did not ratify the contract Although the owner used the solar cells generally a principal cannot ratify an unauthorized transaction with a third person ldquounless the one acting purported to be acting for the ratifierrdquo RESTATEMENT (SECOND) OF AGENCY sect 85(1) (1958)

The result differs under the Third Restatement which expressly rejects the Second Restatement on this issue The Restatement (Third) of Agency sect 403 (2006) states ldquoA person may ratify an act if the actor acted or purported to act as an agent on the personrsquos behalfrdquo According to comment b ldquoan undisclosed principal may ratify an agentrsquos unauthorized actrdquo Under the Restatement (Third) of Agency rule the owner probably ratified the transaction The agent clearly acted on the ownerrsquos behalf and in addition the ownerrsquos conduct in using the solar cells ldquojustifies a reasonable assumption that [she] is manifesting assent that the act shall affect [her] legal relationsrdquo See id sect 401(2)

33

Agency Analysis

The agent also is liable to the solar cell manufacturer for breach of the contract for the solar cells because he is a party to the contract The facts indicate that the agent never told the solar cell manufacturer that he represented the owner or any other principal Consequently even if the agent were authorized (which as discussed above he is not) he would be liable as a party to the contract See RESTATEMENT (SECOND) OF AGENCY sect 322 (1958) RESTATEMENT (THIRD) OF AGENCY sect 603(2) (2006) Here he has no authority or apparent authority and is liable as a party to the contract

The agent would also be liable under the Third Restatement Under Restatement (Third) of Agency sect 402(1) (2006) ratification generally relates back and the transaction is treated as if it were authorized at the time of the transaction However this does not relieve the agent of an undisclosed principal who ratifies an unauthorized transaction of liability under the ratified contract See id sect 603(2) (authorized agent for undisclosed principal is a party to the contract) and sect 403 cmt b (ldquoAn undisclosed principalrsquos ratification does not eliminate the agentrsquos liability to the third party on the transaction rdquo)

[NOTE An examinee may discuss the concept of inherent agency power This concept is recognized by the Restatement (Second) of Agency sect 8 A (1958) but the concept is not used in the Restatement (Third) of Agency (2006) Here there are no facts to support that the agent had inherent authority

As to contracts with agents for partially disclosed principals (eg the contract for the burners) the basic question is whether the acts done ldquousually accompany or are incidental to transactions which the agent is authorized to conductrdquo RESTATEMENT (SECOND) OF AGENCY

sect 161 (1958) If so the principal is bound if the other party ldquoreasonably believes that the agent is authorized to do them and has no notice that he is not so authorizedrdquo Id The purchase of burners without whisper technology was not authorized nor was it incidental to an authorized transaction Therefore there should not be inherent agency power

As to contracts on behalf of undisclosed principals (eg the other two contracts) the basic question is whether the acts done are usual or necessary in the transactions the agent is authorized to transact RESTATEMENT (SECOND) OF AGENCY sect 194 (1958) The other two contracts seem fundamentally different from the authorized transactions Therefore there should not be inherent agency power

Only minimal credit should be given for discussion of inherent agency power]

34

EVIDENCE ANALYSIS _____ (Evidence IIA VA B E F J K)

ANALYSIS

Legal Problems

(1) Is the authenticated copy of the mechanicrsquos text message relevant and admissible

(2) Is the womanrsquos question ldquoIs my scooter safe to drive for a whilerdquo relevant and admissible

(3) Is the womanrsquos testimony describing the mechanicrsquos thumbs-up relevant and admissible

DISCUSSION

Summary

The mechanicrsquos text message to the woman is relevant to whether (1) the woman lost control of the scooter due to its defective brakes (2) the woman knew that the brakes needed repair and (3) it was negligent for the woman to drive the scooter knowing that its brakes needed repair

The mechanicrsquos text message is hearsay if it is offered by the pedestrian to prove that the scooterrsquos brakes needed repair However it fits the hearsay exception for present sense impressions and probably also fits the exception for business records The mechanicrsquos text message is not hearsay if it is instead offered by the pedestrian to prove the womanrsquos state of mind (ie that she had notice that her brakes needed repair)

The womanrsquos question to the mechanic and his response are also relevant to whether the brakes caused the accident and whether the woman was negligent The question is not hearsay because the woman did not make an assertion

The mechanicrsquos thumbs-up response is nonverbal conduct intended by the mechanic as an assertion and is therefore an out-of-court statement If the woman offers the mechanicrsquos statement to prove that the scooter was actually safe to ride the womanrsquos testimony about the statement is hearsay

However the mechanicrsquos statement is not hearsay if it is offered by the woman to prove her state of mind Therefore the womanrsquos question and the mechanicrsquos response are admissible to prove the womanrsquos state of mind

Point One(a) (20) The mechanicrsquos text message to the woman should be admitted because it is relevant

Evidence is relevant if it has ldquoany tendency to make a fact more or less probable than it would be without the evidencerdquo FED R EVID 401 ldquoRelevant evidence is admissiblerdquo unless it is inadmissible pursuant to some other rule FED R EVID 402

The mechanicrsquos text message to the woman ldquoWhen you pick up your scooter you need to schedule a follow-up brake repair Wersquoll order the partsrdquo is relevant for two reasons First this evidence has some tendency to make it more probable that the brakes malfunctioned and

35

Evidence Analysis

caused the accident Second it has some tendency to make it more probable that the woman was negligent in riding her scooter after being told by the mechanic that it required further repair

Point One(b) (30) The mechanicrsquos text message fits either the hearsay exception for present sense impressions or the exception for business records or it is admissible non-hearsay

The mechanicrsquos text message is a statement under Rule 801(a) because it is ldquoa written assertionrdquo FED R EVID 801(a) The text message is hearsay if the pedestrian offers it to prove the ldquotruth of the matter asserted in the statementrdquo (ie that the scooterrsquos brakes required repair) which resulted in the woman losing control of the scooter and causing the accident FED R EVID 801(c)

However the mechanicrsquos text message fits the hearsay exception for ldquopresent sense impressionsrdquo under Rule 803(1) because it is ldquo[a] statement describing or explaining an event or condition made while or immediately after the declarant perceived itrdquo FED R EVID 803(1) Here the mechanicrsquos text message described the condition of the scooter immediately after he perceived it during the maintenance service

The mechanic is a person with knowledge of the condition of the scooter so if text messages regarding repairs were made and kept by the mechanic in the ordinary course of business this text message also fits the business records exception Under Rule 803(6) a business record is a record of an act ldquomade at or near the time by someone with knowledgerdquo and ldquothe record was kept in the course of a regularly conducted activity of a businessrdquo and ldquomaking the record was a regular practice of that activityrdquo FED R EVID 803(6)

However the text message is not hearsay if it is instead offered to prove that the woman was negligent because she rode her scooter after the mechanic told her it required repair If offered for this purpose it would not be offered for the truth of the matter asserted in the statement but to show the womanrsquos belief about the condition of the scooter (her state of mind)

Point Two (10) The womanrsquos question to the mechanic should be admitted because it is not hearsay

The womanrsquos question to the mechanic is relevant because along with the mechanicrsquos thumbs-up response (see Point Three) it has some tendency to make it more probable that the woman was not negligent andor that the scooter brakes did not malfunction and cause the accident FED R EVID 401 The womanrsquos question does not raise hearsay concerns because it is not an assertion

Hearsay is defined under Rule 801(a) as ldquoan oral assertion written assertion or nonverbal conductrdquo Although ldquoassertionrdquo is not further defined ldquoa favorite [definition] of writers in the [evidence] field for at least a century and a half [is that] the word simply means to say that something is so eg that an event happened or a condition existedrdquo 2 MCCORMICK ON

EVIDENCE sect 246 (6th ed 2006) Under this definition the womanrsquos question is not hearsay because it is not an assertion

Point Three(a) (20) The mechanicrsquos thumbs-up to the woman is a nonverbal assertion that is relevant and the womanrsquos testimony about that response is admissible

36

Evidence Analysis

Hearsay is defined under Rule 801(c) as a ldquostatementrdquo that is ldquoa personrsquos oral assertion written assertion or nonverbal conduct if the person intended it as an assertionrdquo FED R EVID 801(a) Here when the mechanic responded to the womanrsquos question (ldquoIs my scooter safe to ride for a whilerdquo) with a thumbs-up gesture the facts suggest that he intended his nonverbal conduct as an assertion that in his opinion the scooter was safe to ride

The mechanicrsquos assertion is relevant and admissible to prove that the woman was not negligent because the evidence makes it more probable that at the time of the accident she believed that the scooter was safe to ride despite the fact that the brakes required repair FED R EVID 401 Admission of the womanrsquos description of the mechanicrsquos thumbs-up for this purpose does not raise hearsay concerns because the evidence would not be offered for the truth of the matter asserted but to show the womanrsquos belief about the condition of the scooter (her state of mind)

Point Three(b) (20) The mechanicrsquos thumbs-up is relevant to determine whether the scooterrsquos brakes malfunctioned causing the accident but if offered for this purpose it is also hearsay

The mechanicrsquos nonverbal assertion is relevant to the determination of whether the scooterrsquos brakes malfunctioned causing the accident However if offered to prove the ldquotruth of the matter asserted in the statementrdquo (ie that the scooter was safe to ride for a while) it is hearsay that does not fit any hearsay exception

37

TRUSTS AND FUTURE INTERESTS ANALYSIS ____________________ (Trusts and Future Interests IC1 amp 4 G IIF)

ANALYSIS

Legal Problems

(1)(a) Was the revocable trust amendable

(1)(b) If the trust was amendable must the amendment have been executed in accordance with the state Statute of Wills in order to be valid

(2) If the trust amendment was valid does the amendment apply to the probate estate assets passing to the trust pursuant to Settlorrsquos will

(3) If the trust amendment was valid should the trust property be distributed to University

(4) If the trust amendment was not valid should the trust property be distributed to Settlorrsquos grandchild (her only heir) or held in further trust in accordance with the terms of the original trust instrument

DISCUSSION

Summary

A revocable trust is amendable even if the trust instrument does not expressly grant to the trust settlor a power to amend Both inter vivos trusts and amendments thereto are valid even though not executed in accordance with the requirements applicable to wills

Under the Uniform Testamentary Additions to Trusts Act a revocable trust may be amended at any time prior to the settlorrsquos death and the amendment applies to the disposition of assets conveyed to the trust pursuant to a will even if the will was executed prior to the date of the amendment

At Settlorrsquos death trust assets including probate assets passing to the trust under Settlorrsquos will would go to University if as is the case here the trust amendment was valid If the amendment was invalid the trust assets would continue to be held in further trust because there is no violation of the common law Rule Against Perpetuities

Point One(a) (30) Settlor retained the right to amend the inter vivos trust despite her failure to expressly reserve this power

At issue here is whether a retained power of revocation includes the power to amend sometimes referred to as the power to modify The Restatement (Second) of Trusts sect 331 cmt g provides that if a settlor has a power to revoke that retained power ordinarily includes a power to modify (amend) as well Comment g also notes that the power to amend includes both a power to withdraw trust assets and a power to ldquomodify the terms of the trustrdquo The Uniform Trust Code which provides that a power to revoke includes the power to amend is consistent with this view

38

Trusts and Future Interests Analysis

UNIF TRUST CODE sect 602 accord RESTATEMENT (THIRD) OF TRUSTS sect 63 cmt The theory is that even though a power to amend was not expressly retained by a settlor the goal of amendment assuming the power was not included in the power to revoke could easily be achieved by first revoking the trust and then creating a new trust with the same terms contemplated by the amendment To require this would put form over substance

Thus by expressly retaining the power to revoke the trust Settlor retained a power to amend the inter vivos trust despite her failure to expressly reserve this power

[NOTE Under the common law a trust is irrevocable unless the settlor expressly retains a power to revoke the trust Conversely under the Uniform Trust Code a trust is revocable unless the terms of the trust expressly provide otherwise See UNIF TRUST CODE sect 602 The Trust Codersquos position on revocation follows the minority view in the United States and is inconsistent with prior Restatements of Trusts (see Restatement (Second) of Trusts sect 330) Here the trust is revocable because Settlor expressly retained a power of revocation

The Uniform Trust Code has been adopted in 24 jurisdictions Alabama Arizona Arkansas District of Columbia Florida Kansas Maine Michigan Missouri Nebraska New Hampshire New Mexico North Carolina North Dakota Ohio Oregon Pennsylvania South Carolina Tennessee Utah Vermont Virginia West Virginia and Wyoming]

Point One(b) (10) Settlorrsquos amendment of the trust was valid despite her failure to have her signature to the trust amendment witnessed

Neither the common law nor state statutes require a trust instrument or an amendment to a trust instrument to be executed in accordance with the formalities prescribed for execution of a will Indeed an inter vivos trust that does not involve real estate can be created orally Under the Uniform Trust Code the only requirements for creating a valid inter vivos trust are intent the specification of beneficiaries and the designation of a trustee See UNIF TRUST CODE sect 402 accord RESTATEMENT (THIRD) OF TRUSTS sect 13

Here the amendment meets the requirements of both the Uniform Trust Code and the common law Thus the fact that Settlorrsquos signature was not witnessed when she signed the amendment to the trust does not make the amendment invalid

Point Two (20) Under the Uniform Testamentary Additions to Trusts Act a revocable trust may be amended at any time prior to the settlorrsquos death and the amendment applies to probate assets poured into the trust at the settlorrsquos death pursuant to the settlorrsquos will even when the will was executed prior to the date of the amendment

Historically property owned by an individual at her death passed to the individualrsquos heirs or to beneficiaries designated in a will executed with the formalities (writing signing witnessing) prescribed by state law However when a will devises property to the trustee of an inter vivos trust then the provisions of the trustmdashwhich may not have been executed in accordance with the formalities required for willsmdasheffectively determine who will receive the property Because of this possibility some early cases held that if an inter vivos trust was not executed with the same formalities required for a valid will then the trust was ineffective to dispose of probate assets poured into the trust at the settlorrsquos death pursuant to the settlorrsquos will

This line of cases has been overturned by the Uniform Testamentary Additions to Trusts Act (the Act) now Uniform Probate Code sect 2-511 Under the Act adopted in almost all

39

Trusts and Future Interest Analysis

jurisdictions a testamentary bequest to the trustee of an inter vivos trust established by the testator during his or her lifetime is valid if the trust is in writing it is identified in the testatorrsquos will and the trust instrument was executed before concurrently with or after the execution of the will Id The Act further specifies that such a bequest is valid even if the trust is amendable or revocable and that a later amendment applies to assets passing to the trust by a previously executed will

Thus because the trust amendment is valid its terms apply to assets received by Bank from Settlorrsquos estate

Point Three (10) If the trust amendment was valid then the trust assets including assets passing to the trust under Settlorrsquos will should go to University

Under the trust amendment all trust assets (including the assets of Settlorrsquos probate estate poured into the trust) pass to University The facts provide no basis for failing to comply with Settlorrsquos stated intentions

Point Four (30) If the trust amendment was invalid trust assets including assets received pursuant to Settlorrsquos will should be held in accordance with the terms of the original trust instrument because those terms do not violate the Rule Against Perpetuities

Under the dispositive terms of the original trust instrument Settlor created successive income interests in her surviving children and grandchildren with a remainder interest in her great-grandchildren Because the trust was revocable the period during which the common law Rule Against Perpetuities requires that interests vest (ie 21 years plus lives in being) began to run from the date Settlor no longer had a power of revocation (here her death) not the date on which the trust was created See JESSE DUKEMINIER STANLEY J JOHANSON JAMES LINDGREN amp ROBERT SITKOFF WILLS TRUSTS AND ESTATES 678 (7th ed 2005)

Under the common law Rule Against Perpetuities Settlorrsquos trust is thus valid At the time of Settlorrsquos death she was survived by no children one granddaughter and no great-grandchildren Because Settlor cannot have more children after her death the only income beneficiary of the trust is Settlorrsquos surviving granddaughter This granddaughter is the only person who can produce great-grandchildren of Settlor thus all great-grandchildren must of necessity be born during the lifetime of Settlorrsquos only surviving granddaughter who is a life in being The granddaughterrsquos interest vested at Settlorrsquos death and the great-grandchildrenrsquos interest will vest at the death of the granddaughter There is no need to wait the additional 21 years permitted under the Rule Thus under the common law and the statute given in the facts the nonvested interest in the great-grandchildren is valid

[NOTE Both modern wait-and-see statutes and the Uniform Statutory Rule Against Perpetuities upon which the statute in the facts is modeled provide that before using either reform to validate an otherwise invalid nonvested interest one should first determine if the nonvested interest violates the common law Rule If it does not then there is no need to reform This proposition which is applicable in all MEE user jurisdictions that have not simply abrogated the rule is tested by this problem]

40

NEGOTIABLE INSTRUMENTS ANALYSIS (Negotiable Instruments III IV V)

ANALYSIS

Legal Problems

(1)(a) What rights does a person in possession of a note that has been indorsed in blank by the payee have against the maker of the note

(1)(b) Which defenses may the maker of a note raise against a person entitled to enforce it who is not a holder in due course but is a transferee from a holder in due course

(2) What rights does a person entitled to enforce a note have against an indorser who transferred it for consideration with no warranties

(3) What rights does a person entitled to enforce a note have against a previous holder who transferred it as a gift without indorsing it

DISCUSSION

Summary

The niece is a holder of the note and is thus a person entitled to enforce it The chef the issuer of the note is obligated to pay it to the niece as the person entitled to enforce it The niece is not subject to any defense or claim of the chef relating to the improper repair of the oven because the niece has the rights of a holder in due course When the buyer bought the note from the repairman the buyer became a holder in due course of the note and thus took it free of any personal defenses the chef had against the repairman Even though the niece is not herself a holder in due course of the note the niece succeeded to the buyerrsquos rights as holder in due course and thus took free of the chefrsquos personal defenses

Because the chef refused to pay the note the niece can recover from the repairman on the repairmanrsquos obligation as indorser The niece cannot recover on the note against the buyer however because the buyer did not indorse the note (and thus incurred no indorserrsquos obligation) and the buyer did not receive any consideration for transfer of the note to the niece (and therefore made no transfer warranty)

[NOTE Although Article 9 of the Uniform Commercial Code governs the sale of promissory notes (a point that might be correctly noted by examinees) that Article does not determine the answer to any of the questions posed]

Point One(a) (20) The niece is the holder of the note and thus may enforce it against the chef who is the issuer of the note

The chef is the maker of the note and thus its issuer See UCC sectsect 3-103 3-105 The issuer of a note is obligated to pay it in accordance with its terms to a ldquoperson entitled to enforcerdquo it UCC sect 3-412 The niece is a ldquoperson entitled to enforcerdquo the note This is because the niece is the holder of the note and a holder of a note is a person entitled to enforce it UCC sect 3-301 The niece is the holder of the note because (i) the repairmanrsquos signature on the back of the note not

41

Negotiable Instruments Analysis

accompanied by words indicating a person to whom the note was made payable was a ldquoblank indorsementrdquo which had the effect of making the note a bearer instrument (ii) anyone in possession of a bearer instrument is a holder of it and (iii) the niece is in possession of the note See UCC sectsect 1-201(b)(21)(A) 3-204 and 3-205 Accordingly the chef has an obligation to the niece to pay the note in accordance with its terms and the niece may enforce that obligation

Point One(b) (40) The niece is not a holder in due course of the note but because she is a transferee from the buyer who was a holder in due course she has the same enforcement rights as the buyer Because the buyer as a holder in due course would have been able to enforce the note against the chef without being subject to defenses or claims arising from the improper repair the niece has the same rights and will not be subject to the chefrsquos defenses or claims about the repair

As noted in Point One(a) the chef has an obligation to the niece to pay the note in accordance with its terms However except against a person with the rights of a holder in due course the chef can raise any defenses or claims in recoupment that he would have if the claim on the note were an ordinary contract claim UCC sect 3-305 Thus except against a holder in due course the chef would be able to raise the improper repair as a defense or a claim in recoupment (a claim in response to the niecersquos claim)

But claims in recoupment and most defenses cannot be raised against a person with the rights of a holder in due course Against a holder in due course the chef can raise only the four ldquorealrdquo defenses listed in UCC sect 3-305(a)(1) (infancy duress lack of legal capacity or illegality that nullifies the obligation of the obligor under other law fraud in the factum discharge in insolvency proceedings) none of which is present here

The niece is not a holder in due course because she did not take the note for value See UCC sectsect 3-302(a)(2)(i) (criteria for holder in due course status) and 3-303(a) (definition of ldquovaluerdquo) But this does not mean that the niece is subject to the chefrsquos claim arising out of the improper repair The buyer was a holder in due course of the note because he took the note for value ($9500) in good faith and without notice of any facts that would have alerted him to the chefrsquos defense against the repairman UCC sect 3-302(a)(2) As a holder in due course the buyer owned the note free of the chefrsquos claim because that claim did not constitute a ldquorealrdquo defense UCC sect 3-305(b) When the buyer gave the note to the niece this constituted a ldquotransferrdquo of the note See UCC sect 3-203(a) When a note is transferred the transferee receives ldquoany right of the transferor to enforce the instrument including any right as a holder in due courserdquo UCC sect 3-203(b) Under this rule (also known as the ldquoshelter principlerdquo) the buyer transferred his freedom from the chefrsquos defenses to the niece and the niece can enforce the note free of the chefrsquos defenses

Point Two (20) Because the chef dishonored the note the niece can recover from the repairman on the repairmanrsquos obligation as indorser

The chefrsquos refusal to pay the note constituted dishonor See UCC sect 3-502 The repairman as an indorser of the note (see Point One(a)) incurred the obligations of an indorser under UCC sect 3-415(a) When a note has been dishonored one of the obligations of an indorser is to pay the amount of the note to a person entitled to enforce it Therefore the repairman is liable for the amount of the note to the niece a person entitled to enforce the note (so long as the niece gives proper notice of dishonor to the repairman)

42

Negotiable Instruments Analysis

[NOTE Because the repairman indorsed the note without warranties there are no transfer warranties UCC sect 3-416 cmt 5]

Point Three (20) The niece cannot recover on the note against the buyer as either indorser or warrantor because the buyer did not indorse the note and did not receive consideration for transferring the note to the niece

The buyer did not indorse the note and therefore did not incur the obligation of an indorser to pay the note upon dishonor

The niece cannot recover from the buyer under a transfer warranty theory because transfer warranties are made only by a person ldquowho transfers an instrument for considerationrdquo Here the buyer gave the instrument to the niece as a gift So the buyer made no transfer warranty UCC sect 3-416(a) Therefore the niece cannot recover from the buyer on that theory

43

National Conference of Bar Examiners 302 South Bedford Street | Madison WI 53703-3622 Phone 608-280-8550 | Fax 608-280-8552 | TDD 608-661-1275

wwwncbexorg e-mail contactncbexorg

  • Contents
  • Preface
  • Description of the MEE
  • Instructions
  • February 2013 Questions
    • Real Property Question
    • Contracts Question
    • Constitutional Law Question
    • Secured Transactions Question
    • Federal Civil Procedure Question
    • Agency Question
    • Evidence Question
    • Trusts and Future Interests Question
    • Negotiable Instruments Question
      • February 2013 Analyses
        • Real Property Analysis
        • Contracts Analysis
        • Constitutional Law Analysis
        • Secured Transactions Analysis
        • Federal Civil Procedure Analysis
        • Agency Analysis
        • Evidence Analysis
        • Trusts and Future Interests Analysis
        • Negotiable Instruments Analysis
            • ltlt ASCII85EncodePages false AllowTransparency false AutoPositionEPSFiles true AutoRotatePages None Binding Left CalGrayProfile (Dot Gain 20) CalRGBProfile (sRGB IEC61966-21) CalCMYKProfile (US Web Coated 050SWOP051 v2) sRGBProfile (sRGB IEC61966-21) CannotEmbedFontPolicy Error CompatibilityLevel 14 CompressObjects Tags CompressPages true ConvertImagesToIndexed true PassThroughJPEGImages true CreateJobTicket false DefaultRenderingIntent Default DetectBlends true DetectCurves 00000 ColorConversionStrategy CMYK DoThumbnails false EmbedAllFonts true EmbedOpenType false ParseICCProfilesInComments true EmbedJobOptions true DSCReportingLevel 0 EmitDSCWarnings false EndPage -1 ImageMemory 1048576 LockDistillerParams false MaxSubsetPct 100 Optimize true OPM 1 ParseDSCComments true ParseDSCCommentsForDocInfo true PreserveCopyPage true PreserveDICMYKValues true PreserveEPSInfo true PreserveFlatness true PreserveHalftoneInfo false PreserveOPIComments true PreserveOverprintSettings true StartPage 1 SubsetFonts true TransferFunctionInfo Apply UCRandBGInfo Preserve UsePrologue false ColorSettingsFile () AlwaysEmbed [ true ] NeverEmbed [ true ] AntiAliasColorImages false CropColorImages true ColorImageMinResolution 300 ColorImageMinResolutionPolicy OK DownsampleColorImages true ColorImageDownsampleType Bicubic ColorImageResolution 300 ColorImageDepth -1 ColorImageMinDownsampleDepth 1 ColorImageDownsampleThreshold 150000 EncodeColorImages true ColorImageFilter DCTEncode AutoFilterColorImages true ColorImageAutoFilterStrategy JPEG ColorACSImageDict ltlt QFactor 015 HSamples [1 1 1 1] VSamples [1 1 1 1] gtgt ColorImageDict ltlt QFactor 015 HSamples [1 1 1 1] VSamples [1 1 1 1] gtgt JPEG2000ColorACSImageDict ltlt TileWidth 256 TileHeight 256 Quality 30 gtgt JPEG2000ColorImageDict ltlt TileWidth 256 TileHeight 256 Quality 30 gtgt AntiAliasGrayImages false CropGrayImages true GrayImageMinResolution 300 GrayImageMinResolutionPolicy OK DownsampleGrayImages true GrayImageDownsampleType Bicubic GrayImageResolution 300 GrayImageDepth -1 GrayImageMinDownsampleDepth 2 GrayImageDownsampleThreshold 150000 EncodeGrayImages true GrayImageFilter DCTEncode AutoFilterGrayImages true GrayImageAutoFilterStrategy JPEG GrayACSImageDict ltlt QFactor 015 HSamples [1 1 1 1] VSamples [1 1 1 1] gtgt GrayImageDict ltlt QFactor 015 HSamples [1 1 1 1] VSamples [1 1 1 1] gtgt JPEG2000GrayACSImageDict ltlt TileWidth 256 TileHeight 256 Quality 30 gtgt JPEG2000GrayImageDict ltlt TileWidth 256 TileHeight 256 Quality 30 gtgt AntiAliasMonoImages false CropMonoImages true MonoImageMinResolution 1200 MonoImageMinResolutionPolicy OK DownsampleMonoImages true MonoImageDownsampleType Bicubic MonoImageResolution 1200 MonoImageDepth -1 MonoImageDownsampleThreshold 150000 EncodeMonoImages true MonoImageFilter CCITTFaxEncode MonoImageDict ltlt K -1 gtgt AllowPSXObjects false CheckCompliance [ None ] PDFX1aCheck false PDFX3Check false PDFXCompliantPDFOnly false PDFXNoTrimBoxError true PDFXTrimBoxToMediaBoxOffset [ 000000 000000 000000 000000 ] PDFXSetBleedBoxToMediaBox true PDFXBleedBoxToTrimBoxOffset [ 000000 000000 000000 000000 ] PDFXOutputIntentProfile () PDFXOutputConditionIdentifier () PDFXOutputCondition () PDFXRegistryName () PDFXTrapped False CreateJDFFile false Description ltlt ARA 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 BGR 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 CHS ltFEFF4f7f75288fd94e9b8bbe5b9a521b5efa7684002000410064006f006200650020005000440046002065876863900275284e8e9ad88d2891cf76845370524d53705237300260a853ef4ee54f7f75280020004100630072006f0062006100740020548c002000410064006f00620065002000520065006100640065007200200035002e003000204ee553ca66f49ad87248672c676562535f00521b5efa768400200050004400460020658768633002gt CHT ltFEFF4f7f752890194e9b8a2d7f6e5efa7acb7684002000410064006f006200650020005000440046002065874ef69069752865bc9ad854c18cea76845370524d5370523786557406300260a853ef4ee54f7f75280020004100630072006f0062006100740020548c002000410064006f00620065002000520065006100640065007200200035002e003000204ee553ca66f49ad87248672c4f86958b555f5df25efa7acb76840020005000440046002065874ef63002gt CZE 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 DAN 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 DEU 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 ESP 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 ETI 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 FRA 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 GRE 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 HEB ltFEFF05D405E905EA05DE05E905D5002005D105D405D205D305E805D505EA002005D005DC05D4002005DB05D305D9002005DC05D905E605D505E8002005DE05E105DE05DB05D9002000410064006F006200650020005000440046002005D405DE05D505EA05D005DE05D905DD002005DC05D405D305E405E105EA002005E705D305DD002D05D305E405D505E1002005D005D905DB05D505EA05D905EA002E002005DE05E105DE05DB05D90020005000440046002005E905E005D505E605E805D5002005E005D905EA05E005D905DD002005DC05E405EA05D905D705D4002005D105D005DE05E605E205D505EA0020004100630072006F006200610074002005D5002D00410064006F00620065002000520065006100640065007200200035002E0030002005D505D205E805E105D005D505EA002005DE05EA05E705D305DE05D505EA002005D905D505EA05E8002E05D005DE05D905DD002005DC002D005000440046002F0058002D0033002C002005E205D905D905E005D5002005D105DE05D305E805D905DA002005DC05DE05E905EA05DE05E9002005E905DC0020004100630072006F006200610074002E002005DE05E105DE05DB05D90020005000440046002005E905E005D505E605E805D5002005E005D905EA05E005D905DD002005DC05E405EA05D905D705D4002005D105D005DE05E605E205D505EA0020004100630072006F006200610074002005D5002D00410064006F00620065002000520065006100640065007200200035002E0030002005D505D205E805E105D005D505EA002005DE05EA05E705D305DE05D505EA002005D905D505EA05E8002Egt HRV (Za stvaranje Adobe PDF dokumenata najpogodnijih za visokokvalitetni ispis prije tiskanja koristite ove postavke Stvoreni PDF dokumenti mogu se otvoriti Acrobat i Adobe Reader 50 i kasnijim verzijama) HUN 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 ITA 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 JPN ltFEFF9ad854c18cea306a30d730ea30d730ec30b951fa529b7528002000410064006f0062006500200050004400460020658766f8306e4f5c6210306b4f7f75283057307e305930023053306e8a2d5b9a30674f5c62103055308c305f0020005000440046002030d530a130a430eb306f3001004100630072006f0062006100740020304a30883073002000410064006f00620065002000520065006100640065007200200035002e003000204ee5964d3067958b304f30533068304c3067304d307e305930023053306e8a2d5b9a306b306f30d530a930f330c8306e57cb30818fbc307f304c5fc59808306730593002gt KOR ltFEFFc7740020c124c815c7440020c0acc6a9d558c5ec0020ace0d488c9c80020c2dcd5d80020c778c1c4c5d00020ac00c7a50020c801d569d55c002000410064006f0062006500200050004400460020bb38c11cb97c0020c791c131d569b2c8b2e4002e0020c774b807ac8c0020c791c131b41c00200050004400460020bb38c11cb2940020004100630072006f0062006100740020bc0f002000410064006f00620065002000520065006100640065007200200035002e00300020c774c0c1c5d0c11c0020c5f40020c2180020c788c2b5b2c8b2e4002egt LTH 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 LVI 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 NLD (Gebruik deze instellingen om Adobe PDF-documenten te maken die zijn geoptimaliseerd voor prepress-afdrukken van hoge kwaliteit De gemaakte PDF-documenten kunnen worden geopend met Acrobat en Adobe Reader 50 en hoger) NOR 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 POL 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 PTB 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 RUM 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 RUS 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 SKY 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 SLV 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 SUO ltFEFF004b00e40079007400e40020006e00e40069007400e4002000610073006500740075006b007300690061002c0020006b0075006e0020006c0075006f00740020006c00e400680069006e006e00e4002000760061006100740069007600610061006e0020007000610069006e006100740075006b00730065006e002000760061006c006d0069007300740065006c00750074007900f6006800f6006e00200073006f00700069007600690061002000410064006f0062006500200050004400460020002d0064006f006b0075006d0065006e007400740065006a0061002e0020004c0075006f0064007500740020005000440046002d0064006f006b0075006d0065006e00740069007400200076006f0069006400610061006e0020006100760061007400610020004100630072006f0062006100740069006c006c00610020006a0061002000410064006f00620065002000520065006100640065007200200035002e0030003a006c006c00610020006a006100200075007500640065006d006d0069006c006c0061002egt SVE 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 TUR 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 UKR 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 ENU (Use these settings to create Adobe PDF documents best suited for high-quality prepress printing Created PDF documents can be opened with Acrobat and Adobe Reader 50 and later) gtgt Namespace [ (Adobe) (Common) (10) ] OtherNamespaces [ ltlt AsReaderSpreads false CropImagesToFrames true ErrorControl WarnAndContinue FlattenerIgnoreSpreadOverrides false IncludeGuidesGrids false IncludeNonPrinting false IncludeSlug false Namespace [ (Adobe) (InDesign) (40) ] OmitPlacedBitmaps false OmitPlacedEPS false OmitPlacedPDF false SimulateOverprint Legacy gtgt ltlt AddBleedMarks false AddColorBars false AddCropMarks false AddPageInfo false AddRegMarks false ConvertColors ConvertToCMYK DestinationProfileName () DestinationProfileSelector DocumentCMYK Downsample16BitImages true FlattenerPreset ltlt PresetSelector MediumResolution gtgt FormElements false GenerateStructure false IncludeBookmarks false IncludeHyperlinks false IncludeInteractive false IncludeLayers false IncludeProfiles false MultimediaHandling UseObjectSettings Namespace [ (Adobe) (CreativeSuite) (20) ] PDFXOutputIntentProfileSelector DocumentCMYK PreserveEditing true UntaggedCMYKHandling LeaveUntagged UntaggedRGBHandling UseDocumentProfile UseDocumentBleed false gtgt ]gtgt setdistillerparamsltlt HWResolution [2400 2400] PageSize [612000 792000]gtgt setpagedevice

Page 2: February 2013 MEE Questions and Analyses

Copyright copy 2013 by the National Conference of Bar Examiners All rights reserved

Contents

Preface ii

Description of the MEE ii

Instructions iii

February 2013 Questions

Real Property Question 3

Contracts Question 4

Constitutional Law Question 5

Secured Transactions Question 6

Federal Civil Procedure Question 7

Agency Question 8

Evidence Question 10

Trusts and Future Interests Question 11

Negotiable Instruments Question 12

February 2013 Analyses

Real Property Analysis 15

Contracts Analysis 19

Constitutional Law Analysis 21

Secured Transactions Analysis 24

Federal Civil Procedure Analysis 27

Agency Analysis 31

Evidence Analysis 35

Trusts and Future Interests Analysis 38

Negotiable Instruments Analysis 41

Used as one of the six questions on the February 2013 Uniform Bar Examination in Alabama Arizona Colorado Idaho Missouri Nebraska North Dakota and Utah

i

Preface

The Multistate Essay Examination (MEE) is developed by the National Conference of Bar Examiners (NCBE) This publication includes the questions and analyses from the February 2013 MEE Each test includes nine 30-minute questions user jurisdictions may elect which of the nine questions they wish to use (Jurisdictions that administer the Uniform Bar Examination [UBE] use a common set of six MEE questions as part of their bar examinations) In the actual test the questions are simply numbered rather than being identified by area of law The instructions for the test appear on page iii For more information see the MEE Information Booklet available on the NCBE website at wwwncbexorg

The model analyses for the MEE are illustrative of the discussions that might appear in excellent answers to the questions They are provided to the user jurisdictions to assist graders in grading the examination They address all the legal and factual issues the drafters intended to raise in the questions

The subjects covered by each question are listed on the first page of its accompanying analysis followed by roman numerals that refer to the MEE subject matter outline for that subject For example the Federal Civil Procedure question on the February 2013 MEE tested the following area from the Federal Civil Procedure outline VIE Verdicts and judgmentsmdashEffect claim and issue preclusion Subject matter outlines are included in the MEE Information Booklet

Description of the MEE

The MEE consists of nine 30-minute essay questions any of which a jurisdiction may select to include as part of its bar examination (UBE jurisdictions use a common set of six MEE questions as part of their bar examinations) It is administered by participating jurisdictions on the Tuesday before the last Wednesday in February and July of each year The areas of law that may be covered by the questions on any MEE are Business Associations (Agency and Partnership Corporations and Limited Liability Companies) Conflict of Laws Constitutional Law Contracts Criminal Law and Procedure Evidence Family Law Federal Civil Procedure Real Property Torts Trusts and Estates (Decedentsrsquo Estates Trusts and Future Interests) and Uniform Commercial Code (Negotiable Instruments and Bank Deposits and Collections Secured Transactions) Some questions may include issues in more than one area of law The particular areas covered vary from exam to exam

The purpose of the MEE is to test the examineersquos ability to (1) identify legal issues raised by a hypothetical factual situation (2) separate material which is relevant from that which is not (3) present a reasoned analysis of the relevant issues in a clear concise and well-organized composition and (4) demonstrate an understanding of the fundamental legal principles relevant to the probable solution of the issues raised by the factual situation The primary distinction between the MEE and the Multistate Bar Examination (MBE) is that the MEE requires the examinee to demonstrate an ability to communicate effectively in writing

ii

_____

Instructions

The back cover of each test booklet contains the following instructions

You will be instructed when to begin and when to stop this test Do not break the seal on this booklet until you are told to begin

You may answer the questions in any order you wish Do not answer more than one question in each answer booklet If you make a mistake or wish to revise your answer simply draw a line through the material you wish to delete

If you are using a laptop computer to answer the questions your jurisdiction will provide you with specific instructions

Read each fact situation very carefully and do not assume facts that are not given in the question Do not assume that each question covers only a single area of the law some of the questions may cover more than one of the areas you are responsible for knowing

Demonstrate your ability to reason and analyze Each of your answers should show an understanding of the facts a recognition of the issues included a knowledge of the applicable principles of law and the reasoning by which you arrive at your conclusion The value of your answer depends not as much upon your conclusions as upon the presence and quality of the elements mentioned above

Clarity and conciseness are important but make your answer complete Do not volunteer irrelevant or immaterial information

Answer all questions according to generally accepted fundamental legal principles unless your jurisdiction has instructed you to answer according to local case or statutory law (UBE instructions Answer all questions according to generally accepted fundamental legal principles rather than local case or statutory law)

iii

February 2013 MEE

QUESTIONS

Real Property Contracts

Constitutional Law Secured Transactions

Federal Civil Procedure Agency

Evidence Trusts and Future Interests

Negotiable Instruments

REAL PROPERTY QUESTION _______________

In 2008 a landlord and a tenant entered into a 10-year written lease commencing September 1 2008 for the exclusive use of a commercial building at a monthly rent of $2500 The lease contained a covenant of quiet enjoyment but no other covenants or promises on the part of the landlord

When the landlord and tenant negotiated the lease the tenant asked the landlord if the building had an air-conditioning system The landlord answered ldquoYes it doesrdquo The tenant responded ldquoGreat I will be using the building to manufacture a product that will be irreparably damaged if the temperature during manufacture exceeds 81 degrees for more than six consecutive hoursrdquo

On April 15 2012 the buildingrsquos air-conditioning system malfunctioned causing the building temperature to rise above 81 degrees for three hours The tenant immediately telephoned the landlord about this malfunction The tenant left a message in which he explained what had happened and asked the landlord ldquoWhat are you going to do about itrdquo The landlord did not respond to the tenantrsquos message

On May 15 2012 the air-conditioning system again malfunctioned This time the malfunction caused the building temperature to rise above 81 degrees for six hours The tenant telephoned the landlord and left a message describing the malfunction As before the landlord did not respond

On August 24 2012 the air-conditioning system malfunctioned again causing the temperature to rise above 81 degrees for 10 hours Again the tenant promptly telephoned the landlord The landlord answered the phone and the tenant begged her to fix the system The landlord refused The tenant then attempted to fix the system himself but he failed As a result of the air-conditioning malfunction products worth $150000 were destroyed

The next day the tenant wrote the following letter to the landlord

Irsquove had enough I told you about the air-conditioning problem twice before yesterdayrsquos disaster and you failed to correct it I will vacate the building by the end of the month and will bring you the keys when I leave

The tenant vacated the building on August 31 2012 and returned the keys to the landlord that day At that time there were six years remaining on the lease

On September 1 2012 the landlord returned the keys to the tenant with a note that said ldquoI repeat the air-conditioning is not my problem You have leased the building and you should fix itrdquo The tenant promptly sent the keys back to the landlord with a letter that said ldquoI have terminated the lease and I will not be returning to the building or making further rent paymentsrdquo After receiving the keys and letter the landlord put the keys into her desk To date she has neither responded to the tenantrsquos letter nor taken steps to lease the building to another tenant

On November 1 2012 two months after the tenant vacated the property the landlord sued the tenant claiming that she is entitled to the remaining unpaid rent ($180000) from September 1 for the balance of the lease term (reduced to present value) or if not that then damages for the tenantrsquos wrongful termination

Is the landlord correct Explain

3

CONTRACTS QUESTION _______________

On January 2 a boat builder and a sailor entered into a contract pursuant to which the builder was to sell to the sailor a boat to be specially manufactured for the sailor by the builder The contract price was $100000 The written contract signed by both parties stated that the builder would tender the boat to the sailor on December 15 at which time payment in full would be due

On October 15 the builderrsquos workers went on strike and there were no available replacements

On October 31 the builderrsquos workers were still on strike and no work was being done on the boat The sailor read a news report about the strike and immediately sent a letter to the builder stating ldquoI am very concerned that my boat will not be completed by December 15 I insist that you provide me with assurance that you will perform in accordance with the contractrdquo The builder received the letter on the next day November 1

On November 25 the builder responded to the letter stating ldquoIrsquom sorry about the strike but it is really out of my hands I hope we settle it soon so that we can get back to workrdquo

Nothing further happened until December 3 when the builder called the sailor and said ldquoMy workers are back and I have two crews working overtime to finish your boat Your boat is task one Donrsquot worry wersquoll deliver your boat by December 15thrdquo The sailor immediately replied ldquoI donrsquot trust you As far as Irsquom concerned our contract is over I am going to buy my boat from a shipyardrdquo Two days later the sailor entered into a contract with a competing manufacturer to buy a boat similar to the boat that was the subject of the contract with the builder

The builder finished the boat on time and tendered it to the sailor on December 15 The sailor reminded the builder about the December 3 conversation in which the sailor had announced that ldquoour contract is overrdquo and refused to take the boat and pay for it

The builder has sued the sailor for breach of contract

1 What was the legal effect of the sailorrsquos October 31 letter to the builder Explain

2 What was the legal effect of the builderrsquos November 25 response to the sailorrsquos October 31 letter Explain

3 What was the legal effect of the sailorrsquos refusal to take and pay for the boat on December 15 Explain

4

CONSTITUTIONAL LAW QUESTION

AutoCo is a privately owned corporation that manufactures automobiles Ten years ago AutoCo purchased a five-square-mile parcel of unincorporated land in a remote region of the state and built a large automobile assembly plant on the land To attract workers to the remote location of the plant AutoCo built apartment buildings and houses on the land and leased them to its employees AutoCo owns and operates a commercial district with shops and streets open to the general public AutoCo named the area Oakwood and provides security fire protection and sanitation services for Oakwoodrsquos residents AutoCo also built operates and fully funds the only school in the region which it makes available free of charge to the children of its employees

A family recently moved to Oakwood The father and mother work in AutoCorsquos plant rent an apartment from AutoCo and have enrolled their 10-year-old son in Oakwoodrsquos school Every morning the students are required to recite the Pledge of Allegiance while standing and saluting an American flag With the approval of his parents the son has politely but insistently refused to recite the Pledge and salute the flag at the school on the grounds that doing so violates his own political beliefs and the political beliefs of his family As a result of his refusal to say the Pledge the son has been expelled from the school

To protest the schoolrsquos actions the father walked into the commercial district of Oakwood While standing on a street corner he handed out leaflets that contained a short essay critical of the schoolrsquos Pledge of Allegiance policy Some of the passersby who took the leaflets dropped them to the ground An AutoCo security guard saw the litter told the father that Oakwoodrsquos anti-litter rule prohibits leaflet distribution that results in littering and directed him to cease distribution of the leaflets and leave the commercial district When the father did not leave and continued to distribute the leaflets the security guard called the state police which sent officers who arrested the father for trespass

1 Did the sonrsquos expulsion from the school violate the First Amendment as applied through the Fourteenth Amendment Explain

2 Did the fatherrsquos arrest violate the First Amendment as applied through the Fourteenth Amendment Explain

5

SECURED TRANSACTIONS QUESTION

On June 1 a bicycle retailer sold two bicycles to a man for a total purchase price of $1500 The man made a $200 down payment and agreed to pay the balance in one year The man also signed a security agreement that identified the bicycles as collateral for the unpaid purchase price and provided that the man ldquoshall not sell or dispose of the collateral until the balance owed is paid in fullrdquo The retailer never filed a financing statement reflecting this security interest

The man had bought the bicycles for him and his girlfriend to use on vacation However shortly after he bought the bicycles the man and his girlfriend broke up The man has never used the bicycles

On August 1 the man sold one of the bicycles at a garage sale to a buyer who paid the man $400 for the bicycle The buyer bought the bicycle to ride for weekend recreation

On October 1 the man gave the other bicycle to his friend as a birthday present The friend began using the bicycle for morning exercise

Neither the buyer nor the friend had any knowledge of the manrsquos dealings with the retailer

1 Does the buyer own the bicycle free of the retailerrsquos security interest Explain

2 Does the friend own the bicycle free of the retailerrsquos security interest Explain

6

FEDERAL CIVIL PROCEDURE QUESTION _____

Mother and Son who are both adults are citizens and residents of State A Mother owned an expensive luxury car valued in excess of $100000 Son borrowed Motherrsquos car to drive to a store in State A As Son approached a traffic light that had just turned yellow he carefully braked and brought the car to a complete stop Driver who was following immediately behind him failed to stop and rear-ended Motherrsquos car which was damaged beyond repair Son was seriously injured Driver is a citizen of State B

Son sued Driver in the United States District Court for the District of State A alleging that she was negligent in the operation of her vehicle Son sought damages in excess of $75000 for his personal injuries exclusive of costs and interest In her answer Driver alleged that Son was contributorily negligent in the operation of Motherrsquos car She further alleged that the brake lights on Motherrsquos car were burned out and that Motherrsquos negligent failure to properly maintain the car was a contributing cause of the accident

Following a trial on the merits in Sonrsquos case against Driver the jury answered the following special interrogatories

Do you find that Driver was negligent in the operation of her vehicle Yes

Do you find that Son was negligent in the operation of Motherrsquos car No

Do you find that Mother negligently failed to ensure that the brake lights on her car were in proper working order Yes

The judge then entered a judgment in favor of Son against Driver Driver did not appeal

Two months later Mother sued Driver in the United States District Court for the District of State A alleging that Driverrsquos negligence in the operation of her vehicle destroyed Motherrsquos luxury car Mother sought damages in excess of $75000 exclusive of costs and interest

State A follows the same preclusion principles that federal courts follow in federal-question cases

1 Is Motherrsquos claim against Driver barred by the judgment in Son v Driver Explain

2 Does the juryrsquos conclusion in Son v Driver that Mother had negligently failed to maintain the brake lights on her car preclude Mother from litigating that issue in her subsequent suit against Driver Explain

3 Does the juryrsquos conclusion in Son v Driver that Driver was negligent preclude Driver from litigating that issue in the Mother v Driver lawsuit Explain

7

_____

AGENCY QUESTION

Over 5000 individuals in the United States operate hot-air balloon businesses A hot-air balloon has four key components the balloon that holds the heated air the basket that houses the riders the propane burner that heats the air in the balloon and the propane storage tanks

The owner of a hot-air balloon business recently notified several basket and burner manufacturers that she or her agent might be contacting them to purchase baskets or burners The owner did not specifically name any person as her agent Basket and burner manufacturers regularly receive such notices from hot-air balloon operators Such notices typically include no restrictions on the types of baskets or burners agents might purchase for their principals

The owner then retained an agent to acquire baskets burners and fuel tanks from various manufacturers The owner authorized the agent to buy only (a) baskets made of woven wicker (not aluminum) (b) burners that use a unique ldquowhisper technologyrdquo (so as not to scare livestock when the balloon sails over farmland) and (c) propane fuel tanks

The agent then entered into three transactions with manufacturers all of whom had no prior dealings with either the owner or the agent

(1) The agent and a large manufacturer of both wicker and aluminum baskets signed a contract for the purchase of four aluminum baskets for a total cost of $60000 The agent never told the manufacturer that he represented the owner or any other principal The contract listed the agent as the buyer and listed the ownerrsquos address as the delivery address but did not indicate that the address was that of the owner rather than the agent When the baskets were delivered to the owner she learned for the first time that the agent had contracted to buy aluminum not wicker baskets The owner immediately rejected the baskets and returned them to the manufacturer Neither the owner nor the agent has paid the basket manufacturer for them

(2) The agent contacted a burner manufacturer and told him that the agent represented a well-known hot-air balloon operator who wanted to purchase burners The agent did not disclose the ownerrsquos name The agent and the burner manufacturer signed a contract for the purchase of four burners that did not have ldquowhisper technologyrdquo for a total price of $70000 The burner contract like the basket contract listed the ownerrsquos address for delivery but did not disclose whose address it was The burners were delivered to the ownerrsquos business and the owner discovered that the agent had ordered the wrong kind of burners The owner rejected the burners and returned them to the manufacturer Neither the owner nor the agent has paid the burner manufacturer for the burners

(3) The agent contracted with a solar cell manufacturer to make three cells advertised as ldquostrong enough to power all your ballooning needsrdquo The agent did not tell the manufacturer that he was acting on behalf of any other person One week after the cells were delivered to the agent he took them to the owner who installed them and discovered that she could save a lot of money using solar cells instead of propane to power her balloons The owner decided to keep the solar cells but she has not paid the manufacturer for them

8

Agency Question

Assume that the rejection of the baskets and the burners and the failure to pay for the solar cells constitute breach of the relevant contracts

1 Is the owner liable to the basket manufacturer for breach of the contract for the aluminum baskets Is the agent liable Explain

2 Is the owner liable to the burner manufacturer for breach of the contract for the burners Is the agent liable Explain

3 Is the owner liable to the solar cell manufacturer for breach of the contract for the solar cells Is the agent liable Explain (Do not address liability based upon restitution or unjust enrichment)

9

EVIDENCE QUESTION _____

A woman who owns a motorized scooter brought her scooter to a mechanic for routine maintenance service As part of the maintenance service the mechanic inspected the braking system on the scooter As soon as the mechanic finished inspecting and servicing the scooter he sent the woman a text message to her cell phone that read ldquoJust finished your service When you pick up your scooter you need to schedule a follow-up brake repair Wersquoll order the partsrdquo

The woman read the mechanicrsquos text message and returned the next day to pick up her scooter As the woman was wheeling her scooter out of the shop she saw the mechanic working nearby and asked ldquoIs my scooter safe to ride for a whilerdquo The mechanic responded by giving her a thumbs-up The woman waved and rode away on the scooter

One week later while the woman was riding her scooter a pedestrian stepped off the curb into a crosswalk and the woman collided with him causing the pedestrian severe injuries The woman had not had the scooterrsquos brakes repaired before the accident

The pedestrian has sued the woman for damages for his injuries resulting from the accident The pedestrian has alleged that (1) the woman lost control of the scooter due to its defective brakes (2) the woman knew that the brakes needed repair and (3) it was negligent for the woman to ride the scooter knowing that its brakes needed to be repaired

The woman claims that the brakes on the scooter worked perfectly and that the accident happened because the pedestrian stepped into the crosswalk without looking and the woman had no time to stop The woman the pedestrian and the mechanic will testify at the upcoming trial

The pedestrian has proffered an authenticated copy of the mechanicrsquos text message to the woman

The woman plans to testify that she asked the mechanic ldquoIs my scooter safe to ride for a whilerdquo and that he gave her a thumbs-up in response

The evidence rules in this jurisdiction are identical to the Federal Rules of Evidence

Analyze whether each of these items of evidence is relevant and admissible at trial

1 The authenticated copy of the mechanicrsquos text message

2 The womanrsquos testimony that she asked the mechanic ldquoIs my scooter safe to ride for a whilerdquo and

3 The womanrsquos testimony describing the mechanicrsquos thumbs-up

10

TRUSTS AND FUTURE INTERESTS QUESTION

Ten years ago Settlor validly created an inter vivos trust and named Bank as trustee The trust instrument provided that Settlor would receive all of the trust income during her lifetime The trust instrument further provided that

Upon Settlorrsquos death the trust income shall be paid in equal shares to Settlorrsquos surviving children for their lives Upon the death of the last surviving child the trust income shall be paid in equal shares to Settlorrsquos then-living grandchildren for their lives Upon the death of the survivor of Settlorrsquos children and grandchildren the trust corpus shall be distributed in equal shares to Settlorrsquos then-living great-grandchildren

The trust instrument expressly specified that the trust was revocable but it was silent regarding whether Settlor could amend the trust instrument

Immediately after creating the trust Settlor validly executed a will leaving her entire estate to Bank as trustee of her inter vivos trust to ldquohold in accordance with the terms of the trustrdquo

Five years ago Settlor signed an amendment to the inter vivos trust The amendment changed the disposition of the remainder interest specifying that all trust assets ldquoshall be paid upon Settlorrsquos death to Universityrdquo Settlorrsquos signature on this amendment was not witnessed

A state statute provides that any trust interest that violates the common law Rule Against Perpetuities ldquois nonetheless valid if the nonvested interest in the trust actually vests or fails to vest either (a) within 21 years of lives in being at the creation of the nonvested interest or (b) within 90 years of its creationrdquo

Recently Settlor died leaving a probate estate of $200000 She was survived by no children one granddaughter (who would be Settlorrsquos only heir) and no great-grandchildren The granddaughter has consulted your law firm and has raised four questions regarding this trust

1 Was Settlorrsquos amendment of the inter vivos trust valid Explain

2 Assuming that the trust amendment was valid do its provisions apply to Settlorrsquos probate assets Explain

3 Assuming that the trust amendment was valid how should trust assets be distributed Explain

4 Assuming that the trust amendment was invalid how should trust assets be distributed Explain

11

NEGOTIABLE INSTRUMENTS QUESTION

A chef entered into a contract with a repairman pursuant to which the repairman agreed to repair the chefrsquos commercial oven for $10000 The repairman agreed to accept as payment a negotiable promissory note for $10000 payable two months after its issuance

After the repairman worked on the oven the chef gave him a $10000 note as payment for the work As agreed the note was signed by the chef as maker was payable to the order of the repairman was payable in two months and fulfilled all criteria for negotiability

The next day the repairman sold the note to a buyer for $9500 To effectuate the sale the repairman wrote ldquono warrantiesrdquo on the back of the note signed his name immediately below that and handed the note to the buyer The buyer bought the note in good faith and without knowledge of any facts relating to the work that the repairman had performed for the chef

Later the buyer gave the note to his niece as a gift To effectuate the gift the buyer handed the note to the niece but did not indorse it

Shortly thereafter the chef discovered that the repair work had been done improperly and the oven still did not function correctly The chef tried repeatedly to get the repairman to return to correct the repair work but the repairman ignored all the chefrsquos calls

On the notersquos due date the niece contacted the chef and demanded that he pay the amount of the note to her The chef refused and told the niece that he would not pay the note because the repairman did not properly repair the oven

1 What are the niecersquos rights against the chef Explain

2 What are the niecersquos rights against the repairman Explain

3 What are the niecersquos rights against the buyer Explain

12

February 2013 MEE

ANALYSES

Real Property Contracts

Constitutional Law Secured Transactions

Federal Civil Procedure Agency

Evidence Trusts and Future Interests

Negotiable Instruments

REAL PROPERTY ANALYSIS (Real Property ID1a 4 amp 5)

ANALYSIS

Legal Problems

(1) Does the tenant have a defense to the landlordrsquos action for unpaid rent based on constructive eviction

(2) Does the tenant have a defense to the landlordrsquos action for unpaid rent based on the tenantrsquos surrender of the premises

(3) What if anything may the landlord recover from the tenant for the period after the tenant vacated the building

DISCUSSION

Summary

Under the common law the tenant does not have a defense to the landlordrsquos action for unpaid rent based on constructive eviction Constructive eviction is based on the tenant proving that (1) the landlord breached a duty to the tenant (2) the breach caused a loss by the tenant of the substantial use and enjoyment of the premises (3) the tenant gave the landlord adequate notice and opportunity to repair and (4) the tenant vacated the leased premises Here there was no constructive eviction because although the tenant vacated and gave the landlord adequate notice the landlord breached no express or implied duty to the tenant to repair the premises

The tenant does not have a defense based on the landlordrsquos acceptance of his surrender of the premises a landlordrsquos retention of keys does not constitute an acceptance of the tenantrsquos surrender unless the landlord so intended and here the landlordrsquos statements to the tenant at the time of the surrender of the keys do not evidence the intent to accept the tenantrsquos surrender

Under the common law a landlord has no duty to mitigate damages but also cannot sue for rents due in the future Under this approach the landlord can sue only for past-due rents Using this approach on November 1 the landlord could recover all the rent past due (ie rent for September and October) but could not recover for rents due in the future However some courts have authorized recovery for future rent minus the fair market rental value of the premises It is thus possible that the landlord could recover damages equal to the amount of rent due from September 1 to the end of the six-year lease term ($180000) minus the propertyrsquos fair-market rental value over that same period

Point One (45) The tenant was not constructively evicted because the landlord had no duty to repair the commercial premises that were the subject of the lease

The landlord and the tenant entered into a term-of-years lease because the lease specified both a beginning and an ending date HERBERT HOVENKAMP amp SHELDON F KURTZ THE LAW OF

PROPERTY 256 (5th ed 2001) Although a term-of-years lease normally cannot be terminated by the tenant prior to the end of the term a tenant may terminate a term-of-years lease if the tenant

15

Real Property Analysis

is constructively evicted See id at 286ndash88 Typically as here a claim of constructive eviction is made as a defense to a landlordrsquos action for damages or unpaid rent

In order to establish a constructive eviction the tenant must prove that the landlord breached a duty to the tenant such as a duty to repair and that the landlordrsquos breach caused a loss of the substantial use and enjoyment of the premises The tenant must also show that he gave the landlord notice adequate to permit the landlord to meet his duty to the tenant and that the tenant vacated the leased premises Id see also JOHN G SPRANKLING UNDERSTANDING

PROPERTY LAW sect 1704 (2d ed 2007) Under the common law there was no implied duty on the part of a landlord to repair

leased premises such a duty arose only if expressly set forth in the lease SPRANKLING supra sect 1702[B] Here the written lease contained no term requiring the landlord to repair the air-conditioning Even if the conversation created a lease term that the building had air-conditioning that itself should not create a duty for the landlord to repair it

Over the past several decades courts have generally implied a duty to repair in residential leases either as part of a revised constructive eviction doctrine or based on an implied warranty of habitability JOSEPH W SINGER PROPERTY 469ndash70 (3d ed 2010) This shift has been justified based on the economic disparity between the typical landlord and tenant as well as the fact that residential tenants generally lack both the authority to authorize repairs to common areas of a building and the incentive to make repairs that will ultimately benefit the landlord

However courts have been more reluctant to imply a duty to repair in commercial leases a context in which the tenant is often a valuable business and in a better position to assess and make repairs than is the landlord But see eg Davidow v Inwood North Professional Group 747 SW2d 373 (Tex 1988) When courts have implied a duty to repair in a commercial lease it is typically when the repair has been mandated by public authorities and involves work so substantial that it would not ordinarily fall within the tenantrsquos common law repair duty andor the value of the repair would primarily inure to the landlordrsquos reversionary interest See Brown v Green 884 P2d 55 (Cal 1994) Eugene L Grant et al The Tenant as Terminator Constructive Eviction in Modern Commercial Leases 2 THE COMMERCIAL PROPERTY LEASE ch 15 (ABA 1997) Some courts have also permitted constructive eviction claims by commercial tenants of office buildings based on repairs required in common areas of the building See id Echo Consulting Services Inc v North Conway Bank 669 A2d 227 (NH 1995)

Here the tenant is the owner of a valuable manufacturing operation and is the exclusive occupant of the building the repair has not been mandated by public authorities and the repair is not structural To the contrary the repair involves a feature of the building of unusual importance in the tenantrsquos manufacturing operation and the tenant is likely far more knowledgeable than the landlord about the air-conditioning specifications necessary for the manufacture of the tenantrsquos product

Based on these facts it is unlikely that a court will find that the tenant in this case was constructively evicted Although the tenant can show that he gave adequate notice to the landlord of the air-conditioning malfunction and vacated the premises the lease was commercial and it did not contain any promises or covenants by the landlord except a covenant of quiet enjoyment a covenant of quiet enjoyment does not entail any repair obligations

[NOTE An examineersquos conclusion is less important than his or her demonstrated awareness of the elements of constructive eviction and the need to imply a repair duty for such a defense to be viable here Although the implied warranty of habitability is not available to this tenant Texas Minnesota and Massachusetts imply a warranty of suitability in commercial leases in limited circumstances and an examinee might argue that this warranty should apply

16

Real Property Analysis

here If an examinee concludes that this warranty applies he or she should discuss the other requirements for constructive eviction

If the examinee wrongly concludes that the first element for a constructive eviction has been met the examinee will then have to discuss the remaining three elements in order to conclude that the tenant can claim constructive eviction The tenant would have a strong argument that the second elementmdashsubstantial interference with the use and enjoyment of the premisesmdashalso is met As indicated above the landlord was aware that a functioning air-conditioning system was vital to the tenantrsquos manufacturing operations The facts further indicate that the system had failed three times in the past few months The landlord may try to argue that the malfunctions did not substantially interfere with the tenantrsquos use of the premises because the malfunctions caused the temperature to climb above 81 degrees for only a short period of timemdash 3 hours 6 hours and 10 hours respectivelymdashon each occasion The tenant will argue however that the landlord was aware that the tenantrsquos manufacturing operations could tolerate temperatures above 81 degrees for no more than 6 hours The final malfunction exceeded that limit destroying $150000 worth of the tenantrsquos products

The tenant would also have a strong argument that the third element is met notice and opportunity to cure The tenant notified the landlord of the problem immediately upon the systemrsquos first malfunction and did so again when it malfunctioned a second time and then a third time The landlord might argue that there was insufficient time to cure the problem because the system corrected itself within a few hours on the first and second times Although the malfunction lasted more than 10 hours the third time the landlord might argue that the time period was insufficient to get a repair person on the premises A court would be likely to find this argument unpersuasive however because the landlord could have attempted to correct the problem after the first and second malfunctions

Assuming that the landlord was given sufficient notice and opportunity to cure a court would be likely to conclude that the tenant also satisfied the final element of vacating the premises within a reasonable time The landlord might argue that the tenant remained in the premises for almost four months after the air conditioning first failed which would suggest that the problem was not so severe as to have constructively evicted the tenant The tenant will argue however that he gave the landlord three months to cure the problem after the first two malfunctions threatened (but did not actually harm) his operations The tenant then moved out shortly after the final malfunction caused temperatures to exceed the tolerance levels of his manufacturing operations]

Point Two (10) The landlord did not accept the tenantrsquos surrender of the lease

When a tenant wrongfully moves from leased premises with the intent to terminate the lease the landlord may either accept the tenantrsquos surrender of the premises and terminate the lease or hold the tenant to the terms of the lease See HOVENKAMP amp KURTZ supra at 295ndash96 Here the tenantrsquos only basis for the claim that the landlord accepted his surrender is the landlordrsquos retention of the keys Many courts have considered whether a landlordrsquos retention of keys delivered by a tenant constitutes acceptance of surrender The weight of the case law holds that retention of the keys alone does not constitute acceptance of surrender without other evidence showing that the landlord intended to accept the surrender See generally 49 AM JUR 2d Landlord and Tenant sect 213

Here the landlordrsquos note saying ldquoI repeat the air-conditioning is not my problem You have leased the building and you should fix itrdquo strongly suggests that the landlord did not intend

17

Some courts have rejected the no-mitigation-of-damages rule based on efficiency concerns and societyrsquos interest in assuring that resources remain in the stream of commerce rather than lying vacant see id at 464ndash65 and allow landlords to sue tenants who have wrongfully terminated a lease for damages equal to the difference between the unpaid rent due under the lease and the propertyrsquos fair market rental value Other courts have abandoned the no-recovery-for-future-rent rule These courts responding to the fact that a tenant may well disappear or be judgment-proof by the time a lease term is concluded have allowed a landlord to collect damages equal to the value of rent over the entire lease term minus the propertyrsquos fair rental value when a tenant has wrongfully terminated a lease and unequivocally shown an intention not to return to the premises or pay future rent Under this approach a landlord receives approximately the same amount he would have received were there a duty to mitigate damages See Sagamore Corp v Willcutt 180 A 464 (Conn 1935)

Real Property Analysis

to accept the tenantrsquos surrender The tenant might argue that the landlordrsquos failure to make a similar statement when the keys were sent to her a second time and she retained them evidences a change of heart However it is likely that a court would find that the landlordrsquos retention of the keys represented a decision to safeguard the keys not to accept the tenantrsquos surrender

[NOTE An examinee should receive credit for arguing the other way with a well-reasoned argument]

Point Three (45) Under the common law the landlord had no duty to mitigate damages Additionally a landlord was not entitled to recover unpaid rents due in the future but was only entitled to recover rents in arrears at the time of the commencement of the suit Applying the common law here the landlord could recover $5000 the amount of rents due at the commencement of the suit ($2500 for September and the same for October) Today some courts allow the landlord under certain circumstances to sue the tenant for damages (not rent) equal to the difference if any between the unpaid promised rent for the balance of the term (here $175000) and the propertyrsquos fair rental value for the balance of the term

Under the common law because a lease was viewed as a conveyance instead of a contract a landlord had no duty to mitigate damages resulting from a tenantrsquos wrongful termination of a lease A landlord could thus recover the full value of rents that were due and unpaid at the time of the suit However under the common law a landlord could not sue a tenant for rents due in the future because there was always a possibility that the tenant might pay the rent when it was due See SINGER supra at 462 Thus using the common law approach on November 1 the landlord could only recover the full value of the two monthsrsquo rent actually due and unpaid ie $5000 for September and October

Here because the tenant returned the keys to the landlord and said ldquoI will not be returning to the building or making further rent paymentsrdquo the landlord could establish abandonment and an intention not to return It is thus possible that the landlord might recover damages in the amount of $5000 (for the months of September and October) plus the present value of $175000 minus the fair market rental value of the property over the remaining months of the lease

18

CONTRACTS ANALYSIS ____ (Contracts II IVE)

ANALYSIS

Legal Problems

(1) What was the legal effect of the sailorrsquos October 31 letter to the builder

(2)(a) What was the legal effect of the builderrsquos November 25 response to the sailorrsquos October 31 letter

(2)(b) What was the legal effect of the sailorrsquos refusal to take and pay for the boat on December 15

DISCUSSION

Summary

This is a sale of goods governed by the Uniform Commercial Code Because the sailor had reasonable grounds for insecurity about the builderrsquos ability to deliver the boat in a timely manner when the sailor learned about the strike on October 31 the sailor was legally justified in sending the letter to the builder seeking adequate assurance of the builderrsquos performance pursuant to the contract The builderrsquos failure to provide such assurance within a reasonable time operated as a repudiation of the contract However the builder was free to retract the repudiation before the sailor either cancelled the contract or materially changed position in reliance on the builderrsquos repudiation The builder retracted the repudiation when he informed the sailor that the workers were back and that the boat would be delivered by the date stipulated in the partiesrsquo contract Because the sailor had taken no action in response to the original repudiation he no longer had the right to cancel the contract with the builder The sailorrsquos subsequent statement that ldquoour contract is overrdquo may have constituted repudiation by the sailor In any event when the sailor failed to perform on December 15 that constituted breach

Point One (35) Because the sailor had reasonable grounds for insecurity with respect to the builderrsquos performance the sailorrsquos letter to the builder was a justified demand seeking assurance of the builderrsquos performance under the contract failure of the builder to provide such assurance within a reasonable time constituted repudiation of the contract

The sailor was legally justified in sending the letter to the builder on October 31 Contract parties are entitled to expect due performance of contractual obligations and are permitted to take steps to protect that expectation UCC sect 2-609 states that ldquo[w]hen reasonable grounds for insecurity arise with respect to the performance of either party the other may in writing demand adequate assurance of due performance rdquo Here the sailor learned on October 31 that the builderrsquos workers were on strike This gave the sailor reasonable grounds for insecurity about the builderrsquos ability to complete performance on time and thus gave the sailor the right to seek adequate assurance from the builder Because the sailorrsquos demand for assurance was justified the builder was required to provide assurance that was adequate under the circumstances within a reasonable time (not to exceed 30 days) or be held to have repudiated the contract UCC sect 2-609(4)

19

Contracts Analysis

Point Two(a) (30) The builder did not within a reasonable time provide the sailor adequate assurance of due performance this failure to provide assurance constituted a repudiation of the contract

Because the sailor with legal justification (see Point One) demanded from the builder assurance of due performance the builderrsquos failure to provide such assurance within a reasonable time was a repudiation of their contract See UCC sect 2-609(4) (ldquoAfter receipt of a justified demand[] failure to provide within a reasonable time not exceeding thirty days assurance of due performance is a repudiation of the contractrdquo) On October 31 the sailor requested that the builder provide adequate assurance regarding the completion of the boat by December 15 The builder did not respond to the sailorrsquos letter until November 25mdashnearly a month later Even if that response had been given in a reasonable time it nonetheless did not provide assurance of due performance It simply stated ldquoIrsquom sorry about the strike but it is really out of my hands I hope we settle it soon so that we can get back to workrdquo Therefore the builderrsquos November 25 response did not provide adequate assurance in response to the sailorrsquos justified request Thus the builder had repudiated the contract

Point Two(b) (35) Although the builder repudiated the contract with the sailor the builder probably retracted that repudiation on December 3 and the sailor was no longer entitled to cancel their contract Thus the sailorrsquos failure to perform the sailorrsquos obligations under the contract constituted a breach

The builderrsquos failure to provide adequate assurance of performance constituted a repudiation of their contract (see UCC sect 2-609(4)) but the builder was free to retract that repudiation until the sailor cancelled the contract or materially changed his position or indicated by communication or action that the sailor considered the repudiation to be final See UCC sect 2-611(1) (ldquoUntil the repudiating partyrsquos next performance is due he can retract his repudiation unless the aggrieved party has since the repudiation cancelled or materially changed his position or otherwise indicated that he considers the repudiation finalrdquo)

Here the facts state that before the builderrsquos December 3 telephone call to the sailor the sailor did nothing in response to the builderrsquos repudiation such as contracting with a third party for a boat The builderrsquos December 3 call informing the sailor that the boat would be timely delivered probably constituted a retraction of the repudiation because it clearly indicated to the sailor that the builder would be able to perform UCC sect 2-611(2) Thus after being so informed the sailor did not have the right to treat their contract as cancelled UCC sect 2-611(3) Accordingly the sailorrsquos failure to perform the sailorrsquos obligations under the contract by taking the boat and paying for it constituted a breach of the contract

20

CONSTITUTIONAL LAW ANALYSIS (Constitutional Law IVA F2b amp e)

ANALYSIS

Legal Problems

(1) Does AutoCorsquos operation of a ldquocompany townrdquo result in its actions counting as those of the state for purposes of constitutional analysis

(2) Does the expulsion of a schoolchild for failure to recite the Pledge of Allegiance violate the First Amendment as applied through the Fourteenth Amendment

(3) Does the arrest of a pamphleteer in connection with violation of an anti-littering rule where the littering is done by the recipients of leaflets distributed by the pamphleteer violate the First Amendment as applied through the Fourteenth Amendment

DISCUSSION

Summary

The First Amendment as applied through the Fourteenth Amendment applies only to state action It does not typically govern private actors However courts have found state action where the private actor has exercised a ldquopublic functionrdquo such as running a privately owned ldquocompany townrdquo as AutoCo has done here Thus First Amendment protections apply By requiring the son to participate in a mandatory Pledge of Allegiance ceremony AutoCo has compelled the expression of political belief in violation of the First Amendment as applied through the Fourteenth Amendment The fatherrsquos arrest in connection with breaching the anti-litter rule also violated the First Amendment as applied through the Fourteenth Amendment Although state actors can regulate the incidental effects of speech on the public streets on a content-neutral basis this power is limited and cannot extend to punishing a distributor of literature because of littering by third parties

Point One (30) AutoCorsquos operation of a company town (including a school) makes it a state actor under the public function strand of the state action doctrine

The individual rights protections of the Constitution apply only where there is ldquostate actionrdquomdash either direct action by the government or some action by a private party that is fairly attributable to the government As a general rule the actions of a private company like AutoCo or of a private school like the school operated by AutoCo would not constitute state action and the protections of the Constitution (in this case the First Amendment) would not apply

However there are situations in which the actions of a private actor are attributed to the state One such situation is when the private actor undertakes a public function There are not many bright-line rules in the Supreme Courtrsquos state action doctrine but one of them is this Where a private actor undertakes a ldquopublic functionrdquo the Constitution applies to those actions Where a corporation operates a privately owned ldquocompany townrdquo that provides essential services typically provided by a state actor the public function doctrine applies and the Constitution

21

Constitutional Law Analysis

binds agents of the town as if they were agents of the government See eg Marsh v Alabama 326 US 501 (1946) Here AutoCo does more than own the town it provides security services fire protection sanitation services and a school Thus the actions of AutoCo constitute state action and are governed by the Fourteenth Amendment

Point Two (35) The sonrsquos expulsion for failure to recite the Pledge of Allegiance violates the First Amendment as applied through the Fourteenth Amendment as a compelled expression of political belief

As explained in Point One the First Amendment applies to the school as a state actor Although children in public schools (and in schools subject to the First Amendment like

the Oakwood school) have some First Amendment rights Tinker v Des Moines Independent Community School District 393 US 503 506 (1969) schools have greater leeway to regulate the speech of students and teachers than the state would have outside the school context Hazelwood School Dist v Kuhlmeier 484 US 260 (1988) Morse v Frederick 551 US 393 (2007) However the Supreme Court has long held that public schools may not force their students to participate in a flag salute ceremony when it offends the political or religious beliefs of the students or their families West Virginia Board of Educ v Barnette 319 US 624 (1943) (invalidating a mandatory public school flag salute ceremony) see also Wooley v Maynard 430 US 705 (1977) (invalidating compelled expression of political belief on state-issued license plates)

In this case the school requires its students to participate in a flag salute and Pledge of Allegiance ceremony and punishes them when they refuse to participate Pursuant to this policy the school has expelled the son This expulsion violates the First Amendment ban on compelled expression

Point Three (35) Because the father was distributing leaflets in a traditional public forum his trespass arrest violated the First Amendment as applied through the Fourteenth Amendment

As explained in Point One AutoCo is treated as a state actor Thus Oakwoodrsquos commercial district is treated as government-owned property for purposes of the First Amendment Thus the leafleting here is subject to the First Amendment because it is an expressive activity Schneider v State of New Jersey Town of Irvington 308 US 147 (1939) When expression takes place on government-owned property government regulation of the expression is assessed under the public forum doctrine Public streets and sidewalks have long been held to be the classic example of a ldquotraditional public forumrdquo open to the public for expression Hague v CIO 307 US 496 515ndash16 (1939) Because the father was distributing leaflets while standing on a street corner in the commercial district his expressive activity occurred in a traditional public forum

When a state tries to regulate expressive activity in a traditional public forum it is prohibited from doing so based on the expressive activityrsquos content unless its regulation is narrowly tailored to achieve a compelling governmental interest (ldquostrict scrutinyrdquo) In this case however AutoCo is regulating the fatherrsquos expressive activity on the ostensibly neutral ground that his expressive activity has produced litter and made the street unsightly When a state tries to regulate expressive activity without regard to its content intermediate scrutiny applies Under intermediate scrutiny the true purpose of the regulation may not be the suppression of ideas (if so then strict scrutiny applies) the regulation must be narrowly tailored to achieve a significant

22

Constitutional Law Analysis

governmental interest and it must leave open ample alternative channels for expressive activity Ward v Rock Against Racism 491 US 781 791 (1989)

Here the application of the ordinance to the father will fail for two reasons First the Supreme Court has held that the governmentrsquos interest in keeping the streets clean is insufficient to ban leafleting in the public streets as the government power to regulate with incidental effects on public sidewalk speech is very limited See eg Schneider 308 US at 162 (leafletinglittering) Second the regulation (a blanket ban on distribution that results in littering) is not narrowly tailored to protect expression A narrowly tailored alternative would be prosecution only of people who litter Moreover the effect of the littering rule is likely to be a ban on all leafleting thus eliminating an entire class of means of expression This raises the possibility that there are not ldquoample alternative channels of communicationrdquo open to the father as required under the Courtrsquos standard of review for content-neutral regulation of speech

[NOTE Some examinees might argue that this is a ldquotime place and mannerrdquo restriction and that AutoCo might have greater latitude to regulate the public sidewalks under this theory This argument is incorrect for two reasons First the Supreme Court has held that the power to regulate speakers through littering laws is very limited for the reasons given and in the cases cited above But more generally a ldquotime place and mannerrdquo restriction involves the shifting of speech from one time and place to another or to another manner here there is no shifting but a direct punishment for expressive activity (albeit one couched in content-neutral terms) In addition some examinees might read the ordinance to be in effect a total ban on leafleting since most leafleting will produce some litter Those examinees might note that the Court has required total bans on an entire mode of expression to satisfy strict scrutiny and analyze the fatherrsquos prosecution here accordingly See United States v Grace 461 US 171 177 (1983) (invalidating ban on display of signs on public sidewalks surrounding US Supreme Court ldquo[a]dditional restrictions such as an absolute prohibition on a particular type of expression will be upheld only if narrowly drawn to accomplish a compelling governmental interestrdquo)]

23

SECURED TRANSACTIONS ANALYSIS (Secured Transactions IID E IVA B C)

ANALYSIS

Legal Problems

(1) Is a purchase-money security interest in consumer goods perfected even though there has been no filing of a financing statement

(2) Does a person who buys consumer goods for personal use take those goods free of a prior perfected purchase-money security interest in the goods

(3) Does a person who receives consumer goods as a gift take those goods subject to a prior perfected security interest in them

DISCUSSION

Summary

The retailerrsquos security interest in the bicycles was perfected even though no financing statement was filed because it was a purchase-money security interest in consumer goods A purchase-money security interest in consumer goods is automatically perfected upon attachment

The buyer is not subject to the retailerrsquos security interest in the bicycle that the buyer bought from the man Because the bicycle was consumer goods in the hands of the man and the retailer never filed a financing statement covering the bicycle the retailerrsquos security interest is not effective against someone like the buyer who bought the bicycle for value without knowledge of the retailerrsquos security interest and for personal use

On the other hand the retailerrsquos security interest continues in the bicycle given to the friend because the friend did not give value for the bicycle or buy it in the ordinary course of business

Point One (35) The retailerrsquos security interest in the bicycles attached on June 1 Because this interest was a purchase-money security interest in consumer goods it was automatically perfected when it attached

The retailerrsquos security interest in the bicycles attached on June 1 when the man bought the bicycles (acquiring rights in the collateral) signed a security agreement containing a description of the collateral and received value from the retailer (by being given credit with which to purchase the bicycles) UCC sect 9-203(a) amp (b)

Despite the retailerrsquos failure to file a financing statement its security interest was perfected Pursuant to UCC sect 9-309(1) a security interest is automatically perfected upon attachment if the goods are ldquoconsumer goodsrdquo and the security interest is a ldquopurchase-money security interestrdquo

In this case the bicycles sold by the retailer to the man were consumer goods at the time of sale The bicycles were ldquogoodsrdquo because they were ldquomovable when a security interest

24

Secured Transactions Analysis

attachesrdquo UCC sect 9-102(a)(44) They were also consumer goods because they were ldquobought for use primarily for personal family or household purposesrdquo UCC sect 9-102(a)(23) The retailerrsquos security interest in these consumer goods was also a ldquopurchase-money security interestrdquo A purchase-money security interest is an interest that secures a debt that was incurred in order to ldquoenable the debtor to acquire rights in or the use of the collateralrdquo UCC sect 9-103(a) (b)(1) Here the man incurred an obligation to the retailer to purchase the bicycles so the security interest he gave the retailer to secure that obligation was a purchase-money security interest

Because the retailerrsquos security interest was a purchase-money security interest in consumer goods it was automatically perfected on June 1 when the interest attached to the bicycles

Point Two (35) The buyer took the bicycle free of the retailerrsquos security interest because (i) the retailer did not file a financing statement covering the bicycle (ii) the bicycle was ldquoconsumer goodsrdquo and (iii) the buyer bought the bicycle for value without knowledge of the retailerrsquos security interest and for personal use

A security interest continues in collateral even after a sale or other disposition of that collateral unless the creditor authorized the disposition ldquofree of the security interestrdquo or another Article 9 exception applies UCC sectsect 9-201(a) and 9-315(a)(1)

However a buyer of goods like the buyer here can take free of a prior security interest in those goods under certain circumstances See UCC sectsect 9-317(b) (buyers who give value and receive delivery of goods without knowledge of an unperfected security interest in the goods) and 9-320(a) amp (b) (buyer in ordinary course of business buyer of consumer goods in a consumer-to-consumer transaction who gives value) In this case the retailerrsquos security interest was perfected when the buyer purchased the bicycle so UCC sect 9-317(b) does not protect the buyer The buyer also is not a protected ldquobuyer in ordinary course of businessrdquo because he did not purchase from a person who is in the business of selling bicycles See UCC sect 1-201(b)(9)

The buyer can however qualify for the protection of UCC sect 9-320(b) That section provides that a buyer of goods from a person who used them for personal family or household purposes takes free of a perfected security interest in the goods if (1) the buyer had no knowledge of the security interest (2) the buyer gave value for the goods (3) the buyer purchased the goods primarily for personal family or household purposes and (4) the purchase occurred before the filing of a financing statement covering the goods

The buyer met all of these criteria The man used the bicycle for personal purposes The buyer purchased the bicycle from the man and the buyer had no knowledge of the retailerrsquos security interest The buyer gave value ($400) for the bicycle and he bought it ldquoprimarily for personal family or household purposesrdquo as he planned to use it for recreation which is a personal rather than a business use Finally no financing statement had been filed Therefore under UCC sect 9-320(b) the buyer took free of the retailerrsquos security interest

Point Three (30) The retailerrsquos security interest continues in the bicycle that the man gave to the friend Thus the retailer can recover the bicycle from the friend because the friend did not give value for the bicycle or buy it in the ordinary course of business

25

Secured Transactions Analysis

As noted in Point Two the retailer did not authorize the man to dispose of the bicycle Consequently the retailerrsquos security interest continued in the bicycle even after the man transferred ownership of the bicycle to the friend See UCC sectsect 9-201(a) and 9-315(a)(1) The retailerrsquos security interest in the bicycle will be effective against the friend unless some other provision of Article 9 allows the friend to take the bicycle free of that security interest

Unfortunately for the friend there is no Article 9 provision that allows him to take free of the retailerrsquos interest The friendrsquos basic problem is that he is not a buyer of the bicyclemdashhe received the bicycle as a gift and did not give value for it Thus the friend is not protected by any of the applicable exceptions See UCC sectsect 9-317(b) (protecting buyers who give value for goods subject to an unperfected security interest) 9-320(a) (protecting buyers in ordinary course of business) and 9-320(b) (protecting buyers of consumer goods who give value)

In short the retailerrsquos security interest continues in the bicycle that the man gave to the friend The friend took the bicycle subject to that security interest

26

FEDERAL CIVIL PROCEDURE ANALYSIS (Federal Civil Procedure VIE)

ANALYSIS

Legal Problems

(1) Does a judgment in a prior action preclude a nonparty from suing the same defendant on a closely related claim when the nonparty and the original plaintiff are in a family relationship

(2) Does a judgment rendered in an earlier action preclude a nonparty from litigating an issue that was actually decided in the first suit

(3) May a nonparty to an earlier action invoke the judgment in that action to preclude a party to the prior action from relitigating an issue that the party had a full and fair opportunity to litigate in the earlier action

DISCUSSION

Summary

Pursuant to the doctrines of claim preclusion (res judicata) and issue preclusion (collateral estoppel) a judgment is binding on the parties thereto In the absence of privity nonparties to a prior suit cannot be bound by a judgment rendered in their absence Thus in the absence of privity a nonparty to the first suit is not precluded from presenting her claim in a second suit even if it is factually related to the claims and defenses presented in the first suit nor is she bound by determinations of issues made in the first suit A family relationship without more does not support a finding of privity For this reason Mother as a nonparty is not bound by the judgment in the Son-Driver action She may bring her separate claim for damage to her car and she is not precluded from litigating the question of whether she was negligent in the maintenance of her car

Driver on the other hand could be precluded from relitigating the issue of her negligence pursuant to the doctrine of non-mutual issue preclusion (also called non-mutual offensive collateral estoppel) which allows a nonparty to a prior action to invoke issue preclusion to prevent a party to that prior action from relitigating determinations of issues made therein However Mother may be prevented from invoking non-mutual collateral estoppel in this case because she could easily have joined her claim in the prior action but did not do so

[NOTE Federal common law governs the preclusive effect of a judgment rendered by a federal court sitting in diversity See Semtek Intrsquol Inc v Lockheed Martin Corp 531 US 497 508 (2001) But the Semtek Court concluded that federal common law in this context incorporates the preclusion law of the state in which the rendering federal court sits (unless the state law is incompatible with federal interests) id at 508ndash09 Thus State Arsquos preclusion law determines the preclusive effect of the judgment rendered in Sonrsquos suit against Driver The problem says that State A preclusion law is identical to federal preclusion law so the following analysis utilizes general principles of preclusion drawn from Supreme Court case law (announcing federal preclusion rules) and the Restatement (Second) of Judgments]

27

Federal Civil Procedure Analysis

Point One (35) Under the doctrine of claim preclusion the judgment rendered in the first action does not preclude Mother a nonparty from suing Driver for the damage to her car because the judgment binds only parties or those in privity with them and Mother and Son are not in privity

Driver may contend that the doctrine of claim preclusion (res judicata) precludes Mother from presenting a claim arising from the same nucleus of facts that was presented in the first action brought by Son According to the doctrine of claim preclusion ldquowhen a court of competent jurisdiction has entered a final judgment on the merits of a cause of action the parties to the suit and their privies are thereafter bound lsquonot only as to every matter which was offered and received to sustain or defeat the claim or demand but as to any other admissible matter which might have been offered for that purposersquordquo Commissioner of Internal Revenue v Sunnen 333 US 591 597 (1948) (citation omitted)

However the doctrine of claim preclusion does not apply to Mother on the facts of this problem First Mother was not a party to the earlier case ldquoIt is a principle of general application in Anglo-American jurisprudence that one is not bound by a judgment in personam in a litigation in which he is not designated as a party or to which he has not been made a party by service of processrdquo Taylor v Sturgell 553 US 880 884 (2008) (citing Hansberry v Lee 311 US 32 40 (1940)) see also RESTATEMENT (SECOND) OF JUDGMENTS sect 34(3) (1982) This rule reflects our ldquodeep-rooted historic tradition that everyone should have his own day in courtrdquo Martin v Wilks 490 US 755 762 (1989) (citation omitted) (superseded by statute on other grounds) Since Mother was not a party to the first suit she is not bound by the judgment unless an exception to the general rule applies

Mother might be bound by the prior judgment if she were considered to have been sufficiently in privity with Son that Son represented her interests in that action ldquoA person who is not a party to an action but who is represented by a party is bound by and entitled to the benefits of a judgment as though he were a partyrdquo RESTATEMENT (SECOND) OF JUDGMENTS sect 41(1) But there is no suggestion in the facts of the problem that Son who is an adult purported to represent Motherrsquos interests in the first suit ldquo[C]lose family relationships are not sufficient by themselves to establish privity with the original suitrsquos party or to bind a nonparty to that suit by the judgment entered therein rdquo Cuauhtli v Chase Home Finance LLC 308 Fed Appx 772 773 (5th Cir 2009) (citation omitted) accord 18A CHARLES ALAN WRIGHT ET AL FEDERAL

PRACTICE AND PROCEDURE sect 4459 (2d ed 2002) In Taylor v Sturgell supra the Supreme Court identified other special circumstances in

which nonparties may be bound by a prior judgmentmdashwhen a nonparty consents to be bound when a nonparty is in a pre-existing substantive legal relationship with a party (such as preceding and succeeding property owners) when a nonparty assumed control of the prior litigation when a party seeks to relitigate through a proxy or where a special statutory scheme seeks to foreclose successive litigation by nonparties See Taylor 553 US at 893ndash95 None of these circumstances exists here

Because Mother was not a party to the first suit and is not in privity with Son who is an adult the judgment in the first action does not preclude her from bringing her own claim against Driver

Point Two (35) Under the doctrine of issue preclusion the judgment rendered in the first action does not preclude Mother a nonparty from litigating the issue of her negligence in maintaining her carrsquos

28

Federal Civil Procedure Analysis

brake lights because the judgment binds only parties or those in privity with them and Mother and Son are not in privity

By its affirmative response to a special interrogatory the jury in the first action expressly concluded that ldquoMother negligently failed to ensure that the brake lights on her car were in proper working orderrdquo Driver may attempt to invoke the doctrine of issue preclusion to preclude Mother from relitigating this issue in the second action

[I]ssue preclusion arises in a second action on the basis of a prior decision when the same lsquoissuersquo is involved in both actions the issue was lsquoactually litigatedrsquo in the first action after a full and fair opportunity for litigation the issue was lsquoactually decidedrsquo in the first action by a disposition that is sufficiently lsquofinalrsquo lsquoon the meritsrsquo and lsquovalidrsquo it was necessary to decide the issue in disposing of the first action and the later litigation is between the same parties or involves nonparties that are subject to the binding effect or benefit of the first action Once these requirements are met issue preclusion is available not only to defend against a demand for relief but also as offensive support for a demand for relief Issue preclusion moreover is available whether or not the second action involves a new claim or cause of action

18 CHARLES ALAN WRIGHT ET AL FEDERAL PRACTICE AND PROCEDURE sect 4416 at 392ndash93 (2d ed) see also RESTATEMENT (SECOND) OF JUDGMENTS sect 27 (1982)

Here several of the elements necessary for issue preclusion are present The same issue is involved in both actionsmdashthe issue of Motherrsquos negligence in failing to maintain the brake lights on her car That issue was actually litigated in the first action and decided by the jury There is nothing to suggest anything less than a full and fair opportunity to litigate The judgment disposing of the issue was final

Nevertheless the judgment will not preclude Mother from relitigating the issue for two reasons First Mother was not a party to the first action and as explained above Mother and Son are not in privity Therefore she cannot be denied an opportunity to litigate the issue of her negligence Second it does not appear that the juryrsquos decision as to Motherrsquos negligence was necessary to the prior judgment against Driver Nothing suggests that the finding on Motherrsquos negligence had any bearing on the outcome of the first action

Point Three (30) Under the doctrine of non-mutual issue preclusion the judgment rendered in the first action might preclude Driver from relitigating the issue of her negligence However Driver has a strong argument that such a result would be inconsistent with the policy against offensive use of non-mutual estoppel when the non-party plaintiff easily could have joined as a plaintiff in the first action

Because Son already convinced the jury in the first action that ldquoDriver was negligent in the operation of her vehiclerdquo Mother may wish to invoke the doctrine of non-mutual issue preclusion to prevent Driver from relitigating the question of her negligence As noted above ldquoissue preclusion arises in a second action on the basis of a prior decision when the same lsquoissuersquo is involved in both actions the issue was lsquoactually litigatedrsquo in the first action after a full and fair opportunity for litigation the issue was lsquoactually decidedrsquo in the first action by a disposition that is sufficiently lsquofinalrsquo lsquoon the meritsrsquo and lsquovalidrsquo it was necessary to decide the issue in disposing of the first action rdquo 18 CHARLES ALAN WRIGHT ET AL FEDERAL PRACTICE AND

PROCEDURE sect 4416 at 392 (2d ed) see also RESTATEMENT (SECOND) OF JUDGMENTS sect 27

29

Federal Civil Procedure Analysis

Here these basic requirements for issue preclusion are met First the same issue is involved in both suits whether Driver was negligent in the operation of her car Second this issue was actually litigated and decided in the first action the jury answered a special interrogatory raising this very question There is nothing to suggest that Driver lacked a full and fair opportunity to litigate the issue Since a judgment was rendered against Driver for the injuries Son sustained as a result of Driverrsquos negligence resolution of the issue was necessary to dispose of the first action Driver was a party to the first action so she may be bound by the judgment

[NOTE Traditionally issue preclusion required mutualitymdashboth the party asserting issue preclusion and the party against whom issue preclusion was asserted were bound by the prior judgment Under the traditional mutuality rule Mother could not assert issue preclusion against Driver because Mother would not be bound by the judgment if Driver sought to rely on it See Point One There is no mutuality between Mother and Driver with respect to the prior judgment

This traditional mutuality requirement has been abandoned in most jurisdictions The Supreme Court rejected a strict mutuality requirement in Blonder-Tongue Laboratories Inc v University of Illinois Foundation 402 US 313 (1971) (non-mutual defensive collateral estoppel used by a defendant to preclude a plaintiff from relitigating a claim the plaintiff previously litigated) and Parklane Hosiery Co v Shore 439 US 322 (1979) (non-mutual offensive collateral estoppel used by a plaintiff to preclude a defendant from relitigating a claim the defendant previously litigated) In Parklane Hosiery the Court concluded (as a matter of federal preclusion law) that trial courts should have ldquobroad discretionrdquo to determine whether or not to permit a plaintiff to invoke non-mutual issue preclusion ldquoThe general rule should be that in cases where a plaintiff could easily have joined in the earlier action or where the application of offensive estoppel would be unfair to a defendant a trial judge should not allow the use of offensive collateral estoppelrdquo Id at 331

The Parklane Hosiery decision identified a number of circumstances that might make it unfair to allow a plaintiff to invoke non-mutual issue preclusion (non-mutual offensive collateral estoppel in the traditional terminology) against a defendant In particular the Parklane Hosiery court suggested that issue preclusion may not be appropriate if the plaintiff in the second action ldquocould easily have joined in the earlier actionrdquo Id Prohibiting plaintiffs from using non-mutual estoppel under such circumstances would promote judicial efficiency by encouraging plaintiffs to join the prior action It would also discourage plaintiffs from staying out of prior litigation in order to secure in effect two bites at the apple using the prior litigation offensively if the defendant loses and forcing the defendant to litigate a second time if the defendant wins the prior action

An exceptional exam answer might therefore argue that non-mutual issue preclusion should be denied on these facts Son and Mother both reside in State A since they are related they know each other well and Son was driving Motherrsquos car when the accident occurred They could have sued together and Rule 20 of the Federal Rules of Civil Procedure would have authorized joinder of their claims because those claims arose from the same transaction or occurrence and raised a common question of law or fact FED R CIV P 20(a) The facts do not suggest that Mother had any reason not to join Sonrsquos suit other than a desire to see how Sonrsquos action concluded before bringing her own claim Cf Nations v Sun Oil Co (Del) 695 F2d 933 938 (5th Cir 1983) (concluding that plaintiff ldquowas entitled to await the development of his injuries and their predictable consequencesrdquo) Because it appears that Mother may be a ldquowait-and-seerdquo plaintiff who could easily have joined the original action a trial court might disallow as a matter of discretion her use of non-mutual issue preclusion]

30

AGENCY ANALYSIS __________ (Agency I II)

ANALYSIS

Legal Problems

(1) Is the principal or the agent or both liable on contracts with a third party when the principal is an ldquoundisclosed principalrdquo

(2) Is the principal or the agent or both liable on contracts with a third party when the principal is ldquopartially disclosedrdquo or an ldquounidentified principalrdquo

(3) Is the principal or the agent or both liable on contracts with a third party for the purchase of goods when the agent exceeded his authority but the principal nonetheless accepts the goods

DISCUSSION

Summary

The agent but not the owner is liable to the basket manufacturer because the owner is an undisclosed principal and the agent acted without actual or apparent authority Both the agent and the owner however are liable on the burner contract because the owner is an unidentified principal and the agent had apparent authority to enter into that contract With respect to the solar cells contract whether the owner is liable depends upon whether a court would follow the Second or Third Restatement of Agency which take different positions on the effect of the ratification of a contract by an undisclosed principal Under either the agent would also be liable on the contract as he was a party to the contract

[NOTE The contracts that are the subject of this question are contracts for the sale of goods and therefore are governed by Article 2 of the Uniform Commercial Code Article 2 however does not contain agency rules Accordingly common law concepts of agency are applicable UCC sect 1-103(b)]

Point One (35) The agent but not the owner is liable to the basket manufacturer The agent had no actual authority to enter into the contract to buy aluminum baskets and because the owner was an undisclosed principal the manufacturer had no reason to believe that the agent had apparent authority Furthermore the manufacturer had no reason to believe that the agent was not contracting for his own benefit

An agent acting on behalf of a principal can bind the principal to contracts if the agent has either actual or apparent authority An agent has actual authority when contracting on behalf of his principal if he ldquoreasonably believes in accordance with the principalrsquos manifestations to the agent that the principal wishes the agent so to actrdquo RESTATEMENT (THIRD) OF AGENCY sect 201 (2006) Here the agent was told to buy only wicker baskets not aluminum baskets Thus when he contracted with the basket manufacturer to buy aluminum baskets he had no actual authority to do so

31

Agency Analysis

An agent acts with apparent authority ldquowhen a third party [with whom the agent acts] reasonably believes the actor has authority to act on behalf of the principal and that belief is traceable to the principalrsquos manifestationsrdquo Id sect 203 Here the owner notified basket manufacturers that she or her agent might contact them to purchase baskets but that notification did not specifically name the agent or any other person as the ownerrsquos agent Furthermore the basket manufacturer had no prior dealings with the agent or the owner or any reason to think that the agent was acting for the benefit of anyone but himself Thus there is no basis to conclude that the basket manufacturer thought that the agent had apparent authority to act for the owner

Generally when an agent acts on behalf of an undisclosed principal and the agent lacks authority to enter into the contract the agent is liable on the contract as a party to the contract but the principal is not liable This rule is consistent with the third partyrsquos expectations ldquoThe third party expected the agent to be a party to the contract because the agent presented the deal as if he were acting for himself Moreover if the third party is unaware of the principalrsquos existence the third party must be relying on the agentrsquos solvency and reliability when entering into the contractrdquo See ROBERT W HAMILTON JONATHAN R MACEY amp DOUGLAS K MOLL CORPORATIONS INCLUDING PARTNERSHIPS AND LIMITED LIABILITY COMPANIES 34 (11th ed 2010) See also RESTATEMENT (THIRD) OF AGENCY sect 603 cmt c Furthermore because the third party has no idea that the agent is acting or is seemingly acting on behalf of another there is no reason to believe that the third party would be expecting an undisclosed principal to be liable on the contract Id

Point Two (35) Because the owner is an unidentified (as opposed to undisclosed) principal both she and the agent (as a party to the contract) probably are liable on the contract with the burner manufacturer

When the agent contracted with the burner manufacturer he did not have actual authority to do so as the owner had expressly restricted the agentrsquos authority to purchase only burners with ldquowhisper technologyrdquo See Point One However the agent may have had apparent authority to buy burners without whisper technology

An agent acts with apparent authority ldquowhen a third party [with whom the agent acts] reasonably believes the actor has authority to act on behalf of the principal and that belief is traceable to the principalrsquos manifestationsrdquo RESTATEMENT (THIRD) OF AGENCY sect 203 (2006) The owner indicated that an agent might contact the burner manufacturer The notice contained no restriction regarding the type of burners that the agent was authorized to purchase The facts indicate that burner manufacturers regularly receive such notices

Although the agent told the burner manufacturer that he represented a well-known hot-air balloon operator he did not disclose the ownerrsquos name Thus the owner was a partially disclosed or unidentified principal See RESTATEMENT (SECOND) OF AGENCY sect 4(2) (1958) (using term ldquopartially disclosed principalrdquo) RESTATEMENT (THIRD) OF AGENCY sect 104(2)(c) (2006) (using term ldquounidentified principalrdquo) An agent for a partially disclosed principal may have apparent authority RESTATEMENT (SECOND) OF AGENCY sect 159 cmt e (1958) Based upon (1) the notice sent by the owner (2) the agentrsquos revelation that he was acting as an agent and (3) the fact that burner manufacturers regularly receive such notices and sell to agents the manufacturer may argue that it reasonably and actually believed that the agent was authorized to purchase burners without whisper technology The manufacturer may also argue that because the agent revealed that he was an agent his listing of the ownerrsquos address as the delivery address connects the agent to the notice given by the owner Arguably this distinguishes the burner contract from the basket

32

Agency Analysis

contract Here there is a strong case to support the conclusion that the agent had apparent authority if he did then the owner is liable to the burner manufacturer

The agent also is liable as a party to the contract because he did not fully disclose his agency relationship Although he told the burner manufacturer that he represented a well-known hot-air balloon operator he did not disclose the ownerrsquos name Generally even an authorized agent of a partially disclosed or unidentified principal is liable as a party to a contract with a third person RESTATEMENT (SECOND) OF AGENCY sect 321 (1958) (ldquounless otherwise agreedrdquo) RESTATEMENT (THIRD) OF AGENCY sect 602(2) (2006) (ldquounless the agent and the third party agree otherwiserdquo)

Point Three (30) Under the Second Restatement of Agency the owner is not liable on the contract for solar cells because the agent did not have actual or apparent authority and the owner as an undisclosed principal cannot ratify the contract Under the Third Restatement the owner could be liable as she ratified the contract Under either Restatement the agent is liable as a party to the contract

The owner is not liable to the solar cell manufacturer for breach of the contract for the solar cells because the agent had no actual or apparent authority to purchase solar cells on the ownerrsquos behalf and the owner under the Second Restatement of Agency did not ratify the contract with knowledge of the material facts Thus she is not liable as a ratifier of the contract

The facts state that the agent had authority to purchase only propane fuel tanks In addition he had no apparent authority to purchase solar cells The owner made no manifestations to the solar cell manufacturer that would lead a reasonable person in the manufacturerrsquos position to believe that the agent had the authority to bind the owner to a contract to purchase solar cells In fact the agent made no manifestations at all to the solar cell manufacturer Unlike with the basket manufacturer and the burner manufacturer the owner did not notify the manufacturer of solar cells that an agent might contact it to purchase solar cells In addition the solar cells were delivered to the agent and not to the ownerrsquos address In sum the manufacturer was unaware of any relationship between the owner and the agent As to the solar cell manufacturer the owner is an undisclosed principal There can be no apparent authority in the case of an undisclosed principal because there are no manifestations from the principal to the third person See RESTATEMENT (SECOND) OF AGENCY sect 8 cmt a (1958) (ldquothere can be no apparent authority created by an undisclosed principalrdquo) RESTATEMENT (THIRD) OF AGENCY sect 203 cmt f (2006) (ldquoapparent authority is not present when a third party believes that an interaction is with an actor who is a principalrdquo)

The owner also did not ratify the contract Although the owner used the solar cells generally a principal cannot ratify an unauthorized transaction with a third person ldquounless the one acting purported to be acting for the ratifierrdquo RESTATEMENT (SECOND) OF AGENCY sect 85(1) (1958)

The result differs under the Third Restatement which expressly rejects the Second Restatement on this issue The Restatement (Third) of Agency sect 403 (2006) states ldquoA person may ratify an act if the actor acted or purported to act as an agent on the personrsquos behalfrdquo According to comment b ldquoan undisclosed principal may ratify an agentrsquos unauthorized actrdquo Under the Restatement (Third) of Agency rule the owner probably ratified the transaction The agent clearly acted on the ownerrsquos behalf and in addition the ownerrsquos conduct in using the solar cells ldquojustifies a reasonable assumption that [she] is manifesting assent that the act shall affect [her] legal relationsrdquo See id sect 401(2)

33

Agency Analysis

The agent also is liable to the solar cell manufacturer for breach of the contract for the solar cells because he is a party to the contract The facts indicate that the agent never told the solar cell manufacturer that he represented the owner or any other principal Consequently even if the agent were authorized (which as discussed above he is not) he would be liable as a party to the contract See RESTATEMENT (SECOND) OF AGENCY sect 322 (1958) RESTATEMENT (THIRD) OF AGENCY sect 603(2) (2006) Here he has no authority or apparent authority and is liable as a party to the contract

The agent would also be liable under the Third Restatement Under Restatement (Third) of Agency sect 402(1) (2006) ratification generally relates back and the transaction is treated as if it were authorized at the time of the transaction However this does not relieve the agent of an undisclosed principal who ratifies an unauthorized transaction of liability under the ratified contract See id sect 603(2) (authorized agent for undisclosed principal is a party to the contract) and sect 403 cmt b (ldquoAn undisclosed principalrsquos ratification does not eliminate the agentrsquos liability to the third party on the transaction rdquo)

[NOTE An examinee may discuss the concept of inherent agency power This concept is recognized by the Restatement (Second) of Agency sect 8 A (1958) but the concept is not used in the Restatement (Third) of Agency (2006) Here there are no facts to support that the agent had inherent authority

As to contracts with agents for partially disclosed principals (eg the contract for the burners) the basic question is whether the acts done ldquousually accompany or are incidental to transactions which the agent is authorized to conductrdquo RESTATEMENT (SECOND) OF AGENCY

sect 161 (1958) If so the principal is bound if the other party ldquoreasonably believes that the agent is authorized to do them and has no notice that he is not so authorizedrdquo Id The purchase of burners without whisper technology was not authorized nor was it incidental to an authorized transaction Therefore there should not be inherent agency power

As to contracts on behalf of undisclosed principals (eg the other two contracts) the basic question is whether the acts done are usual or necessary in the transactions the agent is authorized to transact RESTATEMENT (SECOND) OF AGENCY sect 194 (1958) The other two contracts seem fundamentally different from the authorized transactions Therefore there should not be inherent agency power

Only minimal credit should be given for discussion of inherent agency power]

34

EVIDENCE ANALYSIS _____ (Evidence IIA VA B E F J K)

ANALYSIS

Legal Problems

(1) Is the authenticated copy of the mechanicrsquos text message relevant and admissible

(2) Is the womanrsquos question ldquoIs my scooter safe to drive for a whilerdquo relevant and admissible

(3) Is the womanrsquos testimony describing the mechanicrsquos thumbs-up relevant and admissible

DISCUSSION

Summary

The mechanicrsquos text message to the woman is relevant to whether (1) the woman lost control of the scooter due to its defective brakes (2) the woman knew that the brakes needed repair and (3) it was negligent for the woman to drive the scooter knowing that its brakes needed repair

The mechanicrsquos text message is hearsay if it is offered by the pedestrian to prove that the scooterrsquos brakes needed repair However it fits the hearsay exception for present sense impressions and probably also fits the exception for business records The mechanicrsquos text message is not hearsay if it is instead offered by the pedestrian to prove the womanrsquos state of mind (ie that she had notice that her brakes needed repair)

The womanrsquos question to the mechanic and his response are also relevant to whether the brakes caused the accident and whether the woman was negligent The question is not hearsay because the woman did not make an assertion

The mechanicrsquos thumbs-up response is nonverbal conduct intended by the mechanic as an assertion and is therefore an out-of-court statement If the woman offers the mechanicrsquos statement to prove that the scooter was actually safe to ride the womanrsquos testimony about the statement is hearsay

However the mechanicrsquos statement is not hearsay if it is offered by the woman to prove her state of mind Therefore the womanrsquos question and the mechanicrsquos response are admissible to prove the womanrsquos state of mind

Point One(a) (20) The mechanicrsquos text message to the woman should be admitted because it is relevant

Evidence is relevant if it has ldquoany tendency to make a fact more or less probable than it would be without the evidencerdquo FED R EVID 401 ldquoRelevant evidence is admissiblerdquo unless it is inadmissible pursuant to some other rule FED R EVID 402

The mechanicrsquos text message to the woman ldquoWhen you pick up your scooter you need to schedule a follow-up brake repair Wersquoll order the partsrdquo is relevant for two reasons First this evidence has some tendency to make it more probable that the brakes malfunctioned and

35

Evidence Analysis

caused the accident Second it has some tendency to make it more probable that the woman was negligent in riding her scooter after being told by the mechanic that it required further repair

Point One(b) (30) The mechanicrsquos text message fits either the hearsay exception for present sense impressions or the exception for business records or it is admissible non-hearsay

The mechanicrsquos text message is a statement under Rule 801(a) because it is ldquoa written assertionrdquo FED R EVID 801(a) The text message is hearsay if the pedestrian offers it to prove the ldquotruth of the matter asserted in the statementrdquo (ie that the scooterrsquos brakes required repair) which resulted in the woman losing control of the scooter and causing the accident FED R EVID 801(c)

However the mechanicrsquos text message fits the hearsay exception for ldquopresent sense impressionsrdquo under Rule 803(1) because it is ldquo[a] statement describing or explaining an event or condition made while or immediately after the declarant perceived itrdquo FED R EVID 803(1) Here the mechanicrsquos text message described the condition of the scooter immediately after he perceived it during the maintenance service

The mechanic is a person with knowledge of the condition of the scooter so if text messages regarding repairs were made and kept by the mechanic in the ordinary course of business this text message also fits the business records exception Under Rule 803(6) a business record is a record of an act ldquomade at or near the time by someone with knowledgerdquo and ldquothe record was kept in the course of a regularly conducted activity of a businessrdquo and ldquomaking the record was a regular practice of that activityrdquo FED R EVID 803(6)

However the text message is not hearsay if it is instead offered to prove that the woman was negligent because she rode her scooter after the mechanic told her it required repair If offered for this purpose it would not be offered for the truth of the matter asserted in the statement but to show the womanrsquos belief about the condition of the scooter (her state of mind)

Point Two (10) The womanrsquos question to the mechanic should be admitted because it is not hearsay

The womanrsquos question to the mechanic is relevant because along with the mechanicrsquos thumbs-up response (see Point Three) it has some tendency to make it more probable that the woman was not negligent andor that the scooter brakes did not malfunction and cause the accident FED R EVID 401 The womanrsquos question does not raise hearsay concerns because it is not an assertion

Hearsay is defined under Rule 801(a) as ldquoan oral assertion written assertion or nonverbal conductrdquo Although ldquoassertionrdquo is not further defined ldquoa favorite [definition] of writers in the [evidence] field for at least a century and a half [is that] the word simply means to say that something is so eg that an event happened or a condition existedrdquo 2 MCCORMICK ON

EVIDENCE sect 246 (6th ed 2006) Under this definition the womanrsquos question is not hearsay because it is not an assertion

Point Three(a) (20) The mechanicrsquos thumbs-up to the woman is a nonverbal assertion that is relevant and the womanrsquos testimony about that response is admissible

36

Evidence Analysis

Hearsay is defined under Rule 801(c) as a ldquostatementrdquo that is ldquoa personrsquos oral assertion written assertion or nonverbal conduct if the person intended it as an assertionrdquo FED R EVID 801(a) Here when the mechanic responded to the womanrsquos question (ldquoIs my scooter safe to ride for a whilerdquo) with a thumbs-up gesture the facts suggest that he intended his nonverbal conduct as an assertion that in his opinion the scooter was safe to ride

The mechanicrsquos assertion is relevant and admissible to prove that the woman was not negligent because the evidence makes it more probable that at the time of the accident she believed that the scooter was safe to ride despite the fact that the brakes required repair FED R EVID 401 Admission of the womanrsquos description of the mechanicrsquos thumbs-up for this purpose does not raise hearsay concerns because the evidence would not be offered for the truth of the matter asserted but to show the womanrsquos belief about the condition of the scooter (her state of mind)

Point Three(b) (20) The mechanicrsquos thumbs-up is relevant to determine whether the scooterrsquos brakes malfunctioned causing the accident but if offered for this purpose it is also hearsay

The mechanicrsquos nonverbal assertion is relevant to the determination of whether the scooterrsquos brakes malfunctioned causing the accident However if offered to prove the ldquotruth of the matter asserted in the statementrdquo (ie that the scooter was safe to ride for a while) it is hearsay that does not fit any hearsay exception

37

TRUSTS AND FUTURE INTERESTS ANALYSIS ____________________ (Trusts and Future Interests IC1 amp 4 G IIF)

ANALYSIS

Legal Problems

(1)(a) Was the revocable trust amendable

(1)(b) If the trust was amendable must the amendment have been executed in accordance with the state Statute of Wills in order to be valid

(2) If the trust amendment was valid does the amendment apply to the probate estate assets passing to the trust pursuant to Settlorrsquos will

(3) If the trust amendment was valid should the trust property be distributed to University

(4) If the trust amendment was not valid should the trust property be distributed to Settlorrsquos grandchild (her only heir) or held in further trust in accordance with the terms of the original trust instrument

DISCUSSION

Summary

A revocable trust is amendable even if the trust instrument does not expressly grant to the trust settlor a power to amend Both inter vivos trusts and amendments thereto are valid even though not executed in accordance with the requirements applicable to wills

Under the Uniform Testamentary Additions to Trusts Act a revocable trust may be amended at any time prior to the settlorrsquos death and the amendment applies to the disposition of assets conveyed to the trust pursuant to a will even if the will was executed prior to the date of the amendment

At Settlorrsquos death trust assets including probate assets passing to the trust under Settlorrsquos will would go to University if as is the case here the trust amendment was valid If the amendment was invalid the trust assets would continue to be held in further trust because there is no violation of the common law Rule Against Perpetuities

Point One(a) (30) Settlor retained the right to amend the inter vivos trust despite her failure to expressly reserve this power

At issue here is whether a retained power of revocation includes the power to amend sometimes referred to as the power to modify The Restatement (Second) of Trusts sect 331 cmt g provides that if a settlor has a power to revoke that retained power ordinarily includes a power to modify (amend) as well Comment g also notes that the power to amend includes both a power to withdraw trust assets and a power to ldquomodify the terms of the trustrdquo The Uniform Trust Code which provides that a power to revoke includes the power to amend is consistent with this view

38

Trusts and Future Interests Analysis

UNIF TRUST CODE sect 602 accord RESTATEMENT (THIRD) OF TRUSTS sect 63 cmt The theory is that even though a power to amend was not expressly retained by a settlor the goal of amendment assuming the power was not included in the power to revoke could easily be achieved by first revoking the trust and then creating a new trust with the same terms contemplated by the amendment To require this would put form over substance

Thus by expressly retaining the power to revoke the trust Settlor retained a power to amend the inter vivos trust despite her failure to expressly reserve this power

[NOTE Under the common law a trust is irrevocable unless the settlor expressly retains a power to revoke the trust Conversely under the Uniform Trust Code a trust is revocable unless the terms of the trust expressly provide otherwise See UNIF TRUST CODE sect 602 The Trust Codersquos position on revocation follows the minority view in the United States and is inconsistent with prior Restatements of Trusts (see Restatement (Second) of Trusts sect 330) Here the trust is revocable because Settlor expressly retained a power of revocation

The Uniform Trust Code has been adopted in 24 jurisdictions Alabama Arizona Arkansas District of Columbia Florida Kansas Maine Michigan Missouri Nebraska New Hampshire New Mexico North Carolina North Dakota Ohio Oregon Pennsylvania South Carolina Tennessee Utah Vermont Virginia West Virginia and Wyoming]

Point One(b) (10) Settlorrsquos amendment of the trust was valid despite her failure to have her signature to the trust amendment witnessed

Neither the common law nor state statutes require a trust instrument or an amendment to a trust instrument to be executed in accordance with the formalities prescribed for execution of a will Indeed an inter vivos trust that does not involve real estate can be created orally Under the Uniform Trust Code the only requirements for creating a valid inter vivos trust are intent the specification of beneficiaries and the designation of a trustee See UNIF TRUST CODE sect 402 accord RESTATEMENT (THIRD) OF TRUSTS sect 13

Here the amendment meets the requirements of both the Uniform Trust Code and the common law Thus the fact that Settlorrsquos signature was not witnessed when she signed the amendment to the trust does not make the amendment invalid

Point Two (20) Under the Uniform Testamentary Additions to Trusts Act a revocable trust may be amended at any time prior to the settlorrsquos death and the amendment applies to probate assets poured into the trust at the settlorrsquos death pursuant to the settlorrsquos will even when the will was executed prior to the date of the amendment

Historically property owned by an individual at her death passed to the individualrsquos heirs or to beneficiaries designated in a will executed with the formalities (writing signing witnessing) prescribed by state law However when a will devises property to the trustee of an inter vivos trust then the provisions of the trustmdashwhich may not have been executed in accordance with the formalities required for willsmdasheffectively determine who will receive the property Because of this possibility some early cases held that if an inter vivos trust was not executed with the same formalities required for a valid will then the trust was ineffective to dispose of probate assets poured into the trust at the settlorrsquos death pursuant to the settlorrsquos will

This line of cases has been overturned by the Uniform Testamentary Additions to Trusts Act (the Act) now Uniform Probate Code sect 2-511 Under the Act adopted in almost all

39

Trusts and Future Interest Analysis

jurisdictions a testamentary bequest to the trustee of an inter vivos trust established by the testator during his or her lifetime is valid if the trust is in writing it is identified in the testatorrsquos will and the trust instrument was executed before concurrently with or after the execution of the will Id The Act further specifies that such a bequest is valid even if the trust is amendable or revocable and that a later amendment applies to assets passing to the trust by a previously executed will

Thus because the trust amendment is valid its terms apply to assets received by Bank from Settlorrsquos estate

Point Three (10) If the trust amendment was valid then the trust assets including assets passing to the trust under Settlorrsquos will should go to University

Under the trust amendment all trust assets (including the assets of Settlorrsquos probate estate poured into the trust) pass to University The facts provide no basis for failing to comply with Settlorrsquos stated intentions

Point Four (30) If the trust amendment was invalid trust assets including assets received pursuant to Settlorrsquos will should be held in accordance with the terms of the original trust instrument because those terms do not violate the Rule Against Perpetuities

Under the dispositive terms of the original trust instrument Settlor created successive income interests in her surviving children and grandchildren with a remainder interest in her great-grandchildren Because the trust was revocable the period during which the common law Rule Against Perpetuities requires that interests vest (ie 21 years plus lives in being) began to run from the date Settlor no longer had a power of revocation (here her death) not the date on which the trust was created See JESSE DUKEMINIER STANLEY J JOHANSON JAMES LINDGREN amp ROBERT SITKOFF WILLS TRUSTS AND ESTATES 678 (7th ed 2005)

Under the common law Rule Against Perpetuities Settlorrsquos trust is thus valid At the time of Settlorrsquos death she was survived by no children one granddaughter and no great-grandchildren Because Settlor cannot have more children after her death the only income beneficiary of the trust is Settlorrsquos surviving granddaughter This granddaughter is the only person who can produce great-grandchildren of Settlor thus all great-grandchildren must of necessity be born during the lifetime of Settlorrsquos only surviving granddaughter who is a life in being The granddaughterrsquos interest vested at Settlorrsquos death and the great-grandchildrenrsquos interest will vest at the death of the granddaughter There is no need to wait the additional 21 years permitted under the Rule Thus under the common law and the statute given in the facts the nonvested interest in the great-grandchildren is valid

[NOTE Both modern wait-and-see statutes and the Uniform Statutory Rule Against Perpetuities upon which the statute in the facts is modeled provide that before using either reform to validate an otherwise invalid nonvested interest one should first determine if the nonvested interest violates the common law Rule If it does not then there is no need to reform This proposition which is applicable in all MEE user jurisdictions that have not simply abrogated the rule is tested by this problem]

40

NEGOTIABLE INSTRUMENTS ANALYSIS (Negotiable Instruments III IV V)

ANALYSIS

Legal Problems

(1)(a) What rights does a person in possession of a note that has been indorsed in blank by the payee have against the maker of the note

(1)(b) Which defenses may the maker of a note raise against a person entitled to enforce it who is not a holder in due course but is a transferee from a holder in due course

(2) What rights does a person entitled to enforce a note have against an indorser who transferred it for consideration with no warranties

(3) What rights does a person entitled to enforce a note have against a previous holder who transferred it as a gift without indorsing it

DISCUSSION

Summary

The niece is a holder of the note and is thus a person entitled to enforce it The chef the issuer of the note is obligated to pay it to the niece as the person entitled to enforce it The niece is not subject to any defense or claim of the chef relating to the improper repair of the oven because the niece has the rights of a holder in due course When the buyer bought the note from the repairman the buyer became a holder in due course of the note and thus took it free of any personal defenses the chef had against the repairman Even though the niece is not herself a holder in due course of the note the niece succeeded to the buyerrsquos rights as holder in due course and thus took free of the chefrsquos personal defenses

Because the chef refused to pay the note the niece can recover from the repairman on the repairmanrsquos obligation as indorser The niece cannot recover on the note against the buyer however because the buyer did not indorse the note (and thus incurred no indorserrsquos obligation) and the buyer did not receive any consideration for transfer of the note to the niece (and therefore made no transfer warranty)

[NOTE Although Article 9 of the Uniform Commercial Code governs the sale of promissory notes (a point that might be correctly noted by examinees) that Article does not determine the answer to any of the questions posed]

Point One(a) (20) The niece is the holder of the note and thus may enforce it against the chef who is the issuer of the note

The chef is the maker of the note and thus its issuer See UCC sectsect 3-103 3-105 The issuer of a note is obligated to pay it in accordance with its terms to a ldquoperson entitled to enforcerdquo it UCC sect 3-412 The niece is a ldquoperson entitled to enforcerdquo the note This is because the niece is the holder of the note and a holder of a note is a person entitled to enforce it UCC sect 3-301 The niece is the holder of the note because (i) the repairmanrsquos signature on the back of the note not

41

Negotiable Instruments Analysis

accompanied by words indicating a person to whom the note was made payable was a ldquoblank indorsementrdquo which had the effect of making the note a bearer instrument (ii) anyone in possession of a bearer instrument is a holder of it and (iii) the niece is in possession of the note See UCC sectsect 1-201(b)(21)(A) 3-204 and 3-205 Accordingly the chef has an obligation to the niece to pay the note in accordance with its terms and the niece may enforce that obligation

Point One(b) (40) The niece is not a holder in due course of the note but because she is a transferee from the buyer who was a holder in due course she has the same enforcement rights as the buyer Because the buyer as a holder in due course would have been able to enforce the note against the chef without being subject to defenses or claims arising from the improper repair the niece has the same rights and will not be subject to the chefrsquos defenses or claims about the repair

As noted in Point One(a) the chef has an obligation to the niece to pay the note in accordance with its terms However except against a person with the rights of a holder in due course the chef can raise any defenses or claims in recoupment that he would have if the claim on the note were an ordinary contract claim UCC sect 3-305 Thus except against a holder in due course the chef would be able to raise the improper repair as a defense or a claim in recoupment (a claim in response to the niecersquos claim)

But claims in recoupment and most defenses cannot be raised against a person with the rights of a holder in due course Against a holder in due course the chef can raise only the four ldquorealrdquo defenses listed in UCC sect 3-305(a)(1) (infancy duress lack of legal capacity or illegality that nullifies the obligation of the obligor under other law fraud in the factum discharge in insolvency proceedings) none of which is present here

The niece is not a holder in due course because she did not take the note for value See UCC sectsect 3-302(a)(2)(i) (criteria for holder in due course status) and 3-303(a) (definition of ldquovaluerdquo) But this does not mean that the niece is subject to the chefrsquos claim arising out of the improper repair The buyer was a holder in due course of the note because he took the note for value ($9500) in good faith and without notice of any facts that would have alerted him to the chefrsquos defense against the repairman UCC sect 3-302(a)(2) As a holder in due course the buyer owned the note free of the chefrsquos claim because that claim did not constitute a ldquorealrdquo defense UCC sect 3-305(b) When the buyer gave the note to the niece this constituted a ldquotransferrdquo of the note See UCC sect 3-203(a) When a note is transferred the transferee receives ldquoany right of the transferor to enforce the instrument including any right as a holder in due courserdquo UCC sect 3-203(b) Under this rule (also known as the ldquoshelter principlerdquo) the buyer transferred his freedom from the chefrsquos defenses to the niece and the niece can enforce the note free of the chefrsquos defenses

Point Two (20) Because the chef dishonored the note the niece can recover from the repairman on the repairmanrsquos obligation as indorser

The chefrsquos refusal to pay the note constituted dishonor See UCC sect 3-502 The repairman as an indorser of the note (see Point One(a)) incurred the obligations of an indorser under UCC sect 3-415(a) When a note has been dishonored one of the obligations of an indorser is to pay the amount of the note to a person entitled to enforce it Therefore the repairman is liable for the amount of the note to the niece a person entitled to enforce the note (so long as the niece gives proper notice of dishonor to the repairman)

42

Negotiable Instruments Analysis

[NOTE Because the repairman indorsed the note without warranties there are no transfer warranties UCC sect 3-416 cmt 5]

Point Three (20) The niece cannot recover on the note against the buyer as either indorser or warrantor because the buyer did not indorse the note and did not receive consideration for transferring the note to the niece

The buyer did not indorse the note and therefore did not incur the obligation of an indorser to pay the note upon dishonor

The niece cannot recover from the buyer under a transfer warranty theory because transfer warranties are made only by a person ldquowho transfers an instrument for considerationrdquo Here the buyer gave the instrument to the niece as a gift So the buyer made no transfer warranty UCC sect 3-416(a) Therefore the niece cannot recover from the buyer on that theory

43

National Conference of Bar Examiners 302 South Bedford Street | Madison WI 53703-3622 Phone 608-280-8550 | Fax 608-280-8552 | TDD 608-661-1275

wwwncbexorg e-mail contactncbexorg

  • Contents
  • Preface
  • Description of the MEE
  • Instructions
  • February 2013 Questions
    • Real Property Question
    • Contracts Question
    • Constitutional Law Question
    • Secured Transactions Question
    • Federal Civil Procedure Question
    • Agency Question
    • Evidence Question
    • Trusts and Future Interests Question
    • Negotiable Instruments Question
      • February 2013 Analyses
        • Real Property Analysis
        • Contracts Analysis
        • Constitutional Law Analysis
        • Secured Transactions Analysis
        • Federal Civil Procedure Analysis
        • Agency Analysis
        • Evidence Analysis
        • Trusts and Future Interests Analysis
        • Negotiable Instruments Analysis
            • ltlt ASCII85EncodePages false AllowTransparency false AutoPositionEPSFiles true AutoRotatePages None Binding Left CalGrayProfile (Dot Gain 20) CalRGBProfile (sRGB IEC61966-21) CalCMYKProfile (US Web Coated 050SWOP051 v2) sRGBProfile (sRGB IEC61966-21) CannotEmbedFontPolicy Error CompatibilityLevel 14 CompressObjects Tags CompressPages true ConvertImagesToIndexed true PassThroughJPEGImages true CreateJobTicket false DefaultRenderingIntent Default DetectBlends true DetectCurves 00000 ColorConversionStrategy CMYK DoThumbnails false EmbedAllFonts true EmbedOpenType false ParseICCProfilesInComments true EmbedJobOptions true DSCReportingLevel 0 EmitDSCWarnings false EndPage -1 ImageMemory 1048576 LockDistillerParams false MaxSubsetPct 100 Optimize true OPM 1 ParseDSCComments true ParseDSCCommentsForDocInfo true PreserveCopyPage true PreserveDICMYKValues true PreserveEPSInfo true PreserveFlatness true PreserveHalftoneInfo false PreserveOPIComments true PreserveOverprintSettings true StartPage 1 SubsetFonts true TransferFunctionInfo Apply UCRandBGInfo Preserve UsePrologue false ColorSettingsFile () AlwaysEmbed [ true ] NeverEmbed [ true ] AntiAliasColorImages false CropColorImages true ColorImageMinResolution 300 ColorImageMinResolutionPolicy OK DownsampleColorImages true ColorImageDownsampleType Bicubic ColorImageResolution 300 ColorImageDepth -1 ColorImageMinDownsampleDepth 1 ColorImageDownsampleThreshold 150000 EncodeColorImages true ColorImageFilter DCTEncode AutoFilterColorImages true ColorImageAutoFilterStrategy JPEG ColorACSImageDict ltlt QFactor 015 HSamples [1 1 1 1] VSamples [1 1 1 1] gtgt ColorImageDict ltlt QFactor 015 HSamples [1 1 1 1] VSamples [1 1 1 1] gtgt JPEG2000ColorACSImageDict ltlt TileWidth 256 TileHeight 256 Quality 30 gtgt JPEG2000ColorImageDict ltlt TileWidth 256 TileHeight 256 Quality 30 gtgt AntiAliasGrayImages false CropGrayImages true GrayImageMinResolution 300 GrayImageMinResolutionPolicy OK DownsampleGrayImages true GrayImageDownsampleType Bicubic GrayImageResolution 300 GrayImageDepth -1 GrayImageMinDownsampleDepth 2 GrayImageDownsampleThreshold 150000 EncodeGrayImages true GrayImageFilter DCTEncode AutoFilterGrayImages true GrayImageAutoFilterStrategy JPEG GrayACSImageDict ltlt QFactor 015 HSamples [1 1 1 1] VSamples [1 1 1 1] gtgt GrayImageDict ltlt QFactor 015 HSamples [1 1 1 1] VSamples [1 1 1 1] gtgt JPEG2000GrayACSImageDict ltlt TileWidth 256 TileHeight 256 Quality 30 gtgt JPEG2000GrayImageDict ltlt TileWidth 256 TileHeight 256 Quality 30 gtgt AntiAliasMonoImages false CropMonoImages true MonoImageMinResolution 1200 MonoImageMinResolutionPolicy OK DownsampleMonoImages true MonoImageDownsampleType Bicubic MonoImageResolution 1200 MonoImageDepth -1 MonoImageDownsampleThreshold 150000 EncodeMonoImages true MonoImageFilter CCITTFaxEncode MonoImageDict ltlt K -1 gtgt AllowPSXObjects false CheckCompliance [ None ] PDFX1aCheck false PDFX3Check false PDFXCompliantPDFOnly false PDFXNoTrimBoxError true PDFXTrimBoxToMediaBoxOffset [ 000000 000000 000000 000000 ] PDFXSetBleedBoxToMediaBox true PDFXBleedBoxToTrimBoxOffset [ 000000 000000 000000 000000 ] PDFXOutputIntentProfile () PDFXOutputConditionIdentifier () PDFXOutputCondition () PDFXRegistryName () PDFXTrapped False CreateJDFFile false Description ltlt ARA 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 BGR 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 CHS ltFEFF4f7f75288fd94e9b8bbe5b9a521b5efa7684002000410064006f006200650020005000440046002065876863900275284e8e9ad88d2891cf76845370524d53705237300260a853ef4ee54f7f75280020004100630072006f0062006100740020548c002000410064006f00620065002000520065006100640065007200200035002e003000204ee553ca66f49ad87248672c676562535f00521b5efa768400200050004400460020658768633002gt CHT ltFEFF4f7f752890194e9b8a2d7f6e5efa7acb7684002000410064006f006200650020005000440046002065874ef69069752865bc9ad854c18cea76845370524d5370523786557406300260a853ef4ee54f7f75280020004100630072006f0062006100740020548c002000410064006f00620065002000520065006100640065007200200035002e003000204ee553ca66f49ad87248672c4f86958b555f5df25efa7acb76840020005000440046002065874ef63002gt CZE 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 DAN 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 DEU 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 ESP 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 ETI ltFEFF004b00610073007500740061006700650020006e0065006900640020007300e4007400740065006900640020006b00760061006c006900740065006500740073006500200074007200fc006b006900650065006c007300650020007000720069006e00740069006d0069007300650020006a0061006f006b007300200073006f00620069006c0069006b0065002000410064006f006200650020005000440046002d0064006f006b0075006d0065006e00740069006400650020006c006f006f006d006900730065006b0073002e00200020004c006f006f0064007500640020005000440046002d0064006f006b0075006d0065006e00740065002000730061006100740065002000610076006100640061002000700072006f006700720061006d006d006900640065006700610020004100630072006f0062006100740020006e0069006e0067002000410064006f00620065002000520065006100640065007200200035002e00300020006a00610020007500750065006d006100740065002000760065007200730069006f006f006e00690064006500670061002e000d000agt FRA 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 GRE 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 HEB ltFEFF05D405E905EA05DE05E905D5002005D105D405D205D305E805D505EA002005D005DC05D4002005DB05D305D9002005DC05D905E605D505E8002005DE05E105DE05DB05D9002000410064006F006200650020005000440046002005D405DE05D505EA05D005DE05D905DD002005DC05D405D305E405E105EA002005E705D305DD002D05D305E405D505E1002005D005D905DB05D505EA05D905EA002E002005DE05E105DE05DB05D90020005000440046002005E905E005D505E605E805D5002005E005D905EA05E005D905DD002005DC05E405EA05D905D705D4002005D105D005DE05E605E205D505EA0020004100630072006F006200610074002005D5002D00410064006F00620065002000520065006100640065007200200035002E0030002005D505D205E805E105D005D505EA002005DE05EA05E705D305DE05D505EA002005D905D505EA05E8002E05D005DE05D905DD002005DC002D005000440046002F0058002D0033002C002005E205D905D905E005D5002005D105DE05D305E805D905DA002005DC05DE05E905EA05DE05E9002005E905DC0020004100630072006F006200610074002E002005DE05E105DE05DB05D90020005000440046002005E905E005D505E605E805D5002005E005D905EA05E005D905DD002005DC05E405EA05D905D705D4002005D105D005DE05E605E205D505EA0020004100630072006F006200610074002005D5002D00410064006F00620065002000520065006100640065007200200035002E0030002005D505D205E805E105D005D505EA002005DE05EA05E705D305DE05D505EA002005D905D505EA05E8002Egt HRV (Za stvaranje Adobe PDF dokumenata najpogodnijih za visokokvalitetni ispis prije tiskanja koristite ove postavke Stvoreni PDF dokumenti mogu se otvoriti Acrobat i Adobe Reader 50 i kasnijim verzijama) HUN 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 ITA 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 JPN ltFEFF9ad854c18cea306a30d730ea30d730ec30b951fa529b7528002000410064006f0062006500200050004400460020658766f8306e4f5c6210306b4f7f75283057307e305930023053306e8a2d5b9a30674f5c62103055308c305f0020005000440046002030d530a130a430eb306f3001004100630072006f0062006100740020304a30883073002000410064006f00620065002000520065006100640065007200200035002e003000204ee5964d3067958b304f30533068304c3067304d307e305930023053306e8a2d5b9a306b306f30d530a930f330c8306e57cb30818fbc307f304c5fc59808306730593002gt KOR ltFEFFc7740020c124c815c7440020c0acc6a9d558c5ec0020ace0d488c9c80020c2dcd5d80020c778c1c4c5d00020ac00c7a50020c801d569d55c002000410064006f0062006500200050004400460020bb38c11cb97c0020c791c131d569b2c8b2e4002e0020c774b807ac8c0020c791c131b41c00200050004400460020bb38c11cb2940020004100630072006f0062006100740020bc0f002000410064006f00620065002000520065006100640065007200200035002e00300020c774c0c1c5d0c11c0020c5f40020c2180020c788c2b5b2c8b2e4002egt LTH 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 LVI 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 NLD (Gebruik deze instellingen om Adobe PDF-documenten te maken die zijn geoptimaliseerd voor prepress-afdrukken van hoge kwaliteit De gemaakte PDF-documenten kunnen worden geopend met Acrobat en Adobe Reader 50 en hoger) NOR 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 POL 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 PTB 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 RUM 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 RUS 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 SKY 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 SLV 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 SUO 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 SVE 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 TUR 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 UKR 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 ENU (Use these settings to create Adobe PDF documents best suited for high-quality prepress printing Created PDF documents can be opened with Acrobat and Adobe Reader 50 and later) gtgt Namespace [ (Adobe) (Common) (10) ] OtherNamespaces [ ltlt AsReaderSpreads false CropImagesToFrames true ErrorControl WarnAndContinue FlattenerIgnoreSpreadOverrides false IncludeGuidesGrids false IncludeNonPrinting false IncludeSlug false Namespace [ (Adobe) (InDesign) (40) ] OmitPlacedBitmaps false OmitPlacedEPS false OmitPlacedPDF false SimulateOverprint Legacy gtgt ltlt AddBleedMarks false AddColorBars false AddCropMarks false AddPageInfo false AddRegMarks false ConvertColors ConvertToCMYK DestinationProfileName () DestinationProfileSelector DocumentCMYK Downsample16BitImages true FlattenerPreset ltlt PresetSelector MediumResolution gtgt FormElements false GenerateStructure false IncludeBookmarks false IncludeHyperlinks false IncludeInteractive false IncludeLayers false IncludeProfiles false MultimediaHandling UseObjectSettings Namespace [ (Adobe) (CreativeSuite) (20) ] PDFXOutputIntentProfileSelector DocumentCMYK PreserveEditing true UntaggedCMYKHandling LeaveUntagged UntaggedRGBHandling UseDocumentProfile UseDocumentBleed false gtgt ]gtgt setdistillerparamsltlt HWResolution [2400 2400] PageSize [612000 792000]gtgt setpagedevice

Page 3: February 2013 MEE Questions and Analyses

Contents

Preface ii

Description of the MEE ii

Instructions iii

February 2013 Questions

Real Property Question 3

Contracts Question 4

Constitutional Law Question 5

Secured Transactions Question 6

Federal Civil Procedure Question 7

Agency Question 8

Evidence Question 10

Trusts and Future Interests Question 11

Negotiable Instruments Question 12

February 2013 Analyses

Real Property Analysis 15

Contracts Analysis 19

Constitutional Law Analysis 21

Secured Transactions Analysis 24

Federal Civil Procedure Analysis 27

Agency Analysis 31

Evidence Analysis 35

Trusts and Future Interests Analysis 38

Negotiable Instruments Analysis 41

Used as one of the six questions on the February 2013 Uniform Bar Examination in Alabama Arizona Colorado Idaho Missouri Nebraska North Dakota and Utah

i

Preface

The Multistate Essay Examination (MEE) is developed by the National Conference of Bar Examiners (NCBE) This publication includes the questions and analyses from the February 2013 MEE Each test includes nine 30-minute questions user jurisdictions may elect which of the nine questions they wish to use (Jurisdictions that administer the Uniform Bar Examination [UBE] use a common set of six MEE questions as part of their bar examinations) In the actual test the questions are simply numbered rather than being identified by area of law The instructions for the test appear on page iii For more information see the MEE Information Booklet available on the NCBE website at wwwncbexorg

The model analyses for the MEE are illustrative of the discussions that might appear in excellent answers to the questions They are provided to the user jurisdictions to assist graders in grading the examination They address all the legal and factual issues the drafters intended to raise in the questions

The subjects covered by each question are listed on the first page of its accompanying analysis followed by roman numerals that refer to the MEE subject matter outline for that subject For example the Federal Civil Procedure question on the February 2013 MEE tested the following area from the Federal Civil Procedure outline VIE Verdicts and judgmentsmdashEffect claim and issue preclusion Subject matter outlines are included in the MEE Information Booklet

Description of the MEE

The MEE consists of nine 30-minute essay questions any of which a jurisdiction may select to include as part of its bar examination (UBE jurisdictions use a common set of six MEE questions as part of their bar examinations) It is administered by participating jurisdictions on the Tuesday before the last Wednesday in February and July of each year The areas of law that may be covered by the questions on any MEE are Business Associations (Agency and Partnership Corporations and Limited Liability Companies) Conflict of Laws Constitutional Law Contracts Criminal Law and Procedure Evidence Family Law Federal Civil Procedure Real Property Torts Trusts and Estates (Decedentsrsquo Estates Trusts and Future Interests) and Uniform Commercial Code (Negotiable Instruments and Bank Deposits and Collections Secured Transactions) Some questions may include issues in more than one area of law The particular areas covered vary from exam to exam

The purpose of the MEE is to test the examineersquos ability to (1) identify legal issues raised by a hypothetical factual situation (2) separate material which is relevant from that which is not (3) present a reasoned analysis of the relevant issues in a clear concise and well-organized composition and (4) demonstrate an understanding of the fundamental legal principles relevant to the probable solution of the issues raised by the factual situation The primary distinction between the MEE and the Multistate Bar Examination (MBE) is that the MEE requires the examinee to demonstrate an ability to communicate effectively in writing

ii

_____

Instructions

The back cover of each test booklet contains the following instructions

You will be instructed when to begin and when to stop this test Do not break the seal on this booklet until you are told to begin

You may answer the questions in any order you wish Do not answer more than one question in each answer booklet If you make a mistake or wish to revise your answer simply draw a line through the material you wish to delete

If you are using a laptop computer to answer the questions your jurisdiction will provide you with specific instructions

Read each fact situation very carefully and do not assume facts that are not given in the question Do not assume that each question covers only a single area of the law some of the questions may cover more than one of the areas you are responsible for knowing

Demonstrate your ability to reason and analyze Each of your answers should show an understanding of the facts a recognition of the issues included a knowledge of the applicable principles of law and the reasoning by which you arrive at your conclusion The value of your answer depends not as much upon your conclusions as upon the presence and quality of the elements mentioned above

Clarity and conciseness are important but make your answer complete Do not volunteer irrelevant or immaterial information

Answer all questions according to generally accepted fundamental legal principles unless your jurisdiction has instructed you to answer according to local case or statutory law (UBE instructions Answer all questions according to generally accepted fundamental legal principles rather than local case or statutory law)

iii

February 2013 MEE

QUESTIONS

Real Property Contracts

Constitutional Law Secured Transactions

Federal Civil Procedure Agency

Evidence Trusts and Future Interests

Negotiable Instruments

REAL PROPERTY QUESTION _______________

In 2008 a landlord and a tenant entered into a 10-year written lease commencing September 1 2008 for the exclusive use of a commercial building at a monthly rent of $2500 The lease contained a covenant of quiet enjoyment but no other covenants or promises on the part of the landlord

When the landlord and tenant negotiated the lease the tenant asked the landlord if the building had an air-conditioning system The landlord answered ldquoYes it doesrdquo The tenant responded ldquoGreat I will be using the building to manufacture a product that will be irreparably damaged if the temperature during manufacture exceeds 81 degrees for more than six consecutive hoursrdquo

On April 15 2012 the buildingrsquos air-conditioning system malfunctioned causing the building temperature to rise above 81 degrees for three hours The tenant immediately telephoned the landlord about this malfunction The tenant left a message in which he explained what had happened and asked the landlord ldquoWhat are you going to do about itrdquo The landlord did not respond to the tenantrsquos message

On May 15 2012 the air-conditioning system again malfunctioned This time the malfunction caused the building temperature to rise above 81 degrees for six hours The tenant telephoned the landlord and left a message describing the malfunction As before the landlord did not respond

On August 24 2012 the air-conditioning system malfunctioned again causing the temperature to rise above 81 degrees for 10 hours Again the tenant promptly telephoned the landlord The landlord answered the phone and the tenant begged her to fix the system The landlord refused The tenant then attempted to fix the system himself but he failed As a result of the air-conditioning malfunction products worth $150000 were destroyed

The next day the tenant wrote the following letter to the landlord

Irsquove had enough I told you about the air-conditioning problem twice before yesterdayrsquos disaster and you failed to correct it I will vacate the building by the end of the month and will bring you the keys when I leave

The tenant vacated the building on August 31 2012 and returned the keys to the landlord that day At that time there were six years remaining on the lease

On September 1 2012 the landlord returned the keys to the tenant with a note that said ldquoI repeat the air-conditioning is not my problem You have leased the building and you should fix itrdquo The tenant promptly sent the keys back to the landlord with a letter that said ldquoI have terminated the lease and I will not be returning to the building or making further rent paymentsrdquo After receiving the keys and letter the landlord put the keys into her desk To date she has neither responded to the tenantrsquos letter nor taken steps to lease the building to another tenant

On November 1 2012 two months after the tenant vacated the property the landlord sued the tenant claiming that she is entitled to the remaining unpaid rent ($180000) from September 1 for the balance of the lease term (reduced to present value) or if not that then damages for the tenantrsquos wrongful termination

Is the landlord correct Explain

3

CONTRACTS QUESTION _______________

On January 2 a boat builder and a sailor entered into a contract pursuant to which the builder was to sell to the sailor a boat to be specially manufactured for the sailor by the builder The contract price was $100000 The written contract signed by both parties stated that the builder would tender the boat to the sailor on December 15 at which time payment in full would be due

On October 15 the builderrsquos workers went on strike and there were no available replacements

On October 31 the builderrsquos workers were still on strike and no work was being done on the boat The sailor read a news report about the strike and immediately sent a letter to the builder stating ldquoI am very concerned that my boat will not be completed by December 15 I insist that you provide me with assurance that you will perform in accordance with the contractrdquo The builder received the letter on the next day November 1

On November 25 the builder responded to the letter stating ldquoIrsquom sorry about the strike but it is really out of my hands I hope we settle it soon so that we can get back to workrdquo

Nothing further happened until December 3 when the builder called the sailor and said ldquoMy workers are back and I have two crews working overtime to finish your boat Your boat is task one Donrsquot worry wersquoll deliver your boat by December 15thrdquo The sailor immediately replied ldquoI donrsquot trust you As far as Irsquom concerned our contract is over I am going to buy my boat from a shipyardrdquo Two days later the sailor entered into a contract with a competing manufacturer to buy a boat similar to the boat that was the subject of the contract with the builder

The builder finished the boat on time and tendered it to the sailor on December 15 The sailor reminded the builder about the December 3 conversation in which the sailor had announced that ldquoour contract is overrdquo and refused to take the boat and pay for it

The builder has sued the sailor for breach of contract

1 What was the legal effect of the sailorrsquos October 31 letter to the builder Explain

2 What was the legal effect of the builderrsquos November 25 response to the sailorrsquos October 31 letter Explain

3 What was the legal effect of the sailorrsquos refusal to take and pay for the boat on December 15 Explain

4

CONSTITUTIONAL LAW QUESTION

AutoCo is a privately owned corporation that manufactures automobiles Ten years ago AutoCo purchased a five-square-mile parcel of unincorporated land in a remote region of the state and built a large automobile assembly plant on the land To attract workers to the remote location of the plant AutoCo built apartment buildings and houses on the land and leased them to its employees AutoCo owns and operates a commercial district with shops and streets open to the general public AutoCo named the area Oakwood and provides security fire protection and sanitation services for Oakwoodrsquos residents AutoCo also built operates and fully funds the only school in the region which it makes available free of charge to the children of its employees

A family recently moved to Oakwood The father and mother work in AutoCorsquos plant rent an apartment from AutoCo and have enrolled their 10-year-old son in Oakwoodrsquos school Every morning the students are required to recite the Pledge of Allegiance while standing and saluting an American flag With the approval of his parents the son has politely but insistently refused to recite the Pledge and salute the flag at the school on the grounds that doing so violates his own political beliefs and the political beliefs of his family As a result of his refusal to say the Pledge the son has been expelled from the school

To protest the schoolrsquos actions the father walked into the commercial district of Oakwood While standing on a street corner he handed out leaflets that contained a short essay critical of the schoolrsquos Pledge of Allegiance policy Some of the passersby who took the leaflets dropped them to the ground An AutoCo security guard saw the litter told the father that Oakwoodrsquos anti-litter rule prohibits leaflet distribution that results in littering and directed him to cease distribution of the leaflets and leave the commercial district When the father did not leave and continued to distribute the leaflets the security guard called the state police which sent officers who arrested the father for trespass

1 Did the sonrsquos expulsion from the school violate the First Amendment as applied through the Fourteenth Amendment Explain

2 Did the fatherrsquos arrest violate the First Amendment as applied through the Fourteenth Amendment Explain

5

SECURED TRANSACTIONS QUESTION

On June 1 a bicycle retailer sold two bicycles to a man for a total purchase price of $1500 The man made a $200 down payment and agreed to pay the balance in one year The man also signed a security agreement that identified the bicycles as collateral for the unpaid purchase price and provided that the man ldquoshall not sell or dispose of the collateral until the balance owed is paid in fullrdquo The retailer never filed a financing statement reflecting this security interest

The man had bought the bicycles for him and his girlfriend to use on vacation However shortly after he bought the bicycles the man and his girlfriend broke up The man has never used the bicycles

On August 1 the man sold one of the bicycles at a garage sale to a buyer who paid the man $400 for the bicycle The buyer bought the bicycle to ride for weekend recreation

On October 1 the man gave the other bicycle to his friend as a birthday present The friend began using the bicycle for morning exercise

Neither the buyer nor the friend had any knowledge of the manrsquos dealings with the retailer

1 Does the buyer own the bicycle free of the retailerrsquos security interest Explain

2 Does the friend own the bicycle free of the retailerrsquos security interest Explain

6

FEDERAL CIVIL PROCEDURE QUESTION _____

Mother and Son who are both adults are citizens and residents of State A Mother owned an expensive luxury car valued in excess of $100000 Son borrowed Motherrsquos car to drive to a store in State A As Son approached a traffic light that had just turned yellow he carefully braked and brought the car to a complete stop Driver who was following immediately behind him failed to stop and rear-ended Motherrsquos car which was damaged beyond repair Son was seriously injured Driver is a citizen of State B

Son sued Driver in the United States District Court for the District of State A alleging that she was negligent in the operation of her vehicle Son sought damages in excess of $75000 for his personal injuries exclusive of costs and interest In her answer Driver alleged that Son was contributorily negligent in the operation of Motherrsquos car She further alleged that the brake lights on Motherrsquos car were burned out and that Motherrsquos negligent failure to properly maintain the car was a contributing cause of the accident

Following a trial on the merits in Sonrsquos case against Driver the jury answered the following special interrogatories

Do you find that Driver was negligent in the operation of her vehicle Yes

Do you find that Son was negligent in the operation of Motherrsquos car No

Do you find that Mother negligently failed to ensure that the brake lights on her car were in proper working order Yes

The judge then entered a judgment in favor of Son against Driver Driver did not appeal

Two months later Mother sued Driver in the United States District Court for the District of State A alleging that Driverrsquos negligence in the operation of her vehicle destroyed Motherrsquos luxury car Mother sought damages in excess of $75000 exclusive of costs and interest

State A follows the same preclusion principles that federal courts follow in federal-question cases

1 Is Motherrsquos claim against Driver barred by the judgment in Son v Driver Explain

2 Does the juryrsquos conclusion in Son v Driver that Mother had negligently failed to maintain the brake lights on her car preclude Mother from litigating that issue in her subsequent suit against Driver Explain

3 Does the juryrsquos conclusion in Son v Driver that Driver was negligent preclude Driver from litigating that issue in the Mother v Driver lawsuit Explain

7

_____

AGENCY QUESTION

Over 5000 individuals in the United States operate hot-air balloon businesses A hot-air balloon has four key components the balloon that holds the heated air the basket that houses the riders the propane burner that heats the air in the balloon and the propane storage tanks

The owner of a hot-air balloon business recently notified several basket and burner manufacturers that she or her agent might be contacting them to purchase baskets or burners The owner did not specifically name any person as her agent Basket and burner manufacturers regularly receive such notices from hot-air balloon operators Such notices typically include no restrictions on the types of baskets or burners agents might purchase for their principals

The owner then retained an agent to acquire baskets burners and fuel tanks from various manufacturers The owner authorized the agent to buy only (a) baskets made of woven wicker (not aluminum) (b) burners that use a unique ldquowhisper technologyrdquo (so as not to scare livestock when the balloon sails over farmland) and (c) propane fuel tanks

The agent then entered into three transactions with manufacturers all of whom had no prior dealings with either the owner or the agent

(1) The agent and a large manufacturer of both wicker and aluminum baskets signed a contract for the purchase of four aluminum baskets for a total cost of $60000 The agent never told the manufacturer that he represented the owner or any other principal The contract listed the agent as the buyer and listed the ownerrsquos address as the delivery address but did not indicate that the address was that of the owner rather than the agent When the baskets were delivered to the owner she learned for the first time that the agent had contracted to buy aluminum not wicker baskets The owner immediately rejected the baskets and returned them to the manufacturer Neither the owner nor the agent has paid the basket manufacturer for them

(2) The agent contacted a burner manufacturer and told him that the agent represented a well-known hot-air balloon operator who wanted to purchase burners The agent did not disclose the ownerrsquos name The agent and the burner manufacturer signed a contract for the purchase of four burners that did not have ldquowhisper technologyrdquo for a total price of $70000 The burner contract like the basket contract listed the ownerrsquos address for delivery but did not disclose whose address it was The burners were delivered to the ownerrsquos business and the owner discovered that the agent had ordered the wrong kind of burners The owner rejected the burners and returned them to the manufacturer Neither the owner nor the agent has paid the burner manufacturer for the burners

(3) The agent contracted with a solar cell manufacturer to make three cells advertised as ldquostrong enough to power all your ballooning needsrdquo The agent did not tell the manufacturer that he was acting on behalf of any other person One week after the cells were delivered to the agent he took them to the owner who installed them and discovered that she could save a lot of money using solar cells instead of propane to power her balloons The owner decided to keep the solar cells but she has not paid the manufacturer for them

8

Agency Question

Assume that the rejection of the baskets and the burners and the failure to pay for the solar cells constitute breach of the relevant contracts

1 Is the owner liable to the basket manufacturer for breach of the contract for the aluminum baskets Is the agent liable Explain

2 Is the owner liable to the burner manufacturer for breach of the contract for the burners Is the agent liable Explain

3 Is the owner liable to the solar cell manufacturer for breach of the contract for the solar cells Is the agent liable Explain (Do not address liability based upon restitution or unjust enrichment)

9

EVIDENCE QUESTION _____

A woman who owns a motorized scooter brought her scooter to a mechanic for routine maintenance service As part of the maintenance service the mechanic inspected the braking system on the scooter As soon as the mechanic finished inspecting and servicing the scooter he sent the woman a text message to her cell phone that read ldquoJust finished your service When you pick up your scooter you need to schedule a follow-up brake repair Wersquoll order the partsrdquo

The woman read the mechanicrsquos text message and returned the next day to pick up her scooter As the woman was wheeling her scooter out of the shop she saw the mechanic working nearby and asked ldquoIs my scooter safe to ride for a whilerdquo The mechanic responded by giving her a thumbs-up The woman waved and rode away on the scooter

One week later while the woman was riding her scooter a pedestrian stepped off the curb into a crosswalk and the woman collided with him causing the pedestrian severe injuries The woman had not had the scooterrsquos brakes repaired before the accident

The pedestrian has sued the woman for damages for his injuries resulting from the accident The pedestrian has alleged that (1) the woman lost control of the scooter due to its defective brakes (2) the woman knew that the brakes needed repair and (3) it was negligent for the woman to ride the scooter knowing that its brakes needed to be repaired

The woman claims that the brakes on the scooter worked perfectly and that the accident happened because the pedestrian stepped into the crosswalk without looking and the woman had no time to stop The woman the pedestrian and the mechanic will testify at the upcoming trial

The pedestrian has proffered an authenticated copy of the mechanicrsquos text message to the woman

The woman plans to testify that she asked the mechanic ldquoIs my scooter safe to ride for a whilerdquo and that he gave her a thumbs-up in response

The evidence rules in this jurisdiction are identical to the Federal Rules of Evidence

Analyze whether each of these items of evidence is relevant and admissible at trial

1 The authenticated copy of the mechanicrsquos text message

2 The womanrsquos testimony that she asked the mechanic ldquoIs my scooter safe to ride for a whilerdquo and

3 The womanrsquos testimony describing the mechanicrsquos thumbs-up

10

TRUSTS AND FUTURE INTERESTS QUESTION

Ten years ago Settlor validly created an inter vivos trust and named Bank as trustee The trust instrument provided that Settlor would receive all of the trust income during her lifetime The trust instrument further provided that

Upon Settlorrsquos death the trust income shall be paid in equal shares to Settlorrsquos surviving children for their lives Upon the death of the last surviving child the trust income shall be paid in equal shares to Settlorrsquos then-living grandchildren for their lives Upon the death of the survivor of Settlorrsquos children and grandchildren the trust corpus shall be distributed in equal shares to Settlorrsquos then-living great-grandchildren

The trust instrument expressly specified that the trust was revocable but it was silent regarding whether Settlor could amend the trust instrument

Immediately after creating the trust Settlor validly executed a will leaving her entire estate to Bank as trustee of her inter vivos trust to ldquohold in accordance with the terms of the trustrdquo

Five years ago Settlor signed an amendment to the inter vivos trust The amendment changed the disposition of the remainder interest specifying that all trust assets ldquoshall be paid upon Settlorrsquos death to Universityrdquo Settlorrsquos signature on this amendment was not witnessed

A state statute provides that any trust interest that violates the common law Rule Against Perpetuities ldquois nonetheless valid if the nonvested interest in the trust actually vests or fails to vest either (a) within 21 years of lives in being at the creation of the nonvested interest or (b) within 90 years of its creationrdquo

Recently Settlor died leaving a probate estate of $200000 She was survived by no children one granddaughter (who would be Settlorrsquos only heir) and no great-grandchildren The granddaughter has consulted your law firm and has raised four questions regarding this trust

1 Was Settlorrsquos amendment of the inter vivos trust valid Explain

2 Assuming that the trust amendment was valid do its provisions apply to Settlorrsquos probate assets Explain

3 Assuming that the trust amendment was valid how should trust assets be distributed Explain

4 Assuming that the trust amendment was invalid how should trust assets be distributed Explain

11

NEGOTIABLE INSTRUMENTS QUESTION

A chef entered into a contract with a repairman pursuant to which the repairman agreed to repair the chefrsquos commercial oven for $10000 The repairman agreed to accept as payment a negotiable promissory note for $10000 payable two months after its issuance

After the repairman worked on the oven the chef gave him a $10000 note as payment for the work As agreed the note was signed by the chef as maker was payable to the order of the repairman was payable in two months and fulfilled all criteria for negotiability

The next day the repairman sold the note to a buyer for $9500 To effectuate the sale the repairman wrote ldquono warrantiesrdquo on the back of the note signed his name immediately below that and handed the note to the buyer The buyer bought the note in good faith and without knowledge of any facts relating to the work that the repairman had performed for the chef

Later the buyer gave the note to his niece as a gift To effectuate the gift the buyer handed the note to the niece but did not indorse it

Shortly thereafter the chef discovered that the repair work had been done improperly and the oven still did not function correctly The chef tried repeatedly to get the repairman to return to correct the repair work but the repairman ignored all the chefrsquos calls

On the notersquos due date the niece contacted the chef and demanded that he pay the amount of the note to her The chef refused and told the niece that he would not pay the note because the repairman did not properly repair the oven

1 What are the niecersquos rights against the chef Explain

2 What are the niecersquos rights against the repairman Explain

3 What are the niecersquos rights against the buyer Explain

12

February 2013 MEE

ANALYSES

Real Property Contracts

Constitutional Law Secured Transactions

Federal Civil Procedure Agency

Evidence Trusts and Future Interests

Negotiable Instruments

REAL PROPERTY ANALYSIS (Real Property ID1a 4 amp 5)

ANALYSIS

Legal Problems

(1) Does the tenant have a defense to the landlordrsquos action for unpaid rent based on constructive eviction

(2) Does the tenant have a defense to the landlordrsquos action for unpaid rent based on the tenantrsquos surrender of the premises

(3) What if anything may the landlord recover from the tenant for the period after the tenant vacated the building

DISCUSSION

Summary

Under the common law the tenant does not have a defense to the landlordrsquos action for unpaid rent based on constructive eviction Constructive eviction is based on the tenant proving that (1) the landlord breached a duty to the tenant (2) the breach caused a loss by the tenant of the substantial use and enjoyment of the premises (3) the tenant gave the landlord adequate notice and opportunity to repair and (4) the tenant vacated the leased premises Here there was no constructive eviction because although the tenant vacated and gave the landlord adequate notice the landlord breached no express or implied duty to the tenant to repair the premises

The tenant does not have a defense based on the landlordrsquos acceptance of his surrender of the premises a landlordrsquos retention of keys does not constitute an acceptance of the tenantrsquos surrender unless the landlord so intended and here the landlordrsquos statements to the tenant at the time of the surrender of the keys do not evidence the intent to accept the tenantrsquos surrender

Under the common law a landlord has no duty to mitigate damages but also cannot sue for rents due in the future Under this approach the landlord can sue only for past-due rents Using this approach on November 1 the landlord could recover all the rent past due (ie rent for September and October) but could not recover for rents due in the future However some courts have authorized recovery for future rent minus the fair market rental value of the premises It is thus possible that the landlord could recover damages equal to the amount of rent due from September 1 to the end of the six-year lease term ($180000) minus the propertyrsquos fair-market rental value over that same period

Point One (45) The tenant was not constructively evicted because the landlord had no duty to repair the commercial premises that were the subject of the lease

The landlord and the tenant entered into a term-of-years lease because the lease specified both a beginning and an ending date HERBERT HOVENKAMP amp SHELDON F KURTZ THE LAW OF

PROPERTY 256 (5th ed 2001) Although a term-of-years lease normally cannot be terminated by the tenant prior to the end of the term a tenant may terminate a term-of-years lease if the tenant

15

Real Property Analysis

is constructively evicted See id at 286ndash88 Typically as here a claim of constructive eviction is made as a defense to a landlordrsquos action for damages or unpaid rent

In order to establish a constructive eviction the tenant must prove that the landlord breached a duty to the tenant such as a duty to repair and that the landlordrsquos breach caused a loss of the substantial use and enjoyment of the premises The tenant must also show that he gave the landlord notice adequate to permit the landlord to meet his duty to the tenant and that the tenant vacated the leased premises Id see also JOHN G SPRANKLING UNDERSTANDING

PROPERTY LAW sect 1704 (2d ed 2007) Under the common law there was no implied duty on the part of a landlord to repair

leased premises such a duty arose only if expressly set forth in the lease SPRANKLING supra sect 1702[B] Here the written lease contained no term requiring the landlord to repair the air-conditioning Even if the conversation created a lease term that the building had air-conditioning that itself should not create a duty for the landlord to repair it

Over the past several decades courts have generally implied a duty to repair in residential leases either as part of a revised constructive eviction doctrine or based on an implied warranty of habitability JOSEPH W SINGER PROPERTY 469ndash70 (3d ed 2010) This shift has been justified based on the economic disparity between the typical landlord and tenant as well as the fact that residential tenants generally lack both the authority to authorize repairs to common areas of a building and the incentive to make repairs that will ultimately benefit the landlord

However courts have been more reluctant to imply a duty to repair in commercial leases a context in which the tenant is often a valuable business and in a better position to assess and make repairs than is the landlord But see eg Davidow v Inwood North Professional Group 747 SW2d 373 (Tex 1988) When courts have implied a duty to repair in a commercial lease it is typically when the repair has been mandated by public authorities and involves work so substantial that it would not ordinarily fall within the tenantrsquos common law repair duty andor the value of the repair would primarily inure to the landlordrsquos reversionary interest See Brown v Green 884 P2d 55 (Cal 1994) Eugene L Grant et al The Tenant as Terminator Constructive Eviction in Modern Commercial Leases 2 THE COMMERCIAL PROPERTY LEASE ch 15 (ABA 1997) Some courts have also permitted constructive eviction claims by commercial tenants of office buildings based on repairs required in common areas of the building See id Echo Consulting Services Inc v North Conway Bank 669 A2d 227 (NH 1995)

Here the tenant is the owner of a valuable manufacturing operation and is the exclusive occupant of the building the repair has not been mandated by public authorities and the repair is not structural To the contrary the repair involves a feature of the building of unusual importance in the tenantrsquos manufacturing operation and the tenant is likely far more knowledgeable than the landlord about the air-conditioning specifications necessary for the manufacture of the tenantrsquos product

Based on these facts it is unlikely that a court will find that the tenant in this case was constructively evicted Although the tenant can show that he gave adequate notice to the landlord of the air-conditioning malfunction and vacated the premises the lease was commercial and it did not contain any promises or covenants by the landlord except a covenant of quiet enjoyment a covenant of quiet enjoyment does not entail any repair obligations

[NOTE An examineersquos conclusion is less important than his or her demonstrated awareness of the elements of constructive eviction and the need to imply a repair duty for such a defense to be viable here Although the implied warranty of habitability is not available to this tenant Texas Minnesota and Massachusetts imply a warranty of suitability in commercial leases in limited circumstances and an examinee might argue that this warranty should apply

16

Real Property Analysis

here If an examinee concludes that this warranty applies he or she should discuss the other requirements for constructive eviction

If the examinee wrongly concludes that the first element for a constructive eviction has been met the examinee will then have to discuss the remaining three elements in order to conclude that the tenant can claim constructive eviction The tenant would have a strong argument that the second elementmdashsubstantial interference with the use and enjoyment of the premisesmdashalso is met As indicated above the landlord was aware that a functioning air-conditioning system was vital to the tenantrsquos manufacturing operations The facts further indicate that the system had failed three times in the past few months The landlord may try to argue that the malfunctions did not substantially interfere with the tenantrsquos use of the premises because the malfunctions caused the temperature to climb above 81 degrees for only a short period of timemdash 3 hours 6 hours and 10 hours respectivelymdashon each occasion The tenant will argue however that the landlord was aware that the tenantrsquos manufacturing operations could tolerate temperatures above 81 degrees for no more than 6 hours The final malfunction exceeded that limit destroying $150000 worth of the tenantrsquos products

The tenant would also have a strong argument that the third element is met notice and opportunity to cure The tenant notified the landlord of the problem immediately upon the systemrsquos first malfunction and did so again when it malfunctioned a second time and then a third time The landlord might argue that there was insufficient time to cure the problem because the system corrected itself within a few hours on the first and second times Although the malfunction lasted more than 10 hours the third time the landlord might argue that the time period was insufficient to get a repair person on the premises A court would be likely to find this argument unpersuasive however because the landlord could have attempted to correct the problem after the first and second malfunctions

Assuming that the landlord was given sufficient notice and opportunity to cure a court would be likely to conclude that the tenant also satisfied the final element of vacating the premises within a reasonable time The landlord might argue that the tenant remained in the premises for almost four months after the air conditioning first failed which would suggest that the problem was not so severe as to have constructively evicted the tenant The tenant will argue however that he gave the landlord three months to cure the problem after the first two malfunctions threatened (but did not actually harm) his operations The tenant then moved out shortly after the final malfunction caused temperatures to exceed the tolerance levels of his manufacturing operations]

Point Two (10) The landlord did not accept the tenantrsquos surrender of the lease

When a tenant wrongfully moves from leased premises with the intent to terminate the lease the landlord may either accept the tenantrsquos surrender of the premises and terminate the lease or hold the tenant to the terms of the lease See HOVENKAMP amp KURTZ supra at 295ndash96 Here the tenantrsquos only basis for the claim that the landlord accepted his surrender is the landlordrsquos retention of the keys Many courts have considered whether a landlordrsquos retention of keys delivered by a tenant constitutes acceptance of surrender The weight of the case law holds that retention of the keys alone does not constitute acceptance of surrender without other evidence showing that the landlord intended to accept the surrender See generally 49 AM JUR 2d Landlord and Tenant sect 213

Here the landlordrsquos note saying ldquoI repeat the air-conditioning is not my problem You have leased the building and you should fix itrdquo strongly suggests that the landlord did not intend

17

Some courts have rejected the no-mitigation-of-damages rule based on efficiency concerns and societyrsquos interest in assuring that resources remain in the stream of commerce rather than lying vacant see id at 464ndash65 and allow landlords to sue tenants who have wrongfully terminated a lease for damages equal to the difference between the unpaid rent due under the lease and the propertyrsquos fair market rental value Other courts have abandoned the no-recovery-for-future-rent rule These courts responding to the fact that a tenant may well disappear or be judgment-proof by the time a lease term is concluded have allowed a landlord to collect damages equal to the value of rent over the entire lease term minus the propertyrsquos fair rental value when a tenant has wrongfully terminated a lease and unequivocally shown an intention not to return to the premises or pay future rent Under this approach a landlord receives approximately the same amount he would have received were there a duty to mitigate damages See Sagamore Corp v Willcutt 180 A 464 (Conn 1935)

Real Property Analysis

to accept the tenantrsquos surrender The tenant might argue that the landlordrsquos failure to make a similar statement when the keys were sent to her a second time and she retained them evidences a change of heart However it is likely that a court would find that the landlordrsquos retention of the keys represented a decision to safeguard the keys not to accept the tenantrsquos surrender

[NOTE An examinee should receive credit for arguing the other way with a well-reasoned argument]

Point Three (45) Under the common law the landlord had no duty to mitigate damages Additionally a landlord was not entitled to recover unpaid rents due in the future but was only entitled to recover rents in arrears at the time of the commencement of the suit Applying the common law here the landlord could recover $5000 the amount of rents due at the commencement of the suit ($2500 for September and the same for October) Today some courts allow the landlord under certain circumstances to sue the tenant for damages (not rent) equal to the difference if any between the unpaid promised rent for the balance of the term (here $175000) and the propertyrsquos fair rental value for the balance of the term

Under the common law because a lease was viewed as a conveyance instead of a contract a landlord had no duty to mitigate damages resulting from a tenantrsquos wrongful termination of a lease A landlord could thus recover the full value of rents that were due and unpaid at the time of the suit However under the common law a landlord could not sue a tenant for rents due in the future because there was always a possibility that the tenant might pay the rent when it was due See SINGER supra at 462 Thus using the common law approach on November 1 the landlord could only recover the full value of the two monthsrsquo rent actually due and unpaid ie $5000 for September and October

Here because the tenant returned the keys to the landlord and said ldquoI will not be returning to the building or making further rent paymentsrdquo the landlord could establish abandonment and an intention not to return It is thus possible that the landlord might recover damages in the amount of $5000 (for the months of September and October) plus the present value of $175000 minus the fair market rental value of the property over the remaining months of the lease

18

CONTRACTS ANALYSIS ____ (Contracts II IVE)

ANALYSIS

Legal Problems

(1) What was the legal effect of the sailorrsquos October 31 letter to the builder

(2)(a) What was the legal effect of the builderrsquos November 25 response to the sailorrsquos October 31 letter

(2)(b) What was the legal effect of the sailorrsquos refusal to take and pay for the boat on December 15

DISCUSSION

Summary

This is a sale of goods governed by the Uniform Commercial Code Because the sailor had reasonable grounds for insecurity about the builderrsquos ability to deliver the boat in a timely manner when the sailor learned about the strike on October 31 the sailor was legally justified in sending the letter to the builder seeking adequate assurance of the builderrsquos performance pursuant to the contract The builderrsquos failure to provide such assurance within a reasonable time operated as a repudiation of the contract However the builder was free to retract the repudiation before the sailor either cancelled the contract or materially changed position in reliance on the builderrsquos repudiation The builder retracted the repudiation when he informed the sailor that the workers were back and that the boat would be delivered by the date stipulated in the partiesrsquo contract Because the sailor had taken no action in response to the original repudiation he no longer had the right to cancel the contract with the builder The sailorrsquos subsequent statement that ldquoour contract is overrdquo may have constituted repudiation by the sailor In any event when the sailor failed to perform on December 15 that constituted breach

Point One (35) Because the sailor had reasonable grounds for insecurity with respect to the builderrsquos performance the sailorrsquos letter to the builder was a justified demand seeking assurance of the builderrsquos performance under the contract failure of the builder to provide such assurance within a reasonable time constituted repudiation of the contract

The sailor was legally justified in sending the letter to the builder on October 31 Contract parties are entitled to expect due performance of contractual obligations and are permitted to take steps to protect that expectation UCC sect 2-609 states that ldquo[w]hen reasonable grounds for insecurity arise with respect to the performance of either party the other may in writing demand adequate assurance of due performance rdquo Here the sailor learned on October 31 that the builderrsquos workers were on strike This gave the sailor reasonable grounds for insecurity about the builderrsquos ability to complete performance on time and thus gave the sailor the right to seek adequate assurance from the builder Because the sailorrsquos demand for assurance was justified the builder was required to provide assurance that was adequate under the circumstances within a reasonable time (not to exceed 30 days) or be held to have repudiated the contract UCC sect 2-609(4)

19

Contracts Analysis

Point Two(a) (30) The builder did not within a reasonable time provide the sailor adequate assurance of due performance this failure to provide assurance constituted a repudiation of the contract

Because the sailor with legal justification (see Point One) demanded from the builder assurance of due performance the builderrsquos failure to provide such assurance within a reasonable time was a repudiation of their contract See UCC sect 2-609(4) (ldquoAfter receipt of a justified demand[] failure to provide within a reasonable time not exceeding thirty days assurance of due performance is a repudiation of the contractrdquo) On October 31 the sailor requested that the builder provide adequate assurance regarding the completion of the boat by December 15 The builder did not respond to the sailorrsquos letter until November 25mdashnearly a month later Even if that response had been given in a reasonable time it nonetheless did not provide assurance of due performance It simply stated ldquoIrsquom sorry about the strike but it is really out of my hands I hope we settle it soon so that we can get back to workrdquo Therefore the builderrsquos November 25 response did not provide adequate assurance in response to the sailorrsquos justified request Thus the builder had repudiated the contract

Point Two(b) (35) Although the builder repudiated the contract with the sailor the builder probably retracted that repudiation on December 3 and the sailor was no longer entitled to cancel their contract Thus the sailorrsquos failure to perform the sailorrsquos obligations under the contract constituted a breach

The builderrsquos failure to provide adequate assurance of performance constituted a repudiation of their contract (see UCC sect 2-609(4)) but the builder was free to retract that repudiation until the sailor cancelled the contract or materially changed his position or indicated by communication or action that the sailor considered the repudiation to be final See UCC sect 2-611(1) (ldquoUntil the repudiating partyrsquos next performance is due he can retract his repudiation unless the aggrieved party has since the repudiation cancelled or materially changed his position or otherwise indicated that he considers the repudiation finalrdquo)

Here the facts state that before the builderrsquos December 3 telephone call to the sailor the sailor did nothing in response to the builderrsquos repudiation such as contracting with a third party for a boat The builderrsquos December 3 call informing the sailor that the boat would be timely delivered probably constituted a retraction of the repudiation because it clearly indicated to the sailor that the builder would be able to perform UCC sect 2-611(2) Thus after being so informed the sailor did not have the right to treat their contract as cancelled UCC sect 2-611(3) Accordingly the sailorrsquos failure to perform the sailorrsquos obligations under the contract by taking the boat and paying for it constituted a breach of the contract

20

CONSTITUTIONAL LAW ANALYSIS (Constitutional Law IVA F2b amp e)

ANALYSIS

Legal Problems

(1) Does AutoCorsquos operation of a ldquocompany townrdquo result in its actions counting as those of the state for purposes of constitutional analysis

(2) Does the expulsion of a schoolchild for failure to recite the Pledge of Allegiance violate the First Amendment as applied through the Fourteenth Amendment

(3) Does the arrest of a pamphleteer in connection with violation of an anti-littering rule where the littering is done by the recipients of leaflets distributed by the pamphleteer violate the First Amendment as applied through the Fourteenth Amendment

DISCUSSION

Summary

The First Amendment as applied through the Fourteenth Amendment applies only to state action It does not typically govern private actors However courts have found state action where the private actor has exercised a ldquopublic functionrdquo such as running a privately owned ldquocompany townrdquo as AutoCo has done here Thus First Amendment protections apply By requiring the son to participate in a mandatory Pledge of Allegiance ceremony AutoCo has compelled the expression of political belief in violation of the First Amendment as applied through the Fourteenth Amendment The fatherrsquos arrest in connection with breaching the anti-litter rule also violated the First Amendment as applied through the Fourteenth Amendment Although state actors can regulate the incidental effects of speech on the public streets on a content-neutral basis this power is limited and cannot extend to punishing a distributor of literature because of littering by third parties

Point One (30) AutoCorsquos operation of a company town (including a school) makes it a state actor under the public function strand of the state action doctrine

The individual rights protections of the Constitution apply only where there is ldquostate actionrdquomdash either direct action by the government or some action by a private party that is fairly attributable to the government As a general rule the actions of a private company like AutoCo or of a private school like the school operated by AutoCo would not constitute state action and the protections of the Constitution (in this case the First Amendment) would not apply

However there are situations in which the actions of a private actor are attributed to the state One such situation is when the private actor undertakes a public function There are not many bright-line rules in the Supreme Courtrsquos state action doctrine but one of them is this Where a private actor undertakes a ldquopublic functionrdquo the Constitution applies to those actions Where a corporation operates a privately owned ldquocompany townrdquo that provides essential services typically provided by a state actor the public function doctrine applies and the Constitution

21

Constitutional Law Analysis

binds agents of the town as if they were agents of the government See eg Marsh v Alabama 326 US 501 (1946) Here AutoCo does more than own the town it provides security services fire protection sanitation services and a school Thus the actions of AutoCo constitute state action and are governed by the Fourteenth Amendment

Point Two (35) The sonrsquos expulsion for failure to recite the Pledge of Allegiance violates the First Amendment as applied through the Fourteenth Amendment as a compelled expression of political belief

As explained in Point One the First Amendment applies to the school as a state actor Although children in public schools (and in schools subject to the First Amendment like

the Oakwood school) have some First Amendment rights Tinker v Des Moines Independent Community School District 393 US 503 506 (1969) schools have greater leeway to regulate the speech of students and teachers than the state would have outside the school context Hazelwood School Dist v Kuhlmeier 484 US 260 (1988) Morse v Frederick 551 US 393 (2007) However the Supreme Court has long held that public schools may not force their students to participate in a flag salute ceremony when it offends the political or religious beliefs of the students or their families West Virginia Board of Educ v Barnette 319 US 624 (1943) (invalidating a mandatory public school flag salute ceremony) see also Wooley v Maynard 430 US 705 (1977) (invalidating compelled expression of political belief on state-issued license plates)

In this case the school requires its students to participate in a flag salute and Pledge of Allegiance ceremony and punishes them when they refuse to participate Pursuant to this policy the school has expelled the son This expulsion violates the First Amendment ban on compelled expression

Point Three (35) Because the father was distributing leaflets in a traditional public forum his trespass arrest violated the First Amendment as applied through the Fourteenth Amendment

As explained in Point One AutoCo is treated as a state actor Thus Oakwoodrsquos commercial district is treated as government-owned property for purposes of the First Amendment Thus the leafleting here is subject to the First Amendment because it is an expressive activity Schneider v State of New Jersey Town of Irvington 308 US 147 (1939) When expression takes place on government-owned property government regulation of the expression is assessed under the public forum doctrine Public streets and sidewalks have long been held to be the classic example of a ldquotraditional public forumrdquo open to the public for expression Hague v CIO 307 US 496 515ndash16 (1939) Because the father was distributing leaflets while standing on a street corner in the commercial district his expressive activity occurred in a traditional public forum

When a state tries to regulate expressive activity in a traditional public forum it is prohibited from doing so based on the expressive activityrsquos content unless its regulation is narrowly tailored to achieve a compelling governmental interest (ldquostrict scrutinyrdquo) In this case however AutoCo is regulating the fatherrsquos expressive activity on the ostensibly neutral ground that his expressive activity has produced litter and made the street unsightly When a state tries to regulate expressive activity without regard to its content intermediate scrutiny applies Under intermediate scrutiny the true purpose of the regulation may not be the suppression of ideas (if so then strict scrutiny applies) the regulation must be narrowly tailored to achieve a significant

22

Constitutional Law Analysis

governmental interest and it must leave open ample alternative channels for expressive activity Ward v Rock Against Racism 491 US 781 791 (1989)

Here the application of the ordinance to the father will fail for two reasons First the Supreme Court has held that the governmentrsquos interest in keeping the streets clean is insufficient to ban leafleting in the public streets as the government power to regulate with incidental effects on public sidewalk speech is very limited See eg Schneider 308 US at 162 (leafletinglittering) Second the regulation (a blanket ban on distribution that results in littering) is not narrowly tailored to protect expression A narrowly tailored alternative would be prosecution only of people who litter Moreover the effect of the littering rule is likely to be a ban on all leafleting thus eliminating an entire class of means of expression This raises the possibility that there are not ldquoample alternative channels of communicationrdquo open to the father as required under the Courtrsquos standard of review for content-neutral regulation of speech

[NOTE Some examinees might argue that this is a ldquotime place and mannerrdquo restriction and that AutoCo might have greater latitude to regulate the public sidewalks under this theory This argument is incorrect for two reasons First the Supreme Court has held that the power to regulate speakers through littering laws is very limited for the reasons given and in the cases cited above But more generally a ldquotime place and mannerrdquo restriction involves the shifting of speech from one time and place to another or to another manner here there is no shifting but a direct punishment for expressive activity (albeit one couched in content-neutral terms) In addition some examinees might read the ordinance to be in effect a total ban on leafleting since most leafleting will produce some litter Those examinees might note that the Court has required total bans on an entire mode of expression to satisfy strict scrutiny and analyze the fatherrsquos prosecution here accordingly See United States v Grace 461 US 171 177 (1983) (invalidating ban on display of signs on public sidewalks surrounding US Supreme Court ldquo[a]dditional restrictions such as an absolute prohibition on a particular type of expression will be upheld only if narrowly drawn to accomplish a compelling governmental interestrdquo)]

23

SECURED TRANSACTIONS ANALYSIS (Secured Transactions IID E IVA B C)

ANALYSIS

Legal Problems

(1) Is a purchase-money security interest in consumer goods perfected even though there has been no filing of a financing statement

(2) Does a person who buys consumer goods for personal use take those goods free of a prior perfected purchase-money security interest in the goods

(3) Does a person who receives consumer goods as a gift take those goods subject to a prior perfected security interest in them

DISCUSSION

Summary

The retailerrsquos security interest in the bicycles was perfected even though no financing statement was filed because it was a purchase-money security interest in consumer goods A purchase-money security interest in consumer goods is automatically perfected upon attachment

The buyer is not subject to the retailerrsquos security interest in the bicycle that the buyer bought from the man Because the bicycle was consumer goods in the hands of the man and the retailer never filed a financing statement covering the bicycle the retailerrsquos security interest is not effective against someone like the buyer who bought the bicycle for value without knowledge of the retailerrsquos security interest and for personal use

On the other hand the retailerrsquos security interest continues in the bicycle given to the friend because the friend did not give value for the bicycle or buy it in the ordinary course of business

Point One (35) The retailerrsquos security interest in the bicycles attached on June 1 Because this interest was a purchase-money security interest in consumer goods it was automatically perfected when it attached

The retailerrsquos security interest in the bicycles attached on June 1 when the man bought the bicycles (acquiring rights in the collateral) signed a security agreement containing a description of the collateral and received value from the retailer (by being given credit with which to purchase the bicycles) UCC sect 9-203(a) amp (b)

Despite the retailerrsquos failure to file a financing statement its security interest was perfected Pursuant to UCC sect 9-309(1) a security interest is automatically perfected upon attachment if the goods are ldquoconsumer goodsrdquo and the security interest is a ldquopurchase-money security interestrdquo

In this case the bicycles sold by the retailer to the man were consumer goods at the time of sale The bicycles were ldquogoodsrdquo because they were ldquomovable when a security interest

24

Secured Transactions Analysis

attachesrdquo UCC sect 9-102(a)(44) They were also consumer goods because they were ldquobought for use primarily for personal family or household purposesrdquo UCC sect 9-102(a)(23) The retailerrsquos security interest in these consumer goods was also a ldquopurchase-money security interestrdquo A purchase-money security interest is an interest that secures a debt that was incurred in order to ldquoenable the debtor to acquire rights in or the use of the collateralrdquo UCC sect 9-103(a) (b)(1) Here the man incurred an obligation to the retailer to purchase the bicycles so the security interest he gave the retailer to secure that obligation was a purchase-money security interest

Because the retailerrsquos security interest was a purchase-money security interest in consumer goods it was automatically perfected on June 1 when the interest attached to the bicycles

Point Two (35) The buyer took the bicycle free of the retailerrsquos security interest because (i) the retailer did not file a financing statement covering the bicycle (ii) the bicycle was ldquoconsumer goodsrdquo and (iii) the buyer bought the bicycle for value without knowledge of the retailerrsquos security interest and for personal use

A security interest continues in collateral even after a sale or other disposition of that collateral unless the creditor authorized the disposition ldquofree of the security interestrdquo or another Article 9 exception applies UCC sectsect 9-201(a) and 9-315(a)(1)

However a buyer of goods like the buyer here can take free of a prior security interest in those goods under certain circumstances See UCC sectsect 9-317(b) (buyers who give value and receive delivery of goods without knowledge of an unperfected security interest in the goods) and 9-320(a) amp (b) (buyer in ordinary course of business buyer of consumer goods in a consumer-to-consumer transaction who gives value) In this case the retailerrsquos security interest was perfected when the buyer purchased the bicycle so UCC sect 9-317(b) does not protect the buyer The buyer also is not a protected ldquobuyer in ordinary course of businessrdquo because he did not purchase from a person who is in the business of selling bicycles See UCC sect 1-201(b)(9)

The buyer can however qualify for the protection of UCC sect 9-320(b) That section provides that a buyer of goods from a person who used them for personal family or household purposes takes free of a perfected security interest in the goods if (1) the buyer had no knowledge of the security interest (2) the buyer gave value for the goods (3) the buyer purchased the goods primarily for personal family or household purposes and (4) the purchase occurred before the filing of a financing statement covering the goods

The buyer met all of these criteria The man used the bicycle for personal purposes The buyer purchased the bicycle from the man and the buyer had no knowledge of the retailerrsquos security interest The buyer gave value ($400) for the bicycle and he bought it ldquoprimarily for personal family or household purposesrdquo as he planned to use it for recreation which is a personal rather than a business use Finally no financing statement had been filed Therefore under UCC sect 9-320(b) the buyer took free of the retailerrsquos security interest

Point Three (30) The retailerrsquos security interest continues in the bicycle that the man gave to the friend Thus the retailer can recover the bicycle from the friend because the friend did not give value for the bicycle or buy it in the ordinary course of business

25

Secured Transactions Analysis

As noted in Point Two the retailer did not authorize the man to dispose of the bicycle Consequently the retailerrsquos security interest continued in the bicycle even after the man transferred ownership of the bicycle to the friend See UCC sectsect 9-201(a) and 9-315(a)(1) The retailerrsquos security interest in the bicycle will be effective against the friend unless some other provision of Article 9 allows the friend to take the bicycle free of that security interest

Unfortunately for the friend there is no Article 9 provision that allows him to take free of the retailerrsquos interest The friendrsquos basic problem is that he is not a buyer of the bicyclemdashhe received the bicycle as a gift and did not give value for it Thus the friend is not protected by any of the applicable exceptions See UCC sectsect 9-317(b) (protecting buyers who give value for goods subject to an unperfected security interest) 9-320(a) (protecting buyers in ordinary course of business) and 9-320(b) (protecting buyers of consumer goods who give value)

In short the retailerrsquos security interest continues in the bicycle that the man gave to the friend The friend took the bicycle subject to that security interest

26

FEDERAL CIVIL PROCEDURE ANALYSIS (Federal Civil Procedure VIE)

ANALYSIS

Legal Problems

(1) Does a judgment in a prior action preclude a nonparty from suing the same defendant on a closely related claim when the nonparty and the original plaintiff are in a family relationship

(2) Does a judgment rendered in an earlier action preclude a nonparty from litigating an issue that was actually decided in the first suit

(3) May a nonparty to an earlier action invoke the judgment in that action to preclude a party to the prior action from relitigating an issue that the party had a full and fair opportunity to litigate in the earlier action

DISCUSSION

Summary

Pursuant to the doctrines of claim preclusion (res judicata) and issue preclusion (collateral estoppel) a judgment is binding on the parties thereto In the absence of privity nonparties to a prior suit cannot be bound by a judgment rendered in their absence Thus in the absence of privity a nonparty to the first suit is not precluded from presenting her claim in a second suit even if it is factually related to the claims and defenses presented in the first suit nor is she bound by determinations of issues made in the first suit A family relationship without more does not support a finding of privity For this reason Mother as a nonparty is not bound by the judgment in the Son-Driver action She may bring her separate claim for damage to her car and she is not precluded from litigating the question of whether she was negligent in the maintenance of her car

Driver on the other hand could be precluded from relitigating the issue of her negligence pursuant to the doctrine of non-mutual issue preclusion (also called non-mutual offensive collateral estoppel) which allows a nonparty to a prior action to invoke issue preclusion to prevent a party to that prior action from relitigating determinations of issues made therein However Mother may be prevented from invoking non-mutual collateral estoppel in this case because she could easily have joined her claim in the prior action but did not do so

[NOTE Federal common law governs the preclusive effect of a judgment rendered by a federal court sitting in diversity See Semtek Intrsquol Inc v Lockheed Martin Corp 531 US 497 508 (2001) But the Semtek Court concluded that federal common law in this context incorporates the preclusion law of the state in which the rendering federal court sits (unless the state law is incompatible with federal interests) id at 508ndash09 Thus State Arsquos preclusion law determines the preclusive effect of the judgment rendered in Sonrsquos suit against Driver The problem says that State A preclusion law is identical to federal preclusion law so the following analysis utilizes general principles of preclusion drawn from Supreme Court case law (announcing federal preclusion rules) and the Restatement (Second) of Judgments]

27

Federal Civil Procedure Analysis

Point One (35) Under the doctrine of claim preclusion the judgment rendered in the first action does not preclude Mother a nonparty from suing Driver for the damage to her car because the judgment binds only parties or those in privity with them and Mother and Son are not in privity

Driver may contend that the doctrine of claim preclusion (res judicata) precludes Mother from presenting a claim arising from the same nucleus of facts that was presented in the first action brought by Son According to the doctrine of claim preclusion ldquowhen a court of competent jurisdiction has entered a final judgment on the merits of a cause of action the parties to the suit and their privies are thereafter bound lsquonot only as to every matter which was offered and received to sustain or defeat the claim or demand but as to any other admissible matter which might have been offered for that purposersquordquo Commissioner of Internal Revenue v Sunnen 333 US 591 597 (1948) (citation omitted)

However the doctrine of claim preclusion does not apply to Mother on the facts of this problem First Mother was not a party to the earlier case ldquoIt is a principle of general application in Anglo-American jurisprudence that one is not bound by a judgment in personam in a litigation in which he is not designated as a party or to which he has not been made a party by service of processrdquo Taylor v Sturgell 553 US 880 884 (2008) (citing Hansberry v Lee 311 US 32 40 (1940)) see also RESTATEMENT (SECOND) OF JUDGMENTS sect 34(3) (1982) This rule reflects our ldquodeep-rooted historic tradition that everyone should have his own day in courtrdquo Martin v Wilks 490 US 755 762 (1989) (citation omitted) (superseded by statute on other grounds) Since Mother was not a party to the first suit she is not bound by the judgment unless an exception to the general rule applies

Mother might be bound by the prior judgment if she were considered to have been sufficiently in privity with Son that Son represented her interests in that action ldquoA person who is not a party to an action but who is represented by a party is bound by and entitled to the benefits of a judgment as though he were a partyrdquo RESTATEMENT (SECOND) OF JUDGMENTS sect 41(1) But there is no suggestion in the facts of the problem that Son who is an adult purported to represent Motherrsquos interests in the first suit ldquo[C]lose family relationships are not sufficient by themselves to establish privity with the original suitrsquos party or to bind a nonparty to that suit by the judgment entered therein rdquo Cuauhtli v Chase Home Finance LLC 308 Fed Appx 772 773 (5th Cir 2009) (citation omitted) accord 18A CHARLES ALAN WRIGHT ET AL FEDERAL

PRACTICE AND PROCEDURE sect 4459 (2d ed 2002) In Taylor v Sturgell supra the Supreme Court identified other special circumstances in

which nonparties may be bound by a prior judgmentmdashwhen a nonparty consents to be bound when a nonparty is in a pre-existing substantive legal relationship with a party (such as preceding and succeeding property owners) when a nonparty assumed control of the prior litigation when a party seeks to relitigate through a proxy or where a special statutory scheme seeks to foreclose successive litigation by nonparties See Taylor 553 US at 893ndash95 None of these circumstances exists here

Because Mother was not a party to the first suit and is not in privity with Son who is an adult the judgment in the first action does not preclude her from bringing her own claim against Driver

Point Two (35) Under the doctrine of issue preclusion the judgment rendered in the first action does not preclude Mother a nonparty from litigating the issue of her negligence in maintaining her carrsquos

28

Federal Civil Procedure Analysis

brake lights because the judgment binds only parties or those in privity with them and Mother and Son are not in privity

By its affirmative response to a special interrogatory the jury in the first action expressly concluded that ldquoMother negligently failed to ensure that the brake lights on her car were in proper working orderrdquo Driver may attempt to invoke the doctrine of issue preclusion to preclude Mother from relitigating this issue in the second action

[I]ssue preclusion arises in a second action on the basis of a prior decision when the same lsquoissuersquo is involved in both actions the issue was lsquoactually litigatedrsquo in the first action after a full and fair opportunity for litigation the issue was lsquoactually decidedrsquo in the first action by a disposition that is sufficiently lsquofinalrsquo lsquoon the meritsrsquo and lsquovalidrsquo it was necessary to decide the issue in disposing of the first action and the later litigation is between the same parties or involves nonparties that are subject to the binding effect or benefit of the first action Once these requirements are met issue preclusion is available not only to defend against a demand for relief but also as offensive support for a demand for relief Issue preclusion moreover is available whether or not the second action involves a new claim or cause of action

18 CHARLES ALAN WRIGHT ET AL FEDERAL PRACTICE AND PROCEDURE sect 4416 at 392ndash93 (2d ed) see also RESTATEMENT (SECOND) OF JUDGMENTS sect 27 (1982)

Here several of the elements necessary for issue preclusion are present The same issue is involved in both actionsmdashthe issue of Motherrsquos negligence in failing to maintain the brake lights on her car That issue was actually litigated in the first action and decided by the jury There is nothing to suggest anything less than a full and fair opportunity to litigate The judgment disposing of the issue was final

Nevertheless the judgment will not preclude Mother from relitigating the issue for two reasons First Mother was not a party to the first action and as explained above Mother and Son are not in privity Therefore she cannot be denied an opportunity to litigate the issue of her negligence Second it does not appear that the juryrsquos decision as to Motherrsquos negligence was necessary to the prior judgment against Driver Nothing suggests that the finding on Motherrsquos negligence had any bearing on the outcome of the first action

Point Three (30) Under the doctrine of non-mutual issue preclusion the judgment rendered in the first action might preclude Driver from relitigating the issue of her negligence However Driver has a strong argument that such a result would be inconsistent with the policy against offensive use of non-mutual estoppel when the non-party plaintiff easily could have joined as a plaintiff in the first action

Because Son already convinced the jury in the first action that ldquoDriver was negligent in the operation of her vehiclerdquo Mother may wish to invoke the doctrine of non-mutual issue preclusion to prevent Driver from relitigating the question of her negligence As noted above ldquoissue preclusion arises in a second action on the basis of a prior decision when the same lsquoissuersquo is involved in both actions the issue was lsquoactually litigatedrsquo in the first action after a full and fair opportunity for litigation the issue was lsquoactually decidedrsquo in the first action by a disposition that is sufficiently lsquofinalrsquo lsquoon the meritsrsquo and lsquovalidrsquo it was necessary to decide the issue in disposing of the first action rdquo 18 CHARLES ALAN WRIGHT ET AL FEDERAL PRACTICE AND

PROCEDURE sect 4416 at 392 (2d ed) see also RESTATEMENT (SECOND) OF JUDGMENTS sect 27

29

Federal Civil Procedure Analysis

Here these basic requirements for issue preclusion are met First the same issue is involved in both suits whether Driver was negligent in the operation of her car Second this issue was actually litigated and decided in the first action the jury answered a special interrogatory raising this very question There is nothing to suggest that Driver lacked a full and fair opportunity to litigate the issue Since a judgment was rendered against Driver for the injuries Son sustained as a result of Driverrsquos negligence resolution of the issue was necessary to dispose of the first action Driver was a party to the first action so she may be bound by the judgment

[NOTE Traditionally issue preclusion required mutualitymdashboth the party asserting issue preclusion and the party against whom issue preclusion was asserted were bound by the prior judgment Under the traditional mutuality rule Mother could not assert issue preclusion against Driver because Mother would not be bound by the judgment if Driver sought to rely on it See Point One There is no mutuality between Mother and Driver with respect to the prior judgment

This traditional mutuality requirement has been abandoned in most jurisdictions The Supreme Court rejected a strict mutuality requirement in Blonder-Tongue Laboratories Inc v University of Illinois Foundation 402 US 313 (1971) (non-mutual defensive collateral estoppel used by a defendant to preclude a plaintiff from relitigating a claim the plaintiff previously litigated) and Parklane Hosiery Co v Shore 439 US 322 (1979) (non-mutual offensive collateral estoppel used by a plaintiff to preclude a defendant from relitigating a claim the defendant previously litigated) In Parklane Hosiery the Court concluded (as a matter of federal preclusion law) that trial courts should have ldquobroad discretionrdquo to determine whether or not to permit a plaintiff to invoke non-mutual issue preclusion ldquoThe general rule should be that in cases where a plaintiff could easily have joined in the earlier action or where the application of offensive estoppel would be unfair to a defendant a trial judge should not allow the use of offensive collateral estoppelrdquo Id at 331

The Parklane Hosiery decision identified a number of circumstances that might make it unfair to allow a plaintiff to invoke non-mutual issue preclusion (non-mutual offensive collateral estoppel in the traditional terminology) against a defendant In particular the Parklane Hosiery court suggested that issue preclusion may not be appropriate if the plaintiff in the second action ldquocould easily have joined in the earlier actionrdquo Id Prohibiting plaintiffs from using non-mutual estoppel under such circumstances would promote judicial efficiency by encouraging plaintiffs to join the prior action It would also discourage plaintiffs from staying out of prior litigation in order to secure in effect two bites at the apple using the prior litigation offensively if the defendant loses and forcing the defendant to litigate a second time if the defendant wins the prior action

An exceptional exam answer might therefore argue that non-mutual issue preclusion should be denied on these facts Son and Mother both reside in State A since they are related they know each other well and Son was driving Motherrsquos car when the accident occurred They could have sued together and Rule 20 of the Federal Rules of Civil Procedure would have authorized joinder of their claims because those claims arose from the same transaction or occurrence and raised a common question of law or fact FED R CIV P 20(a) The facts do not suggest that Mother had any reason not to join Sonrsquos suit other than a desire to see how Sonrsquos action concluded before bringing her own claim Cf Nations v Sun Oil Co (Del) 695 F2d 933 938 (5th Cir 1983) (concluding that plaintiff ldquowas entitled to await the development of his injuries and their predictable consequencesrdquo) Because it appears that Mother may be a ldquowait-and-seerdquo plaintiff who could easily have joined the original action a trial court might disallow as a matter of discretion her use of non-mutual issue preclusion]

30

AGENCY ANALYSIS __________ (Agency I II)

ANALYSIS

Legal Problems

(1) Is the principal or the agent or both liable on contracts with a third party when the principal is an ldquoundisclosed principalrdquo

(2) Is the principal or the agent or both liable on contracts with a third party when the principal is ldquopartially disclosedrdquo or an ldquounidentified principalrdquo

(3) Is the principal or the agent or both liable on contracts with a third party for the purchase of goods when the agent exceeded his authority but the principal nonetheless accepts the goods

DISCUSSION

Summary

The agent but not the owner is liable to the basket manufacturer because the owner is an undisclosed principal and the agent acted without actual or apparent authority Both the agent and the owner however are liable on the burner contract because the owner is an unidentified principal and the agent had apparent authority to enter into that contract With respect to the solar cells contract whether the owner is liable depends upon whether a court would follow the Second or Third Restatement of Agency which take different positions on the effect of the ratification of a contract by an undisclosed principal Under either the agent would also be liable on the contract as he was a party to the contract

[NOTE The contracts that are the subject of this question are contracts for the sale of goods and therefore are governed by Article 2 of the Uniform Commercial Code Article 2 however does not contain agency rules Accordingly common law concepts of agency are applicable UCC sect 1-103(b)]

Point One (35) The agent but not the owner is liable to the basket manufacturer The agent had no actual authority to enter into the contract to buy aluminum baskets and because the owner was an undisclosed principal the manufacturer had no reason to believe that the agent had apparent authority Furthermore the manufacturer had no reason to believe that the agent was not contracting for his own benefit

An agent acting on behalf of a principal can bind the principal to contracts if the agent has either actual or apparent authority An agent has actual authority when contracting on behalf of his principal if he ldquoreasonably believes in accordance with the principalrsquos manifestations to the agent that the principal wishes the agent so to actrdquo RESTATEMENT (THIRD) OF AGENCY sect 201 (2006) Here the agent was told to buy only wicker baskets not aluminum baskets Thus when he contracted with the basket manufacturer to buy aluminum baskets he had no actual authority to do so

31

Agency Analysis

An agent acts with apparent authority ldquowhen a third party [with whom the agent acts] reasonably believes the actor has authority to act on behalf of the principal and that belief is traceable to the principalrsquos manifestationsrdquo Id sect 203 Here the owner notified basket manufacturers that she or her agent might contact them to purchase baskets but that notification did not specifically name the agent or any other person as the ownerrsquos agent Furthermore the basket manufacturer had no prior dealings with the agent or the owner or any reason to think that the agent was acting for the benefit of anyone but himself Thus there is no basis to conclude that the basket manufacturer thought that the agent had apparent authority to act for the owner

Generally when an agent acts on behalf of an undisclosed principal and the agent lacks authority to enter into the contract the agent is liable on the contract as a party to the contract but the principal is not liable This rule is consistent with the third partyrsquos expectations ldquoThe third party expected the agent to be a party to the contract because the agent presented the deal as if he were acting for himself Moreover if the third party is unaware of the principalrsquos existence the third party must be relying on the agentrsquos solvency and reliability when entering into the contractrdquo See ROBERT W HAMILTON JONATHAN R MACEY amp DOUGLAS K MOLL CORPORATIONS INCLUDING PARTNERSHIPS AND LIMITED LIABILITY COMPANIES 34 (11th ed 2010) See also RESTATEMENT (THIRD) OF AGENCY sect 603 cmt c Furthermore because the third party has no idea that the agent is acting or is seemingly acting on behalf of another there is no reason to believe that the third party would be expecting an undisclosed principal to be liable on the contract Id

Point Two (35) Because the owner is an unidentified (as opposed to undisclosed) principal both she and the agent (as a party to the contract) probably are liable on the contract with the burner manufacturer

When the agent contracted with the burner manufacturer he did not have actual authority to do so as the owner had expressly restricted the agentrsquos authority to purchase only burners with ldquowhisper technologyrdquo See Point One However the agent may have had apparent authority to buy burners without whisper technology

An agent acts with apparent authority ldquowhen a third party [with whom the agent acts] reasonably believes the actor has authority to act on behalf of the principal and that belief is traceable to the principalrsquos manifestationsrdquo RESTATEMENT (THIRD) OF AGENCY sect 203 (2006) The owner indicated that an agent might contact the burner manufacturer The notice contained no restriction regarding the type of burners that the agent was authorized to purchase The facts indicate that burner manufacturers regularly receive such notices

Although the agent told the burner manufacturer that he represented a well-known hot-air balloon operator he did not disclose the ownerrsquos name Thus the owner was a partially disclosed or unidentified principal See RESTATEMENT (SECOND) OF AGENCY sect 4(2) (1958) (using term ldquopartially disclosed principalrdquo) RESTATEMENT (THIRD) OF AGENCY sect 104(2)(c) (2006) (using term ldquounidentified principalrdquo) An agent for a partially disclosed principal may have apparent authority RESTATEMENT (SECOND) OF AGENCY sect 159 cmt e (1958) Based upon (1) the notice sent by the owner (2) the agentrsquos revelation that he was acting as an agent and (3) the fact that burner manufacturers regularly receive such notices and sell to agents the manufacturer may argue that it reasonably and actually believed that the agent was authorized to purchase burners without whisper technology The manufacturer may also argue that because the agent revealed that he was an agent his listing of the ownerrsquos address as the delivery address connects the agent to the notice given by the owner Arguably this distinguishes the burner contract from the basket

32

Agency Analysis

contract Here there is a strong case to support the conclusion that the agent had apparent authority if he did then the owner is liable to the burner manufacturer

The agent also is liable as a party to the contract because he did not fully disclose his agency relationship Although he told the burner manufacturer that he represented a well-known hot-air balloon operator he did not disclose the ownerrsquos name Generally even an authorized agent of a partially disclosed or unidentified principal is liable as a party to a contract with a third person RESTATEMENT (SECOND) OF AGENCY sect 321 (1958) (ldquounless otherwise agreedrdquo) RESTATEMENT (THIRD) OF AGENCY sect 602(2) (2006) (ldquounless the agent and the third party agree otherwiserdquo)

Point Three (30) Under the Second Restatement of Agency the owner is not liable on the contract for solar cells because the agent did not have actual or apparent authority and the owner as an undisclosed principal cannot ratify the contract Under the Third Restatement the owner could be liable as she ratified the contract Under either Restatement the agent is liable as a party to the contract

The owner is not liable to the solar cell manufacturer for breach of the contract for the solar cells because the agent had no actual or apparent authority to purchase solar cells on the ownerrsquos behalf and the owner under the Second Restatement of Agency did not ratify the contract with knowledge of the material facts Thus she is not liable as a ratifier of the contract

The facts state that the agent had authority to purchase only propane fuel tanks In addition he had no apparent authority to purchase solar cells The owner made no manifestations to the solar cell manufacturer that would lead a reasonable person in the manufacturerrsquos position to believe that the agent had the authority to bind the owner to a contract to purchase solar cells In fact the agent made no manifestations at all to the solar cell manufacturer Unlike with the basket manufacturer and the burner manufacturer the owner did not notify the manufacturer of solar cells that an agent might contact it to purchase solar cells In addition the solar cells were delivered to the agent and not to the ownerrsquos address In sum the manufacturer was unaware of any relationship between the owner and the agent As to the solar cell manufacturer the owner is an undisclosed principal There can be no apparent authority in the case of an undisclosed principal because there are no manifestations from the principal to the third person See RESTATEMENT (SECOND) OF AGENCY sect 8 cmt a (1958) (ldquothere can be no apparent authority created by an undisclosed principalrdquo) RESTATEMENT (THIRD) OF AGENCY sect 203 cmt f (2006) (ldquoapparent authority is not present when a third party believes that an interaction is with an actor who is a principalrdquo)

The owner also did not ratify the contract Although the owner used the solar cells generally a principal cannot ratify an unauthorized transaction with a third person ldquounless the one acting purported to be acting for the ratifierrdquo RESTATEMENT (SECOND) OF AGENCY sect 85(1) (1958)

The result differs under the Third Restatement which expressly rejects the Second Restatement on this issue The Restatement (Third) of Agency sect 403 (2006) states ldquoA person may ratify an act if the actor acted or purported to act as an agent on the personrsquos behalfrdquo According to comment b ldquoan undisclosed principal may ratify an agentrsquos unauthorized actrdquo Under the Restatement (Third) of Agency rule the owner probably ratified the transaction The agent clearly acted on the ownerrsquos behalf and in addition the ownerrsquos conduct in using the solar cells ldquojustifies a reasonable assumption that [she] is manifesting assent that the act shall affect [her] legal relationsrdquo See id sect 401(2)

33

Agency Analysis

The agent also is liable to the solar cell manufacturer for breach of the contract for the solar cells because he is a party to the contract The facts indicate that the agent never told the solar cell manufacturer that he represented the owner or any other principal Consequently even if the agent were authorized (which as discussed above he is not) he would be liable as a party to the contract See RESTATEMENT (SECOND) OF AGENCY sect 322 (1958) RESTATEMENT (THIRD) OF AGENCY sect 603(2) (2006) Here he has no authority or apparent authority and is liable as a party to the contract

The agent would also be liable under the Third Restatement Under Restatement (Third) of Agency sect 402(1) (2006) ratification generally relates back and the transaction is treated as if it were authorized at the time of the transaction However this does not relieve the agent of an undisclosed principal who ratifies an unauthorized transaction of liability under the ratified contract See id sect 603(2) (authorized agent for undisclosed principal is a party to the contract) and sect 403 cmt b (ldquoAn undisclosed principalrsquos ratification does not eliminate the agentrsquos liability to the third party on the transaction rdquo)

[NOTE An examinee may discuss the concept of inherent agency power This concept is recognized by the Restatement (Second) of Agency sect 8 A (1958) but the concept is not used in the Restatement (Third) of Agency (2006) Here there are no facts to support that the agent had inherent authority

As to contracts with agents for partially disclosed principals (eg the contract for the burners) the basic question is whether the acts done ldquousually accompany or are incidental to transactions which the agent is authorized to conductrdquo RESTATEMENT (SECOND) OF AGENCY

sect 161 (1958) If so the principal is bound if the other party ldquoreasonably believes that the agent is authorized to do them and has no notice that he is not so authorizedrdquo Id The purchase of burners without whisper technology was not authorized nor was it incidental to an authorized transaction Therefore there should not be inherent agency power

As to contracts on behalf of undisclosed principals (eg the other two contracts) the basic question is whether the acts done are usual or necessary in the transactions the agent is authorized to transact RESTATEMENT (SECOND) OF AGENCY sect 194 (1958) The other two contracts seem fundamentally different from the authorized transactions Therefore there should not be inherent agency power

Only minimal credit should be given for discussion of inherent agency power]

34

EVIDENCE ANALYSIS _____ (Evidence IIA VA B E F J K)

ANALYSIS

Legal Problems

(1) Is the authenticated copy of the mechanicrsquos text message relevant and admissible

(2) Is the womanrsquos question ldquoIs my scooter safe to drive for a whilerdquo relevant and admissible

(3) Is the womanrsquos testimony describing the mechanicrsquos thumbs-up relevant and admissible

DISCUSSION

Summary

The mechanicrsquos text message to the woman is relevant to whether (1) the woman lost control of the scooter due to its defective brakes (2) the woman knew that the brakes needed repair and (3) it was negligent for the woman to drive the scooter knowing that its brakes needed repair

The mechanicrsquos text message is hearsay if it is offered by the pedestrian to prove that the scooterrsquos brakes needed repair However it fits the hearsay exception for present sense impressions and probably also fits the exception for business records The mechanicrsquos text message is not hearsay if it is instead offered by the pedestrian to prove the womanrsquos state of mind (ie that she had notice that her brakes needed repair)

The womanrsquos question to the mechanic and his response are also relevant to whether the brakes caused the accident and whether the woman was negligent The question is not hearsay because the woman did not make an assertion

The mechanicrsquos thumbs-up response is nonverbal conduct intended by the mechanic as an assertion and is therefore an out-of-court statement If the woman offers the mechanicrsquos statement to prove that the scooter was actually safe to ride the womanrsquos testimony about the statement is hearsay

However the mechanicrsquos statement is not hearsay if it is offered by the woman to prove her state of mind Therefore the womanrsquos question and the mechanicrsquos response are admissible to prove the womanrsquos state of mind

Point One(a) (20) The mechanicrsquos text message to the woman should be admitted because it is relevant

Evidence is relevant if it has ldquoany tendency to make a fact more or less probable than it would be without the evidencerdquo FED R EVID 401 ldquoRelevant evidence is admissiblerdquo unless it is inadmissible pursuant to some other rule FED R EVID 402

The mechanicrsquos text message to the woman ldquoWhen you pick up your scooter you need to schedule a follow-up brake repair Wersquoll order the partsrdquo is relevant for two reasons First this evidence has some tendency to make it more probable that the brakes malfunctioned and

35

Evidence Analysis

caused the accident Second it has some tendency to make it more probable that the woman was negligent in riding her scooter after being told by the mechanic that it required further repair

Point One(b) (30) The mechanicrsquos text message fits either the hearsay exception for present sense impressions or the exception for business records or it is admissible non-hearsay

The mechanicrsquos text message is a statement under Rule 801(a) because it is ldquoa written assertionrdquo FED R EVID 801(a) The text message is hearsay if the pedestrian offers it to prove the ldquotruth of the matter asserted in the statementrdquo (ie that the scooterrsquos brakes required repair) which resulted in the woman losing control of the scooter and causing the accident FED R EVID 801(c)

However the mechanicrsquos text message fits the hearsay exception for ldquopresent sense impressionsrdquo under Rule 803(1) because it is ldquo[a] statement describing or explaining an event or condition made while or immediately after the declarant perceived itrdquo FED R EVID 803(1) Here the mechanicrsquos text message described the condition of the scooter immediately after he perceived it during the maintenance service

The mechanic is a person with knowledge of the condition of the scooter so if text messages regarding repairs were made and kept by the mechanic in the ordinary course of business this text message also fits the business records exception Under Rule 803(6) a business record is a record of an act ldquomade at or near the time by someone with knowledgerdquo and ldquothe record was kept in the course of a regularly conducted activity of a businessrdquo and ldquomaking the record was a regular practice of that activityrdquo FED R EVID 803(6)

However the text message is not hearsay if it is instead offered to prove that the woman was negligent because she rode her scooter after the mechanic told her it required repair If offered for this purpose it would not be offered for the truth of the matter asserted in the statement but to show the womanrsquos belief about the condition of the scooter (her state of mind)

Point Two (10) The womanrsquos question to the mechanic should be admitted because it is not hearsay

The womanrsquos question to the mechanic is relevant because along with the mechanicrsquos thumbs-up response (see Point Three) it has some tendency to make it more probable that the woman was not negligent andor that the scooter brakes did not malfunction and cause the accident FED R EVID 401 The womanrsquos question does not raise hearsay concerns because it is not an assertion

Hearsay is defined under Rule 801(a) as ldquoan oral assertion written assertion or nonverbal conductrdquo Although ldquoassertionrdquo is not further defined ldquoa favorite [definition] of writers in the [evidence] field for at least a century and a half [is that] the word simply means to say that something is so eg that an event happened or a condition existedrdquo 2 MCCORMICK ON

EVIDENCE sect 246 (6th ed 2006) Under this definition the womanrsquos question is not hearsay because it is not an assertion

Point Three(a) (20) The mechanicrsquos thumbs-up to the woman is a nonverbal assertion that is relevant and the womanrsquos testimony about that response is admissible

36

Evidence Analysis

Hearsay is defined under Rule 801(c) as a ldquostatementrdquo that is ldquoa personrsquos oral assertion written assertion or nonverbal conduct if the person intended it as an assertionrdquo FED R EVID 801(a) Here when the mechanic responded to the womanrsquos question (ldquoIs my scooter safe to ride for a whilerdquo) with a thumbs-up gesture the facts suggest that he intended his nonverbal conduct as an assertion that in his opinion the scooter was safe to ride

The mechanicrsquos assertion is relevant and admissible to prove that the woman was not negligent because the evidence makes it more probable that at the time of the accident she believed that the scooter was safe to ride despite the fact that the brakes required repair FED R EVID 401 Admission of the womanrsquos description of the mechanicrsquos thumbs-up for this purpose does not raise hearsay concerns because the evidence would not be offered for the truth of the matter asserted but to show the womanrsquos belief about the condition of the scooter (her state of mind)

Point Three(b) (20) The mechanicrsquos thumbs-up is relevant to determine whether the scooterrsquos brakes malfunctioned causing the accident but if offered for this purpose it is also hearsay

The mechanicrsquos nonverbal assertion is relevant to the determination of whether the scooterrsquos brakes malfunctioned causing the accident However if offered to prove the ldquotruth of the matter asserted in the statementrdquo (ie that the scooter was safe to ride for a while) it is hearsay that does not fit any hearsay exception

37

TRUSTS AND FUTURE INTERESTS ANALYSIS ____________________ (Trusts and Future Interests IC1 amp 4 G IIF)

ANALYSIS

Legal Problems

(1)(a) Was the revocable trust amendable

(1)(b) If the trust was amendable must the amendment have been executed in accordance with the state Statute of Wills in order to be valid

(2) If the trust amendment was valid does the amendment apply to the probate estate assets passing to the trust pursuant to Settlorrsquos will

(3) If the trust amendment was valid should the trust property be distributed to University

(4) If the trust amendment was not valid should the trust property be distributed to Settlorrsquos grandchild (her only heir) or held in further trust in accordance with the terms of the original trust instrument

DISCUSSION

Summary

A revocable trust is amendable even if the trust instrument does not expressly grant to the trust settlor a power to amend Both inter vivos trusts and amendments thereto are valid even though not executed in accordance with the requirements applicable to wills

Under the Uniform Testamentary Additions to Trusts Act a revocable trust may be amended at any time prior to the settlorrsquos death and the amendment applies to the disposition of assets conveyed to the trust pursuant to a will even if the will was executed prior to the date of the amendment

At Settlorrsquos death trust assets including probate assets passing to the trust under Settlorrsquos will would go to University if as is the case here the trust amendment was valid If the amendment was invalid the trust assets would continue to be held in further trust because there is no violation of the common law Rule Against Perpetuities

Point One(a) (30) Settlor retained the right to amend the inter vivos trust despite her failure to expressly reserve this power

At issue here is whether a retained power of revocation includes the power to amend sometimes referred to as the power to modify The Restatement (Second) of Trusts sect 331 cmt g provides that if a settlor has a power to revoke that retained power ordinarily includes a power to modify (amend) as well Comment g also notes that the power to amend includes both a power to withdraw trust assets and a power to ldquomodify the terms of the trustrdquo The Uniform Trust Code which provides that a power to revoke includes the power to amend is consistent with this view

38

Trusts and Future Interests Analysis

UNIF TRUST CODE sect 602 accord RESTATEMENT (THIRD) OF TRUSTS sect 63 cmt The theory is that even though a power to amend was not expressly retained by a settlor the goal of amendment assuming the power was not included in the power to revoke could easily be achieved by first revoking the trust and then creating a new trust with the same terms contemplated by the amendment To require this would put form over substance

Thus by expressly retaining the power to revoke the trust Settlor retained a power to amend the inter vivos trust despite her failure to expressly reserve this power

[NOTE Under the common law a trust is irrevocable unless the settlor expressly retains a power to revoke the trust Conversely under the Uniform Trust Code a trust is revocable unless the terms of the trust expressly provide otherwise See UNIF TRUST CODE sect 602 The Trust Codersquos position on revocation follows the minority view in the United States and is inconsistent with prior Restatements of Trusts (see Restatement (Second) of Trusts sect 330) Here the trust is revocable because Settlor expressly retained a power of revocation

The Uniform Trust Code has been adopted in 24 jurisdictions Alabama Arizona Arkansas District of Columbia Florida Kansas Maine Michigan Missouri Nebraska New Hampshire New Mexico North Carolina North Dakota Ohio Oregon Pennsylvania South Carolina Tennessee Utah Vermont Virginia West Virginia and Wyoming]

Point One(b) (10) Settlorrsquos amendment of the trust was valid despite her failure to have her signature to the trust amendment witnessed

Neither the common law nor state statutes require a trust instrument or an amendment to a trust instrument to be executed in accordance with the formalities prescribed for execution of a will Indeed an inter vivos trust that does not involve real estate can be created orally Under the Uniform Trust Code the only requirements for creating a valid inter vivos trust are intent the specification of beneficiaries and the designation of a trustee See UNIF TRUST CODE sect 402 accord RESTATEMENT (THIRD) OF TRUSTS sect 13

Here the amendment meets the requirements of both the Uniform Trust Code and the common law Thus the fact that Settlorrsquos signature was not witnessed when she signed the amendment to the trust does not make the amendment invalid

Point Two (20) Under the Uniform Testamentary Additions to Trusts Act a revocable trust may be amended at any time prior to the settlorrsquos death and the amendment applies to probate assets poured into the trust at the settlorrsquos death pursuant to the settlorrsquos will even when the will was executed prior to the date of the amendment

Historically property owned by an individual at her death passed to the individualrsquos heirs or to beneficiaries designated in a will executed with the formalities (writing signing witnessing) prescribed by state law However when a will devises property to the trustee of an inter vivos trust then the provisions of the trustmdashwhich may not have been executed in accordance with the formalities required for willsmdasheffectively determine who will receive the property Because of this possibility some early cases held that if an inter vivos trust was not executed with the same formalities required for a valid will then the trust was ineffective to dispose of probate assets poured into the trust at the settlorrsquos death pursuant to the settlorrsquos will

This line of cases has been overturned by the Uniform Testamentary Additions to Trusts Act (the Act) now Uniform Probate Code sect 2-511 Under the Act adopted in almost all

39

Trusts and Future Interest Analysis

jurisdictions a testamentary bequest to the trustee of an inter vivos trust established by the testator during his or her lifetime is valid if the trust is in writing it is identified in the testatorrsquos will and the trust instrument was executed before concurrently with or after the execution of the will Id The Act further specifies that such a bequest is valid even if the trust is amendable or revocable and that a later amendment applies to assets passing to the trust by a previously executed will

Thus because the trust amendment is valid its terms apply to assets received by Bank from Settlorrsquos estate

Point Three (10) If the trust amendment was valid then the trust assets including assets passing to the trust under Settlorrsquos will should go to University

Under the trust amendment all trust assets (including the assets of Settlorrsquos probate estate poured into the trust) pass to University The facts provide no basis for failing to comply with Settlorrsquos stated intentions

Point Four (30) If the trust amendment was invalid trust assets including assets received pursuant to Settlorrsquos will should be held in accordance with the terms of the original trust instrument because those terms do not violate the Rule Against Perpetuities

Under the dispositive terms of the original trust instrument Settlor created successive income interests in her surviving children and grandchildren with a remainder interest in her great-grandchildren Because the trust was revocable the period during which the common law Rule Against Perpetuities requires that interests vest (ie 21 years plus lives in being) began to run from the date Settlor no longer had a power of revocation (here her death) not the date on which the trust was created See JESSE DUKEMINIER STANLEY J JOHANSON JAMES LINDGREN amp ROBERT SITKOFF WILLS TRUSTS AND ESTATES 678 (7th ed 2005)

Under the common law Rule Against Perpetuities Settlorrsquos trust is thus valid At the time of Settlorrsquos death she was survived by no children one granddaughter and no great-grandchildren Because Settlor cannot have more children after her death the only income beneficiary of the trust is Settlorrsquos surviving granddaughter This granddaughter is the only person who can produce great-grandchildren of Settlor thus all great-grandchildren must of necessity be born during the lifetime of Settlorrsquos only surviving granddaughter who is a life in being The granddaughterrsquos interest vested at Settlorrsquos death and the great-grandchildrenrsquos interest will vest at the death of the granddaughter There is no need to wait the additional 21 years permitted under the Rule Thus under the common law and the statute given in the facts the nonvested interest in the great-grandchildren is valid

[NOTE Both modern wait-and-see statutes and the Uniform Statutory Rule Against Perpetuities upon which the statute in the facts is modeled provide that before using either reform to validate an otherwise invalid nonvested interest one should first determine if the nonvested interest violates the common law Rule If it does not then there is no need to reform This proposition which is applicable in all MEE user jurisdictions that have not simply abrogated the rule is tested by this problem]

40

NEGOTIABLE INSTRUMENTS ANALYSIS (Negotiable Instruments III IV V)

ANALYSIS

Legal Problems

(1)(a) What rights does a person in possession of a note that has been indorsed in blank by the payee have against the maker of the note

(1)(b) Which defenses may the maker of a note raise against a person entitled to enforce it who is not a holder in due course but is a transferee from a holder in due course

(2) What rights does a person entitled to enforce a note have against an indorser who transferred it for consideration with no warranties

(3) What rights does a person entitled to enforce a note have against a previous holder who transferred it as a gift without indorsing it

DISCUSSION

Summary

The niece is a holder of the note and is thus a person entitled to enforce it The chef the issuer of the note is obligated to pay it to the niece as the person entitled to enforce it The niece is not subject to any defense or claim of the chef relating to the improper repair of the oven because the niece has the rights of a holder in due course When the buyer bought the note from the repairman the buyer became a holder in due course of the note and thus took it free of any personal defenses the chef had against the repairman Even though the niece is not herself a holder in due course of the note the niece succeeded to the buyerrsquos rights as holder in due course and thus took free of the chefrsquos personal defenses

Because the chef refused to pay the note the niece can recover from the repairman on the repairmanrsquos obligation as indorser The niece cannot recover on the note against the buyer however because the buyer did not indorse the note (and thus incurred no indorserrsquos obligation) and the buyer did not receive any consideration for transfer of the note to the niece (and therefore made no transfer warranty)

[NOTE Although Article 9 of the Uniform Commercial Code governs the sale of promissory notes (a point that might be correctly noted by examinees) that Article does not determine the answer to any of the questions posed]

Point One(a) (20) The niece is the holder of the note and thus may enforce it against the chef who is the issuer of the note

The chef is the maker of the note and thus its issuer See UCC sectsect 3-103 3-105 The issuer of a note is obligated to pay it in accordance with its terms to a ldquoperson entitled to enforcerdquo it UCC sect 3-412 The niece is a ldquoperson entitled to enforcerdquo the note This is because the niece is the holder of the note and a holder of a note is a person entitled to enforce it UCC sect 3-301 The niece is the holder of the note because (i) the repairmanrsquos signature on the back of the note not

41

Negotiable Instruments Analysis

accompanied by words indicating a person to whom the note was made payable was a ldquoblank indorsementrdquo which had the effect of making the note a bearer instrument (ii) anyone in possession of a bearer instrument is a holder of it and (iii) the niece is in possession of the note See UCC sectsect 1-201(b)(21)(A) 3-204 and 3-205 Accordingly the chef has an obligation to the niece to pay the note in accordance with its terms and the niece may enforce that obligation

Point One(b) (40) The niece is not a holder in due course of the note but because she is a transferee from the buyer who was a holder in due course she has the same enforcement rights as the buyer Because the buyer as a holder in due course would have been able to enforce the note against the chef without being subject to defenses or claims arising from the improper repair the niece has the same rights and will not be subject to the chefrsquos defenses or claims about the repair

As noted in Point One(a) the chef has an obligation to the niece to pay the note in accordance with its terms However except against a person with the rights of a holder in due course the chef can raise any defenses or claims in recoupment that he would have if the claim on the note were an ordinary contract claim UCC sect 3-305 Thus except against a holder in due course the chef would be able to raise the improper repair as a defense or a claim in recoupment (a claim in response to the niecersquos claim)

But claims in recoupment and most defenses cannot be raised against a person with the rights of a holder in due course Against a holder in due course the chef can raise only the four ldquorealrdquo defenses listed in UCC sect 3-305(a)(1) (infancy duress lack of legal capacity or illegality that nullifies the obligation of the obligor under other law fraud in the factum discharge in insolvency proceedings) none of which is present here

The niece is not a holder in due course because she did not take the note for value See UCC sectsect 3-302(a)(2)(i) (criteria for holder in due course status) and 3-303(a) (definition of ldquovaluerdquo) But this does not mean that the niece is subject to the chefrsquos claim arising out of the improper repair The buyer was a holder in due course of the note because he took the note for value ($9500) in good faith and without notice of any facts that would have alerted him to the chefrsquos defense against the repairman UCC sect 3-302(a)(2) As a holder in due course the buyer owned the note free of the chefrsquos claim because that claim did not constitute a ldquorealrdquo defense UCC sect 3-305(b) When the buyer gave the note to the niece this constituted a ldquotransferrdquo of the note See UCC sect 3-203(a) When a note is transferred the transferee receives ldquoany right of the transferor to enforce the instrument including any right as a holder in due courserdquo UCC sect 3-203(b) Under this rule (also known as the ldquoshelter principlerdquo) the buyer transferred his freedom from the chefrsquos defenses to the niece and the niece can enforce the note free of the chefrsquos defenses

Point Two (20) Because the chef dishonored the note the niece can recover from the repairman on the repairmanrsquos obligation as indorser

The chefrsquos refusal to pay the note constituted dishonor See UCC sect 3-502 The repairman as an indorser of the note (see Point One(a)) incurred the obligations of an indorser under UCC sect 3-415(a) When a note has been dishonored one of the obligations of an indorser is to pay the amount of the note to a person entitled to enforce it Therefore the repairman is liable for the amount of the note to the niece a person entitled to enforce the note (so long as the niece gives proper notice of dishonor to the repairman)

42

Negotiable Instruments Analysis

[NOTE Because the repairman indorsed the note without warranties there are no transfer warranties UCC sect 3-416 cmt 5]

Point Three (20) The niece cannot recover on the note against the buyer as either indorser or warrantor because the buyer did not indorse the note and did not receive consideration for transferring the note to the niece

The buyer did not indorse the note and therefore did not incur the obligation of an indorser to pay the note upon dishonor

The niece cannot recover from the buyer under a transfer warranty theory because transfer warranties are made only by a person ldquowho transfers an instrument for considerationrdquo Here the buyer gave the instrument to the niece as a gift So the buyer made no transfer warranty UCC sect 3-416(a) Therefore the niece cannot recover from the buyer on that theory

43

National Conference of Bar Examiners 302 South Bedford Street | Madison WI 53703-3622 Phone 608-280-8550 | Fax 608-280-8552 | TDD 608-661-1275

wwwncbexorg e-mail contactncbexorg

  • Contents
  • Preface
  • Description of the MEE
  • Instructions
  • February 2013 Questions
    • Real Property Question
    • Contracts Question
    • Constitutional Law Question
    • Secured Transactions Question
    • Federal Civil Procedure Question
    • Agency Question
    • Evidence Question
    • Trusts and Future Interests Question
    • Negotiable Instruments Question
      • February 2013 Analyses
        • Real Property Analysis
        • Contracts Analysis
        • Constitutional Law Analysis
        • Secured Transactions Analysis
        • Federal Civil Procedure Analysis
        • Agency Analysis
        • Evidence Analysis
        • Trusts and Future Interests Analysis
        • Negotiable Instruments Analysis
            • ltlt ASCII85EncodePages false AllowTransparency false AutoPositionEPSFiles true AutoRotatePages None Binding Left CalGrayProfile (Dot Gain 20) CalRGBProfile (sRGB IEC61966-21) CalCMYKProfile (US Web Coated 050SWOP051 v2) sRGBProfile (sRGB IEC61966-21) CannotEmbedFontPolicy Error CompatibilityLevel 14 CompressObjects Tags CompressPages true ConvertImagesToIndexed true PassThroughJPEGImages true CreateJobTicket false DefaultRenderingIntent Default DetectBlends true DetectCurves 00000 ColorConversionStrategy CMYK DoThumbnails false EmbedAllFonts true EmbedOpenType false ParseICCProfilesInComments true EmbedJobOptions true DSCReportingLevel 0 EmitDSCWarnings false EndPage -1 ImageMemory 1048576 LockDistillerParams false MaxSubsetPct 100 Optimize true OPM 1 ParseDSCComments true ParseDSCCommentsForDocInfo true PreserveCopyPage true PreserveDICMYKValues true PreserveEPSInfo true PreserveFlatness true PreserveHalftoneInfo false PreserveOPIComments true PreserveOverprintSettings true StartPage 1 SubsetFonts true TransferFunctionInfo Apply UCRandBGInfo Preserve UsePrologue false ColorSettingsFile () AlwaysEmbed [ true ] NeverEmbed [ true ] AntiAliasColorImages false CropColorImages true ColorImageMinResolution 300 ColorImageMinResolutionPolicy OK DownsampleColorImages true ColorImageDownsampleType Bicubic ColorImageResolution 300 ColorImageDepth -1 ColorImageMinDownsampleDepth 1 ColorImageDownsampleThreshold 150000 EncodeColorImages true ColorImageFilter DCTEncode AutoFilterColorImages true ColorImageAutoFilterStrategy JPEG ColorACSImageDict ltlt QFactor 015 HSamples [1 1 1 1] VSamples [1 1 1 1] gtgt ColorImageDict ltlt QFactor 015 HSamples [1 1 1 1] VSamples [1 1 1 1] gtgt JPEG2000ColorACSImageDict ltlt TileWidth 256 TileHeight 256 Quality 30 gtgt JPEG2000ColorImageDict ltlt TileWidth 256 TileHeight 256 Quality 30 gtgt AntiAliasGrayImages false CropGrayImages true GrayImageMinResolution 300 GrayImageMinResolutionPolicy OK DownsampleGrayImages true GrayImageDownsampleType Bicubic GrayImageResolution 300 GrayImageDepth -1 GrayImageMinDownsampleDepth 2 GrayImageDownsampleThreshold 150000 EncodeGrayImages true GrayImageFilter DCTEncode AutoFilterGrayImages true GrayImageAutoFilterStrategy JPEG GrayACSImageDict ltlt QFactor 015 HSamples [1 1 1 1] VSamples [1 1 1 1] gtgt GrayImageDict ltlt QFactor 015 HSamples [1 1 1 1] VSamples [1 1 1 1] gtgt JPEG2000GrayACSImageDict ltlt TileWidth 256 TileHeight 256 Quality 30 gtgt JPEG2000GrayImageDict ltlt TileWidth 256 TileHeight 256 Quality 30 gtgt AntiAliasMonoImages false CropMonoImages true MonoImageMinResolution 1200 MonoImageMinResolutionPolicy OK DownsampleMonoImages true MonoImageDownsampleType Bicubic MonoImageResolution 1200 MonoImageDepth -1 MonoImageDownsampleThreshold 150000 EncodeMonoImages true MonoImageFilter CCITTFaxEncode MonoImageDict ltlt K -1 gtgt AllowPSXObjects false CheckCompliance [ None ] PDFX1aCheck false PDFX3Check false PDFXCompliantPDFOnly false PDFXNoTrimBoxError true PDFXTrimBoxToMediaBoxOffset [ 000000 000000 000000 000000 ] PDFXSetBleedBoxToMediaBox true PDFXBleedBoxToTrimBoxOffset [ 000000 000000 000000 000000 ] PDFXOutputIntentProfile () PDFXOutputConditionIdentifier () PDFXOutputCondition () PDFXRegistryName () PDFXTrapped False CreateJDFFile false Description ltlt ARA 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 BGR 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 CHS ltFEFF4f7f75288fd94e9b8bbe5b9a521b5efa7684002000410064006f006200650020005000440046002065876863900275284e8e9ad88d2891cf76845370524d53705237300260a853ef4ee54f7f75280020004100630072006f0062006100740020548c002000410064006f00620065002000520065006100640065007200200035002e003000204ee553ca66f49ad87248672c676562535f00521b5efa768400200050004400460020658768633002gt CHT ltFEFF4f7f752890194e9b8a2d7f6e5efa7acb7684002000410064006f006200650020005000440046002065874ef69069752865bc9ad854c18cea76845370524d5370523786557406300260a853ef4ee54f7f75280020004100630072006f0062006100740020548c002000410064006f00620065002000520065006100640065007200200035002e003000204ee553ca66f49ad87248672c4f86958b555f5df25efa7acb76840020005000440046002065874ef63002gt CZE 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 DAN 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 DEU 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 ESP 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 ETI 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 FRA 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 GRE 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 HEB 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 HRV (Za stvaranje Adobe PDF dokumenata najpogodnijih za visokokvalitetni ispis prije tiskanja koristite ove postavke Stvoreni PDF dokumenti mogu se otvoriti Acrobat i Adobe Reader 50 i kasnijim verzijama) HUN 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 ITA 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 JPN ltFEFF9ad854c18cea306a30d730ea30d730ec30b951fa529b7528002000410064006f0062006500200050004400460020658766f8306e4f5c6210306b4f7f75283057307e305930023053306e8a2d5b9a30674f5c62103055308c305f0020005000440046002030d530a130a430eb306f3001004100630072006f0062006100740020304a30883073002000410064006f00620065002000520065006100640065007200200035002e003000204ee5964d3067958b304f30533068304c3067304d307e305930023053306e8a2d5b9a306b306f30d530a930f330c8306e57cb30818fbc307f304c5fc59808306730593002gt KOR ltFEFFc7740020c124c815c7440020c0acc6a9d558c5ec0020ace0d488c9c80020c2dcd5d80020c778c1c4c5d00020ac00c7a50020c801d569d55c002000410064006f0062006500200050004400460020bb38c11cb97c0020c791c131d569b2c8b2e4002e0020c774b807ac8c0020c791c131b41c00200050004400460020bb38c11cb2940020004100630072006f0062006100740020bc0f002000410064006f00620065002000520065006100640065007200200035002e00300020c774c0c1c5d0c11c0020c5f40020c2180020c788c2b5b2c8b2e4002egt LTH 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 LVI 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 NLD (Gebruik deze instellingen om Adobe PDF-documenten te maken die zijn geoptimaliseerd voor prepress-afdrukken van hoge kwaliteit De gemaakte PDF-documenten kunnen worden geopend met Acrobat en Adobe Reader 50 en hoger) NOR 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 POL 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 PTB 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 RUM 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 RUS ltFEFF04180441043f043e043b044c04370443043904420435002004340430043d043d044b04350020043d0430044104420440043e0439043a043800200434043b044f00200441043e043704340430043d0438044f00200434043e043a0443043c0435043d0442043e0432002000410064006f006200650020005000440046002c0020043c0430043a04410438043c0430043b044c043d043e0020043f043e04340445043e0434044f04490438044500200434043b044f00200432044b0441043e043a043e043a0430044704350441044204320435043d043d043e0433043e00200434043e043f0435044704300442043d043e0433043e00200432044b0432043e04340430002e002000200421043e043704340430043d043d044b04350020005000440046002d0434043e043a0443043c0435043d0442044b0020043c043e0436043d043e0020043e0442043a0440044b043204300442044c002004410020043f043e043c043e0449044c044e0020004100630072006f00620061007400200438002000410064006f00620065002000520065006100640065007200200035002e00300020043800200431043e043b043504350020043f043e04370434043d043804450020043204350440044104380439002egt SKY 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 SLV 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 SUO 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 SVE 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 TUR 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 UKR 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 ENU (Use these settings to create Adobe PDF documents best suited for high-quality prepress printing Created PDF documents can be opened with Acrobat and Adobe Reader 50 and later) gtgt Namespace [ (Adobe) (Common) (10) ] OtherNamespaces [ ltlt AsReaderSpreads false CropImagesToFrames true ErrorControl WarnAndContinue FlattenerIgnoreSpreadOverrides false IncludeGuidesGrids false IncludeNonPrinting false IncludeSlug false Namespace [ (Adobe) (InDesign) (40) ] OmitPlacedBitmaps false OmitPlacedEPS false OmitPlacedPDF false SimulateOverprint Legacy gtgt ltlt AddBleedMarks false AddColorBars false AddCropMarks false AddPageInfo false AddRegMarks false ConvertColors ConvertToCMYK DestinationProfileName () DestinationProfileSelector DocumentCMYK Downsample16BitImages true FlattenerPreset ltlt PresetSelector MediumResolution gtgt FormElements false GenerateStructure false IncludeBookmarks false IncludeHyperlinks false IncludeInteractive false IncludeLayers false IncludeProfiles false MultimediaHandling UseObjectSettings Namespace [ (Adobe) (CreativeSuite) (20) ] PDFXOutputIntentProfileSelector DocumentCMYK PreserveEditing true UntaggedCMYKHandling LeaveUntagged UntaggedRGBHandling UseDocumentProfile UseDocumentBleed false gtgt ]gtgt setdistillerparamsltlt HWResolution [2400 2400] PageSize [612000 792000]gtgt setpagedevice

Page 4: February 2013 MEE Questions and Analyses

Preface

The Multistate Essay Examination (MEE) is developed by the National Conference of Bar Examiners (NCBE) This publication includes the questions and analyses from the February 2013 MEE Each test includes nine 30-minute questions user jurisdictions may elect which of the nine questions they wish to use (Jurisdictions that administer the Uniform Bar Examination [UBE] use a common set of six MEE questions as part of their bar examinations) In the actual test the questions are simply numbered rather than being identified by area of law The instructions for the test appear on page iii For more information see the MEE Information Booklet available on the NCBE website at wwwncbexorg

The model analyses for the MEE are illustrative of the discussions that might appear in excellent answers to the questions They are provided to the user jurisdictions to assist graders in grading the examination They address all the legal and factual issues the drafters intended to raise in the questions

The subjects covered by each question are listed on the first page of its accompanying analysis followed by roman numerals that refer to the MEE subject matter outline for that subject For example the Federal Civil Procedure question on the February 2013 MEE tested the following area from the Federal Civil Procedure outline VIE Verdicts and judgmentsmdashEffect claim and issue preclusion Subject matter outlines are included in the MEE Information Booklet

Description of the MEE

The MEE consists of nine 30-minute essay questions any of which a jurisdiction may select to include as part of its bar examination (UBE jurisdictions use a common set of six MEE questions as part of their bar examinations) It is administered by participating jurisdictions on the Tuesday before the last Wednesday in February and July of each year The areas of law that may be covered by the questions on any MEE are Business Associations (Agency and Partnership Corporations and Limited Liability Companies) Conflict of Laws Constitutional Law Contracts Criminal Law and Procedure Evidence Family Law Federal Civil Procedure Real Property Torts Trusts and Estates (Decedentsrsquo Estates Trusts and Future Interests) and Uniform Commercial Code (Negotiable Instruments and Bank Deposits and Collections Secured Transactions) Some questions may include issues in more than one area of law The particular areas covered vary from exam to exam

The purpose of the MEE is to test the examineersquos ability to (1) identify legal issues raised by a hypothetical factual situation (2) separate material which is relevant from that which is not (3) present a reasoned analysis of the relevant issues in a clear concise and well-organized composition and (4) demonstrate an understanding of the fundamental legal principles relevant to the probable solution of the issues raised by the factual situation The primary distinction between the MEE and the Multistate Bar Examination (MBE) is that the MEE requires the examinee to demonstrate an ability to communicate effectively in writing

ii

_____

Instructions

The back cover of each test booklet contains the following instructions

You will be instructed when to begin and when to stop this test Do not break the seal on this booklet until you are told to begin

You may answer the questions in any order you wish Do not answer more than one question in each answer booklet If you make a mistake or wish to revise your answer simply draw a line through the material you wish to delete

If you are using a laptop computer to answer the questions your jurisdiction will provide you with specific instructions

Read each fact situation very carefully and do not assume facts that are not given in the question Do not assume that each question covers only a single area of the law some of the questions may cover more than one of the areas you are responsible for knowing

Demonstrate your ability to reason and analyze Each of your answers should show an understanding of the facts a recognition of the issues included a knowledge of the applicable principles of law and the reasoning by which you arrive at your conclusion The value of your answer depends not as much upon your conclusions as upon the presence and quality of the elements mentioned above

Clarity and conciseness are important but make your answer complete Do not volunteer irrelevant or immaterial information

Answer all questions according to generally accepted fundamental legal principles unless your jurisdiction has instructed you to answer according to local case or statutory law (UBE instructions Answer all questions according to generally accepted fundamental legal principles rather than local case or statutory law)

iii

February 2013 MEE

QUESTIONS

Real Property Contracts

Constitutional Law Secured Transactions

Federal Civil Procedure Agency

Evidence Trusts and Future Interests

Negotiable Instruments

REAL PROPERTY QUESTION _______________

In 2008 a landlord and a tenant entered into a 10-year written lease commencing September 1 2008 for the exclusive use of a commercial building at a monthly rent of $2500 The lease contained a covenant of quiet enjoyment but no other covenants or promises on the part of the landlord

When the landlord and tenant negotiated the lease the tenant asked the landlord if the building had an air-conditioning system The landlord answered ldquoYes it doesrdquo The tenant responded ldquoGreat I will be using the building to manufacture a product that will be irreparably damaged if the temperature during manufacture exceeds 81 degrees for more than six consecutive hoursrdquo

On April 15 2012 the buildingrsquos air-conditioning system malfunctioned causing the building temperature to rise above 81 degrees for three hours The tenant immediately telephoned the landlord about this malfunction The tenant left a message in which he explained what had happened and asked the landlord ldquoWhat are you going to do about itrdquo The landlord did not respond to the tenantrsquos message

On May 15 2012 the air-conditioning system again malfunctioned This time the malfunction caused the building temperature to rise above 81 degrees for six hours The tenant telephoned the landlord and left a message describing the malfunction As before the landlord did not respond

On August 24 2012 the air-conditioning system malfunctioned again causing the temperature to rise above 81 degrees for 10 hours Again the tenant promptly telephoned the landlord The landlord answered the phone and the tenant begged her to fix the system The landlord refused The tenant then attempted to fix the system himself but he failed As a result of the air-conditioning malfunction products worth $150000 were destroyed

The next day the tenant wrote the following letter to the landlord

Irsquove had enough I told you about the air-conditioning problem twice before yesterdayrsquos disaster and you failed to correct it I will vacate the building by the end of the month and will bring you the keys when I leave

The tenant vacated the building on August 31 2012 and returned the keys to the landlord that day At that time there were six years remaining on the lease

On September 1 2012 the landlord returned the keys to the tenant with a note that said ldquoI repeat the air-conditioning is not my problem You have leased the building and you should fix itrdquo The tenant promptly sent the keys back to the landlord with a letter that said ldquoI have terminated the lease and I will not be returning to the building or making further rent paymentsrdquo After receiving the keys and letter the landlord put the keys into her desk To date she has neither responded to the tenantrsquos letter nor taken steps to lease the building to another tenant

On November 1 2012 two months after the tenant vacated the property the landlord sued the tenant claiming that she is entitled to the remaining unpaid rent ($180000) from September 1 for the balance of the lease term (reduced to present value) or if not that then damages for the tenantrsquos wrongful termination

Is the landlord correct Explain

3

CONTRACTS QUESTION _______________

On January 2 a boat builder and a sailor entered into a contract pursuant to which the builder was to sell to the sailor a boat to be specially manufactured for the sailor by the builder The contract price was $100000 The written contract signed by both parties stated that the builder would tender the boat to the sailor on December 15 at which time payment in full would be due

On October 15 the builderrsquos workers went on strike and there were no available replacements

On October 31 the builderrsquos workers were still on strike and no work was being done on the boat The sailor read a news report about the strike and immediately sent a letter to the builder stating ldquoI am very concerned that my boat will not be completed by December 15 I insist that you provide me with assurance that you will perform in accordance with the contractrdquo The builder received the letter on the next day November 1

On November 25 the builder responded to the letter stating ldquoIrsquom sorry about the strike but it is really out of my hands I hope we settle it soon so that we can get back to workrdquo

Nothing further happened until December 3 when the builder called the sailor and said ldquoMy workers are back and I have two crews working overtime to finish your boat Your boat is task one Donrsquot worry wersquoll deliver your boat by December 15thrdquo The sailor immediately replied ldquoI donrsquot trust you As far as Irsquom concerned our contract is over I am going to buy my boat from a shipyardrdquo Two days later the sailor entered into a contract with a competing manufacturer to buy a boat similar to the boat that was the subject of the contract with the builder

The builder finished the boat on time and tendered it to the sailor on December 15 The sailor reminded the builder about the December 3 conversation in which the sailor had announced that ldquoour contract is overrdquo and refused to take the boat and pay for it

The builder has sued the sailor for breach of contract

1 What was the legal effect of the sailorrsquos October 31 letter to the builder Explain

2 What was the legal effect of the builderrsquos November 25 response to the sailorrsquos October 31 letter Explain

3 What was the legal effect of the sailorrsquos refusal to take and pay for the boat on December 15 Explain

4

CONSTITUTIONAL LAW QUESTION

AutoCo is a privately owned corporation that manufactures automobiles Ten years ago AutoCo purchased a five-square-mile parcel of unincorporated land in a remote region of the state and built a large automobile assembly plant on the land To attract workers to the remote location of the plant AutoCo built apartment buildings and houses on the land and leased them to its employees AutoCo owns and operates a commercial district with shops and streets open to the general public AutoCo named the area Oakwood and provides security fire protection and sanitation services for Oakwoodrsquos residents AutoCo also built operates and fully funds the only school in the region which it makes available free of charge to the children of its employees

A family recently moved to Oakwood The father and mother work in AutoCorsquos plant rent an apartment from AutoCo and have enrolled their 10-year-old son in Oakwoodrsquos school Every morning the students are required to recite the Pledge of Allegiance while standing and saluting an American flag With the approval of his parents the son has politely but insistently refused to recite the Pledge and salute the flag at the school on the grounds that doing so violates his own political beliefs and the political beliefs of his family As a result of his refusal to say the Pledge the son has been expelled from the school

To protest the schoolrsquos actions the father walked into the commercial district of Oakwood While standing on a street corner he handed out leaflets that contained a short essay critical of the schoolrsquos Pledge of Allegiance policy Some of the passersby who took the leaflets dropped them to the ground An AutoCo security guard saw the litter told the father that Oakwoodrsquos anti-litter rule prohibits leaflet distribution that results in littering and directed him to cease distribution of the leaflets and leave the commercial district When the father did not leave and continued to distribute the leaflets the security guard called the state police which sent officers who arrested the father for trespass

1 Did the sonrsquos expulsion from the school violate the First Amendment as applied through the Fourteenth Amendment Explain

2 Did the fatherrsquos arrest violate the First Amendment as applied through the Fourteenth Amendment Explain

5

SECURED TRANSACTIONS QUESTION

On June 1 a bicycle retailer sold two bicycles to a man for a total purchase price of $1500 The man made a $200 down payment and agreed to pay the balance in one year The man also signed a security agreement that identified the bicycles as collateral for the unpaid purchase price and provided that the man ldquoshall not sell or dispose of the collateral until the balance owed is paid in fullrdquo The retailer never filed a financing statement reflecting this security interest

The man had bought the bicycles for him and his girlfriend to use on vacation However shortly after he bought the bicycles the man and his girlfriend broke up The man has never used the bicycles

On August 1 the man sold one of the bicycles at a garage sale to a buyer who paid the man $400 for the bicycle The buyer bought the bicycle to ride for weekend recreation

On October 1 the man gave the other bicycle to his friend as a birthday present The friend began using the bicycle for morning exercise

Neither the buyer nor the friend had any knowledge of the manrsquos dealings with the retailer

1 Does the buyer own the bicycle free of the retailerrsquos security interest Explain

2 Does the friend own the bicycle free of the retailerrsquos security interest Explain

6

FEDERAL CIVIL PROCEDURE QUESTION _____

Mother and Son who are both adults are citizens and residents of State A Mother owned an expensive luxury car valued in excess of $100000 Son borrowed Motherrsquos car to drive to a store in State A As Son approached a traffic light that had just turned yellow he carefully braked and brought the car to a complete stop Driver who was following immediately behind him failed to stop and rear-ended Motherrsquos car which was damaged beyond repair Son was seriously injured Driver is a citizen of State B

Son sued Driver in the United States District Court for the District of State A alleging that she was negligent in the operation of her vehicle Son sought damages in excess of $75000 for his personal injuries exclusive of costs and interest In her answer Driver alleged that Son was contributorily negligent in the operation of Motherrsquos car She further alleged that the brake lights on Motherrsquos car were burned out and that Motherrsquos negligent failure to properly maintain the car was a contributing cause of the accident

Following a trial on the merits in Sonrsquos case against Driver the jury answered the following special interrogatories

Do you find that Driver was negligent in the operation of her vehicle Yes

Do you find that Son was negligent in the operation of Motherrsquos car No

Do you find that Mother negligently failed to ensure that the brake lights on her car were in proper working order Yes

The judge then entered a judgment in favor of Son against Driver Driver did not appeal

Two months later Mother sued Driver in the United States District Court for the District of State A alleging that Driverrsquos negligence in the operation of her vehicle destroyed Motherrsquos luxury car Mother sought damages in excess of $75000 exclusive of costs and interest

State A follows the same preclusion principles that federal courts follow in federal-question cases

1 Is Motherrsquos claim against Driver barred by the judgment in Son v Driver Explain

2 Does the juryrsquos conclusion in Son v Driver that Mother had negligently failed to maintain the brake lights on her car preclude Mother from litigating that issue in her subsequent suit against Driver Explain

3 Does the juryrsquos conclusion in Son v Driver that Driver was negligent preclude Driver from litigating that issue in the Mother v Driver lawsuit Explain

7

_____

AGENCY QUESTION

Over 5000 individuals in the United States operate hot-air balloon businesses A hot-air balloon has four key components the balloon that holds the heated air the basket that houses the riders the propane burner that heats the air in the balloon and the propane storage tanks

The owner of a hot-air balloon business recently notified several basket and burner manufacturers that she or her agent might be contacting them to purchase baskets or burners The owner did not specifically name any person as her agent Basket and burner manufacturers regularly receive such notices from hot-air balloon operators Such notices typically include no restrictions on the types of baskets or burners agents might purchase for their principals

The owner then retained an agent to acquire baskets burners and fuel tanks from various manufacturers The owner authorized the agent to buy only (a) baskets made of woven wicker (not aluminum) (b) burners that use a unique ldquowhisper technologyrdquo (so as not to scare livestock when the balloon sails over farmland) and (c) propane fuel tanks

The agent then entered into three transactions with manufacturers all of whom had no prior dealings with either the owner or the agent

(1) The agent and a large manufacturer of both wicker and aluminum baskets signed a contract for the purchase of four aluminum baskets for a total cost of $60000 The agent never told the manufacturer that he represented the owner or any other principal The contract listed the agent as the buyer and listed the ownerrsquos address as the delivery address but did not indicate that the address was that of the owner rather than the agent When the baskets were delivered to the owner she learned for the first time that the agent had contracted to buy aluminum not wicker baskets The owner immediately rejected the baskets and returned them to the manufacturer Neither the owner nor the agent has paid the basket manufacturer for them

(2) The agent contacted a burner manufacturer and told him that the agent represented a well-known hot-air balloon operator who wanted to purchase burners The agent did not disclose the ownerrsquos name The agent and the burner manufacturer signed a contract for the purchase of four burners that did not have ldquowhisper technologyrdquo for a total price of $70000 The burner contract like the basket contract listed the ownerrsquos address for delivery but did not disclose whose address it was The burners were delivered to the ownerrsquos business and the owner discovered that the agent had ordered the wrong kind of burners The owner rejected the burners and returned them to the manufacturer Neither the owner nor the agent has paid the burner manufacturer for the burners

(3) The agent contracted with a solar cell manufacturer to make three cells advertised as ldquostrong enough to power all your ballooning needsrdquo The agent did not tell the manufacturer that he was acting on behalf of any other person One week after the cells were delivered to the agent he took them to the owner who installed them and discovered that she could save a lot of money using solar cells instead of propane to power her balloons The owner decided to keep the solar cells but she has not paid the manufacturer for them

8

Agency Question

Assume that the rejection of the baskets and the burners and the failure to pay for the solar cells constitute breach of the relevant contracts

1 Is the owner liable to the basket manufacturer for breach of the contract for the aluminum baskets Is the agent liable Explain

2 Is the owner liable to the burner manufacturer for breach of the contract for the burners Is the agent liable Explain

3 Is the owner liable to the solar cell manufacturer for breach of the contract for the solar cells Is the agent liable Explain (Do not address liability based upon restitution or unjust enrichment)

9

EVIDENCE QUESTION _____

A woman who owns a motorized scooter brought her scooter to a mechanic for routine maintenance service As part of the maintenance service the mechanic inspected the braking system on the scooter As soon as the mechanic finished inspecting and servicing the scooter he sent the woman a text message to her cell phone that read ldquoJust finished your service When you pick up your scooter you need to schedule a follow-up brake repair Wersquoll order the partsrdquo

The woman read the mechanicrsquos text message and returned the next day to pick up her scooter As the woman was wheeling her scooter out of the shop she saw the mechanic working nearby and asked ldquoIs my scooter safe to ride for a whilerdquo The mechanic responded by giving her a thumbs-up The woman waved and rode away on the scooter

One week later while the woman was riding her scooter a pedestrian stepped off the curb into a crosswalk and the woman collided with him causing the pedestrian severe injuries The woman had not had the scooterrsquos brakes repaired before the accident

The pedestrian has sued the woman for damages for his injuries resulting from the accident The pedestrian has alleged that (1) the woman lost control of the scooter due to its defective brakes (2) the woman knew that the brakes needed repair and (3) it was negligent for the woman to ride the scooter knowing that its brakes needed to be repaired

The woman claims that the brakes on the scooter worked perfectly and that the accident happened because the pedestrian stepped into the crosswalk without looking and the woman had no time to stop The woman the pedestrian and the mechanic will testify at the upcoming trial

The pedestrian has proffered an authenticated copy of the mechanicrsquos text message to the woman

The woman plans to testify that she asked the mechanic ldquoIs my scooter safe to ride for a whilerdquo and that he gave her a thumbs-up in response

The evidence rules in this jurisdiction are identical to the Federal Rules of Evidence

Analyze whether each of these items of evidence is relevant and admissible at trial

1 The authenticated copy of the mechanicrsquos text message

2 The womanrsquos testimony that she asked the mechanic ldquoIs my scooter safe to ride for a whilerdquo and

3 The womanrsquos testimony describing the mechanicrsquos thumbs-up

10

TRUSTS AND FUTURE INTERESTS QUESTION

Ten years ago Settlor validly created an inter vivos trust and named Bank as trustee The trust instrument provided that Settlor would receive all of the trust income during her lifetime The trust instrument further provided that

Upon Settlorrsquos death the trust income shall be paid in equal shares to Settlorrsquos surviving children for their lives Upon the death of the last surviving child the trust income shall be paid in equal shares to Settlorrsquos then-living grandchildren for their lives Upon the death of the survivor of Settlorrsquos children and grandchildren the trust corpus shall be distributed in equal shares to Settlorrsquos then-living great-grandchildren

The trust instrument expressly specified that the trust was revocable but it was silent regarding whether Settlor could amend the trust instrument

Immediately after creating the trust Settlor validly executed a will leaving her entire estate to Bank as trustee of her inter vivos trust to ldquohold in accordance with the terms of the trustrdquo

Five years ago Settlor signed an amendment to the inter vivos trust The amendment changed the disposition of the remainder interest specifying that all trust assets ldquoshall be paid upon Settlorrsquos death to Universityrdquo Settlorrsquos signature on this amendment was not witnessed

A state statute provides that any trust interest that violates the common law Rule Against Perpetuities ldquois nonetheless valid if the nonvested interest in the trust actually vests or fails to vest either (a) within 21 years of lives in being at the creation of the nonvested interest or (b) within 90 years of its creationrdquo

Recently Settlor died leaving a probate estate of $200000 She was survived by no children one granddaughter (who would be Settlorrsquos only heir) and no great-grandchildren The granddaughter has consulted your law firm and has raised four questions regarding this trust

1 Was Settlorrsquos amendment of the inter vivos trust valid Explain

2 Assuming that the trust amendment was valid do its provisions apply to Settlorrsquos probate assets Explain

3 Assuming that the trust amendment was valid how should trust assets be distributed Explain

4 Assuming that the trust amendment was invalid how should trust assets be distributed Explain

11

NEGOTIABLE INSTRUMENTS QUESTION

A chef entered into a contract with a repairman pursuant to which the repairman agreed to repair the chefrsquos commercial oven for $10000 The repairman agreed to accept as payment a negotiable promissory note for $10000 payable two months after its issuance

After the repairman worked on the oven the chef gave him a $10000 note as payment for the work As agreed the note was signed by the chef as maker was payable to the order of the repairman was payable in two months and fulfilled all criteria for negotiability

The next day the repairman sold the note to a buyer for $9500 To effectuate the sale the repairman wrote ldquono warrantiesrdquo on the back of the note signed his name immediately below that and handed the note to the buyer The buyer bought the note in good faith and without knowledge of any facts relating to the work that the repairman had performed for the chef

Later the buyer gave the note to his niece as a gift To effectuate the gift the buyer handed the note to the niece but did not indorse it

Shortly thereafter the chef discovered that the repair work had been done improperly and the oven still did not function correctly The chef tried repeatedly to get the repairman to return to correct the repair work but the repairman ignored all the chefrsquos calls

On the notersquos due date the niece contacted the chef and demanded that he pay the amount of the note to her The chef refused and told the niece that he would not pay the note because the repairman did not properly repair the oven

1 What are the niecersquos rights against the chef Explain

2 What are the niecersquos rights against the repairman Explain

3 What are the niecersquos rights against the buyer Explain

12

February 2013 MEE

ANALYSES

Real Property Contracts

Constitutional Law Secured Transactions

Federal Civil Procedure Agency

Evidence Trusts and Future Interests

Negotiable Instruments

REAL PROPERTY ANALYSIS (Real Property ID1a 4 amp 5)

ANALYSIS

Legal Problems

(1) Does the tenant have a defense to the landlordrsquos action for unpaid rent based on constructive eviction

(2) Does the tenant have a defense to the landlordrsquos action for unpaid rent based on the tenantrsquos surrender of the premises

(3) What if anything may the landlord recover from the tenant for the period after the tenant vacated the building

DISCUSSION

Summary

Under the common law the tenant does not have a defense to the landlordrsquos action for unpaid rent based on constructive eviction Constructive eviction is based on the tenant proving that (1) the landlord breached a duty to the tenant (2) the breach caused a loss by the tenant of the substantial use and enjoyment of the premises (3) the tenant gave the landlord adequate notice and opportunity to repair and (4) the tenant vacated the leased premises Here there was no constructive eviction because although the tenant vacated and gave the landlord adequate notice the landlord breached no express or implied duty to the tenant to repair the premises

The tenant does not have a defense based on the landlordrsquos acceptance of his surrender of the premises a landlordrsquos retention of keys does not constitute an acceptance of the tenantrsquos surrender unless the landlord so intended and here the landlordrsquos statements to the tenant at the time of the surrender of the keys do not evidence the intent to accept the tenantrsquos surrender

Under the common law a landlord has no duty to mitigate damages but also cannot sue for rents due in the future Under this approach the landlord can sue only for past-due rents Using this approach on November 1 the landlord could recover all the rent past due (ie rent for September and October) but could not recover for rents due in the future However some courts have authorized recovery for future rent minus the fair market rental value of the premises It is thus possible that the landlord could recover damages equal to the amount of rent due from September 1 to the end of the six-year lease term ($180000) minus the propertyrsquos fair-market rental value over that same period

Point One (45) The tenant was not constructively evicted because the landlord had no duty to repair the commercial premises that were the subject of the lease

The landlord and the tenant entered into a term-of-years lease because the lease specified both a beginning and an ending date HERBERT HOVENKAMP amp SHELDON F KURTZ THE LAW OF

PROPERTY 256 (5th ed 2001) Although a term-of-years lease normally cannot be terminated by the tenant prior to the end of the term a tenant may terminate a term-of-years lease if the tenant

15

Real Property Analysis

is constructively evicted See id at 286ndash88 Typically as here a claim of constructive eviction is made as a defense to a landlordrsquos action for damages or unpaid rent

In order to establish a constructive eviction the tenant must prove that the landlord breached a duty to the tenant such as a duty to repair and that the landlordrsquos breach caused a loss of the substantial use and enjoyment of the premises The tenant must also show that he gave the landlord notice adequate to permit the landlord to meet his duty to the tenant and that the tenant vacated the leased premises Id see also JOHN G SPRANKLING UNDERSTANDING

PROPERTY LAW sect 1704 (2d ed 2007) Under the common law there was no implied duty on the part of a landlord to repair

leased premises such a duty arose only if expressly set forth in the lease SPRANKLING supra sect 1702[B] Here the written lease contained no term requiring the landlord to repair the air-conditioning Even if the conversation created a lease term that the building had air-conditioning that itself should not create a duty for the landlord to repair it

Over the past several decades courts have generally implied a duty to repair in residential leases either as part of a revised constructive eviction doctrine or based on an implied warranty of habitability JOSEPH W SINGER PROPERTY 469ndash70 (3d ed 2010) This shift has been justified based on the economic disparity between the typical landlord and tenant as well as the fact that residential tenants generally lack both the authority to authorize repairs to common areas of a building and the incentive to make repairs that will ultimately benefit the landlord

However courts have been more reluctant to imply a duty to repair in commercial leases a context in which the tenant is often a valuable business and in a better position to assess and make repairs than is the landlord But see eg Davidow v Inwood North Professional Group 747 SW2d 373 (Tex 1988) When courts have implied a duty to repair in a commercial lease it is typically when the repair has been mandated by public authorities and involves work so substantial that it would not ordinarily fall within the tenantrsquos common law repair duty andor the value of the repair would primarily inure to the landlordrsquos reversionary interest See Brown v Green 884 P2d 55 (Cal 1994) Eugene L Grant et al The Tenant as Terminator Constructive Eviction in Modern Commercial Leases 2 THE COMMERCIAL PROPERTY LEASE ch 15 (ABA 1997) Some courts have also permitted constructive eviction claims by commercial tenants of office buildings based on repairs required in common areas of the building See id Echo Consulting Services Inc v North Conway Bank 669 A2d 227 (NH 1995)

Here the tenant is the owner of a valuable manufacturing operation and is the exclusive occupant of the building the repair has not been mandated by public authorities and the repair is not structural To the contrary the repair involves a feature of the building of unusual importance in the tenantrsquos manufacturing operation and the tenant is likely far more knowledgeable than the landlord about the air-conditioning specifications necessary for the manufacture of the tenantrsquos product

Based on these facts it is unlikely that a court will find that the tenant in this case was constructively evicted Although the tenant can show that he gave adequate notice to the landlord of the air-conditioning malfunction and vacated the premises the lease was commercial and it did not contain any promises or covenants by the landlord except a covenant of quiet enjoyment a covenant of quiet enjoyment does not entail any repair obligations

[NOTE An examineersquos conclusion is less important than his or her demonstrated awareness of the elements of constructive eviction and the need to imply a repair duty for such a defense to be viable here Although the implied warranty of habitability is not available to this tenant Texas Minnesota and Massachusetts imply a warranty of suitability in commercial leases in limited circumstances and an examinee might argue that this warranty should apply

16

Real Property Analysis

here If an examinee concludes that this warranty applies he or she should discuss the other requirements for constructive eviction

If the examinee wrongly concludes that the first element for a constructive eviction has been met the examinee will then have to discuss the remaining three elements in order to conclude that the tenant can claim constructive eviction The tenant would have a strong argument that the second elementmdashsubstantial interference with the use and enjoyment of the premisesmdashalso is met As indicated above the landlord was aware that a functioning air-conditioning system was vital to the tenantrsquos manufacturing operations The facts further indicate that the system had failed three times in the past few months The landlord may try to argue that the malfunctions did not substantially interfere with the tenantrsquos use of the premises because the malfunctions caused the temperature to climb above 81 degrees for only a short period of timemdash 3 hours 6 hours and 10 hours respectivelymdashon each occasion The tenant will argue however that the landlord was aware that the tenantrsquos manufacturing operations could tolerate temperatures above 81 degrees for no more than 6 hours The final malfunction exceeded that limit destroying $150000 worth of the tenantrsquos products

The tenant would also have a strong argument that the third element is met notice and opportunity to cure The tenant notified the landlord of the problem immediately upon the systemrsquos first malfunction and did so again when it malfunctioned a second time and then a third time The landlord might argue that there was insufficient time to cure the problem because the system corrected itself within a few hours on the first and second times Although the malfunction lasted more than 10 hours the third time the landlord might argue that the time period was insufficient to get a repair person on the premises A court would be likely to find this argument unpersuasive however because the landlord could have attempted to correct the problem after the first and second malfunctions

Assuming that the landlord was given sufficient notice and opportunity to cure a court would be likely to conclude that the tenant also satisfied the final element of vacating the premises within a reasonable time The landlord might argue that the tenant remained in the premises for almost four months after the air conditioning first failed which would suggest that the problem was not so severe as to have constructively evicted the tenant The tenant will argue however that he gave the landlord three months to cure the problem after the first two malfunctions threatened (but did not actually harm) his operations The tenant then moved out shortly after the final malfunction caused temperatures to exceed the tolerance levels of his manufacturing operations]

Point Two (10) The landlord did not accept the tenantrsquos surrender of the lease

When a tenant wrongfully moves from leased premises with the intent to terminate the lease the landlord may either accept the tenantrsquos surrender of the premises and terminate the lease or hold the tenant to the terms of the lease See HOVENKAMP amp KURTZ supra at 295ndash96 Here the tenantrsquos only basis for the claim that the landlord accepted his surrender is the landlordrsquos retention of the keys Many courts have considered whether a landlordrsquos retention of keys delivered by a tenant constitutes acceptance of surrender The weight of the case law holds that retention of the keys alone does not constitute acceptance of surrender without other evidence showing that the landlord intended to accept the surrender See generally 49 AM JUR 2d Landlord and Tenant sect 213

Here the landlordrsquos note saying ldquoI repeat the air-conditioning is not my problem You have leased the building and you should fix itrdquo strongly suggests that the landlord did not intend

17

Some courts have rejected the no-mitigation-of-damages rule based on efficiency concerns and societyrsquos interest in assuring that resources remain in the stream of commerce rather than lying vacant see id at 464ndash65 and allow landlords to sue tenants who have wrongfully terminated a lease for damages equal to the difference between the unpaid rent due under the lease and the propertyrsquos fair market rental value Other courts have abandoned the no-recovery-for-future-rent rule These courts responding to the fact that a tenant may well disappear or be judgment-proof by the time a lease term is concluded have allowed a landlord to collect damages equal to the value of rent over the entire lease term minus the propertyrsquos fair rental value when a tenant has wrongfully terminated a lease and unequivocally shown an intention not to return to the premises or pay future rent Under this approach a landlord receives approximately the same amount he would have received were there a duty to mitigate damages See Sagamore Corp v Willcutt 180 A 464 (Conn 1935)

Real Property Analysis

to accept the tenantrsquos surrender The tenant might argue that the landlordrsquos failure to make a similar statement when the keys were sent to her a second time and she retained them evidences a change of heart However it is likely that a court would find that the landlordrsquos retention of the keys represented a decision to safeguard the keys not to accept the tenantrsquos surrender

[NOTE An examinee should receive credit for arguing the other way with a well-reasoned argument]

Point Three (45) Under the common law the landlord had no duty to mitigate damages Additionally a landlord was not entitled to recover unpaid rents due in the future but was only entitled to recover rents in arrears at the time of the commencement of the suit Applying the common law here the landlord could recover $5000 the amount of rents due at the commencement of the suit ($2500 for September and the same for October) Today some courts allow the landlord under certain circumstances to sue the tenant for damages (not rent) equal to the difference if any between the unpaid promised rent for the balance of the term (here $175000) and the propertyrsquos fair rental value for the balance of the term

Under the common law because a lease was viewed as a conveyance instead of a contract a landlord had no duty to mitigate damages resulting from a tenantrsquos wrongful termination of a lease A landlord could thus recover the full value of rents that were due and unpaid at the time of the suit However under the common law a landlord could not sue a tenant for rents due in the future because there was always a possibility that the tenant might pay the rent when it was due See SINGER supra at 462 Thus using the common law approach on November 1 the landlord could only recover the full value of the two monthsrsquo rent actually due and unpaid ie $5000 for September and October

Here because the tenant returned the keys to the landlord and said ldquoI will not be returning to the building or making further rent paymentsrdquo the landlord could establish abandonment and an intention not to return It is thus possible that the landlord might recover damages in the amount of $5000 (for the months of September and October) plus the present value of $175000 minus the fair market rental value of the property over the remaining months of the lease

18

CONTRACTS ANALYSIS ____ (Contracts II IVE)

ANALYSIS

Legal Problems

(1) What was the legal effect of the sailorrsquos October 31 letter to the builder

(2)(a) What was the legal effect of the builderrsquos November 25 response to the sailorrsquos October 31 letter

(2)(b) What was the legal effect of the sailorrsquos refusal to take and pay for the boat on December 15

DISCUSSION

Summary

This is a sale of goods governed by the Uniform Commercial Code Because the sailor had reasonable grounds for insecurity about the builderrsquos ability to deliver the boat in a timely manner when the sailor learned about the strike on October 31 the sailor was legally justified in sending the letter to the builder seeking adequate assurance of the builderrsquos performance pursuant to the contract The builderrsquos failure to provide such assurance within a reasonable time operated as a repudiation of the contract However the builder was free to retract the repudiation before the sailor either cancelled the contract or materially changed position in reliance on the builderrsquos repudiation The builder retracted the repudiation when he informed the sailor that the workers were back and that the boat would be delivered by the date stipulated in the partiesrsquo contract Because the sailor had taken no action in response to the original repudiation he no longer had the right to cancel the contract with the builder The sailorrsquos subsequent statement that ldquoour contract is overrdquo may have constituted repudiation by the sailor In any event when the sailor failed to perform on December 15 that constituted breach

Point One (35) Because the sailor had reasonable grounds for insecurity with respect to the builderrsquos performance the sailorrsquos letter to the builder was a justified demand seeking assurance of the builderrsquos performance under the contract failure of the builder to provide such assurance within a reasonable time constituted repudiation of the contract

The sailor was legally justified in sending the letter to the builder on October 31 Contract parties are entitled to expect due performance of contractual obligations and are permitted to take steps to protect that expectation UCC sect 2-609 states that ldquo[w]hen reasonable grounds for insecurity arise with respect to the performance of either party the other may in writing demand adequate assurance of due performance rdquo Here the sailor learned on October 31 that the builderrsquos workers were on strike This gave the sailor reasonable grounds for insecurity about the builderrsquos ability to complete performance on time and thus gave the sailor the right to seek adequate assurance from the builder Because the sailorrsquos demand for assurance was justified the builder was required to provide assurance that was adequate under the circumstances within a reasonable time (not to exceed 30 days) or be held to have repudiated the contract UCC sect 2-609(4)

19

Contracts Analysis

Point Two(a) (30) The builder did not within a reasonable time provide the sailor adequate assurance of due performance this failure to provide assurance constituted a repudiation of the contract

Because the sailor with legal justification (see Point One) demanded from the builder assurance of due performance the builderrsquos failure to provide such assurance within a reasonable time was a repudiation of their contract See UCC sect 2-609(4) (ldquoAfter receipt of a justified demand[] failure to provide within a reasonable time not exceeding thirty days assurance of due performance is a repudiation of the contractrdquo) On October 31 the sailor requested that the builder provide adequate assurance regarding the completion of the boat by December 15 The builder did not respond to the sailorrsquos letter until November 25mdashnearly a month later Even if that response had been given in a reasonable time it nonetheless did not provide assurance of due performance It simply stated ldquoIrsquom sorry about the strike but it is really out of my hands I hope we settle it soon so that we can get back to workrdquo Therefore the builderrsquos November 25 response did not provide adequate assurance in response to the sailorrsquos justified request Thus the builder had repudiated the contract

Point Two(b) (35) Although the builder repudiated the contract with the sailor the builder probably retracted that repudiation on December 3 and the sailor was no longer entitled to cancel their contract Thus the sailorrsquos failure to perform the sailorrsquos obligations under the contract constituted a breach

The builderrsquos failure to provide adequate assurance of performance constituted a repudiation of their contract (see UCC sect 2-609(4)) but the builder was free to retract that repudiation until the sailor cancelled the contract or materially changed his position or indicated by communication or action that the sailor considered the repudiation to be final See UCC sect 2-611(1) (ldquoUntil the repudiating partyrsquos next performance is due he can retract his repudiation unless the aggrieved party has since the repudiation cancelled or materially changed his position or otherwise indicated that he considers the repudiation finalrdquo)

Here the facts state that before the builderrsquos December 3 telephone call to the sailor the sailor did nothing in response to the builderrsquos repudiation such as contracting with a third party for a boat The builderrsquos December 3 call informing the sailor that the boat would be timely delivered probably constituted a retraction of the repudiation because it clearly indicated to the sailor that the builder would be able to perform UCC sect 2-611(2) Thus after being so informed the sailor did not have the right to treat their contract as cancelled UCC sect 2-611(3) Accordingly the sailorrsquos failure to perform the sailorrsquos obligations under the contract by taking the boat and paying for it constituted a breach of the contract

20

CONSTITUTIONAL LAW ANALYSIS (Constitutional Law IVA F2b amp e)

ANALYSIS

Legal Problems

(1) Does AutoCorsquos operation of a ldquocompany townrdquo result in its actions counting as those of the state for purposes of constitutional analysis

(2) Does the expulsion of a schoolchild for failure to recite the Pledge of Allegiance violate the First Amendment as applied through the Fourteenth Amendment

(3) Does the arrest of a pamphleteer in connection with violation of an anti-littering rule where the littering is done by the recipients of leaflets distributed by the pamphleteer violate the First Amendment as applied through the Fourteenth Amendment

DISCUSSION

Summary

The First Amendment as applied through the Fourteenth Amendment applies only to state action It does not typically govern private actors However courts have found state action where the private actor has exercised a ldquopublic functionrdquo such as running a privately owned ldquocompany townrdquo as AutoCo has done here Thus First Amendment protections apply By requiring the son to participate in a mandatory Pledge of Allegiance ceremony AutoCo has compelled the expression of political belief in violation of the First Amendment as applied through the Fourteenth Amendment The fatherrsquos arrest in connection with breaching the anti-litter rule also violated the First Amendment as applied through the Fourteenth Amendment Although state actors can regulate the incidental effects of speech on the public streets on a content-neutral basis this power is limited and cannot extend to punishing a distributor of literature because of littering by third parties

Point One (30) AutoCorsquos operation of a company town (including a school) makes it a state actor under the public function strand of the state action doctrine

The individual rights protections of the Constitution apply only where there is ldquostate actionrdquomdash either direct action by the government or some action by a private party that is fairly attributable to the government As a general rule the actions of a private company like AutoCo or of a private school like the school operated by AutoCo would not constitute state action and the protections of the Constitution (in this case the First Amendment) would not apply

However there are situations in which the actions of a private actor are attributed to the state One such situation is when the private actor undertakes a public function There are not many bright-line rules in the Supreme Courtrsquos state action doctrine but one of them is this Where a private actor undertakes a ldquopublic functionrdquo the Constitution applies to those actions Where a corporation operates a privately owned ldquocompany townrdquo that provides essential services typically provided by a state actor the public function doctrine applies and the Constitution

21

Constitutional Law Analysis

binds agents of the town as if they were agents of the government See eg Marsh v Alabama 326 US 501 (1946) Here AutoCo does more than own the town it provides security services fire protection sanitation services and a school Thus the actions of AutoCo constitute state action and are governed by the Fourteenth Amendment

Point Two (35) The sonrsquos expulsion for failure to recite the Pledge of Allegiance violates the First Amendment as applied through the Fourteenth Amendment as a compelled expression of political belief

As explained in Point One the First Amendment applies to the school as a state actor Although children in public schools (and in schools subject to the First Amendment like

the Oakwood school) have some First Amendment rights Tinker v Des Moines Independent Community School District 393 US 503 506 (1969) schools have greater leeway to regulate the speech of students and teachers than the state would have outside the school context Hazelwood School Dist v Kuhlmeier 484 US 260 (1988) Morse v Frederick 551 US 393 (2007) However the Supreme Court has long held that public schools may not force their students to participate in a flag salute ceremony when it offends the political or religious beliefs of the students or their families West Virginia Board of Educ v Barnette 319 US 624 (1943) (invalidating a mandatory public school flag salute ceremony) see also Wooley v Maynard 430 US 705 (1977) (invalidating compelled expression of political belief on state-issued license plates)

In this case the school requires its students to participate in a flag salute and Pledge of Allegiance ceremony and punishes them when they refuse to participate Pursuant to this policy the school has expelled the son This expulsion violates the First Amendment ban on compelled expression

Point Three (35) Because the father was distributing leaflets in a traditional public forum his trespass arrest violated the First Amendment as applied through the Fourteenth Amendment

As explained in Point One AutoCo is treated as a state actor Thus Oakwoodrsquos commercial district is treated as government-owned property for purposes of the First Amendment Thus the leafleting here is subject to the First Amendment because it is an expressive activity Schneider v State of New Jersey Town of Irvington 308 US 147 (1939) When expression takes place on government-owned property government regulation of the expression is assessed under the public forum doctrine Public streets and sidewalks have long been held to be the classic example of a ldquotraditional public forumrdquo open to the public for expression Hague v CIO 307 US 496 515ndash16 (1939) Because the father was distributing leaflets while standing on a street corner in the commercial district his expressive activity occurred in a traditional public forum

When a state tries to regulate expressive activity in a traditional public forum it is prohibited from doing so based on the expressive activityrsquos content unless its regulation is narrowly tailored to achieve a compelling governmental interest (ldquostrict scrutinyrdquo) In this case however AutoCo is regulating the fatherrsquos expressive activity on the ostensibly neutral ground that his expressive activity has produced litter and made the street unsightly When a state tries to regulate expressive activity without regard to its content intermediate scrutiny applies Under intermediate scrutiny the true purpose of the regulation may not be the suppression of ideas (if so then strict scrutiny applies) the regulation must be narrowly tailored to achieve a significant

22

Constitutional Law Analysis

governmental interest and it must leave open ample alternative channels for expressive activity Ward v Rock Against Racism 491 US 781 791 (1989)

Here the application of the ordinance to the father will fail for two reasons First the Supreme Court has held that the governmentrsquos interest in keeping the streets clean is insufficient to ban leafleting in the public streets as the government power to regulate with incidental effects on public sidewalk speech is very limited See eg Schneider 308 US at 162 (leafletinglittering) Second the regulation (a blanket ban on distribution that results in littering) is not narrowly tailored to protect expression A narrowly tailored alternative would be prosecution only of people who litter Moreover the effect of the littering rule is likely to be a ban on all leafleting thus eliminating an entire class of means of expression This raises the possibility that there are not ldquoample alternative channels of communicationrdquo open to the father as required under the Courtrsquos standard of review for content-neutral regulation of speech

[NOTE Some examinees might argue that this is a ldquotime place and mannerrdquo restriction and that AutoCo might have greater latitude to regulate the public sidewalks under this theory This argument is incorrect for two reasons First the Supreme Court has held that the power to regulate speakers through littering laws is very limited for the reasons given and in the cases cited above But more generally a ldquotime place and mannerrdquo restriction involves the shifting of speech from one time and place to another or to another manner here there is no shifting but a direct punishment for expressive activity (albeit one couched in content-neutral terms) In addition some examinees might read the ordinance to be in effect a total ban on leafleting since most leafleting will produce some litter Those examinees might note that the Court has required total bans on an entire mode of expression to satisfy strict scrutiny and analyze the fatherrsquos prosecution here accordingly See United States v Grace 461 US 171 177 (1983) (invalidating ban on display of signs on public sidewalks surrounding US Supreme Court ldquo[a]dditional restrictions such as an absolute prohibition on a particular type of expression will be upheld only if narrowly drawn to accomplish a compelling governmental interestrdquo)]

23

SECURED TRANSACTIONS ANALYSIS (Secured Transactions IID E IVA B C)

ANALYSIS

Legal Problems

(1) Is a purchase-money security interest in consumer goods perfected even though there has been no filing of a financing statement

(2) Does a person who buys consumer goods for personal use take those goods free of a prior perfected purchase-money security interest in the goods

(3) Does a person who receives consumer goods as a gift take those goods subject to a prior perfected security interest in them

DISCUSSION

Summary

The retailerrsquos security interest in the bicycles was perfected even though no financing statement was filed because it was a purchase-money security interest in consumer goods A purchase-money security interest in consumer goods is automatically perfected upon attachment

The buyer is not subject to the retailerrsquos security interest in the bicycle that the buyer bought from the man Because the bicycle was consumer goods in the hands of the man and the retailer never filed a financing statement covering the bicycle the retailerrsquos security interest is not effective against someone like the buyer who bought the bicycle for value without knowledge of the retailerrsquos security interest and for personal use

On the other hand the retailerrsquos security interest continues in the bicycle given to the friend because the friend did not give value for the bicycle or buy it in the ordinary course of business

Point One (35) The retailerrsquos security interest in the bicycles attached on June 1 Because this interest was a purchase-money security interest in consumer goods it was automatically perfected when it attached

The retailerrsquos security interest in the bicycles attached on June 1 when the man bought the bicycles (acquiring rights in the collateral) signed a security agreement containing a description of the collateral and received value from the retailer (by being given credit with which to purchase the bicycles) UCC sect 9-203(a) amp (b)

Despite the retailerrsquos failure to file a financing statement its security interest was perfected Pursuant to UCC sect 9-309(1) a security interest is automatically perfected upon attachment if the goods are ldquoconsumer goodsrdquo and the security interest is a ldquopurchase-money security interestrdquo

In this case the bicycles sold by the retailer to the man were consumer goods at the time of sale The bicycles were ldquogoodsrdquo because they were ldquomovable when a security interest

24

Secured Transactions Analysis

attachesrdquo UCC sect 9-102(a)(44) They were also consumer goods because they were ldquobought for use primarily for personal family or household purposesrdquo UCC sect 9-102(a)(23) The retailerrsquos security interest in these consumer goods was also a ldquopurchase-money security interestrdquo A purchase-money security interest is an interest that secures a debt that was incurred in order to ldquoenable the debtor to acquire rights in or the use of the collateralrdquo UCC sect 9-103(a) (b)(1) Here the man incurred an obligation to the retailer to purchase the bicycles so the security interest he gave the retailer to secure that obligation was a purchase-money security interest

Because the retailerrsquos security interest was a purchase-money security interest in consumer goods it was automatically perfected on June 1 when the interest attached to the bicycles

Point Two (35) The buyer took the bicycle free of the retailerrsquos security interest because (i) the retailer did not file a financing statement covering the bicycle (ii) the bicycle was ldquoconsumer goodsrdquo and (iii) the buyer bought the bicycle for value without knowledge of the retailerrsquos security interest and for personal use

A security interest continues in collateral even after a sale or other disposition of that collateral unless the creditor authorized the disposition ldquofree of the security interestrdquo or another Article 9 exception applies UCC sectsect 9-201(a) and 9-315(a)(1)

However a buyer of goods like the buyer here can take free of a prior security interest in those goods under certain circumstances See UCC sectsect 9-317(b) (buyers who give value and receive delivery of goods without knowledge of an unperfected security interest in the goods) and 9-320(a) amp (b) (buyer in ordinary course of business buyer of consumer goods in a consumer-to-consumer transaction who gives value) In this case the retailerrsquos security interest was perfected when the buyer purchased the bicycle so UCC sect 9-317(b) does not protect the buyer The buyer also is not a protected ldquobuyer in ordinary course of businessrdquo because he did not purchase from a person who is in the business of selling bicycles See UCC sect 1-201(b)(9)

The buyer can however qualify for the protection of UCC sect 9-320(b) That section provides that a buyer of goods from a person who used them for personal family or household purposes takes free of a perfected security interest in the goods if (1) the buyer had no knowledge of the security interest (2) the buyer gave value for the goods (3) the buyer purchased the goods primarily for personal family or household purposes and (4) the purchase occurred before the filing of a financing statement covering the goods

The buyer met all of these criteria The man used the bicycle for personal purposes The buyer purchased the bicycle from the man and the buyer had no knowledge of the retailerrsquos security interest The buyer gave value ($400) for the bicycle and he bought it ldquoprimarily for personal family or household purposesrdquo as he planned to use it for recreation which is a personal rather than a business use Finally no financing statement had been filed Therefore under UCC sect 9-320(b) the buyer took free of the retailerrsquos security interest

Point Three (30) The retailerrsquos security interest continues in the bicycle that the man gave to the friend Thus the retailer can recover the bicycle from the friend because the friend did not give value for the bicycle or buy it in the ordinary course of business

25

Secured Transactions Analysis

As noted in Point Two the retailer did not authorize the man to dispose of the bicycle Consequently the retailerrsquos security interest continued in the bicycle even after the man transferred ownership of the bicycle to the friend See UCC sectsect 9-201(a) and 9-315(a)(1) The retailerrsquos security interest in the bicycle will be effective against the friend unless some other provision of Article 9 allows the friend to take the bicycle free of that security interest

Unfortunately for the friend there is no Article 9 provision that allows him to take free of the retailerrsquos interest The friendrsquos basic problem is that he is not a buyer of the bicyclemdashhe received the bicycle as a gift and did not give value for it Thus the friend is not protected by any of the applicable exceptions See UCC sectsect 9-317(b) (protecting buyers who give value for goods subject to an unperfected security interest) 9-320(a) (protecting buyers in ordinary course of business) and 9-320(b) (protecting buyers of consumer goods who give value)

In short the retailerrsquos security interest continues in the bicycle that the man gave to the friend The friend took the bicycle subject to that security interest

26

FEDERAL CIVIL PROCEDURE ANALYSIS (Federal Civil Procedure VIE)

ANALYSIS

Legal Problems

(1) Does a judgment in a prior action preclude a nonparty from suing the same defendant on a closely related claim when the nonparty and the original plaintiff are in a family relationship

(2) Does a judgment rendered in an earlier action preclude a nonparty from litigating an issue that was actually decided in the first suit

(3) May a nonparty to an earlier action invoke the judgment in that action to preclude a party to the prior action from relitigating an issue that the party had a full and fair opportunity to litigate in the earlier action

DISCUSSION

Summary

Pursuant to the doctrines of claim preclusion (res judicata) and issue preclusion (collateral estoppel) a judgment is binding on the parties thereto In the absence of privity nonparties to a prior suit cannot be bound by a judgment rendered in their absence Thus in the absence of privity a nonparty to the first suit is not precluded from presenting her claim in a second suit even if it is factually related to the claims and defenses presented in the first suit nor is she bound by determinations of issues made in the first suit A family relationship without more does not support a finding of privity For this reason Mother as a nonparty is not bound by the judgment in the Son-Driver action She may bring her separate claim for damage to her car and she is not precluded from litigating the question of whether she was negligent in the maintenance of her car

Driver on the other hand could be precluded from relitigating the issue of her negligence pursuant to the doctrine of non-mutual issue preclusion (also called non-mutual offensive collateral estoppel) which allows a nonparty to a prior action to invoke issue preclusion to prevent a party to that prior action from relitigating determinations of issues made therein However Mother may be prevented from invoking non-mutual collateral estoppel in this case because she could easily have joined her claim in the prior action but did not do so

[NOTE Federal common law governs the preclusive effect of a judgment rendered by a federal court sitting in diversity See Semtek Intrsquol Inc v Lockheed Martin Corp 531 US 497 508 (2001) But the Semtek Court concluded that federal common law in this context incorporates the preclusion law of the state in which the rendering federal court sits (unless the state law is incompatible with federal interests) id at 508ndash09 Thus State Arsquos preclusion law determines the preclusive effect of the judgment rendered in Sonrsquos suit against Driver The problem says that State A preclusion law is identical to federal preclusion law so the following analysis utilizes general principles of preclusion drawn from Supreme Court case law (announcing federal preclusion rules) and the Restatement (Second) of Judgments]

27

Federal Civil Procedure Analysis

Point One (35) Under the doctrine of claim preclusion the judgment rendered in the first action does not preclude Mother a nonparty from suing Driver for the damage to her car because the judgment binds only parties or those in privity with them and Mother and Son are not in privity

Driver may contend that the doctrine of claim preclusion (res judicata) precludes Mother from presenting a claim arising from the same nucleus of facts that was presented in the first action brought by Son According to the doctrine of claim preclusion ldquowhen a court of competent jurisdiction has entered a final judgment on the merits of a cause of action the parties to the suit and their privies are thereafter bound lsquonot only as to every matter which was offered and received to sustain or defeat the claim or demand but as to any other admissible matter which might have been offered for that purposersquordquo Commissioner of Internal Revenue v Sunnen 333 US 591 597 (1948) (citation omitted)

However the doctrine of claim preclusion does not apply to Mother on the facts of this problem First Mother was not a party to the earlier case ldquoIt is a principle of general application in Anglo-American jurisprudence that one is not bound by a judgment in personam in a litigation in which he is not designated as a party or to which he has not been made a party by service of processrdquo Taylor v Sturgell 553 US 880 884 (2008) (citing Hansberry v Lee 311 US 32 40 (1940)) see also RESTATEMENT (SECOND) OF JUDGMENTS sect 34(3) (1982) This rule reflects our ldquodeep-rooted historic tradition that everyone should have his own day in courtrdquo Martin v Wilks 490 US 755 762 (1989) (citation omitted) (superseded by statute on other grounds) Since Mother was not a party to the first suit she is not bound by the judgment unless an exception to the general rule applies

Mother might be bound by the prior judgment if she were considered to have been sufficiently in privity with Son that Son represented her interests in that action ldquoA person who is not a party to an action but who is represented by a party is bound by and entitled to the benefits of a judgment as though he were a partyrdquo RESTATEMENT (SECOND) OF JUDGMENTS sect 41(1) But there is no suggestion in the facts of the problem that Son who is an adult purported to represent Motherrsquos interests in the first suit ldquo[C]lose family relationships are not sufficient by themselves to establish privity with the original suitrsquos party or to bind a nonparty to that suit by the judgment entered therein rdquo Cuauhtli v Chase Home Finance LLC 308 Fed Appx 772 773 (5th Cir 2009) (citation omitted) accord 18A CHARLES ALAN WRIGHT ET AL FEDERAL

PRACTICE AND PROCEDURE sect 4459 (2d ed 2002) In Taylor v Sturgell supra the Supreme Court identified other special circumstances in

which nonparties may be bound by a prior judgmentmdashwhen a nonparty consents to be bound when a nonparty is in a pre-existing substantive legal relationship with a party (such as preceding and succeeding property owners) when a nonparty assumed control of the prior litigation when a party seeks to relitigate through a proxy or where a special statutory scheme seeks to foreclose successive litigation by nonparties See Taylor 553 US at 893ndash95 None of these circumstances exists here

Because Mother was not a party to the first suit and is not in privity with Son who is an adult the judgment in the first action does not preclude her from bringing her own claim against Driver

Point Two (35) Under the doctrine of issue preclusion the judgment rendered in the first action does not preclude Mother a nonparty from litigating the issue of her negligence in maintaining her carrsquos

28

Federal Civil Procedure Analysis

brake lights because the judgment binds only parties or those in privity with them and Mother and Son are not in privity

By its affirmative response to a special interrogatory the jury in the first action expressly concluded that ldquoMother negligently failed to ensure that the brake lights on her car were in proper working orderrdquo Driver may attempt to invoke the doctrine of issue preclusion to preclude Mother from relitigating this issue in the second action

[I]ssue preclusion arises in a second action on the basis of a prior decision when the same lsquoissuersquo is involved in both actions the issue was lsquoactually litigatedrsquo in the first action after a full and fair opportunity for litigation the issue was lsquoactually decidedrsquo in the first action by a disposition that is sufficiently lsquofinalrsquo lsquoon the meritsrsquo and lsquovalidrsquo it was necessary to decide the issue in disposing of the first action and the later litigation is between the same parties or involves nonparties that are subject to the binding effect or benefit of the first action Once these requirements are met issue preclusion is available not only to defend against a demand for relief but also as offensive support for a demand for relief Issue preclusion moreover is available whether or not the second action involves a new claim or cause of action

18 CHARLES ALAN WRIGHT ET AL FEDERAL PRACTICE AND PROCEDURE sect 4416 at 392ndash93 (2d ed) see also RESTATEMENT (SECOND) OF JUDGMENTS sect 27 (1982)

Here several of the elements necessary for issue preclusion are present The same issue is involved in both actionsmdashthe issue of Motherrsquos negligence in failing to maintain the brake lights on her car That issue was actually litigated in the first action and decided by the jury There is nothing to suggest anything less than a full and fair opportunity to litigate The judgment disposing of the issue was final

Nevertheless the judgment will not preclude Mother from relitigating the issue for two reasons First Mother was not a party to the first action and as explained above Mother and Son are not in privity Therefore she cannot be denied an opportunity to litigate the issue of her negligence Second it does not appear that the juryrsquos decision as to Motherrsquos negligence was necessary to the prior judgment against Driver Nothing suggests that the finding on Motherrsquos negligence had any bearing on the outcome of the first action

Point Three (30) Under the doctrine of non-mutual issue preclusion the judgment rendered in the first action might preclude Driver from relitigating the issue of her negligence However Driver has a strong argument that such a result would be inconsistent with the policy against offensive use of non-mutual estoppel when the non-party plaintiff easily could have joined as a plaintiff in the first action

Because Son already convinced the jury in the first action that ldquoDriver was negligent in the operation of her vehiclerdquo Mother may wish to invoke the doctrine of non-mutual issue preclusion to prevent Driver from relitigating the question of her negligence As noted above ldquoissue preclusion arises in a second action on the basis of a prior decision when the same lsquoissuersquo is involved in both actions the issue was lsquoactually litigatedrsquo in the first action after a full and fair opportunity for litigation the issue was lsquoactually decidedrsquo in the first action by a disposition that is sufficiently lsquofinalrsquo lsquoon the meritsrsquo and lsquovalidrsquo it was necessary to decide the issue in disposing of the first action rdquo 18 CHARLES ALAN WRIGHT ET AL FEDERAL PRACTICE AND

PROCEDURE sect 4416 at 392 (2d ed) see also RESTATEMENT (SECOND) OF JUDGMENTS sect 27

29

Federal Civil Procedure Analysis

Here these basic requirements for issue preclusion are met First the same issue is involved in both suits whether Driver was negligent in the operation of her car Second this issue was actually litigated and decided in the first action the jury answered a special interrogatory raising this very question There is nothing to suggest that Driver lacked a full and fair opportunity to litigate the issue Since a judgment was rendered against Driver for the injuries Son sustained as a result of Driverrsquos negligence resolution of the issue was necessary to dispose of the first action Driver was a party to the first action so she may be bound by the judgment

[NOTE Traditionally issue preclusion required mutualitymdashboth the party asserting issue preclusion and the party against whom issue preclusion was asserted were bound by the prior judgment Under the traditional mutuality rule Mother could not assert issue preclusion against Driver because Mother would not be bound by the judgment if Driver sought to rely on it See Point One There is no mutuality between Mother and Driver with respect to the prior judgment

This traditional mutuality requirement has been abandoned in most jurisdictions The Supreme Court rejected a strict mutuality requirement in Blonder-Tongue Laboratories Inc v University of Illinois Foundation 402 US 313 (1971) (non-mutual defensive collateral estoppel used by a defendant to preclude a plaintiff from relitigating a claim the plaintiff previously litigated) and Parklane Hosiery Co v Shore 439 US 322 (1979) (non-mutual offensive collateral estoppel used by a plaintiff to preclude a defendant from relitigating a claim the defendant previously litigated) In Parklane Hosiery the Court concluded (as a matter of federal preclusion law) that trial courts should have ldquobroad discretionrdquo to determine whether or not to permit a plaintiff to invoke non-mutual issue preclusion ldquoThe general rule should be that in cases where a plaintiff could easily have joined in the earlier action or where the application of offensive estoppel would be unfair to a defendant a trial judge should not allow the use of offensive collateral estoppelrdquo Id at 331

The Parklane Hosiery decision identified a number of circumstances that might make it unfair to allow a plaintiff to invoke non-mutual issue preclusion (non-mutual offensive collateral estoppel in the traditional terminology) against a defendant In particular the Parklane Hosiery court suggested that issue preclusion may not be appropriate if the plaintiff in the second action ldquocould easily have joined in the earlier actionrdquo Id Prohibiting plaintiffs from using non-mutual estoppel under such circumstances would promote judicial efficiency by encouraging plaintiffs to join the prior action It would also discourage plaintiffs from staying out of prior litigation in order to secure in effect two bites at the apple using the prior litigation offensively if the defendant loses and forcing the defendant to litigate a second time if the defendant wins the prior action

An exceptional exam answer might therefore argue that non-mutual issue preclusion should be denied on these facts Son and Mother both reside in State A since they are related they know each other well and Son was driving Motherrsquos car when the accident occurred They could have sued together and Rule 20 of the Federal Rules of Civil Procedure would have authorized joinder of their claims because those claims arose from the same transaction or occurrence and raised a common question of law or fact FED R CIV P 20(a) The facts do not suggest that Mother had any reason not to join Sonrsquos suit other than a desire to see how Sonrsquos action concluded before bringing her own claim Cf Nations v Sun Oil Co (Del) 695 F2d 933 938 (5th Cir 1983) (concluding that plaintiff ldquowas entitled to await the development of his injuries and their predictable consequencesrdquo) Because it appears that Mother may be a ldquowait-and-seerdquo plaintiff who could easily have joined the original action a trial court might disallow as a matter of discretion her use of non-mutual issue preclusion]

30

AGENCY ANALYSIS __________ (Agency I II)

ANALYSIS

Legal Problems

(1) Is the principal or the agent or both liable on contracts with a third party when the principal is an ldquoundisclosed principalrdquo

(2) Is the principal or the agent or both liable on contracts with a third party when the principal is ldquopartially disclosedrdquo or an ldquounidentified principalrdquo

(3) Is the principal or the agent or both liable on contracts with a third party for the purchase of goods when the agent exceeded his authority but the principal nonetheless accepts the goods

DISCUSSION

Summary

The agent but not the owner is liable to the basket manufacturer because the owner is an undisclosed principal and the agent acted without actual or apparent authority Both the agent and the owner however are liable on the burner contract because the owner is an unidentified principal and the agent had apparent authority to enter into that contract With respect to the solar cells contract whether the owner is liable depends upon whether a court would follow the Second or Third Restatement of Agency which take different positions on the effect of the ratification of a contract by an undisclosed principal Under either the agent would also be liable on the contract as he was a party to the contract

[NOTE The contracts that are the subject of this question are contracts for the sale of goods and therefore are governed by Article 2 of the Uniform Commercial Code Article 2 however does not contain agency rules Accordingly common law concepts of agency are applicable UCC sect 1-103(b)]

Point One (35) The agent but not the owner is liable to the basket manufacturer The agent had no actual authority to enter into the contract to buy aluminum baskets and because the owner was an undisclosed principal the manufacturer had no reason to believe that the agent had apparent authority Furthermore the manufacturer had no reason to believe that the agent was not contracting for his own benefit

An agent acting on behalf of a principal can bind the principal to contracts if the agent has either actual or apparent authority An agent has actual authority when contracting on behalf of his principal if he ldquoreasonably believes in accordance with the principalrsquos manifestations to the agent that the principal wishes the agent so to actrdquo RESTATEMENT (THIRD) OF AGENCY sect 201 (2006) Here the agent was told to buy only wicker baskets not aluminum baskets Thus when he contracted with the basket manufacturer to buy aluminum baskets he had no actual authority to do so

31

Agency Analysis

An agent acts with apparent authority ldquowhen a third party [with whom the agent acts] reasonably believes the actor has authority to act on behalf of the principal and that belief is traceable to the principalrsquos manifestationsrdquo Id sect 203 Here the owner notified basket manufacturers that she or her agent might contact them to purchase baskets but that notification did not specifically name the agent or any other person as the ownerrsquos agent Furthermore the basket manufacturer had no prior dealings with the agent or the owner or any reason to think that the agent was acting for the benefit of anyone but himself Thus there is no basis to conclude that the basket manufacturer thought that the agent had apparent authority to act for the owner

Generally when an agent acts on behalf of an undisclosed principal and the agent lacks authority to enter into the contract the agent is liable on the contract as a party to the contract but the principal is not liable This rule is consistent with the third partyrsquos expectations ldquoThe third party expected the agent to be a party to the contract because the agent presented the deal as if he were acting for himself Moreover if the third party is unaware of the principalrsquos existence the third party must be relying on the agentrsquos solvency and reliability when entering into the contractrdquo See ROBERT W HAMILTON JONATHAN R MACEY amp DOUGLAS K MOLL CORPORATIONS INCLUDING PARTNERSHIPS AND LIMITED LIABILITY COMPANIES 34 (11th ed 2010) See also RESTATEMENT (THIRD) OF AGENCY sect 603 cmt c Furthermore because the third party has no idea that the agent is acting or is seemingly acting on behalf of another there is no reason to believe that the third party would be expecting an undisclosed principal to be liable on the contract Id

Point Two (35) Because the owner is an unidentified (as opposed to undisclosed) principal both she and the agent (as a party to the contract) probably are liable on the contract with the burner manufacturer

When the agent contracted with the burner manufacturer he did not have actual authority to do so as the owner had expressly restricted the agentrsquos authority to purchase only burners with ldquowhisper technologyrdquo See Point One However the agent may have had apparent authority to buy burners without whisper technology

An agent acts with apparent authority ldquowhen a third party [with whom the agent acts] reasonably believes the actor has authority to act on behalf of the principal and that belief is traceable to the principalrsquos manifestationsrdquo RESTATEMENT (THIRD) OF AGENCY sect 203 (2006) The owner indicated that an agent might contact the burner manufacturer The notice contained no restriction regarding the type of burners that the agent was authorized to purchase The facts indicate that burner manufacturers regularly receive such notices

Although the agent told the burner manufacturer that he represented a well-known hot-air balloon operator he did not disclose the ownerrsquos name Thus the owner was a partially disclosed or unidentified principal See RESTATEMENT (SECOND) OF AGENCY sect 4(2) (1958) (using term ldquopartially disclosed principalrdquo) RESTATEMENT (THIRD) OF AGENCY sect 104(2)(c) (2006) (using term ldquounidentified principalrdquo) An agent for a partially disclosed principal may have apparent authority RESTATEMENT (SECOND) OF AGENCY sect 159 cmt e (1958) Based upon (1) the notice sent by the owner (2) the agentrsquos revelation that he was acting as an agent and (3) the fact that burner manufacturers regularly receive such notices and sell to agents the manufacturer may argue that it reasonably and actually believed that the agent was authorized to purchase burners without whisper technology The manufacturer may also argue that because the agent revealed that he was an agent his listing of the ownerrsquos address as the delivery address connects the agent to the notice given by the owner Arguably this distinguishes the burner contract from the basket

32

Agency Analysis

contract Here there is a strong case to support the conclusion that the agent had apparent authority if he did then the owner is liable to the burner manufacturer

The agent also is liable as a party to the contract because he did not fully disclose his agency relationship Although he told the burner manufacturer that he represented a well-known hot-air balloon operator he did not disclose the ownerrsquos name Generally even an authorized agent of a partially disclosed or unidentified principal is liable as a party to a contract with a third person RESTATEMENT (SECOND) OF AGENCY sect 321 (1958) (ldquounless otherwise agreedrdquo) RESTATEMENT (THIRD) OF AGENCY sect 602(2) (2006) (ldquounless the agent and the third party agree otherwiserdquo)

Point Three (30) Under the Second Restatement of Agency the owner is not liable on the contract for solar cells because the agent did not have actual or apparent authority and the owner as an undisclosed principal cannot ratify the contract Under the Third Restatement the owner could be liable as she ratified the contract Under either Restatement the agent is liable as a party to the contract

The owner is not liable to the solar cell manufacturer for breach of the contract for the solar cells because the agent had no actual or apparent authority to purchase solar cells on the ownerrsquos behalf and the owner under the Second Restatement of Agency did not ratify the contract with knowledge of the material facts Thus she is not liable as a ratifier of the contract

The facts state that the agent had authority to purchase only propane fuel tanks In addition he had no apparent authority to purchase solar cells The owner made no manifestations to the solar cell manufacturer that would lead a reasonable person in the manufacturerrsquos position to believe that the agent had the authority to bind the owner to a contract to purchase solar cells In fact the agent made no manifestations at all to the solar cell manufacturer Unlike with the basket manufacturer and the burner manufacturer the owner did not notify the manufacturer of solar cells that an agent might contact it to purchase solar cells In addition the solar cells were delivered to the agent and not to the ownerrsquos address In sum the manufacturer was unaware of any relationship between the owner and the agent As to the solar cell manufacturer the owner is an undisclosed principal There can be no apparent authority in the case of an undisclosed principal because there are no manifestations from the principal to the third person See RESTATEMENT (SECOND) OF AGENCY sect 8 cmt a (1958) (ldquothere can be no apparent authority created by an undisclosed principalrdquo) RESTATEMENT (THIRD) OF AGENCY sect 203 cmt f (2006) (ldquoapparent authority is not present when a third party believes that an interaction is with an actor who is a principalrdquo)

The owner also did not ratify the contract Although the owner used the solar cells generally a principal cannot ratify an unauthorized transaction with a third person ldquounless the one acting purported to be acting for the ratifierrdquo RESTATEMENT (SECOND) OF AGENCY sect 85(1) (1958)

The result differs under the Third Restatement which expressly rejects the Second Restatement on this issue The Restatement (Third) of Agency sect 403 (2006) states ldquoA person may ratify an act if the actor acted or purported to act as an agent on the personrsquos behalfrdquo According to comment b ldquoan undisclosed principal may ratify an agentrsquos unauthorized actrdquo Under the Restatement (Third) of Agency rule the owner probably ratified the transaction The agent clearly acted on the ownerrsquos behalf and in addition the ownerrsquos conduct in using the solar cells ldquojustifies a reasonable assumption that [she] is manifesting assent that the act shall affect [her] legal relationsrdquo See id sect 401(2)

33

Agency Analysis

The agent also is liable to the solar cell manufacturer for breach of the contract for the solar cells because he is a party to the contract The facts indicate that the agent never told the solar cell manufacturer that he represented the owner or any other principal Consequently even if the agent were authorized (which as discussed above he is not) he would be liable as a party to the contract See RESTATEMENT (SECOND) OF AGENCY sect 322 (1958) RESTATEMENT (THIRD) OF AGENCY sect 603(2) (2006) Here he has no authority or apparent authority and is liable as a party to the contract

The agent would also be liable under the Third Restatement Under Restatement (Third) of Agency sect 402(1) (2006) ratification generally relates back and the transaction is treated as if it were authorized at the time of the transaction However this does not relieve the agent of an undisclosed principal who ratifies an unauthorized transaction of liability under the ratified contract See id sect 603(2) (authorized agent for undisclosed principal is a party to the contract) and sect 403 cmt b (ldquoAn undisclosed principalrsquos ratification does not eliminate the agentrsquos liability to the third party on the transaction rdquo)

[NOTE An examinee may discuss the concept of inherent agency power This concept is recognized by the Restatement (Second) of Agency sect 8 A (1958) but the concept is not used in the Restatement (Third) of Agency (2006) Here there are no facts to support that the agent had inherent authority

As to contracts with agents for partially disclosed principals (eg the contract for the burners) the basic question is whether the acts done ldquousually accompany or are incidental to transactions which the agent is authorized to conductrdquo RESTATEMENT (SECOND) OF AGENCY

sect 161 (1958) If so the principal is bound if the other party ldquoreasonably believes that the agent is authorized to do them and has no notice that he is not so authorizedrdquo Id The purchase of burners without whisper technology was not authorized nor was it incidental to an authorized transaction Therefore there should not be inherent agency power

As to contracts on behalf of undisclosed principals (eg the other two contracts) the basic question is whether the acts done are usual or necessary in the transactions the agent is authorized to transact RESTATEMENT (SECOND) OF AGENCY sect 194 (1958) The other two contracts seem fundamentally different from the authorized transactions Therefore there should not be inherent agency power

Only minimal credit should be given for discussion of inherent agency power]

34

EVIDENCE ANALYSIS _____ (Evidence IIA VA B E F J K)

ANALYSIS

Legal Problems

(1) Is the authenticated copy of the mechanicrsquos text message relevant and admissible

(2) Is the womanrsquos question ldquoIs my scooter safe to drive for a whilerdquo relevant and admissible

(3) Is the womanrsquos testimony describing the mechanicrsquos thumbs-up relevant and admissible

DISCUSSION

Summary

The mechanicrsquos text message to the woman is relevant to whether (1) the woman lost control of the scooter due to its defective brakes (2) the woman knew that the brakes needed repair and (3) it was negligent for the woman to drive the scooter knowing that its brakes needed repair

The mechanicrsquos text message is hearsay if it is offered by the pedestrian to prove that the scooterrsquos brakes needed repair However it fits the hearsay exception for present sense impressions and probably also fits the exception for business records The mechanicrsquos text message is not hearsay if it is instead offered by the pedestrian to prove the womanrsquos state of mind (ie that she had notice that her brakes needed repair)

The womanrsquos question to the mechanic and his response are also relevant to whether the brakes caused the accident and whether the woman was negligent The question is not hearsay because the woman did not make an assertion

The mechanicrsquos thumbs-up response is nonverbal conduct intended by the mechanic as an assertion and is therefore an out-of-court statement If the woman offers the mechanicrsquos statement to prove that the scooter was actually safe to ride the womanrsquos testimony about the statement is hearsay

However the mechanicrsquos statement is not hearsay if it is offered by the woman to prove her state of mind Therefore the womanrsquos question and the mechanicrsquos response are admissible to prove the womanrsquos state of mind

Point One(a) (20) The mechanicrsquos text message to the woman should be admitted because it is relevant

Evidence is relevant if it has ldquoany tendency to make a fact more or less probable than it would be without the evidencerdquo FED R EVID 401 ldquoRelevant evidence is admissiblerdquo unless it is inadmissible pursuant to some other rule FED R EVID 402

The mechanicrsquos text message to the woman ldquoWhen you pick up your scooter you need to schedule a follow-up brake repair Wersquoll order the partsrdquo is relevant for two reasons First this evidence has some tendency to make it more probable that the brakes malfunctioned and

35

Evidence Analysis

caused the accident Second it has some tendency to make it more probable that the woman was negligent in riding her scooter after being told by the mechanic that it required further repair

Point One(b) (30) The mechanicrsquos text message fits either the hearsay exception for present sense impressions or the exception for business records or it is admissible non-hearsay

The mechanicrsquos text message is a statement under Rule 801(a) because it is ldquoa written assertionrdquo FED R EVID 801(a) The text message is hearsay if the pedestrian offers it to prove the ldquotruth of the matter asserted in the statementrdquo (ie that the scooterrsquos brakes required repair) which resulted in the woman losing control of the scooter and causing the accident FED R EVID 801(c)

However the mechanicrsquos text message fits the hearsay exception for ldquopresent sense impressionsrdquo under Rule 803(1) because it is ldquo[a] statement describing or explaining an event or condition made while or immediately after the declarant perceived itrdquo FED R EVID 803(1) Here the mechanicrsquos text message described the condition of the scooter immediately after he perceived it during the maintenance service

The mechanic is a person with knowledge of the condition of the scooter so if text messages regarding repairs were made and kept by the mechanic in the ordinary course of business this text message also fits the business records exception Under Rule 803(6) a business record is a record of an act ldquomade at or near the time by someone with knowledgerdquo and ldquothe record was kept in the course of a regularly conducted activity of a businessrdquo and ldquomaking the record was a regular practice of that activityrdquo FED R EVID 803(6)

However the text message is not hearsay if it is instead offered to prove that the woman was negligent because she rode her scooter after the mechanic told her it required repair If offered for this purpose it would not be offered for the truth of the matter asserted in the statement but to show the womanrsquos belief about the condition of the scooter (her state of mind)

Point Two (10) The womanrsquos question to the mechanic should be admitted because it is not hearsay

The womanrsquos question to the mechanic is relevant because along with the mechanicrsquos thumbs-up response (see Point Three) it has some tendency to make it more probable that the woman was not negligent andor that the scooter brakes did not malfunction and cause the accident FED R EVID 401 The womanrsquos question does not raise hearsay concerns because it is not an assertion

Hearsay is defined under Rule 801(a) as ldquoan oral assertion written assertion or nonverbal conductrdquo Although ldquoassertionrdquo is not further defined ldquoa favorite [definition] of writers in the [evidence] field for at least a century and a half [is that] the word simply means to say that something is so eg that an event happened or a condition existedrdquo 2 MCCORMICK ON

EVIDENCE sect 246 (6th ed 2006) Under this definition the womanrsquos question is not hearsay because it is not an assertion

Point Three(a) (20) The mechanicrsquos thumbs-up to the woman is a nonverbal assertion that is relevant and the womanrsquos testimony about that response is admissible

36

Evidence Analysis

Hearsay is defined under Rule 801(c) as a ldquostatementrdquo that is ldquoa personrsquos oral assertion written assertion or nonverbal conduct if the person intended it as an assertionrdquo FED R EVID 801(a) Here when the mechanic responded to the womanrsquos question (ldquoIs my scooter safe to ride for a whilerdquo) with a thumbs-up gesture the facts suggest that he intended his nonverbal conduct as an assertion that in his opinion the scooter was safe to ride

The mechanicrsquos assertion is relevant and admissible to prove that the woman was not negligent because the evidence makes it more probable that at the time of the accident she believed that the scooter was safe to ride despite the fact that the brakes required repair FED R EVID 401 Admission of the womanrsquos description of the mechanicrsquos thumbs-up for this purpose does not raise hearsay concerns because the evidence would not be offered for the truth of the matter asserted but to show the womanrsquos belief about the condition of the scooter (her state of mind)

Point Three(b) (20) The mechanicrsquos thumbs-up is relevant to determine whether the scooterrsquos brakes malfunctioned causing the accident but if offered for this purpose it is also hearsay

The mechanicrsquos nonverbal assertion is relevant to the determination of whether the scooterrsquos brakes malfunctioned causing the accident However if offered to prove the ldquotruth of the matter asserted in the statementrdquo (ie that the scooter was safe to ride for a while) it is hearsay that does not fit any hearsay exception

37

TRUSTS AND FUTURE INTERESTS ANALYSIS ____________________ (Trusts and Future Interests IC1 amp 4 G IIF)

ANALYSIS

Legal Problems

(1)(a) Was the revocable trust amendable

(1)(b) If the trust was amendable must the amendment have been executed in accordance with the state Statute of Wills in order to be valid

(2) If the trust amendment was valid does the amendment apply to the probate estate assets passing to the trust pursuant to Settlorrsquos will

(3) If the trust amendment was valid should the trust property be distributed to University

(4) If the trust amendment was not valid should the trust property be distributed to Settlorrsquos grandchild (her only heir) or held in further trust in accordance with the terms of the original trust instrument

DISCUSSION

Summary

A revocable trust is amendable even if the trust instrument does not expressly grant to the trust settlor a power to amend Both inter vivos trusts and amendments thereto are valid even though not executed in accordance with the requirements applicable to wills

Under the Uniform Testamentary Additions to Trusts Act a revocable trust may be amended at any time prior to the settlorrsquos death and the amendment applies to the disposition of assets conveyed to the trust pursuant to a will even if the will was executed prior to the date of the amendment

At Settlorrsquos death trust assets including probate assets passing to the trust under Settlorrsquos will would go to University if as is the case here the trust amendment was valid If the amendment was invalid the trust assets would continue to be held in further trust because there is no violation of the common law Rule Against Perpetuities

Point One(a) (30) Settlor retained the right to amend the inter vivos trust despite her failure to expressly reserve this power

At issue here is whether a retained power of revocation includes the power to amend sometimes referred to as the power to modify The Restatement (Second) of Trusts sect 331 cmt g provides that if a settlor has a power to revoke that retained power ordinarily includes a power to modify (amend) as well Comment g also notes that the power to amend includes both a power to withdraw trust assets and a power to ldquomodify the terms of the trustrdquo The Uniform Trust Code which provides that a power to revoke includes the power to amend is consistent with this view

38

Trusts and Future Interests Analysis

UNIF TRUST CODE sect 602 accord RESTATEMENT (THIRD) OF TRUSTS sect 63 cmt The theory is that even though a power to amend was not expressly retained by a settlor the goal of amendment assuming the power was not included in the power to revoke could easily be achieved by first revoking the trust and then creating a new trust with the same terms contemplated by the amendment To require this would put form over substance

Thus by expressly retaining the power to revoke the trust Settlor retained a power to amend the inter vivos trust despite her failure to expressly reserve this power

[NOTE Under the common law a trust is irrevocable unless the settlor expressly retains a power to revoke the trust Conversely under the Uniform Trust Code a trust is revocable unless the terms of the trust expressly provide otherwise See UNIF TRUST CODE sect 602 The Trust Codersquos position on revocation follows the minority view in the United States and is inconsistent with prior Restatements of Trusts (see Restatement (Second) of Trusts sect 330) Here the trust is revocable because Settlor expressly retained a power of revocation

The Uniform Trust Code has been adopted in 24 jurisdictions Alabama Arizona Arkansas District of Columbia Florida Kansas Maine Michigan Missouri Nebraska New Hampshire New Mexico North Carolina North Dakota Ohio Oregon Pennsylvania South Carolina Tennessee Utah Vermont Virginia West Virginia and Wyoming]

Point One(b) (10) Settlorrsquos amendment of the trust was valid despite her failure to have her signature to the trust amendment witnessed

Neither the common law nor state statutes require a trust instrument or an amendment to a trust instrument to be executed in accordance with the formalities prescribed for execution of a will Indeed an inter vivos trust that does not involve real estate can be created orally Under the Uniform Trust Code the only requirements for creating a valid inter vivos trust are intent the specification of beneficiaries and the designation of a trustee See UNIF TRUST CODE sect 402 accord RESTATEMENT (THIRD) OF TRUSTS sect 13

Here the amendment meets the requirements of both the Uniform Trust Code and the common law Thus the fact that Settlorrsquos signature was not witnessed when she signed the amendment to the trust does not make the amendment invalid

Point Two (20) Under the Uniform Testamentary Additions to Trusts Act a revocable trust may be amended at any time prior to the settlorrsquos death and the amendment applies to probate assets poured into the trust at the settlorrsquos death pursuant to the settlorrsquos will even when the will was executed prior to the date of the amendment

Historically property owned by an individual at her death passed to the individualrsquos heirs or to beneficiaries designated in a will executed with the formalities (writing signing witnessing) prescribed by state law However when a will devises property to the trustee of an inter vivos trust then the provisions of the trustmdashwhich may not have been executed in accordance with the formalities required for willsmdasheffectively determine who will receive the property Because of this possibility some early cases held that if an inter vivos trust was not executed with the same formalities required for a valid will then the trust was ineffective to dispose of probate assets poured into the trust at the settlorrsquos death pursuant to the settlorrsquos will

This line of cases has been overturned by the Uniform Testamentary Additions to Trusts Act (the Act) now Uniform Probate Code sect 2-511 Under the Act adopted in almost all

39

Trusts and Future Interest Analysis

jurisdictions a testamentary bequest to the trustee of an inter vivos trust established by the testator during his or her lifetime is valid if the trust is in writing it is identified in the testatorrsquos will and the trust instrument was executed before concurrently with or after the execution of the will Id The Act further specifies that such a bequest is valid even if the trust is amendable or revocable and that a later amendment applies to assets passing to the trust by a previously executed will

Thus because the trust amendment is valid its terms apply to assets received by Bank from Settlorrsquos estate

Point Three (10) If the trust amendment was valid then the trust assets including assets passing to the trust under Settlorrsquos will should go to University

Under the trust amendment all trust assets (including the assets of Settlorrsquos probate estate poured into the trust) pass to University The facts provide no basis for failing to comply with Settlorrsquos stated intentions

Point Four (30) If the trust amendment was invalid trust assets including assets received pursuant to Settlorrsquos will should be held in accordance with the terms of the original trust instrument because those terms do not violate the Rule Against Perpetuities

Under the dispositive terms of the original trust instrument Settlor created successive income interests in her surviving children and grandchildren with a remainder interest in her great-grandchildren Because the trust was revocable the period during which the common law Rule Against Perpetuities requires that interests vest (ie 21 years plus lives in being) began to run from the date Settlor no longer had a power of revocation (here her death) not the date on which the trust was created See JESSE DUKEMINIER STANLEY J JOHANSON JAMES LINDGREN amp ROBERT SITKOFF WILLS TRUSTS AND ESTATES 678 (7th ed 2005)

Under the common law Rule Against Perpetuities Settlorrsquos trust is thus valid At the time of Settlorrsquos death she was survived by no children one granddaughter and no great-grandchildren Because Settlor cannot have more children after her death the only income beneficiary of the trust is Settlorrsquos surviving granddaughter This granddaughter is the only person who can produce great-grandchildren of Settlor thus all great-grandchildren must of necessity be born during the lifetime of Settlorrsquos only surviving granddaughter who is a life in being The granddaughterrsquos interest vested at Settlorrsquos death and the great-grandchildrenrsquos interest will vest at the death of the granddaughter There is no need to wait the additional 21 years permitted under the Rule Thus under the common law and the statute given in the facts the nonvested interest in the great-grandchildren is valid

[NOTE Both modern wait-and-see statutes and the Uniform Statutory Rule Against Perpetuities upon which the statute in the facts is modeled provide that before using either reform to validate an otherwise invalid nonvested interest one should first determine if the nonvested interest violates the common law Rule If it does not then there is no need to reform This proposition which is applicable in all MEE user jurisdictions that have not simply abrogated the rule is tested by this problem]

40

NEGOTIABLE INSTRUMENTS ANALYSIS (Negotiable Instruments III IV V)

ANALYSIS

Legal Problems

(1)(a) What rights does a person in possession of a note that has been indorsed in blank by the payee have against the maker of the note

(1)(b) Which defenses may the maker of a note raise against a person entitled to enforce it who is not a holder in due course but is a transferee from a holder in due course

(2) What rights does a person entitled to enforce a note have against an indorser who transferred it for consideration with no warranties

(3) What rights does a person entitled to enforce a note have against a previous holder who transferred it as a gift without indorsing it

DISCUSSION

Summary

The niece is a holder of the note and is thus a person entitled to enforce it The chef the issuer of the note is obligated to pay it to the niece as the person entitled to enforce it The niece is not subject to any defense or claim of the chef relating to the improper repair of the oven because the niece has the rights of a holder in due course When the buyer bought the note from the repairman the buyer became a holder in due course of the note and thus took it free of any personal defenses the chef had against the repairman Even though the niece is not herself a holder in due course of the note the niece succeeded to the buyerrsquos rights as holder in due course and thus took free of the chefrsquos personal defenses

Because the chef refused to pay the note the niece can recover from the repairman on the repairmanrsquos obligation as indorser The niece cannot recover on the note against the buyer however because the buyer did not indorse the note (and thus incurred no indorserrsquos obligation) and the buyer did not receive any consideration for transfer of the note to the niece (and therefore made no transfer warranty)

[NOTE Although Article 9 of the Uniform Commercial Code governs the sale of promissory notes (a point that might be correctly noted by examinees) that Article does not determine the answer to any of the questions posed]

Point One(a) (20) The niece is the holder of the note and thus may enforce it against the chef who is the issuer of the note

The chef is the maker of the note and thus its issuer See UCC sectsect 3-103 3-105 The issuer of a note is obligated to pay it in accordance with its terms to a ldquoperson entitled to enforcerdquo it UCC sect 3-412 The niece is a ldquoperson entitled to enforcerdquo the note This is because the niece is the holder of the note and a holder of a note is a person entitled to enforce it UCC sect 3-301 The niece is the holder of the note because (i) the repairmanrsquos signature on the back of the note not

41

Negotiable Instruments Analysis

accompanied by words indicating a person to whom the note was made payable was a ldquoblank indorsementrdquo which had the effect of making the note a bearer instrument (ii) anyone in possession of a bearer instrument is a holder of it and (iii) the niece is in possession of the note See UCC sectsect 1-201(b)(21)(A) 3-204 and 3-205 Accordingly the chef has an obligation to the niece to pay the note in accordance with its terms and the niece may enforce that obligation

Point One(b) (40) The niece is not a holder in due course of the note but because she is a transferee from the buyer who was a holder in due course she has the same enforcement rights as the buyer Because the buyer as a holder in due course would have been able to enforce the note against the chef without being subject to defenses or claims arising from the improper repair the niece has the same rights and will not be subject to the chefrsquos defenses or claims about the repair

As noted in Point One(a) the chef has an obligation to the niece to pay the note in accordance with its terms However except against a person with the rights of a holder in due course the chef can raise any defenses or claims in recoupment that he would have if the claim on the note were an ordinary contract claim UCC sect 3-305 Thus except against a holder in due course the chef would be able to raise the improper repair as a defense or a claim in recoupment (a claim in response to the niecersquos claim)

But claims in recoupment and most defenses cannot be raised against a person with the rights of a holder in due course Against a holder in due course the chef can raise only the four ldquorealrdquo defenses listed in UCC sect 3-305(a)(1) (infancy duress lack of legal capacity or illegality that nullifies the obligation of the obligor under other law fraud in the factum discharge in insolvency proceedings) none of which is present here

The niece is not a holder in due course because she did not take the note for value See UCC sectsect 3-302(a)(2)(i) (criteria for holder in due course status) and 3-303(a) (definition of ldquovaluerdquo) But this does not mean that the niece is subject to the chefrsquos claim arising out of the improper repair The buyer was a holder in due course of the note because he took the note for value ($9500) in good faith and without notice of any facts that would have alerted him to the chefrsquos defense against the repairman UCC sect 3-302(a)(2) As a holder in due course the buyer owned the note free of the chefrsquos claim because that claim did not constitute a ldquorealrdquo defense UCC sect 3-305(b) When the buyer gave the note to the niece this constituted a ldquotransferrdquo of the note See UCC sect 3-203(a) When a note is transferred the transferee receives ldquoany right of the transferor to enforce the instrument including any right as a holder in due courserdquo UCC sect 3-203(b) Under this rule (also known as the ldquoshelter principlerdquo) the buyer transferred his freedom from the chefrsquos defenses to the niece and the niece can enforce the note free of the chefrsquos defenses

Point Two (20) Because the chef dishonored the note the niece can recover from the repairman on the repairmanrsquos obligation as indorser

The chefrsquos refusal to pay the note constituted dishonor See UCC sect 3-502 The repairman as an indorser of the note (see Point One(a)) incurred the obligations of an indorser under UCC sect 3-415(a) When a note has been dishonored one of the obligations of an indorser is to pay the amount of the note to a person entitled to enforce it Therefore the repairman is liable for the amount of the note to the niece a person entitled to enforce the note (so long as the niece gives proper notice of dishonor to the repairman)

42

Negotiable Instruments Analysis

[NOTE Because the repairman indorsed the note without warranties there are no transfer warranties UCC sect 3-416 cmt 5]

Point Three (20) The niece cannot recover on the note against the buyer as either indorser or warrantor because the buyer did not indorse the note and did not receive consideration for transferring the note to the niece

The buyer did not indorse the note and therefore did not incur the obligation of an indorser to pay the note upon dishonor

The niece cannot recover from the buyer under a transfer warranty theory because transfer warranties are made only by a person ldquowho transfers an instrument for considerationrdquo Here the buyer gave the instrument to the niece as a gift So the buyer made no transfer warranty UCC sect 3-416(a) Therefore the niece cannot recover from the buyer on that theory

43

National Conference of Bar Examiners 302 South Bedford Street | Madison WI 53703-3622 Phone 608-280-8550 | Fax 608-280-8552 | TDD 608-661-1275

wwwncbexorg e-mail contactncbexorg

  • Contents
  • Preface
  • Description of the MEE
  • Instructions
  • February 2013 Questions
    • Real Property Question
    • Contracts Question
    • Constitutional Law Question
    • Secured Transactions Question
    • Federal Civil Procedure Question
    • Agency Question
    • Evidence Question
    • Trusts and Future Interests Question
    • Negotiable Instruments Question
      • February 2013 Analyses
        • Real Property Analysis
        • Contracts Analysis
        • Constitutional Law Analysis
        • Secured Transactions Analysis
        • Federal Civil Procedure Analysis
        • Agency Analysis
        • Evidence Analysis
        • Trusts and Future Interests Analysis
        • Negotiable Instruments Analysis
            • ltlt ASCII85EncodePages false AllowTransparency false AutoPositionEPSFiles true AutoRotatePages None Binding Left CalGrayProfile (Dot Gain 20) CalRGBProfile (sRGB IEC61966-21) CalCMYKProfile (US Web Coated 050SWOP051 v2) sRGBProfile (sRGB IEC61966-21) CannotEmbedFontPolicy Error CompatibilityLevel 14 CompressObjects Tags CompressPages true ConvertImagesToIndexed true PassThroughJPEGImages true CreateJobTicket false DefaultRenderingIntent Default DetectBlends true DetectCurves 00000 ColorConversionStrategy CMYK DoThumbnails false EmbedAllFonts true EmbedOpenType false ParseICCProfilesInComments true EmbedJobOptions true DSCReportingLevel 0 EmitDSCWarnings false EndPage -1 ImageMemory 1048576 LockDistillerParams false MaxSubsetPct 100 Optimize true OPM 1 ParseDSCComments true ParseDSCCommentsForDocInfo true PreserveCopyPage true PreserveDICMYKValues true PreserveEPSInfo true PreserveFlatness true PreserveHalftoneInfo false PreserveOPIComments true PreserveOverprintSettings true StartPage 1 SubsetFonts true TransferFunctionInfo Apply UCRandBGInfo Preserve UsePrologue false ColorSettingsFile () AlwaysEmbed [ true ] NeverEmbed [ true ] AntiAliasColorImages false CropColorImages true ColorImageMinResolution 300 ColorImageMinResolutionPolicy OK DownsampleColorImages true ColorImageDownsampleType Bicubic ColorImageResolution 300 ColorImageDepth -1 ColorImageMinDownsampleDepth 1 ColorImageDownsampleThreshold 150000 EncodeColorImages true ColorImageFilter DCTEncode AutoFilterColorImages true ColorImageAutoFilterStrategy JPEG ColorACSImageDict ltlt QFactor 015 HSamples [1 1 1 1] VSamples [1 1 1 1] gtgt ColorImageDict ltlt QFactor 015 HSamples [1 1 1 1] VSamples [1 1 1 1] gtgt JPEG2000ColorACSImageDict ltlt TileWidth 256 TileHeight 256 Quality 30 gtgt JPEG2000ColorImageDict ltlt TileWidth 256 TileHeight 256 Quality 30 gtgt AntiAliasGrayImages false CropGrayImages true GrayImageMinResolution 300 GrayImageMinResolutionPolicy OK DownsampleGrayImages true GrayImageDownsampleType Bicubic GrayImageResolution 300 GrayImageDepth -1 GrayImageMinDownsampleDepth 2 GrayImageDownsampleThreshold 150000 EncodeGrayImages true GrayImageFilter DCTEncode AutoFilterGrayImages true GrayImageAutoFilterStrategy JPEG GrayACSImageDict ltlt QFactor 015 HSamples [1 1 1 1] VSamples [1 1 1 1] gtgt GrayImageDict ltlt QFactor 015 HSamples [1 1 1 1] VSamples [1 1 1 1] gtgt JPEG2000GrayACSImageDict ltlt TileWidth 256 TileHeight 256 Quality 30 gtgt JPEG2000GrayImageDict ltlt TileWidth 256 TileHeight 256 Quality 30 gtgt AntiAliasMonoImages false CropMonoImages true MonoImageMinResolution 1200 MonoImageMinResolutionPolicy OK DownsampleMonoImages true MonoImageDownsampleType Bicubic MonoImageResolution 1200 MonoImageDepth -1 MonoImageDownsampleThreshold 150000 EncodeMonoImages true MonoImageFilter CCITTFaxEncode MonoImageDict ltlt K -1 gtgt AllowPSXObjects false CheckCompliance [ None ] PDFX1aCheck false PDFX3Check false PDFXCompliantPDFOnly false PDFXNoTrimBoxError true PDFXTrimBoxToMediaBoxOffset [ 000000 000000 000000 000000 ] PDFXSetBleedBoxToMediaBox true PDFXBleedBoxToTrimBoxOffset [ 000000 000000 000000 000000 ] PDFXOutputIntentProfile () PDFXOutputConditionIdentifier () PDFXOutputCondition () PDFXRegistryName () PDFXTrapped False CreateJDFFile false Description ltlt ARA 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 BGR ltFEFF04180437043f043e043b043704320430043904420435002004420435043704380020043d0430044104420440043e0439043a0438002c00200437043000200434043000200441044a0437043404300432043004420435002000410064006f00620065002000500044004600200434043e043a0443043c0435043d04420438002c0020043c0430043a04410438043c0430043b043d043e0020043f044004380433043e04340435043d04380020043704300020043204380441043e043a043e043a0430044704350441044204320435043d0020043f04350447043004420020043704300020043f044004350434043f0435044704300442043d04300020043f043e04340433043e0442043e0432043a0430002e002000200421044a04370434043004340435043d043804420435002000500044004600200434043e043a0443043c0435043d044204380020043c043e0433043004420020043404300020044104350020043e0442043204300440044f0442002004410020004100630072006f00620061007400200438002000410064006f00620065002000520065006100640065007200200035002e00300020043800200441043b0435043404320430044904380020043204350440044104380438002egt CHS ltFEFF4f7f75288fd94e9b8bbe5b9a521b5efa7684002000410064006f006200650020005000440046002065876863900275284e8e9ad88d2891cf76845370524d53705237300260a853ef4ee54f7f75280020004100630072006f0062006100740020548c002000410064006f00620065002000520065006100640065007200200035002e003000204ee553ca66f49ad87248672c676562535f00521b5efa768400200050004400460020658768633002gt CHT ltFEFF4f7f752890194e9b8a2d7f6e5efa7acb7684002000410064006f006200650020005000440046002065874ef69069752865bc9ad854c18cea76845370524d5370523786557406300260a853ef4ee54f7f75280020004100630072006f0062006100740020548c002000410064006f00620065002000520065006100640065007200200035002e003000204ee553ca66f49ad87248672c4f86958b555f5df25efa7acb76840020005000440046002065874ef63002gt CZE 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 DAN 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 DEU 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 ESP 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 ETI 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 FRA 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 GRE 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 HEB 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 HRV (Za stvaranje Adobe PDF dokumenata najpogodnijih za visokokvalitetni ispis prije tiskanja koristite ove postavke Stvoreni PDF dokumenti mogu se otvoriti Acrobat i Adobe Reader 50 i kasnijim verzijama) HUN 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 ITA 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 JPN ltFEFF9ad854c18cea306a30d730ea30d730ec30b951fa529b7528002000410064006f0062006500200050004400460020658766f8306e4f5c6210306b4f7f75283057307e305930023053306e8a2d5b9a30674f5c62103055308c305f0020005000440046002030d530a130a430eb306f3001004100630072006f0062006100740020304a30883073002000410064006f00620065002000520065006100640065007200200035002e003000204ee5964d3067958b304f30533068304c3067304d307e305930023053306e8a2d5b9a306b306f30d530a930f330c8306e57cb30818fbc307f304c5fc59808306730593002gt KOR ltFEFFc7740020c124c815c7440020c0acc6a9d558c5ec0020ace0d488c9c80020c2dcd5d80020c778c1c4c5d00020ac00c7a50020c801d569d55c002000410064006f0062006500200050004400460020bb38c11cb97c0020c791c131d569b2c8b2e4002e0020c774b807ac8c0020c791c131b41c00200050004400460020bb38c11cb2940020004100630072006f0062006100740020bc0f002000410064006f00620065002000520065006100640065007200200035002e00300020c774c0c1c5d0c11c0020c5f40020c2180020c788c2b5b2c8b2e4002egt LTH 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 LVI 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 NLD (Gebruik deze instellingen om Adobe PDF-documenten te maken die zijn geoptimaliseerd voor prepress-afdrukken van hoge kwaliteit De gemaakte PDF-documenten kunnen worden geopend met Acrobat en Adobe Reader 50 en hoger) NOR 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 POL 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 PTB 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 RUM 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 RUS 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 SKY ltFEFF0054006900650074006f0020006e006100730074006100760065006e0069006100200070006f0075017e0069007400650020006e00610020007600790074007600e100720061006e0069006500200064006f006b0075006d0065006e0074006f0076002000410064006f006200650020005000440046002c0020006b0074006f007200e90020007300610020006e0061006a006c0065007001610069006500200068006f0064006900610020006e00610020006b00760061006c00690074006e00fa00200074006c0061010d00200061002000700072006500700072006500730073002e00200056007900740076006f00720065006e00e900200064006f006b0075006d0065006e007400790020005000440046002000620075006400650020006d006f017e006e00e90020006f00740076006f00720069016500200076002000700072006f006700720061006d006f006300680020004100630072006f00620061007400200061002000410064006f00620065002000520065006100640065007200200035002e0030002000610020006e006f0076016100ed00630068002egt SLV 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 SUO 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 SVE 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 TUR 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 UKR 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 ENU (Use these settings to create Adobe PDF documents best suited for high-quality prepress printing Created PDF documents can be opened with Acrobat and Adobe Reader 50 and later) gtgt Namespace [ (Adobe) (Common) (10) ] OtherNamespaces [ ltlt AsReaderSpreads false CropImagesToFrames true ErrorControl WarnAndContinue FlattenerIgnoreSpreadOverrides false IncludeGuidesGrids false IncludeNonPrinting false IncludeSlug false Namespace [ (Adobe) (InDesign) (40) ] OmitPlacedBitmaps false OmitPlacedEPS false OmitPlacedPDF false SimulateOverprint Legacy gtgt ltlt AddBleedMarks false AddColorBars false AddCropMarks false AddPageInfo false AddRegMarks false ConvertColors ConvertToCMYK DestinationProfileName () DestinationProfileSelector DocumentCMYK Downsample16BitImages true FlattenerPreset ltlt PresetSelector MediumResolution gtgt FormElements false GenerateStructure false IncludeBookmarks false IncludeHyperlinks false IncludeInteractive false IncludeLayers false IncludeProfiles false MultimediaHandling UseObjectSettings Namespace [ (Adobe) (CreativeSuite) (20) ] PDFXOutputIntentProfileSelector DocumentCMYK PreserveEditing true UntaggedCMYKHandling LeaveUntagged UntaggedRGBHandling UseDocumentProfile UseDocumentBleed false gtgt ]gtgt setdistillerparamsltlt HWResolution [2400 2400] PageSize [612000 792000]gtgt setpagedevice

Page 5: February 2013 MEE Questions and Analyses

_____

Instructions

The back cover of each test booklet contains the following instructions

You will be instructed when to begin and when to stop this test Do not break the seal on this booklet until you are told to begin

You may answer the questions in any order you wish Do not answer more than one question in each answer booklet If you make a mistake or wish to revise your answer simply draw a line through the material you wish to delete

If you are using a laptop computer to answer the questions your jurisdiction will provide you with specific instructions

Read each fact situation very carefully and do not assume facts that are not given in the question Do not assume that each question covers only a single area of the law some of the questions may cover more than one of the areas you are responsible for knowing

Demonstrate your ability to reason and analyze Each of your answers should show an understanding of the facts a recognition of the issues included a knowledge of the applicable principles of law and the reasoning by which you arrive at your conclusion The value of your answer depends not as much upon your conclusions as upon the presence and quality of the elements mentioned above

Clarity and conciseness are important but make your answer complete Do not volunteer irrelevant or immaterial information

Answer all questions according to generally accepted fundamental legal principles unless your jurisdiction has instructed you to answer according to local case or statutory law (UBE instructions Answer all questions according to generally accepted fundamental legal principles rather than local case or statutory law)

iii

February 2013 MEE

QUESTIONS

Real Property Contracts

Constitutional Law Secured Transactions

Federal Civil Procedure Agency

Evidence Trusts and Future Interests

Negotiable Instruments

REAL PROPERTY QUESTION _______________

In 2008 a landlord and a tenant entered into a 10-year written lease commencing September 1 2008 for the exclusive use of a commercial building at a monthly rent of $2500 The lease contained a covenant of quiet enjoyment but no other covenants or promises on the part of the landlord

When the landlord and tenant negotiated the lease the tenant asked the landlord if the building had an air-conditioning system The landlord answered ldquoYes it doesrdquo The tenant responded ldquoGreat I will be using the building to manufacture a product that will be irreparably damaged if the temperature during manufacture exceeds 81 degrees for more than six consecutive hoursrdquo

On April 15 2012 the buildingrsquos air-conditioning system malfunctioned causing the building temperature to rise above 81 degrees for three hours The tenant immediately telephoned the landlord about this malfunction The tenant left a message in which he explained what had happened and asked the landlord ldquoWhat are you going to do about itrdquo The landlord did not respond to the tenantrsquos message

On May 15 2012 the air-conditioning system again malfunctioned This time the malfunction caused the building temperature to rise above 81 degrees for six hours The tenant telephoned the landlord and left a message describing the malfunction As before the landlord did not respond

On August 24 2012 the air-conditioning system malfunctioned again causing the temperature to rise above 81 degrees for 10 hours Again the tenant promptly telephoned the landlord The landlord answered the phone and the tenant begged her to fix the system The landlord refused The tenant then attempted to fix the system himself but he failed As a result of the air-conditioning malfunction products worth $150000 were destroyed

The next day the tenant wrote the following letter to the landlord

Irsquove had enough I told you about the air-conditioning problem twice before yesterdayrsquos disaster and you failed to correct it I will vacate the building by the end of the month and will bring you the keys when I leave

The tenant vacated the building on August 31 2012 and returned the keys to the landlord that day At that time there were six years remaining on the lease

On September 1 2012 the landlord returned the keys to the tenant with a note that said ldquoI repeat the air-conditioning is not my problem You have leased the building and you should fix itrdquo The tenant promptly sent the keys back to the landlord with a letter that said ldquoI have terminated the lease and I will not be returning to the building or making further rent paymentsrdquo After receiving the keys and letter the landlord put the keys into her desk To date she has neither responded to the tenantrsquos letter nor taken steps to lease the building to another tenant

On November 1 2012 two months after the tenant vacated the property the landlord sued the tenant claiming that she is entitled to the remaining unpaid rent ($180000) from September 1 for the balance of the lease term (reduced to present value) or if not that then damages for the tenantrsquos wrongful termination

Is the landlord correct Explain

3

CONTRACTS QUESTION _______________

On January 2 a boat builder and a sailor entered into a contract pursuant to which the builder was to sell to the sailor a boat to be specially manufactured for the sailor by the builder The contract price was $100000 The written contract signed by both parties stated that the builder would tender the boat to the sailor on December 15 at which time payment in full would be due

On October 15 the builderrsquos workers went on strike and there were no available replacements

On October 31 the builderrsquos workers were still on strike and no work was being done on the boat The sailor read a news report about the strike and immediately sent a letter to the builder stating ldquoI am very concerned that my boat will not be completed by December 15 I insist that you provide me with assurance that you will perform in accordance with the contractrdquo The builder received the letter on the next day November 1

On November 25 the builder responded to the letter stating ldquoIrsquom sorry about the strike but it is really out of my hands I hope we settle it soon so that we can get back to workrdquo

Nothing further happened until December 3 when the builder called the sailor and said ldquoMy workers are back and I have two crews working overtime to finish your boat Your boat is task one Donrsquot worry wersquoll deliver your boat by December 15thrdquo The sailor immediately replied ldquoI donrsquot trust you As far as Irsquom concerned our contract is over I am going to buy my boat from a shipyardrdquo Two days later the sailor entered into a contract with a competing manufacturer to buy a boat similar to the boat that was the subject of the contract with the builder

The builder finished the boat on time and tendered it to the sailor on December 15 The sailor reminded the builder about the December 3 conversation in which the sailor had announced that ldquoour contract is overrdquo and refused to take the boat and pay for it

The builder has sued the sailor for breach of contract

1 What was the legal effect of the sailorrsquos October 31 letter to the builder Explain

2 What was the legal effect of the builderrsquos November 25 response to the sailorrsquos October 31 letter Explain

3 What was the legal effect of the sailorrsquos refusal to take and pay for the boat on December 15 Explain

4

CONSTITUTIONAL LAW QUESTION

AutoCo is a privately owned corporation that manufactures automobiles Ten years ago AutoCo purchased a five-square-mile parcel of unincorporated land in a remote region of the state and built a large automobile assembly plant on the land To attract workers to the remote location of the plant AutoCo built apartment buildings and houses on the land and leased them to its employees AutoCo owns and operates a commercial district with shops and streets open to the general public AutoCo named the area Oakwood and provides security fire protection and sanitation services for Oakwoodrsquos residents AutoCo also built operates and fully funds the only school in the region which it makes available free of charge to the children of its employees

A family recently moved to Oakwood The father and mother work in AutoCorsquos plant rent an apartment from AutoCo and have enrolled their 10-year-old son in Oakwoodrsquos school Every morning the students are required to recite the Pledge of Allegiance while standing and saluting an American flag With the approval of his parents the son has politely but insistently refused to recite the Pledge and salute the flag at the school on the grounds that doing so violates his own political beliefs and the political beliefs of his family As a result of his refusal to say the Pledge the son has been expelled from the school

To protest the schoolrsquos actions the father walked into the commercial district of Oakwood While standing on a street corner he handed out leaflets that contained a short essay critical of the schoolrsquos Pledge of Allegiance policy Some of the passersby who took the leaflets dropped them to the ground An AutoCo security guard saw the litter told the father that Oakwoodrsquos anti-litter rule prohibits leaflet distribution that results in littering and directed him to cease distribution of the leaflets and leave the commercial district When the father did not leave and continued to distribute the leaflets the security guard called the state police which sent officers who arrested the father for trespass

1 Did the sonrsquos expulsion from the school violate the First Amendment as applied through the Fourteenth Amendment Explain

2 Did the fatherrsquos arrest violate the First Amendment as applied through the Fourteenth Amendment Explain

5

SECURED TRANSACTIONS QUESTION

On June 1 a bicycle retailer sold two bicycles to a man for a total purchase price of $1500 The man made a $200 down payment and agreed to pay the balance in one year The man also signed a security agreement that identified the bicycles as collateral for the unpaid purchase price and provided that the man ldquoshall not sell or dispose of the collateral until the balance owed is paid in fullrdquo The retailer never filed a financing statement reflecting this security interest

The man had bought the bicycles for him and his girlfriend to use on vacation However shortly after he bought the bicycles the man and his girlfriend broke up The man has never used the bicycles

On August 1 the man sold one of the bicycles at a garage sale to a buyer who paid the man $400 for the bicycle The buyer bought the bicycle to ride for weekend recreation

On October 1 the man gave the other bicycle to his friend as a birthday present The friend began using the bicycle for morning exercise

Neither the buyer nor the friend had any knowledge of the manrsquos dealings with the retailer

1 Does the buyer own the bicycle free of the retailerrsquos security interest Explain

2 Does the friend own the bicycle free of the retailerrsquos security interest Explain

6

FEDERAL CIVIL PROCEDURE QUESTION _____

Mother and Son who are both adults are citizens and residents of State A Mother owned an expensive luxury car valued in excess of $100000 Son borrowed Motherrsquos car to drive to a store in State A As Son approached a traffic light that had just turned yellow he carefully braked and brought the car to a complete stop Driver who was following immediately behind him failed to stop and rear-ended Motherrsquos car which was damaged beyond repair Son was seriously injured Driver is a citizen of State B

Son sued Driver in the United States District Court for the District of State A alleging that she was negligent in the operation of her vehicle Son sought damages in excess of $75000 for his personal injuries exclusive of costs and interest In her answer Driver alleged that Son was contributorily negligent in the operation of Motherrsquos car She further alleged that the brake lights on Motherrsquos car were burned out and that Motherrsquos negligent failure to properly maintain the car was a contributing cause of the accident

Following a trial on the merits in Sonrsquos case against Driver the jury answered the following special interrogatories

Do you find that Driver was negligent in the operation of her vehicle Yes

Do you find that Son was negligent in the operation of Motherrsquos car No

Do you find that Mother negligently failed to ensure that the brake lights on her car were in proper working order Yes

The judge then entered a judgment in favor of Son against Driver Driver did not appeal

Two months later Mother sued Driver in the United States District Court for the District of State A alleging that Driverrsquos negligence in the operation of her vehicle destroyed Motherrsquos luxury car Mother sought damages in excess of $75000 exclusive of costs and interest

State A follows the same preclusion principles that federal courts follow in federal-question cases

1 Is Motherrsquos claim against Driver barred by the judgment in Son v Driver Explain

2 Does the juryrsquos conclusion in Son v Driver that Mother had negligently failed to maintain the brake lights on her car preclude Mother from litigating that issue in her subsequent suit against Driver Explain

3 Does the juryrsquos conclusion in Son v Driver that Driver was negligent preclude Driver from litigating that issue in the Mother v Driver lawsuit Explain

7

_____

AGENCY QUESTION

Over 5000 individuals in the United States operate hot-air balloon businesses A hot-air balloon has four key components the balloon that holds the heated air the basket that houses the riders the propane burner that heats the air in the balloon and the propane storage tanks

The owner of a hot-air balloon business recently notified several basket and burner manufacturers that she or her agent might be contacting them to purchase baskets or burners The owner did not specifically name any person as her agent Basket and burner manufacturers regularly receive such notices from hot-air balloon operators Such notices typically include no restrictions on the types of baskets or burners agents might purchase for their principals

The owner then retained an agent to acquire baskets burners and fuel tanks from various manufacturers The owner authorized the agent to buy only (a) baskets made of woven wicker (not aluminum) (b) burners that use a unique ldquowhisper technologyrdquo (so as not to scare livestock when the balloon sails over farmland) and (c) propane fuel tanks

The agent then entered into three transactions with manufacturers all of whom had no prior dealings with either the owner or the agent

(1) The agent and a large manufacturer of both wicker and aluminum baskets signed a contract for the purchase of four aluminum baskets for a total cost of $60000 The agent never told the manufacturer that he represented the owner or any other principal The contract listed the agent as the buyer and listed the ownerrsquos address as the delivery address but did not indicate that the address was that of the owner rather than the agent When the baskets were delivered to the owner she learned for the first time that the agent had contracted to buy aluminum not wicker baskets The owner immediately rejected the baskets and returned them to the manufacturer Neither the owner nor the agent has paid the basket manufacturer for them

(2) The agent contacted a burner manufacturer and told him that the agent represented a well-known hot-air balloon operator who wanted to purchase burners The agent did not disclose the ownerrsquos name The agent and the burner manufacturer signed a contract for the purchase of four burners that did not have ldquowhisper technologyrdquo for a total price of $70000 The burner contract like the basket contract listed the ownerrsquos address for delivery but did not disclose whose address it was The burners were delivered to the ownerrsquos business and the owner discovered that the agent had ordered the wrong kind of burners The owner rejected the burners and returned them to the manufacturer Neither the owner nor the agent has paid the burner manufacturer for the burners

(3) The agent contracted with a solar cell manufacturer to make three cells advertised as ldquostrong enough to power all your ballooning needsrdquo The agent did not tell the manufacturer that he was acting on behalf of any other person One week after the cells were delivered to the agent he took them to the owner who installed them and discovered that she could save a lot of money using solar cells instead of propane to power her balloons The owner decided to keep the solar cells but she has not paid the manufacturer for them

8

Agency Question

Assume that the rejection of the baskets and the burners and the failure to pay for the solar cells constitute breach of the relevant contracts

1 Is the owner liable to the basket manufacturer for breach of the contract for the aluminum baskets Is the agent liable Explain

2 Is the owner liable to the burner manufacturer for breach of the contract for the burners Is the agent liable Explain

3 Is the owner liable to the solar cell manufacturer for breach of the contract for the solar cells Is the agent liable Explain (Do not address liability based upon restitution or unjust enrichment)

9

EVIDENCE QUESTION _____

A woman who owns a motorized scooter brought her scooter to a mechanic for routine maintenance service As part of the maintenance service the mechanic inspected the braking system on the scooter As soon as the mechanic finished inspecting and servicing the scooter he sent the woman a text message to her cell phone that read ldquoJust finished your service When you pick up your scooter you need to schedule a follow-up brake repair Wersquoll order the partsrdquo

The woman read the mechanicrsquos text message and returned the next day to pick up her scooter As the woman was wheeling her scooter out of the shop she saw the mechanic working nearby and asked ldquoIs my scooter safe to ride for a whilerdquo The mechanic responded by giving her a thumbs-up The woman waved and rode away on the scooter

One week later while the woman was riding her scooter a pedestrian stepped off the curb into a crosswalk and the woman collided with him causing the pedestrian severe injuries The woman had not had the scooterrsquos brakes repaired before the accident

The pedestrian has sued the woman for damages for his injuries resulting from the accident The pedestrian has alleged that (1) the woman lost control of the scooter due to its defective brakes (2) the woman knew that the brakes needed repair and (3) it was negligent for the woman to ride the scooter knowing that its brakes needed to be repaired

The woman claims that the brakes on the scooter worked perfectly and that the accident happened because the pedestrian stepped into the crosswalk without looking and the woman had no time to stop The woman the pedestrian and the mechanic will testify at the upcoming trial

The pedestrian has proffered an authenticated copy of the mechanicrsquos text message to the woman

The woman plans to testify that she asked the mechanic ldquoIs my scooter safe to ride for a whilerdquo and that he gave her a thumbs-up in response

The evidence rules in this jurisdiction are identical to the Federal Rules of Evidence

Analyze whether each of these items of evidence is relevant and admissible at trial

1 The authenticated copy of the mechanicrsquos text message

2 The womanrsquos testimony that she asked the mechanic ldquoIs my scooter safe to ride for a whilerdquo and

3 The womanrsquos testimony describing the mechanicrsquos thumbs-up

10

TRUSTS AND FUTURE INTERESTS QUESTION

Ten years ago Settlor validly created an inter vivos trust and named Bank as trustee The trust instrument provided that Settlor would receive all of the trust income during her lifetime The trust instrument further provided that

Upon Settlorrsquos death the trust income shall be paid in equal shares to Settlorrsquos surviving children for their lives Upon the death of the last surviving child the trust income shall be paid in equal shares to Settlorrsquos then-living grandchildren for their lives Upon the death of the survivor of Settlorrsquos children and grandchildren the trust corpus shall be distributed in equal shares to Settlorrsquos then-living great-grandchildren

The trust instrument expressly specified that the trust was revocable but it was silent regarding whether Settlor could amend the trust instrument

Immediately after creating the trust Settlor validly executed a will leaving her entire estate to Bank as trustee of her inter vivos trust to ldquohold in accordance with the terms of the trustrdquo

Five years ago Settlor signed an amendment to the inter vivos trust The amendment changed the disposition of the remainder interest specifying that all trust assets ldquoshall be paid upon Settlorrsquos death to Universityrdquo Settlorrsquos signature on this amendment was not witnessed

A state statute provides that any trust interest that violates the common law Rule Against Perpetuities ldquois nonetheless valid if the nonvested interest in the trust actually vests or fails to vest either (a) within 21 years of lives in being at the creation of the nonvested interest or (b) within 90 years of its creationrdquo

Recently Settlor died leaving a probate estate of $200000 She was survived by no children one granddaughter (who would be Settlorrsquos only heir) and no great-grandchildren The granddaughter has consulted your law firm and has raised four questions regarding this trust

1 Was Settlorrsquos amendment of the inter vivos trust valid Explain

2 Assuming that the trust amendment was valid do its provisions apply to Settlorrsquos probate assets Explain

3 Assuming that the trust amendment was valid how should trust assets be distributed Explain

4 Assuming that the trust amendment was invalid how should trust assets be distributed Explain

11

NEGOTIABLE INSTRUMENTS QUESTION

A chef entered into a contract with a repairman pursuant to which the repairman agreed to repair the chefrsquos commercial oven for $10000 The repairman agreed to accept as payment a negotiable promissory note for $10000 payable two months after its issuance

After the repairman worked on the oven the chef gave him a $10000 note as payment for the work As agreed the note was signed by the chef as maker was payable to the order of the repairman was payable in two months and fulfilled all criteria for negotiability

The next day the repairman sold the note to a buyer for $9500 To effectuate the sale the repairman wrote ldquono warrantiesrdquo on the back of the note signed his name immediately below that and handed the note to the buyer The buyer bought the note in good faith and without knowledge of any facts relating to the work that the repairman had performed for the chef

Later the buyer gave the note to his niece as a gift To effectuate the gift the buyer handed the note to the niece but did not indorse it

Shortly thereafter the chef discovered that the repair work had been done improperly and the oven still did not function correctly The chef tried repeatedly to get the repairman to return to correct the repair work but the repairman ignored all the chefrsquos calls

On the notersquos due date the niece contacted the chef and demanded that he pay the amount of the note to her The chef refused and told the niece that he would not pay the note because the repairman did not properly repair the oven

1 What are the niecersquos rights against the chef Explain

2 What are the niecersquos rights against the repairman Explain

3 What are the niecersquos rights against the buyer Explain

12

February 2013 MEE

ANALYSES

Real Property Contracts

Constitutional Law Secured Transactions

Federal Civil Procedure Agency

Evidence Trusts and Future Interests

Negotiable Instruments

REAL PROPERTY ANALYSIS (Real Property ID1a 4 amp 5)

ANALYSIS

Legal Problems

(1) Does the tenant have a defense to the landlordrsquos action for unpaid rent based on constructive eviction

(2) Does the tenant have a defense to the landlordrsquos action for unpaid rent based on the tenantrsquos surrender of the premises

(3) What if anything may the landlord recover from the tenant for the period after the tenant vacated the building

DISCUSSION

Summary

Under the common law the tenant does not have a defense to the landlordrsquos action for unpaid rent based on constructive eviction Constructive eviction is based on the tenant proving that (1) the landlord breached a duty to the tenant (2) the breach caused a loss by the tenant of the substantial use and enjoyment of the premises (3) the tenant gave the landlord adequate notice and opportunity to repair and (4) the tenant vacated the leased premises Here there was no constructive eviction because although the tenant vacated and gave the landlord adequate notice the landlord breached no express or implied duty to the tenant to repair the premises

The tenant does not have a defense based on the landlordrsquos acceptance of his surrender of the premises a landlordrsquos retention of keys does not constitute an acceptance of the tenantrsquos surrender unless the landlord so intended and here the landlordrsquos statements to the tenant at the time of the surrender of the keys do not evidence the intent to accept the tenantrsquos surrender

Under the common law a landlord has no duty to mitigate damages but also cannot sue for rents due in the future Under this approach the landlord can sue only for past-due rents Using this approach on November 1 the landlord could recover all the rent past due (ie rent for September and October) but could not recover for rents due in the future However some courts have authorized recovery for future rent minus the fair market rental value of the premises It is thus possible that the landlord could recover damages equal to the amount of rent due from September 1 to the end of the six-year lease term ($180000) minus the propertyrsquos fair-market rental value over that same period

Point One (45) The tenant was not constructively evicted because the landlord had no duty to repair the commercial premises that were the subject of the lease

The landlord and the tenant entered into a term-of-years lease because the lease specified both a beginning and an ending date HERBERT HOVENKAMP amp SHELDON F KURTZ THE LAW OF

PROPERTY 256 (5th ed 2001) Although a term-of-years lease normally cannot be terminated by the tenant prior to the end of the term a tenant may terminate a term-of-years lease if the tenant

15

Real Property Analysis

is constructively evicted See id at 286ndash88 Typically as here a claim of constructive eviction is made as a defense to a landlordrsquos action for damages or unpaid rent

In order to establish a constructive eviction the tenant must prove that the landlord breached a duty to the tenant such as a duty to repair and that the landlordrsquos breach caused a loss of the substantial use and enjoyment of the premises The tenant must also show that he gave the landlord notice adequate to permit the landlord to meet his duty to the tenant and that the tenant vacated the leased premises Id see also JOHN G SPRANKLING UNDERSTANDING

PROPERTY LAW sect 1704 (2d ed 2007) Under the common law there was no implied duty on the part of a landlord to repair

leased premises such a duty arose only if expressly set forth in the lease SPRANKLING supra sect 1702[B] Here the written lease contained no term requiring the landlord to repair the air-conditioning Even if the conversation created a lease term that the building had air-conditioning that itself should not create a duty for the landlord to repair it

Over the past several decades courts have generally implied a duty to repair in residential leases either as part of a revised constructive eviction doctrine or based on an implied warranty of habitability JOSEPH W SINGER PROPERTY 469ndash70 (3d ed 2010) This shift has been justified based on the economic disparity between the typical landlord and tenant as well as the fact that residential tenants generally lack both the authority to authorize repairs to common areas of a building and the incentive to make repairs that will ultimately benefit the landlord

However courts have been more reluctant to imply a duty to repair in commercial leases a context in which the tenant is often a valuable business and in a better position to assess and make repairs than is the landlord But see eg Davidow v Inwood North Professional Group 747 SW2d 373 (Tex 1988) When courts have implied a duty to repair in a commercial lease it is typically when the repair has been mandated by public authorities and involves work so substantial that it would not ordinarily fall within the tenantrsquos common law repair duty andor the value of the repair would primarily inure to the landlordrsquos reversionary interest See Brown v Green 884 P2d 55 (Cal 1994) Eugene L Grant et al The Tenant as Terminator Constructive Eviction in Modern Commercial Leases 2 THE COMMERCIAL PROPERTY LEASE ch 15 (ABA 1997) Some courts have also permitted constructive eviction claims by commercial tenants of office buildings based on repairs required in common areas of the building See id Echo Consulting Services Inc v North Conway Bank 669 A2d 227 (NH 1995)

Here the tenant is the owner of a valuable manufacturing operation and is the exclusive occupant of the building the repair has not been mandated by public authorities and the repair is not structural To the contrary the repair involves a feature of the building of unusual importance in the tenantrsquos manufacturing operation and the tenant is likely far more knowledgeable than the landlord about the air-conditioning specifications necessary for the manufacture of the tenantrsquos product

Based on these facts it is unlikely that a court will find that the tenant in this case was constructively evicted Although the tenant can show that he gave adequate notice to the landlord of the air-conditioning malfunction and vacated the premises the lease was commercial and it did not contain any promises or covenants by the landlord except a covenant of quiet enjoyment a covenant of quiet enjoyment does not entail any repair obligations

[NOTE An examineersquos conclusion is less important than his or her demonstrated awareness of the elements of constructive eviction and the need to imply a repair duty for such a defense to be viable here Although the implied warranty of habitability is not available to this tenant Texas Minnesota and Massachusetts imply a warranty of suitability in commercial leases in limited circumstances and an examinee might argue that this warranty should apply

16

Real Property Analysis

here If an examinee concludes that this warranty applies he or she should discuss the other requirements for constructive eviction

If the examinee wrongly concludes that the first element for a constructive eviction has been met the examinee will then have to discuss the remaining three elements in order to conclude that the tenant can claim constructive eviction The tenant would have a strong argument that the second elementmdashsubstantial interference with the use and enjoyment of the premisesmdashalso is met As indicated above the landlord was aware that a functioning air-conditioning system was vital to the tenantrsquos manufacturing operations The facts further indicate that the system had failed three times in the past few months The landlord may try to argue that the malfunctions did not substantially interfere with the tenantrsquos use of the premises because the malfunctions caused the temperature to climb above 81 degrees for only a short period of timemdash 3 hours 6 hours and 10 hours respectivelymdashon each occasion The tenant will argue however that the landlord was aware that the tenantrsquos manufacturing operations could tolerate temperatures above 81 degrees for no more than 6 hours The final malfunction exceeded that limit destroying $150000 worth of the tenantrsquos products

The tenant would also have a strong argument that the third element is met notice and opportunity to cure The tenant notified the landlord of the problem immediately upon the systemrsquos first malfunction and did so again when it malfunctioned a second time and then a third time The landlord might argue that there was insufficient time to cure the problem because the system corrected itself within a few hours on the first and second times Although the malfunction lasted more than 10 hours the third time the landlord might argue that the time period was insufficient to get a repair person on the premises A court would be likely to find this argument unpersuasive however because the landlord could have attempted to correct the problem after the first and second malfunctions

Assuming that the landlord was given sufficient notice and opportunity to cure a court would be likely to conclude that the tenant also satisfied the final element of vacating the premises within a reasonable time The landlord might argue that the tenant remained in the premises for almost four months after the air conditioning first failed which would suggest that the problem was not so severe as to have constructively evicted the tenant The tenant will argue however that he gave the landlord three months to cure the problem after the first two malfunctions threatened (but did not actually harm) his operations The tenant then moved out shortly after the final malfunction caused temperatures to exceed the tolerance levels of his manufacturing operations]

Point Two (10) The landlord did not accept the tenantrsquos surrender of the lease

When a tenant wrongfully moves from leased premises with the intent to terminate the lease the landlord may either accept the tenantrsquos surrender of the premises and terminate the lease or hold the tenant to the terms of the lease See HOVENKAMP amp KURTZ supra at 295ndash96 Here the tenantrsquos only basis for the claim that the landlord accepted his surrender is the landlordrsquos retention of the keys Many courts have considered whether a landlordrsquos retention of keys delivered by a tenant constitutes acceptance of surrender The weight of the case law holds that retention of the keys alone does not constitute acceptance of surrender without other evidence showing that the landlord intended to accept the surrender See generally 49 AM JUR 2d Landlord and Tenant sect 213

Here the landlordrsquos note saying ldquoI repeat the air-conditioning is not my problem You have leased the building and you should fix itrdquo strongly suggests that the landlord did not intend

17

Some courts have rejected the no-mitigation-of-damages rule based on efficiency concerns and societyrsquos interest in assuring that resources remain in the stream of commerce rather than lying vacant see id at 464ndash65 and allow landlords to sue tenants who have wrongfully terminated a lease for damages equal to the difference between the unpaid rent due under the lease and the propertyrsquos fair market rental value Other courts have abandoned the no-recovery-for-future-rent rule These courts responding to the fact that a tenant may well disappear or be judgment-proof by the time a lease term is concluded have allowed a landlord to collect damages equal to the value of rent over the entire lease term minus the propertyrsquos fair rental value when a tenant has wrongfully terminated a lease and unequivocally shown an intention not to return to the premises or pay future rent Under this approach a landlord receives approximately the same amount he would have received were there a duty to mitigate damages See Sagamore Corp v Willcutt 180 A 464 (Conn 1935)

Real Property Analysis

to accept the tenantrsquos surrender The tenant might argue that the landlordrsquos failure to make a similar statement when the keys were sent to her a second time and she retained them evidences a change of heart However it is likely that a court would find that the landlordrsquos retention of the keys represented a decision to safeguard the keys not to accept the tenantrsquos surrender

[NOTE An examinee should receive credit for arguing the other way with a well-reasoned argument]

Point Three (45) Under the common law the landlord had no duty to mitigate damages Additionally a landlord was not entitled to recover unpaid rents due in the future but was only entitled to recover rents in arrears at the time of the commencement of the suit Applying the common law here the landlord could recover $5000 the amount of rents due at the commencement of the suit ($2500 for September and the same for October) Today some courts allow the landlord under certain circumstances to sue the tenant for damages (not rent) equal to the difference if any between the unpaid promised rent for the balance of the term (here $175000) and the propertyrsquos fair rental value for the balance of the term

Under the common law because a lease was viewed as a conveyance instead of a contract a landlord had no duty to mitigate damages resulting from a tenantrsquos wrongful termination of a lease A landlord could thus recover the full value of rents that were due and unpaid at the time of the suit However under the common law a landlord could not sue a tenant for rents due in the future because there was always a possibility that the tenant might pay the rent when it was due See SINGER supra at 462 Thus using the common law approach on November 1 the landlord could only recover the full value of the two monthsrsquo rent actually due and unpaid ie $5000 for September and October

Here because the tenant returned the keys to the landlord and said ldquoI will not be returning to the building or making further rent paymentsrdquo the landlord could establish abandonment and an intention not to return It is thus possible that the landlord might recover damages in the amount of $5000 (for the months of September and October) plus the present value of $175000 minus the fair market rental value of the property over the remaining months of the lease

18

CONTRACTS ANALYSIS ____ (Contracts II IVE)

ANALYSIS

Legal Problems

(1) What was the legal effect of the sailorrsquos October 31 letter to the builder

(2)(a) What was the legal effect of the builderrsquos November 25 response to the sailorrsquos October 31 letter

(2)(b) What was the legal effect of the sailorrsquos refusal to take and pay for the boat on December 15

DISCUSSION

Summary

This is a sale of goods governed by the Uniform Commercial Code Because the sailor had reasonable grounds for insecurity about the builderrsquos ability to deliver the boat in a timely manner when the sailor learned about the strike on October 31 the sailor was legally justified in sending the letter to the builder seeking adequate assurance of the builderrsquos performance pursuant to the contract The builderrsquos failure to provide such assurance within a reasonable time operated as a repudiation of the contract However the builder was free to retract the repudiation before the sailor either cancelled the contract or materially changed position in reliance on the builderrsquos repudiation The builder retracted the repudiation when he informed the sailor that the workers were back and that the boat would be delivered by the date stipulated in the partiesrsquo contract Because the sailor had taken no action in response to the original repudiation he no longer had the right to cancel the contract with the builder The sailorrsquos subsequent statement that ldquoour contract is overrdquo may have constituted repudiation by the sailor In any event when the sailor failed to perform on December 15 that constituted breach

Point One (35) Because the sailor had reasonable grounds for insecurity with respect to the builderrsquos performance the sailorrsquos letter to the builder was a justified demand seeking assurance of the builderrsquos performance under the contract failure of the builder to provide such assurance within a reasonable time constituted repudiation of the contract

The sailor was legally justified in sending the letter to the builder on October 31 Contract parties are entitled to expect due performance of contractual obligations and are permitted to take steps to protect that expectation UCC sect 2-609 states that ldquo[w]hen reasonable grounds for insecurity arise with respect to the performance of either party the other may in writing demand adequate assurance of due performance rdquo Here the sailor learned on October 31 that the builderrsquos workers were on strike This gave the sailor reasonable grounds for insecurity about the builderrsquos ability to complete performance on time and thus gave the sailor the right to seek adequate assurance from the builder Because the sailorrsquos demand for assurance was justified the builder was required to provide assurance that was adequate under the circumstances within a reasonable time (not to exceed 30 days) or be held to have repudiated the contract UCC sect 2-609(4)

19

Contracts Analysis

Point Two(a) (30) The builder did not within a reasonable time provide the sailor adequate assurance of due performance this failure to provide assurance constituted a repudiation of the contract

Because the sailor with legal justification (see Point One) demanded from the builder assurance of due performance the builderrsquos failure to provide such assurance within a reasonable time was a repudiation of their contract See UCC sect 2-609(4) (ldquoAfter receipt of a justified demand[] failure to provide within a reasonable time not exceeding thirty days assurance of due performance is a repudiation of the contractrdquo) On October 31 the sailor requested that the builder provide adequate assurance regarding the completion of the boat by December 15 The builder did not respond to the sailorrsquos letter until November 25mdashnearly a month later Even if that response had been given in a reasonable time it nonetheless did not provide assurance of due performance It simply stated ldquoIrsquom sorry about the strike but it is really out of my hands I hope we settle it soon so that we can get back to workrdquo Therefore the builderrsquos November 25 response did not provide adequate assurance in response to the sailorrsquos justified request Thus the builder had repudiated the contract

Point Two(b) (35) Although the builder repudiated the contract with the sailor the builder probably retracted that repudiation on December 3 and the sailor was no longer entitled to cancel their contract Thus the sailorrsquos failure to perform the sailorrsquos obligations under the contract constituted a breach

The builderrsquos failure to provide adequate assurance of performance constituted a repudiation of their contract (see UCC sect 2-609(4)) but the builder was free to retract that repudiation until the sailor cancelled the contract or materially changed his position or indicated by communication or action that the sailor considered the repudiation to be final See UCC sect 2-611(1) (ldquoUntil the repudiating partyrsquos next performance is due he can retract his repudiation unless the aggrieved party has since the repudiation cancelled or materially changed his position or otherwise indicated that he considers the repudiation finalrdquo)

Here the facts state that before the builderrsquos December 3 telephone call to the sailor the sailor did nothing in response to the builderrsquos repudiation such as contracting with a third party for a boat The builderrsquos December 3 call informing the sailor that the boat would be timely delivered probably constituted a retraction of the repudiation because it clearly indicated to the sailor that the builder would be able to perform UCC sect 2-611(2) Thus after being so informed the sailor did not have the right to treat their contract as cancelled UCC sect 2-611(3) Accordingly the sailorrsquos failure to perform the sailorrsquos obligations under the contract by taking the boat and paying for it constituted a breach of the contract

20

CONSTITUTIONAL LAW ANALYSIS (Constitutional Law IVA F2b amp e)

ANALYSIS

Legal Problems

(1) Does AutoCorsquos operation of a ldquocompany townrdquo result in its actions counting as those of the state for purposes of constitutional analysis

(2) Does the expulsion of a schoolchild for failure to recite the Pledge of Allegiance violate the First Amendment as applied through the Fourteenth Amendment

(3) Does the arrest of a pamphleteer in connection with violation of an anti-littering rule where the littering is done by the recipients of leaflets distributed by the pamphleteer violate the First Amendment as applied through the Fourteenth Amendment

DISCUSSION

Summary

The First Amendment as applied through the Fourteenth Amendment applies only to state action It does not typically govern private actors However courts have found state action where the private actor has exercised a ldquopublic functionrdquo such as running a privately owned ldquocompany townrdquo as AutoCo has done here Thus First Amendment protections apply By requiring the son to participate in a mandatory Pledge of Allegiance ceremony AutoCo has compelled the expression of political belief in violation of the First Amendment as applied through the Fourteenth Amendment The fatherrsquos arrest in connection with breaching the anti-litter rule also violated the First Amendment as applied through the Fourteenth Amendment Although state actors can regulate the incidental effects of speech on the public streets on a content-neutral basis this power is limited and cannot extend to punishing a distributor of literature because of littering by third parties

Point One (30) AutoCorsquos operation of a company town (including a school) makes it a state actor under the public function strand of the state action doctrine

The individual rights protections of the Constitution apply only where there is ldquostate actionrdquomdash either direct action by the government or some action by a private party that is fairly attributable to the government As a general rule the actions of a private company like AutoCo or of a private school like the school operated by AutoCo would not constitute state action and the protections of the Constitution (in this case the First Amendment) would not apply

However there are situations in which the actions of a private actor are attributed to the state One such situation is when the private actor undertakes a public function There are not many bright-line rules in the Supreme Courtrsquos state action doctrine but one of them is this Where a private actor undertakes a ldquopublic functionrdquo the Constitution applies to those actions Where a corporation operates a privately owned ldquocompany townrdquo that provides essential services typically provided by a state actor the public function doctrine applies and the Constitution

21

Constitutional Law Analysis

binds agents of the town as if they were agents of the government See eg Marsh v Alabama 326 US 501 (1946) Here AutoCo does more than own the town it provides security services fire protection sanitation services and a school Thus the actions of AutoCo constitute state action and are governed by the Fourteenth Amendment

Point Two (35) The sonrsquos expulsion for failure to recite the Pledge of Allegiance violates the First Amendment as applied through the Fourteenth Amendment as a compelled expression of political belief

As explained in Point One the First Amendment applies to the school as a state actor Although children in public schools (and in schools subject to the First Amendment like

the Oakwood school) have some First Amendment rights Tinker v Des Moines Independent Community School District 393 US 503 506 (1969) schools have greater leeway to regulate the speech of students and teachers than the state would have outside the school context Hazelwood School Dist v Kuhlmeier 484 US 260 (1988) Morse v Frederick 551 US 393 (2007) However the Supreme Court has long held that public schools may not force their students to participate in a flag salute ceremony when it offends the political or religious beliefs of the students or their families West Virginia Board of Educ v Barnette 319 US 624 (1943) (invalidating a mandatory public school flag salute ceremony) see also Wooley v Maynard 430 US 705 (1977) (invalidating compelled expression of political belief on state-issued license plates)

In this case the school requires its students to participate in a flag salute and Pledge of Allegiance ceremony and punishes them when they refuse to participate Pursuant to this policy the school has expelled the son This expulsion violates the First Amendment ban on compelled expression

Point Three (35) Because the father was distributing leaflets in a traditional public forum his trespass arrest violated the First Amendment as applied through the Fourteenth Amendment

As explained in Point One AutoCo is treated as a state actor Thus Oakwoodrsquos commercial district is treated as government-owned property for purposes of the First Amendment Thus the leafleting here is subject to the First Amendment because it is an expressive activity Schneider v State of New Jersey Town of Irvington 308 US 147 (1939) When expression takes place on government-owned property government regulation of the expression is assessed under the public forum doctrine Public streets and sidewalks have long been held to be the classic example of a ldquotraditional public forumrdquo open to the public for expression Hague v CIO 307 US 496 515ndash16 (1939) Because the father was distributing leaflets while standing on a street corner in the commercial district his expressive activity occurred in a traditional public forum

When a state tries to regulate expressive activity in a traditional public forum it is prohibited from doing so based on the expressive activityrsquos content unless its regulation is narrowly tailored to achieve a compelling governmental interest (ldquostrict scrutinyrdquo) In this case however AutoCo is regulating the fatherrsquos expressive activity on the ostensibly neutral ground that his expressive activity has produced litter and made the street unsightly When a state tries to regulate expressive activity without regard to its content intermediate scrutiny applies Under intermediate scrutiny the true purpose of the regulation may not be the suppression of ideas (if so then strict scrutiny applies) the regulation must be narrowly tailored to achieve a significant

22

Constitutional Law Analysis

governmental interest and it must leave open ample alternative channels for expressive activity Ward v Rock Against Racism 491 US 781 791 (1989)

Here the application of the ordinance to the father will fail for two reasons First the Supreme Court has held that the governmentrsquos interest in keeping the streets clean is insufficient to ban leafleting in the public streets as the government power to regulate with incidental effects on public sidewalk speech is very limited See eg Schneider 308 US at 162 (leafletinglittering) Second the regulation (a blanket ban on distribution that results in littering) is not narrowly tailored to protect expression A narrowly tailored alternative would be prosecution only of people who litter Moreover the effect of the littering rule is likely to be a ban on all leafleting thus eliminating an entire class of means of expression This raises the possibility that there are not ldquoample alternative channels of communicationrdquo open to the father as required under the Courtrsquos standard of review for content-neutral regulation of speech

[NOTE Some examinees might argue that this is a ldquotime place and mannerrdquo restriction and that AutoCo might have greater latitude to regulate the public sidewalks under this theory This argument is incorrect for two reasons First the Supreme Court has held that the power to regulate speakers through littering laws is very limited for the reasons given and in the cases cited above But more generally a ldquotime place and mannerrdquo restriction involves the shifting of speech from one time and place to another or to another manner here there is no shifting but a direct punishment for expressive activity (albeit one couched in content-neutral terms) In addition some examinees might read the ordinance to be in effect a total ban on leafleting since most leafleting will produce some litter Those examinees might note that the Court has required total bans on an entire mode of expression to satisfy strict scrutiny and analyze the fatherrsquos prosecution here accordingly See United States v Grace 461 US 171 177 (1983) (invalidating ban on display of signs on public sidewalks surrounding US Supreme Court ldquo[a]dditional restrictions such as an absolute prohibition on a particular type of expression will be upheld only if narrowly drawn to accomplish a compelling governmental interestrdquo)]

23

SECURED TRANSACTIONS ANALYSIS (Secured Transactions IID E IVA B C)

ANALYSIS

Legal Problems

(1) Is a purchase-money security interest in consumer goods perfected even though there has been no filing of a financing statement

(2) Does a person who buys consumer goods for personal use take those goods free of a prior perfected purchase-money security interest in the goods

(3) Does a person who receives consumer goods as a gift take those goods subject to a prior perfected security interest in them

DISCUSSION

Summary

The retailerrsquos security interest in the bicycles was perfected even though no financing statement was filed because it was a purchase-money security interest in consumer goods A purchase-money security interest in consumer goods is automatically perfected upon attachment

The buyer is not subject to the retailerrsquos security interest in the bicycle that the buyer bought from the man Because the bicycle was consumer goods in the hands of the man and the retailer never filed a financing statement covering the bicycle the retailerrsquos security interest is not effective against someone like the buyer who bought the bicycle for value without knowledge of the retailerrsquos security interest and for personal use

On the other hand the retailerrsquos security interest continues in the bicycle given to the friend because the friend did not give value for the bicycle or buy it in the ordinary course of business

Point One (35) The retailerrsquos security interest in the bicycles attached on June 1 Because this interest was a purchase-money security interest in consumer goods it was automatically perfected when it attached

The retailerrsquos security interest in the bicycles attached on June 1 when the man bought the bicycles (acquiring rights in the collateral) signed a security agreement containing a description of the collateral and received value from the retailer (by being given credit with which to purchase the bicycles) UCC sect 9-203(a) amp (b)

Despite the retailerrsquos failure to file a financing statement its security interest was perfected Pursuant to UCC sect 9-309(1) a security interest is automatically perfected upon attachment if the goods are ldquoconsumer goodsrdquo and the security interest is a ldquopurchase-money security interestrdquo

In this case the bicycles sold by the retailer to the man were consumer goods at the time of sale The bicycles were ldquogoodsrdquo because they were ldquomovable when a security interest

24

Secured Transactions Analysis

attachesrdquo UCC sect 9-102(a)(44) They were also consumer goods because they were ldquobought for use primarily for personal family or household purposesrdquo UCC sect 9-102(a)(23) The retailerrsquos security interest in these consumer goods was also a ldquopurchase-money security interestrdquo A purchase-money security interest is an interest that secures a debt that was incurred in order to ldquoenable the debtor to acquire rights in or the use of the collateralrdquo UCC sect 9-103(a) (b)(1) Here the man incurred an obligation to the retailer to purchase the bicycles so the security interest he gave the retailer to secure that obligation was a purchase-money security interest

Because the retailerrsquos security interest was a purchase-money security interest in consumer goods it was automatically perfected on June 1 when the interest attached to the bicycles

Point Two (35) The buyer took the bicycle free of the retailerrsquos security interest because (i) the retailer did not file a financing statement covering the bicycle (ii) the bicycle was ldquoconsumer goodsrdquo and (iii) the buyer bought the bicycle for value without knowledge of the retailerrsquos security interest and for personal use

A security interest continues in collateral even after a sale or other disposition of that collateral unless the creditor authorized the disposition ldquofree of the security interestrdquo or another Article 9 exception applies UCC sectsect 9-201(a) and 9-315(a)(1)

However a buyer of goods like the buyer here can take free of a prior security interest in those goods under certain circumstances See UCC sectsect 9-317(b) (buyers who give value and receive delivery of goods without knowledge of an unperfected security interest in the goods) and 9-320(a) amp (b) (buyer in ordinary course of business buyer of consumer goods in a consumer-to-consumer transaction who gives value) In this case the retailerrsquos security interest was perfected when the buyer purchased the bicycle so UCC sect 9-317(b) does not protect the buyer The buyer also is not a protected ldquobuyer in ordinary course of businessrdquo because he did not purchase from a person who is in the business of selling bicycles See UCC sect 1-201(b)(9)

The buyer can however qualify for the protection of UCC sect 9-320(b) That section provides that a buyer of goods from a person who used them for personal family or household purposes takes free of a perfected security interest in the goods if (1) the buyer had no knowledge of the security interest (2) the buyer gave value for the goods (3) the buyer purchased the goods primarily for personal family or household purposes and (4) the purchase occurred before the filing of a financing statement covering the goods

The buyer met all of these criteria The man used the bicycle for personal purposes The buyer purchased the bicycle from the man and the buyer had no knowledge of the retailerrsquos security interest The buyer gave value ($400) for the bicycle and he bought it ldquoprimarily for personal family or household purposesrdquo as he planned to use it for recreation which is a personal rather than a business use Finally no financing statement had been filed Therefore under UCC sect 9-320(b) the buyer took free of the retailerrsquos security interest

Point Three (30) The retailerrsquos security interest continues in the bicycle that the man gave to the friend Thus the retailer can recover the bicycle from the friend because the friend did not give value for the bicycle or buy it in the ordinary course of business

25

Secured Transactions Analysis

As noted in Point Two the retailer did not authorize the man to dispose of the bicycle Consequently the retailerrsquos security interest continued in the bicycle even after the man transferred ownership of the bicycle to the friend See UCC sectsect 9-201(a) and 9-315(a)(1) The retailerrsquos security interest in the bicycle will be effective against the friend unless some other provision of Article 9 allows the friend to take the bicycle free of that security interest

Unfortunately for the friend there is no Article 9 provision that allows him to take free of the retailerrsquos interest The friendrsquos basic problem is that he is not a buyer of the bicyclemdashhe received the bicycle as a gift and did not give value for it Thus the friend is not protected by any of the applicable exceptions See UCC sectsect 9-317(b) (protecting buyers who give value for goods subject to an unperfected security interest) 9-320(a) (protecting buyers in ordinary course of business) and 9-320(b) (protecting buyers of consumer goods who give value)

In short the retailerrsquos security interest continues in the bicycle that the man gave to the friend The friend took the bicycle subject to that security interest

26

FEDERAL CIVIL PROCEDURE ANALYSIS (Federal Civil Procedure VIE)

ANALYSIS

Legal Problems

(1) Does a judgment in a prior action preclude a nonparty from suing the same defendant on a closely related claim when the nonparty and the original plaintiff are in a family relationship

(2) Does a judgment rendered in an earlier action preclude a nonparty from litigating an issue that was actually decided in the first suit

(3) May a nonparty to an earlier action invoke the judgment in that action to preclude a party to the prior action from relitigating an issue that the party had a full and fair opportunity to litigate in the earlier action

DISCUSSION

Summary

Pursuant to the doctrines of claim preclusion (res judicata) and issue preclusion (collateral estoppel) a judgment is binding on the parties thereto In the absence of privity nonparties to a prior suit cannot be bound by a judgment rendered in their absence Thus in the absence of privity a nonparty to the first suit is not precluded from presenting her claim in a second suit even if it is factually related to the claims and defenses presented in the first suit nor is she bound by determinations of issues made in the first suit A family relationship without more does not support a finding of privity For this reason Mother as a nonparty is not bound by the judgment in the Son-Driver action She may bring her separate claim for damage to her car and she is not precluded from litigating the question of whether she was negligent in the maintenance of her car

Driver on the other hand could be precluded from relitigating the issue of her negligence pursuant to the doctrine of non-mutual issue preclusion (also called non-mutual offensive collateral estoppel) which allows a nonparty to a prior action to invoke issue preclusion to prevent a party to that prior action from relitigating determinations of issues made therein However Mother may be prevented from invoking non-mutual collateral estoppel in this case because she could easily have joined her claim in the prior action but did not do so

[NOTE Federal common law governs the preclusive effect of a judgment rendered by a federal court sitting in diversity See Semtek Intrsquol Inc v Lockheed Martin Corp 531 US 497 508 (2001) But the Semtek Court concluded that federal common law in this context incorporates the preclusion law of the state in which the rendering federal court sits (unless the state law is incompatible with federal interests) id at 508ndash09 Thus State Arsquos preclusion law determines the preclusive effect of the judgment rendered in Sonrsquos suit against Driver The problem says that State A preclusion law is identical to federal preclusion law so the following analysis utilizes general principles of preclusion drawn from Supreme Court case law (announcing federal preclusion rules) and the Restatement (Second) of Judgments]

27

Federal Civil Procedure Analysis

Point One (35) Under the doctrine of claim preclusion the judgment rendered in the first action does not preclude Mother a nonparty from suing Driver for the damage to her car because the judgment binds only parties or those in privity with them and Mother and Son are not in privity

Driver may contend that the doctrine of claim preclusion (res judicata) precludes Mother from presenting a claim arising from the same nucleus of facts that was presented in the first action brought by Son According to the doctrine of claim preclusion ldquowhen a court of competent jurisdiction has entered a final judgment on the merits of a cause of action the parties to the suit and their privies are thereafter bound lsquonot only as to every matter which was offered and received to sustain or defeat the claim or demand but as to any other admissible matter which might have been offered for that purposersquordquo Commissioner of Internal Revenue v Sunnen 333 US 591 597 (1948) (citation omitted)

However the doctrine of claim preclusion does not apply to Mother on the facts of this problem First Mother was not a party to the earlier case ldquoIt is a principle of general application in Anglo-American jurisprudence that one is not bound by a judgment in personam in a litigation in which he is not designated as a party or to which he has not been made a party by service of processrdquo Taylor v Sturgell 553 US 880 884 (2008) (citing Hansberry v Lee 311 US 32 40 (1940)) see also RESTATEMENT (SECOND) OF JUDGMENTS sect 34(3) (1982) This rule reflects our ldquodeep-rooted historic tradition that everyone should have his own day in courtrdquo Martin v Wilks 490 US 755 762 (1989) (citation omitted) (superseded by statute on other grounds) Since Mother was not a party to the first suit she is not bound by the judgment unless an exception to the general rule applies

Mother might be bound by the prior judgment if she were considered to have been sufficiently in privity with Son that Son represented her interests in that action ldquoA person who is not a party to an action but who is represented by a party is bound by and entitled to the benefits of a judgment as though he were a partyrdquo RESTATEMENT (SECOND) OF JUDGMENTS sect 41(1) But there is no suggestion in the facts of the problem that Son who is an adult purported to represent Motherrsquos interests in the first suit ldquo[C]lose family relationships are not sufficient by themselves to establish privity with the original suitrsquos party or to bind a nonparty to that suit by the judgment entered therein rdquo Cuauhtli v Chase Home Finance LLC 308 Fed Appx 772 773 (5th Cir 2009) (citation omitted) accord 18A CHARLES ALAN WRIGHT ET AL FEDERAL

PRACTICE AND PROCEDURE sect 4459 (2d ed 2002) In Taylor v Sturgell supra the Supreme Court identified other special circumstances in

which nonparties may be bound by a prior judgmentmdashwhen a nonparty consents to be bound when a nonparty is in a pre-existing substantive legal relationship with a party (such as preceding and succeeding property owners) when a nonparty assumed control of the prior litigation when a party seeks to relitigate through a proxy or where a special statutory scheme seeks to foreclose successive litigation by nonparties See Taylor 553 US at 893ndash95 None of these circumstances exists here

Because Mother was not a party to the first suit and is not in privity with Son who is an adult the judgment in the first action does not preclude her from bringing her own claim against Driver

Point Two (35) Under the doctrine of issue preclusion the judgment rendered in the first action does not preclude Mother a nonparty from litigating the issue of her negligence in maintaining her carrsquos

28

Federal Civil Procedure Analysis

brake lights because the judgment binds only parties or those in privity with them and Mother and Son are not in privity

By its affirmative response to a special interrogatory the jury in the first action expressly concluded that ldquoMother negligently failed to ensure that the brake lights on her car were in proper working orderrdquo Driver may attempt to invoke the doctrine of issue preclusion to preclude Mother from relitigating this issue in the second action

[I]ssue preclusion arises in a second action on the basis of a prior decision when the same lsquoissuersquo is involved in both actions the issue was lsquoactually litigatedrsquo in the first action after a full and fair opportunity for litigation the issue was lsquoactually decidedrsquo in the first action by a disposition that is sufficiently lsquofinalrsquo lsquoon the meritsrsquo and lsquovalidrsquo it was necessary to decide the issue in disposing of the first action and the later litigation is between the same parties or involves nonparties that are subject to the binding effect or benefit of the first action Once these requirements are met issue preclusion is available not only to defend against a demand for relief but also as offensive support for a demand for relief Issue preclusion moreover is available whether or not the second action involves a new claim or cause of action

18 CHARLES ALAN WRIGHT ET AL FEDERAL PRACTICE AND PROCEDURE sect 4416 at 392ndash93 (2d ed) see also RESTATEMENT (SECOND) OF JUDGMENTS sect 27 (1982)

Here several of the elements necessary for issue preclusion are present The same issue is involved in both actionsmdashthe issue of Motherrsquos negligence in failing to maintain the brake lights on her car That issue was actually litigated in the first action and decided by the jury There is nothing to suggest anything less than a full and fair opportunity to litigate The judgment disposing of the issue was final

Nevertheless the judgment will not preclude Mother from relitigating the issue for two reasons First Mother was not a party to the first action and as explained above Mother and Son are not in privity Therefore she cannot be denied an opportunity to litigate the issue of her negligence Second it does not appear that the juryrsquos decision as to Motherrsquos negligence was necessary to the prior judgment against Driver Nothing suggests that the finding on Motherrsquos negligence had any bearing on the outcome of the first action

Point Three (30) Under the doctrine of non-mutual issue preclusion the judgment rendered in the first action might preclude Driver from relitigating the issue of her negligence However Driver has a strong argument that such a result would be inconsistent with the policy against offensive use of non-mutual estoppel when the non-party plaintiff easily could have joined as a plaintiff in the first action

Because Son already convinced the jury in the first action that ldquoDriver was negligent in the operation of her vehiclerdquo Mother may wish to invoke the doctrine of non-mutual issue preclusion to prevent Driver from relitigating the question of her negligence As noted above ldquoissue preclusion arises in a second action on the basis of a prior decision when the same lsquoissuersquo is involved in both actions the issue was lsquoactually litigatedrsquo in the first action after a full and fair opportunity for litigation the issue was lsquoactually decidedrsquo in the first action by a disposition that is sufficiently lsquofinalrsquo lsquoon the meritsrsquo and lsquovalidrsquo it was necessary to decide the issue in disposing of the first action rdquo 18 CHARLES ALAN WRIGHT ET AL FEDERAL PRACTICE AND

PROCEDURE sect 4416 at 392 (2d ed) see also RESTATEMENT (SECOND) OF JUDGMENTS sect 27

29

Federal Civil Procedure Analysis

Here these basic requirements for issue preclusion are met First the same issue is involved in both suits whether Driver was negligent in the operation of her car Second this issue was actually litigated and decided in the first action the jury answered a special interrogatory raising this very question There is nothing to suggest that Driver lacked a full and fair opportunity to litigate the issue Since a judgment was rendered against Driver for the injuries Son sustained as a result of Driverrsquos negligence resolution of the issue was necessary to dispose of the first action Driver was a party to the first action so she may be bound by the judgment

[NOTE Traditionally issue preclusion required mutualitymdashboth the party asserting issue preclusion and the party against whom issue preclusion was asserted were bound by the prior judgment Under the traditional mutuality rule Mother could not assert issue preclusion against Driver because Mother would not be bound by the judgment if Driver sought to rely on it See Point One There is no mutuality between Mother and Driver with respect to the prior judgment

This traditional mutuality requirement has been abandoned in most jurisdictions The Supreme Court rejected a strict mutuality requirement in Blonder-Tongue Laboratories Inc v University of Illinois Foundation 402 US 313 (1971) (non-mutual defensive collateral estoppel used by a defendant to preclude a plaintiff from relitigating a claim the plaintiff previously litigated) and Parklane Hosiery Co v Shore 439 US 322 (1979) (non-mutual offensive collateral estoppel used by a plaintiff to preclude a defendant from relitigating a claim the defendant previously litigated) In Parklane Hosiery the Court concluded (as a matter of federal preclusion law) that trial courts should have ldquobroad discretionrdquo to determine whether or not to permit a plaintiff to invoke non-mutual issue preclusion ldquoThe general rule should be that in cases where a plaintiff could easily have joined in the earlier action or where the application of offensive estoppel would be unfair to a defendant a trial judge should not allow the use of offensive collateral estoppelrdquo Id at 331

The Parklane Hosiery decision identified a number of circumstances that might make it unfair to allow a plaintiff to invoke non-mutual issue preclusion (non-mutual offensive collateral estoppel in the traditional terminology) against a defendant In particular the Parklane Hosiery court suggested that issue preclusion may not be appropriate if the plaintiff in the second action ldquocould easily have joined in the earlier actionrdquo Id Prohibiting plaintiffs from using non-mutual estoppel under such circumstances would promote judicial efficiency by encouraging plaintiffs to join the prior action It would also discourage plaintiffs from staying out of prior litigation in order to secure in effect two bites at the apple using the prior litigation offensively if the defendant loses and forcing the defendant to litigate a second time if the defendant wins the prior action

An exceptional exam answer might therefore argue that non-mutual issue preclusion should be denied on these facts Son and Mother both reside in State A since they are related they know each other well and Son was driving Motherrsquos car when the accident occurred They could have sued together and Rule 20 of the Federal Rules of Civil Procedure would have authorized joinder of their claims because those claims arose from the same transaction or occurrence and raised a common question of law or fact FED R CIV P 20(a) The facts do not suggest that Mother had any reason not to join Sonrsquos suit other than a desire to see how Sonrsquos action concluded before bringing her own claim Cf Nations v Sun Oil Co (Del) 695 F2d 933 938 (5th Cir 1983) (concluding that plaintiff ldquowas entitled to await the development of his injuries and their predictable consequencesrdquo) Because it appears that Mother may be a ldquowait-and-seerdquo plaintiff who could easily have joined the original action a trial court might disallow as a matter of discretion her use of non-mutual issue preclusion]

30

AGENCY ANALYSIS __________ (Agency I II)

ANALYSIS

Legal Problems

(1) Is the principal or the agent or both liable on contracts with a third party when the principal is an ldquoundisclosed principalrdquo

(2) Is the principal or the agent or both liable on contracts with a third party when the principal is ldquopartially disclosedrdquo or an ldquounidentified principalrdquo

(3) Is the principal or the agent or both liable on contracts with a third party for the purchase of goods when the agent exceeded his authority but the principal nonetheless accepts the goods

DISCUSSION

Summary

The agent but not the owner is liable to the basket manufacturer because the owner is an undisclosed principal and the agent acted without actual or apparent authority Both the agent and the owner however are liable on the burner contract because the owner is an unidentified principal and the agent had apparent authority to enter into that contract With respect to the solar cells contract whether the owner is liable depends upon whether a court would follow the Second or Third Restatement of Agency which take different positions on the effect of the ratification of a contract by an undisclosed principal Under either the agent would also be liable on the contract as he was a party to the contract

[NOTE The contracts that are the subject of this question are contracts for the sale of goods and therefore are governed by Article 2 of the Uniform Commercial Code Article 2 however does not contain agency rules Accordingly common law concepts of agency are applicable UCC sect 1-103(b)]

Point One (35) The agent but not the owner is liable to the basket manufacturer The agent had no actual authority to enter into the contract to buy aluminum baskets and because the owner was an undisclosed principal the manufacturer had no reason to believe that the agent had apparent authority Furthermore the manufacturer had no reason to believe that the agent was not contracting for his own benefit

An agent acting on behalf of a principal can bind the principal to contracts if the agent has either actual or apparent authority An agent has actual authority when contracting on behalf of his principal if he ldquoreasonably believes in accordance with the principalrsquos manifestations to the agent that the principal wishes the agent so to actrdquo RESTATEMENT (THIRD) OF AGENCY sect 201 (2006) Here the agent was told to buy only wicker baskets not aluminum baskets Thus when he contracted with the basket manufacturer to buy aluminum baskets he had no actual authority to do so

31

Agency Analysis

An agent acts with apparent authority ldquowhen a third party [with whom the agent acts] reasonably believes the actor has authority to act on behalf of the principal and that belief is traceable to the principalrsquos manifestationsrdquo Id sect 203 Here the owner notified basket manufacturers that she or her agent might contact them to purchase baskets but that notification did not specifically name the agent or any other person as the ownerrsquos agent Furthermore the basket manufacturer had no prior dealings with the agent or the owner or any reason to think that the agent was acting for the benefit of anyone but himself Thus there is no basis to conclude that the basket manufacturer thought that the agent had apparent authority to act for the owner

Generally when an agent acts on behalf of an undisclosed principal and the agent lacks authority to enter into the contract the agent is liable on the contract as a party to the contract but the principal is not liable This rule is consistent with the third partyrsquos expectations ldquoThe third party expected the agent to be a party to the contract because the agent presented the deal as if he were acting for himself Moreover if the third party is unaware of the principalrsquos existence the third party must be relying on the agentrsquos solvency and reliability when entering into the contractrdquo See ROBERT W HAMILTON JONATHAN R MACEY amp DOUGLAS K MOLL CORPORATIONS INCLUDING PARTNERSHIPS AND LIMITED LIABILITY COMPANIES 34 (11th ed 2010) See also RESTATEMENT (THIRD) OF AGENCY sect 603 cmt c Furthermore because the third party has no idea that the agent is acting or is seemingly acting on behalf of another there is no reason to believe that the third party would be expecting an undisclosed principal to be liable on the contract Id

Point Two (35) Because the owner is an unidentified (as opposed to undisclosed) principal both she and the agent (as a party to the contract) probably are liable on the contract with the burner manufacturer

When the agent contracted with the burner manufacturer he did not have actual authority to do so as the owner had expressly restricted the agentrsquos authority to purchase only burners with ldquowhisper technologyrdquo See Point One However the agent may have had apparent authority to buy burners without whisper technology

An agent acts with apparent authority ldquowhen a third party [with whom the agent acts] reasonably believes the actor has authority to act on behalf of the principal and that belief is traceable to the principalrsquos manifestationsrdquo RESTATEMENT (THIRD) OF AGENCY sect 203 (2006) The owner indicated that an agent might contact the burner manufacturer The notice contained no restriction regarding the type of burners that the agent was authorized to purchase The facts indicate that burner manufacturers regularly receive such notices

Although the agent told the burner manufacturer that he represented a well-known hot-air balloon operator he did not disclose the ownerrsquos name Thus the owner was a partially disclosed or unidentified principal See RESTATEMENT (SECOND) OF AGENCY sect 4(2) (1958) (using term ldquopartially disclosed principalrdquo) RESTATEMENT (THIRD) OF AGENCY sect 104(2)(c) (2006) (using term ldquounidentified principalrdquo) An agent for a partially disclosed principal may have apparent authority RESTATEMENT (SECOND) OF AGENCY sect 159 cmt e (1958) Based upon (1) the notice sent by the owner (2) the agentrsquos revelation that he was acting as an agent and (3) the fact that burner manufacturers regularly receive such notices and sell to agents the manufacturer may argue that it reasonably and actually believed that the agent was authorized to purchase burners without whisper technology The manufacturer may also argue that because the agent revealed that he was an agent his listing of the ownerrsquos address as the delivery address connects the agent to the notice given by the owner Arguably this distinguishes the burner contract from the basket

32

Agency Analysis

contract Here there is a strong case to support the conclusion that the agent had apparent authority if he did then the owner is liable to the burner manufacturer

The agent also is liable as a party to the contract because he did not fully disclose his agency relationship Although he told the burner manufacturer that he represented a well-known hot-air balloon operator he did not disclose the ownerrsquos name Generally even an authorized agent of a partially disclosed or unidentified principal is liable as a party to a contract with a third person RESTATEMENT (SECOND) OF AGENCY sect 321 (1958) (ldquounless otherwise agreedrdquo) RESTATEMENT (THIRD) OF AGENCY sect 602(2) (2006) (ldquounless the agent and the third party agree otherwiserdquo)

Point Three (30) Under the Second Restatement of Agency the owner is not liable on the contract for solar cells because the agent did not have actual or apparent authority and the owner as an undisclosed principal cannot ratify the contract Under the Third Restatement the owner could be liable as she ratified the contract Under either Restatement the agent is liable as a party to the contract

The owner is not liable to the solar cell manufacturer for breach of the contract for the solar cells because the agent had no actual or apparent authority to purchase solar cells on the ownerrsquos behalf and the owner under the Second Restatement of Agency did not ratify the contract with knowledge of the material facts Thus she is not liable as a ratifier of the contract

The facts state that the agent had authority to purchase only propane fuel tanks In addition he had no apparent authority to purchase solar cells The owner made no manifestations to the solar cell manufacturer that would lead a reasonable person in the manufacturerrsquos position to believe that the agent had the authority to bind the owner to a contract to purchase solar cells In fact the agent made no manifestations at all to the solar cell manufacturer Unlike with the basket manufacturer and the burner manufacturer the owner did not notify the manufacturer of solar cells that an agent might contact it to purchase solar cells In addition the solar cells were delivered to the agent and not to the ownerrsquos address In sum the manufacturer was unaware of any relationship between the owner and the agent As to the solar cell manufacturer the owner is an undisclosed principal There can be no apparent authority in the case of an undisclosed principal because there are no manifestations from the principal to the third person See RESTATEMENT (SECOND) OF AGENCY sect 8 cmt a (1958) (ldquothere can be no apparent authority created by an undisclosed principalrdquo) RESTATEMENT (THIRD) OF AGENCY sect 203 cmt f (2006) (ldquoapparent authority is not present when a third party believes that an interaction is with an actor who is a principalrdquo)

The owner also did not ratify the contract Although the owner used the solar cells generally a principal cannot ratify an unauthorized transaction with a third person ldquounless the one acting purported to be acting for the ratifierrdquo RESTATEMENT (SECOND) OF AGENCY sect 85(1) (1958)

The result differs under the Third Restatement which expressly rejects the Second Restatement on this issue The Restatement (Third) of Agency sect 403 (2006) states ldquoA person may ratify an act if the actor acted or purported to act as an agent on the personrsquos behalfrdquo According to comment b ldquoan undisclosed principal may ratify an agentrsquos unauthorized actrdquo Under the Restatement (Third) of Agency rule the owner probably ratified the transaction The agent clearly acted on the ownerrsquos behalf and in addition the ownerrsquos conduct in using the solar cells ldquojustifies a reasonable assumption that [she] is manifesting assent that the act shall affect [her] legal relationsrdquo See id sect 401(2)

33

Agency Analysis

The agent also is liable to the solar cell manufacturer for breach of the contract for the solar cells because he is a party to the contract The facts indicate that the agent never told the solar cell manufacturer that he represented the owner or any other principal Consequently even if the agent were authorized (which as discussed above he is not) he would be liable as a party to the contract See RESTATEMENT (SECOND) OF AGENCY sect 322 (1958) RESTATEMENT (THIRD) OF AGENCY sect 603(2) (2006) Here he has no authority or apparent authority and is liable as a party to the contract

The agent would also be liable under the Third Restatement Under Restatement (Third) of Agency sect 402(1) (2006) ratification generally relates back and the transaction is treated as if it were authorized at the time of the transaction However this does not relieve the agent of an undisclosed principal who ratifies an unauthorized transaction of liability under the ratified contract See id sect 603(2) (authorized agent for undisclosed principal is a party to the contract) and sect 403 cmt b (ldquoAn undisclosed principalrsquos ratification does not eliminate the agentrsquos liability to the third party on the transaction rdquo)

[NOTE An examinee may discuss the concept of inherent agency power This concept is recognized by the Restatement (Second) of Agency sect 8 A (1958) but the concept is not used in the Restatement (Third) of Agency (2006) Here there are no facts to support that the agent had inherent authority

As to contracts with agents for partially disclosed principals (eg the contract for the burners) the basic question is whether the acts done ldquousually accompany or are incidental to transactions which the agent is authorized to conductrdquo RESTATEMENT (SECOND) OF AGENCY

sect 161 (1958) If so the principal is bound if the other party ldquoreasonably believes that the agent is authorized to do them and has no notice that he is not so authorizedrdquo Id The purchase of burners without whisper technology was not authorized nor was it incidental to an authorized transaction Therefore there should not be inherent agency power

As to contracts on behalf of undisclosed principals (eg the other two contracts) the basic question is whether the acts done are usual or necessary in the transactions the agent is authorized to transact RESTATEMENT (SECOND) OF AGENCY sect 194 (1958) The other two contracts seem fundamentally different from the authorized transactions Therefore there should not be inherent agency power

Only minimal credit should be given for discussion of inherent agency power]

34

EVIDENCE ANALYSIS _____ (Evidence IIA VA B E F J K)

ANALYSIS

Legal Problems

(1) Is the authenticated copy of the mechanicrsquos text message relevant and admissible

(2) Is the womanrsquos question ldquoIs my scooter safe to drive for a whilerdquo relevant and admissible

(3) Is the womanrsquos testimony describing the mechanicrsquos thumbs-up relevant and admissible

DISCUSSION

Summary

The mechanicrsquos text message to the woman is relevant to whether (1) the woman lost control of the scooter due to its defective brakes (2) the woman knew that the brakes needed repair and (3) it was negligent for the woman to drive the scooter knowing that its brakes needed repair

The mechanicrsquos text message is hearsay if it is offered by the pedestrian to prove that the scooterrsquos brakes needed repair However it fits the hearsay exception for present sense impressions and probably also fits the exception for business records The mechanicrsquos text message is not hearsay if it is instead offered by the pedestrian to prove the womanrsquos state of mind (ie that she had notice that her brakes needed repair)

The womanrsquos question to the mechanic and his response are also relevant to whether the brakes caused the accident and whether the woman was negligent The question is not hearsay because the woman did not make an assertion

The mechanicrsquos thumbs-up response is nonverbal conduct intended by the mechanic as an assertion and is therefore an out-of-court statement If the woman offers the mechanicrsquos statement to prove that the scooter was actually safe to ride the womanrsquos testimony about the statement is hearsay

However the mechanicrsquos statement is not hearsay if it is offered by the woman to prove her state of mind Therefore the womanrsquos question and the mechanicrsquos response are admissible to prove the womanrsquos state of mind

Point One(a) (20) The mechanicrsquos text message to the woman should be admitted because it is relevant

Evidence is relevant if it has ldquoany tendency to make a fact more or less probable than it would be without the evidencerdquo FED R EVID 401 ldquoRelevant evidence is admissiblerdquo unless it is inadmissible pursuant to some other rule FED R EVID 402

The mechanicrsquos text message to the woman ldquoWhen you pick up your scooter you need to schedule a follow-up brake repair Wersquoll order the partsrdquo is relevant for two reasons First this evidence has some tendency to make it more probable that the brakes malfunctioned and

35

Evidence Analysis

caused the accident Second it has some tendency to make it more probable that the woman was negligent in riding her scooter after being told by the mechanic that it required further repair

Point One(b) (30) The mechanicrsquos text message fits either the hearsay exception for present sense impressions or the exception for business records or it is admissible non-hearsay

The mechanicrsquos text message is a statement under Rule 801(a) because it is ldquoa written assertionrdquo FED R EVID 801(a) The text message is hearsay if the pedestrian offers it to prove the ldquotruth of the matter asserted in the statementrdquo (ie that the scooterrsquos brakes required repair) which resulted in the woman losing control of the scooter and causing the accident FED R EVID 801(c)

However the mechanicrsquos text message fits the hearsay exception for ldquopresent sense impressionsrdquo under Rule 803(1) because it is ldquo[a] statement describing or explaining an event or condition made while or immediately after the declarant perceived itrdquo FED R EVID 803(1) Here the mechanicrsquos text message described the condition of the scooter immediately after he perceived it during the maintenance service

The mechanic is a person with knowledge of the condition of the scooter so if text messages regarding repairs were made and kept by the mechanic in the ordinary course of business this text message also fits the business records exception Under Rule 803(6) a business record is a record of an act ldquomade at or near the time by someone with knowledgerdquo and ldquothe record was kept in the course of a regularly conducted activity of a businessrdquo and ldquomaking the record was a regular practice of that activityrdquo FED R EVID 803(6)

However the text message is not hearsay if it is instead offered to prove that the woman was negligent because she rode her scooter after the mechanic told her it required repair If offered for this purpose it would not be offered for the truth of the matter asserted in the statement but to show the womanrsquos belief about the condition of the scooter (her state of mind)

Point Two (10) The womanrsquos question to the mechanic should be admitted because it is not hearsay

The womanrsquos question to the mechanic is relevant because along with the mechanicrsquos thumbs-up response (see Point Three) it has some tendency to make it more probable that the woman was not negligent andor that the scooter brakes did not malfunction and cause the accident FED R EVID 401 The womanrsquos question does not raise hearsay concerns because it is not an assertion

Hearsay is defined under Rule 801(a) as ldquoan oral assertion written assertion or nonverbal conductrdquo Although ldquoassertionrdquo is not further defined ldquoa favorite [definition] of writers in the [evidence] field for at least a century and a half [is that] the word simply means to say that something is so eg that an event happened or a condition existedrdquo 2 MCCORMICK ON

EVIDENCE sect 246 (6th ed 2006) Under this definition the womanrsquos question is not hearsay because it is not an assertion

Point Three(a) (20) The mechanicrsquos thumbs-up to the woman is a nonverbal assertion that is relevant and the womanrsquos testimony about that response is admissible

36

Evidence Analysis

Hearsay is defined under Rule 801(c) as a ldquostatementrdquo that is ldquoa personrsquos oral assertion written assertion or nonverbal conduct if the person intended it as an assertionrdquo FED R EVID 801(a) Here when the mechanic responded to the womanrsquos question (ldquoIs my scooter safe to ride for a whilerdquo) with a thumbs-up gesture the facts suggest that he intended his nonverbal conduct as an assertion that in his opinion the scooter was safe to ride

The mechanicrsquos assertion is relevant and admissible to prove that the woman was not negligent because the evidence makes it more probable that at the time of the accident she believed that the scooter was safe to ride despite the fact that the brakes required repair FED R EVID 401 Admission of the womanrsquos description of the mechanicrsquos thumbs-up for this purpose does not raise hearsay concerns because the evidence would not be offered for the truth of the matter asserted but to show the womanrsquos belief about the condition of the scooter (her state of mind)

Point Three(b) (20) The mechanicrsquos thumbs-up is relevant to determine whether the scooterrsquos brakes malfunctioned causing the accident but if offered for this purpose it is also hearsay

The mechanicrsquos nonverbal assertion is relevant to the determination of whether the scooterrsquos brakes malfunctioned causing the accident However if offered to prove the ldquotruth of the matter asserted in the statementrdquo (ie that the scooter was safe to ride for a while) it is hearsay that does not fit any hearsay exception

37

TRUSTS AND FUTURE INTERESTS ANALYSIS ____________________ (Trusts and Future Interests IC1 amp 4 G IIF)

ANALYSIS

Legal Problems

(1)(a) Was the revocable trust amendable

(1)(b) If the trust was amendable must the amendment have been executed in accordance with the state Statute of Wills in order to be valid

(2) If the trust amendment was valid does the amendment apply to the probate estate assets passing to the trust pursuant to Settlorrsquos will

(3) If the trust amendment was valid should the trust property be distributed to University

(4) If the trust amendment was not valid should the trust property be distributed to Settlorrsquos grandchild (her only heir) or held in further trust in accordance with the terms of the original trust instrument

DISCUSSION

Summary

A revocable trust is amendable even if the trust instrument does not expressly grant to the trust settlor a power to amend Both inter vivos trusts and amendments thereto are valid even though not executed in accordance with the requirements applicable to wills

Under the Uniform Testamentary Additions to Trusts Act a revocable trust may be amended at any time prior to the settlorrsquos death and the amendment applies to the disposition of assets conveyed to the trust pursuant to a will even if the will was executed prior to the date of the amendment

At Settlorrsquos death trust assets including probate assets passing to the trust under Settlorrsquos will would go to University if as is the case here the trust amendment was valid If the amendment was invalid the trust assets would continue to be held in further trust because there is no violation of the common law Rule Against Perpetuities

Point One(a) (30) Settlor retained the right to amend the inter vivos trust despite her failure to expressly reserve this power

At issue here is whether a retained power of revocation includes the power to amend sometimes referred to as the power to modify The Restatement (Second) of Trusts sect 331 cmt g provides that if a settlor has a power to revoke that retained power ordinarily includes a power to modify (amend) as well Comment g also notes that the power to amend includes both a power to withdraw trust assets and a power to ldquomodify the terms of the trustrdquo The Uniform Trust Code which provides that a power to revoke includes the power to amend is consistent with this view

38

Trusts and Future Interests Analysis

UNIF TRUST CODE sect 602 accord RESTATEMENT (THIRD) OF TRUSTS sect 63 cmt The theory is that even though a power to amend was not expressly retained by a settlor the goal of amendment assuming the power was not included in the power to revoke could easily be achieved by first revoking the trust and then creating a new trust with the same terms contemplated by the amendment To require this would put form over substance

Thus by expressly retaining the power to revoke the trust Settlor retained a power to amend the inter vivos trust despite her failure to expressly reserve this power

[NOTE Under the common law a trust is irrevocable unless the settlor expressly retains a power to revoke the trust Conversely under the Uniform Trust Code a trust is revocable unless the terms of the trust expressly provide otherwise See UNIF TRUST CODE sect 602 The Trust Codersquos position on revocation follows the minority view in the United States and is inconsistent with prior Restatements of Trusts (see Restatement (Second) of Trusts sect 330) Here the trust is revocable because Settlor expressly retained a power of revocation

The Uniform Trust Code has been adopted in 24 jurisdictions Alabama Arizona Arkansas District of Columbia Florida Kansas Maine Michigan Missouri Nebraska New Hampshire New Mexico North Carolina North Dakota Ohio Oregon Pennsylvania South Carolina Tennessee Utah Vermont Virginia West Virginia and Wyoming]

Point One(b) (10) Settlorrsquos amendment of the trust was valid despite her failure to have her signature to the trust amendment witnessed

Neither the common law nor state statutes require a trust instrument or an amendment to a trust instrument to be executed in accordance with the formalities prescribed for execution of a will Indeed an inter vivos trust that does not involve real estate can be created orally Under the Uniform Trust Code the only requirements for creating a valid inter vivos trust are intent the specification of beneficiaries and the designation of a trustee See UNIF TRUST CODE sect 402 accord RESTATEMENT (THIRD) OF TRUSTS sect 13

Here the amendment meets the requirements of both the Uniform Trust Code and the common law Thus the fact that Settlorrsquos signature was not witnessed when she signed the amendment to the trust does not make the amendment invalid

Point Two (20) Under the Uniform Testamentary Additions to Trusts Act a revocable trust may be amended at any time prior to the settlorrsquos death and the amendment applies to probate assets poured into the trust at the settlorrsquos death pursuant to the settlorrsquos will even when the will was executed prior to the date of the amendment

Historically property owned by an individual at her death passed to the individualrsquos heirs or to beneficiaries designated in a will executed with the formalities (writing signing witnessing) prescribed by state law However when a will devises property to the trustee of an inter vivos trust then the provisions of the trustmdashwhich may not have been executed in accordance with the formalities required for willsmdasheffectively determine who will receive the property Because of this possibility some early cases held that if an inter vivos trust was not executed with the same formalities required for a valid will then the trust was ineffective to dispose of probate assets poured into the trust at the settlorrsquos death pursuant to the settlorrsquos will

This line of cases has been overturned by the Uniform Testamentary Additions to Trusts Act (the Act) now Uniform Probate Code sect 2-511 Under the Act adopted in almost all

39

Trusts and Future Interest Analysis

jurisdictions a testamentary bequest to the trustee of an inter vivos trust established by the testator during his or her lifetime is valid if the trust is in writing it is identified in the testatorrsquos will and the trust instrument was executed before concurrently with or after the execution of the will Id The Act further specifies that such a bequest is valid even if the trust is amendable or revocable and that a later amendment applies to assets passing to the trust by a previously executed will

Thus because the trust amendment is valid its terms apply to assets received by Bank from Settlorrsquos estate

Point Three (10) If the trust amendment was valid then the trust assets including assets passing to the trust under Settlorrsquos will should go to University

Under the trust amendment all trust assets (including the assets of Settlorrsquos probate estate poured into the trust) pass to University The facts provide no basis for failing to comply with Settlorrsquos stated intentions

Point Four (30) If the trust amendment was invalid trust assets including assets received pursuant to Settlorrsquos will should be held in accordance with the terms of the original trust instrument because those terms do not violate the Rule Against Perpetuities

Under the dispositive terms of the original trust instrument Settlor created successive income interests in her surviving children and grandchildren with a remainder interest in her great-grandchildren Because the trust was revocable the period during which the common law Rule Against Perpetuities requires that interests vest (ie 21 years plus lives in being) began to run from the date Settlor no longer had a power of revocation (here her death) not the date on which the trust was created See JESSE DUKEMINIER STANLEY J JOHANSON JAMES LINDGREN amp ROBERT SITKOFF WILLS TRUSTS AND ESTATES 678 (7th ed 2005)

Under the common law Rule Against Perpetuities Settlorrsquos trust is thus valid At the time of Settlorrsquos death she was survived by no children one granddaughter and no great-grandchildren Because Settlor cannot have more children after her death the only income beneficiary of the trust is Settlorrsquos surviving granddaughter This granddaughter is the only person who can produce great-grandchildren of Settlor thus all great-grandchildren must of necessity be born during the lifetime of Settlorrsquos only surviving granddaughter who is a life in being The granddaughterrsquos interest vested at Settlorrsquos death and the great-grandchildrenrsquos interest will vest at the death of the granddaughter There is no need to wait the additional 21 years permitted under the Rule Thus under the common law and the statute given in the facts the nonvested interest in the great-grandchildren is valid

[NOTE Both modern wait-and-see statutes and the Uniform Statutory Rule Against Perpetuities upon which the statute in the facts is modeled provide that before using either reform to validate an otherwise invalid nonvested interest one should first determine if the nonvested interest violates the common law Rule If it does not then there is no need to reform This proposition which is applicable in all MEE user jurisdictions that have not simply abrogated the rule is tested by this problem]

40

NEGOTIABLE INSTRUMENTS ANALYSIS (Negotiable Instruments III IV V)

ANALYSIS

Legal Problems

(1)(a) What rights does a person in possession of a note that has been indorsed in blank by the payee have against the maker of the note

(1)(b) Which defenses may the maker of a note raise against a person entitled to enforce it who is not a holder in due course but is a transferee from a holder in due course

(2) What rights does a person entitled to enforce a note have against an indorser who transferred it for consideration with no warranties

(3) What rights does a person entitled to enforce a note have against a previous holder who transferred it as a gift without indorsing it

DISCUSSION

Summary

The niece is a holder of the note and is thus a person entitled to enforce it The chef the issuer of the note is obligated to pay it to the niece as the person entitled to enforce it The niece is not subject to any defense or claim of the chef relating to the improper repair of the oven because the niece has the rights of a holder in due course When the buyer bought the note from the repairman the buyer became a holder in due course of the note and thus took it free of any personal defenses the chef had against the repairman Even though the niece is not herself a holder in due course of the note the niece succeeded to the buyerrsquos rights as holder in due course and thus took free of the chefrsquos personal defenses

Because the chef refused to pay the note the niece can recover from the repairman on the repairmanrsquos obligation as indorser The niece cannot recover on the note against the buyer however because the buyer did not indorse the note (and thus incurred no indorserrsquos obligation) and the buyer did not receive any consideration for transfer of the note to the niece (and therefore made no transfer warranty)

[NOTE Although Article 9 of the Uniform Commercial Code governs the sale of promissory notes (a point that might be correctly noted by examinees) that Article does not determine the answer to any of the questions posed]

Point One(a) (20) The niece is the holder of the note and thus may enforce it against the chef who is the issuer of the note

The chef is the maker of the note and thus its issuer See UCC sectsect 3-103 3-105 The issuer of a note is obligated to pay it in accordance with its terms to a ldquoperson entitled to enforcerdquo it UCC sect 3-412 The niece is a ldquoperson entitled to enforcerdquo the note This is because the niece is the holder of the note and a holder of a note is a person entitled to enforce it UCC sect 3-301 The niece is the holder of the note because (i) the repairmanrsquos signature on the back of the note not

41

Negotiable Instruments Analysis

accompanied by words indicating a person to whom the note was made payable was a ldquoblank indorsementrdquo which had the effect of making the note a bearer instrument (ii) anyone in possession of a bearer instrument is a holder of it and (iii) the niece is in possession of the note See UCC sectsect 1-201(b)(21)(A) 3-204 and 3-205 Accordingly the chef has an obligation to the niece to pay the note in accordance with its terms and the niece may enforce that obligation

Point One(b) (40) The niece is not a holder in due course of the note but because she is a transferee from the buyer who was a holder in due course she has the same enforcement rights as the buyer Because the buyer as a holder in due course would have been able to enforce the note against the chef without being subject to defenses or claims arising from the improper repair the niece has the same rights and will not be subject to the chefrsquos defenses or claims about the repair

As noted in Point One(a) the chef has an obligation to the niece to pay the note in accordance with its terms However except against a person with the rights of a holder in due course the chef can raise any defenses or claims in recoupment that he would have if the claim on the note were an ordinary contract claim UCC sect 3-305 Thus except against a holder in due course the chef would be able to raise the improper repair as a defense or a claim in recoupment (a claim in response to the niecersquos claim)

But claims in recoupment and most defenses cannot be raised against a person with the rights of a holder in due course Against a holder in due course the chef can raise only the four ldquorealrdquo defenses listed in UCC sect 3-305(a)(1) (infancy duress lack of legal capacity or illegality that nullifies the obligation of the obligor under other law fraud in the factum discharge in insolvency proceedings) none of which is present here

The niece is not a holder in due course because she did not take the note for value See UCC sectsect 3-302(a)(2)(i) (criteria for holder in due course status) and 3-303(a) (definition of ldquovaluerdquo) But this does not mean that the niece is subject to the chefrsquos claim arising out of the improper repair The buyer was a holder in due course of the note because he took the note for value ($9500) in good faith and without notice of any facts that would have alerted him to the chefrsquos defense against the repairman UCC sect 3-302(a)(2) As a holder in due course the buyer owned the note free of the chefrsquos claim because that claim did not constitute a ldquorealrdquo defense UCC sect 3-305(b) When the buyer gave the note to the niece this constituted a ldquotransferrdquo of the note See UCC sect 3-203(a) When a note is transferred the transferee receives ldquoany right of the transferor to enforce the instrument including any right as a holder in due courserdquo UCC sect 3-203(b) Under this rule (also known as the ldquoshelter principlerdquo) the buyer transferred his freedom from the chefrsquos defenses to the niece and the niece can enforce the note free of the chefrsquos defenses

Point Two (20) Because the chef dishonored the note the niece can recover from the repairman on the repairmanrsquos obligation as indorser

The chefrsquos refusal to pay the note constituted dishonor See UCC sect 3-502 The repairman as an indorser of the note (see Point One(a)) incurred the obligations of an indorser under UCC sect 3-415(a) When a note has been dishonored one of the obligations of an indorser is to pay the amount of the note to a person entitled to enforce it Therefore the repairman is liable for the amount of the note to the niece a person entitled to enforce the note (so long as the niece gives proper notice of dishonor to the repairman)

42

Negotiable Instruments Analysis

[NOTE Because the repairman indorsed the note without warranties there are no transfer warranties UCC sect 3-416 cmt 5]

Point Three (20) The niece cannot recover on the note against the buyer as either indorser or warrantor because the buyer did not indorse the note and did not receive consideration for transferring the note to the niece

The buyer did not indorse the note and therefore did not incur the obligation of an indorser to pay the note upon dishonor

The niece cannot recover from the buyer under a transfer warranty theory because transfer warranties are made only by a person ldquowho transfers an instrument for considerationrdquo Here the buyer gave the instrument to the niece as a gift So the buyer made no transfer warranty UCC sect 3-416(a) Therefore the niece cannot recover from the buyer on that theory

43

National Conference of Bar Examiners 302 South Bedford Street | Madison WI 53703-3622 Phone 608-280-8550 | Fax 608-280-8552 | TDD 608-661-1275

wwwncbexorg e-mail contactncbexorg

  • Contents
  • Preface
  • Description of the MEE
  • Instructions
  • February 2013 Questions
    • Real Property Question
    • Contracts Question
    • Constitutional Law Question
    • Secured Transactions Question
    • Federal Civil Procedure Question
    • Agency Question
    • Evidence Question
    • Trusts and Future Interests Question
    • Negotiable Instruments Question
      • February 2013 Analyses
        • Real Property Analysis
        • Contracts Analysis
        • Constitutional Law Analysis
        • Secured Transactions Analysis
        • Federal Civil Procedure Analysis
        • Agency Analysis
        • Evidence Analysis
        • Trusts and Future Interests Analysis
        • Negotiable Instruments Analysis
            • ltlt ASCII85EncodePages false AllowTransparency false AutoPositionEPSFiles true AutoRotatePages None Binding Left CalGrayProfile (Dot Gain 20) CalRGBProfile (sRGB IEC61966-21) CalCMYKProfile (US Web Coated 050SWOP051 v2) sRGBProfile (sRGB IEC61966-21) CannotEmbedFontPolicy Error CompatibilityLevel 14 CompressObjects Tags CompressPages true ConvertImagesToIndexed true PassThroughJPEGImages true CreateJobTicket false DefaultRenderingIntent Default DetectBlends true DetectCurves 00000 ColorConversionStrategy CMYK DoThumbnails false EmbedAllFonts true EmbedOpenType false ParseICCProfilesInComments true EmbedJobOptions true DSCReportingLevel 0 EmitDSCWarnings false EndPage -1 ImageMemory 1048576 LockDistillerParams false MaxSubsetPct 100 Optimize true OPM 1 ParseDSCComments true ParseDSCCommentsForDocInfo true PreserveCopyPage true PreserveDICMYKValues true PreserveEPSInfo true PreserveFlatness true PreserveHalftoneInfo false PreserveOPIComments true PreserveOverprintSettings true StartPage 1 SubsetFonts true TransferFunctionInfo Apply UCRandBGInfo Preserve UsePrologue false ColorSettingsFile () AlwaysEmbed [ true ] NeverEmbed [ true ] AntiAliasColorImages false CropColorImages true ColorImageMinResolution 300 ColorImageMinResolutionPolicy OK DownsampleColorImages true ColorImageDownsampleType Bicubic ColorImageResolution 300 ColorImageDepth -1 ColorImageMinDownsampleDepth 1 ColorImageDownsampleThreshold 150000 EncodeColorImages true ColorImageFilter DCTEncode AutoFilterColorImages true ColorImageAutoFilterStrategy JPEG ColorACSImageDict ltlt QFactor 015 HSamples [1 1 1 1] VSamples [1 1 1 1] gtgt ColorImageDict ltlt QFactor 015 HSamples [1 1 1 1] VSamples [1 1 1 1] gtgt JPEG2000ColorACSImageDict ltlt TileWidth 256 TileHeight 256 Quality 30 gtgt JPEG2000ColorImageDict ltlt TileWidth 256 TileHeight 256 Quality 30 gtgt AntiAliasGrayImages false CropGrayImages true GrayImageMinResolution 300 GrayImageMinResolutionPolicy OK DownsampleGrayImages true GrayImageDownsampleType Bicubic GrayImageResolution 300 GrayImageDepth -1 GrayImageMinDownsampleDepth 2 GrayImageDownsampleThreshold 150000 EncodeGrayImages true GrayImageFilter DCTEncode AutoFilterGrayImages true GrayImageAutoFilterStrategy JPEG GrayACSImageDict ltlt QFactor 015 HSamples [1 1 1 1] VSamples [1 1 1 1] gtgt GrayImageDict ltlt QFactor 015 HSamples [1 1 1 1] VSamples [1 1 1 1] gtgt JPEG2000GrayACSImageDict ltlt TileWidth 256 TileHeight 256 Quality 30 gtgt JPEG2000GrayImageDict ltlt TileWidth 256 TileHeight 256 Quality 30 gtgt AntiAliasMonoImages false CropMonoImages true MonoImageMinResolution 1200 MonoImageMinResolutionPolicy OK DownsampleMonoImages true MonoImageDownsampleType Bicubic MonoImageResolution 1200 MonoImageDepth -1 MonoImageDownsampleThreshold 150000 EncodeMonoImages true MonoImageFilter CCITTFaxEncode MonoImageDict ltlt K -1 gtgt AllowPSXObjects false CheckCompliance [ None ] PDFX1aCheck false PDFX3Check false PDFXCompliantPDFOnly false PDFXNoTrimBoxError true PDFXTrimBoxToMediaBoxOffset [ 000000 000000 000000 000000 ] PDFXSetBleedBoxToMediaBox true PDFXBleedBoxToTrimBoxOffset [ 000000 000000 000000 000000 ] PDFXOutputIntentProfile () PDFXOutputConditionIdentifier () PDFXOutputCondition () PDFXRegistryName () PDFXTrapped False CreateJDFFile false Description ltlt ARA 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 BGR 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 CHS ltFEFF4f7f75288fd94e9b8bbe5b9a521b5efa7684002000410064006f006200650020005000440046002065876863900275284e8e9ad88d2891cf76845370524d53705237300260a853ef4ee54f7f75280020004100630072006f0062006100740020548c002000410064006f00620065002000520065006100640065007200200035002e003000204ee553ca66f49ad87248672c676562535f00521b5efa768400200050004400460020658768633002gt CHT ltFEFF4f7f752890194e9b8a2d7f6e5efa7acb7684002000410064006f006200650020005000440046002065874ef69069752865bc9ad854c18cea76845370524d5370523786557406300260a853ef4ee54f7f75280020004100630072006f0062006100740020548c002000410064006f00620065002000520065006100640065007200200035002e003000204ee553ca66f49ad87248672c4f86958b555f5df25efa7acb76840020005000440046002065874ef63002gt CZE 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 DAN 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 DEU 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 ESP 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 ETI 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 FRA 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 GRE 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 HEB ltFEFF05D405E905EA05DE05E905D5002005D105D405D205D305E805D505EA002005D005DC05D4002005DB05D305D9002005DC05D905E605D505E8002005DE05E105DE05DB05D9002000410064006F006200650020005000440046002005D405DE05D505EA05D005DE05D905DD002005DC05D405D305E405E105EA002005E705D305DD002D05D305E405D505E1002005D005D905DB05D505EA05D905EA002E002005DE05E105DE05DB05D90020005000440046002005E905E005D505E605E805D5002005E005D905EA05E005D905DD002005DC05E405EA05D905D705D4002005D105D005DE05E605E205D505EA0020004100630072006F006200610074002005D5002D00410064006F00620065002000520065006100640065007200200035002E0030002005D505D205E805E105D005D505EA002005DE05EA05E705D305DE05D505EA002005D905D505EA05E8002E05D005DE05D905DD002005DC002D005000440046002F0058002D0033002C002005E205D905D905E005D5002005D105DE05D305E805D905DA002005DC05DE05E905EA05DE05E9002005E905DC0020004100630072006F006200610074002E002005DE05E105DE05DB05D90020005000440046002005E905E005D505E605E805D5002005E005D905EA05E005D905DD002005DC05E405EA05D905D705D4002005D105D005DE05E605E205D505EA0020004100630072006F006200610074002005D5002D00410064006F00620065002000520065006100640065007200200035002E0030002005D505D205E805E105D005D505EA002005DE05EA05E705D305DE05D505EA002005D905D505EA05E8002Egt HRV (Za stvaranje Adobe PDF dokumenata najpogodnijih za visokokvalitetni ispis prije tiskanja koristite ove postavke Stvoreni PDF dokumenti mogu se otvoriti Acrobat i Adobe Reader 50 i kasnijim verzijama) HUN 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 ITA 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 JPN ltFEFF9ad854c18cea306a30d730ea30d730ec30b951fa529b7528002000410064006f0062006500200050004400460020658766f8306e4f5c6210306b4f7f75283057307e305930023053306e8a2d5b9a30674f5c62103055308c305f0020005000440046002030d530a130a430eb306f3001004100630072006f0062006100740020304a30883073002000410064006f00620065002000520065006100640065007200200035002e003000204ee5964d3067958b304f30533068304c3067304d307e305930023053306e8a2d5b9a306b306f30d530a930f330c8306e57cb30818fbc307f304c5fc59808306730593002gt KOR ltFEFFc7740020c124c815c7440020c0acc6a9d558c5ec0020ace0d488c9c80020c2dcd5d80020c778c1c4c5d00020ac00c7a50020c801d569d55c002000410064006f0062006500200050004400460020bb38c11cb97c0020c791c131d569b2c8b2e4002e0020c774b807ac8c0020c791c131b41c00200050004400460020bb38c11cb2940020004100630072006f0062006100740020bc0f002000410064006f00620065002000520065006100640065007200200035002e00300020c774c0c1c5d0c11c0020c5f40020c2180020c788c2b5b2c8b2e4002egt LTH 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 LVI 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 NLD (Gebruik deze instellingen om Adobe PDF-documenten te maken die zijn geoptimaliseerd voor prepress-afdrukken van hoge kwaliteit De gemaakte PDF-documenten kunnen worden geopend met Acrobat en Adobe Reader 50 en hoger) NOR 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 POL ltFEFF0055007300740061007700690065006e0069006100200064006f002000740077006f0072007a0065006e0069006100200064006f006b0075006d0065006e007400f300770020005000440046002000700072007a0065007a006e00610063007a006f006e00790063006800200064006f002000770079006400720075006b00f30077002000770020007700790073006f006b00690065006a0020006a0061006b006f015b00630069002e002000200044006f006b0075006d0065006e0074007900200050004400460020006d006f017c006e00610020006f007400770069006500720061010700200077002000700072006f006700720061006d006900650020004100630072006f00620061007400200069002000410064006f00620065002000520065006100640065007200200035002e0030002000690020006e006f00770073007a0079006d002egt PTB ltFEFF005500740069006c0069007a006500200065007300730061007300200063006f006e00660069006700750072006100e700f50065007300200064006500200066006f0072006d00610020006100200063007200690061007200200064006f00630075006d0065006e0074006f0073002000410064006f0062006500200050004400460020006d00610069007300200061006400650071007500610064006f00730020007000610072006100200070007200e9002d0069006d0070007200650073007300f50065007300200064006500200061006c007400610020007100750061006c00690064006100640065002e0020004f007300200064006f00630075006d0065006e0074006f00730020005000440046002000630072006900610064006f007300200070006f00640065006d0020007300650072002000610062006500720074006f007300200063006f006d0020006f0020004100630072006f006200610074002000650020006f002000410064006f00620065002000520065006100640065007200200035002e0030002000650020007600650072007300f50065007300200070006f00730074006500720069006f007200650073002egt RUM 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 RUS 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 SKY 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 SLV 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 SUO 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 SVE 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 TUR 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 UKR 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 ENU (Use these settings to create Adobe PDF documents best suited for high-quality prepress printing Created PDF documents can be opened with Acrobat and Adobe Reader 50 and later) gtgt Namespace [ (Adobe) (Common) (10) ] OtherNamespaces [ ltlt AsReaderSpreads false CropImagesToFrames true ErrorControl WarnAndContinue FlattenerIgnoreSpreadOverrides false IncludeGuidesGrids false IncludeNonPrinting false IncludeSlug false Namespace [ (Adobe) (InDesign) (40) ] OmitPlacedBitmaps false OmitPlacedEPS false OmitPlacedPDF false SimulateOverprint Legacy gtgt ltlt AddBleedMarks false AddColorBars false AddCropMarks false AddPageInfo false AddRegMarks false ConvertColors ConvertToCMYK DestinationProfileName () DestinationProfileSelector DocumentCMYK Downsample16BitImages true FlattenerPreset ltlt PresetSelector MediumResolution gtgt FormElements false GenerateStructure false IncludeBookmarks false IncludeHyperlinks false IncludeInteractive false IncludeLayers false IncludeProfiles false MultimediaHandling UseObjectSettings Namespace [ (Adobe) (CreativeSuite) (20) ] PDFXOutputIntentProfileSelector DocumentCMYK PreserveEditing true UntaggedCMYKHandling LeaveUntagged UntaggedRGBHandling UseDocumentProfile UseDocumentBleed false gtgt ]gtgt setdistillerparamsltlt HWResolution [2400 2400] PageSize [612000 792000]gtgt setpagedevice

Page 6: February 2013 MEE Questions and Analyses

February 2013 MEE

QUESTIONS

Real Property Contracts

Constitutional Law Secured Transactions

Federal Civil Procedure Agency

Evidence Trusts and Future Interests

Negotiable Instruments

REAL PROPERTY QUESTION _______________

In 2008 a landlord and a tenant entered into a 10-year written lease commencing September 1 2008 for the exclusive use of a commercial building at a monthly rent of $2500 The lease contained a covenant of quiet enjoyment but no other covenants or promises on the part of the landlord

When the landlord and tenant negotiated the lease the tenant asked the landlord if the building had an air-conditioning system The landlord answered ldquoYes it doesrdquo The tenant responded ldquoGreat I will be using the building to manufacture a product that will be irreparably damaged if the temperature during manufacture exceeds 81 degrees for more than six consecutive hoursrdquo

On April 15 2012 the buildingrsquos air-conditioning system malfunctioned causing the building temperature to rise above 81 degrees for three hours The tenant immediately telephoned the landlord about this malfunction The tenant left a message in which he explained what had happened and asked the landlord ldquoWhat are you going to do about itrdquo The landlord did not respond to the tenantrsquos message

On May 15 2012 the air-conditioning system again malfunctioned This time the malfunction caused the building temperature to rise above 81 degrees for six hours The tenant telephoned the landlord and left a message describing the malfunction As before the landlord did not respond

On August 24 2012 the air-conditioning system malfunctioned again causing the temperature to rise above 81 degrees for 10 hours Again the tenant promptly telephoned the landlord The landlord answered the phone and the tenant begged her to fix the system The landlord refused The tenant then attempted to fix the system himself but he failed As a result of the air-conditioning malfunction products worth $150000 were destroyed

The next day the tenant wrote the following letter to the landlord

Irsquove had enough I told you about the air-conditioning problem twice before yesterdayrsquos disaster and you failed to correct it I will vacate the building by the end of the month and will bring you the keys when I leave

The tenant vacated the building on August 31 2012 and returned the keys to the landlord that day At that time there were six years remaining on the lease

On September 1 2012 the landlord returned the keys to the tenant with a note that said ldquoI repeat the air-conditioning is not my problem You have leased the building and you should fix itrdquo The tenant promptly sent the keys back to the landlord with a letter that said ldquoI have terminated the lease and I will not be returning to the building or making further rent paymentsrdquo After receiving the keys and letter the landlord put the keys into her desk To date she has neither responded to the tenantrsquos letter nor taken steps to lease the building to another tenant

On November 1 2012 two months after the tenant vacated the property the landlord sued the tenant claiming that she is entitled to the remaining unpaid rent ($180000) from September 1 for the balance of the lease term (reduced to present value) or if not that then damages for the tenantrsquos wrongful termination

Is the landlord correct Explain

3

CONTRACTS QUESTION _______________

On January 2 a boat builder and a sailor entered into a contract pursuant to which the builder was to sell to the sailor a boat to be specially manufactured for the sailor by the builder The contract price was $100000 The written contract signed by both parties stated that the builder would tender the boat to the sailor on December 15 at which time payment in full would be due

On October 15 the builderrsquos workers went on strike and there were no available replacements

On October 31 the builderrsquos workers were still on strike and no work was being done on the boat The sailor read a news report about the strike and immediately sent a letter to the builder stating ldquoI am very concerned that my boat will not be completed by December 15 I insist that you provide me with assurance that you will perform in accordance with the contractrdquo The builder received the letter on the next day November 1

On November 25 the builder responded to the letter stating ldquoIrsquom sorry about the strike but it is really out of my hands I hope we settle it soon so that we can get back to workrdquo

Nothing further happened until December 3 when the builder called the sailor and said ldquoMy workers are back and I have two crews working overtime to finish your boat Your boat is task one Donrsquot worry wersquoll deliver your boat by December 15thrdquo The sailor immediately replied ldquoI donrsquot trust you As far as Irsquom concerned our contract is over I am going to buy my boat from a shipyardrdquo Two days later the sailor entered into a contract with a competing manufacturer to buy a boat similar to the boat that was the subject of the contract with the builder

The builder finished the boat on time and tendered it to the sailor on December 15 The sailor reminded the builder about the December 3 conversation in which the sailor had announced that ldquoour contract is overrdquo and refused to take the boat and pay for it

The builder has sued the sailor for breach of contract

1 What was the legal effect of the sailorrsquos October 31 letter to the builder Explain

2 What was the legal effect of the builderrsquos November 25 response to the sailorrsquos October 31 letter Explain

3 What was the legal effect of the sailorrsquos refusal to take and pay for the boat on December 15 Explain

4

CONSTITUTIONAL LAW QUESTION

AutoCo is a privately owned corporation that manufactures automobiles Ten years ago AutoCo purchased a five-square-mile parcel of unincorporated land in a remote region of the state and built a large automobile assembly plant on the land To attract workers to the remote location of the plant AutoCo built apartment buildings and houses on the land and leased them to its employees AutoCo owns and operates a commercial district with shops and streets open to the general public AutoCo named the area Oakwood and provides security fire protection and sanitation services for Oakwoodrsquos residents AutoCo also built operates and fully funds the only school in the region which it makes available free of charge to the children of its employees

A family recently moved to Oakwood The father and mother work in AutoCorsquos plant rent an apartment from AutoCo and have enrolled their 10-year-old son in Oakwoodrsquos school Every morning the students are required to recite the Pledge of Allegiance while standing and saluting an American flag With the approval of his parents the son has politely but insistently refused to recite the Pledge and salute the flag at the school on the grounds that doing so violates his own political beliefs and the political beliefs of his family As a result of his refusal to say the Pledge the son has been expelled from the school

To protest the schoolrsquos actions the father walked into the commercial district of Oakwood While standing on a street corner he handed out leaflets that contained a short essay critical of the schoolrsquos Pledge of Allegiance policy Some of the passersby who took the leaflets dropped them to the ground An AutoCo security guard saw the litter told the father that Oakwoodrsquos anti-litter rule prohibits leaflet distribution that results in littering and directed him to cease distribution of the leaflets and leave the commercial district When the father did not leave and continued to distribute the leaflets the security guard called the state police which sent officers who arrested the father for trespass

1 Did the sonrsquos expulsion from the school violate the First Amendment as applied through the Fourteenth Amendment Explain

2 Did the fatherrsquos arrest violate the First Amendment as applied through the Fourteenth Amendment Explain

5

SECURED TRANSACTIONS QUESTION

On June 1 a bicycle retailer sold two bicycles to a man for a total purchase price of $1500 The man made a $200 down payment and agreed to pay the balance in one year The man also signed a security agreement that identified the bicycles as collateral for the unpaid purchase price and provided that the man ldquoshall not sell or dispose of the collateral until the balance owed is paid in fullrdquo The retailer never filed a financing statement reflecting this security interest

The man had bought the bicycles for him and his girlfriend to use on vacation However shortly after he bought the bicycles the man and his girlfriend broke up The man has never used the bicycles

On August 1 the man sold one of the bicycles at a garage sale to a buyer who paid the man $400 for the bicycle The buyer bought the bicycle to ride for weekend recreation

On October 1 the man gave the other bicycle to his friend as a birthday present The friend began using the bicycle for morning exercise

Neither the buyer nor the friend had any knowledge of the manrsquos dealings with the retailer

1 Does the buyer own the bicycle free of the retailerrsquos security interest Explain

2 Does the friend own the bicycle free of the retailerrsquos security interest Explain

6

FEDERAL CIVIL PROCEDURE QUESTION _____

Mother and Son who are both adults are citizens and residents of State A Mother owned an expensive luxury car valued in excess of $100000 Son borrowed Motherrsquos car to drive to a store in State A As Son approached a traffic light that had just turned yellow he carefully braked and brought the car to a complete stop Driver who was following immediately behind him failed to stop and rear-ended Motherrsquos car which was damaged beyond repair Son was seriously injured Driver is a citizen of State B

Son sued Driver in the United States District Court for the District of State A alleging that she was negligent in the operation of her vehicle Son sought damages in excess of $75000 for his personal injuries exclusive of costs and interest In her answer Driver alleged that Son was contributorily negligent in the operation of Motherrsquos car She further alleged that the brake lights on Motherrsquos car were burned out and that Motherrsquos negligent failure to properly maintain the car was a contributing cause of the accident

Following a trial on the merits in Sonrsquos case against Driver the jury answered the following special interrogatories

Do you find that Driver was negligent in the operation of her vehicle Yes

Do you find that Son was negligent in the operation of Motherrsquos car No

Do you find that Mother negligently failed to ensure that the brake lights on her car were in proper working order Yes

The judge then entered a judgment in favor of Son against Driver Driver did not appeal

Two months later Mother sued Driver in the United States District Court for the District of State A alleging that Driverrsquos negligence in the operation of her vehicle destroyed Motherrsquos luxury car Mother sought damages in excess of $75000 exclusive of costs and interest

State A follows the same preclusion principles that federal courts follow in federal-question cases

1 Is Motherrsquos claim against Driver barred by the judgment in Son v Driver Explain

2 Does the juryrsquos conclusion in Son v Driver that Mother had negligently failed to maintain the brake lights on her car preclude Mother from litigating that issue in her subsequent suit against Driver Explain

3 Does the juryrsquos conclusion in Son v Driver that Driver was negligent preclude Driver from litigating that issue in the Mother v Driver lawsuit Explain

7

_____

AGENCY QUESTION

Over 5000 individuals in the United States operate hot-air balloon businesses A hot-air balloon has four key components the balloon that holds the heated air the basket that houses the riders the propane burner that heats the air in the balloon and the propane storage tanks

The owner of a hot-air balloon business recently notified several basket and burner manufacturers that she or her agent might be contacting them to purchase baskets or burners The owner did not specifically name any person as her agent Basket and burner manufacturers regularly receive such notices from hot-air balloon operators Such notices typically include no restrictions on the types of baskets or burners agents might purchase for their principals

The owner then retained an agent to acquire baskets burners and fuel tanks from various manufacturers The owner authorized the agent to buy only (a) baskets made of woven wicker (not aluminum) (b) burners that use a unique ldquowhisper technologyrdquo (so as not to scare livestock when the balloon sails over farmland) and (c) propane fuel tanks

The agent then entered into three transactions with manufacturers all of whom had no prior dealings with either the owner or the agent

(1) The agent and a large manufacturer of both wicker and aluminum baskets signed a contract for the purchase of four aluminum baskets for a total cost of $60000 The agent never told the manufacturer that he represented the owner or any other principal The contract listed the agent as the buyer and listed the ownerrsquos address as the delivery address but did not indicate that the address was that of the owner rather than the agent When the baskets were delivered to the owner she learned for the first time that the agent had contracted to buy aluminum not wicker baskets The owner immediately rejected the baskets and returned them to the manufacturer Neither the owner nor the agent has paid the basket manufacturer for them

(2) The agent contacted a burner manufacturer and told him that the agent represented a well-known hot-air balloon operator who wanted to purchase burners The agent did not disclose the ownerrsquos name The agent and the burner manufacturer signed a contract for the purchase of four burners that did not have ldquowhisper technologyrdquo for a total price of $70000 The burner contract like the basket contract listed the ownerrsquos address for delivery but did not disclose whose address it was The burners were delivered to the ownerrsquos business and the owner discovered that the agent had ordered the wrong kind of burners The owner rejected the burners and returned them to the manufacturer Neither the owner nor the agent has paid the burner manufacturer for the burners

(3) The agent contracted with a solar cell manufacturer to make three cells advertised as ldquostrong enough to power all your ballooning needsrdquo The agent did not tell the manufacturer that he was acting on behalf of any other person One week after the cells were delivered to the agent he took them to the owner who installed them and discovered that she could save a lot of money using solar cells instead of propane to power her balloons The owner decided to keep the solar cells but she has not paid the manufacturer for them

8

Agency Question

Assume that the rejection of the baskets and the burners and the failure to pay for the solar cells constitute breach of the relevant contracts

1 Is the owner liable to the basket manufacturer for breach of the contract for the aluminum baskets Is the agent liable Explain

2 Is the owner liable to the burner manufacturer for breach of the contract for the burners Is the agent liable Explain

3 Is the owner liable to the solar cell manufacturer for breach of the contract for the solar cells Is the agent liable Explain (Do not address liability based upon restitution or unjust enrichment)

9

EVIDENCE QUESTION _____

A woman who owns a motorized scooter brought her scooter to a mechanic for routine maintenance service As part of the maintenance service the mechanic inspected the braking system on the scooter As soon as the mechanic finished inspecting and servicing the scooter he sent the woman a text message to her cell phone that read ldquoJust finished your service When you pick up your scooter you need to schedule a follow-up brake repair Wersquoll order the partsrdquo

The woman read the mechanicrsquos text message and returned the next day to pick up her scooter As the woman was wheeling her scooter out of the shop she saw the mechanic working nearby and asked ldquoIs my scooter safe to ride for a whilerdquo The mechanic responded by giving her a thumbs-up The woman waved and rode away on the scooter

One week later while the woman was riding her scooter a pedestrian stepped off the curb into a crosswalk and the woman collided with him causing the pedestrian severe injuries The woman had not had the scooterrsquos brakes repaired before the accident

The pedestrian has sued the woman for damages for his injuries resulting from the accident The pedestrian has alleged that (1) the woman lost control of the scooter due to its defective brakes (2) the woman knew that the brakes needed repair and (3) it was negligent for the woman to ride the scooter knowing that its brakes needed to be repaired

The woman claims that the brakes on the scooter worked perfectly and that the accident happened because the pedestrian stepped into the crosswalk without looking and the woman had no time to stop The woman the pedestrian and the mechanic will testify at the upcoming trial

The pedestrian has proffered an authenticated copy of the mechanicrsquos text message to the woman

The woman plans to testify that she asked the mechanic ldquoIs my scooter safe to ride for a whilerdquo and that he gave her a thumbs-up in response

The evidence rules in this jurisdiction are identical to the Federal Rules of Evidence

Analyze whether each of these items of evidence is relevant and admissible at trial

1 The authenticated copy of the mechanicrsquos text message

2 The womanrsquos testimony that she asked the mechanic ldquoIs my scooter safe to ride for a whilerdquo and

3 The womanrsquos testimony describing the mechanicrsquos thumbs-up

10

TRUSTS AND FUTURE INTERESTS QUESTION

Ten years ago Settlor validly created an inter vivos trust and named Bank as trustee The trust instrument provided that Settlor would receive all of the trust income during her lifetime The trust instrument further provided that

Upon Settlorrsquos death the trust income shall be paid in equal shares to Settlorrsquos surviving children for their lives Upon the death of the last surviving child the trust income shall be paid in equal shares to Settlorrsquos then-living grandchildren for their lives Upon the death of the survivor of Settlorrsquos children and grandchildren the trust corpus shall be distributed in equal shares to Settlorrsquos then-living great-grandchildren

The trust instrument expressly specified that the trust was revocable but it was silent regarding whether Settlor could amend the trust instrument

Immediately after creating the trust Settlor validly executed a will leaving her entire estate to Bank as trustee of her inter vivos trust to ldquohold in accordance with the terms of the trustrdquo

Five years ago Settlor signed an amendment to the inter vivos trust The amendment changed the disposition of the remainder interest specifying that all trust assets ldquoshall be paid upon Settlorrsquos death to Universityrdquo Settlorrsquos signature on this amendment was not witnessed

A state statute provides that any trust interest that violates the common law Rule Against Perpetuities ldquois nonetheless valid if the nonvested interest in the trust actually vests or fails to vest either (a) within 21 years of lives in being at the creation of the nonvested interest or (b) within 90 years of its creationrdquo

Recently Settlor died leaving a probate estate of $200000 She was survived by no children one granddaughter (who would be Settlorrsquos only heir) and no great-grandchildren The granddaughter has consulted your law firm and has raised four questions regarding this trust

1 Was Settlorrsquos amendment of the inter vivos trust valid Explain

2 Assuming that the trust amendment was valid do its provisions apply to Settlorrsquos probate assets Explain

3 Assuming that the trust amendment was valid how should trust assets be distributed Explain

4 Assuming that the trust amendment was invalid how should trust assets be distributed Explain

11

NEGOTIABLE INSTRUMENTS QUESTION

A chef entered into a contract with a repairman pursuant to which the repairman agreed to repair the chefrsquos commercial oven for $10000 The repairman agreed to accept as payment a negotiable promissory note for $10000 payable two months after its issuance

After the repairman worked on the oven the chef gave him a $10000 note as payment for the work As agreed the note was signed by the chef as maker was payable to the order of the repairman was payable in two months and fulfilled all criteria for negotiability

The next day the repairman sold the note to a buyer for $9500 To effectuate the sale the repairman wrote ldquono warrantiesrdquo on the back of the note signed his name immediately below that and handed the note to the buyer The buyer bought the note in good faith and without knowledge of any facts relating to the work that the repairman had performed for the chef

Later the buyer gave the note to his niece as a gift To effectuate the gift the buyer handed the note to the niece but did not indorse it

Shortly thereafter the chef discovered that the repair work had been done improperly and the oven still did not function correctly The chef tried repeatedly to get the repairman to return to correct the repair work but the repairman ignored all the chefrsquos calls

On the notersquos due date the niece contacted the chef and demanded that he pay the amount of the note to her The chef refused and told the niece that he would not pay the note because the repairman did not properly repair the oven

1 What are the niecersquos rights against the chef Explain

2 What are the niecersquos rights against the repairman Explain

3 What are the niecersquos rights against the buyer Explain

12

February 2013 MEE

ANALYSES

Real Property Contracts

Constitutional Law Secured Transactions

Federal Civil Procedure Agency

Evidence Trusts and Future Interests

Negotiable Instruments

REAL PROPERTY ANALYSIS (Real Property ID1a 4 amp 5)

ANALYSIS

Legal Problems

(1) Does the tenant have a defense to the landlordrsquos action for unpaid rent based on constructive eviction

(2) Does the tenant have a defense to the landlordrsquos action for unpaid rent based on the tenantrsquos surrender of the premises

(3) What if anything may the landlord recover from the tenant for the period after the tenant vacated the building

DISCUSSION

Summary

Under the common law the tenant does not have a defense to the landlordrsquos action for unpaid rent based on constructive eviction Constructive eviction is based on the tenant proving that (1) the landlord breached a duty to the tenant (2) the breach caused a loss by the tenant of the substantial use and enjoyment of the premises (3) the tenant gave the landlord adequate notice and opportunity to repair and (4) the tenant vacated the leased premises Here there was no constructive eviction because although the tenant vacated and gave the landlord adequate notice the landlord breached no express or implied duty to the tenant to repair the premises

The tenant does not have a defense based on the landlordrsquos acceptance of his surrender of the premises a landlordrsquos retention of keys does not constitute an acceptance of the tenantrsquos surrender unless the landlord so intended and here the landlordrsquos statements to the tenant at the time of the surrender of the keys do not evidence the intent to accept the tenantrsquos surrender

Under the common law a landlord has no duty to mitigate damages but also cannot sue for rents due in the future Under this approach the landlord can sue only for past-due rents Using this approach on November 1 the landlord could recover all the rent past due (ie rent for September and October) but could not recover for rents due in the future However some courts have authorized recovery for future rent minus the fair market rental value of the premises It is thus possible that the landlord could recover damages equal to the amount of rent due from September 1 to the end of the six-year lease term ($180000) minus the propertyrsquos fair-market rental value over that same period

Point One (45) The tenant was not constructively evicted because the landlord had no duty to repair the commercial premises that were the subject of the lease

The landlord and the tenant entered into a term-of-years lease because the lease specified both a beginning and an ending date HERBERT HOVENKAMP amp SHELDON F KURTZ THE LAW OF

PROPERTY 256 (5th ed 2001) Although a term-of-years lease normally cannot be terminated by the tenant prior to the end of the term a tenant may terminate a term-of-years lease if the tenant

15

Real Property Analysis

is constructively evicted See id at 286ndash88 Typically as here a claim of constructive eviction is made as a defense to a landlordrsquos action for damages or unpaid rent

In order to establish a constructive eviction the tenant must prove that the landlord breached a duty to the tenant such as a duty to repair and that the landlordrsquos breach caused a loss of the substantial use and enjoyment of the premises The tenant must also show that he gave the landlord notice adequate to permit the landlord to meet his duty to the tenant and that the tenant vacated the leased premises Id see also JOHN G SPRANKLING UNDERSTANDING

PROPERTY LAW sect 1704 (2d ed 2007) Under the common law there was no implied duty on the part of a landlord to repair

leased premises such a duty arose only if expressly set forth in the lease SPRANKLING supra sect 1702[B] Here the written lease contained no term requiring the landlord to repair the air-conditioning Even if the conversation created a lease term that the building had air-conditioning that itself should not create a duty for the landlord to repair it

Over the past several decades courts have generally implied a duty to repair in residential leases either as part of a revised constructive eviction doctrine or based on an implied warranty of habitability JOSEPH W SINGER PROPERTY 469ndash70 (3d ed 2010) This shift has been justified based on the economic disparity between the typical landlord and tenant as well as the fact that residential tenants generally lack both the authority to authorize repairs to common areas of a building and the incentive to make repairs that will ultimately benefit the landlord

However courts have been more reluctant to imply a duty to repair in commercial leases a context in which the tenant is often a valuable business and in a better position to assess and make repairs than is the landlord But see eg Davidow v Inwood North Professional Group 747 SW2d 373 (Tex 1988) When courts have implied a duty to repair in a commercial lease it is typically when the repair has been mandated by public authorities and involves work so substantial that it would not ordinarily fall within the tenantrsquos common law repair duty andor the value of the repair would primarily inure to the landlordrsquos reversionary interest See Brown v Green 884 P2d 55 (Cal 1994) Eugene L Grant et al The Tenant as Terminator Constructive Eviction in Modern Commercial Leases 2 THE COMMERCIAL PROPERTY LEASE ch 15 (ABA 1997) Some courts have also permitted constructive eviction claims by commercial tenants of office buildings based on repairs required in common areas of the building See id Echo Consulting Services Inc v North Conway Bank 669 A2d 227 (NH 1995)

Here the tenant is the owner of a valuable manufacturing operation and is the exclusive occupant of the building the repair has not been mandated by public authorities and the repair is not structural To the contrary the repair involves a feature of the building of unusual importance in the tenantrsquos manufacturing operation and the tenant is likely far more knowledgeable than the landlord about the air-conditioning specifications necessary for the manufacture of the tenantrsquos product

Based on these facts it is unlikely that a court will find that the tenant in this case was constructively evicted Although the tenant can show that he gave adequate notice to the landlord of the air-conditioning malfunction and vacated the premises the lease was commercial and it did not contain any promises or covenants by the landlord except a covenant of quiet enjoyment a covenant of quiet enjoyment does not entail any repair obligations

[NOTE An examineersquos conclusion is less important than his or her demonstrated awareness of the elements of constructive eviction and the need to imply a repair duty for such a defense to be viable here Although the implied warranty of habitability is not available to this tenant Texas Minnesota and Massachusetts imply a warranty of suitability in commercial leases in limited circumstances and an examinee might argue that this warranty should apply

16

Real Property Analysis

here If an examinee concludes that this warranty applies he or she should discuss the other requirements for constructive eviction

If the examinee wrongly concludes that the first element for a constructive eviction has been met the examinee will then have to discuss the remaining three elements in order to conclude that the tenant can claim constructive eviction The tenant would have a strong argument that the second elementmdashsubstantial interference with the use and enjoyment of the premisesmdashalso is met As indicated above the landlord was aware that a functioning air-conditioning system was vital to the tenantrsquos manufacturing operations The facts further indicate that the system had failed three times in the past few months The landlord may try to argue that the malfunctions did not substantially interfere with the tenantrsquos use of the premises because the malfunctions caused the temperature to climb above 81 degrees for only a short period of timemdash 3 hours 6 hours and 10 hours respectivelymdashon each occasion The tenant will argue however that the landlord was aware that the tenantrsquos manufacturing operations could tolerate temperatures above 81 degrees for no more than 6 hours The final malfunction exceeded that limit destroying $150000 worth of the tenantrsquos products

The tenant would also have a strong argument that the third element is met notice and opportunity to cure The tenant notified the landlord of the problem immediately upon the systemrsquos first malfunction and did so again when it malfunctioned a second time and then a third time The landlord might argue that there was insufficient time to cure the problem because the system corrected itself within a few hours on the first and second times Although the malfunction lasted more than 10 hours the third time the landlord might argue that the time period was insufficient to get a repair person on the premises A court would be likely to find this argument unpersuasive however because the landlord could have attempted to correct the problem after the first and second malfunctions

Assuming that the landlord was given sufficient notice and opportunity to cure a court would be likely to conclude that the tenant also satisfied the final element of vacating the premises within a reasonable time The landlord might argue that the tenant remained in the premises for almost four months after the air conditioning first failed which would suggest that the problem was not so severe as to have constructively evicted the tenant The tenant will argue however that he gave the landlord three months to cure the problem after the first two malfunctions threatened (but did not actually harm) his operations The tenant then moved out shortly after the final malfunction caused temperatures to exceed the tolerance levels of his manufacturing operations]

Point Two (10) The landlord did not accept the tenantrsquos surrender of the lease

When a tenant wrongfully moves from leased premises with the intent to terminate the lease the landlord may either accept the tenantrsquos surrender of the premises and terminate the lease or hold the tenant to the terms of the lease See HOVENKAMP amp KURTZ supra at 295ndash96 Here the tenantrsquos only basis for the claim that the landlord accepted his surrender is the landlordrsquos retention of the keys Many courts have considered whether a landlordrsquos retention of keys delivered by a tenant constitutes acceptance of surrender The weight of the case law holds that retention of the keys alone does not constitute acceptance of surrender without other evidence showing that the landlord intended to accept the surrender See generally 49 AM JUR 2d Landlord and Tenant sect 213

Here the landlordrsquos note saying ldquoI repeat the air-conditioning is not my problem You have leased the building and you should fix itrdquo strongly suggests that the landlord did not intend

17

Some courts have rejected the no-mitigation-of-damages rule based on efficiency concerns and societyrsquos interest in assuring that resources remain in the stream of commerce rather than lying vacant see id at 464ndash65 and allow landlords to sue tenants who have wrongfully terminated a lease for damages equal to the difference between the unpaid rent due under the lease and the propertyrsquos fair market rental value Other courts have abandoned the no-recovery-for-future-rent rule These courts responding to the fact that a tenant may well disappear or be judgment-proof by the time a lease term is concluded have allowed a landlord to collect damages equal to the value of rent over the entire lease term minus the propertyrsquos fair rental value when a tenant has wrongfully terminated a lease and unequivocally shown an intention not to return to the premises or pay future rent Under this approach a landlord receives approximately the same amount he would have received were there a duty to mitigate damages See Sagamore Corp v Willcutt 180 A 464 (Conn 1935)

Real Property Analysis

to accept the tenantrsquos surrender The tenant might argue that the landlordrsquos failure to make a similar statement when the keys were sent to her a second time and she retained them evidences a change of heart However it is likely that a court would find that the landlordrsquos retention of the keys represented a decision to safeguard the keys not to accept the tenantrsquos surrender

[NOTE An examinee should receive credit for arguing the other way with a well-reasoned argument]

Point Three (45) Under the common law the landlord had no duty to mitigate damages Additionally a landlord was not entitled to recover unpaid rents due in the future but was only entitled to recover rents in arrears at the time of the commencement of the suit Applying the common law here the landlord could recover $5000 the amount of rents due at the commencement of the suit ($2500 for September and the same for October) Today some courts allow the landlord under certain circumstances to sue the tenant for damages (not rent) equal to the difference if any between the unpaid promised rent for the balance of the term (here $175000) and the propertyrsquos fair rental value for the balance of the term

Under the common law because a lease was viewed as a conveyance instead of a contract a landlord had no duty to mitigate damages resulting from a tenantrsquos wrongful termination of a lease A landlord could thus recover the full value of rents that were due and unpaid at the time of the suit However under the common law a landlord could not sue a tenant for rents due in the future because there was always a possibility that the tenant might pay the rent when it was due See SINGER supra at 462 Thus using the common law approach on November 1 the landlord could only recover the full value of the two monthsrsquo rent actually due and unpaid ie $5000 for September and October

Here because the tenant returned the keys to the landlord and said ldquoI will not be returning to the building or making further rent paymentsrdquo the landlord could establish abandonment and an intention not to return It is thus possible that the landlord might recover damages in the amount of $5000 (for the months of September and October) plus the present value of $175000 minus the fair market rental value of the property over the remaining months of the lease

18

CONTRACTS ANALYSIS ____ (Contracts II IVE)

ANALYSIS

Legal Problems

(1) What was the legal effect of the sailorrsquos October 31 letter to the builder

(2)(a) What was the legal effect of the builderrsquos November 25 response to the sailorrsquos October 31 letter

(2)(b) What was the legal effect of the sailorrsquos refusal to take and pay for the boat on December 15

DISCUSSION

Summary

This is a sale of goods governed by the Uniform Commercial Code Because the sailor had reasonable grounds for insecurity about the builderrsquos ability to deliver the boat in a timely manner when the sailor learned about the strike on October 31 the sailor was legally justified in sending the letter to the builder seeking adequate assurance of the builderrsquos performance pursuant to the contract The builderrsquos failure to provide such assurance within a reasonable time operated as a repudiation of the contract However the builder was free to retract the repudiation before the sailor either cancelled the contract or materially changed position in reliance on the builderrsquos repudiation The builder retracted the repudiation when he informed the sailor that the workers were back and that the boat would be delivered by the date stipulated in the partiesrsquo contract Because the sailor had taken no action in response to the original repudiation he no longer had the right to cancel the contract with the builder The sailorrsquos subsequent statement that ldquoour contract is overrdquo may have constituted repudiation by the sailor In any event when the sailor failed to perform on December 15 that constituted breach

Point One (35) Because the sailor had reasonable grounds for insecurity with respect to the builderrsquos performance the sailorrsquos letter to the builder was a justified demand seeking assurance of the builderrsquos performance under the contract failure of the builder to provide such assurance within a reasonable time constituted repudiation of the contract

The sailor was legally justified in sending the letter to the builder on October 31 Contract parties are entitled to expect due performance of contractual obligations and are permitted to take steps to protect that expectation UCC sect 2-609 states that ldquo[w]hen reasonable grounds for insecurity arise with respect to the performance of either party the other may in writing demand adequate assurance of due performance rdquo Here the sailor learned on October 31 that the builderrsquos workers were on strike This gave the sailor reasonable grounds for insecurity about the builderrsquos ability to complete performance on time and thus gave the sailor the right to seek adequate assurance from the builder Because the sailorrsquos demand for assurance was justified the builder was required to provide assurance that was adequate under the circumstances within a reasonable time (not to exceed 30 days) or be held to have repudiated the contract UCC sect 2-609(4)

19

Contracts Analysis

Point Two(a) (30) The builder did not within a reasonable time provide the sailor adequate assurance of due performance this failure to provide assurance constituted a repudiation of the contract

Because the sailor with legal justification (see Point One) demanded from the builder assurance of due performance the builderrsquos failure to provide such assurance within a reasonable time was a repudiation of their contract See UCC sect 2-609(4) (ldquoAfter receipt of a justified demand[] failure to provide within a reasonable time not exceeding thirty days assurance of due performance is a repudiation of the contractrdquo) On October 31 the sailor requested that the builder provide adequate assurance regarding the completion of the boat by December 15 The builder did not respond to the sailorrsquos letter until November 25mdashnearly a month later Even if that response had been given in a reasonable time it nonetheless did not provide assurance of due performance It simply stated ldquoIrsquom sorry about the strike but it is really out of my hands I hope we settle it soon so that we can get back to workrdquo Therefore the builderrsquos November 25 response did not provide adequate assurance in response to the sailorrsquos justified request Thus the builder had repudiated the contract

Point Two(b) (35) Although the builder repudiated the contract with the sailor the builder probably retracted that repudiation on December 3 and the sailor was no longer entitled to cancel their contract Thus the sailorrsquos failure to perform the sailorrsquos obligations under the contract constituted a breach

The builderrsquos failure to provide adequate assurance of performance constituted a repudiation of their contract (see UCC sect 2-609(4)) but the builder was free to retract that repudiation until the sailor cancelled the contract or materially changed his position or indicated by communication or action that the sailor considered the repudiation to be final See UCC sect 2-611(1) (ldquoUntil the repudiating partyrsquos next performance is due he can retract his repudiation unless the aggrieved party has since the repudiation cancelled or materially changed his position or otherwise indicated that he considers the repudiation finalrdquo)

Here the facts state that before the builderrsquos December 3 telephone call to the sailor the sailor did nothing in response to the builderrsquos repudiation such as contracting with a third party for a boat The builderrsquos December 3 call informing the sailor that the boat would be timely delivered probably constituted a retraction of the repudiation because it clearly indicated to the sailor that the builder would be able to perform UCC sect 2-611(2) Thus after being so informed the sailor did not have the right to treat their contract as cancelled UCC sect 2-611(3) Accordingly the sailorrsquos failure to perform the sailorrsquos obligations under the contract by taking the boat and paying for it constituted a breach of the contract

20

CONSTITUTIONAL LAW ANALYSIS (Constitutional Law IVA F2b amp e)

ANALYSIS

Legal Problems

(1) Does AutoCorsquos operation of a ldquocompany townrdquo result in its actions counting as those of the state for purposes of constitutional analysis

(2) Does the expulsion of a schoolchild for failure to recite the Pledge of Allegiance violate the First Amendment as applied through the Fourteenth Amendment

(3) Does the arrest of a pamphleteer in connection with violation of an anti-littering rule where the littering is done by the recipients of leaflets distributed by the pamphleteer violate the First Amendment as applied through the Fourteenth Amendment

DISCUSSION

Summary

The First Amendment as applied through the Fourteenth Amendment applies only to state action It does not typically govern private actors However courts have found state action where the private actor has exercised a ldquopublic functionrdquo such as running a privately owned ldquocompany townrdquo as AutoCo has done here Thus First Amendment protections apply By requiring the son to participate in a mandatory Pledge of Allegiance ceremony AutoCo has compelled the expression of political belief in violation of the First Amendment as applied through the Fourteenth Amendment The fatherrsquos arrest in connection with breaching the anti-litter rule also violated the First Amendment as applied through the Fourteenth Amendment Although state actors can regulate the incidental effects of speech on the public streets on a content-neutral basis this power is limited and cannot extend to punishing a distributor of literature because of littering by third parties

Point One (30) AutoCorsquos operation of a company town (including a school) makes it a state actor under the public function strand of the state action doctrine

The individual rights protections of the Constitution apply only where there is ldquostate actionrdquomdash either direct action by the government or some action by a private party that is fairly attributable to the government As a general rule the actions of a private company like AutoCo or of a private school like the school operated by AutoCo would not constitute state action and the protections of the Constitution (in this case the First Amendment) would not apply

However there are situations in which the actions of a private actor are attributed to the state One such situation is when the private actor undertakes a public function There are not many bright-line rules in the Supreme Courtrsquos state action doctrine but one of them is this Where a private actor undertakes a ldquopublic functionrdquo the Constitution applies to those actions Where a corporation operates a privately owned ldquocompany townrdquo that provides essential services typically provided by a state actor the public function doctrine applies and the Constitution

21

Constitutional Law Analysis

binds agents of the town as if they were agents of the government See eg Marsh v Alabama 326 US 501 (1946) Here AutoCo does more than own the town it provides security services fire protection sanitation services and a school Thus the actions of AutoCo constitute state action and are governed by the Fourteenth Amendment

Point Two (35) The sonrsquos expulsion for failure to recite the Pledge of Allegiance violates the First Amendment as applied through the Fourteenth Amendment as a compelled expression of political belief

As explained in Point One the First Amendment applies to the school as a state actor Although children in public schools (and in schools subject to the First Amendment like

the Oakwood school) have some First Amendment rights Tinker v Des Moines Independent Community School District 393 US 503 506 (1969) schools have greater leeway to regulate the speech of students and teachers than the state would have outside the school context Hazelwood School Dist v Kuhlmeier 484 US 260 (1988) Morse v Frederick 551 US 393 (2007) However the Supreme Court has long held that public schools may not force their students to participate in a flag salute ceremony when it offends the political or religious beliefs of the students or their families West Virginia Board of Educ v Barnette 319 US 624 (1943) (invalidating a mandatory public school flag salute ceremony) see also Wooley v Maynard 430 US 705 (1977) (invalidating compelled expression of political belief on state-issued license plates)

In this case the school requires its students to participate in a flag salute and Pledge of Allegiance ceremony and punishes them when they refuse to participate Pursuant to this policy the school has expelled the son This expulsion violates the First Amendment ban on compelled expression

Point Three (35) Because the father was distributing leaflets in a traditional public forum his trespass arrest violated the First Amendment as applied through the Fourteenth Amendment

As explained in Point One AutoCo is treated as a state actor Thus Oakwoodrsquos commercial district is treated as government-owned property for purposes of the First Amendment Thus the leafleting here is subject to the First Amendment because it is an expressive activity Schneider v State of New Jersey Town of Irvington 308 US 147 (1939) When expression takes place on government-owned property government regulation of the expression is assessed under the public forum doctrine Public streets and sidewalks have long been held to be the classic example of a ldquotraditional public forumrdquo open to the public for expression Hague v CIO 307 US 496 515ndash16 (1939) Because the father was distributing leaflets while standing on a street corner in the commercial district his expressive activity occurred in a traditional public forum

When a state tries to regulate expressive activity in a traditional public forum it is prohibited from doing so based on the expressive activityrsquos content unless its regulation is narrowly tailored to achieve a compelling governmental interest (ldquostrict scrutinyrdquo) In this case however AutoCo is regulating the fatherrsquos expressive activity on the ostensibly neutral ground that his expressive activity has produced litter and made the street unsightly When a state tries to regulate expressive activity without regard to its content intermediate scrutiny applies Under intermediate scrutiny the true purpose of the regulation may not be the suppression of ideas (if so then strict scrutiny applies) the regulation must be narrowly tailored to achieve a significant

22

Constitutional Law Analysis

governmental interest and it must leave open ample alternative channels for expressive activity Ward v Rock Against Racism 491 US 781 791 (1989)

Here the application of the ordinance to the father will fail for two reasons First the Supreme Court has held that the governmentrsquos interest in keeping the streets clean is insufficient to ban leafleting in the public streets as the government power to regulate with incidental effects on public sidewalk speech is very limited See eg Schneider 308 US at 162 (leafletinglittering) Second the regulation (a blanket ban on distribution that results in littering) is not narrowly tailored to protect expression A narrowly tailored alternative would be prosecution only of people who litter Moreover the effect of the littering rule is likely to be a ban on all leafleting thus eliminating an entire class of means of expression This raises the possibility that there are not ldquoample alternative channels of communicationrdquo open to the father as required under the Courtrsquos standard of review for content-neutral regulation of speech

[NOTE Some examinees might argue that this is a ldquotime place and mannerrdquo restriction and that AutoCo might have greater latitude to regulate the public sidewalks under this theory This argument is incorrect for two reasons First the Supreme Court has held that the power to regulate speakers through littering laws is very limited for the reasons given and in the cases cited above But more generally a ldquotime place and mannerrdquo restriction involves the shifting of speech from one time and place to another or to another manner here there is no shifting but a direct punishment for expressive activity (albeit one couched in content-neutral terms) In addition some examinees might read the ordinance to be in effect a total ban on leafleting since most leafleting will produce some litter Those examinees might note that the Court has required total bans on an entire mode of expression to satisfy strict scrutiny and analyze the fatherrsquos prosecution here accordingly See United States v Grace 461 US 171 177 (1983) (invalidating ban on display of signs on public sidewalks surrounding US Supreme Court ldquo[a]dditional restrictions such as an absolute prohibition on a particular type of expression will be upheld only if narrowly drawn to accomplish a compelling governmental interestrdquo)]

23

SECURED TRANSACTIONS ANALYSIS (Secured Transactions IID E IVA B C)

ANALYSIS

Legal Problems

(1) Is a purchase-money security interest in consumer goods perfected even though there has been no filing of a financing statement

(2) Does a person who buys consumer goods for personal use take those goods free of a prior perfected purchase-money security interest in the goods

(3) Does a person who receives consumer goods as a gift take those goods subject to a prior perfected security interest in them

DISCUSSION

Summary

The retailerrsquos security interest in the bicycles was perfected even though no financing statement was filed because it was a purchase-money security interest in consumer goods A purchase-money security interest in consumer goods is automatically perfected upon attachment

The buyer is not subject to the retailerrsquos security interest in the bicycle that the buyer bought from the man Because the bicycle was consumer goods in the hands of the man and the retailer never filed a financing statement covering the bicycle the retailerrsquos security interest is not effective against someone like the buyer who bought the bicycle for value without knowledge of the retailerrsquos security interest and for personal use

On the other hand the retailerrsquos security interest continues in the bicycle given to the friend because the friend did not give value for the bicycle or buy it in the ordinary course of business

Point One (35) The retailerrsquos security interest in the bicycles attached on June 1 Because this interest was a purchase-money security interest in consumer goods it was automatically perfected when it attached

The retailerrsquos security interest in the bicycles attached on June 1 when the man bought the bicycles (acquiring rights in the collateral) signed a security agreement containing a description of the collateral and received value from the retailer (by being given credit with which to purchase the bicycles) UCC sect 9-203(a) amp (b)

Despite the retailerrsquos failure to file a financing statement its security interest was perfected Pursuant to UCC sect 9-309(1) a security interest is automatically perfected upon attachment if the goods are ldquoconsumer goodsrdquo and the security interest is a ldquopurchase-money security interestrdquo

In this case the bicycles sold by the retailer to the man were consumer goods at the time of sale The bicycles were ldquogoodsrdquo because they were ldquomovable when a security interest

24

Secured Transactions Analysis

attachesrdquo UCC sect 9-102(a)(44) They were also consumer goods because they were ldquobought for use primarily for personal family or household purposesrdquo UCC sect 9-102(a)(23) The retailerrsquos security interest in these consumer goods was also a ldquopurchase-money security interestrdquo A purchase-money security interest is an interest that secures a debt that was incurred in order to ldquoenable the debtor to acquire rights in or the use of the collateralrdquo UCC sect 9-103(a) (b)(1) Here the man incurred an obligation to the retailer to purchase the bicycles so the security interest he gave the retailer to secure that obligation was a purchase-money security interest

Because the retailerrsquos security interest was a purchase-money security interest in consumer goods it was automatically perfected on June 1 when the interest attached to the bicycles

Point Two (35) The buyer took the bicycle free of the retailerrsquos security interest because (i) the retailer did not file a financing statement covering the bicycle (ii) the bicycle was ldquoconsumer goodsrdquo and (iii) the buyer bought the bicycle for value without knowledge of the retailerrsquos security interest and for personal use

A security interest continues in collateral even after a sale or other disposition of that collateral unless the creditor authorized the disposition ldquofree of the security interestrdquo or another Article 9 exception applies UCC sectsect 9-201(a) and 9-315(a)(1)

However a buyer of goods like the buyer here can take free of a prior security interest in those goods under certain circumstances See UCC sectsect 9-317(b) (buyers who give value and receive delivery of goods without knowledge of an unperfected security interest in the goods) and 9-320(a) amp (b) (buyer in ordinary course of business buyer of consumer goods in a consumer-to-consumer transaction who gives value) In this case the retailerrsquos security interest was perfected when the buyer purchased the bicycle so UCC sect 9-317(b) does not protect the buyer The buyer also is not a protected ldquobuyer in ordinary course of businessrdquo because he did not purchase from a person who is in the business of selling bicycles See UCC sect 1-201(b)(9)

The buyer can however qualify for the protection of UCC sect 9-320(b) That section provides that a buyer of goods from a person who used them for personal family or household purposes takes free of a perfected security interest in the goods if (1) the buyer had no knowledge of the security interest (2) the buyer gave value for the goods (3) the buyer purchased the goods primarily for personal family or household purposes and (4) the purchase occurred before the filing of a financing statement covering the goods

The buyer met all of these criteria The man used the bicycle for personal purposes The buyer purchased the bicycle from the man and the buyer had no knowledge of the retailerrsquos security interest The buyer gave value ($400) for the bicycle and he bought it ldquoprimarily for personal family or household purposesrdquo as he planned to use it for recreation which is a personal rather than a business use Finally no financing statement had been filed Therefore under UCC sect 9-320(b) the buyer took free of the retailerrsquos security interest

Point Three (30) The retailerrsquos security interest continues in the bicycle that the man gave to the friend Thus the retailer can recover the bicycle from the friend because the friend did not give value for the bicycle or buy it in the ordinary course of business

25

Secured Transactions Analysis

As noted in Point Two the retailer did not authorize the man to dispose of the bicycle Consequently the retailerrsquos security interest continued in the bicycle even after the man transferred ownership of the bicycle to the friend See UCC sectsect 9-201(a) and 9-315(a)(1) The retailerrsquos security interest in the bicycle will be effective against the friend unless some other provision of Article 9 allows the friend to take the bicycle free of that security interest

Unfortunately for the friend there is no Article 9 provision that allows him to take free of the retailerrsquos interest The friendrsquos basic problem is that he is not a buyer of the bicyclemdashhe received the bicycle as a gift and did not give value for it Thus the friend is not protected by any of the applicable exceptions See UCC sectsect 9-317(b) (protecting buyers who give value for goods subject to an unperfected security interest) 9-320(a) (protecting buyers in ordinary course of business) and 9-320(b) (protecting buyers of consumer goods who give value)

In short the retailerrsquos security interest continues in the bicycle that the man gave to the friend The friend took the bicycle subject to that security interest

26

FEDERAL CIVIL PROCEDURE ANALYSIS (Federal Civil Procedure VIE)

ANALYSIS

Legal Problems

(1) Does a judgment in a prior action preclude a nonparty from suing the same defendant on a closely related claim when the nonparty and the original plaintiff are in a family relationship

(2) Does a judgment rendered in an earlier action preclude a nonparty from litigating an issue that was actually decided in the first suit

(3) May a nonparty to an earlier action invoke the judgment in that action to preclude a party to the prior action from relitigating an issue that the party had a full and fair opportunity to litigate in the earlier action

DISCUSSION

Summary

Pursuant to the doctrines of claim preclusion (res judicata) and issue preclusion (collateral estoppel) a judgment is binding on the parties thereto In the absence of privity nonparties to a prior suit cannot be bound by a judgment rendered in their absence Thus in the absence of privity a nonparty to the first suit is not precluded from presenting her claim in a second suit even if it is factually related to the claims and defenses presented in the first suit nor is she bound by determinations of issues made in the first suit A family relationship without more does not support a finding of privity For this reason Mother as a nonparty is not bound by the judgment in the Son-Driver action She may bring her separate claim for damage to her car and she is not precluded from litigating the question of whether she was negligent in the maintenance of her car

Driver on the other hand could be precluded from relitigating the issue of her negligence pursuant to the doctrine of non-mutual issue preclusion (also called non-mutual offensive collateral estoppel) which allows a nonparty to a prior action to invoke issue preclusion to prevent a party to that prior action from relitigating determinations of issues made therein However Mother may be prevented from invoking non-mutual collateral estoppel in this case because she could easily have joined her claim in the prior action but did not do so

[NOTE Federal common law governs the preclusive effect of a judgment rendered by a federal court sitting in diversity See Semtek Intrsquol Inc v Lockheed Martin Corp 531 US 497 508 (2001) But the Semtek Court concluded that federal common law in this context incorporates the preclusion law of the state in which the rendering federal court sits (unless the state law is incompatible with federal interests) id at 508ndash09 Thus State Arsquos preclusion law determines the preclusive effect of the judgment rendered in Sonrsquos suit against Driver The problem says that State A preclusion law is identical to federal preclusion law so the following analysis utilizes general principles of preclusion drawn from Supreme Court case law (announcing federal preclusion rules) and the Restatement (Second) of Judgments]

27

Federal Civil Procedure Analysis

Point One (35) Under the doctrine of claim preclusion the judgment rendered in the first action does not preclude Mother a nonparty from suing Driver for the damage to her car because the judgment binds only parties or those in privity with them and Mother and Son are not in privity

Driver may contend that the doctrine of claim preclusion (res judicata) precludes Mother from presenting a claim arising from the same nucleus of facts that was presented in the first action brought by Son According to the doctrine of claim preclusion ldquowhen a court of competent jurisdiction has entered a final judgment on the merits of a cause of action the parties to the suit and their privies are thereafter bound lsquonot only as to every matter which was offered and received to sustain or defeat the claim or demand but as to any other admissible matter which might have been offered for that purposersquordquo Commissioner of Internal Revenue v Sunnen 333 US 591 597 (1948) (citation omitted)

However the doctrine of claim preclusion does not apply to Mother on the facts of this problem First Mother was not a party to the earlier case ldquoIt is a principle of general application in Anglo-American jurisprudence that one is not bound by a judgment in personam in a litigation in which he is not designated as a party or to which he has not been made a party by service of processrdquo Taylor v Sturgell 553 US 880 884 (2008) (citing Hansberry v Lee 311 US 32 40 (1940)) see also RESTATEMENT (SECOND) OF JUDGMENTS sect 34(3) (1982) This rule reflects our ldquodeep-rooted historic tradition that everyone should have his own day in courtrdquo Martin v Wilks 490 US 755 762 (1989) (citation omitted) (superseded by statute on other grounds) Since Mother was not a party to the first suit she is not bound by the judgment unless an exception to the general rule applies

Mother might be bound by the prior judgment if she were considered to have been sufficiently in privity with Son that Son represented her interests in that action ldquoA person who is not a party to an action but who is represented by a party is bound by and entitled to the benefits of a judgment as though he were a partyrdquo RESTATEMENT (SECOND) OF JUDGMENTS sect 41(1) But there is no suggestion in the facts of the problem that Son who is an adult purported to represent Motherrsquos interests in the first suit ldquo[C]lose family relationships are not sufficient by themselves to establish privity with the original suitrsquos party or to bind a nonparty to that suit by the judgment entered therein rdquo Cuauhtli v Chase Home Finance LLC 308 Fed Appx 772 773 (5th Cir 2009) (citation omitted) accord 18A CHARLES ALAN WRIGHT ET AL FEDERAL

PRACTICE AND PROCEDURE sect 4459 (2d ed 2002) In Taylor v Sturgell supra the Supreme Court identified other special circumstances in

which nonparties may be bound by a prior judgmentmdashwhen a nonparty consents to be bound when a nonparty is in a pre-existing substantive legal relationship with a party (such as preceding and succeeding property owners) when a nonparty assumed control of the prior litigation when a party seeks to relitigate through a proxy or where a special statutory scheme seeks to foreclose successive litigation by nonparties See Taylor 553 US at 893ndash95 None of these circumstances exists here

Because Mother was not a party to the first suit and is not in privity with Son who is an adult the judgment in the first action does not preclude her from bringing her own claim against Driver

Point Two (35) Under the doctrine of issue preclusion the judgment rendered in the first action does not preclude Mother a nonparty from litigating the issue of her negligence in maintaining her carrsquos

28

Federal Civil Procedure Analysis

brake lights because the judgment binds only parties or those in privity with them and Mother and Son are not in privity

By its affirmative response to a special interrogatory the jury in the first action expressly concluded that ldquoMother negligently failed to ensure that the brake lights on her car were in proper working orderrdquo Driver may attempt to invoke the doctrine of issue preclusion to preclude Mother from relitigating this issue in the second action

[I]ssue preclusion arises in a second action on the basis of a prior decision when the same lsquoissuersquo is involved in both actions the issue was lsquoactually litigatedrsquo in the first action after a full and fair opportunity for litigation the issue was lsquoactually decidedrsquo in the first action by a disposition that is sufficiently lsquofinalrsquo lsquoon the meritsrsquo and lsquovalidrsquo it was necessary to decide the issue in disposing of the first action and the later litigation is between the same parties or involves nonparties that are subject to the binding effect or benefit of the first action Once these requirements are met issue preclusion is available not only to defend against a demand for relief but also as offensive support for a demand for relief Issue preclusion moreover is available whether or not the second action involves a new claim or cause of action

18 CHARLES ALAN WRIGHT ET AL FEDERAL PRACTICE AND PROCEDURE sect 4416 at 392ndash93 (2d ed) see also RESTATEMENT (SECOND) OF JUDGMENTS sect 27 (1982)

Here several of the elements necessary for issue preclusion are present The same issue is involved in both actionsmdashthe issue of Motherrsquos negligence in failing to maintain the brake lights on her car That issue was actually litigated in the first action and decided by the jury There is nothing to suggest anything less than a full and fair opportunity to litigate The judgment disposing of the issue was final

Nevertheless the judgment will not preclude Mother from relitigating the issue for two reasons First Mother was not a party to the first action and as explained above Mother and Son are not in privity Therefore she cannot be denied an opportunity to litigate the issue of her negligence Second it does not appear that the juryrsquos decision as to Motherrsquos negligence was necessary to the prior judgment against Driver Nothing suggests that the finding on Motherrsquos negligence had any bearing on the outcome of the first action

Point Three (30) Under the doctrine of non-mutual issue preclusion the judgment rendered in the first action might preclude Driver from relitigating the issue of her negligence However Driver has a strong argument that such a result would be inconsistent with the policy against offensive use of non-mutual estoppel when the non-party plaintiff easily could have joined as a plaintiff in the first action

Because Son already convinced the jury in the first action that ldquoDriver was negligent in the operation of her vehiclerdquo Mother may wish to invoke the doctrine of non-mutual issue preclusion to prevent Driver from relitigating the question of her negligence As noted above ldquoissue preclusion arises in a second action on the basis of a prior decision when the same lsquoissuersquo is involved in both actions the issue was lsquoactually litigatedrsquo in the first action after a full and fair opportunity for litigation the issue was lsquoactually decidedrsquo in the first action by a disposition that is sufficiently lsquofinalrsquo lsquoon the meritsrsquo and lsquovalidrsquo it was necessary to decide the issue in disposing of the first action rdquo 18 CHARLES ALAN WRIGHT ET AL FEDERAL PRACTICE AND

PROCEDURE sect 4416 at 392 (2d ed) see also RESTATEMENT (SECOND) OF JUDGMENTS sect 27

29

Federal Civil Procedure Analysis

Here these basic requirements for issue preclusion are met First the same issue is involved in both suits whether Driver was negligent in the operation of her car Second this issue was actually litigated and decided in the first action the jury answered a special interrogatory raising this very question There is nothing to suggest that Driver lacked a full and fair opportunity to litigate the issue Since a judgment was rendered against Driver for the injuries Son sustained as a result of Driverrsquos negligence resolution of the issue was necessary to dispose of the first action Driver was a party to the first action so she may be bound by the judgment

[NOTE Traditionally issue preclusion required mutualitymdashboth the party asserting issue preclusion and the party against whom issue preclusion was asserted were bound by the prior judgment Under the traditional mutuality rule Mother could not assert issue preclusion against Driver because Mother would not be bound by the judgment if Driver sought to rely on it See Point One There is no mutuality between Mother and Driver with respect to the prior judgment

This traditional mutuality requirement has been abandoned in most jurisdictions The Supreme Court rejected a strict mutuality requirement in Blonder-Tongue Laboratories Inc v University of Illinois Foundation 402 US 313 (1971) (non-mutual defensive collateral estoppel used by a defendant to preclude a plaintiff from relitigating a claim the plaintiff previously litigated) and Parklane Hosiery Co v Shore 439 US 322 (1979) (non-mutual offensive collateral estoppel used by a plaintiff to preclude a defendant from relitigating a claim the defendant previously litigated) In Parklane Hosiery the Court concluded (as a matter of federal preclusion law) that trial courts should have ldquobroad discretionrdquo to determine whether or not to permit a plaintiff to invoke non-mutual issue preclusion ldquoThe general rule should be that in cases where a plaintiff could easily have joined in the earlier action or where the application of offensive estoppel would be unfair to a defendant a trial judge should not allow the use of offensive collateral estoppelrdquo Id at 331

The Parklane Hosiery decision identified a number of circumstances that might make it unfair to allow a plaintiff to invoke non-mutual issue preclusion (non-mutual offensive collateral estoppel in the traditional terminology) against a defendant In particular the Parklane Hosiery court suggested that issue preclusion may not be appropriate if the plaintiff in the second action ldquocould easily have joined in the earlier actionrdquo Id Prohibiting plaintiffs from using non-mutual estoppel under such circumstances would promote judicial efficiency by encouraging plaintiffs to join the prior action It would also discourage plaintiffs from staying out of prior litigation in order to secure in effect two bites at the apple using the prior litigation offensively if the defendant loses and forcing the defendant to litigate a second time if the defendant wins the prior action

An exceptional exam answer might therefore argue that non-mutual issue preclusion should be denied on these facts Son and Mother both reside in State A since they are related they know each other well and Son was driving Motherrsquos car when the accident occurred They could have sued together and Rule 20 of the Federal Rules of Civil Procedure would have authorized joinder of their claims because those claims arose from the same transaction or occurrence and raised a common question of law or fact FED R CIV P 20(a) The facts do not suggest that Mother had any reason not to join Sonrsquos suit other than a desire to see how Sonrsquos action concluded before bringing her own claim Cf Nations v Sun Oil Co (Del) 695 F2d 933 938 (5th Cir 1983) (concluding that plaintiff ldquowas entitled to await the development of his injuries and their predictable consequencesrdquo) Because it appears that Mother may be a ldquowait-and-seerdquo plaintiff who could easily have joined the original action a trial court might disallow as a matter of discretion her use of non-mutual issue preclusion]

30

AGENCY ANALYSIS __________ (Agency I II)

ANALYSIS

Legal Problems

(1) Is the principal or the agent or both liable on contracts with a third party when the principal is an ldquoundisclosed principalrdquo

(2) Is the principal or the agent or both liable on contracts with a third party when the principal is ldquopartially disclosedrdquo or an ldquounidentified principalrdquo

(3) Is the principal or the agent or both liable on contracts with a third party for the purchase of goods when the agent exceeded his authority but the principal nonetheless accepts the goods

DISCUSSION

Summary

The agent but not the owner is liable to the basket manufacturer because the owner is an undisclosed principal and the agent acted without actual or apparent authority Both the agent and the owner however are liable on the burner contract because the owner is an unidentified principal and the agent had apparent authority to enter into that contract With respect to the solar cells contract whether the owner is liable depends upon whether a court would follow the Second or Third Restatement of Agency which take different positions on the effect of the ratification of a contract by an undisclosed principal Under either the agent would also be liable on the contract as he was a party to the contract

[NOTE The contracts that are the subject of this question are contracts for the sale of goods and therefore are governed by Article 2 of the Uniform Commercial Code Article 2 however does not contain agency rules Accordingly common law concepts of agency are applicable UCC sect 1-103(b)]

Point One (35) The agent but not the owner is liable to the basket manufacturer The agent had no actual authority to enter into the contract to buy aluminum baskets and because the owner was an undisclosed principal the manufacturer had no reason to believe that the agent had apparent authority Furthermore the manufacturer had no reason to believe that the agent was not contracting for his own benefit

An agent acting on behalf of a principal can bind the principal to contracts if the agent has either actual or apparent authority An agent has actual authority when contracting on behalf of his principal if he ldquoreasonably believes in accordance with the principalrsquos manifestations to the agent that the principal wishes the agent so to actrdquo RESTATEMENT (THIRD) OF AGENCY sect 201 (2006) Here the agent was told to buy only wicker baskets not aluminum baskets Thus when he contracted with the basket manufacturer to buy aluminum baskets he had no actual authority to do so

31

Agency Analysis

An agent acts with apparent authority ldquowhen a third party [with whom the agent acts] reasonably believes the actor has authority to act on behalf of the principal and that belief is traceable to the principalrsquos manifestationsrdquo Id sect 203 Here the owner notified basket manufacturers that she or her agent might contact them to purchase baskets but that notification did not specifically name the agent or any other person as the ownerrsquos agent Furthermore the basket manufacturer had no prior dealings with the agent or the owner or any reason to think that the agent was acting for the benefit of anyone but himself Thus there is no basis to conclude that the basket manufacturer thought that the agent had apparent authority to act for the owner

Generally when an agent acts on behalf of an undisclosed principal and the agent lacks authority to enter into the contract the agent is liable on the contract as a party to the contract but the principal is not liable This rule is consistent with the third partyrsquos expectations ldquoThe third party expected the agent to be a party to the contract because the agent presented the deal as if he were acting for himself Moreover if the third party is unaware of the principalrsquos existence the third party must be relying on the agentrsquos solvency and reliability when entering into the contractrdquo See ROBERT W HAMILTON JONATHAN R MACEY amp DOUGLAS K MOLL CORPORATIONS INCLUDING PARTNERSHIPS AND LIMITED LIABILITY COMPANIES 34 (11th ed 2010) See also RESTATEMENT (THIRD) OF AGENCY sect 603 cmt c Furthermore because the third party has no idea that the agent is acting or is seemingly acting on behalf of another there is no reason to believe that the third party would be expecting an undisclosed principal to be liable on the contract Id

Point Two (35) Because the owner is an unidentified (as opposed to undisclosed) principal both she and the agent (as a party to the contract) probably are liable on the contract with the burner manufacturer

When the agent contracted with the burner manufacturer he did not have actual authority to do so as the owner had expressly restricted the agentrsquos authority to purchase only burners with ldquowhisper technologyrdquo See Point One However the agent may have had apparent authority to buy burners without whisper technology

An agent acts with apparent authority ldquowhen a third party [with whom the agent acts] reasonably believes the actor has authority to act on behalf of the principal and that belief is traceable to the principalrsquos manifestationsrdquo RESTATEMENT (THIRD) OF AGENCY sect 203 (2006) The owner indicated that an agent might contact the burner manufacturer The notice contained no restriction regarding the type of burners that the agent was authorized to purchase The facts indicate that burner manufacturers regularly receive such notices

Although the agent told the burner manufacturer that he represented a well-known hot-air balloon operator he did not disclose the ownerrsquos name Thus the owner was a partially disclosed or unidentified principal See RESTATEMENT (SECOND) OF AGENCY sect 4(2) (1958) (using term ldquopartially disclosed principalrdquo) RESTATEMENT (THIRD) OF AGENCY sect 104(2)(c) (2006) (using term ldquounidentified principalrdquo) An agent for a partially disclosed principal may have apparent authority RESTATEMENT (SECOND) OF AGENCY sect 159 cmt e (1958) Based upon (1) the notice sent by the owner (2) the agentrsquos revelation that he was acting as an agent and (3) the fact that burner manufacturers regularly receive such notices and sell to agents the manufacturer may argue that it reasonably and actually believed that the agent was authorized to purchase burners without whisper technology The manufacturer may also argue that because the agent revealed that he was an agent his listing of the ownerrsquos address as the delivery address connects the agent to the notice given by the owner Arguably this distinguishes the burner contract from the basket

32

Agency Analysis

contract Here there is a strong case to support the conclusion that the agent had apparent authority if he did then the owner is liable to the burner manufacturer

The agent also is liable as a party to the contract because he did not fully disclose his agency relationship Although he told the burner manufacturer that he represented a well-known hot-air balloon operator he did not disclose the ownerrsquos name Generally even an authorized agent of a partially disclosed or unidentified principal is liable as a party to a contract with a third person RESTATEMENT (SECOND) OF AGENCY sect 321 (1958) (ldquounless otherwise agreedrdquo) RESTATEMENT (THIRD) OF AGENCY sect 602(2) (2006) (ldquounless the agent and the third party agree otherwiserdquo)

Point Three (30) Under the Second Restatement of Agency the owner is not liable on the contract for solar cells because the agent did not have actual or apparent authority and the owner as an undisclosed principal cannot ratify the contract Under the Third Restatement the owner could be liable as she ratified the contract Under either Restatement the agent is liable as a party to the contract

The owner is not liable to the solar cell manufacturer for breach of the contract for the solar cells because the agent had no actual or apparent authority to purchase solar cells on the ownerrsquos behalf and the owner under the Second Restatement of Agency did not ratify the contract with knowledge of the material facts Thus she is not liable as a ratifier of the contract

The facts state that the agent had authority to purchase only propane fuel tanks In addition he had no apparent authority to purchase solar cells The owner made no manifestations to the solar cell manufacturer that would lead a reasonable person in the manufacturerrsquos position to believe that the agent had the authority to bind the owner to a contract to purchase solar cells In fact the agent made no manifestations at all to the solar cell manufacturer Unlike with the basket manufacturer and the burner manufacturer the owner did not notify the manufacturer of solar cells that an agent might contact it to purchase solar cells In addition the solar cells were delivered to the agent and not to the ownerrsquos address In sum the manufacturer was unaware of any relationship between the owner and the agent As to the solar cell manufacturer the owner is an undisclosed principal There can be no apparent authority in the case of an undisclosed principal because there are no manifestations from the principal to the third person See RESTATEMENT (SECOND) OF AGENCY sect 8 cmt a (1958) (ldquothere can be no apparent authority created by an undisclosed principalrdquo) RESTATEMENT (THIRD) OF AGENCY sect 203 cmt f (2006) (ldquoapparent authority is not present when a third party believes that an interaction is with an actor who is a principalrdquo)

The owner also did not ratify the contract Although the owner used the solar cells generally a principal cannot ratify an unauthorized transaction with a third person ldquounless the one acting purported to be acting for the ratifierrdquo RESTATEMENT (SECOND) OF AGENCY sect 85(1) (1958)

The result differs under the Third Restatement which expressly rejects the Second Restatement on this issue The Restatement (Third) of Agency sect 403 (2006) states ldquoA person may ratify an act if the actor acted or purported to act as an agent on the personrsquos behalfrdquo According to comment b ldquoan undisclosed principal may ratify an agentrsquos unauthorized actrdquo Under the Restatement (Third) of Agency rule the owner probably ratified the transaction The agent clearly acted on the ownerrsquos behalf and in addition the ownerrsquos conduct in using the solar cells ldquojustifies a reasonable assumption that [she] is manifesting assent that the act shall affect [her] legal relationsrdquo See id sect 401(2)

33

Agency Analysis

The agent also is liable to the solar cell manufacturer for breach of the contract for the solar cells because he is a party to the contract The facts indicate that the agent never told the solar cell manufacturer that he represented the owner or any other principal Consequently even if the agent were authorized (which as discussed above he is not) he would be liable as a party to the contract See RESTATEMENT (SECOND) OF AGENCY sect 322 (1958) RESTATEMENT (THIRD) OF AGENCY sect 603(2) (2006) Here he has no authority or apparent authority and is liable as a party to the contract

The agent would also be liable under the Third Restatement Under Restatement (Third) of Agency sect 402(1) (2006) ratification generally relates back and the transaction is treated as if it were authorized at the time of the transaction However this does not relieve the agent of an undisclosed principal who ratifies an unauthorized transaction of liability under the ratified contract See id sect 603(2) (authorized agent for undisclosed principal is a party to the contract) and sect 403 cmt b (ldquoAn undisclosed principalrsquos ratification does not eliminate the agentrsquos liability to the third party on the transaction rdquo)

[NOTE An examinee may discuss the concept of inherent agency power This concept is recognized by the Restatement (Second) of Agency sect 8 A (1958) but the concept is not used in the Restatement (Third) of Agency (2006) Here there are no facts to support that the agent had inherent authority

As to contracts with agents for partially disclosed principals (eg the contract for the burners) the basic question is whether the acts done ldquousually accompany or are incidental to transactions which the agent is authorized to conductrdquo RESTATEMENT (SECOND) OF AGENCY

sect 161 (1958) If so the principal is bound if the other party ldquoreasonably believes that the agent is authorized to do them and has no notice that he is not so authorizedrdquo Id The purchase of burners without whisper technology was not authorized nor was it incidental to an authorized transaction Therefore there should not be inherent agency power

As to contracts on behalf of undisclosed principals (eg the other two contracts) the basic question is whether the acts done are usual or necessary in the transactions the agent is authorized to transact RESTATEMENT (SECOND) OF AGENCY sect 194 (1958) The other two contracts seem fundamentally different from the authorized transactions Therefore there should not be inherent agency power

Only minimal credit should be given for discussion of inherent agency power]

34

EVIDENCE ANALYSIS _____ (Evidence IIA VA B E F J K)

ANALYSIS

Legal Problems

(1) Is the authenticated copy of the mechanicrsquos text message relevant and admissible

(2) Is the womanrsquos question ldquoIs my scooter safe to drive for a whilerdquo relevant and admissible

(3) Is the womanrsquos testimony describing the mechanicrsquos thumbs-up relevant and admissible

DISCUSSION

Summary

The mechanicrsquos text message to the woman is relevant to whether (1) the woman lost control of the scooter due to its defective brakes (2) the woman knew that the brakes needed repair and (3) it was negligent for the woman to drive the scooter knowing that its brakes needed repair

The mechanicrsquos text message is hearsay if it is offered by the pedestrian to prove that the scooterrsquos brakes needed repair However it fits the hearsay exception for present sense impressions and probably also fits the exception for business records The mechanicrsquos text message is not hearsay if it is instead offered by the pedestrian to prove the womanrsquos state of mind (ie that she had notice that her brakes needed repair)

The womanrsquos question to the mechanic and his response are also relevant to whether the brakes caused the accident and whether the woman was negligent The question is not hearsay because the woman did not make an assertion

The mechanicrsquos thumbs-up response is nonverbal conduct intended by the mechanic as an assertion and is therefore an out-of-court statement If the woman offers the mechanicrsquos statement to prove that the scooter was actually safe to ride the womanrsquos testimony about the statement is hearsay

However the mechanicrsquos statement is not hearsay if it is offered by the woman to prove her state of mind Therefore the womanrsquos question and the mechanicrsquos response are admissible to prove the womanrsquos state of mind

Point One(a) (20) The mechanicrsquos text message to the woman should be admitted because it is relevant

Evidence is relevant if it has ldquoany tendency to make a fact more or less probable than it would be without the evidencerdquo FED R EVID 401 ldquoRelevant evidence is admissiblerdquo unless it is inadmissible pursuant to some other rule FED R EVID 402

The mechanicrsquos text message to the woman ldquoWhen you pick up your scooter you need to schedule a follow-up brake repair Wersquoll order the partsrdquo is relevant for two reasons First this evidence has some tendency to make it more probable that the brakes malfunctioned and

35

Evidence Analysis

caused the accident Second it has some tendency to make it more probable that the woman was negligent in riding her scooter after being told by the mechanic that it required further repair

Point One(b) (30) The mechanicrsquos text message fits either the hearsay exception for present sense impressions or the exception for business records or it is admissible non-hearsay

The mechanicrsquos text message is a statement under Rule 801(a) because it is ldquoa written assertionrdquo FED R EVID 801(a) The text message is hearsay if the pedestrian offers it to prove the ldquotruth of the matter asserted in the statementrdquo (ie that the scooterrsquos brakes required repair) which resulted in the woman losing control of the scooter and causing the accident FED R EVID 801(c)

However the mechanicrsquos text message fits the hearsay exception for ldquopresent sense impressionsrdquo under Rule 803(1) because it is ldquo[a] statement describing or explaining an event or condition made while or immediately after the declarant perceived itrdquo FED R EVID 803(1) Here the mechanicrsquos text message described the condition of the scooter immediately after he perceived it during the maintenance service

The mechanic is a person with knowledge of the condition of the scooter so if text messages regarding repairs were made and kept by the mechanic in the ordinary course of business this text message also fits the business records exception Under Rule 803(6) a business record is a record of an act ldquomade at or near the time by someone with knowledgerdquo and ldquothe record was kept in the course of a regularly conducted activity of a businessrdquo and ldquomaking the record was a regular practice of that activityrdquo FED R EVID 803(6)

However the text message is not hearsay if it is instead offered to prove that the woman was negligent because she rode her scooter after the mechanic told her it required repair If offered for this purpose it would not be offered for the truth of the matter asserted in the statement but to show the womanrsquos belief about the condition of the scooter (her state of mind)

Point Two (10) The womanrsquos question to the mechanic should be admitted because it is not hearsay

The womanrsquos question to the mechanic is relevant because along with the mechanicrsquos thumbs-up response (see Point Three) it has some tendency to make it more probable that the woman was not negligent andor that the scooter brakes did not malfunction and cause the accident FED R EVID 401 The womanrsquos question does not raise hearsay concerns because it is not an assertion

Hearsay is defined under Rule 801(a) as ldquoan oral assertion written assertion or nonverbal conductrdquo Although ldquoassertionrdquo is not further defined ldquoa favorite [definition] of writers in the [evidence] field for at least a century and a half [is that] the word simply means to say that something is so eg that an event happened or a condition existedrdquo 2 MCCORMICK ON

EVIDENCE sect 246 (6th ed 2006) Under this definition the womanrsquos question is not hearsay because it is not an assertion

Point Three(a) (20) The mechanicrsquos thumbs-up to the woman is a nonverbal assertion that is relevant and the womanrsquos testimony about that response is admissible

36

Evidence Analysis

Hearsay is defined under Rule 801(c) as a ldquostatementrdquo that is ldquoa personrsquos oral assertion written assertion or nonverbal conduct if the person intended it as an assertionrdquo FED R EVID 801(a) Here when the mechanic responded to the womanrsquos question (ldquoIs my scooter safe to ride for a whilerdquo) with a thumbs-up gesture the facts suggest that he intended his nonverbal conduct as an assertion that in his opinion the scooter was safe to ride

The mechanicrsquos assertion is relevant and admissible to prove that the woman was not negligent because the evidence makes it more probable that at the time of the accident she believed that the scooter was safe to ride despite the fact that the brakes required repair FED R EVID 401 Admission of the womanrsquos description of the mechanicrsquos thumbs-up for this purpose does not raise hearsay concerns because the evidence would not be offered for the truth of the matter asserted but to show the womanrsquos belief about the condition of the scooter (her state of mind)

Point Three(b) (20) The mechanicrsquos thumbs-up is relevant to determine whether the scooterrsquos brakes malfunctioned causing the accident but if offered for this purpose it is also hearsay

The mechanicrsquos nonverbal assertion is relevant to the determination of whether the scooterrsquos brakes malfunctioned causing the accident However if offered to prove the ldquotruth of the matter asserted in the statementrdquo (ie that the scooter was safe to ride for a while) it is hearsay that does not fit any hearsay exception

37

TRUSTS AND FUTURE INTERESTS ANALYSIS ____________________ (Trusts and Future Interests IC1 amp 4 G IIF)

ANALYSIS

Legal Problems

(1)(a) Was the revocable trust amendable

(1)(b) If the trust was amendable must the amendment have been executed in accordance with the state Statute of Wills in order to be valid

(2) If the trust amendment was valid does the amendment apply to the probate estate assets passing to the trust pursuant to Settlorrsquos will

(3) If the trust amendment was valid should the trust property be distributed to University

(4) If the trust amendment was not valid should the trust property be distributed to Settlorrsquos grandchild (her only heir) or held in further trust in accordance with the terms of the original trust instrument

DISCUSSION

Summary

A revocable trust is amendable even if the trust instrument does not expressly grant to the trust settlor a power to amend Both inter vivos trusts and amendments thereto are valid even though not executed in accordance with the requirements applicable to wills

Under the Uniform Testamentary Additions to Trusts Act a revocable trust may be amended at any time prior to the settlorrsquos death and the amendment applies to the disposition of assets conveyed to the trust pursuant to a will even if the will was executed prior to the date of the amendment

At Settlorrsquos death trust assets including probate assets passing to the trust under Settlorrsquos will would go to University if as is the case here the trust amendment was valid If the amendment was invalid the trust assets would continue to be held in further trust because there is no violation of the common law Rule Against Perpetuities

Point One(a) (30) Settlor retained the right to amend the inter vivos trust despite her failure to expressly reserve this power

At issue here is whether a retained power of revocation includes the power to amend sometimes referred to as the power to modify The Restatement (Second) of Trusts sect 331 cmt g provides that if a settlor has a power to revoke that retained power ordinarily includes a power to modify (amend) as well Comment g also notes that the power to amend includes both a power to withdraw trust assets and a power to ldquomodify the terms of the trustrdquo The Uniform Trust Code which provides that a power to revoke includes the power to amend is consistent with this view

38

Trusts and Future Interests Analysis

UNIF TRUST CODE sect 602 accord RESTATEMENT (THIRD) OF TRUSTS sect 63 cmt The theory is that even though a power to amend was not expressly retained by a settlor the goal of amendment assuming the power was not included in the power to revoke could easily be achieved by first revoking the trust and then creating a new trust with the same terms contemplated by the amendment To require this would put form over substance

Thus by expressly retaining the power to revoke the trust Settlor retained a power to amend the inter vivos trust despite her failure to expressly reserve this power

[NOTE Under the common law a trust is irrevocable unless the settlor expressly retains a power to revoke the trust Conversely under the Uniform Trust Code a trust is revocable unless the terms of the trust expressly provide otherwise See UNIF TRUST CODE sect 602 The Trust Codersquos position on revocation follows the minority view in the United States and is inconsistent with prior Restatements of Trusts (see Restatement (Second) of Trusts sect 330) Here the trust is revocable because Settlor expressly retained a power of revocation

The Uniform Trust Code has been adopted in 24 jurisdictions Alabama Arizona Arkansas District of Columbia Florida Kansas Maine Michigan Missouri Nebraska New Hampshire New Mexico North Carolina North Dakota Ohio Oregon Pennsylvania South Carolina Tennessee Utah Vermont Virginia West Virginia and Wyoming]

Point One(b) (10) Settlorrsquos amendment of the trust was valid despite her failure to have her signature to the trust amendment witnessed

Neither the common law nor state statutes require a trust instrument or an amendment to a trust instrument to be executed in accordance with the formalities prescribed for execution of a will Indeed an inter vivos trust that does not involve real estate can be created orally Under the Uniform Trust Code the only requirements for creating a valid inter vivos trust are intent the specification of beneficiaries and the designation of a trustee See UNIF TRUST CODE sect 402 accord RESTATEMENT (THIRD) OF TRUSTS sect 13

Here the amendment meets the requirements of both the Uniform Trust Code and the common law Thus the fact that Settlorrsquos signature was not witnessed when she signed the amendment to the trust does not make the amendment invalid

Point Two (20) Under the Uniform Testamentary Additions to Trusts Act a revocable trust may be amended at any time prior to the settlorrsquos death and the amendment applies to probate assets poured into the trust at the settlorrsquos death pursuant to the settlorrsquos will even when the will was executed prior to the date of the amendment

Historically property owned by an individual at her death passed to the individualrsquos heirs or to beneficiaries designated in a will executed with the formalities (writing signing witnessing) prescribed by state law However when a will devises property to the trustee of an inter vivos trust then the provisions of the trustmdashwhich may not have been executed in accordance with the formalities required for willsmdasheffectively determine who will receive the property Because of this possibility some early cases held that if an inter vivos trust was not executed with the same formalities required for a valid will then the trust was ineffective to dispose of probate assets poured into the trust at the settlorrsquos death pursuant to the settlorrsquos will

This line of cases has been overturned by the Uniform Testamentary Additions to Trusts Act (the Act) now Uniform Probate Code sect 2-511 Under the Act adopted in almost all

39

Trusts and Future Interest Analysis

jurisdictions a testamentary bequest to the trustee of an inter vivos trust established by the testator during his or her lifetime is valid if the trust is in writing it is identified in the testatorrsquos will and the trust instrument was executed before concurrently with or after the execution of the will Id The Act further specifies that such a bequest is valid even if the trust is amendable or revocable and that a later amendment applies to assets passing to the trust by a previously executed will

Thus because the trust amendment is valid its terms apply to assets received by Bank from Settlorrsquos estate

Point Three (10) If the trust amendment was valid then the trust assets including assets passing to the trust under Settlorrsquos will should go to University

Under the trust amendment all trust assets (including the assets of Settlorrsquos probate estate poured into the trust) pass to University The facts provide no basis for failing to comply with Settlorrsquos stated intentions

Point Four (30) If the trust amendment was invalid trust assets including assets received pursuant to Settlorrsquos will should be held in accordance with the terms of the original trust instrument because those terms do not violate the Rule Against Perpetuities

Under the dispositive terms of the original trust instrument Settlor created successive income interests in her surviving children and grandchildren with a remainder interest in her great-grandchildren Because the trust was revocable the period during which the common law Rule Against Perpetuities requires that interests vest (ie 21 years plus lives in being) began to run from the date Settlor no longer had a power of revocation (here her death) not the date on which the trust was created See JESSE DUKEMINIER STANLEY J JOHANSON JAMES LINDGREN amp ROBERT SITKOFF WILLS TRUSTS AND ESTATES 678 (7th ed 2005)

Under the common law Rule Against Perpetuities Settlorrsquos trust is thus valid At the time of Settlorrsquos death she was survived by no children one granddaughter and no great-grandchildren Because Settlor cannot have more children after her death the only income beneficiary of the trust is Settlorrsquos surviving granddaughter This granddaughter is the only person who can produce great-grandchildren of Settlor thus all great-grandchildren must of necessity be born during the lifetime of Settlorrsquos only surviving granddaughter who is a life in being The granddaughterrsquos interest vested at Settlorrsquos death and the great-grandchildrenrsquos interest will vest at the death of the granddaughter There is no need to wait the additional 21 years permitted under the Rule Thus under the common law and the statute given in the facts the nonvested interest in the great-grandchildren is valid

[NOTE Both modern wait-and-see statutes and the Uniform Statutory Rule Against Perpetuities upon which the statute in the facts is modeled provide that before using either reform to validate an otherwise invalid nonvested interest one should first determine if the nonvested interest violates the common law Rule If it does not then there is no need to reform This proposition which is applicable in all MEE user jurisdictions that have not simply abrogated the rule is tested by this problem]

40

NEGOTIABLE INSTRUMENTS ANALYSIS (Negotiable Instruments III IV V)

ANALYSIS

Legal Problems

(1)(a) What rights does a person in possession of a note that has been indorsed in blank by the payee have against the maker of the note

(1)(b) Which defenses may the maker of a note raise against a person entitled to enforce it who is not a holder in due course but is a transferee from a holder in due course

(2) What rights does a person entitled to enforce a note have against an indorser who transferred it for consideration with no warranties

(3) What rights does a person entitled to enforce a note have against a previous holder who transferred it as a gift without indorsing it

DISCUSSION

Summary

The niece is a holder of the note and is thus a person entitled to enforce it The chef the issuer of the note is obligated to pay it to the niece as the person entitled to enforce it The niece is not subject to any defense or claim of the chef relating to the improper repair of the oven because the niece has the rights of a holder in due course When the buyer bought the note from the repairman the buyer became a holder in due course of the note and thus took it free of any personal defenses the chef had against the repairman Even though the niece is not herself a holder in due course of the note the niece succeeded to the buyerrsquos rights as holder in due course and thus took free of the chefrsquos personal defenses

Because the chef refused to pay the note the niece can recover from the repairman on the repairmanrsquos obligation as indorser The niece cannot recover on the note against the buyer however because the buyer did not indorse the note (and thus incurred no indorserrsquos obligation) and the buyer did not receive any consideration for transfer of the note to the niece (and therefore made no transfer warranty)

[NOTE Although Article 9 of the Uniform Commercial Code governs the sale of promissory notes (a point that might be correctly noted by examinees) that Article does not determine the answer to any of the questions posed]

Point One(a) (20) The niece is the holder of the note and thus may enforce it against the chef who is the issuer of the note

The chef is the maker of the note and thus its issuer See UCC sectsect 3-103 3-105 The issuer of a note is obligated to pay it in accordance with its terms to a ldquoperson entitled to enforcerdquo it UCC sect 3-412 The niece is a ldquoperson entitled to enforcerdquo the note This is because the niece is the holder of the note and a holder of a note is a person entitled to enforce it UCC sect 3-301 The niece is the holder of the note because (i) the repairmanrsquos signature on the back of the note not

41

Negotiable Instruments Analysis

accompanied by words indicating a person to whom the note was made payable was a ldquoblank indorsementrdquo which had the effect of making the note a bearer instrument (ii) anyone in possession of a bearer instrument is a holder of it and (iii) the niece is in possession of the note See UCC sectsect 1-201(b)(21)(A) 3-204 and 3-205 Accordingly the chef has an obligation to the niece to pay the note in accordance with its terms and the niece may enforce that obligation

Point One(b) (40) The niece is not a holder in due course of the note but because she is a transferee from the buyer who was a holder in due course she has the same enforcement rights as the buyer Because the buyer as a holder in due course would have been able to enforce the note against the chef without being subject to defenses or claims arising from the improper repair the niece has the same rights and will not be subject to the chefrsquos defenses or claims about the repair

As noted in Point One(a) the chef has an obligation to the niece to pay the note in accordance with its terms However except against a person with the rights of a holder in due course the chef can raise any defenses or claims in recoupment that he would have if the claim on the note were an ordinary contract claim UCC sect 3-305 Thus except against a holder in due course the chef would be able to raise the improper repair as a defense or a claim in recoupment (a claim in response to the niecersquos claim)

But claims in recoupment and most defenses cannot be raised against a person with the rights of a holder in due course Against a holder in due course the chef can raise only the four ldquorealrdquo defenses listed in UCC sect 3-305(a)(1) (infancy duress lack of legal capacity or illegality that nullifies the obligation of the obligor under other law fraud in the factum discharge in insolvency proceedings) none of which is present here

The niece is not a holder in due course because she did not take the note for value See UCC sectsect 3-302(a)(2)(i) (criteria for holder in due course status) and 3-303(a) (definition of ldquovaluerdquo) But this does not mean that the niece is subject to the chefrsquos claim arising out of the improper repair The buyer was a holder in due course of the note because he took the note for value ($9500) in good faith and without notice of any facts that would have alerted him to the chefrsquos defense against the repairman UCC sect 3-302(a)(2) As a holder in due course the buyer owned the note free of the chefrsquos claim because that claim did not constitute a ldquorealrdquo defense UCC sect 3-305(b) When the buyer gave the note to the niece this constituted a ldquotransferrdquo of the note See UCC sect 3-203(a) When a note is transferred the transferee receives ldquoany right of the transferor to enforce the instrument including any right as a holder in due courserdquo UCC sect 3-203(b) Under this rule (also known as the ldquoshelter principlerdquo) the buyer transferred his freedom from the chefrsquos defenses to the niece and the niece can enforce the note free of the chefrsquos defenses

Point Two (20) Because the chef dishonored the note the niece can recover from the repairman on the repairmanrsquos obligation as indorser

The chefrsquos refusal to pay the note constituted dishonor See UCC sect 3-502 The repairman as an indorser of the note (see Point One(a)) incurred the obligations of an indorser under UCC sect 3-415(a) When a note has been dishonored one of the obligations of an indorser is to pay the amount of the note to a person entitled to enforce it Therefore the repairman is liable for the amount of the note to the niece a person entitled to enforce the note (so long as the niece gives proper notice of dishonor to the repairman)

42

Negotiable Instruments Analysis

[NOTE Because the repairman indorsed the note without warranties there are no transfer warranties UCC sect 3-416 cmt 5]

Point Three (20) The niece cannot recover on the note against the buyer as either indorser or warrantor because the buyer did not indorse the note and did not receive consideration for transferring the note to the niece

The buyer did not indorse the note and therefore did not incur the obligation of an indorser to pay the note upon dishonor

The niece cannot recover from the buyer under a transfer warranty theory because transfer warranties are made only by a person ldquowho transfers an instrument for considerationrdquo Here the buyer gave the instrument to the niece as a gift So the buyer made no transfer warranty UCC sect 3-416(a) Therefore the niece cannot recover from the buyer on that theory

43

National Conference of Bar Examiners 302 South Bedford Street | Madison WI 53703-3622 Phone 608-280-8550 | Fax 608-280-8552 | TDD 608-661-1275

wwwncbexorg e-mail contactncbexorg

  • Contents
  • Preface
  • Description of the MEE
  • Instructions
  • February 2013 Questions
    • Real Property Question
    • Contracts Question
    • Constitutional Law Question
    • Secured Transactions Question
    • Federal Civil Procedure Question
    • Agency Question
    • Evidence Question
    • Trusts and Future Interests Question
    • Negotiable Instruments Question
      • February 2013 Analyses
        • Real Property Analysis
        • Contracts Analysis
        • Constitutional Law Analysis
        • Secured Transactions Analysis
        • Federal Civil Procedure Analysis
        • Agency Analysis
        • Evidence Analysis
        • Trusts and Future Interests Analysis
        • Negotiable Instruments Analysis
            • ltlt ASCII85EncodePages false AllowTransparency false AutoPositionEPSFiles true AutoRotatePages None Binding Left CalGrayProfile (Dot Gain 20) CalRGBProfile (sRGB IEC61966-21) CalCMYKProfile (US Web Coated 050SWOP051 v2) sRGBProfile (sRGB IEC61966-21) CannotEmbedFontPolicy Error CompatibilityLevel 14 CompressObjects Tags CompressPages true ConvertImagesToIndexed true PassThroughJPEGImages true CreateJobTicket false DefaultRenderingIntent Default DetectBlends true DetectCurves 00000 ColorConversionStrategy CMYK DoThumbnails false EmbedAllFonts true EmbedOpenType false ParseICCProfilesInComments true EmbedJobOptions true DSCReportingLevel 0 EmitDSCWarnings false EndPage -1 ImageMemory 1048576 LockDistillerParams false MaxSubsetPct 100 Optimize true OPM 1 ParseDSCComments true ParseDSCCommentsForDocInfo true PreserveCopyPage true PreserveDICMYKValues true PreserveEPSInfo true PreserveFlatness true PreserveHalftoneInfo false PreserveOPIComments true PreserveOverprintSettings true StartPage 1 SubsetFonts true TransferFunctionInfo Apply UCRandBGInfo Preserve UsePrologue false ColorSettingsFile () AlwaysEmbed [ true ] NeverEmbed [ true ] AntiAliasColorImages false CropColorImages true ColorImageMinResolution 300 ColorImageMinResolutionPolicy OK DownsampleColorImages true ColorImageDownsampleType Bicubic ColorImageResolution 300 ColorImageDepth -1 ColorImageMinDownsampleDepth 1 ColorImageDownsampleThreshold 150000 EncodeColorImages true ColorImageFilter DCTEncode AutoFilterColorImages true ColorImageAutoFilterStrategy JPEG ColorACSImageDict ltlt QFactor 015 HSamples [1 1 1 1] VSamples [1 1 1 1] gtgt ColorImageDict ltlt QFactor 015 HSamples [1 1 1 1] VSamples [1 1 1 1] gtgt JPEG2000ColorACSImageDict ltlt TileWidth 256 TileHeight 256 Quality 30 gtgt JPEG2000ColorImageDict ltlt TileWidth 256 TileHeight 256 Quality 30 gtgt AntiAliasGrayImages false CropGrayImages true GrayImageMinResolution 300 GrayImageMinResolutionPolicy OK DownsampleGrayImages true GrayImageDownsampleType Bicubic GrayImageResolution 300 GrayImageDepth -1 GrayImageMinDownsampleDepth 2 GrayImageDownsampleThreshold 150000 EncodeGrayImages true GrayImageFilter DCTEncode AutoFilterGrayImages true GrayImageAutoFilterStrategy JPEG GrayACSImageDict ltlt QFactor 015 HSamples [1 1 1 1] VSamples [1 1 1 1] gtgt GrayImageDict ltlt QFactor 015 HSamples [1 1 1 1] VSamples [1 1 1 1] gtgt JPEG2000GrayACSImageDict ltlt TileWidth 256 TileHeight 256 Quality 30 gtgt JPEG2000GrayImageDict ltlt TileWidth 256 TileHeight 256 Quality 30 gtgt AntiAliasMonoImages false CropMonoImages true MonoImageMinResolution 1200 MonoImageMinResolutionPolicy OK DownsampleMonoImages true MonoImageDownsampleType Bicubic MonoImageResolution 1200 MonoImageDepth -1 MonoImageDownsampleThreshold 150000 EncodeMonoImages true MonoImageFilter CCITTFaxEncode MonoImageDict ltlt K -1 gtgt AllowPSXObjects false CheckCompliance [ None ] PDFX1aCheck false PDFX3Check false PDFXCompliantPDFOnly false PDFXNoTrimBoxError true PDFXTrimBoxToMediaBoxOffset [ 000000 000000 000000 000000 ] PDFXSetBleedBoxToMediaBox true PDFXBleedBoxToTrimBoxOffset [ 000000 000000 000000 000000 ] PDFXOutputIntentProfile () PDFXOutputConditionIdentifier () PDFXOutputCondition () PDFXRegistryName () PDFXTrapped False CreateJDFFile false Description ltlt ARA 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 BGR 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 CHS ltFEFF4f7f75288fd94e9b8bbe5b9a521b5efa7684002000410064006f006200650020005000440046002065876863900275284e8e9ad88d2891cf76845370524d53705237300260a853ef4ee54f7f75280020004100630072006f0062006100740020548c002000410064006f00620065002000520065006100640065007200200035002e003000204ee553ca66f49ad87248672c676562535f00521b5efa768400200050004400460020658768633002gt CHT ltFEFF4f7f752890194e9b8a2d7f6e5efa7acb7684002000410064006f006200650020005000440046002065874ef69069752865bc9ad854c18cea76845370524d5370523786557406300260a853ef4ee54f7f75280020004100630072006f0062006100740020548c002000410064006f00620065002000520065006100640065007200200035002e003000204ee553ca66f49ad87248672c4f86958b555f5df25efa7acb76840020005000440046002065874ef63002gt CZE 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 DAN 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 DEU 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 ESP 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 ETI 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 FRA 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 GRE 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 HEB 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 HRV (Za stvaranje Adobe PDF dokumenata najpogodnijih za visokokvalitetni ispis prije tiskanja koristite ove postavke Stvoreni PDF dokumenti mogu se otvoriti Acrobat i Adobe Reader 50 i kasnijim verzijama) HUN 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 ITA 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 JPN ltFEFF9ad854c18cea306a30d730ea30d730ec30b951fa529b7528002000410064006f0062006500200050004400460020658766f8306e4f5c6210306b4f7f75283057307e305930023053306e8a2d5b9a30674f5c62103055308c305f0020005000440046002030d530a130a430eb306f3001004100630072006f0062006100740020304a30883073002000410064006f00620065002000520065006100640065007200200035002e003000204ee5964d3067958b304f30533068304c3067304d307e305930023053306e8a2d5b9a306b306f30d530a930f330c8306e57cb30818fbc307f304c5fc59808306730593002gt KOR ltFEFFc7740020c124c815c7440020c0acc6a9d558c5ec0020ace0d488c9c80020c2dcd5d80020c778c1c4c5d00020ac00c7a50020c801d569d55c002000410064006f0062006500200050004400460020bb38c11cb97c0020c791c131d569b2c8b2e4002e0020c774b807ac8c0020c791c131b41c00200050004400460020bb38c11cb2940020004100630072006f0062006100740020bc0f002000410064006f00620065002000520065006100640065007200200035002e00300020c774c0c1c5d0c11c0020c5f40020c2180020c788c2b5b2c8b2e4002egt LTH ltFEFF004e006100750064006f006b0069007400650020016100690075006f007300200070006100720061006d006500740072007500730020006e006f0072011700640061006d00690020006b0075007200740069002000410064006f00620065002000500044004600200064006f006b0075006d0065006e007400750073002c0020006b00750072006900650020006c0061006200690061007500730069006100690020007000720069007400610069006b007900740069002000610075006b01610074006f00730020006b006f006b007900620117007300200070006100720065006e006700740069006e00690061006d00200073007000610075007300640069006e0069006d00750069002e0020002000530075006b0075007200740069002000500044004600200064006f006b0075006d0065006e007400610069002000670061006c006900200062016b007400690020006100740069006400610072006f006d00690020004100630072006f006200610074002000690072002000410064006f00620065002000520065006100640065007200200035002e0030002000610072002000760117006c00650073006e0117006d00690073002000760065007200730069006a006f006d00690073002egt LVI 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 NLD (Gebruik deze instellingen om Adobe PDF-documenten te maken die zijn geoptimaliseerd voor prepress-afdrukken van hoge kwaliteit De gemaakte PDF-documenten kunnen worden geopend met Acrobat en Adobe Reader 50 en hoger) NOR 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 POL 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 PTB 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 RUM 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 RUS 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 SKY ltFEFF0054006900650074006f0020006e006100730074006100760065006e0069006100200070006f0075017e0069007400650020006e00610020007600790074007600e100720061006e0069006500200064006f006b0075006d0065006e0074006f0076002000410064006f006200650020005000440046002c0020006b0074006f007200e90020007300610020006e0061006a006c0065007001610069006500200068006f0064006900610020006e00610020006b00760061006c00690074006e00fa00200074006c0061010d00200061002000700072006500700072006500730073002e00200056007900740076006f00720065006e00e900200064006f006b0075006d0065006e007400790020005000440046002000620075006400650020006d006f017e006e00e90020006f00740076006f00720069016500200076002000700072006f006700720061006d006f006300680020004100630072006f00620061007400200061002000410064006f00620065002000520065006100640065007200200035002e0030002000610020006e006f0076016100ed00630068002egt SLV 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 SUO 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 SVE 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 TUR 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 UKR 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 ENU (Use these settings to create Adobe PDF documents best suited for high-quality prepress printing Created PDF documents can be opened with Acrobat and Adobe Reader 50 and later) gtgt Namespace [ (Adobe) (Common) (10) ] OtherNamespaces [ ltlt AsReaderSpreads false CropImagesToFrames true ErrorControl WarnAndContinue FlattenerIgnoreSpreadOverrides false IncludeGuidesGrids false IncludeNonPrinting false IncludeSlug false Namespace [ (Adobe) (InDesign) (40) ] OmitPlacedBitmaps false OmitPlacedEPS false OmitPlacedPDF false SimulateOverprint Legacy gtgt ltlt AddBleedMarks false AddColorBars false AddCropMarks false AddPageInfo false AddRegMarks false ConvertColors ConvertToCMYK DestinationProfileName () DestinationProfileSelector DocumentCMYK Downsample16BitImages true FlattenerPreset ltlt PresetSelector MediumResolution gtgt FormElements false GenerateStructure false IncludeBookmarks false IncludeHyperlinks false IncludeInteractive false IncludeLayers false IncludeProfiles false MultimediaHandling UseObjectSettings Namespace [ (Adobe) (CreativeSuite) (20) ] PDFXOutputIntentProfileSelector DocumentCMYK PreserveEditing true UntaggedCMYKHandling LeaveUntagged UntaggedRGBHandling UseDocumentProfile UseDocumentBleed false gtgt ]gtgt setdistillerparamsltlt HWResolution [2400 2400] PageSize [612000 792000]gtgt setpagedevice

Page 7: February 2013 MEE Questions and Analyses

REAL PROPERTY QUESTION _______________

In 2008 a landlord and a tenant entered into a 10-year written lease commencing September 1 2008 for the exclusive use of a commercial building at a monthly rent of $2500 The lease contained a covenant of quiet enjoyment but no other covenants or promises on the part of the landlord

When the landlord and tenant negotiated the lease the tenant asked the landlord if the building had an air-conditioning system The landlord answered ldquoYes it doesrdquo The tenant responded ldquoGreat I will be using the building to manufacture a product that will be irreparably damaged if the temperature during manufacture exceeds 81 degrees for more than six consecutive hoursrdquo

On April 15 2012 the buildingrsquos air-conditioning system malfunctioned causing the building temperature to rise above 81 degrees for three hours The tenant immediately telephoned the landlord about this malfunction The tenant left a message in which he explained what had happened and asked the landlord ldquoWhat are you going to do about itrdquo The landlord did not respond to the tenantrsquos message

On May 15 2012 the air-conditioning system again malfunctioned This time the malfunction caused the building temperature to rise above 81 degrees for six hours The tenant telephoned the landlord and left a message describing the malfunction As before the landlord did not respond

On August 24 2012 the air-conditioning system malfunctioned again causing the temperature to rise above 81 degrees for 10 hours Again the tenant promptly telephoned the landlord The landlord answered the phone and the tenant begged her to fix the system The landlord refused The tenant then attempted to fix the system himself but he failed As a result of the air-conditioning malfunction products worth $150000 were destroyed

The next day the tenant wrote the following letter to the landlord

Irsquove had enough I told you about the air-conditioning problem twice before yesterdayrsquos disaster and you failed to correct it I will vacate the building by the end of the month and will bring you the keys when I leave

The tenant vacated the building on August 31 2012 and returned the keys to the landlord that day At that time there were six years remaining on the lease

On September 1 2012 the landlord returned the keys to the tenant with a note that said ldquoI repeat the air-conditioning is not my problem You have leased the building and you should fix itrdquo The tenant promptly sent the keys back to the landlord with a letter that said ldquoI have terminated the lease and I will not be returning to the building or making further rent paymentsrdquo After receiving the keys and letter the landlord put the keys into her desk To date she has neither responded to the tenantrsquos letter nor taken steps to lease the building to another tenant

On November 1 2012 two months after the tenant vacated the property the landlord sued the tenant claiming that she is entitled to the remaining unpaid rent ($180000) from September 1 for the balance of the lease term (reduced to present value) or if not that then damages for the tenantrsquos wrongful termination

Is the landlord correct Explain

3

CONTRACTS QUESTION _______________

On January 2 a boat builder and a sailor entered into a contract pursuant to which the builder was to sell to the sailor a boat to be specially manufactured for the sailor by the builder The contract price was $100000 The written contract signed by both parties stated that the builder would tender the boat to the sailor on December 15 at which time payment in full would be due

On October 15 the builderrsquos workers went on strike and there were no available replacements

On October 31 the builderrsquos workers were still on strike and no work was being done on the boat The sailor read a news report about the strike and immediately sent a letter to the builder stating ldquoI am very concerned that my boat will not be completed by December 15 I insist that you provide me with assurance that you will perform in accordance with the contractrdquo The builder received the letter on the next day November 1

On November 25 the builder responded to the letter stating ldquoIrsquom sorry about the strike but it is really out of my hands I hope we settle it soon so that we can get back to workrdquo

Nothing further happened until December 3 when the builder called the sailor and said ldquoMy workers are back and I have two crews working overtime to finish your boat Your boat is task one Donrsquot worry wersquoll deliver your boat by December 15thrdquo The sailor immediately replied ldquoI donrsquot trust you As far as Irsquom concerned our contract is over I am going to buy my boat from a shipyardrdquo Two days later the sailor entered into a contract with a competing manufacturer to buy a boat similar to the boat that was the subject of the contract with the builder

The builder finished the boat on time and tendered it to the sailor on December 15 The sailor reminded the builder about the December 3 conversation in which the sailor had announced that ldquoour contract is overrdquo and refused to take the boat and pay for it

The builder has sued the sailor for breach of contract

1 What was the legal effect of the sailorrsquos October 31 letter to the builder Explain

2 What was the legal effect of the builderrsquos November 25 response to the sailorrsquos October 31 letter Explain

3 What was the legal effect of the sailorrsquos refusal to take and pay for the boat on December 15 Explain

4

CONSTITUTIONAL LAW QUESTION

AutoCo is a privately owned corporation that manufactures automobiles Ten years ago AutoCo purchased a five-square-mile parcel of unincorporated land in a remote region of the state and built a large automobile assembly plant on the land To attract workers to the remote location of the plant AutoCo built apartment buildings and houses on the land and leased them to its employees AutoCo owns and operates a commercial district with shops and streets open to the general public AutoCo named the area Oakwood and provides security fire protection and sanitation services for Oakwoodrsquos residents AutoCo also built operates and fully funds the only school in the region which it makes available free of charge to the children of its employees

A family recently moved to Oakwood The father and mother work in AutoCorsquos plant rent an apartment from AutoCo and have enrolled their 10-year-old son in Oakwoodrsquos school Every morning the students are required to recite the Pledge of Allegiance while standing and saluting an American flag With the approval of his parents the son has politely but insistently refused to recite the Pledge and salute the flag at the school on the grounds that doing so violates his own political beliefs and the political beliefs of his family As a result of his refusal to say the Pledge the son has been expelled from the school

To protest the schoolrsquos actions the father walked into the commercial district of Oakwood While standing on a street corner he handed out leaflets that contained a short essay critical of the schoolrsquos Pledge of Allegiance policy Some of the passersby who took the leaflets dropped them to the ground An AutoCo security guard saw the litter told the father that Oakwoodrsquos anti-litter rule prohibits leaflet distribution that results in littering and directed him to cease distribution of the leaflets and leave the commercial district When the father did not leave and continued to distribute the leaflets the security guard called the state police which sent officers who arrested the father for trespass

1 Did the sonrsquos expulsion from the school violate the First Amendment as applied through the Fourteenth Amendment Explain

2 Did the fatherrsquos arrest violate the First Amendment as applied through the Fourteenth Amendment Explain

5

SECURED TRANSACTIONS QUESTION

On June 1 a bicycle retailer sold two bicycles to a man for a total purchase price of $1500 The man made a $200 down payment and agreed to pay the balance in one year The man also signed a security agreement that identified the bicycles as collateral for the unpaid purchase price and provided that the man ldquoshall not sell or dispose of the collateral until the balance owed is paid in fullrdquo The retailer never filed a financing statement reflecting this security interest

The man had bought the bicycles for him and his girlfriend to use on vacation However shortly after he bought the bicycles the man and his girlfriend broke up The man has never used the bicycles

On August 1 the man sold one of the bicycles at a garage sale to a buyer who paid the man $400 for the bicycle The buyer bought the bicycle to ride for weekend recreation

On October 1 the man gave the other bicycle to his friend as a birthday present The friend began using the bicycle for morning exercise

Neither the buyer nor the friend had any knowledge of the manrsquos dealings with the retailer

1 Does the buyer own the bicycle free of the retailerrsquos security interest Explain

2 Does the friend own the bicycle free of the retailerrsquos security interest Explain

6

FEDERAL CIVIL PROCEDURE QUESTION _____

Mother and Son who are both adults are citizens and residents of State A Mother owned an expensive luxury car valued in excess of $100000 Son borrowed Motherrsquos car to drive to a store in State A As Son approached a traffic light that had just turned yellow he carefully braked and brought the car to a complete stop Driver who was following immediately behind him failed to stop and rear-ended Motherrsquos car which was damaged beyond repair Son was seriously injured Driver is a citizen of State B

Son sued Driver in the United States District Court for the District of State A alleging that she was negligent in the operation of her vehicle Son sought damages in excess of $75000 for his personal injuries exclusive of costs and interest In her answer Driver alleged that Son was contributorily negligent in the operation of Motherrsquos car She further alleged that the brake lights on Motherrsquos car were burned out and that Motherrsquos negligent failure to properly maintain the car was a contributing cause of the accident

Following a trial on the merits in Sonrsquos case against Driver the jury answered the following special interrogatories

Do you find that Driver was negligent in the operation of her vehicle Yes

Do you find that Son was negligent in the operation of Motherrsquos car No

Do you find that Mother negligently failed to ensure that the brake lights on her car were in proper working order Yes

The judge then entered a judgment in favor of Son against Driver Driver did not appeal

Two months later Mother sued Driver in the United States District Court for the District of State A alleging that Driverrsquos negligence in the operation of her vehicle destroyed Motherrsquos luxury car Mother sought damages in excess of $75000 exclusive of costs and interest

State A follows the same preclusion principles that federal courts follow in federal-question cases

1 Is Motherrsquos claim against Driver barred by the judgment in Son v Driver Explain

2 Does the juryrsquos conclusion in Son v Driver that Mother had negligently failed to maintain the brake lights on her car preclude Mother from litigating that issue in her subsequent suit against Driver Explain

3 Does the juryrsquos conclusion in Son v Driver that Driver was negligent preclude Driver from litigating that issue in the Mother v Driver lawsuit Explain

7

_____

AGENCY QUESTION

Over 5000 individuals in the United States operate hot-air balloon businesses A hot-air balloon has four key components the balloon that holds the heated air the basket that houses the riders the propane burner that heats the air in the balloon and the propane storage tanks

The owner of a hot-air balloon business recently notified several basket and burner manufacturers that she or her agent might be contacting them to purchase baskets or burners The owner did not specifically name any person as her agent Basket and burner manufacturers regularly receive such notices from hot-air balloon operators Such notices typically include no restrictions on the types of baskets or burners agents might purchase for their principals

The owner then retained an agent to acquire baskets burners and fuel tanks from various manufacturers The owner authorized the agent to buy only (a) baskets made of woven wicker (not aluminum) (b) burners that use a unique ldquowhisper technologyrdquo (so as not to scare livestock when the balloon sails over farmland) and (c) propane fuel tanks

The agent then entered into three transactions with manufacturers all of whom had no prior dealings with either the owner or the agent

(1) The agent and a large manufacturer of both wicker and aluminum baskets signed a contract for the purchase of four aluminum baskets for a total cost of $60000 The agent never told the manufacturer that he represented the owner or any other principal The contract listed the agent as the buyer and listed the ownerrsquos address as the delivery address but did not indicate that the address was that of the owner rather than the agent When the baskets were delivered to the owner she learned for the first time that the agent had contracted to buy aluminum not wicker baskets The owner immediately rejected the baskets and returned them to the manufacturer Neither the owner nor the agent has paid the basket manufacturer for them

(2) The agent contacted a burner manufacturer and told him that the agent represented a well-known hot-air balloon operator who wanted to purchase burners The agent did not disclose the ownerrsquos name The agent and the burner manufacturer signed a contract for the purchase of four burners that did not have ldquowhisper technologyrdquo for a total price of $70000 The burner contract like the basket contract listed the ownerrsquos address for delivery but did not disclose whose address it was The burners were delivered to the ownerrsquos business and the owner discovered that the agent had ordered the wrong kind of burners The owner rejected the burners and returned them to the manufacturer Neither the owner nor the agent has paid the burner manufacturer for the burners

(3) The agent contracted with a solar cell manufacturer to make three cells advertised as ldquostrong enough to power all your ballooning needsrdquo The agent did not tell the manufacturer that he was acting on behalf of any other person One week after the cells were delivered to the agent he took them to the owner who installed them and discovered that she could save a lot of money using solar cells instead of propane to power her balloons The owner decided to keep the solar cells but she has not paid the manufacturer for them

8

Agency Question

Assume that the rejection of the baskets and the burners and the failure to pay for the solar cells constitute breach of the relevant contracts

1 Is the owner liable to the basket manufacturer for breach of the contract for the aluminum baskets Is the agent liable Explain

2 Is the owner liable to the burner manufacturer for breach of the contract for the burners Is the agent liable Explain

3 Is the owner liable to the solar cell manufacturer for breach of the contract for the solar cells Is the agent liable Explain (Do not address liability based upon restitution or unjust enrichment)

9

EVIDENCE QUESTION _____

A woman who owns a motorized scooter brought her scooter to a mechanic for routine maintenance service As part of the maintenance service the mechanic inspected the braking system on the scooter As soon as the mechanic finished inspecting and servicing the scooter he sent the woman a text message to her cell phone that read ldquoJust finished your service When you pick up your scooter you need to schedule a follow-up brake repair Wersquoll order the partsrdquo

The woman read the mechanicrsquos text message and returned the next day to pick up her scooter As the woman was wheeling her scooter out of the shop she saw the mechanic working nearby and asked ldquoIs my scooter safe to ride for a whilerdquo The mechanic responded by giving her a thumbs-up The woman waved and rode away on the scooter

One week later while the woman was riding her scooter a pedestrian stepped off the curb into a crosswalk and the woman collided with him causing the pedestrian severe injuries The woman had not had the scooterrsquos brakes repaired before the accident

The pedestrian has sued the woman for damages for his injuries resulting from the accident The pedestrian has alleged that (1) the woman lost control of the scooter due to its defective brakes (2) the woman knew that the brakes needed repair and (3) it was negligent for the woman to ride the scooter knowing that its brakes needed to be repaired

The woman claims that the brakes on the scooter worked perfectly and that the accident happened because the pedestrian stepped into the crosswalk without looking and the woman had no time to stop The woman the pedestrian and the mechanic will testify at the upcoming trial

The pedestrian has proffered an authenticated copy of the mechanicrsquos text message to the woman

The woman plans to testify that she asked the mechanic ldquoIs my scooter safe to ride for a whilerdquo and that he gave her a thumbs-up in response

The evidence rules in this jurisdiction are identical to the Federal Rules of Evidence

Analyze whether each of these items of evidence is relevant and admissible at trial

1 The authenticated copy of the mechanicrsquos text message

2 The womanrsquos testimony that she asked the mechanic ldquoIs my scooter safe to ride for a whilerdquo and

3 The womanrsquos testimony describing the mechanicrsquos thumbs-up

10

TRUSTS AND FUTURE INTERESTS QUESTION

Ten years ago Settlor validly created an inter vivos trust and named Bank as trustee The trust instrument provided that Settlor would receive all of the trust income during her lifetime The trust instrument further provided that

Upon Settlorrsquos death the trust income shall be paid in equal shares to Settlorrsquos surviving children for their lives Upon the death of the last surviving child the trust income shall be paid in equal shares to Settlorrsquos then-living grandchildren for their lives Upon the death of the survivor of Settlorrsquos children and grandchildren the trust corpus shall be distributed in equal shares to Settlorrsquos then-living great-grandchildren

The trust instrument expressly specified that the trust was revocable but it was silent regarding whether Settlor could amend the trust instrument

Immediately after creating the trust Settlor validly executed a will leaving her entire estate to Bank as trustee of her inter vivos trust to ldquohold in accordance with the terms of the trustrdquo

Five years ago Settlor signed an amendment to the inter vivos trust The amendment changed the disposition of the remainder interest specifying that all trust assets ldquoshall be paid upon Settlorrsquos death to Universityrdquo Settlorrsquos signature on this amendment was not witnessed

A state statute provides that any trust interest that violates the common law Rule Against Perpetuities ldquois nonetheless valid if the nonvested interest in the trust actually vests or fails to vest either (a) within 21 years of lives in being at the creation of the nonvested interest or (b) within 90 years of its creationrdquo

Recently Settlor died leaving a probate estate of $200000 She was survived by no children one granddaughter (who would be Settlorrsquos only heir) and no great-grandchildren The granddaughter has consulted your law firm and has raised four questions regarding this trust

1 Was Settlorrsquos amendment of the inter vivos trust valid Explain

2 Assuming that the trust amendment was valid do its provisions apply to Settlorrsquos probate assets Explain

3 Assuming that the trust amendment was valid how should trust assets be distributed Explain

4 Assuming that the trust amendment was invalid how should trust assets be distributed Explain

11

NEGOTIABLE INSTRUMENTS QUESTION

A chef entered into a contract with a repairman pursuant to which the repairman agreed to repair the chefrsquos commercial oven for $10000 The repairman agreed to accept as payment a negotiable promissory note for $10000 payable two months after its issuance

After the repairman worked on the oven the chef gave him a $10000 note as payment for the work As agreed the note was signed by the chef as maker was payable to the order of the repairman was payable in two months and fulfilled all criteria for negotiability

The next day the repairman sold the note to a buyer for $9500 To effectuate the sale the repairman wrote ldquono warrantiesrdquo on the back of the note signed his name immediately below that and handed the note to the buyer The buyer bought the note in good faith and without knowledge of any facts relating to the work that the repairman had performed for the chef

Later the buyer gave the note to his niece as a gift To effectuate the gift the buyer handed the note to the niece but did not indorse it

Shortly thereafter the chef discovered that the repair work had been done improperly and the oven still did not function correctly The chef tried repeatedly to get the repairman to return to correct the repair work but the repairman ignored all the chefrsquos calls

On the notersquos due date the niece contacted the chef and demanded that he pay the amount of the note to her The chef refused and told the niece that he would not pay the note because the repairman did not properly repair the oven

1 What are the niecersquos rights against the chef Explain

2 What are the niecersquos rights against the repairman Explain

3 What are the niecersquos rights against the buyer Explain

12

February 2013 MEE

ANALYSES

Real Property Contracts

Constitutional Law Secured Transactions

Federal Civil Procedure Agency

Evidence Trusts and Future Interests

Negotiable Instruments

REAL PROPERTY ANALYSIS (Real Property ID1a 4 amp 5)

ANALYSIS

Legal Problems

(1) Does the tenant have a defense to the landlordrsquos action for unpaid rent based on constructive eviction

(2) Does the tenant have a defense to the landlordrsquos action for unpaid rent based on the tenantrsquos surrender of the premises

(3) What if anything may the landlord recover from the tenant for the period after the tenant vacated the building

DISCUSSION

Summary

Under the common law the tenant does not have a defense to the landlordrsquos action for unpaid rent based on constructive eviction Constructive eviction is based on the tenant proving that (1) the landlord breached a duty to the tenant (2) the breach caused a loss by the tenant of the substantial use and enjoyment of the premises (3) the tenant gave the landlord adequate notice and opportunity to repair and (4) the tenant vacated the leased premises Here there was no constructive eviction because although the tenant vacated and gave the landlord adequate notice the landlord breached no express or implied duty to the tenant to repair the premises

The tenant does not have a defense based on the landlordrsquos acceptance of his surrender of the premises a landlordrsquos retention of keys does not constitute an acceptance of the tenantrsquos surrender unless the landlord so intended and here the landlordrsquos statements to the tenant at the time of the surrender of the keys do not evidence the intent to accept the tenantrsquos surrender

Under the common law a landlord has no duty to mitigate damages but also cannot sue for rents due in the future Under this approach the landlord can sue only for past-due rents Using this approach on November 1 the landlord could recover all the rent past due (ie rent for September and October) but could not recover for rents due in the future However some courts have authorized recovery for future rent minus the fair market rental value of the premises It is thus possible that the landlord could recover damages equal to the amount of rent due from September 1 to the end of the six-year lease term ($180000) minus the propertyrsquos fair-market rental value over that same period

Point One (45) The tenant was not constructively evicted because the landlord had no duty to repair the commercial premises that were the subject of the lease

The landlord and the tenant entered into a term-of-years lease because the lease specified both a beginning and an ending date HERBERT HOVENKAMP amp SHELDON F KURTZ THE LAW OF

PROPERTY 256 (5th ed 2001) Although a term-of-years lease normally cannot be terminated by the tenant prior to the end of the term a tenant may terminate a term-of-years lease if the tenant

15

Real Property Analysis

is constructively evicted See id at 286ndash88 Typically as here a claim of constructive eviction is made as a defense to a landlordrsquos action for damages or unpaid rent

In order to establish a constructive eviction the tenant must prove that the landlord breached a duty to the tenant such as a duty to repair and that the landlordrsquos breach caused a loss of the substantial use and enjoyment of the premises The tenant must also show that he gave the landlord notice adequate to permit the landlord to meet his duty to the tenant and that the tenant vacated the leased premises Id see also JOHN G SPRANKLING UNDERSTANDING

PROPERTY LAW sect 1704 (2d ed 2007) Under the common law there was no implied duty on the part of a landlord to repair

leased premises such a duty arose only if expressly set forth in the lease SPRANKLING supra sect 1702[B] Here the written lease contained no term requiring the landlord to repair the air-conditioning Even if the conversation created a lease term that the building had air-conditioning that itself should not create a duty for the landlord to repair it

Over the past several decades courts have generally implied a duty to repair in residential leases either as part of a revised constructive eviction doctrine or based on an implied warranty of habitability JOSEPH W SINGER PROPERTY 469ndash70 (3d ed 2010) This shift has been justified based on the economic disparity between the typical landlord and tenant as well as the fact that residential tenants generally lack both the authority to authorize repairs to common areas of a building and the incentive to make repairs that will ultimately benefit the landlord

However courts have been more reluctant to imply a duty to repair in commercial leases a context in which the tenant is often a valuable business and in a better position to assess and make repairs than is the landlord But see eg Davidow v Inwood North Professional Group 747 SW2d 373 (Tex 1988) When courts have implied a duty to repair in a commercial lease it is typically when the repair has been mandated by public authorities and involves work so substantial that it would not ordinarily fall within the tenantrsquos common law repair duty andor the value of the repair would primarily inure to the landlordrsquos reversionary interest See Brown v Green 884 P2d 55 (Cal 1994) Eugene L Grant et al The Tenant as Terminator Constructive Eviction in Modern Commercial Leases 2 THE COMMERCIAL PROPERTY LEASE ch 15 (ABA 1997) Some courts have also permitted constructive eviction claims by commercial tenants of office buildings based on repairs required in common areas of the building See id Echo Consulting Services Inc v North Conway Bank 669 A2d 227 (NH 1995)

Here the tenant is the owner of a valuable manufacturing operation and is the exclusive occupant of the building the repair has not been mandated by public authorities and the repair is not structural To the contrary the repair involves a feature of the building of unusual importance in the tenantrsquos manufacturing operation and the tenant is likely far more knowledgeable than the landlord about the air-conditioning specifications necessary for the manufacture of the tenantrsquos product

Based on these facts it is unlikely that a court will find that the tenant in this case was constructively evicted Although the tenant can show that he gave adequate notice to the landlord of the air-conditioning malfunction and vacated the premises the lease was commercial and it did not contain any promises or covenants by the landlord except a covenant of quiet enjoyment a covenant of quiet enjoyment does not entail any repair obligations

[NOTE An examineersquos conclusion is less important than his or her demonstrated awareness of the elements of constructive eviction and the need to imply a repair duty for such a defense to be viable here Although the implied warranty of habitability is not available to this tenant Texas Minnesota and Massachusetts imply a warranty of suitability in commercial leases in limited circumstances and an examinee might argue that this warranty should apply

16

Real Property Analysis

here If an examinee concludes that this warranty applies he or she should discuss the other requirements for constructive eviction

If the examinee wrongly concludes that the first element for a constructive eviction has been met the examinee will then have to discuss the remaining three elements in order to conclude that the tenant can claim constructive eviction The tenant would have a strong argument that the second elementmdashsubstantial interference with the use and enjoyment of the premisesmdashalso is met As indicated above the landlord was aware that a functioning air-conditioning system was vital to the tenantrsquos manufacturing operations The facts further indicate that the system had failed three times in the past few months The landlord may try to argue that the malfunctions did not substantially interfere with the tenantrsquos use of the premises because the malfunctions caused the temperature to climb above 81 degrees for only a short period of timemdash 3 hours 6 hours and 10 hours respectivelymdashon each occasion The tenant will argue however that the landlord was aware that the tenantrsquos manufacturing operations could tolerate temperatures above 81 degrees for no more than 6 hours The final malfunction exceeded that limit destroying $150000 worth of the tenantrsquos products

The tenant would also have a strong argument that the third element is met notice and opportunity to cure The tenant notified the landlord of the problem immediately upon the systemrsquos first malfunction and did so again when it malfunctioned a second time and then a third time The landlord might argue that there was insufficient time to cure the problem because the system corrected itself within a few hours on the first and second times Although the malfunction lasted more than 10 hours the third time the landlord might argue that the time period was insufficient to get a repair person on the premises A court would be likely to find this argument unpersuasive however because the landlord could have attempted to correct the problem after the first and second malfunctions

Assuming that the landlord was given sufficient notice and opportunity to cure a court would be likely to conclude that the tenant also satisfied the final element of vacating the premises within a reasonable time The landlord might argue that the tenant remained in the premises for almost four months after the air conditioning first failed which would suggest that the problem was not so severe as to have constructively evicted the tenant The tenant will argue however that he gave the landlord three months to cure the problem after the first two malfunctions threatened (but did not actually harm) his operations The tenant then moved out shortly after the final malfunction caused temperatures to exceed the tolerance levels of his manufacturing operations]

Point Two (10) The landlord did not accept the tenantrsquos surrender of the lease

When a tenant wrongfully moves from leased premises with the intent to terminate the lease the landlord may either accept the tenantrsquos surrender of the premises and terminate the lease or hold the tenant to the terms of the lease See HOVENKAMP amp KURTZ supra at 295ndash96 Here the tenantrsquos only basis for the claim that the landlord accepted his surrender is the landlordrsquos retention of the keys Many courts have considered whether a landlordrsquos retention of keys delivered by a tenant constitutes acceptance of surrender The weight of the case law holds that retention of the keys alone does not constitute acceptance of surrender without other evidence showing that the landlord intended to accept the surrender See generally 49 AM JUR 2d Landlord and Tenant sect 213

Here the landlordrsquos note saying ldquoI repeat the air-conditioning is not my problem You have leased the building and you should fix itrdquo strongly suggests that the landlord did not intend

17

Some courts have rejected the no-mitigation-of-damages rule based on efficiency concerns and societyrsquos interest in assuring that resources remain in the stream of commerce rather than lying vacant see id at 464ndash65 and allow landlords to sue tenants who have wrongfully terminated a lease for damages equal to the difference between the unpaid rent due under the lease and the propertyrsquos fair market rental value Other courts have abandoned the no-recovery-for-future-rent rule These courts responding to the fact that a tenant may well disappear or be judgment-proof by the time a lease term is concluded have allowed a landlord to collect damages equal to the value of rent over the entire lease term minus the propertyrsquos fair rental value when a tenant has wrongfully terminated a lease and unequivocally shown an intention not to return to the premises or pay future rent Under this approach a landlord receives approximately the same amount he would have received were there a duty to mitigate damages See Sagamore Corp v Willcutt 180 A 464 (Conn 1935)

Real Property Analysis

to accept the tenantrsquos surrender The tenant might argue that the landlordrsquos failure to make a similar statement when the keys were sent to her a second time and she retained them evidences a change of heart However it is likely that a court would find that the landlordrsquos retention of the keys represented a decision to safeguard the keys not to accept the tenantrsquos surrender

[NOTE An examinee should receive credit for arguing the other way with a well-reasoned argument]

Point Three (45) Under the common law the landlord had no duty to mitigate damages Additionally a landlord was not entitled to recover unpaid rents due in the future but was only entitled to recover rents in arrears at the time of the commencement of the suit Applying the common law here the landlord could recover $5000 the amount of rents due at the commencement of the suit ($2500 for September and the same for October) Today some courts allow the landlord under certain circumstances to sue the tenant for damages (not rent) equal to the difference if any between the unpaid promised rent for the balance of the term (here $175000) and the propertyrsquos fair rental value for the balance of the term

Under the common law because a lease was viewed as a conveyance instead of a contract a landlord had no duty to mitigate damages resulting from a tenantrsquos wrongful termination of a lease A landlord could thus recover the full value of rents that were due and unpaid at the time of the suit However under the common law a landlord could not sue a tenant for rents due in the future because there was always a possibility that the tenant might pay the rent when it was due See SINGER supra at 462 Thus using the common law approach on November 1 the landlord could only recover the full value of the two monthsrsquo rent actually due and unpaid ie $5000 for September and October

Here because the tenant returned the keys to the landlord and said ldquoI will not be returning to the building or making further rent paymentsrdquo the landlord could establish abandonment and an intention not to return It is thus possible that the landlord might recover damages in the amount of $5000 (for the months of September and October) plus the present value of $175000 minus the fair market rental value of the property over the remaining months of the lease

18

CONTRACTS ANALYSIS ____ (Contracts II IVE)

ANALYSIS

Legal Problems

(1) What was the legal effect of the sailorrsquos October 31 letter to the builder

(2)(a) What was the legal effect of the builderrsquos November 25 response to the sailorrsquos October 31 letter

(2)(b) What was the legal effect of the sailorrsquos refusal to take and pay for the boat on December 15

DISCUSSION

Summary

This is a sale of goods governed by the Uniform Commercial Code Because the sailor had reasonable grounds for insecurity about the builderrsquos ability to deliver the boat in a timely manner when the sailor learned about the strike on October 31 the sailor was legally justified in sending the letter to the builder seeking adequate assurance of the builderrsquos performance pursuant to the contract The builderrsquos failure to provide such assurance within a reasonable time operated as a repudiation of the contract However the builder was free to retract the repudiation before the sailor either cancelled the contract or materially changed position in reliance on the builderrsquos repudiation The builder retracted the repudiation when he informed the sailor that the workers were back and that the boat would be delivered by the date stipulated in the partiesrsquo contract Because the sailor had taken no action in response to the original repudiation he no longer had the right to cancel the contract with the builder The sailorrsquos subsequent statement that ldquoour contract is overrdquo may have constituted repudiation by the sailor In any event when the sailor failed to perform on December 15 that constituted breach

Point One (35) Because the sailor had reasonable grounds for insecurity with respect to the builderrsquos performance the sailorrsquos letter to the builder was a justified demand seeking assurance of the builderrsquos performance under the contract failure of the builder to provide such assurance within a reasonable time constituted repudiation of the contract

The sailor was legally justified in sending the letter to the builder on October 31 Contract parties are entitled to expect due performance of contractual obligations and are permitted to take steps to protect that expectation UCC sect 2-609 states that ldquo[w]hen reasonable grounds for insecurity arise with respect to the performance of either party the other may in writing demand adequate assurance of due performance rdquo Here the sailor learned on October 31 that the builderrsquos workers were on strike This gave the sailor reasonable grounds for insecurity about the builderrsquos ability to complete performance on time and thus gave the sailor the right to seek adequate assurance from the builder Because the sailorrsquos demand for assurance was justified the builder was required to provide assurance that was adequate under the circumstances within a reasonable time (not to exceed 30 days) or be held to have repudiated the contract UCC sect 2-609(4)

19

Contracts Analysis

Point Two(a) (30) The builder did not within a reasonable time provide the sailor adequate assurance of due performance this failure to provide assurance constituted a repudiation of the contract

Because the sailor with legal justification (see Point One) demanded from the builder assurance of due performance the builderrsquos failure to provide such assurance within a reasonable time was a repudiation of their contract See UCC sect 2-609(4) (ldquoAfter receipt of a justified demand[] failure to provide within a reasonable time not exceeding thirty days assurance of due performance is a repudiation of the contractrdquo) On October 31 the sailor requested that the builder provide adequate assurance regarding the completion of the boat by December 15 The builder did not respond to the sailorrsquos letter until November 25mdashnearly a month later Even if that response had been given in a reasonable time it nonetheless did not provide assurance of due performance It simply stated ldquoIrsquom sorry about the strike but it is really out of my hands I hope we settle it soon so that we can get back to workrdquo Therefore the builderrsquos November 25 response did not provide adequate assurance in response to the sailorrsquos justified request Thus the builder had repudiated the contract

Point Two(b) (35) Although the builder repudiated the contract with the sailor the builder probably retracted that repudiation on December 3 and the sailor was no longer entitled to cancel their contract Thus the sailorrsquos failure to perform the sailorrsquos obligations under the contract constituted a breach

The builderrsquos failure to provide adequate assurance of performance constituted a repudiation of their contract (see UCC sect 2-609(4)) but the builder was free to retract that repudiation until the sailor cancelled the contract or materially changed his position or indicated by communication or action that the sailor considered the repudiation to be final See UCC sect 2-611(1) (ldquoUntil the repudiating partyrsquos next performance is due he can retract his repudiation unless the aggrieved party has since the repudiation cancelled or materially changed his position or otherwise indicated that he considers the repudiation finalrdquo)

Here the facts state that before the builderrsquos December 3 telephone call to the sailor the sailor did nothing in response to the builderrsquos repudiation such as contracting with a third party for a boat The builderrsquos December 3 call informing the sailor that the boat would be timely delivered probably constituted a retraction of the repudiation because it clearly indicated to the sailor that the builder would be able to perform UCC sect 2-611(2) Thus after being so informed the sailor did not have the right to treat their contract as cancelled UCC sect 2-611(3) Accordingly the sailorrsquos failure to perform the sailorrsquos obligations under the contract by taking the boat and paying for it constituted a breach of the contract

20

CONSTITUTIONAL LAW ANALYSIS (Constitutional Law IVA F2b amp e)

ANALYSIS

Legal Problems

(1) Does AutoCorsquos operation of a ldquocompany townrdquo result in its actions counting as those of the state for purposes of constitutional analysis

(2) Does the expulsion of a schoolchild for failure to recite the Pledge of Allegiance violate the First Amendment as applied through the Fourteenth Amendment

(3) Does the arrest of a pamphleteer in connection with violation of an anti-littering rule where the littering is done by the recipients of leaflets distributed by the pamphleteer violate the First Amendment as applied through the Fourteenth Amendment

DISCUSSION

Summary

The First Amendment as applied through the Fourteenth Amendment applies only to state action It does not typically govern private actors However courts have found state action where the private actor has exercised a ldquopublic functionrdquo such as running a privately owned ldquocompany townrdquo as AutoCo has done here Thus First Amendment protections apply By requiring the son to participate in a mandatory Pledge of Allegiance ceremony AutoCo has compelled the expression of political belief in violation of the First Amendment as applied through the Fourteenth Amendment The fatherrsquos arrest in connection with breaching the anti-litter rule also violated the First Amendment as applied through the Fourteenth Amendment Although state actors can regulate the incidental effects of speech on the public streets on a content-neutral basis this power is limited and cannot extend to punishing a distributor of literature because of littering by third parties

Point One (30) AutoCorsquos operation of a company town (including a school) makes it a state actor under the public function strand of the state action doctrine

The individual rights protections of the Constitution apply only where there is ldquostate actionrdquomdash either direct action by the government or some action by a private party that is fairly attributable to the government As a general rule the actions of a private company like AutoCo or of a private school like the school operated by AutoCo would not constitute state action and the protections of the Constitution (in this case the First Amendment) would not apply

However there are situations in which the actions of a private actor are attributed to the state One such situation is when the private actor undertakes a public function There are not many bright-line rules in the Supreme Courtrsquos state action doctrine but one of them is this Where a private actor undertakes a ldquopublic functionrdquo the Constitution applies to those actions Where a corporation operates a privately owned ldquocompany townrdquo that provides essential services typically provided by a state actor the public function doctrine applies and the Constitution

21

Constitutional Law Analysis

binds agents of the town as if they were agents of the government See eg Marsh v Alabama 326 US 501 (1946) Here AutoCo does more than own the town it provides security services fire protection sanitation services and a school Thus the actions of AutoCo constitute state action and are governed by the Fourteenth Amendment

Point Two (35) The sonrsquos expulsion for failure to recite the Pledge of Allegiance violates the First Amendment as applied through the Fourteenth Amendment as a compelled expression of political belief

As explained in Point One the First Amendment applies to the school as a state actor Although children in public schools (and in schools subject to the First Amendment like

the Oakwood school) have some First Amendment rights Tinker v Des Moines Independent Community School District 393 US 503 506 (1969) schools have greater leeway to regulate the speech of students and teachers than the state would have outside the school context Hazelwood School Dist v Kuhlmeier 484 US 260 (1988) Morse v Frederick 551 US 393 (2007) However the Supreme Court has long held that public schools may not force their students to participate in a flag salute ceremony when it offends the political or religious beliefs of the students or their families West Virginia Board of Educ v Barnette 319 US 624 (1943) (invalidating a mandatory public school flag salute ceremony) see also Wooley v Maynard 430 US 705 (1977) (invalidating compelled expression of political belief on state-issued license plates)

In this case the school requires its students to participate in a flag salute and Pledge of Allegiance ceremony and punishes them when they refuse to participate Pursuant to this policy the school has expelled the son This expulsion violates the First Amendment ban on compelled expression

Point Three (35) Because the father was distributing leaflets in a traditional public forum his trespass arrest violated the First Amendment as applied through the Fourteenth Amendment

As explained in Point One AutoCo is treated as a state actor Thus Oakwoodrsquos commercial district is treated as government-owned property for purposes of the First Amendment Thus the leafleting here is subject to the First Amendment because it is an expressive activity Schneider v State of New Jersey Town of Irvington 308 US 147 (1939) When expression takes place on government-owned property government regulation of the expression is assessed under the public forum doctrine Public streets and sidewalks have long been held to be the classic example of a ldquotraditional public forumrdquo open to the public for expression Hague v CIO 307 US 496 515ndash16 (1939) Because the father was distributing leaflets while standing on a street corner in the commercial district his expressive activity occurred in a traditional public forum

When a state tries to regulate expressive activity in a traditional public forum it is prohibited from doing so based on the expressive activityrsquos content unless its regulation is narrowly tailored to achieve a compelling governmental interest (ldquostrict scrutinyrdquo) In this case however AutoCo is regulating the fatherrsquos expressive activity on the ostensibly neutral ground that his expressive activity has produced litter and made the street unsightly When a state tries to regulate expressive activity without regard to its content intermediate scrutiny applies Under intermediate scrutiny the true purpose of the regulation may not be the suppression of ideas (if so then strict scrutiny applies) the regulation must be narrowly tailored to achieve a significant

22

Constitutional Law Analysis

governmental interest and it must leave open ample alternative channels for expressive activity Ward v Rock Against Racism 491 US 781 791 (1989)

Here the application of the ordinance to the father will fail for two reasons First the Supreme Court has held that the governmentrsquos interest in keeping the streets clean is insufficient to ban leafleting in the public streets as the government power to regulate with incidental effects on public sidewalk speech is very limited See eg Schneider 308 US at 162 (leafletinglittering) Second the regulation (a blanket ban on distribution that results in littering) is not narrowly tailored to protect expression A narrowly tailored alternative would be prosecution only of people who litter Moreover the effect of the littering rule is likely to be a ban on all leafleting thus eliminating an entire class of means of expression This raises the possibility that there are not ldquoample alternative channels of communicationrdquo open to the father as required under the Courtrsquos standard of review for content-neutral regulation of speech

[NOTE Some examinees might argue that this is a ldquotime place and mannerrdquo restriction and that AutoCo might have greater latitude to regulate the public sidewalks under this theory This argument is incorrect for two reasons First the Supreme Court has held that the power to regulate speakers through littering laws is very limited for the reasons given and in the cases cited above But more generally a ldquotime place and mannerrdquo restriction involves the shifting of speech from one time and place to another or to another manner here there is no shifting but a direct punishment for expressive activity (albeit one couched in content-neutral terms) In addition some examinees might read the ordinance to be in effect a total ban on leafleting since most leafleting will produce some litter Those examinees might note that the Court has required total bans on an entire mode of expression to satisfy strict scrutiny and analyze the fatherrsquos prosecution here accordingly See United States v Grace 461 US 171 177 (1983) (invalidating ban on display of signs on public sidewalks surrounding US Supreme Court ldquo[a]dditional restrictions such as an absolute prohibition on a particular type of expression will be upheld only if narrowly drawn to accomplish a compelling governmental interestrdquo)]

23

SECURED TRANSACTIONS ANALYSIS (Secured Transactions IID E IVA B C)

ANALYSIS

Legal Problems

(1) Is a purchase-money security interest in consumer goods perfected even though there has been no filing of a financing statement

(2) Does a person who buys consumer goods for personal use take those goods free of a prior perfected purchase-money security interest in the goods

(3) Does a person who receives consumer goods as a gift take those goods subject to a prior perfected security interest in them

DISCUSSION

Summary

The retailerrsquos security interest in the bicycles was perfected even though no financing statement was filed because it was a purchase-money security interest in consumer goods A purchase-money security interest in consumer goods is automatically perfected upon attachment

The buyer is not subject to the retailerrsquos security interest in the bicycle that the buyer bought from the man Because the bicycle was consumer goods in the hands of the man and the retailer never filed a financing statement covering the bicycle the retailerrsquos security interest is not effective against someone like the buyer who bought the bicycle for value without knowledge of the retailerrsquos security interest and for personal use

On the other hand the retailerrsquos security interest continues in the bicycle given to the friend because the friend did not give value for the bicycle or buy it in the ordinary course of business

Point One (35) The retailerrsquos security interest in the bicycles attached on June 1 Because this interest was a purchase-money security interest in consumer goods it was automatically perfected when it attached

The retailerrsquos security interest in the bicycles attached on June 1 when the man bought the bicycles (acquiring rights in the collateral) signed a security agreement containing a description of the collateral and received value from the retailer (by being given credit with which to purchase the bicycles) UCC sect 9-203(a) amp (b)

Despite the retailerrsquos failure to file a financing statement its security interest was perfected Pursuant to UCC sect 9-309(1) a security interest is automatically perfected upon attachment if the goods are ldquoconsumer goodsrdquo and the security interest is a ldquopurchase-money security interestrdquo

In this case the bicycles sold by the retailer to the man were consumer goods at the time of sale The bicycles were ldquogoodsrdquo because they were ldquomovable when a security interest

24

Secured Transactions Analysis

attachesrdquo UCC sect 9-102(a)(44) They were also consumer goods because they were ldquobought for use primarily for personal family or household purposesrdquo UCC sect 9-102(a)(23) The retailerrsquos security interest in these consumer goods was also a ldquopurchase-money security interestrdquo A purchase-money security interest is an interest that secures a debt that was incurred in order to ldquoenable the debtor to acquire rights in or the use of the collateralrdquo UCC sect 9-103(a) (b)(1) Here the man incurred an obligation to the retailer to purchase the bicycles so the security interest he gave the retailer to secure that obligation was a purchase-money security interest

Because the retailerrsquos security interest was a purchase-money security interest in consumer goods it was automatically perfected on June 1 when the interest attached to the bicycles

Point Two (35) The buyer took the bicycle free of the retailerrsquos security interest because (i) the retailer did not file a financing statement covering the bicycle (ii) the bicycle was ldquoconsumer goodsrdquo and (iii) the buyer bought the bicycle for value without knowledge of the retailerrsquos security interest and for personal use

A security interest continues in collateral even after a sale or other disposition of that collateral unless the creditor authorized the disposition ldquofree of the security interestrdquo or another Article 9 exception applies UCC sectsect 9-201(a) and 9-315(a)(1)

However a buyer of goods like the buyer here can take free of a prior security interest in those goods under certain circumstances See UCC sectsect 9-317(b) (buyers who give value and receive delivery of goods without knowledge of an unperfected security interest in the goods) and 9-320(a) amp (b) (buyer in ordinary course of business buyer of consumer goods in a consumer-to-consumer transaction who gives value) In this case the retailerrsquos security interest was perfected when the buyer purchased the bicycle so UCC sect 9-317(b) does not protect the buyer The buyer also is not a protected ldquobuyer in ordinary course of businessrdquo because he did not purchase from a person who is in the business of selling bicycles See UCC sect 1-201(b)(9)

The buyer can however qualify for the protection of UCC sect 9-320(b) That section provides that a buyer of goods from a person who used them for personal family or household purposes takes free of a perfected security interest in the goods if (1) the buyer had no knowledge of the security interest (2) the buyer gave value for the goods (3) the buyer purchased the goods primarily for personal family or household purposes and (4) the purchase occurred before the filing of a financing statement covering the goods

The buyer met all of these criteria The man used the bicycle for personal purposes The buyer purchased the bicycle from the man and the buyer had no knowledge of the retailerrsquos security interest The buyer gave value ($400) for the bicycle and he bought it ldquoprimarily for personal family or household purposesrdquo as he planned to use it for recreation which is a personal rather than a business use Finally no financing statement had been filed Therefore under UCC sect 9-320(b) the buyer took free of the retailerrsquos security interest

Point Three (30) The retailerrsquos security interest continues in the bicycle that the man gave to the friend Thus the retailer can recover the bicycle from the friend because the friend did not give value for the bicycle or buy it in the ordinary course of business

25

Secured Transactions Analysis

As noted in Point Two the retailer did not authorize the man to dispose of the bicycle Consequently the retailerrsquos security interest continued in the bicycle even after the man transferred ownership of the bicycle to the friend See UCC sectsect 9-201(a) and 9-315(a)(1) The retailerrsquos security interest in the bicycle will be effective against the friend unless some other provision of Article 9 allows the friend to take the bicycle free of that security interest

Unfortunately for the friend there is no Article 9 provision that allows him to take free of the retailerrsquos interest The friendrsquos basic problem is that he is not a buyer of the bicyclemdashhe received the bicycle as a gift and did not give value for it Thus the friend is not protected by any of the applicable exceptions See UCC sectsect 9-317(b) (protecting buyers who give value for goods subject to an unperfected security interest) 9-320(a) (protecting buyers in ordinary course of business) and 9-320(b) (protecting buyers of consumer goods who give value)

In short the retailerrsquos security interest continues in the bicycle that the man gave to the friend The friend took the bicycle subject to that security interest

26

FEDERAL CIVIL PROCEDURE ANALYSIS (Federal Civil Procedure VIE)

ANALYSIS

Legal Problems

(1) Does a judgment in a prior action preclude a nonparty from suing the same defendant on a closely related claim when the nonparty and the original plaintiff are in a family relationship

(2) Does a judgment rendered in an earlier action preclude a nonparty from litigating an issue that was actually decided in the first suit

(3) May a nonparty to an earlier action invoke the judgment in that action to preclude a party to the prior action from relitigating an issue that the party had a full and fair opportunity to litigate in the earlier action

DISCUSSION

Summary

Pursuant to the doctrines of claim preclusion (res judicata) and issue preclusion (collateral estoppel) a judgment is binding on the parties thereto In the absence of privity nonparties to a prior suit cannot be bound by a judgment rendered in their absence Thus in the absence of privity a nonparty to the first suit is not precluded from presenting her claim in a second suit even if it is factually related to the claims and defenses presented in the first suit nor is she bound by determinations of issues made in the first suit A family relationship without more does not support a finding of privity For this reason Mother as a nonparty is not bound by the judgment in the Son-Driver action She may bring her separate claim for damage to her car and she is not precluded from litigating the question of whether she was negligent in the maintenance of her car

Driver on the other hand could be precluded from relitigating the issue of her negligence pursuant to the doctrine of non-mutual issue preclusion (also called non-mutual offensive collateral estoppel) which allows a nonparty to a prior action to invoke issue preclusion to prevent a party to that prior action from relitigating determinations of issues made therein However Mother may be prevented from invoking non-mutual collateral estoppel in this case because she could easily have joined her claim in the prior action but did not do so

[NOTE Federal common law governs the preclusive effect of a judgment rendered by a federal court sitting in diversity See Semtek Intrsquol Inc v Lockheed Martin Corp 531 US 497 508 (2001) But the Semtek Court concluded that federal common law in this context incorporates the preclusion law of the state in which the rendering federal court sits (unless the state law is incompatible with federal interests) id at 508ndash09 Thus State Arsquos preclusion law determines the preclusive effect of the judgment rendered in Sonrsquos suit against Driver The problem says that State A preclusion law is identical to federal preclusion law so the following analysis utilizes general principles of preclusion drawn from Supreme Court case law (announcing federal preclusion rules) and the Restatement (Second) of Judgments]

27

Federal Civil Procedure Analysis

Point One (35) Under the doctrine of claim preclusion the judgment rendered in the first action does not preclude Mother a nonparty from suing Driver for the damage to her car because the judgment binds only parties or those in privity with them and Mother and Son are not in privity

Driver may contend that the doctrine of claim preclusion (res judicata) precludes Mother from presenting a claim arising from the same nucleus of facts that was presented in the first action brought by Son According to the doctrine of claim preclusion ldquowhen a court of competent jurisdiction has entered a final judgment on the merits of a cause of action the parties to the suit and their privies are thereafter bound lsquonot only as to every matter which was offered and received to sustain or defeat the claim or demand but as to any other admissible matter which might have been offered for that purposersquordquo Commissioner of Internal Revenue v Sunnen 333 US 591 597 (1948) (citation omitted)

However the doctrine of claim preclusion does not apply to Mother on the facts of this problem First Mother was not a party to the earlier case ldquoIt is a principle of general application in Anglo-American jurisprudence that one is not bound by a judgment in personam in a litigation in which he is not designated as a party or to which he has not been made a party by service of processrdquo Taylor v Sturgell 553 US 880 884 (2008) (citing Hansberry v Lee 311 US 32 40 (1940)) see also RESTATEMENT (SECOND) OF JUDGMENTS sect 34(3) (1982) This rule reflects our ldquodeep-rooted historic tradition that everyone should have his own day in courtrdquo Martin v Wilks 490 US 755 762 (1989) (citation omitted) (superseded by statute on other grounds) Since Mother was not a party to the first suit she is not bound by the judgment unless an exception to the general rule applies

Mother might be bound by the prior judgment if she were considered to have been sufficiently in privity with Son that Son represented her interests in that action ldquoA person who is not a party to an action but who is represented by a party is bound by and entitled to the benefits of a judgment as though he were a partyrdquo RESTATEMENT (SECOND) OF JUDGMENTS sect 41(1) But there is no suggestion in the facts of the problem that Son who is an adult purported to represent Motherrsquos interests in the first suit ldquo[C]lose family relationships are not sufficient by themselves to establish privity with the original suitrsquos party or to bind a nonparty to that suit by the judgment entered therein rdquo Cuauhtli v Chase Home Finance LLC 308 Fed Appx 772 773 (5th Cir 2009) (citation omitted) accord 18A CHARLES ALAN WRIGHT ET AL FEDERAL

PRACTICE AND PROCEDURE sect 4459 (2d ed 2002) In Taylor v Sturgell supra the Supreme Court identified other special circumstances in

which nonparties may be bound by a prior judgmentmdashwhen a nonparty consents to be bound when a nonparty is in a pre-existing substantive legal relationship with a party (such as preceding and succeeding property owners) when a nonparty assumed control of the prior litigation when a party seeks to relitigate through a proxy or where a special statutory scheme seeks to foreclose successive litigation by nonparties See Taylor 553 US at 893ndash95 None of these circumstances exists here

Because Mother was not a party to the first suit and is not in privity with Son who is an adult the judgment in the first action does not preclude her from bringing her own claim against Driver

Point Two (35) Under the doctrine of issue preclusion the judgment rendered in the first action does not preclude Mother a nonparty from litigating the issue of her negligence in maintaining her carrsquos

28

Federal Civil Procedure Analysis

brake lights because the judgment binds only parties or those in privity with them and Mother and Son are not in privity

By its affirmative response to a special interrogatory the jury in the first action expressly concluded that ldquoMother negligently failed to ensure that the brake lights on her car were in proper working orderrdquo Driver may attempt to invoke the doctrine of issue preclusion to preclude Mother from relitigating this issue in the second action

[I]ssue preclusion arises in a second action on the basis of a prior decision when the same lsquoissuersquo is involved in both actions the issue was lsquoactually litigatedrsquo in the first action after a full and fair opportunity for litigation the issue was lsquoactually decidedrsquo in the first action by a disposition that is sufficiently lsquofinalrsquo lsquoon the meritsrsquo and lsquovalidrsquo it was necessary to decide the issue in disposing of the first action and the later litigation is between the same parties or involves nonparties that are subject to the binding effect or benefit of the first action Once these requirements are met issue preclusion is available not only to defend against a demand for relief but also as offensive support for a demand for relief Issue preclusion moreover is available whether or not the second action involves a new claim or cause of action

18 CHARLES ALAN WRIGHT ET AL FEDERAL PRACTICE AND PROCEDURE sect 4416 at 392ndash93 (2d ed) see also RESTATEMENT (SECOND) OF JUDGMENTS sect 27 (1982)

Here several of the elements necessary for issue preclusion are present The same issue is involved in both actionsmdashthe issue of Motherrsquos negligence in failing to maintain the brake lights on her car That issue was actually litigated in the first action and decided by the jury There is nothing to suggest anything less than a full and fair opportunity to litigate The judgment disposing of the issue was final

Nevertheless the judgment will not preclude Mother from relitigating the issue for two reasons First Mother was not a party to the first action and as explained above Mother and Son are not in privity Therefore she cannot be denied an opportunity to litigate the issue of her negligence Second it does not appear that the juryrsquos decision as to Motherrsquos negligence was necessary to the prior judgment against Driver Nothing suggests that the finding on Motherrsquos negligence had any bearing on the outcome of the first action

Point Three (30) Under the doctrine of non-mutual issue preclusion the judgment rendered in the first action might preclude Driver from relitigating the issue of her negligence However Driver has a strong argument that such a result would be inconsistent with the policy against offensive use of non-mutual estoppel when the non-party plaintiff easily could have joined as a plaintiff in the first action

Because Son already convinced the jury in the first action that ldquoDriver was negligent in the operation of her vehiclerdquo Mother may wish to invoke the doctrine of non-mutual issue preclusion to prevent Driver from relitigating the question of her negligence As noted above ldquoissue preclusion arises in a second action on the basis of a prior decision when the same lsquoissuersquo is involved in both actions the issue was lsquoactually litigatedrsquo in the first action after a full and fair opportunity for litigation the issue was lsquoactually decidedrsquo in the first action by a disposition that is sufficiently lsquofinalrsquo lsquoon the meritsrsquo and lsquovalidrsquo it was necessary to decide the issue in disposing of the first action rdquo 18 CHARLES ALAN WRIGHT ET AL FEDERAL PRACTICE AND

PROCEDURE sect 4416 at 392 (2d ed) see also RESTATEMENT (SECOND) OF JUDGMENTS sect 27

29

Federal Civil Procedure Analysis

Here these basic requirements for issue preclusion are met First the same issue is involved in both suits whether Driver was negligent in the operation of her car Second this issue was actually litigated and decided in the first action the jury answered a special interrogatory raising this very question There is nothing to suggest that Driver lacked a full and fair opportunity to litigate the issue Since a judgment was rendered against Driver for the injuries Son sustained as a result of Driverrsquos negligence resolution of the issue was necessary to dispose of the first action Driver was a party to the first action so she may be bound by the judgment

[NOTE Traditionally issue preclusion required mutualitymdashboth the party asserting issue preclusion and the party against whom issue preclusion was asserted were bound by the prior judgment Under the traditional mutuality rule Mother could not assert issue preclusion against Driver because Mother would not be bound by the judgment if Driver sought to rely on it See Point One There is no mutuality between Mother and Driver with respect to the prior judgment

This traditional mutuality requirement has been abandoned in most jurisdictions The Supreme Court rejected a strict mutuality requirement in Blonder-Tongue Laboratories Inc v University of Illinois Foundation 402 US 313 (1971) (non-mutual defensive collateral estoppel used by a defendant to preclude a plaintiff from relitigating a claim the plaintiff previously litigated) and Parklane Hosiery Co v Shore 439 US 322 (1979) (non-mutual offensive collateral estoppel used by a plaintiff to preclude a defendant from relitigating a claim the defendant previously litigated) In Parklane Hosiery the Court concluded (as a matter of federal preclusion law) that trial courts should have ldquobroad discretionrdquo to determine whether or not to permit a plaintiff to invoke non-mutual issue preclusion ldquoThe general rule should be that in cases where a plaintiff could easily have joined in the earlier action or where the application of offensive estoppel would be unfair to a defendant a trial judge should not allow the use of offensive collateral estoppelrdquo Id at 331

The Parklane Hosiery decision identified a number of circumstances that might make it unfair to allow a plaintiff to invoke non-mutual issue preclusion (non-mutual offensive collateral estoppel in the traditional terminology) against a defendant In particular the Parklane Hosiery court suggested that issue preclusion may not be appropriate if the plaintiff in the second action ldquocould easily have joined in the earlier actionrdquo Id Prohibiting plaintiffs from using non-mutual estoppel under such circumstances would promote judicial efficiency by encouraging plaintiffs to join the prior action It would also discourage plaintiffs from staying out of prior litigation in order to secure in effect two bites at the apple using the prior litigation offensively if the defendant loses and forcing the defendant to litigate a second time if the defendant wins the prior action

An exceptional exam answer might therefore argue that non-mutual issue preclusion should be denied on these facts Son and Mother both reside in State A since they are related they know each other well and Son was driving Motherrsquos car when the accident occurred They could have sued together and Rule 20 of the Federal Rules of Civil Procedure would have authorized joinder of their claims because those claims arose from the same transaction or occurrence and raised a common question of law or fact FED R CIV P 20(a) The facts do not suggest that Mother had any reason not to join Sonrsquos suit other than a desire to see how Sonrsquos action concluded before bringing her own claim Cf Nations v Sun Oil Co (Del) 695 F2d 933 938 (5th Cir 1983) (concluding that plaintiff ldquowas entitled to await the development of his injuries and their predictable consequencesrdquo) Because it appears that Mother may be a ldquowait-and-seerdquo plaintiff who could easily have joined the original action a trial court might disallow as a matter of discretion her use of non-mutual issue preclusion]

30

AGENCY ANALYSIS __________ (Agency I II)

ANALYSIS

Legal Problems

(1) Is the principal or the agent or both liable on contracts with a third party when the principal is an ldquoundisclosed principalrdquo

(2) Is the principal or the agent or both liable on contracts with a third party when the principal is ldquopartially disclosedrdquo or an ldquounidentified principalrdquo

(3) Is the principal or the agent or both liable on contracts with a third party for the purchase of goods when the agent exceeded his authority but the principal nonetheless accepts the goods

DISCUSSION

Summary

The agent but not the owner is liable to the basket manufacturer because the owner is an undisclosed principal and the agent acted without actual or apparent authority Both the agent and the owner however are liable on the burner contract because the owner is an unidentified principal and the agent had apparent authority to enter into that contract With respect to the solar cells contract whether the owner is liable depends upon whether a court would follow the Second or Third Restatement of Agency which take different positions on the effect of the ratification of a contract by an undisclosed principal Under either the agent would also be liable on the contract as he was a party to the contract

[NOTE The contracts that are the subject of this question are contracts for the sale of goods and therefore are governed by Article 2 of the Uniform Commercial Code Article 2 however does not contain agency rules Accordingly common law concepts of agency are applicable UCC sect 1-103(b)]

Point One (35) The agent but not the owner is liable to the basket manufacturer The agent had no actual authority to enter into the contract to buy aluminum baskets and because the owner was an undisclosed principal the manufacturer had no reason to believe that the agent had apparent authority Furthermore the manufacturer had no reason to believe that the agent was not contracting for his own benefit

An agent acting on behalf of a principal can bind the principal to contracts if the agent has either actual or apparent authority An agent has actual authority when contracting on behalf of his principal if he ldquoreasonably believes in accordance with the principalrsquos manifestations to the agent that the principal wishes the agent so to actrdquo RESTATEMENT (THIRD) OF AGENCY sect 201 (2006) Here the agent was told to buy only wicker baskets not aluminum baskets Thus when he contracted with the basket manufacturer to buy aluminum baskets he had no actual authority to do so

31

Agency Analysis

An agent acts with apparent authority ldquowhen a third party [with whom the agent acts] reasonably believes the actor has authority to act on behalf of the principal and that belief is traceable to the principalrsquos manifestationsrdquo Id sect 203 Here the owner notified basket manufacturers that she or her agent might contact them to purchase baskets but that notification did not specifically name the agent or any other person as the ownerrsquos agent Furthermore the basket manufacturer had no prior dealings with the agent or the owner or any reason to think that the agent was acting for the benefit of anyone but himself Thus there is no basis to conclude that the basket manufacturer thought that the agent had apparent authority to act for the owner

Generally when an agent acts on behalf of an undisclosed principal and the agent lacks authority to enter into the contract the agent is liable on the contract as a party to the contract but the principal is not liable This rule is consistent with the third partyrsquos expectations ldquoThe third party expected the agent to be a party to the contract because the agent presented the deal as if he were acting for himself Moreover if the third party is unaware of the principalrsquos existence the third party must be relying on the agentrsquos solvency and reliability when entering into the contractrdquo See ROBERT W HAMILTON JONATHAN R MACEY amp DOUGLAS K MOLL CORPORATIONS INCLUDING PARTNERSHIPS AND LIMITED LIABILITY COMPANIES 34 (11th ed 2010) See also RESTATEMENT (THIRD) OF AGENCY sect 603 cmt c Furthermore because the third party has no idea that the agent is acting or is seemingly acting on behalf of another there is no reason to believe that the third party would be expecting an undisclosed principal to be liable on the contract Id

Point Two (35) Because the owner is an unidentified (as opposed to undisclosed) principal both she and the agent (as a party to the contract) probably are liable on the contract with the burner manufacturer

When the agent contracted with the burner manufacturer he did not have actual authority to do so as the owner had expressly restricted the agentrsquos authority to purchase only burners with ldquowhisper technologyrdquo See Point One However the agent may have had apparent authority to buy burners without whisper technology

An agent acts with apparent authority ldquowhen a third party [with whom the agent acts] reasonably believes the actor has authority to act on behalf of the principal and that belief is traceable to the principalrsquos manifestationsrdquo RESTATEMENT (THIRD) OF AGENCY sect 203 (2006) The owner indicated that an agent might contact the burner manufacturer The notice contained no restriction regarding the type of burners that the agent was authorized to purchase The facts indicate that burner manufacturers regularly receive such notices

Although the agent told the burner manufacturer that he represented a well-known hot-air balloon operator he did not disclose the ownerrsquos name Thus the owner was a partially disclosed or unidentified principal See RESTATEMENT (SECOND) OF AGENCY sect 4(2) (1958) (using term ldquopartially disclosed principalrdquo) RESTATEMENT (THIRD) OF AGENCY sect 104(2)(c) (2006) (using term ldquounidentified principalrdquo) An agent for a partially disclosed principal may have apparent authority RESTATEMENT (SECOND) OF AGENCY sect 159 cmt e (1958) Based upon (1) the notice sent by the owner (2) the agentrsquos revelation that he was acting as an agent and (3) the fact that burner manufacturers regularly receive such notices and sell to agents the manufacturer may argue that it reasonably and actually believed that the agent was authorized to purchase burners without whisper technology The manufacturer may also argue that because the agent revealed that he was an agent his listing of the ownerrsquos address as the delivery address connects the agent to the notice given by the owner Arguably this distinguishes the burner contract from the basket

32

Agency Analysis

contract Here there is a strong case to support the conclusion that the agent had apparent authority if he did then the owner is liable to the burner manufacturer

The agent also is liable as a party to the contract because he did not fully disclose his agency relationship Although he told the burner manufacturer that he represented a well-known hot-air balloon operator he did not disclose the ownerrsquos name Generally even an authorized agent of a partially disclosed or unidentified principal is liable as a party to a contract with a third person RESTATEMENT (SECOND) OF AGENCY sect 321 (1958) (ldquounless otherwise agreedrdquo) RESTATEMENT (THIRD) OF AGENCY sect 602(2) (2006) (ldquounless the agent and the third party agree otherwiserdquo)

Point Three (30) Under the Second Restatement of Agency the owner is not liable on the contract for solar cells because the agent did not have actual or apparent authority and the owner as an undisclosed principal cannot ratify the contract Under the Third Restatement the owner could be liable as she ratified the contract Under either Restatement the agent is liable as a party to the contract

The owner is not liable to the solar cell manufacturer for breach of the contract for the solar cells because the agent had no actual or apparent authority to purchase solar cells on the ownerrsquos behalf and the owner under the Second Restatement of Agency did not ratify the contract with knowledge of the material facts Thus she is not liable as a ratifier of the contract

The facts state that the agent had authority to purchase only propane fuel tanks In addition he had no apparent authority to purchase solar cells The owner made no manifestations to the solar cell manufacturer that would lead a reasonable person in the manufacturerrsquos position to believe that the agent had the authority to bind the owner to a contract to purchase solar cells In fact the agent made no manifestations at all to the solar cell manufacturer Unlike with the basket manufacturer and the burner manufacturer the owner did not notify the manufacturer of solar cells that an agent might contact it to purchase solar cells In addition the solar cells were delivered to the agent and not to the ownerrsquos address In sum the manufacturer was unaware of any relationship between the owner and the agent As to the solar cell manufacturer the owner is an undisclosed principal There can be no apparent authority in the case of an undisclosed principal because there are no manifestations from the principal to the third person See RESTATEMENT (SECOND) OF AGENCY sect 8 cmt a (1958) (ldquothere can be no apparent authority created by an undisclosed principalrdquo) RESTATEMENT (THIRD) OF AGENCY sect 203 cmt f (2006) (ldquoapparent authority is not present when a third party believes that an interaction is with an actor who is a principalrdquo)

The owner also did not ratify the contract Although the owner used the solar cells generally a principal cannot ratify an unauthorized transaction with a third person ldquounless the one acting purported to be acting for the ratifierrdquo RESTATEMENT (SECOND) OF AGENCY sect 85(1) (1958)

The result differs under the Third Restatement which expressly rejects the Second Restatement on this issue The Restatement (Third) of Agency sect 403 (2006) states ldquoA person may ratify an act if the actor acted or purported to act as an agent on the personrsquos behalfrdquo According to comment b ldquoan undisclosed principal may ratify an agentrsquos unauthorized actrdquo Under the Restatement (Third) of Agency rule the owner probably ratified the transaction The agent clearly acted on the ownerrsquos behalf and in addition the ownerrsquos conduct in using the solar cells ldquojustifies a reasonable assumption that [she] is manifesting assent that the act shall affect [her] legal relationsrdquo See id sect 401(2)

33

Agency Analysis

The agent also is liable to the solar cell manufacturer for breach of the contract for the solar cells because he is a party to the contract The facts indicate that the agent never told the solar cell manufacturer that he represented the owner or any other principal Consequently even if the agent were authorized (which as discussed above he is not) he would be liable as a party to the contract See RESTATEMENT (SECOND) OF AGENCY sect 322 (1958) RESTATEMENT (THIRD) OF AGENCY sect 603(2) (2006) Here he has no authority or apparent authority and is liable as a party to the contract

The agent would also be liable under the Third Restatement Under Restatement (Third) of Agency sect 402(1) (2006) ratification generally relates back and the transaction is treated as if it were authorized at the time of the transaction However this does not relieve the agent of an undisclosed principal who ratifies an unauthorized transaction of liability under the ratified contract See id sect 603(2) (authorized agent for undisclosed principal is a party to the contract) and sect 403 cmt b (ldquoAn undisclosed principalrsquos ratification does not eliminate the agentrsquos liability to the third party on the transaction rdquo)

[NOTE An examinee may discuss the concept of inherent agency power This concept is recognized by the Restatement (Second) of Agency sect 8 A (1958) but the concept is not used in the Restatement (Third) of Agency (2006) Here there are no facts to support that the agent had inherent authority

As to contracts with agents for partially disclosed principals (eg the contract for the burners) the basic question is whether the acts done ldquousually accompany or are incidental to transactions which the agent is authorized to conductrdquo RESTATEMENT (SECOND) OF AGENCY

sect 161 (1958) If so the principal is bound if the other party ldquoreasonably believes that the agent is authorized to do them and has no notice that he is not so authorizedrdquo Id The purchase of burners without whisper technology was not authorized nor was it incidental to an authorized transaction Therefore there should not be inherent agency power

As to contracts on behalf of undisclosed principals (eg the other two contracts) the basic question is whether the acts done are usual or necessary in the transactions the agent is authorized to transact RESTATEMENT (SECOND) OF AGENCY sect 194 (1958) The other two contracts seem fundamentally different from the authorized transactions Therefore there should not be inherent agency power

Only minimal credit should be given for discussion of inherent agency power]

34

EVIDENCE ANALYSIS _____ (Evidence IIA VA B E F J K)

ANALYSIS

Legal Problems

(1) Is the authenticated copy of the mechanicrsquos text message relevant and admissible

(2) Is the womanrsquos question ldquoIs my scooter safe to drive for a whilerdquo relevant and admissible

(3) Is the womanrsquos testimony describing the mechanicrsquos thumbs-up relevant and admissible

DISCUSSION

Summary

The mechanicrsquos text message to the woman is relevant to whether (1) the woman lost control of the scooter due to its defective brakes (2) the woman knew that the brakes needed repair and (3) it was negligent for the woman to drive the scooter knowing that its brakes needed repair

The mechanicrsquos text message is hearsay if it is offered by the pedestrian to prove that the scooterrsquos brakes needed repair However it fits the hearsay exception for present sense impressions and probably also fits the exception for business records The mechanicrsquos text message is not hearsay if it is instead offered by the pedestrian to prove the womanrsquos state of mind (ie that she had notice that her brakes needed repair)

The womanrsquos question to the mechanic and his response are also relevant to whether the brakes caused the accident and whether the woman was negligent The question is not hearsay because the woman did not make an assertion

The mechanicrsquos thumbs-up response is nonverbal conduct intended by the mechanic as an assertion and is therefore an out-of-court statement If the woman offers the mechanicrsquos statement to prove that the scooter was actually safe to ride the womanrsquos testimony about the statement is hearsay

However the mechanicrsquos statement is not hearsay if it is offered by the woman to prove her state of mind Therefore the womanrsquos question and the mechanicrsquos response are admissible to prove the womanrsquos state of mind

Point One(a) (20) The mechanicrsquos text message to the woman should be admitted because it is relevant

Evidence is relevant if it has ldquoany tendency to make a fact more or less probable than it would be without the evidencerdquo FED R EVID 401 ldquoRelevant evidence is admissiblerdquo unless it is inadmissible pursuant to some other rule FED R EVID 402

The mechanicrsquos text message to the woman ldquoWhen you pick up your scooter you need to schedule a follow-up brake repair Wersquoll order the partsrdquo is relevant for two reasons First this evidence has some tendency to make it more probable that the brakes malfunctioned and

35

Evidence Analysis

caused the accident Second it has some tendency to make it more probable that the woman was negligent in riding her scooter after being told by the mechanic that it required further repair

Point One(b) (30) The mechanicrsquos text message fits either the hearsay exception for present sense impressions or the exception for business records or it is admissible non-hearsay

The mechanicrsquos text message is a statement under Rule 801(a) because it is ldquoa written assertionrdquo FED R EVID 801(a) The text message is hearsay if the pedestrian offers it to prove the ldquotruth of the matter asserted in the statementrdquo (ie that the scooterrsquos brakes required repair) which resulted in the woman losing control of the scooter and causing the accident FED R EVID 801(c)

However the mechanicrsquos text message fits the hearsay exception for ldquopresent sense impressionsrdquo under Rule 803(1) because it is ldquo[a] statement describing or explaining an event or condition made while or immediately after the declarant perceived itrdquo FED R EVID 803(1) Here the mechanicrsquos text message described the condition of the scooter immediately after he perceived it during the maintenance service

The mechanic is a person with knowledge of the condition of the scooter so if text messages regarding repairs were made and kept by the mechanic in the ordinary course of business this text message also fits the business records exception Under Rule 803(6) a business record is a record of an act ldquomade at or near the time by someone with knowledgerdquo and ldquothe record was kept in the course of a regularly conducted activity of a businessrdquo and ldquomaking the record was a regular practice of that activityrdquo FED R EVID 803(6)

However the text message is not hearsay if it is instead offered to prove that the woman was negligent because she rode her scooter after the mechanic told her it required repair If offered for this purpose it would not be offered for the truth of the matter asserted in the statement but to show the womanrsquos belief about the condition of the scooter (her state of mind)

Point Two (10) The womanrsquos question to the mechanic should be admitted because it is not hearsay

The womanrsquos question to the mechanic is relevant because along with the mechanicrsquos thumbs-up response (see Point Three) it has some tendency to make it more probable that the woman was not negligent andor that the scooter brakes did not malfunction and cause the accident FED R EVID 401 The womanrsquos question does not raise hearsay concerns because it is not an assertion

Hearsay is defined under Rule 801(a) as ldquoan oral assertion written assertion or nonverbal conductrdquo Although ldquoassertionrdquo is not further defined ldquoa favorite [definition] of writers in the [evidence] field for at least a century and a half [is that] the word simply means to say that something is so eg that an event happened or a condition existedrdquo 2 MCCORMICK ON

EVIDENCE sect 246 (6th ed 2006) Under this definition the womanrsquos question is not hearsay because it is not an assertion

Point Three(a) (20) The mechanicrsquos thumbs-up to the woman is a nonverbal assertion that is relevant and the womanrsquos testimony about that response is admissible

36

Evidence Analysis

Hearsay is defined under Rule 801(c) as a ldquostatementrdquo that is ldquoa personrsquos oral assertion written assertion or nonverbal conduct if the person intended it as an assertionrdquo FED R EVID 801(a) Here when the mechanic responded to the womanrsquos question (ldquoIs my scooter safe to ride for a whilerdquo) with a thumbs-up gesture the facts suggest that he intended his nonverbal conduct as an assertion that in his opinion the scooter was safe to ride

The mechanicrsquos assertion is relevant and admissible to prove that the woman was not negligent because the evidence makes it more probable that at the time of the accident she believed that the scooter was safe to ride despite the fact that the brakes required repair FED R EVID 401 Admission of the womanrsquos description of the mechanicrsquos thumbs-up for this purpose does not raise hearsay concerns because the evidence would not be offered for the truth of the matter asserted but to show the womanrsquos belief about the condition of the scooter (her state of mind)

Point Three(b) (20) The mechanicrsquos thumbs-up is relevant to determine whether the scooterrsquos brakes malfunctioned causing the accident but if offered for this purpose it is also hearsay

The mechanicrsquos nonverbal assertion is relevant to the determination of whether the scooterrsquos brakes malfunctioned causing the accident However if offered to prove the ldquotruth of the matter asserted in the statementrdquo (ie that the scooter was safe to ride for a while) it is hearsay that does not fit any hearsay exception

37

TRUSTS AND FUTURE INTERESTS ANALYSIS ____________________ (Trusts and Future Interests IC1 amp 4 G IIF)

ANALYSIS

Legal Problems

(1)(a) Was the revocable trust amendable

(1)(b) If the trust was amendable must the amendment have been executed in accordance with the state Statute of Wills in order to be valid

(2) If the trust amendment was valid does the amendment apply to the probate estate assets passing to the trust pursuant to Settlorrsquos will

(3) If the trust amendment was valid should the trust property be distributed to University

(4) If the trust amendment was not valid should the trust property be distributed to Settlorrsquos grandchild (her only heir) or held in further trust in accordance with the terms of the original trust instrument

DISCUSSION

Summary

A revocable trust is amendable even if the trust instrument does not expressly grant to the trust settlor a power to amend Both inter vivos trusts and amendments thereto are valid even though not executed in accordance with the requirements applicable to wills

Under the Uniform Testamentary Additions to Trusts Act a revocable trust may be amended at any time prior to the settlorrsquos death and the amendment applies to the disposition of assets conveyed to the trust pursuant to a will even if the will was executed prior to the date of the amendment

At Settlorrsquos death trust assets including probate assets passing to the trust under Settlorrsquos will would go to University if as is the case here the trust amendment was valid If the amendment was invalid the trust assets would continue to be held in further trust because there is no violation of the common law Rule Against Perpetuities

Point One(a) (30) Settlor retained the right to amend the inter vivos trust despite her failure to expressly reserve this power

At issue here is whether a retained power of revocation includes the power to amend sometimes referred to as the power to modify The Restatement (Second) of Trusts sect 331 cmt g provides that if a settlor has a power to revoke that retained power ordinarily includes a power to modify (amend) as well Comment g also notes that the power to amend includes both a power to withdraw trust assets and a power to ldquomodify the terms of the trustrdquo The Uniform Trust Code which provides that a power to revoke includes the power to amend is consistent with this view

38

Trusts and Future Interests Analysis

UNIF TRUST CODE sect 602 accord RESTATEMENT (THIRD) OF TRUSTS sect 63 cmt The theory is that even though a power to amend was not expressly retained by a settlor the goal of amendment assuming the power was not included in the power to revoke could easily be achieved by first revoking the trust and then creating a new trust with the same terms contemplated by the amendment To require this would put form over substance

Thus by expressly retaining the power to revoke the trust Settlor retained a power to amend the inter vivos trust despite her failure to expressly reserve this power

[NOTE Under the common law a trust is irrevocable unless the settlor expressly retains a power to revoke the trust Conversely under the Uniform Trust Code a trust is revocable unless the terms of the trust expressly provide otherwise See UNIF TRUST CODE sect 602 The Trust Codersquos position on revocation follows the minority view in the United States and is inconsistent with prior Restatements of Trusts (see Restatement (Second) of Trusts sect 330) Here the trust is revocable because Settlor expressly retained a power of revocation

The Uniform Trust Code has been adopted in 24 jurisdictions Alabama Arizona Arkansas District of Columbia Florida Kansas Maine Michigan Missouri Nebraska New Hampshire New Mexico North Carolina North Dakota Ohio Oregon Pennsylvania South Carolina Tennessee Utah Vermont Virginia West Virginia and Wyoming]

Point One(b) (10) Settlorrsquos amendment of the trust was valid despite her failure to have her signature to the trust amendment witnessed

Neither the common law nor state statutes require a trust instrument or an amendment to a trust instrument to be executed in accordance with the formalities prescribed for execution of a will Indeed an inter vivos trust that does not involve real estate can be created orally Under the Uniform Trust Code the only requirements for creating a valid inter vivos trust are intent the specification of beneficiaries and the designation of a trustee See UNIF TRUST CODE sect 402 accord RESTATEMENT (THIRD) OF TRUSTS sect 13

Here the amendment meets the requirements of both the Uniform Trust Code and the common law Thus the fact that Settlorrsquos signature was not witnessed when she signed the amendment to the trust does not make the amendment invalid

Point Two (20) Under the Uniform Testamentary Additions to Trusts Act a revocable trust may be amended at any time prior to the settlorrsquos death and the amendment applies to probate assets poured into the trust at the settlorrsquos death pursuant to the settlorrsquos will even when the will was executed prior to the date of the amendment

Historically property owned by an individual at her death passed to the individualrsquos heirs or to beneficiaries designated in a will executed with the formalities (writing signing witnessing) prescribed by state law However when a will devises property to the trustee of an inter vivos trust then the provisions of the trustmdashwhich may not have been executed in accordance with the formalities required for willsmdasheffectively determine who will receive the property Because of this possibility some early cases held that if an inter vivos trust was not executed with the same formalities required for a valid will then the trust was ineffective to dispose of probate assets poured into the trust at the settlorrsquos death pursuant to the settlorrsquos will

This line of cases has been overturned by the Uniform Testamentary Additions to Trusts Act (the Act) now Uniform Probate Code sect 2-511 Under the Act adopted in almost all

39

Trusts and Future Interest Analysis

jurisdictions a testamentary bequest to the trustee of an inter vivos trust established by the testator during his or her lifetime is valid if the trust is in writing it is identified in the testatorrsquos will and the trust instrument was executed before concurrently with or after the execution of the will Id The Act further specifies that such a bequest is valid even if the trust is amendable or revocable and that a later amendment applies to assets passing to the trust by a previously executed will

Thus because the trust amendment is valid its terms apply to assets received by Bank from Settlorrsquos estate

Point Three (10) If the trust amendment was valid then the trust assets including assets passing to the trust under Settlorrsquos will should go to University

Under the trust amendment all trust assets (including the assets of Settlorrsquos probate estate poured into the trust) pass to University The facts provide no basis for failing to comply with Settlorrsquos stated intentions

Point Four (30) If the trust amendment was invalid trust assets including assets received pursuant to Settlorrsquos will should be held in accordance with the terms of the original trust instrument because those terms do not violate the Rule Against Perpetuities

Under the dispositive terms of the original trust instrument Settlor created successive income interests in her surviving children and grandchildren with a remainder interest in her great-grandchildren Because the trust was revocable the period during which the common law Rule Against Perpetuities requires that interests vest (ie 21 years plus lives in being) began to run from the date Settlor no longer had a power of revocation (here her death) not the date on which the trust was created See JESSE DUKEMINIER STANLEY J JOHANSON JAMES LINDGREN amp ROBERT SITKOFF WILLS TRUSTS AND ESTATES 678 (7th ed 2005)

Under the common law Rule Against Perpetuities Settlorrsquos trust is thus valid At the time of Settlorrsquos death she was survived by no children one granddaughter and no great-grandchildren Because Settlor cannot have more children after her death the only income beneficiary of the trust is Settlorrsquos surviving granddaughter This granddaughter is the only person who can produce great-grandchildren of Settlor thus all great-grandchildren must of necessity be born during the lifetime of Settlorrsquos only surviving granddaughter who is a life in being The granddaughterrsquos interest vested at Settlorrsquos death and the great-grandchildrenrsquos interest will vest at the death of the granddaughter There is no need to wait the additional 21 years permitted under the Rule Thus under the common law and the statute given in the facts the nonvested interest in the great-grandchildren is valid

[NOTE Both modern wait-and-see statutes and the Uniform Statutory Rule Against Perpetuities upon which the statute in the facts is modeled provide that before using either reform to validate an otherwise invalid nonvested interest one should first determine if the nonvested interest violates the common law Rule If it does not then there is no need to reform This proposition which is applicable in all MEE user jurisdictions that have not simply abrogated the rule is tested by this problem]

40

NEGOTIABLE INSTRUMENTS ANALYSIS (Negotiable Instruments III IV V)

ANALYSIS

Legal Problems

(1)(a) What rights does a person in possession of a note that has been indorsed in blank by the payee have against the maker of the note

(1)(b) Which defenses may the maker of a note raise against a person entitled to enforce it who is not a holder in due course but is a transferee from a holder in due course

(2) What rights does a person entitled to enforce a note have against an indorser who transferred it for consideration with no warranties

(3) What rights does a person entitled to enforce a note have against a previous holder who transferred it as a gift without indorsing it

DISCUSSION

Summary

The niece is a holder of the note and is thus a person entitled to enforce it The chef the issuer of the note is obligated to pay it to the niece as the person entitled to enforce it The niece is not subject to any defense or claim of the chef relating to the improper repair of the oven because the niece has the rights of a holder in due course When the buyer bought the note from the repairman the buyer became a holder in due course of the note and thus took it free of any personal defenses the chef had against the repairman Even though the niece is not herself a holder in due course of the note the niece succeeded to the buyerrsquos rights as holder in due course and thus took free of the chefrsquos personal defenses

Because the chef refused to pay the note the niece can recover from the repairman on the repairmanrsquos obligation as indorser The niece cannot recover on the note against the buyer however because the buyer did not indorse the note (and thus incurred no indorserrsquos obligation) and the buyer did not receive any consideration for transfer of the note to the niece (and therefore made no transfer warranty)

[NOTE Although Article 9 of the Uniform Commercial Code governs the sale of promissory notes (a point that might be correctly noted by examinees) that Article does not determine the answer to any of the questions posed]

Point One(a) (20) The niece is the holder of the note and thus may enforce it against the chef who is the issuer of the note

The chef is the maker of the note and thus its issuer See UCC sectsect 3-103 3-105 The issuer of a note is obligated to pay it in accordance with its terms to a ldquoperson entitled to enforcerdquo it UCC sect 3-412 The niece is a ldquoperson entitled to enforcerdquo the note This is because the niece is the holder of the note and a holder of a note is a person entitled to enforce it UCC sect 3-301 The niece is the holder of the note because (i) the repairmanrsquos signature on the back of the note not

41

Negotiable Instruments Analysis

accompanied by words indicating a person to whom the note was made payable was a ldquoblank indorsementrdquo which had the effect of making the note a bearer instrument (ii) anyone in possession of a bearer instrument is a holder of it and (iii) the niece is in possession of the note See UCC sectsect 1-201(b)(21)(A) 3-204 and 3-205 Accordingly the chef has an obligation to the niece to pay the note in accordance with its terms and the niece may enforce that obligation

Point One(b) (40) The niece is not a holder in due course of the note but because she is a transferee from the buyer who was a holder in due course she has the same enforcement rights as the buyer Because the buyer as a holder in due course would have been able to enforce the note against the chef without being subject to defenses or claims arising from the improper repair the niece has the same rights and will not be subject to the chefrsquos defenses or claims about the repair

As noted in Point One(a) the chef has an obligation to the niece to pay the note in accordance with its terms However except against a person with the rights of a holder in due course the chef can raise any defenses or claims in recoupment that he would have if the claim on the note were an ordinary contract claim UCC sect 3-305 Thus except against a holder in due course the chef would be able to raise the improper repair as a defense or a claim in recoupment (a claim in response to the niecersquos claim)

But claims in recoupment and most defenses cannot be raised against a person with the rights of a holder in due course Against a holder in due course the chef can raise only the four ldquorealrdquo defenses listed in UCC sect 3-305(a)(1) (infancy duress lack of legal capacity or illegality that nullifies the obligation of the obligor under other law fraud in the factum discharge in insolvency proceedings) none of which is present here

The niece is not a holder in due course because she did not take the note for value See UCC sectsect 3-302(a)(2)(i) (criteria for holder in due course status) and 3-303(a) (definition of ldquovaluerdquo) But this does not mean that the niece is subject to the chefrsquos claim arising out of the improper repair The buyer was a holder in due course of the note because he took the note for value ($9500) in good faith and without notice of any facts that would have alerted him to the chefrsquos defense against the repairman UCC sect 3-302(a)(2) As a holder in due course the buyer owned the note free of the chefrsquos claim because that claim did not constitute a ldquorealrdquo defense UCC sect 3-305(b) When the buyer gave the note to the niece this constituted a ldquotransferrdquo of the note See UCC sect 3-203(a) When a note is transferred the transferee receives ldquoany right of the transferor to enforce the instrument including any right as a holder in due courserdquo UCC sect 3-203(b) Under this rule (also known as the ldquoshelter principlerdquo) the buyer transferred his freedom from the chefrsquos defenses to the niece and the niece can enforce the note free of the chefrsquos defenses

Point Two (20) Because the chef dishonored the note the niece can recover from the repairman on the repairmanrsquos obligation as indorser

The chefrsquos refusal to pay the note constituted dishonor See UCC sect 3-502 The repairman as an indorser of the note (see Point One(a)) incurred the obligations of an indorser under UCC sect 3-415(a) When a note has been dishonored one of the obligations of an indorser is to pay the amount of the note to a person entitled to enforce it Therefore the repairman is liable for the amount of the note to the niece a person entitled to enforce the note (so long as the niece gives proper notice of dishonor to the repairman)

42

Negotiable Instruments Analysis

[NOTE Because the repairman indorsed the note without warranties there are no transfer warranties UCC sect 3-416 cmt 5]

Point Three (20) The niece cannot recover on the note against the buyer as either indorser or warrantor because the buyer did not indorse the note and did not receive consideration for transferring the note to the niece

The buyer did not indorse the note and therefore did not incur the obligation of an indorser to pay the note upon dishonor

The niece cannot recover from the buyer under a transfer warranty theory because transfer warranties are made only by a person ldquowho transfers an instrument for considerationrdquo Here the buyer gave the instrument to the niece as a gift So the buyer made no transfer warranty UCC sect 3-416(a) Therefore the niece cannot recover from the buyer on that theory

43

National Conference of Bar Examiners 302 South Bedford Street | Madison WI 53703-3622 Phone 608-280-8550 | Fax 608-280-8552 | TDD 608-661-1275

wwwncbexorg e-mail contactncbexorg

  • Contents
  • Preface
  • Description of the MEE
  • Instructions
  • February 2013 Questions
    • Real Property Question
    • Contracts Question
    • Constitutional Law Question
    • Secured Transactions Question
    • Federal Civil Procedure Question
    • Agency Question
    • Evidence Question
    • Trusts and Future Interests Question
    • Negotiable Instruments Question
      • February 2013 Analyses
        • Real Property Analysis
        • Contracts Analysis
        • Constitutional Law Analysis
        • Secured Transactions Analysis
        • Federal Civil Procedure Analysis
        • Agency Analysis
        • Evidence Analysis
        • Trusts and Future Interests Analysis
        • Negotiable Instruments Analysis
            • ltlt ASCII85EncodePages false AllowTransparency false AutoPositionEPSFiles true AutoRotatePages None Binding Left CalGrayProfile (Dot Gain 20) CalRGBProfile (sRGB IEC61966-21) CalCMYKProfile (US Web Coated 050SWOP051 v2) sRGBProfile (sRGB IEC61966-21) CannotEmbedFontPolicy Error CompatibilityLevel 14 CompressObjects Tags CompressPages true ConvertImagesToIndexed true PassThroughJPEGImages true CreateJobTicket false DefaultRenderingIntent Default DetectBlends true DetectCurves 00000 ColorConversionStrategy CMYK DoThumbnails false EmbedAllFonts true EmbedOpenType false ParseICCProfilesInComments true EmbedJobOptions true DSCReportingLevel 0 EmitDSCWarnings false EndPage -1 ImageMemory 1048576 LockDistillerParams false MaxSubsetPct 100 Optimize true OPM 1 ParseDSCComments true ParseDSCCommentsForDocInfo true PreserveCopyPage true PreserveDICMYKValues true PreserveEPSInfo true PreserveFlatness true PreserveHalftoneInfo false PreserveOPIComments true PreserveOverprintSettings true StartPage 1 SubsetFonts true TransferFunctionInfo Apply UCRandBGInfo Preserve UsePrologue false ColorSettingsFile () AlwaysEmbed [ true ] NeverEmbed [ true ] AntiAliasColorImages false CropColorImages true ColorImageMinResolution 300 ColorImageMinResolutionPolicy OK DownsampleColorImages true ColorImageDownsampleType Bicubic ColorImageResolution 300 ColorImageDepth -1 ColorImageMinDownsampleDepth 1 ColorImageDownsampleThreshold 150000 EncodeColorImages true ColorImageFilter DCTEncode AutoFilterColorImages true ColorImageAutoFilterStrategy JPEG ColorACSImageDict ltlt QFactor 015 HSamples [1 1 1 1] VSamples [1 1 1 1] gtgt ColorImageDict ltlt QFactor 015 HSamples [1 1 1 1] VSamples [1 1 1 1] gtgt JPEG2000ColorACSImageDict ltlt TileWidth 256 TileHeight 256 Quality 30 gtgt JPEG2000ColorImageDict ltlt TileWidth 256 TileHeight 256 Quality 30 gtgt AntiAliasGrayImages false CropGrayImages true GrayImageMinResolution 300 GrayImageMinResolutionPolicy OK DownsampleGrayImages true GrayImageDownsampleType Bicubic GrayImageResolution 300 GrayImageDepth -1 GrayImageMinDownsampleDepth 2 GrayImageDownsampleThreshold 150000 EncodeGrayImages true GrayImageFilter DCTEncode AutoFilterGrayImages true GrayImageAutoFilterStrategy JPEG GrayACSImageDict ltlt QFactor 015 HSamples [1 1 1 1] VSamples [1 1 1 1] gtgt GrayImageDict ltlt QFactor 015 HSamples [1 1 1 1] VSamples [1 1 1 1] gtgt JPEG2000GrayACSImageDict ltlt TileWidth 256 TileHeight 256 Quality 30 gtgt JPEG2000GrayImageDict ltlt TileWidth 256 TileHeight 256 Quality 30 gtgt AntiAliasMonoImages false CropMonoImages true MonoImageMinResolution 1200 MonoImageMinResolutionPolicy OK DownsampleMonoImages true MonoImageDownsampleType Bicubic MonoImageResolution 1200 MonoImageDepth -1 MonoImageDownsampleThreshold 150000 EncodeMonoImages true MonoImageFilter CCITTFaxEncode MonoImageDict ltlt K -1 gtgt AllowPSXObjects false CheckCompliance [ None ] PDFX1aCheck false PDFX3Check false PDFXCompliantPDFOnly false PDFXNoTrimBoxError true PDFXTrimBoxToMediaBoxOffset [ 000000 000000 000000 000000 ] PDFXSetBleedBoxToMediaBox true PDFXBleedBoxToTrimBoxOffset [ 000000 000000 000000 000000 ] PDFXOutputIntentProfile () PDFXOutputConditionIdentifier () PDFXOutputCondition () PDFXRegistryName () PDFXTrapped False CreateJDFFile false Description ltlt ARA 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 BGR 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 CHS ltFEFF4f7f75288fd94e9b8bbe5b9a521b5efa7684002000410064006f006200650020005000440046002065876863900275284e8e9ad88d2891cf76845370524d53705237300260a853ef4ee54f7f75280020004100630072006f0062006100740020548c002000410064006f00620065002000520065006100640065007200200035002e003000204ee553ca66f49ad87248672c676562535f00521b5efa768400200050004400460020658768633002gt CHT ltFEFF4f7f752890194e9b8a2d7f6e5efa7acb7684002000410064006f006200650020005000440046002065874ef69069752865bc9ad854c18cea76845370524d5370523786557406300260a853ef4ee54f7f75280020004100630072006f0062006100740020548c002000410064006f00620065002000520065006100640065007200200035002e003000204ee553ca66f49ad87248672c4f86958b555f5df25efa7acb76840020005000440046002065874ef63002gt CZE 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 DAN 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 DEU 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 ESP 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 ETI 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 FRA 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 GRE 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 HEB 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 HRV (Za stvaranje Adobe PDF dokumenata najpogodnijih za visokokvalitetni ispis prije tiskanja koristite ove postavke Stvoreni PDF dokumenti mogu se otvoriti Acrobat i Adobe Reader 50 i kasnijim verzijama) HUN ltFEFF004b0069007600e1006c00f30020006d0069006e0151007300e9006701710020006e0079006f006d00640061006900200065006c0151006b00e90073007a00ed007401510020006e0079006f006d00740061007400e100730068006f007a0020006c006500670069006e006b00e1006200620020006d0065006700660065006c0065006c0151002000410064006f00620065002000500044004600200064006f006b0075006d0065006e00740075006d006f006b0061007400200065007a0065006b006b0065006c0020006100200062006500e1006c006c00ed007400e10073006f006b006b0061006c0020006b00e90073007a00ed0074006800650074002e0020002000410020006c00e90074007200650068006f007a006f00740074002000500044004600200064006f006b0075006d0065006e00740075006d006f006b00200061007a0020004100630072006f006200610074002000e9007300200061007a002000410064006f00620065002000520065006100640065007200200035002e0030002c0020007600610067007900200061007a002000610074007400f3006c0020006b00e9007301510062006200690020007600650072007a006900f3006b006b0061006c0020006e00790069007400680061007400f3006b0020006d00650067002egt ITA 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 JPN ltFEFF9ad854c18cea306a30d730ea30d730ec30b951fa529b7528002000410064006f0062006500200050004400460020658766f8306e4f5c6210306b4f7f75283057307e305930023053306e8a2d5b9a30674f5c62103055308c305f0020005000440046002030d530a130a430eb306f3001004100630072006f0062006100740020304a30883073002000410064006f00620065002000520065006100640065007200200035002e003000204ee5964d3067958b304f30533068304c3067304d307e305930023053306e8a2d5b9a306b306f30d530a930f330c8306e57cb30818fbc307f304c5fc59808306730593002gt KOR ltFEFFc7740020c124c815c7440020c0acc6a9d558c5ec0020ace0d488c9c80020c2dcd5d80020c778c1c4c5d00020ac00c7a50020c801d569d55c002000410064006f0062006500200050004400460020bb38c11cb97c0020c791c131d569b2c8b2e4002e0020c774b807ac8c0020c791c131b41c00200050004400460020bb38c11cb2940020004100630072006f0062006100740020bc0f002000410064006f00620065002000520065006100640065007200200035002e00300020c774c0c1c5d0c11c0020c5f40020c2180020c788c2b5b2c8b2e4002egt LTH 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 LVI 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 NLD (Gebruik deze instellingen om Adobe PDF-documenten te maken die zijn geoptimaliseerd voor prepress-afdrukken van hoge kwaliteit De gemaakte PDF-documenten kunnen worden geopend met Acrobat en Adobe Reader 50 en hoger) NOR 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 POL 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 PTB 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 RUM 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 RUS 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 SKY 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 SLV 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 SUO 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 SVE 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 TUR ltFEFF005900fc006b00730065006b0020006b0061006c006900740065006c0069002000f6006e002000790061007a006401310072006d00610020006200610073006b013100730131006e006100200065006e0020006900790069002000750079006100620069006c006500630065006b002000410064006f006200650020005000440046002000620065006c00670065006c0065007200690020006f006c0075015f007400750072006d0061006b0020006900e70069006e00200062007500200061007900610072006c0061007201310020006b0075006c006c0061006e0131006e002e00200020004f006c0075015f0074007500720075006c0061006e0020005000440046002000620065006c00670065006c0065007200690020004100630072006f006200610074002000760065002000410064006f00620065002000520065006100640065007200200035002e003000200076006500200073006f006e0072006100730131006e00640061006b00690020007300fc007200fc006d006c00650072006c00650020006100e70131006c006100620069006c00690072002egt UKR 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 ENU (Use these settings to create Adobe PDF documents best suited for high-quality prepress printing Created PDF documents can be opened with Acrobat and Adobe Reader 50 and later) gtgt Namespace [ (Adobe) (Common) (10) ] OtherNamespaces [ ltlt AsReaderSpreads false CropImagesToFrames true ErrorControl WarnAndContinue FlattenerIgnoreSpreadOverrides false IncludeGuidesGrids false IncludeNonPrinting false IncludeSlug false Namespace [ (Adobe) (InDesign) (40) ] OmitPlacedBitmaps false OmitPlacedEPS false OmitPlacedPDF false SimulateOverprint Legacy gtgt ltlt AddBleedMarks false AddColorBars false AddCropMarks false AddPageInfo false AddRegMarks false ConvertColors ConvertToCMYK DestinationProfileName () DestinationProfileSelector DocumentCMYK Downsample16BitImages true FlattenerPreset ltlt PresetSelector MediumResolution gtgt FormElements false GenerateStructure false IncludeBookmarks false IncludeHyperlinks false IncludeInteractive false IncludeLayers false IncludeProfiles false MultimediaHandling UseObjectSettings Namespace [ (Adobe) (CreativeSuite) (20) ] PDFXOutputIntentProfileSelector DocumentCMYK PreserveEditing true UntaggedCMYKHandling LeaveUntagged UntaggedRGBHandling UseDocumentProfile UseDocumentBleed false gtgt ]gtgt setdistillerparamsltlt HWResolution [2400 2400] PageSize [612000 792000]gtgt setpagedevice

Page 8: February 2013 MEE Questions and Analyses

CONTRACTS QUESTION _______________

On January 2 a boat builder and a sailor entered into a contract pursuant to which the builder was to sell to the sailor a boat to be specially manufactured for the sailor by the builder The contract price was $100000 The written contract signed by both parties stated that the builder would tender the boat to the sailor on December 15 at which time payment in full would be due

On October 15 the builderrsquos workers went on strike and there were no available replacements

On October 31 the builderrsquos workers were still on strike and no work was being done on the boat The sailor read a news report about the strike and immediately sent a letter to the builder stating ldquoI am very concerned that my boat will not be completed by December 15 I insist that you provide me with assurance that you will perform in accordance with the contractrdquo The builder received the letter on the next day November 1

On November 25 the builder responded to the letter stating ldquoIrsquom sorry about the strike but it is really out of my hands I hope we settle it soon so that we can get back to workrdquo

Nothing further happened until December 3 when the builder called the sailor and said ldquoMy workers are back and I have two crews working overtime to finish your boat Your boat is task one Donrsquot worry wersquoll deliver your boat by December 15thrdquo The sailor immediately replied ldquoI donrsquot trust you As far as Irsquom concerned our contract is over I am going to buy my boat from a shipyardrdquo Two days later the sailor entered into a contract with a competing manufacturer to buy a boat similar to the boat that was the subject of the contract with the builder

The builder finished the boat on time and tendered it to the sailor on December 15 The sailor reminded the builder about the December 3 conversation in which the sailor had announced that ldquoour contract is overrdquo and refused to take the boat and pay for it

The builder has sued the sailor for breach of contract

1 What was the legal effect of the sailorrsquos October 31 letter to the builder Explain

2 What was the legal effect of the builderrsquos November 25 response to the sailorrsquos October 31 letter Explain

3 What was the legal effect of the sailorrsquos refusal to take and pay for the boat on December 15 Explain

4

CONSTITUTIONAL LAW QUESTION

AutoCo is a privately owned corporation that manufactures automobiles Ten years ago AutoCo purchased a five-square-mile parcel of unincorporated land in a remote region of the state and built a large automobile assembly plant on the land To attract workers to the remote location of the plant AutoCo built apartment buildings and houses on the land and leased them to its employees AutoCo owns and operates a commercial district with shops and streets open to the general public AutoCo named the area Oakwood and provides security fire protection and sanitation services for Oakwoodrsquos residents AutoCo also built operates and fully funds the only school in the region which it makes available free of charge to the children of its employees

A family recently moved to Oakwood The father and mother work in AutoCorsquos plant rent an apartment from AutoCo and have enrolled their 10-year-old son in Oakwoodrsquos school Every morning the students are required to recite the Pledge of Allegiance while standing and saluting an American flag With the approval of his parents the son has politely but insistently refused to recite the Pledge and salute the flag at the school on the grounds that doing so violates his own political beliefs and the political beliefs of his family As a result of his refusal to say the Pledge the son has been expelled from the school

To protest the schoolrsquos actions the father walked into the commercial district of Oakwood While standing on a street corner he handed out leaflets that contained a short essay critical of the schoolrsquos Pledge of Allegiance policy Some of the passersby who took the leaflets dropped them to the ground An AutoCo security guard saw the litter told the father that Oakwoodrsquos anti-litter rule prohibits leaflet distribution that results in littering and directed him to cease distribution of the leaflets and leave the commercial district When the father did not leave and continued to distribute the leaflets the security guard called the state police which sent officers who arrested the father for trespass

1 Did the sonrsquos expulsion from the school violate the First Amendment as applied through the Fourteenth Amendment Explain

2 Did the fatherrsquos arrest violate the First Amendment as applied through the Fourteenth Amendment Explain

5

SECURED TRANSACTIONS QUESTION

On June 1 a bicycle retailer sold two bicycles to a man for a total purchase price of $1500 The man made a $200 down payment and agreed to pay the balance in one year The man also signed a security agreement that identified the bicycles as collateral for the unpaid purchase price and provided that the man ldquoshall not sell or dispose of the collateral until the balance owed is paid in fullrdquo The retailer never filed a financing statement reflecting this security interest

The man had bought the bicycles for him and his girlfriend to use on vacation However shortly after he bought the bicycles the man and his girlfriend broke up The man has never used the bicycles

On August 1 the man sold one of the bicycles at a garage sale to a buyer who paid the man $400 for the bicycle The buyer bought the bicycle to ride for weekend recreation

On October 1 the man gave the other bicycle to his friend as a birthday present The friend began using the bicycle for morning exercise

Neither the buyer nor the friend had any knowledge of the manrsquos dealings with the retailer

1 Does the buyer own the bicycle free of the retailerrsquos security interest Explain

2 Does the friend own the bicycle free of the retailerrsquos security interest Explain

6

FEDERAL CIVIL PROCEDURE QUESTION _____

Mother and Son who are both adults are citizens and residents of State A Mother owned an expensive luxury car valued in excess of $100000 Son borrowed Motherrsquos car to drive to a store in State A As Son approached a traffic light that had just turned yellow he carefully braked and brought the car to a complete stop Driver who was following immediately behind him failed to stop and rear-ended Motherrsquos car which was damaged beyond repair Son was seriously injured Driver is a citizen of State B

Son sued Driver in the United States District Court for the District of State A alleging that she was negligent in the operation of her vehicle Son sought damages in excess of $75000 for his personal injuries exclusive of costs and interest In her answer Driver alleged that Son was contributorily negligent in the operation of Motherrsquos car She further alleged that the brake lights on Motherrsquos car were burned out and that Motherrsquos negligent failure to properly maintain the car was a contributing cause of the accident

Following a trial on the merits in Sonrsquos case against Driver the jury answered the following special interrogatories

Do you find that Driver was negligent in the operation of her vehicle Yes

Do you find that Son was negligent in the operation of Motherrsquos car No

Do you find that Mother negligently failed to ensure that the brake lights on her car were in proper working order Yes

The judge then entered a judgment in favor of Son against Driver Driver did not appeal

Two months later Mother sued Driver in the United States District Court for the District of State A alleging that Driverrsquos negligence in the operation of her vehicle destroyed Motherrsquos luxury car Mother sought damages in excess of $75000 exclusive of costs and interest

State A follows the same preclusion principles that federal courts follow in federal-question cases

1 Is Motherrsquos claim against Driver barred by the judgment in Son v Driver Explain

2 Does the juryrsquos conclusion in Son v Driver that Mother had negligently failed to maintain the brake lights on her car preclude Mother from litigating that issue in her subsequent suit against Driver Explain

3 Does the juryrsquos conclusion in Son v Driver that Driver was negligent preclude Driver from litigating that issue in the Mother v Driver lawsuit Explain

7

_____

AGENCY QUESTION

Over 5000 individuals in the United States operate hot-air balloon businesses A hot-air balloon has four key components the balloon that holds the heated air the basket that houses the riders the propane burner that heats the air in the balloon and the propane storage tanks

The owner of a hot-air balloon business recently notified several basket and burner manufacturers that she or her agent might be contacting them to purchase baskets or burners The owner did not specifically name any person as her agent Basket and burner manufacturers regularly receive such notices from hot-air balloon operators Such notices typically include no restrictions on the types of baskets or burners agents might purchase for their principals

The owner then retained an agent to acquire baskets burners and fuel tanks from various manufacturers The owner authorized the agent to buy only (a) baskets made of woven wicker (not aluminum) (b) burners that use a unique ldquowhisper technologyrdquo (so as not to scare livestock when the balloon sails over farmland) and (c) propane fuel tanks

The agent then entered into three transactions with manufacturers all of whom had no prior dealings with either the owner or the agent

(1) The agent and a large manufacturer of both wicker and aluminum baskets signed a contract for the purchase of four aluminum baskets for a total cost of $60000 The agent never told the manufacturer that he represented the owner or any other principal The contract listed the agent as the buyer and listed the ownerrsquos address as the delivery address but did not indicate that the address was that of the owner rather than the agent When the baskets were delivered to the owner she learned for the first time that the agent had contracted to buy aluminum not wicker baskets The owner immediately rejected the baskets and returned them to the manufacturer Neither the owner nor the agent has paid the basket manufacturer for them

(2) The agent contacted a burner manufacturer and told him that the agent represented a well-known hot-air balloon operator who wanted to purchase burners The agent did not disclose the ownerrsquos name The agent and the burner manufacturer signed a contract for the purchase of four burners that did not have ldquowhisper technologyrdquo for a total price of $70000 The burner contract like the basket contract listed the ownerrsquos address for delivery but did not disclose whose address it was The burners were delivered to the ownerrsquos business and the owner discovered that the agent had ordered the wrong kind of burners The owner rejected the burners and returned them to the manufacturer Neither the owner nor the agent has paid the burner manufacturer for the burners

(3) The agent contracted with a solar cell manufacturer to make three cells advertised as ldquostrong enough to power all your ballooning needsrdquo The agent did not tell the manufacturer that he was acting on behalf of any other person One week after the cells were delivered to the agent he took them to the owner who installed them and discovered that she could save a lot of money using solar cells instead of propane to power her balloons The owner decided to keep the solar cells but she has not paid the manufacturer for them

8

Agency Question

Assume that the rejection of the baskets and the burners and the failure to pay for the solar cells constitute breach of the relevant contracts

1 Is the owner liable to the basket manufacturer for breach of the contract for the aluminum baskets Is the agent liable Explain

2 Is the owner liable to the burner manufacturer for breach of the contract for the burners Is the agent liable Explain

3 Is the owner liable to the solar cell manufacturer for breach of the contract for the solar cells Is the agent liable Explain (Do not address liability based upon restitution or unjust enrichment)

9

EVIDENCE QUESTION _____

A woman who owns a motorized scooter brought her scooter to a mechanic for routine maintenance service As part of the maintenance service the mechanic inspected the braking system on the scooter As soon as the mechanic finished inspecting and servicing the scooter he sent the woman a text message to her cell phone that read ldquoJust finished your service When you pick up your scooter you need to schedule a follow-up brake repair Wersquoll order the partsrdquo

The woman read the mechanicrsquos text message and returned the next day to pick up her scooter As the woman was wheeling her scooter out of the shop she saw the mechanic working nearby and asked ldquoIs my scooter safe to ride for a whilerdquo The mechanic responded by giving her a thumbs-up The woman waved and rode away on the scooter

One week later while the woman was riding her scooter a pedestrian stepped off the curb into a crosswalk and the woman collided with him causing the pedestrian severe injuries The woman had not had the scooterrsquos brakes repaired before the accident

The pedestrian has sued the woman for damages for his injuries resulting from the accident The pedestrian has alleged that (1) the woman lost control of the scooter due to its defective brakes (2) the woman knew that the brakes needed repair and (3) it was negligent for the woman to ride the scooter knowing that its brakes needed to be repaired

The woman claims that the brakes on the scooter worked perfectly and that the accident happened because the pedestrian stepped into the crosswalk without looking and the woman had no time to stop The woman the pedestrian and the mechanic will testify at the upcoming trial

The pedestrian has proffered an authenticated copy of the mechanicrsquos text message to the woman

The woman plans to testify that she asked the mechanic ldquoIs my scooter safe to ride for a whilerdquo and that he gave her a thumbs-up in response

The evidence rules in this jurisdiction are identical to the Federal Rules of Evidence

Analyze whether each of these items of evidence is relevant and admissible at trial

1 The authenticated copy of the mechanicrsquos text message

2 The womanrsquos testimony that she asked the mechanic ldquoIs my scooter safe to ride for a whilerdquo and

3 The womanrsquos testimony describing the mechanicrsquos thumbs-up

10

TRUSTS AND FUTURE INTERESTS QUESTION

Ten years ago Settlor validly created an inter vivos trust and named Bank as trustee The trust instrument provided that Settlor would receive all of the trust income during her lifetime The trust instrument further provided that

Upon Settlorrsquos death the trust income shall be paid in equal shares to Settlorrsquos surviving children for their lives Upon the death of the last surviving child the trust income shall be paid in equal shares to Settlorrsquos then-living grandchildren for their lives Upon the death of the survivor of Settlorrsquos children and grandchildren the trust corpus shall be distributed in equal shares to Settlorrsquos then-living great-grandchildren

The trust instrument expressly specified that the trust was revocable but it was silent regarding whether Settlor could amend the trust instrument

Immediately after creating the trust Settlor validly executed a will leaving her entire estate to Bank as trustee of her inter vivos trust to ldquohold in accordance with the terms of the trustrdquo

Five years ago Settlor signed an amendment to the inter vivos trust The amendment changed the disposition of the remainder interest specifying that all trust assets ldquoshall be paid upon Settlorrsquos death to Universityrdquo Settlorrsquos signature on this amendment was not witnessed

A state statute provides that any trust interest that violates the common law Rule Against Perpetuities ldquois nonetheless valid if the nonvested interest in the trust actually vests or fails to vest either (a) within 21 years of lives in being at the creation of the nonvested interest or (b) within 90 years of its creationrdquo

Recently Settlor died leaving a probate estate of $200000 She was survived by no children one granddaughter (who would be Settlorrsquos only heir) and no great-grandchildren The granddaughter has consulted your law firm and has raised four questions regarding this trust

1 Was Settlorrsquos amendment of the inter vivos trust valid Explain

2 Assuming that the trust amendment was valid do its provisions apply to Settlorrsquos probate assets Explain

3 Assuming that the trust amendment was valid how should trust assets be distributed Explain

4 Assuming that the trust amendment was invalid how should trust assets be distributed Explain

11

NEGOTIABLE INSTRUMENTS QUESTION

A chef entered into a contract with a repairman pursuant to which the repairman agreed to repair the chefrsquos commercial oven for $10000 The repairman agreed to accept as payment a negotiable promissory note for $10000 payable two months after its issuance

After the repairman worked on the oven the chef gave him a $10000 note as payment for the work As agreed the note was signed by the chef as maker was payable to the order of the repairman was payable in two months and fulfilled all criteria for negotiability

The next day the repairman sold the note to a buyer for $9500 To effectuate the sale the repairman wrote ldquono warrantiesrdquo on the back of the note signed his name immediately below that and handed the note to the buyer The buyer bought the note in good faith and without knowledge of any facts relating to the work that the repairman had performed for the chef

Later the buyer gave the note to his niece as a gift To effectuate the gift the buyer handed the note to the niece but did not indorse it

Shortly thereafter the chef discovered that the repair work had been done improperly and the oven still did not function correctly The chef tried repeatedly to get the repairman to return to correct the repair work but the repairman ignored all the chefrsquos calls

On the notersquos due date the niece contacted the chef and demanded that he pay the amount of the note to her The chef refused and told the niece that he would not pay the note because the repairman did not properly repair the oven

1 What are the niecersquos rights against the chef Explain

2 What are the niecersquos rights against the repairman Explain

3 What are the niecersquos rights against the buyer Explain

12

February 2013 MEE

ANALYSES

Real Property Contracts

Constitutional Law Secured Transactions

Federal Civil Procedure Agency

Evidence Trusts and Future Interests

Negotiable Instruments

REAL PROPERTY ANALYSIS (Real Property ID1a 4 amp 5)

ANALYSIS

Legal Problems

(1) Does the tenant have a defense to the landlordrsquos action for unpaid rent based on constructive eviction

(2) Does the tenant have a defense to the landlordrsquos action for unpaid rent based on the tenantrsquos surrender of the premises

(3) What if anything may the landlord recover from the tenant for the period after the tenant vacated the building

DISCUSSION

Summary

Under the common law the tenant does not have a defense to the landlordrsquos action for unpaid rent based on constructive eviction Constructive eviction is based on the tenant proving that (1) the landlord breached a duty to the tenant (2) the breach caused a loss by the tenant of the substantial use and enjoyment of the premises (3) the tenant gave the landlord adequate notice and opportunity to repair and (4) the tenant vacated the leased premises Here there was no constructive eviction because although the tenant vacated and gave the landlord adequate notice the landlord breached no express or implied duty to the tenant to repair the premises

The tenant does not have a defense based on the landlordrsquos acceptance of his surrender of the premises a landlordrsquos retention of keys does not constitute an acceptance of the tenantrsquos surrender unless the landlord so intended and here the landlordrsquos statements to the tenant at the time of the surrender of the keys do not evidence the intent to accept the tenantrsquos surrender

Under the common law a landlord has no duty to mitigate damages but also cannot sue for rents due in the future Under this approach the landlord can sue only for past-due rents Using this approach on November 1 the landlord could recover all the rent past due (ie rent for September and October) but could not recover for rents due in the future However some courts have authorized recovery for future rent minus the fair market rental value of the premises It is thus possible that the landlord could recover damages equal to the amount of rent due from September 1 to the end of the six-year lease term ($180000) minus the propertyrsquos fair-market rental value over that same period

Point One (45) The tenant was not constructively evicted because the landlord had no duty to repair the commercial premises that were the subject of the lease

The landlord and the tenant entered into a term-of-years lease because the lease specified both a beginning and an ending date HERBERT HOVENKAMP amp SHELDON F KURTZ THE LAW OF

PROPERTY 256 (5th ed 2001) Although a term-of-years lease normally cannot be terminated by the tenant prior to the end of the term a tenant may terminate a term-of-years lease if the tenant

15

Real Property Analysis

is constructively evicted See id at 286ndash88 Typically as here a claim of constructive eviction is made as a defense to a landlordrsquos action for damages or unpaid rent

In order to establish a constructive eviction the tenant must prove that the landlord breached a duty to the tenant such as a duty to repair and that the landlordrsquos breach caused a loss of the substantial use and enjoyment of the premises The tenant must also show that he gave the landlord notice adequate to permit the landlord to meet his duty to the tenant and that the tenant vacated the leased premises Id see also JOHN G SPRANKLING UNDERSTANDING

PROPERTY LAW sect 1704 (2d ed 2007) Under the common law there was no implied duty on the part of a landlord to repair

leased premises such a duty arose only if expressly set forth in the lease SPRANKLING supra sect 1702[B] Here the written lease contained no term requiring the landlord to repair the air-conditioning Even if the conversation created a lease term that the building had air-conditioning that itself should not create a duty for the landlord to repair it

Over the past several decades courts have generally implied a duty to repair in residential leases either as part of a revised constructive eviction doctrine or based on an implied warranty of habitability JOSEPH W SINGER PROPERTY 469ndash70 (3d ed 2010) This shift has been justified based on the economic disparity between the typical landlord and tenant as well as the fact that residential tenants generally lack both the authority to authorize repairs to common areas of a building and the incentive to make repairs that will ultimately benefit the landlord

However courts have been more reluctant to imply a duty to repair in commercial leases a context in which the tenant is often a valuable business and in a better position to assess and make repairs than is the landlord But see eg Davidow v Inwood North Professional Group 747 SW2d 373 (Tex 1988) When courts have implied a duty to repair in a commercial lease it is typically when the repair has been mandated by public authorities and involves work so substantial that it would not ordinarily fall within the tenantrsquos common law repair duty andor the value of the repair would primarily inure to the landlordrsquos reversionary interest See Brown v Green 884 P2d 55 (Cal 1994) Eugene L Grant et al The Tenant as Terminator Constructive Eviction in Modern Commercial Leases 2 THE COMMERCIAL PROPERTY LEASE ch 15 (ABA 1997) Some courts have also permitted constructive eviction claims by commercial tenants of office buildings based on repairs required in common areas of the building See id Echo Consulting Services Inc v North Conway Bank 669 A2d 227 (NH 1995)

Here the tenant is the owner of a valuable manufacturing operation and is the exclusive occupant of the building the repair has not been mandated by public authorities and the repair is not structural To the contrary the repair involves a feature of the building of unusual importance in the tenantrsquos manufacturing operation and the tenant is likely far more knowledgeable than the landlord about the air-conditioning specifications necessary for the manufacture of the tenantrsquos product

Based on these facts it is unlikely that a court will find that the tenant in this case was constructively evicted Although the tenant can show that he gave adequate notice to the landlord of the air-conditioning malfunction and vacated the premises the lease was commercial and it did not contain any promises or covenants by the landlord except a covenant of quiet enjoyment a covenant of quiet enjoyment does not entail any repair obligations

[NOTE An examineersquos conclusion is less important than his or her demonstrated awareness of the elements of constructive eviction and the need to imply a repair duty for such a defense to be viable here Although the implied warranty of habitability is not available to this tenant Texas Minnesota and Massachusetts imply a warranty of suitability in commercial leases in limited circumstances and an examinee might argue that this warranty should apply

16

Real Property Analysis

here If an examinee concludes that this warranty applies he or she should discuss the other requirements for constructive eviction

If the examinee wrongly concludes that the first element for a constructive eviction has been met the examinee will then have to discuss the remaining three elements in order to conclude that the tenant can claim constructive eviction The tenant would have a strong argument that the second elementmdashsubstantial interference with the use and enjoyment of the premisesmdashalso is met As indicated above the landlord was aware that a functioning air-conditioning system was vital to the tenantrsquos manufacturing operations The facts further indicate that the system had failed three times in the past few months The landlord may try to argue that the malfunctions did not substantially interfere with the tenantrsquos use of the premises because the malfunctions caused the temperature to climb above 81 degrees for only a short period of timemdash 3 hours 6 hours and 10 hours respectivelymdashon each occasion The tenant will argue however that the landlord was aware that the tenantrsquos manufacturing operations could tolerate temperatures above 81 degrees for no more than 6 hours The final malfunction exceeded that limit destroying $150000 worth of the tenantrsquos products

The tenant would also have a strong argument that the third element is met notice and opportunity to cure The tenant notified the landlord of the problem immediately upon the systemrsquos first malfunction and did so again when it malfunctioned a second time and then a third time The landlord might argue that there was insufficient time to cure the problem because the system corrected itself within a few hours on the first and second times Although the malfunction lasted more than 10 hours the third time the landlord might argue that the time period was insufficient to get a repair person on the premises A court would be likely to find this argument unpersuasive however because the landlord could have attempted to correct the problem after the first and second malfunctions

Assuming that the landlord was given sufficient notice and opportunity to cure a court would be likely to conclude that the tenant also satisfied the final element of vacating the premises within a reasonable time The landlord might argue that the tenant remained in the premises for almost four months after the air conditioning first failed which would suggest that the problem was not so severe as to have constructively evicted the tenant The tenant will argue however that he gave the landlord three months to cure the problem after the first two malfunctions threatened (but did not actually harm) his operations The tenant then moved out shortly after the final malfunction caused temperatures to exceed the tolerance levels of his manufacturing operations]

Point Two (10) The landlord did not accept the tenantrsquos surrender of the lease

When a tenant wrongfully moves from leased premises with the intent to terminate the lease the landlord may either accept the tenantrsquos surrender of the premises and terminate the lease or hold the tenant to the terms of the lease See HOVENKAMP amp KURTZ supra at 295ndash96 Here the tenantrsquos only basis for the claim that the landlord accepted his surrender is the landlordrsquos retention of the keys Many courts have considered whether a landlordrsquos retention of keys delivered by a tenant constitutes acceptance of surrender The weight of the case law holds that retention of the keys alone does not constitute acceptance of surrender without other evidence showing that the landlord intended to accept the surrender See generally 49 AM JUR 2d Landlord and Tenant sect 213

Here the landlordrsquos note saying ldquoI repeat the air-conditioning is not my problem You have leased the building and you should fix itrdquo strongly suggests that the landlord did not intend

17

Some courts have rejected the no-mitigation-of-damages rule based on efficiency concerns and societyrsquos interest in assuring that resources remain in the stream of commerce rather than lying vacant see id at 464ndash65 and allow landlords to sue tenants who have wrongfully terminated a lease for damages equal to the difference between the unpaid rent due under the lease and the propertyrsquos fair market rental value Other courts have abandoned the no-recovery-for-future-rent rule These courts responding to the fact that a tenant may well disappear or be judgment-proof by the time a lease term is concluded have allowed a landlord to collect damages equal to the value of rent over the entire lease term minus the propertyrsquos fair rental value when a tenant has wrongfully terminated a lease and unequivocally shown an intention not to return to the premises or pay future rent Under this approach a landlord receives approximately the same amount he would have received were there a duty to mitigate damages See Sagamore Corp v Willcutt 180 A 464 (Conn 1935)

Real Property Analysis

to accept the tenantrsquos surrender The tenant might argue that the landlordrsquos failure to make a similar statement when the keys were sent to her a second time and she retained them evidences a change of heart However it is likely that a court would find that the landlordrsquos retention of the keys represented a decision to safeguard the keys not to accept the tenantrsquos surrender

[NOTE An examinee should receive credit for arguing the other way with a well-reasoned argument]

Point Three (45) Under the common law the landlord had no duty to mitigate damages Additionally a landlord was not entitled to recover unpaid rents due in the future but was only entitled to recover rents in arrears at the time of the commencement of the suit Applying the common law here the landlord could recover $5000 the amount of rents due at the commencement of the suit ($2500 for September and the same for October) Today some courts allow the landlord under certain circumstances to sue the tenant for damages (not rent) equal to the difference if any between the unpaid promised rent for the balance of the term (here $175000) and the propertyrsquos fair rental value for the balance of the term

Under the common law because a lease was viewed as a conveyance instead of a contract a landlord had no duty to mitigate damages resulting from a tenantrsquos wrongful termination of a lease A landlord could thus recover the full value of rents that were due and unpaid at the time of the suit However under the common law a landlord could not sue a tenant for rents due in the future because there was always a possibility that the tenant might pay the rent when it was due See SINGER supra at 462 Thus using the common law approach on November 1 the landlord could only recover the full value of the two monthsrsquo rent actually due and unpaid ie $5000 for September and October

Here because the tenant returned the keys to the landlord and said ldquoI will not be returning to the building or making further rent paymentsrdquo the landlord could establish abandonment and an intention not to return It is thus possible that the landlord might recover damages in the amount of $5000 (for the months of September and October) plus the present value of $175000 minus the fair market rental value of the property over the remaining months of the lease

18

CONTRACTS ANALYSIS ____ (Contracts II IVE)

ANALYSIS

Legal Problems

(1) What was the legal effect of the sailorrsquos October 31 letter to the builder

(2)(a) What was the legal effect of the builderrsquos November 25 response to the sailorrsquos October 31 letter

(2)(b) What was the legal effect of the sailorrsquos refusal to take and pay for the boat on December 15

DISCUSSION

Summary

This is a sale of goods governed by the Uniform Commercial Code Because the sailor had reasonable grounds for insecurity about the builderrsquos ability to deliver the boat in a timely manner when the sailor learned about the strike on October 31 the sailor was legally justified in sending the letter to the builder seeking adequate assurance of the builderrsquos performance pursuant to the contract The builderrsquos failure to provide such assurance within a reasonable time operated as a repudiation of the contract However the builder was free to retract the repudiation before the sailor either cancelled the contract or materially changed position in reliance on the builderrsquos repudiation The builder retracted the repudiation when he informed the sailor that the workers were back and that the boat would be delivered by the date stipulated in the partiesrsquo contract Because the sailor had taken no action in response to the original repudiation he no longer had the right to cancel the contract with the builder The sailorrsquos subsequent statement that ldquoour contract is overrdquo may have constituted repudiation by the sailor In any event when the sailor failed to perform on December 15 that constituted breach

Point One (35) Because the sailor had reasonable grounds for insecurity with respect to the builderrsquos performance the sailorrsquos letter to the builder was a justified demand seeking assurance of the builderrsquos performance under the contract failure of the builder to provide such assurance within a reasonable time constituted repudiation of the contract

The sailor was legally justified in sending the letter to the builder on October 31 Contract parties are entitled to expect due performance of contractual obligations and are permitted to take steps to protect that expectation UCC sect 2-609 states that ldquo[w]hen reasonable grounds for insecurity arise with respect to the performance of either party the other may in writing demand adequate assurance of due performance rdquo Here the sailor learned on October 31 that the builderrsquos workers were on strike This gave the sailor reasonable grounds for insecurity about the builderrsquos ability to complete performance on time and thus gave the sailor the right to seek adequate assurance from the builder Because the sailorrsquos demand for assurance was justified the builder was required to provide assurance that was adequate under the circumstances within a reasonable time (not to exceed 30 days) or be held to have repudiated the contract UCC sect 2-609(4)

19

Contracts Analysis

Point Two(a) (30) The builder did not within a reasonable time provide the sailor adequate assurance of due performance this failure to provide assurance constituted a repudiation of the contract

Because the sailor with legal justification (see Point One) demanded from the builder assurance of due performance the builderrsquos failure to provide such assurance within a reasonable time was a repudiation of their contract See UCC sect 2-609(4) (ldquoAfter receipt of a justified demand[] failure to provide within a reasonable time not exceeding thirty days assurance of due performance is a repudiation of the contractrdquo) On October 31 the sailor requested that the builder provide adequate assurance regarding the completion of the boat by December 15 The builder did not respond to the sailorrsquos letter until November 25mdashnearly a month later Even if that response had been given in a reasonable time it nonetheless did not provide assurance of due performance It simply stated ldquoIrsquom sorry about the strike but it is really out of my hands I hope we settle it soon so that we can get back to workrdquo Therefore the builderrsquos November 25 response did not provide adequate assurance in response to the sailorrsquos justified request Thus the builder had repudiated the contract

Point Two(b) (35) Although the builder repudiated the contract with the sailor the builder probably retracted that repudiation on December 3 and the sailor was no longer entitled to cancel their contract Thus the sailorrsquos failure to perform the sailorrsquos obligations under the contract constituted a breach

The builderrsquos failure to provide adequate assurance of performance constituted a repudiation of their contract (see UCC sect 2-609(4)) but the builder was free to retract that repudiation until the sailor cancelled the contract or materially changed his position or indicated by communication or action that the sailor considered the repudiation to be final See UCC sect 2-611(1) (ldquoUntil the repudiating partyrsquos next performance is due he can retract his repudiation unless the aggrieved party has since the repudiation cancelled or materially changed his position or otherwise indicated that he considers the repudiation finalrdquo)

Here the facts state that before the builderrsquos December 3 telephone call to the sailor the sailor did nothing in response to the builderrsquos repudiation such as contracting with a third party for a boat The builderrsquos December 3 call informing the sailor that the boat would be timely delivered probably constituted a retraction of the repudiation because it clearly indicated to the sailor that the builder would be able to perform UCC sect 2-611(2) Thus after being so informed the sailor did not have the right to treat their contract as cancelled UCC sect 2-611(3) Accordingly the sailorrsquos failure to perform the sailorrsquos obligations under the contract by taking the boat and paying for it constituted a breach of the contract

20

CONSTITUTIONAL LAW ANALYSIS (Constitutional Law IVA F2b amp e)

ANALYSIS

Legal Problems

(1) Does AutoCorsquos operation of a ldquocompany townrdquo result in its actions counting as those of the state for purposes of constitutional analysis

(2) Does the expulsion of a schoolchild for failure to recite the Pledge of Allegiance violate the First Amendment as applied through the Fourteenth Amendment

(3) Does the arrest of a pamphleteer in connection with violation of an anti-littering rule where the littering is done by the recipients of leaflets distributed by the pamphleteer violate the First Amendment as applied through the Fourteenth Amendment

DISCUSSION

Summary

The First Amendment as applied through the Fourteenth Amendment applies only to state action It does not typically govern private actors However courts have found state action where the private actor has exercised a ldquopublic functionrdquo such as running a privately owned ldquocompany townrdquo as AutoCo has done here Thus First Amendment protections apply By requiring the son to participate in a mandatory Pledge of Allegiance ceremony AutoCo has compelled the expression of political belief in violation of the First Amendment as applied through the Fourteenth Amendment The fatherrsquos arrest in connection with breaching the anti-litter rule also violated the First Amendment as applied through the Fourteenth Amendment Although state actors can regulate the incidental effects of speech on the public streets on a content-neutral basis this power is limited and cannot extend to punishing a distributor of literature because of littering by third parties

Point One (30) AutoCorsquos operation of a company town (including a school) makes it a state actor under the public function strand of the state action doctrine

The individual rights protections of the Constitution apply only where there is ldquostate actionrdquomdash either direct action by the government or some action by a private party that is fairly attributable to the government As a general rule the actions of a private company like AutoCo or of a private school like the school operated by AutoCo would not constitute state action and the protections of the Constitution (in this case the First Amendment) would not apply

However there are situations in which the actions of a private actor are attributed to the state One such situation is when the private actor undertakes a public function There are not many bright-line rules in the Supreme Courtrsquos state action doctrine but one of them is this Where a private actor undertakes a ldquopublic functionrdquo the Constitution applies to those actions Where a corporation operates a privately owned ldquocompany townrdquo that provides essential services typically provided by a state actor the public function doctrine applies and the Constitution

21

Constitutional Law Analysis

binds agents of the town as if they were agents of the government See eg Marsh v Alabama 326 US 501 (1946) Here AutoCo does more than own the town it provides security services fire protection sanitation services and a school Thus the actions of AutoCo constitute state action and are governed by the Fourteenth Amendment

Point Two (35) The sonrsquos expulsion for failure to recite the Pledge of Allegiance violates the First Amendment as applied through the Fourteenth Amendment as a compelled expression of political belief

As explained in Point One the First Amendment applies to the school as a state actor Although children in public schools (and in schools subject to the First Amendment like

the Oakwood school) have some First Amendment rights Tinker v Des Moines Independent Community School District 393 US 503 506 (1969) schools have greater leeway to regulate the speech of students and teachers than the state would have outside the school context Hazelwood School Dist v Kuhlmeier 484 US 260 (1988) Morse v Frederick 551 US 393 (2007) However the Supreme Court has long held that public schools may not force their students to participate in a flag salute ceremony when it offends the political or religious beliefs of the students or their families West Virginia Board of Educ v Barnette 319 US 624 (1943) (invalidating a mandatory public school flag salute ceremony) see also Wooley v Maynard 430 US 705 (1977) (invalidating compelled expression of political belief on state-issued license plates)

In this case the school requires its students to participate in a flag salute and Pledge of Allegiance ceremony and punishes them when they refuse to participate Pursuant to this policy the school has expelled the son This expulsion violates the First Amendment ban on compelled expression

Point Three (35) Because the father was distributing leaflets in a traditional public forum his trespass arrest violated the First Amendment as applied through the Fourteenth Amendment

As explained in Point One AutoCo is treated as a state actor Thus Oakwoodrsquos commercial district is treated as government-owned property for purposes of the First Amendment Thus the leafleting here is subject to the First Amendment because it is an expressive activity Schneider v State of New Jersey Town of Irvington 308 US 147 (1939) When expression takes place on government-owned property government regulation of the expression is assessed under the public forum doctrine Public streets and sidewalks have long been held to be the classic example of a ldquotraditional public forumrdquo open to the public for expression Hague v CIO 307 US 496 515ndash16 (1939) Because the father was distributing leaflets while standing on a street corner in the commercial district his expressive activity occurred in a traditional public forum

When a state tries to regulate expressive activity in a traditional public forum it is prohibited from doing so based on the expressive activityrsquos content unless its regulation is narrowly tailored to achieve a compelling governmental interest (ldquostrict scrutinyrdquo) In this case however AutoCo is regulating the fatherrsquos expressive activity on the ostensibly neutral ground that his expressive activity has produced litter and made the street unsightly When a state tries to regulate expressive activity without regard to its content intermediate scrutiny applies Under intermediate scrutiny the true purpose of the regulation may not be the suppression of ideas (if so then strict scrutiny applies) the regulation must be narrowly tailored to achieve a significant

22

Constitutional Law Analysis

governmental interest and it must leave open ample alternative channels for expressive activity Ward v Rock Against Racism 491 US 781 791 (1989)

Here the application of the ordinance to the father will fail for two reasons First the Supreme Court has held that the governmentrsquos interest in keeping the streets clean is insufficient to ban leafleting in the public streets as the government power to regulate with incidental effects on public sidewalk speech is very limited See eg Schneider 308 US at 162 (leafletinglittering) Second the regulation (a blanket ban on distribution that results in littering) is not narrowly tailored to protect expression A narrowly tailored alternative would be prosecution only of people who litter Moreover the effect of the littering rule is likely to be a ban on all leafleting thus eliminating an entire class of means of expression This raises the possibility that there are not ldquoample alternative channels of communicationrdquo open to the father as required under the Courtrsquos standard of review for content-neutral regulation of speech

[NOTE Some examinees might argue that this is a ldquotime place and mannerrdquo restriction and that AutoCo might have greater latitude to regulate the public sidewalks under this theory This argument is incorrect for two reasons First the Supreme Court has held that the power to regulate speakers through littering laws is very limited for the reasons given and in the cases cited above But more generally a ldquotime place and mannerrdquo restriction involves the shifting of speech from one time and place to another or to another manner here there is no shifting but a direct punishment for expressive activity (albeit one couched in content-neutral terms) In addition some examinees might read the ordinance to be in effect a total ban on leafleting since most leafleting will produce some litter Those examinees might note that the Court has required total bans on an entire mode of expression to satisfy strict scrutiny and analyze the fatherrsquos prosecution here accordingly See United States v Grace 461 US 171 177 (1983) (invalidating ban on display of signs on public sidewalks surrounding US Supreme Court ldquo[a]dditional restrictions such as an absolute prohibition on a particular type of expression will be upheld only if narrowly drawn to accomplish a compelling governmental interestrdquo)]

23

SECURED TRANSACTIONS ANALYSIS (Secured Transactions IID E IVA B C)

ANALYSIS

Legal Problems

(1) Is a purchase-money security interest in consumer goods perfected even though there has been no filing of a financing statement

(2) Does a person who buys consumer goods for personal use take those goods free of a prior perfected purchase-money security interest in the goods

(3) Does a person who receives consumer goods as a gift take those goods subject to a prior perfected security interest in them

DISCUSSION

Summary

The retailerrsquos security interest in the bicycles was perfected even though no financing statement was filed because it was a purchase-money security interest in consumer goods A purchase-money security interest in consumer goods is automatically perfected upon attachment

The buyer is not subject to the retailerrsquos security interest in the bicycle that the buyer bought from the man Because the bicycle was consumer goods in the hands of the man and the retailer never filed a financing statement covering the bicycle the retailerrsquos security interest is not effective against someone like the buyer who bought the bicycle for value without knowledge of the retailerrsquos security interest and for personal use

On the other hand the retailerrsquos security interest continues in the bicycle given to the friend because the friend did not give value for the bicycle or buy it in the ordinary course of business

Point One (35) The retailerrsquos security interest in the bicycles attached on June 1 Because this interest was a purchase-money security interest in consumer goods it was automatically perfected when it attached

The retailerrsquos security interest in the bicycles attached on June 1 when the man bought the bicycles (acquiring rights in the collateral) signed a security agreement containing a description of the collateral and received value from the retailer (by being given credit with which to purchase the bicycles) UCC sect 9-203(a) amp (b)

Despite the retailerrsquos failure to file a financing statement its security interest was perfected Pursuant to UCC sect 9-309(1) a security interest is automatically perfected upon attachment if the goods are ldquoconsumer goodsrdquo and the security interest is a ldquopurchase-money security interestrdquo

In this case the bicycles sold by the retailer to the man were consumer goods at the time of sale The bicycles were ldquogoodsrdquo because they were ldquomovable when a security interest

24

Secured Transactions Analysis

attachesrdquo UCC sect 9-102(a)(44) They were also consumer goods because they were ldquobought for use primarily for personal family or household purposesrdquo UCC sect 9-102(a)(23) The retailerrsquos security interest in these consumer goods was also a ldquopurchase-money security interestrdquo A purchase-money security interest is an interest that secures a debt that was incurred in order to ldquoenable the debtor to acquire rights in or the use of the collateralrdquo UCC sect 9-103(a) (b)(1) Here the man incurred an obligation to the retailer to purchase the bicycles so the security interest he gave the retailer to secure that obligation was a purchase-money security interest

Because the retailerrsquos security interest was a purchase-money security interest in consumer goods it was automatically perfected on June 1 when the interest attached to the bicycles

Point Two (35) The buyer took the bicycle free of the retailerrsquos security interest because (i) the retailer did not file a financing statement covering the bicycle (ii) the bicycle was ldquoconsumer goodsrdquo and (iii) the buyer bought the bicycle for value without knowledge of the retailerrsquos security interest and for personal use

A security interest continues in collateral even after a sale or other disposition of that collateral unless the creditor authorized the disposition ldquofree of the security interestrdquo or another Article 9 exception applies UCC sectsect 9-201(a) and 9-315(a)(1)

However a buyer of goods like the buyer here can take free of a prior security interest in those goods under certain circumstances See UCC sectsect 9-317(b) (buyers who give value and receive delivery of goods without knowledge of an unperfected security interest in the goods) and 9-320(a) amp (b) (buyer in ordinary course of business buyer of consumer goods in a consumer-to-consumer transaction who gives value) In this case the retailerrsquos security interest was perfected when the buyer purchased the bicycle so UCC sect 9-317(b) does not protect the buyer The buyer also is not a protected ldquobuyer in ordinary course of businessrdquo because he did not purchase from a person who is in the business of selling bicycles See UCC sect 1-201(b)(9)

The buyer can however qualify for the protection of UCC sect 9-320(b) That section provides that a buyer of goods from a person who used them for personal family or household purposes takes free of a perfected security interest in the goods if (1) the buyer had no knowledge of the security interest (2) the buyer gave value for the goods (3) the buyer purchased the goods primarily for personal family or household purposes and (4) the purchase occurred before the filing of a financing statement covering the goods

The buyer met all of these criteria The man used the bicycle for personal purposes The buyer purchased the bicycle from the man and the buyer had no knowledge of the retailerrsquos security interest The buyer gave value ($400) for the bicycle and he bought it ldquoprimarily for personal family or household purposesrdquo as he planned to use it for recreation which is a personal rather than a business use Finally no financing statement had been filed Therefore under UCC sect 9-320(b) the buyer took free of the retailerrsquos security interest

Point Three (30) The retailerrsquos security interest continues in the bicycle that the man gave to the friend Thus the retailer can recover the bicycle from the friend because the friend did not give value for the bicycle or buy it in the ordinary course of business

25

Secured Transactions Analysis

As noted in Point Two the retailer did not authorize the man to dispose of the bicycle Consequently the retailerrsquos security interest continued in the bicycle even after the man transferred ownership of the bicycle to the friend See UCC sectsect 9-201(a) and 9-315(a)(1) The retailerrsquos security interest in the bicycle will be effective against the friend unless some other provision of Article 9 allows the friend to take the bicycle free of that security interest

Unfortunately for the friend there is no Article 9 provision that allows him to take free of the retailerrsquos interest The friendrsquos basic problem is that he is not a buyer of the bicyclemdashhe received the bicycle as a gift and did not give value for it Thus the friend is not protected by any of the applicable exceptions See UCC sectsect 9-317(b) (protecting buyers who give value for goods subject to an unperfected security interest) 9-320(a) (protecting buyers in ordinary course of business) and 9-320(b) (protecting buyers of consumer goods who give value)

In short the retailerrsquos security interest continues in the bicycle that the man gave to the friend The friend took the bicycle subject to that security interest

26

FEDERAL CIVIL PROCEDURE ANALYSIS (Federal Civil Procedure VIE)

ANALYSIS

Legal Problems

(1) Does a judgment in a prior action preclude a nonparty from suing the same defendant on a closely related claim when the nonparty and the original plaintiff are in a family relationship

(2) Does a judgment rendered in an earlier action preclude a nonparty from litigating an issue that was actually decided in the first suit

(3) May a nonparty to an earlier action invoke the judgment in that action to preclude a party to the prior action from relitigating an issue that the party had a full and fair opportunity to litigate in the earlier action

DISCUSSION

Summary

Pursuant to the doctrines of claim preclusion (res judicata) and issue preclusion (collateral estoppel) a judgment is binding on the parties thereto In the absence of privity nonparties to a prior suit cannot be bound by a judgment rendered in their absence Thus in the absence of privity a nonparty to the first suit is not precluded from presenting her claim in a second suit even if it is factually related to the claims and defenses presented in the first suit nor is she bound by determinations of issues made in the first suit A family relationship without more does not support a finding of privity For this reason Mother as a nonparty is not bound by the judgment in the Son-Driver action She may bring her separate claim for damage to her car and she is not precluded from litigating the question of whether she was negligent in the maintenance of her car

Driver on the other hand could be precluded from relitigating the issue of her negligence pursuant to the doctrine of non-mutual issue preclusion (also called non-mutual offensive collateral estoppel) which allows a nonparty to a prior action to invoke issue preclusion to prevent a party to that prior action from relitigating determinations of issues made therein However Mother may be prevented from invoking non-mutual collateral estoppel in this case because she could easily have joined her claim in the prior action but did not do so

[NOTE Federal common law governs the preclusive effect of a judgment rendered by a federal court sitting in diversity See Semtek Intrsquol Inc v Lockheed Martin Corp 531 US 497 508 (2001) But the Semtek Court concluded that federal common law in this context incorporates the preclusion law of the state in which the rendering federal court sits (unless the state law is incompatible with federal interests) id at 508ndash09 Thus State Arsquos preclusion law determines the preclusive effect of the judgment rendered in Sonrsquos suit against Driver The problem says that State A preclusion law is identical to federal preclusion law so the following analysis utilizes general principles of preclusion drawn from Supreme Court case law (announcing federal preclusion rules) and the Restatement (Second) of Judgments]

27

Federal Civil Procedure Analysis

Point One (35) Under the doctrine of claim preclusion the judgment rendered in the first action does not preclude Mother a nonparty from suing Driver for the damage to her car because the judgment binds only parties or those in privity with them and Mother and Son are not in privity

Driver may contend that the doctrine of claim preclusion (res judicata) precludes Mother from presenting a claim arising from the same nucleus of facts that was presented in the first action brought by Son According to the doctrine of claim preclusion ldquowhen a court of competent jurisdiction has entered a final judgment on the merits of a cause of action the parties to the suit and their privies are thereafter bound lsquonot only as to every matter which was offered and received to sustain or defeat the claim or demand but as to any other admissible matter which might have been offered for that purposersquordquo Commissioner of Internal Revenue v Sunnen 333 US 591 597 (1948) (citation omitted)

However the doctrine of claim preclusion does not apply to Mother on the facts of this problem First Mother was not a party to the earlier case ldquoIt is a principle of general application in Anglo-American jurisprudence that one is not bound by a judgment in personam in a litigation in which he is not designated as a party or to which he has not been made a party by service of processrdquo Taylor v Sturgell 553 US 880 884 (2008) (citing Hansberry v Lee 311 US 32 40 (1940)) see also RESTATEMENT (SECOND) OF JUDGMENTS sect 34(3) (1982) This rule reflects our ldquodeep-rooted historic tradition that everyone should have his own day in courtrdquo Martin v Wilks 490 US 755 762 (1989) (citation omitted) (superseded by statute on other grounds) Since Mother was not a party to the first suit she is not bound by the judgment unless an exception to the general rule applies

Mother might be bound by the prior judgment if she were considered to have been sufficiently in privity with Son that Son represented her interests in that action ldquoA person who is not a party to an action but who is represented by a party is bound by and entitled to the benefits of a judgment as though he were a partyrdquo RESTATEMENT (SECOND) OF JUDGMENTS sect 41(1) But there is no suggestion in the facts of the problem that Son who is an adult purported to represent Motherrsquos interests in the first suit ldquo[C]lose family relationships are not sufficient by themselves to establish privity with the original suitrsquos party or to bind a nonparty to that suit by the judgment entered therein rdquo Cuauhtli v Chase Home Finance LLC 308 Fed Appx 772 773 (5th Cir 2009) (citation omitted) accord 18A CHARLES ALAN WRIGHT ET AL FEDERAL

PRACTICE AND PROCEDURE sect 4459 (2d ed 2002) In Taylor v Sturgell supra the Supreme Court identified other special circumstances in

which nonparties may be bound by a prior judgmentmdashwhen a nonparty consents to be bound when a nonparty is in a pre-existing substantive legal relationship with a party (such as preceding and succeeding property owners) when a nonparty assumed control of the prior litigation when a party seeks to relitigate through a proxy or where a special statutory scheme seeks to foreclose successive litigation by nonparties See Taylor 553 US at 893ndash95 None of these circumstances exists here

Because Mother was not a party to the first suit and is not in privity with Son who is an adult the judgment in the first action does not preclude her from bringing her own claim against Driver

Point Two (35) Under the doctrine of issue preclusion the judgment rendered in the first action does not preclude Mother a nonparty from litigating the issue of her negligence in maintaining her carrsquos

28

Federal Civil Procedure Analysis

brake lights because the judgment binds only parties or those in privity with them and Mother and Son are not in privity

By its affirmative response to a special interrogatory the jury in the first action expressly concluded that ldquoMother negligently failed to ensure that the brake lights on her car were in proper working orderrdquo Driver may attempt to invoke the doctrine of issue preclusion to preclude Mother from relitigating this issue in the second action

[I]ssue preclusion arises in a second action on the basis of a prior decision when the same lsquoissuersquo is involved in both actions the issue was lsquoactually litigatedrsquo in the first action after a full and fair opportunity for litigation the issue was lsquoactually decidedrsquo in the first action by a disposition that is sufficiently lsquofinalrsquo lsquoon the meritsrsquo and lsquovalidrsquo it was necessary to decide the issue in disposing of the first action and the later litigation is between the same parties or involves nonparties that are subject to the binding effect or benefit of the first action Once these requirements are met issue preclusion is available not only to defend against a demand for relief but also as offensive support for a demand for relief Issue preclusion moreover is available whether or not the second action involves a new claim or cause of action

18 CHARLES ALAN WRIGHT ET AL FEDERAL PRACTICE AND PROCEDURE sect 4416 at 392ndash93 (2d ed) see also RESTATEMENT (SECOND) OF JUDGMENTS sect 27 (1982)

Here several of the elements necessary for issue preclusion are present The same issue is involved in both actionsmdashthe issue of Motherrsquos negligence in failing to maintain the brake lights on her car That issue was actually litigated in the first action and decided by the jury There is nothing to suggest anything less than a full and fair opportunity to litigate The judgment disposing of the issue was final

Nevertheless the judgment will not preclude Mother from relitigating the issue for two reasons First Mother was not a party to the first action and as explained above Mother and Son are not in privity Therefore she cannot be denied an opportunity to litigate the issue of her negligence Second it does not appear that the juryrsquos decision as to Motherrsquos negligence was necessary to the prior judgment against Driver Nothing suggests that the finding on Motherrsquos negligence had any bearing on the outcome of the first action

Point Three (30) Under the doctrine of non-mutual issue preclusion the judgment rendered in the first action might preclude Driver from relitigating the issue of her negligence However Driver has a strong argument that such a result would be inconsistent with the policy against offensive use of non-mutual estoppel when the non-party plaintiff easily could have joined as a plaintiff in the first action

Because Son already convinced the jury in the first action that ldquoDriver was negligent in the operation of her vehiclerdquo Mother may wish to invoke the doctrine of non-mutual issue preclusion to prevent Driver from relitigating the question of her negligence As noted above ldquoissue preclusion arises in a second action on the basis of a prior decision when the same lsquoissuersquo is involved in both actions the issue was lsquoactually litigatedrsquo in the first action after a full and fair opportunity for litigation the issue was lsquoactually decidedrsquo in the first action by a disposition that is sufficiently lsquofinalrsquo lsquoon the meritsrsquo and lsquovalidrsquo it was necessary to decide the issue in disposing of the first action rdquo 18 CHARLES ALAN WRIGHT ET AL FEDERAL PRACTICE AND

PROCEDURE sect 4416 at 392 (2d ed) see also RESTATEMENT (SECOND) OF JUDGMENTS sect 27

29

Federal Civil Procedure Analysis

Here these basic requirements for issue preclusion are met First the same issue is involved in both suits whether Driver was negligent in the operation of her car Second this issue was actually litigated and decided in the first action the jury answered a special interrogatory raising this very question There is nothing to suggest that Driver lacked a full and fair opportunity to litigate the issue Since a judgment was rendered against Driver for the injuries Son sustained as a result of Driverrsquos negligence resolution of the issue was necessary to dispose of the first action Driver was a party to the first action so she may be bound by the judgment

[NOTE Traditionally issue preclusion required mutualitymdashboth the party asserting issue preclusion and the party against whom issue preclusion was asserted were bound by the prior judgment Under the traditional mutuality rule Mother could not assert issue preclusion against Driver because Mother would not be bound by the judgment if Driver sought to rely on it See Point One There is no mutuality between Mother and Driver with respect to the prior judgment

This traditional mutuality requirement has been abandoned in most jurisdictions The Supreme Court rejected a strict mutuality requirement in Blonder-Tongue Laboratories Inc v University of Illinois Foundation 402 US 313 (1971) (non-mutual defensive collateral estoppel used by a defendant to preclude a plaintiff from relitigating a claim the plaintiff previously litigated) and Parklane Hosiery Co v Shore 439 US 322 (1979) (non-mutual offensive collateral estoppel used by a plaintiff to preclude a defendant from relitigating a claim the defendant previously litigated) In Parklane Hosiery the Court concluded (as a matter of federal preclusion law) that trial courts should have ldquobroad discretionrdquo to determine whether or not to permit a plaintiff to invoke non-mutual issue preclusion ldquoThe general rule should be that in cases where a plaintiff could easily have joined in the earlier action or where the application of offensive estoppel would be unfair to a defendant a trial judge should not allow the use of offensive collateral estoppelrdquo Id at 331

The Parklane Hosiery decision identified a number of circumstances that might make it unfair to allow a plaintiff to invoke non-mutual issue preclusion (non-mutual offensive collateral estoppel in the traditional terminology) against a defendant In particular the Parklane Hosiery court suggested that issue preclusion may not be appropriate if the plaintiff in the second action ldquocould easily have joined in the earlier actionrdquo Id Prohibiting plaintiffs from using non-mutual estoppel under such circumstances would promote judicial efficiency by encouraging plaintiffs to join the prior action It would also discourage plaintiffs from staying out of prior litigation in order to secure in effect two bites at the apple using the prior litigation offensively if the defendant loses and forcing the defendant to litigate a second time if the defendant wins the prior action

An exceptional exam answer might therefore argue that non-mutual issue preclusion should be denied on these facts Son and Mother both reside in State A since they are related they know each other well and Son was driving Motherrsquos car when the accident occurred They could have sued together and Rule 20 of the Federal Rules of Civil Procedure would have authorized joinder of their claims because those claims arose from the same transaction or occurrence and raised a common question of law or fact FED R CIV P 20(a) The facts do not suggest that Mother had any reason not to join Sonrsquos suit other than a desire to see how Sonrsquos action concluded before bringing her own claim Cf Nations v Sun Oil Co (Del) 695 F2d 933 938 (5th Cir 1983) (concluding that plaintiff ldquowas entitled to await the development of his injuries and their predictable consequencesrdquo) Because it appears that Mother may be a ldquowait-and-seerdquo plaintiff who could easily have joined the original action a trial court might disallow as a matter of discretion her use of non-mutual issue preclusion]

30

AGENCY ANALYSIS __________ (Agency I II)

ANALYSIS

Legal Problems

(1) Is the principal or the agent or both liable on contracts with a third party when the principal is an ldquoundisclosed principalrdquo

(2) Is the principal or the agent or both liable on contracts with a third party when the principal is ldquopartially disclosedrdquo or an ldquounidentified principalrdquo

(3) Is the principal or the agent or both liable on contracts with a third party for the purchase of goods when the agent exceeded his authority but the principal nonetheless accepts the goods

DISCUSSION

Summary

The agent but not the owner is liable to the basket manufacturer because the owner is an undisclosed principal and the agent acted without actual or apparent authority Both the agent and the owner however are liable on the burner contract because the owner is an unidentified principal and the agent had apparent authority to enter into that contract With respect to the solar cells contract whether the owner is liable depends upon whether a court would follow the Second or Third Restatement of Agency which take different positions on the effect of the ratification of a contract by an undisclosed principal Under either the agent would also be liable on the contract as he was a party to the contract

[NOTE The contracts that are the subject of this question are contracts for the sale of goods and therefore are governed by Article 2 of the Uniform Commercial Code Article 2 however does not contain agency rules Accordingly common law concepts of agency are applicable UCC sect 1-103(b)]

Point One (35) The agent but not the owner is liable to the basket manufacturer The agent had no actual authority to enter into the contract to buy aluminum baskets and because the owner was an undisclosed principal the manufacturer had no reason to believe that the agent had apparent authority Furthermore the manufacturer had no reason to believe that the agent was not contracting for his own benefit

An agent acting on behalf of a principal can bind the principal to contracts if the agent has either actual or apparent authority An agent has actual authority when contracting on behalf of his principal if he ldquoreasonably believes in accordance with the principalrsquos manifestations to the agent that the principal wishes the agent so to actrdquo RESTATEMENT (THIRD) OF AGENCY sect 201 (2006) Here the agent was told to buy only wicker baskets not aluminum baskets Thus when he contracted with the basket manufacturer to buy aluminum baskets he had no actual authority to do so

31

Agency Analysis

An agent acts with apparent authority ldquowhen a third party [with whom the agent acts] reasonably believes the actor has authority to act on behalf of the principal and that belief is traceable to the principalrsquos manifestationsrdquo Id sect 203 Here the owner notified basket manufacturers that she or her agent might contact them to purchase baskets but that notification did not specifically name the agent or any other person as the ownerrsquos agent Furthermore the basket manufacturer had no prior dealings with the agent or the owner or any reason to think that the agent was acting for the benefit of anyone but himself Thus there is no basis to conclude that the basket manufacturer thought that the agent had apparent authority to act for the owner

Generally when an agent acts on behalf of an undisclosed principal and the agent lacks authority to enter into the contract the agent is liable on the contract as a party to the contract but the principal is not liable This rule is consistent with the third partyrsquos expectations ldquoThe third party expected the agent to be a party to the contract because the agent presented the deal as if he were acting for himself Moreover if the third party is unaware of the principalrsquos existence the third party must be relying on the agentrsquos solvency and reliability when entering into the contractrdquo See ROBERT W HAMILTON JONATHAN R MACEY amp DOUGLAS K MOLL CORPORATIONS INCLUDING PARTNERSHIPS AND LIMITED LIABILITY COMPANIES 34 (11th ed 2010) See also RESTATEMENT (THIRD) OF AGENCY sect 603 cmt c Furthermore because the third party has no idea that the agent is acting or is seemingly acting on behalf of another there is no reason to believe that the third party would be expecting an undisclosed principal to be liable on the contract Id

Point Two (35) Because the owner is an unidentified (as opposed to undisclosed) principal both she and the agent (as a party to the contract) probably are liable on the contract with the burner manufacturer

When the agent contracted with the burner manufacturer he did not have actual authority to do so as the owner had expressly restricted the agentrsquos authority to purchase only burners with ldquowhisper technologyrdquo See Point One However the agent may have had apparent authority to buy burners without whisper technology

An agent acts with apparent authority ldquowhen a third party [with whom the agent acts] reasonably believes the actor has authority to act on behalf of the principal and that belief is traceable to the principalrsquos manifestationsrdquo RESTATEMENT (THIRD) OF AGENCY sect 203 (2006) The owner indicated that an agent might contact the burner manufacturer The notice contained no restriction regarding the type of burners that the agent was authorized to purchase The facts indicate that burner manufacturers regularly receive such notices

Although the agent told the burner manufacturer that he represented a well-known hot-air balloon operator he did not disclose the ownerrsquos name Thus the owner was a partially disclosed or unidentified principal See RESTATEMENT (SECOND) OF AGENCY sect 4(2) (1958) (using term ldquopartially disclosed principalrdquo) RESTATEMENT (THIRD) OF AGENCY sect 104(2)(c) (2006) (using term ldquounidentified principalrdquo) An agent for a partially disclosed principal may have apparent authority RESTATEMENT (SECOND) OF AGENCY sect 159 cmt e (1958) Based upon (1) the notice sent by the owner (2) the agentrsquos revelation that he was acting as an agent and (3) the fact that burner manufacturers regularly receive such notices and sell to agents the manufacturer may argue that it reasonably and actually believed that the agent was authorized to purchase burners without whisper technology The manufacturer may also argue that because the agent revealed that he was an agent his listing of the ownerrsquos address as the delivery address connects the agent to the notice given by the owner Arguably this distinguishes the burner contract from the basket

32

Agency Analysis

contract Here there is a strong case to support the conclusion that the agent had apparent authority if he did then the owner is liable to the burner manufacturer

The agent also is liable as a party to the contract because he did not fully disclose his agency relationship Although he told the burner manufacturer that he represented a well-known hot-air balloon operator he did not disclose the ownerrsquos name Generally even an authorized agent of a partially disclosed or unidentified principal is liable as a party to a contract with a third person RESTATEMENT (SECOND) OF AGENCY sect 321 (1958) (ldquounless otherwise agreedrdquo) RESTATEMENT (THIRD) OF AGENCY sect 602(2) (2006) (ldquounless the agent and the third party agree otherwiserdquo)

Point Three (30) Under the Second Restatement of Agency the owner is not liable on the contract for solar cells because the agent did not have actual or apparent authority and the owner as an undisclosed principal cannot ratify the contract Under the Third Restatement the owner could be liable as she ratified the contract Under either Restatement the agent is liable as a party to the contract

The owner is not liable to the solar cell manufacturer for breach of the contract for the solar cells because the agent had no actual or apparent authority to purchase solar cells on the ownerrsquos behalf and the owner under the Second Restatement of Agency did not ratify the contract with knowledge of the material facts Thus she is not liable as a ratifier of the contract

The facts state that the agent had authority to purchase only propane fuel tanks In addition he had no apparent authority to purchase solar cells The owner made no manifestations to the solar cell manufacturer that would lead a reasonable person in the manufacturerrsquos position to believe that the agent had the authority to bind the owner to a contract to purchase solar cells In fact the agent made no manifestations at all to the solar cell manufacturer Unlike with the basket manufacturer and the burner manufacturer the owner did not notify the manufacturer of solar cells that an agent might contact it to purchase solar cells In addition the solar cells were delivered to the agent and not to the ownerrsquos address In sum the manufacturer was unaware of any relationship between the owner and the agent As to the solar cell manufacturer the owner is an undisclosed principal There can be no apparent authority in the case of an undisclosed principal because there are no manifestations from the principal to the third person See RESTATEMENT (SECOND) OF AGENCY sect 8 cmt a (1958) (ldquothere can be no apparent authority created by an undisclosed principalrdquo) RESTATEMENT (THIRD) OF AGENCY sect 203 cmt f (2006) (ldquoapparent authority is not present when a third party believes that an interaction is with an actor who is a principalrdquo)

The owner also did not ratify the contract Although the owner used the solar cells generally a principal cannot ratify an unauthorized transaction with a third person ldquounless the one acting purported to be acting for the ratifierrdquo RESTATEMENT (SECOND) OF AGENCY sect 85(1) (1958)

The result differs under the Third Restatement which expressly rejects the Second Restatement on this issue The Restatement (Third) of Agency sect 403 (2006) states ldquoA person may ratify an act if the actor acted or purported to act as an agent on the personrsquos behalfrdquo According to comment b ldquoan undisclosed principal may ratify an agentrsquos unauthorized actrdquo Under the Restatement (Third) of Agency rule the owner probably ratified the transaction The agent clearly acted on the ownerrsquos behalf and in addition the ownerrsquos conduct in using the solar cells ldquojustifies a reasonable assumption that [she] is manifesting assent that the act shall affect [her] legal relationsrdquo See id sect 401(2)

33

Agency Analysis

The agent also is liable to the solar cell manufacturer for breach of the contract for the solar cells because he is a party to the contract The facts indicate that the agent never told the solar cell manufacturer that he represented the owner or any other principal Consequently even if the agent were authorized (which as discussed above he is not) he would be liable as a party to the contract See RESTATEMENT (SECOND) OF AGENCY sect 322 (1958) RESTATEMENT (THIRD) OF AGENCY sect 603(2) (2006) Here he has no authority or apparent authority and is liable as a party to the contract

The agent would also be liable under the Third Restatement Under Restatement (Third) of Agency sect 402(1) (2006) ratification generally relates back and the transaction is treated as if it were authorized at the time of the transaction However this does not relieve the agent of an undisclosed principal who ratifies an unauthorized transaction of liability under the ratified contract See id sect 603(2) (authorized agent for undisclosed principal is a party to the contract) and sect 403 cmt b (ldquoAn undisclosed principalrsquos ratification does not eliminate the agentrsquos liability to the third party on the transaction rdquo)

[NOTE An examinee may discuss the concept of inherent agency power This concept is recognized by the Restatement (Second) of Agency sect 8 A (1958) but the concept is not used in the Restatement (Third) of Agency (2006) Here there are no facts to support that the agent had inherent authority

As to contracts with agents for partially disclosed principals (eg the contract for the burners) the basic question is whether the acts done ldquousually accompany or are incidental to transactions which the agent is authorized to conductrdquo RESTATEMENT (SECOND) OF AGENCY

sect 161 (1958) If so the principal is bound if the other party ldquoreasonably believes that the agent is authorized to do them and has no notice that he is not so authorizedrdquo Id The purchase of burners without whisper technology was not authorized nor was it incidental to an authorized transaction Therefore there should not be inherent agency power

As to contracts on behalf of undisclosed principals (eg the other two contracts) the basic question is whether the acts done are usual or necessary in the transactions the agent is authorized to transact RESTATEMENT (SECOND) OF AGENCY sect 194 (1958) The other two contracts seem fundamentally different from the authorized transactions Therefore there should not be inherent agency power

Only minimal credit should be given for discussion of inherent agency power]

34

EVIDENCE ANALYSIS _____ (Evidence IIA VA B E F J K)

ANALYSIS

Legal Problems

(1) Is the authenticated copy of the mechanicrsquos text message relevant and admissible

(2) Is the womanrsquos question ldquoIs my scooter safe to drive for a whilerdquo relevant and admissible

(3) Is the womanrsquos testimony describing the mechanicrsquos thumbs-up relevant and admissible

DISCUSSION

Summary

The mechanicrsquos text message to the woman is relevant to whether (1) the woman lost control of the scooter due to its defective brakes (2) the woman knew that the brakes needed repair and (3) it was negligent for the woman to drive the scooter knowing that its brakes needed repair

The mechanicrsquos text message is hearsay if it is offered by the pedestrian to prove that the scooterrsquos brakes needed repair However it fits the hearsay exception for present sense impressions and probably also fits the exception for business records The mechanicrsquos text message is not hearsay if it is instead offered by the pedestrian to prove the womanrsquos state of mind (ie that she had notice that her brakes needed repair)

The womanrsquos question to the mechanic and his response are also relevant to whether the brakes caused the accident and whether the woman was negligent The question is not hearsay because the woman did not make an assertion

The mechanicrsquos thumbs-up response is nonverbal conduct intended by the mechanic as an assertion and is therefore an out-of-court statement If the woman offers the mechanicrsquos statement to prove that the scooter was actually safe to ride the womanrsquos testimony about the statement is hearsay

However the mechanicrsquos statement is not hearsay if it is offered by the woman to prove her state of mind Therefore the womanrsquos question and the mechanicrsquos response are admissible to prove the womanrsquos state of mind

Point One(a) (20) The mechanicrsquos text message to the woman should be admitted because it is relevant

Evidence is relevant if it has ldquoany tendency to make a fact more or less probable than it would be without the evidencerdquo FED R EVID 401 ldquoRelevant evidence is admissiblerdquo unless it is inadmissible pursuant to some other rule FED R EVID 402

The mechanicrsquos text message to the woman ldquoWhen you pick up your scooter you need to schedule a follow-up brake repair Wersquoll order the partsrdquo is relevant for two reasons First this evidence has some tendency to make it more probable that the brakes malfunctioned and

35

Evidence Analysis

caused the accident Second it has some tendency to make it more probable that the woman was negligent in riding her scooter after being told by the mechanic that it required further repair

Point One(b) (30) The mechanicrsquos text message fits either the hearsay exception for present sense impressions or the exception for business records or it is admissible non-hearsay

The mechanicrsquos text message is a statement under Rule 801(a) because it is ldquoa written assertionrdquo FED R EVID 801(a) The text message is hearsay if the pedestrian offers it to prove the ldquotruth of the matter asserted in the statementrdquo (ie that the scooterrsquos brakes required repair) which resulted in the woman losing control of the scooter and causing the accident FED R EVID 801(c)

However the mechanicrsquos text message fits the hearsay exception for ldquopresent sense impressionsrdquo under Rule 803(1) because it is ldquo[a] statement describing or explaining an event or condition made while or immediately after the declarant perceived itrdquo FED R EVID 803(1) Here the mechanicrsquos text message described the condition of the scooter immediately after he perceived it during the maintenance service

The mechanic is a person with knowledge of the condition of the scooter so if text messages regarding repairs were made and kept by the mechanic in the ordinary course of business this text message also fits the business records exception Under Rule 803(6) a business record is a record of an act ldquomade at or near the time by someone with knowledgerdquo and ldquothe record was kept in the course of a regularly conducted activity of a businessrdquo and ldquomaking the record was a regular practice of that activityrdquo FED R EVID 803(6)

However the text message is not hearsay if it is instead offered to prove that the woman was negligent because she rode her scooter after the mechanic told her it required repair If offered for this purpose it would not be offered for the truth of the matter asserted in the statement but to show the womanrsquos belief about the condition of the scooter (her state of mind)

Point Two (10) The womanrsquos question to the mechanic should be admitted because it is not hearsay

The womanrsquos question to the mechanic is relevant because along with the mechanicrsquos thumbs-up response (see Point Three) it has some tendency to make it more probable that the woman was not negligent andor that the scooter brakes did not malfunction and cause the accident FED R EVID 401 The womanrsquos question does not raise hearsay concerns because it is not an assertion

Hearsay is defined under Rule 801(a) as ldquoan oral assertion written assertion or nonverbal conductrdquo Although ldquoassertionrdquo is not further defined ldquoa favorite [definition] of writers in the [evidence] field for at least a century and a half [is that] the word simply means to say that something is so eg that an event happened or a condition existedrdquo 2 MCCORMICK ON

EVIDENCE sect 246 (6th ed 2006) Under this definition the womanrsquos question is not hearsay because it is not an assertion

Point Three(a) (20) The mechanicrsquos thumbs-up to the woman is a nonverbal assertion that is relevant and the womanrsquos testimony about that response is admissible

36

Evidence Analysis

Hearsay is defined under Rule 801(c) as a ldquostatementrdquo that is ldquoa personrsquos oral assertion written assertion or nonverbal conduct if the person intended it as an assertionrdquo FED R EVID 801(a) Here when the mechanic responded to the womanrsquos question (ldquoIs my scooter safe to ride for a whilerdquo) with a thumbs-up gesture the facts suggest that he intended his nonverbal conduct as an assertion that in his opinion the scooter was safe to ride

The mechanicrsquos assertion is relevant and admissible to prove that the woman was not negligent because the evidence makes it more probable that at the time of the accident she believed that the scooter was safe to ride despite the fact that the brakes required repair FED R EVID 401 Admission of the womanrsquos description of the mechanicrsquos thumbs-up for this purpose does not raise hearsay concerns because the evidence would not be offered for the truth of the matter asserted but to show the womanrsquos belief about the condition of the scooter (her state of mind)

Point Three(b) (20) The mechanicrsquos thumbs-up is relevant to determine whether the scooterrsquos brakes malfunctioned causing the accident but if offered for this purpose it is also hearsay

The mechanicrsquos nonverbal assertion is relevant to the determination of whether the scooterrsquos brakes malfunctioned causing the accident However if offered to prove the ldquotruth of the matter asserted in the statementrdquo (ie that the scooter was safe to ride for a while) it is hearsay that does not fit any hearsay exception

37

TRUSTS AND FUTURE INTERESTS ANALYSIS ____________________ (Trusts and Future Interests IC1 amp 4 G IIF)

ANALYSIS

Legal Problems

(1)(a) Was the revocable trust amendable

(1)(b) If the trust was amendable must the amendment have been executed in accordance with the state Statute of Wills in order to be valid

(2) If the trust amendment was valid does the amendment apply to the probate estate assets passing to the trust pursuant to Settlorrsquos will

(3) If the trust amendment was valid should the trust property be distributed to University

(4) If the trust amendment was not valid should the trust property be distributed to Settlorrsquos grandchild (her only heir) or held in further trust in accordance with the terms of the original trust instrument

DISCUSSION

Summary

A revocable trust is amendable even if the trust instrument does not expressly grant to the trust settlor a power to amend Both inter vivos trusts and amendments thereto are valid even though not executed in accordance with the requirements applicable to wills

Under the Uniform Testamentary Additions to Trusts Act a revocable trust may be amended at any time prior to the settlorrsquos death and the amendment applies to the disposition of assets conveyed to the trust pursuant to a will even if the will was executed prior to the date of the amendment

At Settlorrsquos death trust assets including probate assets passing to the trust under Settlorrsquos will would go to University if as is the case here the trust amendment was valid If the amendment was invalid the trust assets would continue to be held in further trust because there is no violation of the common law Rule Against Perpetuities

Point One(a) (30) Settlor retained the right to amend the inter vivos trust despite her failure to expressly reserve this power

At issue here is whether a retained power of revocation includes the power to amend sometimes referred to as the power to modify The Restatement (Second) of Trusts sect 331 cmt g provides that if a settlor has a power to revoke that retained power ordinarily includes a power to modify (amend) as well Comment g also notes that the power to amend includes both a power to withdraw trust assets and a power to ldquomodify the terms of the trustrdquo The Uniform Trust Code which provides that a power to revoke includes the power to amend is consistent with this view

38

Trusts and Future Interests Analysis

UNIF TRUST CODE sect 602 accord RESTATEMENT (THIRD) OF TRUSTS sect 63 cmt The theory is that even though a power to amend was not expressly retained by a settlor the goal of amendment assuming the power was not included in the power to revoke could easily be achieved by first revoking the trust and then creating a new trust with the same terms contemplated by the amendment To require this would put form over substance

Thus by expressly retaining the power to revoke the trust Settlor retained a power to amend the inter vivos trust despite her failure to expressly reserve this power

[NOTE Under the common law a trust is irrevocable unless the settlor expressly retains a power to revoke the trust Conversely under the Uniform Trust Code a trust is revocable unless the terms of the trust expressly provide otherwise See UNIF TRUST CODE sect 602 The Trust Codersquos position on revocation follows the minority view in the United States and is inconsistent with prior Restatements of Trusts (see Restatement (Second) of Trusts sect 330) Here the trust is revocable because Settlor expressly retained a power of revocation

The Uniform Trust Code has been adopted in 24 jurisdictions Alabama Arizona Arkansas District of Columbia Florida Kansas Maine Michigan Missouri Nebraska New Hampshire New Mexico North Carolina North Dakota Ohio Oregon Pennsylvania South Carolina Tennessee Utah Vermont Virginia West Virginia and Wyoming]

Point One(b) (10) Settlorrsquos amendment of the trust was valid despite her failure to have her signature to the trust amendment witnessed

Neither the common law nor state statutes require a trust instrument or an amendment to a trust instrument to be executed in accordance with the formalities prescribed for execution of a will Indeed an inter vivos trust that does not involve real estate can be created orally Under the Uniform Trust Code the only requirements for creating a valid inter vivos trust are intent the specification of beneficiaries and the designation of a trustee See UNIF TRUST CODE sect 402 accord RESTATEMENT (THIRD) OF TRUSTS sect 13

Here the amendment meets the requirements of both the Uniform Trust Code and the common law Thus the fact that Settlorrsquos signature was not witnessed when she signed the amendment to the trust does not make the amendment invalid

Point Two (20) Under the Uniform Testamentary Additions to Trusts Act a revocable trust may be amended at any time prior to the settlorrsquos death and the amendment applies to probate assets poured into the trust at the settlorrsquos death pursuant to the settlorrsquos will even when the will was executed prior to the date of the amendment

Historically property owned by an individual at her death passed to the individualrsquos heirs or to beneficiaries designated in a will executed with the formalities (writing signing witnessing) prescribed by state law However when a will devises property to the trustee of an inter vivos trust then the provisions of the trustmdashwhich may not have been executed in accordance with the formalities required for willsmdasheffectively determine who will receive the property Because of this possibility some early cases held that if an inter vivos trust was not executed with the same formalities required for a valid will then the trust was ineffective to dispose of probate assets poured into the trust at the settlorrsquos death pursuant to the settlorrsquos will

This line of cases has been overturned by the Uniform Testamentary Additions to Trusts Act (the Act) now Uniform Probate Code sect 2-511 Under the Act adopted in almost all

39

Trusts and Future Interest Analysis

jurisdictions a testamentary bequest to the trustee of an inter vivos trust established by the testator during his or her lifetime is valid if the trust is in writing it is identified in the testatorrsquos will and the trust instrument was executed before concurrently with or after the execution of the will Id The Act further specifies that such a bequest is valid even if the trust is amendable or revocable and that a later amendment applies to assets passing to the trust by a previously executed will

Thus because the trust amendment is valid its terms apply to assets received by Bank from Settlorrsquos estate

Point Three (10) If the trust amendment was valid then the trust assets including assets passing to the trust under Settlorrsquos will should go to University

Under the trust amendment all trust assets (including the assets of Settlorrsquos probate estate poured into the trust) pass to University The facts provide no basis for failing to comply with Settlorrsquos stated intentions

Point Four (30) If the trust amendment was invalid trust assets including assets received pursuant to Settlorrsquos will should be held in accordance with the terms of the original trust instrument because those terms do not violate the Rule Against Perpetuities

Under the dispositive terms of the original trust instrument Settlor created successive income interests in her surviving children and grandchildren with a remainder interest in her great-grandchildren Because the trust was revocable the period during which the common law Rule Against Perpetuities requires that interests vest (ie 21 years plus lives in being) began to run from the date Settlor no longer had a power of revocation (here her death) not the date on which the trust was created See JESSE DUKEMINIER STANLEY J JOHANSON JAMES LINDGREN amp ROBERT SITKOFF WILLS TRUSTS AND ESTATES 678 (7th ed 2005)

Under the common law Rule Against Perpetuities Settlorrsquos trust is thus valid At the time of Settlorrsquos death she was survived by no children one granddaughter and no great-grandchildren Because Settlor cannot have more children after her death the only income beneficiary of the trust is Settlorrsquos surviving granddaughter This granddaughter is the only person who can produce great-grandchildren of Settlor thus all great-grandchildren must of necessity be born during the lifetime of Settlorrsquos only surviving granddaughter who is a life in being The granddaughterrsquos interest vested at Settlorrsquos death and the great-grandchildrenrsquos interest will vest at the death of the granddaughter There is no need to wait the additional 21 years permitted under the Rule Thus under the common law and the statute given in the facts the nonvested interest in the great-grandchildren is valid

[NOTE Both modern wait-and-see statutes and the Uniform Statutory Rule Against Perpetuities upon which the statute in the facts is modeled provide that before using either reform to validate an otherwise invalid nonvested interest one should first determine if the nonvested interest violates the common law Rule If it does not then there is no need to reform This proposition which is applicable in all MEE user jurisdictions that have not simply abrogated the rule is tested by this problem]

40

NEGOTIABLE INSTRUMENTS ANALYSIS (Negotiable Instruments III IV V)

ANALYSIS

Legal Problems

(1)(a) What rights does a person in possession of a note that has been indorsed in blank by the payee have against the maker of the note

(1)(b) Which defenses may the maker of a note raise against a person entitled to enforce it who is not a holder in due course but is a transferee from a holder in due course

(2) What rights does a person entitled to enforce a note have against an indorser who transferred it for consideration with no warranties

(3) What rights does a person entitled to enforce a note have against a previous holder who transferred it as a gift without indorsing it

DISCUSSION

Summary

The niece is a holder of the note and is thus a person entitled to enforce it The chef the issuer of the note is obligated to pay it to the niece as the person entitled to enforce it The niece is not subject to any defense or claim of the chef relating to the improper repair of the oven because the niece has the rights of a holder in due course When the buyer bought the note from the repairman the buyer became a holder in due course of the note and thus took it free of any personal defenses the chef had against the repairman Even though the niece is not herself a holder in due course of the note the niece succeeded to the buyerrsquos rights as holder in due course and thus took free of the chefrsquos personal defenses

Because the chef refused to pay the note the niece can recover from the repairman on the repairmanrsquos obligation as indorser The niece cannot recover on the note against the buyer however because the buyer did not indorse the note (and thus incurred no indorserrsquos obligation) and the buyer did not receive any consideration for transfer of the note to the niece (and therefore made no transfer warranty)

[NOTE Although Article 9 of the Uniform Commercial Code governs the sale of promissory notes (a point that might be correctly noted by examinees) that Article does not determine the answer to any of the questions posed]

Point One(a) (20) The niece is the holder of the note and thus may enforce it against the chef who is the issuer of the note

The chef is the maker of the note and thus its issuer See UCC sectsect 3-103 3-105 The issuer of a note is obligated to pay it in accordance with its terms to a ldquoperson entitled to enforcerdquo it UCC sect 3-412 The niece is a ldquoperson entitled to enforcerdquo the note This is because the niece is the holder of the note and a holder of a note is a person entitled to enforce it UCC sect 3-301 The niece is the holder of the note because (i) the repairmanrsquos signature on the back of the note not

41

Negotiable Instruments Analysis

accompanied by words indicating a person to whom the note was made payable was a ldquoblank indorsementrdquo which had the effect of making the note a bearer instrument (ii) anyone in possession of a bearer instrument is a holder of it and (iii) the niece is in possession of the note See UCC sectsect 1-201(b)(21)(A) 3-204 and 3-205 Accordingly the chef has an obligation to the niece to pay the note in accordance with its terms and the niece may enforce that obligation

Point One(b) (40) The niece is not a holder in due course of the note but because she is a transferee from the buyer who was a holder in due course she has the same enforcement rights as the buyer Because the buyer as a holder in due course would have been able to enforce the note against the chef without being subject to defenses or claims arising from the improper repair the niece has the same rights and will not be subject to the chefrsquos defenses or claims about the repair

As noted in Point One(a) the chef has an obligation to the niece to pay the note in accordance with its terms However except against a person with the rights of a holder in due course the chef can raise any defenses or claims in recoupment that he would have if the claim on the note were an ordinary contract claim UCC sect 3-305 Thus except against a holder in due course the chef would be able to raise the improper repair as a defense or a claim in recoupment (a claim in response to the niecersquos claim)

But claims in recoupment and most defenses cannot be raised against a person with the rights of a holder in due course Against a holder in due course the chef can raise only the four ldquorealrdquo defenses listed in UCC sect 3-305(a)(1) (infancy duress lack of legal capacity or illegality that nullifies the obligation of the obligor under other law fraud in the factum discharge in insolvency proceedings) none of which is present here

The niece is not a holder in due course because she did not take the note for value See UCC sectsect 3-302(a)(2)(i) (criteria for holder in due course status) and 3-303(a) (definition of ldquovaluerdquo) But this does not mean that the niece is subject to the chefrsquos claim arising out of the improper repair The buyer was a holder in due course of the note because he took the note for value ($9500) in good faith and without notice of any facts that would have alerted him to the chefrsquos defense against the repairman UCC sect 3-302(a)(2) As a holder in due course the buyer owned the note free of the chefrsquos claim because that claim did not constitute a ldquorealrdquo defense UCC sect 3-305(b) When the buyer gave the note to the niece this constituted a ldquotransferrdquo of the note See UCC sect 3-203(a) When a note is transferred the transferee receives ldquoany right of the transferor to enforce the instrument including any right as a holder in due courserdquo UCC sect 3-203(b) Under this rule (also known as the ldquoshelter principlerdquo) the buyer transferred his freedom from the chefrsquos defenses to the niece and the niece can enforce the note free of the chefrsquos defenses

Point Two (20) Because the chef dishonored the note the niece can recover from the repairman on the repairmanrsquos obligation as indorser

The chefrsquos refusal to pay the note constituted dishonor See UCC sect 3-502 The repairman as an indorser of the note (see Point One(a)) incurred the obligations of an indorser under UCC sect 3-415(a) When a note has been dishonored one of the obligations of an indorser is to pay the amount of the note to a person entitled to enforce it Therefore the repairman is liable for the amount of the note to the niece a person entitled to enforce the note (so long as the niece gives proper notice of dishonor to the repairman)

42

Negotiable Instruments Analysis

[NOTE Because the repairman indorsed the note without warranties there are no transfer warranties UCC sect 3-416 cmt 5]

Point Three (20) The niece cannot recover on the note against the buyer as either indorser or warrantor because the buyer did not indorse the note and did not receive consideration for transferring the note to the niece

The buyer did not indorse the note and therefore did not incur the obligation of an indorser to pay the note upon dishonor

The niece cannot recover from the buyer under a transfer warranty theory because transfer warranties are made only by a person ldquowho transfers an instrument for considerationrdquo Here the buyer gave the instrument to the niece as a gift So the buyer made no transfer warranty UCC sect 3-416(a) Therefore the niece cannot recover from the buyer on that theory

43

National Conference of Bar Examiners 302 South Bedford Street | Madison WI 53703-3622 Phone 608-280-8550 | Fax 608-280-8552 | TDD 608-661-1275

wwwncbexorg e-mail contactncbexorg

  • Contents
  • Preface
  • Description of the MEE
  • Instructions
  • February 2013 Questions
    • Real Property Question
    • Contracts Question
    • Constitutional Law Question
    • Secured Transactions Question
    • Federal Civil Procedure Question
    • Agency Question
    • Evidence Question
    • Trusts and Future Interests Question
    • Negotiable Instruments Question
      • February 2013 Analyses
        • Real Property Analysis
        • Contracts Analysis
        • Constitutional Law Analysis
        • Secured Transactions Analysis
        • Federal Civil Procedure Analysis
        • Agency Analysis
        • Evidence Analysis
        • Trusts and Future Interests Analysis
        • Negotiable Instruments Analysis
            • ltlt ASCII85EncodePages false AllowTransparency false AutoPositionEPSFiles true AutoRotatePages None Binding Left CalGrayProfile (Dot Gain 20) CalRGBProfile (sRGB IEC61966-21) CalCMYKProfile (US Web Coated 050SWOP051 v2) sRGBProfile (sRGB IEC61966-21) CannotEmbedFontPolicy Error CompatibilityLevel 14 CompressObjects Tags CompressPages true ConvertImagesToIndexed true PassThroughJPEGImages true CreateJobTicket false DefaultRenderingIntent Default DetectBlends true DetectCurves 00000 ColorConversionStrategy CMYK DoThumbnails false EmbedAllFonts true EmbedOpenType false ParseICCProfilesInComments true EmbedJobOptions true DSCReportingLevel 0 EmitDSCWarnings false EndPage -1 ImageMemory 1048576 LockDistillerParams false MaxSubsetPct 100 Optimize true OPM 1 ParseDSCComments true ParseDSCCommentsForDocInfo true PreserveCopyPage true PreserveDICMYKValues true PreserveEPSInfo true PreserveFlatness true PreserveHalftoneInfo false PreserveOPIComments true PreserveOverprintSettings true StartPage 1 SubsetFonts true TransferFunctionInfo Apply UCRandBGInfo Preserve UsePrologue false ColorSettingsFile () AlwaysEmbed [ true ] NeverEmbed [ true ] AntiAliasColorImages false CropColorImages true ColorImageMinResolution 300 ColorImageMinResolutionPolicy OK DownsampleColorImages true ColorImageDownsampleType Bicubic ColorImageResolution 300 ColorImageDepth -1 ColorImageMinDownsampleDepth 1 ColorImageDownsampleThreshold 150000 EncodeColorImages true ColorImageFilter DCTEncode AutoFilterColorImages true ColorImageAutoFilterStrategy JPEG ColorACSImageDict ltlt QFactor 015 HSamples [1 1 1 1] VSamples [1 1 1 1] gtgt ColorImageDict ltlt QFactor 015 HSamples [1 1 1 1] VSamples [1 1 1 1] gtgt JPEG2000ColorACSImageDict ltlt TileWidth 256 TileHeight 256 Quality 30 gtgt JPEG2000ColorImageDict ltlt TileWidth 256 TileHeight 256 Quality 30 gtgt AntiAliasGrayImages false CropGrayImages true GrayImageMinResolution 300 GrayImageMinResolutionPolicy OK DownsampleGrayImages true GrayImageDownsampleType Bicubic GrayImageResolution 300 GrayImageDepth -1 GrayImageMinDownsampleDepth 2 GrayImageDownsampleThreshold 150000 EncodeGrayImages true GrayImageFilter DCTEncode AutoFilterGrayImages true GrayImageAutoFilterStrategy JPEG GrayACSImageDict ltlt QFactor 015 HSamples [1 1 1 1] VSamples [1 1 1 1] gtgt GrayImageDict ltlt QFactor 015 HSamples [1 1 1 1] VSamples [1 1 1 1] gtgt JPEG2000GrayACSImageDict ltlt TileWidth 256 TileHeight 256 Quality 30 gtgt JPEG2000GrayImageDict ltlt TileWidth 256 TileHeight 256 Quality 30 gtgt AntiAliasMonoImages false CropMonoImages true MonoImageMinResolution 1200 MonoImageMinResolutionPolicy OK DownsampleMonoImages true MonoImageDownsampleType Bicubic MonoImageResolution 1200 MonoImageDepth -1 MonoImageDownsampleThreshold 150000 EncodeMonoImages true MonoImageFilter CCITTFaxEncode MonoImageDict ltlt K -1 gtgt AllowPSXObjects false CheckCompliance [ None ] PDFX1aCheck false PDFX3Check false PDFXCompliantPDFOnly false PDFXNoTrimBoxError true PDFXTrimBoxToMediaBoxOffset [ 000000 000000 000000 000000 ] PDFXSetBleedBoxToMediaBox true PDFXBleedBoxToTrimBoxOffset [ 000000 000000 000000 000000 ] PDFXOutputIntentProfile () PDFXOutputConditionIdentifier () PDFXOutputCondition () PDFXRegistryName () PDFXTrapped False CreateJDFFile false Description ltlt ARA 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 BGR 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 CHS ltFEFF4f7f75288fd94e9b8bbe5b9a521b5efa7684002000410064006f006200650020005000440046002065876863900275284e8e9ad88d2891cf76845370524d53705237300260a853ef4ee54f7f75280020004100630072006f0062006100740020548c002000410064006f00620065002000520065006100640065007200200035002e003000204ee553ca66f49ad87248672c676562535f00521b5efa768400200050004400460020658768633002gt CHT ltFEFF4f7f752890194e9b8a2d7f6e5efa7acb7684002000410064006f006200650020005000440046002065874ef69069752865bc9ad854c18cea76845370524d5370523786557406300260a853ef4ee54f7f75280020004100630072006f0062006100740020548c002000410064006f00620065002000520065006100640065007200200035002e003000204ee553ca66f49ad87248672c4f86958b555f5df25efa7acb76840020005000440046002065874ef63002gt CZE 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 DAN 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 DEU ltFEFF00560065007200770065006e00640065006e0020005300690065002000640069006500730065002000450069006e007300740065006c006c0075006e00670065006e0020007a0075006d002000450072007300740065006c006c0065006e00200076006f006e002000410064006f006200650020005000440046002d0044006f006b0075006d0065006e00740065006e002c00200076006f006e002000640065006e0065006e002000530069006500200068006f006300680077006500720074006900670065002000500072006500700072006500730073002d0044007200750063006b0065002000650072007a0065007500670065006e0020006d00f60063006800740065006e002e002000450072007300740065006c006c007400650020005000440046002d0044006f006b0075006d0065006e007400650020006b00f6006e006e0065006e0020006d006900740020004100630072006f00620061007400200075006e0064002000410064006f00620065002000520065006100640065007200200035002e00300020006f0064006500720020006800f600680065007200200067006500f600660066006e00650074002000770065007200640065006e002egt ESP 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 ETI 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 FRA 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 GRE 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 HEB ltFEFF05D405E905EA05DE05E905D5002005D105D405D205D305E805D505EA002005D005DC05D4002005DB05D305D9002005DC05D905E605D505E8002005DE05E105DE05DB05D9002000410064006F006200650020005000440046002005D405DE05D505EA05D005DE05D905DD002005DC05D405D305E405E105EA002005E705D305DD002D05D305E405D505E1002005D005D905DB05D505EA05D905EA002E002005DE05E105DE05DB05D90020005000440046002005E905E005D505E605E805D5002005E005D905EA05E005D905DD002005DC05E405EA05D905D705D4002005D105D005DE05E605E205D505EA0020004100630072006F006200610074002005D5002D00410064006F00620065002000520065006100640065007200200035002E0030002005D505D205E805E105D005D505EA002005DE05EA05E705D305DE05D505EA002005D905D505EA05E8002E05D005DE05D905DD002005DC002D005000440046002F0058002D0033002C002005E205D905D905E005D5002005D105DE05D305E805D905DA002005DC05DE05E905EA05DE05E9002005E905DC0020004100630072006F006200610074002E002005DE05E105DE05DB05D90020005000440046002005E905E005D505E605E805D5002005E005D905EA05E005D905DD002005DC05E405EA05D905D705D4002005D105D005DE05E605E205D505EA0020004100630072006F006200610074002005D5002D00410064006F00620065002000520065006100640065007200200035002E0030002005D505D205E805E105D005D505EA002005DE05EA05E705D305DE05D505EA002005D905D505EA05E8002Egt HRV (Za stvaranje Adobe PDF dokumenata najpogodnijih za visokokvalitetni ispis prije tiskanja koristite ove postavke Stvoreni PDF dokumenti mogu se otvoriti Acrobat i Adobe Reader 50 i kasnijim verzijama) HUN ltFEFF004b0069007600e1006c00f30020006d0069006e0151007300e9006701710020006e0079006f006d00640061006900200065006c0151006b00e90073007a00ed007401510020006e0079006f006d00740061007400e100730068006f007a0020006c006500670069006e006b00e1006200620020006d0065006700660065006c0065006c0151002000410064006f00620065002000500044004600200064006f006b0075006d0065006e00740075006d006f006b0061007400200065007a0065006b006b0065006c0020006100200062006500e1006c006c00ed007400e10073006f006b006b0061006c0020006b00e90073007a00ed0074006800650074002e0020002000410020006c00e90074007200650068006f007a006f00740074002000500044004600200064006f006b0075006d0065006e00740075006d006f006b00200061007a0020004100630072006f006200610074002000e9007300200061007a002000410064006f00620065002000520065006100640065007200200035002e0030002c0020007600610067007900200061007a002000610074007400f3006c0020006b00e9007301510062006200690020007600650072007a006900f3006b006b0061006c0020006e00790069007400680061007400f3006b0020006d00650067002egt ITA ltFEFF005500740069006c0069007a007a006100720065002000710075006500730074006500200069006d0070006f007300740061007a0069006f006e00690020007000650072002000630072006500610072006500200064006f00630075006d0065006e00740069002000410064006f00620065002000500044004600200070006900f900200061006400610074007400690020006100200075006e00610020007000720065007300740061006d0070006100200064006900200061006c007400610020007100750061006c0069007400e0002e0020004900200064006f00630075006d0065006e007400690020005000440046002000630072006500610074006900200070006f00730073006f006e006f0020006500730073006500720065002000610070006500720074006900200063006f006e0020004100630072006f00620061007400200065002000410064006f00620065002000520065006100640065007200200035002e003000200065002000760065007200730069006f006e006900200073007500630063006500730073006900760065002egt JPN ltFEFF9ad854c18cea306a30d730ea30d730ec30b951fa529b7528002000410064006f0062006500200050004400460020658766f8306e4f5c6210306b4f7f75283057307e305930023053306e8a2d5b9a30674f5c62103055308c305f0020005000440046002030d530a130a430eb306f3001004100630072006f0062006100740020304a30883073002000410064006f00620065002000520065006100640065007200200035002e003000204ee5964d3067958b304f30533068304c3067304d307e305930023053306e8a2d5b9a306b306f30d530a930f330c8306e57cb30818fbc307f304c5fc59808306730593002gt KOR ltFEFFc7740020c124c815c7440020c0acc6a9d558c5ec0020ace0d488c9c80020c2dcd5d80020c778c1c4c5d00020ac00c7a50020c801d569d55c002000410064006f0062006500200050004400460020bb38c11cb97c0020c791c131d569b2c8b2e4002e0020c774b807ac8c0020c791c131b41c00200050004400460020bb38c11cb2940020004100630072006f0062006100740020bc0f002000410064006f00620065002000520065006100640065007200200035002e00300020c774c0c1c5d0c11c0020c5f40020c2180020c788c2b5b2c8b2e4002egt LTH ltFEFF004e006100750064006f006b0069007400650020016100690075006f007300200070006100720061006d006500740072007500730020006e006f0072011700640061006d00690020006b0075007200740069002000410064006f00620065002000500044004600200064006f006b0075006d0065006e007400750073002c0020006b00750072006900650020006c0061006200690061007500730069006100690020007000720069007400610069006b007900740069002000610075006b01610074006f00730020006b006f006b007900620117007300200070006100720065006e006700740069006e00690061006d00200073007000610075007300640069006e0069006d00750069002e0020002000530075006b0075007200740069002000500044004600200064006f006b0075006d0065006e007400610069002000670061006c006900200062016b007400690020006100740069006400610072006f006d00690020004100630072006f006200610074002000690072002000410064006f00620065002000520065006100640065007200200035002e0030002000610072002000760117006c00650073006e0117006d00690073002000760065007200730069006a006f006d00690073002egt LVI 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 NLD (Gebruik deze instellingen om Adobe PDF-documenten te maken die zijn geoptimaliseerd voor prepress-afdrukken van hoge kwaliteit De gemaakte PDF-documenten kunnen worden geopend met Acrobat en Adobe Reader 50 en hoger) NOR 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 POL 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 PTB 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 RUM ltFEFF005500740069006c0069007a00610163006900200061006300650073007400650020007300650074010300720069002000700065006e007400720075002000610020006300720065006100200064006f00630075006d0065006e00740065002000410064006f006200650020005000440046002000610064006500630076006100740065002000700065006e0074007200750020007400690070010300720069007200650061002000700072006500700072006500730073002000640065002000630061006c006900740061007400650020007300750070006500720069006f006100720103002e002000200044006f00630075006d0065006e00740065006c00650020005000440046002000630072006500610074006500200070006f00740020006600690020006400650073006300680069007300650020006300750020004100630072006f006200610074002c002000410064006f00620065002000520065006100640065007200200035002e00300020015f00690020007600650072007300690075006e0069006c006500200075006c0074006500720069006f006100720065002egt RUS 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 SKY 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 SLV 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 SUO 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 SVE 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 TUR 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 UKR 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 ENU (Use these settings to create Adobe PDF documents best suited for high-quality prepress printing Created PDF documents can be opened with Acrobat and Adobe Reader 50 and later) gtgt Namespace [ (Adobe) (Common) (10) ] OtherNamespaces [ ltlt AsReaderSpreads false CropImagesToFrames true ErrorControl WarnAndContinue FlattenerIgnoreSpreadOverrides false IncludeGuidesGrids false IncludeNonPrinting false IncludeSlug false Namespace [ (Adobe) (InDesign) (40) ] OmitPlacedBitmaps false OmitPlacedEPS false OmitPlacedPDF false SimulateOverprint Legacy gtgt ltlt AddBleedMarks false AddColorBars false AddCropMarks false AddPageInfo false AddRegMarks false ConvertColors ConvertToCMYK DestinationProfileName () DestinationProfileSelector DocumentCMYK Downsample16BitImages true FlattenerPreset ltlt PresetSelector MediumResolution gtgt FormElements false GenerateStructure false IncludeBookmarks false IncludeHyperlinks false IncludeInteractive false IncludeLayers false IncludeProfiles false MultimediaHandling UseObjectSettings Namespace [ (Adobe) (CreativeSuite) (20) ] PDFXOutputIntentProfileSelector DocumentCMYK PreserveEditing true UntaggedCMYKHandling LeaveUntagged UntaggedRGBHandling UseDocumentProfile UseDocumentBleed false gtgt ]gtgt setdistillerparamsltlt HWResolution [2400 2400] PageSize [612000 792000]gtgt setpagedevice

Page 9: February 2013 MEE Questions and Analyses

CONSTITUTIONAL LAW QUESTION

AutoCo is a privately owned corporation that manufactures automobiles Ten years ago AutoCo purchased a five-square-mile parcel of unincorporated land in a remote region of the state and built a large automobile assembly plant on the land To attract workers to the remote location of the plant AutoCo built apartment buildings and houses on the land and leased them to its employees AutoCo owns and operates a commercial district with shops and streets open to the general public AutoCo named the area Oakwood and provides security fire protection and sanitation services for Oakwoodrsquos residents AutoCo also built operates and fully funds the only school in the region which it makes available free of charge to the children of its employees

A family recently moved to Oakwood The father and mother work in AutoCorsquos plant rent an apartment from AutoCo and have enrolled their 10-year-old son in Oakwoodrsquos school Every morning the students are required to recite the Pledge of Allegiance while standing and saluting an American flag With the approval of his parents the son has politely but insistently refused to recite the Pledge and salute the flag at the school on the grounds that doing so violates his own political beliefs and the political beliefs of his family As a result of his refusal to say the Pledge the son has been expelled from the school

To protest the schoolrsquos actions the father walked into the commercial district of Oakwood While standing on a street corner he handed out leaflets that contained a short essay critical of the schoolrsquos Pledge of Allegiance policy Some of the passersby who took the leaflets dropped them to the ground An AutoCo security guard saw the litter told the father that Oakwoodrsquos anti-litter rule prohibits leaflet distribution that results in littering and directed him to cease distribution of the leaflets and leave the commercial district When the father did not leave and continued to distribute the leaflets the security guard called the state police which sent officers who arrested the father for trespass

1 Did the sonrsquos expulsion from the school violate the First Amendment as applied through the Fourteenth Amendment Explain

2 Did the fatherrsquos arrest violate the First Amendment as applied through the Fourteenth Amendment Explain

5

SECURED TRANSACTIONS QUESTION

On June 1 a bicycle retailer sold two bicycles to a man for a total purchase price of $1500 The man made a $200 down payment and agreed to pay the balance in one year The man also signed a security agreement that identified the bicycles as collateral for the unpaid purchase price and provided that the man ldquoshall not sell or dispose of the collateral until the balance owed is paid in fullrdquo The retailer never filed a financing statement reflecting this security interest

The man had bought the bicycles for him and his girlfriend to use on vacation However shortly after he bought the bicycles the man and his girlfriend broke up The man has never used the bicycles

On August 1 the man sold one of the bicycles at a garage sale to a buyer who paid the man $400 for the bicycle The buyer bought the bicycle to ride for weekend recreation

On October 1 the man gave the other bicycle to his friend as a birthday present The friend began using the bicycle for morning exercise

Neither the buyer nor the friend had any knowledge of the manrsquos dealings with the retailer

1 Does the buyer own the bicycle free of the retailerrsquos security interest Explain

2 Does the friend own the bicycle free of the retailerrsquos security interest Explain

6

FEDERAL CIVIL PROCEDURE QUESTION _____

Mother and Son who are both adults are citizens and residents of State A Mother owned an expensive luxury car valued in excess of $100000 Son borrowed Motherrsquos car to drive to a store in State A As Son approached a traffic light that had just turned yellow he carefully braked and brought the car to a complete stop Driver who was following immediately behind him failed to stop and rear-ended Motherrsquos car which was damaged beyond repair Son was seriously injured Driver is a citizen of State B

Son sued Driver in the United States District Court for the District of State A alleging that she was negligent in the operation of her vehicle Son sought damages in excess of $75000 for his personal injuries exclusive of costs and interest In her answer Driver alleged that Son was contributorily negligent in the operation of Motherrsquos car She further alleged that the brake lights on Motherrsquos car were burned out and that Motherrsquos negligent failure to properly maintain the car was a contributing cause of the accident

Following a trial on the merits in Sonrsquos case against Driver the jury answered the following special interrogatories

Do you find that Driver was negligent in the operation of her vehicle Yes

Do you find that Son was negligent in the operation of Motherrsquos car No

Do you find that Mother negligently failed to ensure that the brake lights on her car were in proper working order Yes

The judge then entered a judgment in favor of Son against Driver Driver did not appeal

Two months later Mother sued Driver in the United States District Court for the District of State A alleging that Driverrsquos negligence in the operation of her vehicle destroyed Motherrsquos luxury car Mother sought damages in excess of $75000 exclusive of costs and interest

State A follows the same preclusion principles that federal courts follow in federal-question cases

1 Is Motherrsquos claim against Driver barred by the judgment in Son v Driver Explain

2 Does the juryrsquos conclusion in Son v Driver that Mother had negligently failed to maintain the brake lights on her car preclude Mother from litigating that issue in her subsequent suit against Driver Explain

3 Does the juryrsquos conclusion in Son v Driver that Driver was negligent preclude Driver from litigating that issue in the Mother v Driver lawsuit Explain

7

_____

AGENCY QUESTION

Over 5000 individuals in the United States operate hot-air balloon businesses A hot-air balloon has four key components the balloon that holds the heated air the basket that houses the riders the propane burner that heats the air in the balloon and the propane storage tanks

The owner of a hot-air balloon business recently notified several basket and burner manufacturers that she or her agent might be contacting them to purchase baskets or burners The owner did not specifically name any person as her agent Basket and burner manufacturers regularly receive such notices from hot-air balloon operators Such notices typically include no restrictions on the types of baskets or burners agents might purchase for their principals

The owner then retained an agent to acquire baskets burners and fuel tanks from various manufacturers The owner authorized the agent to buy only (a) baskets made of woven wicker (not aluminum) (b) burners that use a unique ldquowhisper technologyrdquo (so as not to scare livestock when the balloon sails over farmland) and (c) propane fuel tanks

The agent then entered into three transactions with manufacturers all of whom had no prior dealings with either the owner or the agent

(1) The agent and a large manufacturer of both wicker and aluminum baskets signed a contract for the purchase of four aluminum baskets for a total cost of $60000 The agent never told the manufacturer that he represented the owner or any other principal The contract listed the agent as the buyer and listed the ownerrsquos address as the delivery address but did not indicate that the address was that of the owner rather than the agent When the baskets were delivered to the owner she learned for the first time that the agent had contracted to buy aluminum not wicker baskets The owner immediately rejected the baskets and returned them to the manufacturer Neither the owner nor the agent has paid the basket manufacturer for them

(2) The agent contacted a burner manufacturer and told him that the agent represented a well-known hot-air balloon operator who wanted to purchase burners The agent did not disclose the ownerrsquos name The agent and the burner manufacturer signed a contract for the purchase of four burners that did not have ldquowhisper technologyrdquo for a total price of $70000 The burner contract like the basket contract listed the ownerrsquos address for delivery but did not disclose whose address it was The burners were delivered to the ownerrsquos business and the owner discovered that the agent had ordered the wrong kind of burners The owner rejected the burners and returned them to the manufacturer Neither the owner nor the agent has paid the burner manufacturer for the burners

(3) The agent contracted with a solar cell manufacturer to make three cells advertised as ldquostrong enough to power all your ballooning needsrdquo The agent did not tell the manufacturer that he was acting on behalf of any other person One week after the cells were delivered to the agent he took them to the owner who installed them and discovered that she could save a lot of money using solar cells instead of propane to power her balloons The owner decided to keep the solar cells but she has not paid the manufacturer for them

8

Agency Question

Assume that the rejection of the baskets and the burners and the failure to pay for the solar cells constitute breach of the relevant contracts

1 Is the owner liable to the basket manufacturer for breach of the contract for the aluminum baskets Is the agent liable Explain

2 Is the owner liable to the burner manufacturer for breach of the contract for the burners Is the agent liable Explain

3 Is the owner liable to the solar cell manufacturer for breach of the contract for the solar cells Is the agent liable Explain (Do not address liability based upon restitution or unjust enrichment)

9

EVIDENCE QUESTION _____

A woman who owns a motorized scooter brought her scooter to a mechanic for routine maintenance service As part of the maintenance service the mechanic inspected the braking system on the scooter As soon as the mechanic finished inspecting and servicing the scooter he sent the woman a text message to her cell phone that read ldquoJust finished your service When you pick up your scooter you need to schedule a follow-up brake repair Wersquoll order the partsrdquo

The woman read the mechanicrsquos text message and returned the next day to pick up her scooter As the woman was wheeling her scooter out of the shop she saw the mechanic working nearby and asked ldquoIs my scooter safe to ride for a whilerdquo The mechanic responded by giving her a thumbs-up The woman waved and rode away on the scooter

One week later while the woman was riding her scooter a pedestrian stepped off the curb into a crosswalk and the woman collided with him causing the pedestrian severe injuries The woman had not had the scooterrsquos brakes repaired before the accident

The pedestrian has sued the woman for damages for his injuries resulting from the accident The pedestrian has alleged that (1) the woman lost control of the scooter due to its defective brakes (2) the woman knew that the brakes needed repair and (3) it was negligent for the woman to ride the scooter knowing that its brakes needed to be repaired

The woman claims that the brakes on the scooter worked perfectly and that the accident happened because the pedestrian stepped into the crosswalk without looking and the woman had no time to stop The woman the pedestrian and the mechanic will testify at the upcoming trial

The pedestrian has proffered an authenticated copy of the mechanicrsquos text message to the woman

The woman plans to testify that she asked the mechanic ldquoIs my scooter safe to ride for a whilerdquo and that he gave her a thumbs-up in response

The evidence rules in this jurisdiction are identical to the Federal Rules of Evidence

Analyze whether each of these items of evidence is relevant and admissible at trial

1 The authenticated copy of the mechanicrsquos text message

2 The womanrsquos testimony that she asked the mechanic ldquoIs my scooter safe to ride for a whilerdquo and

3 The womanrsquos testimony describing the mechanicrsquos thumbs-up

10

TRUSTS AND FUTURE INTERESTS QUESTION

Ten years ago Settlor validly created an inter vivos trust and named Bank as trustee The trust instrument provided that Settlor would receive all of the trust income during her lifetime The trust instrument further provided that

Upon Settlorrsquos death the trust income shall be paid in equal shares to Settlorrsquos surviving children for their lives Upon the death of the last surviving child the trust income shall be paid in equal shares to Settlorrsquos then-living grandchildren for their lives Upon the death of the survivor of Settlorrsquos children and grandchildren the trust corpus shall be distributed in equal shares to Settlorrsquos then-living great-grandchildren

The trust instrument expressly specified that the trust was revocable but it was silent regarding whether Settlor could amend the trust instrument

Immediately after creating the trust Settlor validly executed a will leaving her entire estate to Bank as trustee of her inter vivos trust to ldquohold in accordance with the terms of the trustrdquo

Five years ago Settlor signed an amendment to the inter vivos trust The amendment changed the disposition of the remainder interest specifying that all trust assets ldquoshall be paid upon Settlorrsquos death to Universityrdquo Settlorrsquos signature on this amendment was not witnessed

A state statute provides that any trust interest that violates the common law Rule Against Perpetuities ldquois nonetheless valid if the nonvested interest in the trust actually vests or fails to vest either (a) within 21 years of lives in being at the creation of the nonvested interest or (b) within 90 years of its creationrdquo

Recently Settlor died leaving a probate estate of $200000 She was survived by no children one granddaughter (who would be Settlorrsquos only heir) and no great-grandchildren The granddaughter has consulted your law firm and has raised four questions regarding this trust

1 Was Settlorrsquos amendment of the inter vivos trust valid Explain

2 Assuming that the trust amendment was valid do its provisions apply to Settlorrsquos probate assets Explain

3 Assuming that the trust amendment was valid how should trust assets be distributed Explain

4 Assuming that the trust amendment was invalid how should trust assets be distributed Explain

11

NEGOTIABLE INSTRUMENTS QUESTION

A chef entered into a contract with a repairman pursuant to which the repairman agreed to repair the chefrsquos commercial oven for $10000 The repairman agreed to accept as payment a negotiable promissory note for $10000 payable two months after its issuance

After the repairman worked on the oven the chef gave him a $10000 note as payment for the work As agreed the note was signed by the chef as maker was payable to the order of the repairman was payable in two months and fulfilled all criteria for negotiability

The next day the repairman sold the note to a buyer for $9500 To effectuate the sale the repairman wrote ldquono warrantiesrdquo on the back of the note signed his name immediately below that and handed the note to the buyer The buyer bought the note in good faith and without knowledge of any facts relating to the work that the repairman had performed for the chef

Later the buyer gave the note to his niece as a gift To effectuate the gift the buyer handed the note to the niece but did not indorse it

Shortly thereafter the chef discovered that the repair work had been done improperly and the oven still did not function correctly The chef tried repeatedly to get the repairman to return to correct the repair work but the repairman ignored all the chefrsquos calls

On the notersquos due date the niece contacted the chef and demanded that he pay the amount of the note to her The chef refused and told the niece that he would not pay the note because the repairman did not properly repair the oven

1 What are the niecersquos rights against the chef Explain

2 What are the niecersquos rights against the repairman Explain

3 What are the niecersquos rights against the buyer Explain

12

February 2013 MEE

ANALYSES

Real Property Contracts

Constitutional Law Secured Transactions

Federal Civil Procedure Agency

Evidence Trusts and Future Interests

Negotiable Instruments

REAL PROPERTY ANALYSIS (Real Property ID1a 4 amp 5)

ANALYSIS

Legal Problems

(1) Does the tenant have a defense to the landlordrsquos action for unpaid rent based on constructive eviction

(2) Does the tenant have a defense to the landlordrsquos action for unpaid rent based on the tenantrsquos surrender of the premises

(3) What if anything may the landlord recover from the tenant for the period after the tenant vacated the building

DISCUSSION

Summary

Under the common law the tenant does not have a defense to the landlordrsquos action for unpaid rent based on constructive eviction Constructive eviction is based on the tenant proving that (1) the landlord breached a duty to the tenant (2) the breach caused a loss by the tenant of the substantial use and enjoyment of the premises (3) the tenant gave the landlord adequate notice and opportunity to repair and (4) the tenant vacated the leased premises Here there was no constructive eviction because although the tenant vacated and gave the landlord adequate notice the landlord breached no express or implied duty to the tenant to repair the premises

The tenant does not have a defense based on the landlordrsquos acceptance of his surrender of the premises a landlordrsquos retention of keys does not constitute an acceptance of the tenantrsquos surrender unless the landlord so intended and here the landlordrsquos statements to the tenant at the time of the surrender of the keys do not evidence the intent to accept the tenantrsquos surrender

Under the common law a landlord has no duty to mitigate damages but also cannot sue for rents due in the future Under this approach the landlord can sue only for past-due rents Using this approach on November 1 the landlord could recover all the rent past due (ie rent for September and October) but could not recover for rents due in the future However some courts have authorized recovery for future rent minus the fair market rental value of the premises It is thus possible that the landlord could recover damages equal to the amount of rent due from September 1 to the end of the six-year lease term ($180000) minus the propertyrsquos fair-market rental value over that same period

Point One (45) The tenant was not constructively evicted because the landlord had no duty to repair the commercial premises that were the subject of the lease

The landlord and the tenant entered into a term-of-years lease because the lease specified both a beginning and an ending date HERBERT HOVENKAMP amp SHELDON F KURTZ THE LAW OF

PROPERTY 256 (5th ed 2001) Although a term-of-years lease normally cannot be terminated by the tenant prior to the end of the term a tenant may terminate a term-of-years lease if the tenant

15

Real Property Analysis

is constructively evicted See id at 286ndash88 Typically as here a claim of constructive eviction is made as a defense to a landlordrsquos action for damages or unpaid rent

In order to establish a constructive eviction the tenant must prove that the landlord breached a duty to the tenant such as a duty to repair and that the landlordrsquos breach caused a loss of the substantial use and enjoyment of the premises The tenant must also show that he gave the landlord notice adequate to permit the landlord to meet his duty to the tenant and that the tenant vacated the leased premises Id see also JOHN G SPRANKLING UNDERSTANDING

PROPERTY LAW sect 1704 (2d ed 2007) Under the common law there was no implied duty on the part of a landlord to repair

leased premises such a duty arose only if expressly set forth in the lease SPRANKLING supra sect 1702[B] Here the written lease contained no term requiring the landlord to repair the air-conditioning Even if the conversation created a lease term that the building had air-conditioning that itself should not create a duty for the landlord to repair it

Over the past several decades courts have generally implied a duty to repair in residential leases either as part of a revised constructive eviction doctrine or based on an implied warranty of habitability JOSEPH W SINGER PROPERTY 469ndash70 (3d ed 2010) This shift has been justified based on the economic disparity between the typical landlord and tenant as well as the fact that residential tenants generally lack both the authority to authorize repairs to common areas of a building and the incentive to make repairs that will ultimately benefit the landlord

However courts have been more reluctant to imply a duty to repair in commercial leases a context in which the tenant is often a valuable business and in a better position to assess and make repairs than is the landlord But see eg Davidow v Inwood North Professional Group 747 SW2d 373 (Tex 1988) When courts have implied a duty to repair in a commercial lease it is typically when the repair has been mandated by public authorities and involves work so substantial that it would not ordinarily fall within the tenantrsquos common law repair duty andor the value of the repair would primarily inure to the landlordrsquos reversionary interest See Brown v Green 884 P2d 55 (Cal 1994) Eugene L Grant et al The Tenant as Terminator Constructive Eviction in Modern Commercial Leases 2 THE COMMERCIAL PROPERTY LEASE ch 15 (ABA 1997) Some courts have also permitted constructive eviction claims by commercial tenants of office buildings based on repairs required in common areas of the building See id Echo Consulting Services Inc v North Conway Bank 669 A2d 227 (NH 1995)

Here the tenant is the owner of a valuable manufacturing operation and is the exclusive occupant of the building the repair has not been mandated by public authorities and the repair is not structural To the contrary the repair involves a feature of the building of unusual importance in the tenantrsquos manufacturing operation and the tenant is likely far more knowledgeable than the landlord about the air-conditioning specifications necessary for the manufacture of the tenantrsquos product

Based on these facts it is unlikely that a court will find that the tenant in this case was constructively evicted Although the tenant can show that he gave adequate notice to the landlord of the air-conditioning malfunction and vacated the premises the lease was commercial and it did not contain any promises or covenants by the landlord except a covenant of quiet enjoyment a covenant of quiet enjoyment does not entail any repair obligations

[NOTE An examineersquos conclusion is less important than his or her demonstrated awareness of the elements of constructive eviction and the need to imply a repair duty for such a defense to be viable here Although the implied warranty of habitability is not available to this tenant Texas Minnesota and Massachusetts imply a warranty of suitability in commercial leases in limited circumstances and an examinee might argue that this warranty should apply

16

Real Property Analysis

here If an examinee concludes that this warranty applies he or she should discuss the other requirements for constructive eviction

If the examinee wrongly concludes that the first element for a constructive eviction has been met the examinee will then have to discuss the remaining three elements in order to conclude that the tenant can claim constructive eviction The tenant would have a strong argument that the second elementmdashsubstantial interference with the use and enjoyment of the premisesmdashalso is met As indicated above the landlord was aware that a functioning air-conditioning system was vital to the tenantrsquos manufacturing operations The facts further indicate that the system had failed three times in the past few months The landlord may try to argue that the malfunctions did not substantially interfere with the tenantrsquos use of the premises because the malfunctions caused the temperature to climb above 81 degrees for only a short period of timemdash 3 hours 6 hours and 10 hours respectivelymdashon each occasion The tenant will argue however that the landlord was aware that the tenantrsquos manufacturing operations could tolerate temperatures above 81 degrees for no more than 6 hours The final malfunction exceeded that limit destroying $150000 worth of the tenantrsquos products

The tenant would also have a strong argument that the third element is met notice and opportunity to cure The tenant notified the landlord of the problem immediately upon the systemrsquos first malfunction and did so again when it malfunctioned a second time and then a third time The landlord might argue that there was insufficient time to cure the problem because the system corrected itself within a few hours on the first and second times Although the malfunction lasted more than 10 hours the third time the landlord might argue that the time period was insufficient to get a repair person on the premises A court would be likely to find this argument unpersuasive however because the landlord could have attempted to correct the problem after the first and second malfunctions

Assuming that the landlord was given sufficient notice and opportunity to cure a court would be likely to conclude that the tenant also satisfied the final element of vacating the premises within a reasonable time The landlord might argue that the tenant remained in the premises for almost four months after the air conditioning first failed which would suggest that the problem was not so severe as to have constructively evicted the tenant The tenant will argue however that he gave the landlord three months to cure the problem after the first two malfunctions threatened (but did not actually harm) his operations The tenant then moved out shortly after the final malfunction caused temperatures to exceed the tolerance levels of his manufacturing operations]

Point Two (10) The landlord did not accept the tenantrsquos surrender of the lease

When a tenant wrongfully moves from leased premises with the intent to terminate the lease the landlord may either accept the tenantrsquos surrender of the premises and terminate the lease or hold the tenant to the terms of the lease See HOVENKAMP amp KURTZ supra at 295ndash96 Here the tenantrsquos only basis for the claim that the landlord accepted his surrender is the landlordrsquos retention of the keys Many courts have considered whether a landlordrsquos retention of keys delivered by a tenant constitutes acceptance of surrender The weight of the case law holds that retention of the keys alone does not constitute acceptance of surrender without other evidence showing that the landlord intended to accept the surrender See generally 49 AM JUR 2d Landlord and Tenant sect 213

Here the landlordrsquos note saying ldquoI repeat the air-conditioning is not my problem You have leased the building and you should fix itrdquo strongly suggests that the landlord did not intend

17

Some courts have rejected the no-mitigation-of-damages rule based on efficiency concerns and societyrsquos interest in assuring that resources remain in the stream of commerce rather than lying vacant see id at 464ndash65 and allow landlords to sue tenants who have wrongfully terminated a lease for damages equal to the difference between the unpaid rent due under the lease and the propertyrsquos fair market rental value Other courts have abandoned the no-recovery-for-future-rent rule These courts responding to the fact that a tenant may well disappear or be judgment-proof by the time a lease term is concluded have allowed a landlord to collect damages equal to the value of rent over the entire lease term minus the propertyrsquos fair rental value when a tenant has wrongfully terminated a lease and unequivocally shown an intention not to return to the premises or pay future rent Under this approach a landlord receives approximately the same amount he would have received were there a duty to mitigate damages See Sagamore Corp v Willcutt 180 A 464 (Conn 1935)

Real Property Analysis

to accept the tenantrsquos surrender The tenant might argue that the landlordrsquos failure to make a similar statement when the keys were sent to her a second time and she retained them evidences a change of heart However it is likely that a court would find that the landlordrsquos retention of the keys represented a decision to safeguard the keys not to accept the tenantrsquos surrender

[NOTE An examinee should receive credit for arguing the other way with a well-reasoned argument]

Point Three (45) Under the common law the landlord had no duty to mitigate damages Additionally a landlord was not entitled to recover unpaid rents due in the future but was only entitled to recover rents in arrears at the time of the commencement of the suit Applying the common law here the landlord could recover $5000 the amount of rents due at the commencement of the suit ($2500 for September and the same for October) Today some courts allow the landlord under certain circumstances to sue the tenant for damages (not rent) equal to the difference if any between the unpaid promised rent for the balance of the term (here $175000) and the propertyrsquos fair rental value for the balance of the term

Under the common law because a lease was viewed as a conveyance instead of a contract a landlord had no duty to mitigate damages resulting from a tenantrsquos wrongful termination of a lease A landlord could thus recover the full value of rents that were due and unpaid at the time of the suit However under the common law a landlord could not sue a tenant for rents due in the future because there was always a possibility that the tenant might pay the rent when it was due See SINGER supra at 462 Thus using the common law approach on November 1 the landlord could only recover the full value of the two monthsrsquo rent actually due and unpaid ie $5000 for September and October

Here because the tenant returned the keys to the landlord and said ldquoI will not be returning to the building or making further rent paymentsrdquo the landlord could establish abandonment and an intention not to return It is thus possible that the landlord might recover damages in the amount of $5000 (for the months of September and October) plus the present value of $175000 minus the fair market rental value of the property over the remaining months of the lease

18

CONTRACTS ANALYSIS ____ (Contracts II IVE)

ANALYSIS

Legal Problems

(1) What was the legal effect of the sailorrsquos October 31 letter to the builder

(2)(a) What was the legal effect of the builderrsquos November 25 response to the sailorrsquos October 31 letter

(2)(b) What was the legal effect of the sailorrsquos refusal to take and pay for the boat on December 15

DISCUSSION

Summary

This is a sale of goods governed by the Uniform Commercial Code Because the sailor had reasonable grounds for insecurity about the builderrsquos ability to deliver the boat in a timely manner when the sailor learned about the strike on October 31 the sailor was legally justified in sending the letter to the builder seeking adequate assurance of the builderrsquos performance pursuant to the contract The builderrsquos failure to provide such assurance within a reasonable time operated as a repudiation of the contract However the builder was free to retract the repudiation before the sailor either cancelled the contract or materially changed position in reliance on the builderrsquos repudiation The builder retracted the repudiation when he informed the sailor that the workers were back and that the boat would be delivered by the date stipulated in the partiesrsquo contract Because the sailor had taken no action in response to the original repudiation he no longer had the right to cancel the contract with the builder The sailorrsquos subsequent statement that ldquoour contract is overrdquo may have constituted repudiation by the sailor In any event when the sailor failed to perform on December 15 that constituted breach

Point One (35) Because the sailor had reasonable grounds for insecurity with respect to the builderrsquos performance the sailorrsquos letter to the builder was a justified demand seeking assurance of the builderrsquos performance under the contract failure of the builder to provide such assurance within a reasonable time constituted repudiation of the contract

The sailor was legally justified in sending the letter to the builder on October 31 Contract parties are entitled to expect due performance of contractual obligations and are permitted to take steps to protect that expectation UCC sect 2-609 states that ldquo[w]hen reasonable grounds for insecurity arise with respect to the performance of either party the other may in writing demand adequate assurance of due performance rdquo Here the sailor learned on October 31 that the builderrsquos workers were on strike This gave the sailor reasonable grounds for insecurity about the builderrsquos ability to complete performance on time and thus gave the sailor the right to seek adequate assurance from the builder Because the sailorrsquos demand for assurance was justified the builder was required to provide assurance that was adequate under the circumstances within a reasonable time (not to exceed 30 days) or be held to have repudiated the contract UCC sect 2-609(4)

19

Contracts Analysis

Point Two(a) (30) The builder did not within a reasonable time provide the sailor adequate assurance of due performance this failure to provide assurance constituted a repudiation of the contract

Because the sailor with legal justification (see Point One) demanded from the builder assurance of due performance the builderrsquos failure to provide such assurance within a reasonable time was a repudiation of their contract See UCC sect 2-609(4) (ldquoAfter receipt of a justified demand[] failure to provide within a reasonable time not exceeding thirty days assurance of due performance is a repudiation of the contractrdquo) On October 31 the sailor requested that the builder provide adequate assurance regarding the completion of the boat by December 15 The builder did not respond to the sailorrsquos letter until November 25mdashnearly a month later Even if that response had been given in a reasonable time it nonetheless did not provide assurance of due performance It simply stated ldquoIrsquom sorry about the strike but it is really out of my hands I hope we settle it soon so that we can get back to workrdquo Therefore the builderrsquos November 25 response did not provide adequate assurance in response to the sailorrsquos justified request Thus the builder had repudiated the contract

Point Two(b) (35) Although the builder repudiated the contract with the sailor the builder probably retracted that repudiation on December 3 and the sailor was no longer entitled to cancel their contract Thus the sailorrsquos failure to perform the sailorrsquos obligations under the contract constituted a breach

The builderrsquos failure to provide adequate assurance of performance constituted a repudiation of their contract (see UCC sect 2-609(4)) but the builder was free to retract that repudiation until the sailor cancelled the contract or materially changed his position or indicated by communication or action that the sailor considered the repudiation to be final See UCC sect 2-611(1) (ldquoUntil the repudiating partyrsquos next performance is due he can retract his repudiation unless the aggrieved party has since the repudiation cancelled or materially changed his position or otherwise indicated that he considers the repudiation finalrdquo)

Here the facts state that before the builderrsquos December 3 telephone call to the sailor the sailor did nothing in response to the builderrsquos repudiation such as contracting with a third party for a boat The builderrsquos December 3 call informing the sailor that the boat would be timely delivered probably constituted a retraction of the repudiation because it clearly indicated to the sailor that the builder would be able to perform UCC sect 2-611(2) Thus after being so informed the sailor did not have the right to treat their contract as cancelled UCC sect 2-611(3) Accordingly the sailorrsquos failure to perform the sailorrsquos obligations under the contract by taking the boat and paying for it constituted a breach of the contract

20

CONSTITUTIONAL LAW ANALYSIS (Constitutional Law IVA F2b amp e)

ANALYSIS

Legal Problems

(1) Does AutoCorsquos operation of a ldquocompany townrdquo result in its actions counting as those of the state for purposes of constitutional analysis

(2) Does the expulsion of a schoolchild for failure to recite the Pledge of Allegiance violate the First Amendment as applied through the Fourteenth Amendment

(3) Does the arrest of a pamphleteer in connection with violation of an anti-littering rule where the littering is done by the recipients of leaflets distributed by the pamphleteer violate the First Amendment as applied through the Fourteenth Amendment

DISCUSSION

Summary

The First Amendment as applied through the Fourteenth Amendment applies only to state action It does not typically govern private actors However courts have found state action where the private actor has exercised a ldquopublic functionrdquo such as running a privately owned ldquocompany townrdquo as AutoCo has done here Thus First Amendment protections apply By requiring the son to participate in a mandatory Pledge of Allegiance ceremony AutoCo has compelled the expression of political belief in violation of the First Amendment as applied through the Fourteenth Amendment The fatherrsquos arrest in connection with breaching the anti-litter rule also violated the First Amendment as applied through the Fourteenth Amendment Although state actors can regulate the incidental effects of speech on the public streets on a content-neutral basis this power is limited and cannot extend to punishing a distributor of literature because of littering by third parties

Point One (30) AutoCorsquos operation of a company town (including a school) makes it a state actor under the public function strand of the state action doctrine

The individual rights protections of the Constitution apply only where there is ldquostate actionrdquomdash either direct action by the government or some action by a private party that is fairly attributable to the government As a general rule the actions of a private company like AutoCo or of a private school like the school operated by AutoCo would not constitute state action and the protections of the Constitution (in this case the First Amendment) would not apply

However there are situations in which the actions of a private actor are attributed to the state One such situation is when the private actor undertakes a public function There are not many bright-line rules in the Supreme Courtrsquos state action doctrine but one of them is this Where a private actor undertakes a ldquopublic functionrdquo the Constitution applies to those actions Where a corporation operates a privately owned ldquocompany townrdquo that provides essential services typically provided by a state actor the public function doctrine applies and the Constitution

21

Constitutional Law Analysis

binds agents of the town as if they were agents of the government See eg Marsh v Alabama 326 US 501 (1946) Here AutoCo does more than own the town it provides security services fire protection sanitation services and a school Thus the actions of AutoCo constitute state action and are governed by the Fourteenth Amendment

Point Two (35) The sonrsquos expulsion for failure to recite the Pledge of Allegiance violates the First Amendment as applied through the Fourteenth Amendment as a compelled expression of political belief

As explained in Point One the First Amendment applies to the school as a state actor Although children in public schools (and in schools subject to the First Amendment like

the Oakwood school) have some First Amendment rights Tinker v Des Moines Independent Community School District 393 US 503 506 (1969) schools have greater leeway to regulate the speech of students and teachers than the state would have outside the school context Hazelwood School Dist v Kuhlmeier 484 US 260 (1988) Morse v Frederick 551 US 393 (2007) However the Supreme Court has long held that public schools may not force their students to participate in a flag salute ceremony when it offends the political or religious beliefs of the students or their families West Virginia Board of Educ v Barnette 319 US 624 (1943) (invalidating a mandatory public school flag salute ceremony) see also Wooley v Maynard 430 US 705 (1977) (invalidating compelled expression of political belief on state-issued license plates)

In this case the school requires its students to participate in a flag salute and Pledge of Allegiance ceremony and punishes them when they refuse to participate Pursuant to this policy the school has expelled the son This expulsion violates the First Amendment ban on compelled expression

Point Three (35) Because the father was distributing leaflets in a traditional public forum his trespass arrest violated the First Amendment as applied through the Fourteenth Amendment

As explained in Point One AutoCo is treated as a state actor Thus Oakwoodrsquos commercial district is treated as government-owned property for purposes of the First Amendment Thus the leafleting here is subject to the First Amendment because it is an expressive activity Schneider v State of New Jersey Town of Irvington 308 US 147 (1939) When expression takes place on government-owned property government regulation of the expression is assessed under the public forum doctrine Public streets and sidewalks have long been held to be the classic example of a ldquotraditional public forumrdquo open to the public for expression Hague v CIO 307 US 496 515ndash16 (1939) Because the father was distributing leaflets while standing on a street corner in the commercial district his expressive activity occurred in a traditional public forum

When a state tries to regulate expressive activity in a traditional public forum it is prohibited from doing so based on the expressive activityrsquos content unless its regulation is narrowly tailored to achieve a compelling governmental interest (ldquostrict scrutinyrdquo) In this case however AutoCo is regulating the fatherrsquos expressive activity on the ostensibly neutral ground that his expressive activity has produced litter and made the street unsightly When a state tries to regulate expressive activity without regard to its content intermediate scrutiny applies Under intermediate scrutiny the true purpose of the regulation may not be the suppression of ideas (if so then strict scrutiny applies) the regulation must be narrowly tailored to achieve a significant

22

Constitutional Law Analysis

governmental interest and it must leave open ample alternative channels for expressive activity Ward v Rock Against Racism 491 US 781 791 (1989)

Here the application of the ordinance to the father will fail for two reasons First the Supreme Court has held that the governmentrsquos interest in keeping the streets clean is insufficient to ban leafleting in the public streets as the government power to regulate with incidental effects on public sidewalk speech is very limited See eg Schneider 308 US at 162 (leafletinglittering) Second the regulation (a blanket ban on distribution that results in littering) is not narrowly tailored to protect expression A narrowly tailored alternative would be prosecution only of people who litter Moreover the effect of the littering rule is likely to be a ban on all leafleting thus eliminating an entire class of means of expression This raises the possibility that there are not ldquoample alternative channels of communicationrdquo open to the father as required under the Courtrsquos standard of review for content-neutral regulation of speech

[NOTE Some examinees might argue that this is a ldquotime place and mannerrdquo restriction and that AutoCo might have greater latitude to regulate the public sidewalks under this theory This argument is incorrect for two reasons First the Supreme Court has held that the power to regulate speakers through littering laws is very limited for the reasons given and in the cases cited above But more generally a ldquotime place and mannerrdquo restriction involves the shifting of speech from one time and place to another or to another manner here there is no shifting but a direct punishment for expressive activity (albeit one couched in content-neutral terms) In addition some examinees might read the ordinance to be in effect a total ban on leafleting since most leafleting will produce some litter Those examinees might note that the Court has required total bans on an entire mode of expression to satisfy strict scrutiny and analyze the fatherrsquos prosecution here accordingly See United States v Grace 461 US 171 177 (1983) (invalidating ban on display of signs on public sidewalks surrounding US Supreme Court ldquo[a]dditional restrictions such as an absolute prohibition on a particular type of expression will be upheld only if narrowly drawn to accomplish a compelling governmental interestrdquo)]

23

SECURED TRANSACTIONS ANALYSIS (Secured Transactions IID E IVA B C)

ANALYSIS

Legal Problems

(1) Is a purchase-money security interest in consumer goods perfected even though there has been no filing of a financing statement

(2) Does a person who buys consumer goods for personal use take those goods free of a prior perfected purchase-money security interest in the goods

(3) Does a person who receives consumer goods as a gift take those goods subject to a prior perfected security interest in them

DISCUSSION

Summary

The retailerrsquos security interest in the bicycles was perfected even though no financing statement was filed because it was a purchase-money security interest in consumer goods A purchase-money security interest in consumer goods is automatically perfected upon attachment

The buyer is not subject to the retailerrsquos security interest in the bicycle that the buyer bought from the man Because the bicycle was consumer goods in the hands of the man and the retailer never filed a financing statement covering the bicycle the retailerrsquos security interest is not effective against someone like the buyer who bought the bicycle for value without knowledge of the retailerrsquos security interest and for personal use

On the other hand the retailerrsquos security interest continues in the bicycle given to the friend because the friend did not give value for the bicycle or buy it in the ordinary course of business

Point One (35) The retailerrsquos security interest in the bicycles attached on June 1 Because this interest was a purchase-money security interest in consumer goods it was automatically perfected when it attached

The retailerrsquos security interest in the bicycles attached on June 1 when the man bought the bicycles (acquiring rights in the collateral) signed a security agreement containing a description of the collateral and received value from the retailer (by being given credit with which to purchase the bicycles) UCC sect 9-203(a) amp (b)

Despite the retailerrsquos failure to file a financing statement its security interest was perfected Pursuant to UCC sect 9-309(1) a security interest is automatically perfected upon attachment if the goods are ldquoconsumer goodsrdquo and the security interest is a ldquopurchase-money security interestrdquo

In this case the bicycles sold by the retailer to the man were consumer goods at the time of sale The bicycles were ldquogoodsrdquo because they were ldquomovable when a security interest

24

Secured Transactions Analysis

attachesrdquo UCC sect 9-102(a)(44) They were also consumer goods because they were ldquobought for use primarily for personal family or household purposesrdquo UCC sect 9-102(a)(23) The retailerrsquos security interest in these consumer goods was also a ldquopurchase-money security interestrdquo A purchase-money security interest is an interest that secures a debt that was incurred in order to ldquoenable the debtor to acquire rights in or the use of the collateralrdquo UCC sect 9-103(a) (b)(1) Here the man incurred an obligation to the retailer to purchase the bicycles so the security interest he gave the retailer to secure that obligation was a purchase-money security interest

Because the retailerrsquos security interest was a purchase-money security interest in consumer goods it was automatically perfected on June 1 when the interest attached to the bicycles

Point Two (35) The buyer took the bicycle free of the retailerrsquos security interest because (i) the retailer did not file a financing statement covering the bicycle (ii) the bicycle was ldquoconsumer goodsrdquo and (iii) the buyer bought the bicycle for value without knowledge of the retailerrsquos security interest and for personal use

A security interest continues in collateral even after a sale or other disposition of that collateral unless the creditor authorized the disposition ldquofree of the security interestrdquo or another Article 9 exception applies UCC sectsect 9-201(a) and 9-315(a)(1)

However a buyer of goods like the buyer here can take free of a prior security interest in those goods under certain circumstances See UCC sectsect 9-317(b) (buyers who give value and receive delivery of goods without knowledge of an unperfected security interest in the goods) and 9-320(a) amp (b) (buyer in ordinary course of business buyer of consumer goods in a consumer-to-consumer transaction who gives value) In this case the retailerrsquos security interest was perfected when the buyer purchased the bicycle so UCC sect 9-317(b) does not protect the buyer The buyer also is not a protected ldquobuyer in ordinary course of businessrdquo because he did not purchase from a person who is in the business of selling bicycles See UCC sect 1-201(b)(9)

The buyer can however qualify for the protection of UCC sect 9-320(b) That section provides that a buyer of goods from a person who used them for personal family or household purposes takes free of a perfected security interest in the goods if (1) the buyer had no knowledge of the security interest (2) the buyer gave value for the goods (3) the buyer purchased the goods primarily for personal family or household purposes and (4) the purchase occurred before the filing of a financing statement covering the goods

The buyer met all of these criteria The man used the bicycle for personal purposes The buyer purchased the bicycle from the man and the buyer had no knowledge of the retailerrsquos security interest The buyer gave value ($400) for the bicycle and he bought it ldquoprimarily for personal family or household purposesrdquo as he planned to use it for recreation which is a personal rather than a business use Finally no financing statement had been filed Therefore under UCC sect 9-320(b) the buyer took free of the retailerrsquos security interest

Point Three (30) The retailerrsquos security interest continues in the bicycle that the man gave to the friend Thus the retailer can recover the bicycle from the friend because the friend did not give value for the bicycle or buy it in the ordinary course of business

25

Secured Transactions Analysis

As noted in Point Two the retailer did not authorize the man to dispose of the bicycle Consequently the retailerrsquos security interest continued in the bicycle even after the man transferred ownership of the bicycle to the friend See UCC sectsect 9-201(a) and 9-315(a)(1) The retailerrsquos security interest in the bicycle will be effective against the friend unless some other provision of Article 9 allows the friend to take the bicycle free of that security interest

Unfortunately for the friend there is no Article 9 provision that allows him to take free of the retailerrsquos interest The friendrsquos basic problem is that he is not a buyer of the bicyclemdashhe received the bicycle as a gift and did not give value for it Thus the friend is not protected by any of the applicable exceptions See UCC sectsect 9-317(b) (protecting buyers who give value for goods subject to an unperfected security interest) 9-320(a) (protecting buyers in ordinary course of business) and 9-320(b) (protecting buyers of consumer goods who give value)

In short the retailerrsquos security interest continues in the bicycle that the man gave to the friend The friend took the bicycle subject to that security interest

26

FEDERAL CIVIL PROCEDURE ANALYSIS (Federal Civil Procedure VIE)

ANALYSIS

Legal Problems

(1) Does a judgment in a prior action preclude a nonparty from suing the same defendant on a closely related claim when the nonparty and the original plaintiff are in a family relationship

(2) Does a judgment rendered in an earlier action preclude a nonparty from litigating an issue that was actually decided in the first suit

(3) May a nonparty to an earlier action invoke the judgment in that action to preclude a party to the prior action from relitigating an issue that the party had a full and fair opportunity to litigate in the earlier action

DISCUSSION

Summary

Pursuant to the doctrines of claim preclusion (res judicata) and issue preclusion (collateral estoppel) a judgment is binding on the parties thereto In the absence of privity nonparties to a prior suit cannot be bound by a judgment rendered in their absence Thus in the absence of privity a nonparty to the first suit is not precluded from presenting her claim in a second suit even if it is factually related to the claims and defenses presented in the first suit nor is she bound by determinations of issues made in the first suit A family relationship without more does not support a finding of privity For this reason Mother as a nonparty is not bound by the judgment in the Son-Driver action She may bring her separate claim for damage to her car and she is not precluded from litigating the question of whether she was negligent in the maintenance of her car

Driver on the other hand could be precluded from relitigating the issue of her negligence pursuant to the doctrine of non-mutual issue preclusion (also called non-mutual offensive collateral estoppel) which allows a nonparty to a prior action to invoke issue preclusion to prevent a party to that prior action from relitigating determinations of issues made therein However Mother may be prevented from invoking non-mutual collateral estoppel in this case because she could easily have joined her claim in the prior action but did not do so

[NOTE Federal common law governs the preclusive effect of a judgment rendered by a federal court sitting in diversity See Semtek Intrsquol Inc v Lockheed Martin Corp 531 US 497 508 (2001) But the Semtek Court concluded that federal common law in this context incorporates the preclusion law of the state in which the rendering federal court sits (unless the state law is incompatible with federal interests) id at 508ndash09 Thus State Arsquos preclusion law determines the preclusive effect of the judgment rendered in Sonrsquos suit against Driver The problem says that State A preclusion law is identical to federal preclusion law so the following analysis utilizes general principles of preclusion drawn from Supreme Court case law (announcing federal preclusion rules) and the Restatement (Second) of Judgments]

27

Federal Civil Procedure Analysis

Point One (35) Under the doctrine of claim preclusion the judgment rendered in the first action does not preclude Mother a nonparty from suing Driver for the damage to her car because the judgment binds only parties or those in privity with them and Mother and Son are not in privity

Driver may contend that the doctrine of claim preclusion (res judicata) precludes Mother from presenting a claim arising from the same nucleus of facts that was presented in the first action brought by Son According to the doctrine of claim preclusion ldquowhen a court of competent jurisdiction has entered a final judgment on the merits of a cause of action the parties to the suit and their privies are thereafter bound lsquonot only as to every matter which was offered and received to sustain or defeat the claim or demand but as to any other admissible matter which might have been offered for that purposersquordquo Commissioner of Internal Revenue v Sunnen 333 US 591 597 (1948) (citation omitted)

However the doctrine of claim preclusion does not apply to Mother on the facts of this problem First Mother was not a party to the earlier case ldquoIt is a principle of general application in Anglo-American jurisprudence that one is not bound by a judgment in personam in a litigation in which he is not designated as a party or to which he has not been made a party by service of processrdquo Taylor v Sturgell 553 US 880 884 (2008) (citing Hansberry v Lee 311 US 32 40 (1940)) see also RESTATEMENT (SECOND) OF JUDGMENTS sect 34(3) (1982) This rule reflects our ldquodeep-rooted historic tradition that everyone should have his own day in courtrdquo Martin v Wilks 490 US 755 762 (1989) (citation omitted) (superseded by statute on other grounds) Since Mother was not a party to the first suit she is not bound by the judgment unless an exception to the general rule applies

Mother might be bound by the prior judgment if she were considered to have been sufficiently in privity with Son that Son represented her interests in that action ldquoA person who is not a party to an action but who is represented by a party is bound by and entitled to the benefits of a judgment as though he were a partyrdquo RESTATEMENT (SECOND) OF JUDGMENTS sect 41(1) But there is no suggestion in the facts of the problem that Son who is an adult purported to represent Motherrsquos interests in the first suit ldquo[C]lose family relationships are not sufficient by themselves to establish privity with the original suitrsquos party or to bind a nonparty to that suit by the judgment entered therein rdquo Cuauhtli v Chase Home Finance LLC 308 Fed Appx 772 773 (5th Cir 2009) (citation omitted) accord 18A CHARLES ALAN WRIGHT ET AL FEDERAL

PRACTICE AND PROCEDURE sect 4459 (2d ed 2002) In Taylor v Sturgell supra the Supreme Court identified other special circumstances in

which nonparties may be bound by a prior judgmentmdashwhen a nonparty consents to be bound when a nonparty is in a pre-existing substantive legal relationship with a party (such as preceding and succeeding property owners) when a nonparty assumed control of the prior litigation when a party seeks to relitigate through a proxy or where a special statutory scheme seeks to foreclose successive litigation by nonparties See Taylor 553 US at 893ndash95 None of these circumstances exists here

Because Mother was not a party to the first suit and is not in privity with Son who is an adult the judgment in the first action does not preclude her from bringing her own claim against Driver

Point Two (35) Under the doctrine of issue preclusion the judgment rendered in the first action does not preclude Mother a nonparty from litigating the issue of her negligence in maintaining her carrsquos

28

Federal Civil Procedure Analysis

brake lights because the judgment binds only parties or those in privity with them and Mother and Son are not in privity

By its affirmative response to a special interrogatory the jury in the first action expressly concluded that ldquoMother negligently failed to ensure that the brake lights on her car were in proper working orderrdquo Driver may attempt to invoke the doctrine of issue preclusion to preclude Mother from relitigating this issue in the second action

[I]ssue preclusion arises in a second action on the basis of a prior decision when the same lsquoissuersquo is involved in both actions the issue was lsquoactually litigatedrsquo in the first action after a full and fair opportunity for litigation the issue was lsquoactually decidedrsquo in the first action by a disposition that is sufficiently lsquofinalrsquo lsquoon the meritsrsquo and lsquovalidrsquo it was necessary to decide the issue in disposing of the first action and the later litigation is between the same parties or involves nonparties that are subject to the binding effect or benefit of the first action Once these requirements are met issue preclusion is available not only to defend against a demand for relief but also as offensive support for a demand for relief Issue preclusion moreover is available whether or not the second action involves a new claim or cause of action

18 CHARLES ALAN WRIGHT ET AL FEDERAL PRACTICE AND PROCEDURE sect 4416 at 392ndash93 (2d ed) see also RESTATEMENT (SECOND) OF JUDGMENTS sect 27 (1982)

Here several of the elements necessary for issue preclusion are present The same issue is involved in both actionsmdashthe issue of Motherrsquos negligence in failing to maintain the brake lights on her car That issue was actually litigated in the first action and decided by the jury There is nothing to suggest anything less than a full and fair opportunity to litigate The judgment disposing of the issue was final

Nevertheless the judgment will not preclude Mother from relitigating the issue for two reasons First Mother was not a party to the first action and as explained above Mother and Son are not in privity Therefore she cannot be denied an opportunity to litigate the issue of her negligence Second it does not appear that the juryrsquos decision as to Motherrsquos negligence was necessary to the prior judgment against Driver Nothing suggests that the finding on Motherrsquos negligence had any bearing on the outcome of the first action

Point Three (30) Under the doctrine of non-mutual issue preclusion the judgment rendered in the first action might preclude Driver from relitigating the issue of her negligence However Driver has a strong argument that such a result would be inconsistent with the policy against offensive use of non-mutual estoppel when the non-party plaintiff easily could have joined as a plaintiff in the first action

Because Son already convinced the jury in the first action that ldquoDriver was negligent in the operation of her vehiclerdquo Mother may wish to invoke the doctrine of non-mutual issue preclusion to prevent Driver from relitigating the question of her negligence As noted above ldquoissue preclusion arises in a second action on the basis of a prior decision when the same lsquoissuersquo is involved in both actions the issue was lsquoactually litigatedrsquo in the first action after a full and fair opportunity for litigation the issue was lsquoactually decidedrsquo in the first action by a disposition that is sufficiently lsquofinalrsquo lsquoon the meritsrsquo and lsquovalidrsquo it was necessary to decide the issue in disposing of the first action rdquo 18 CHARLES ALAN WRIGHT ET AL FEDERAL PRACTICE AND

PROCEDURE sect 4416 at 392 (2d ed) see also RESTATEMENT (SECOND) OF JUDGMENTS sect 27

29

Federal Civil Procedure Analysis

Here these basic requirements for issue preclusion are met First the same issue is involved in both suits whether Driver was negligent in the operation of her car Second this issue was actually litigated and decided in the first action the jury answered a special interrogatory raising this very question There is nothing to suggest that Driver lacked a full and fair opportunity to litigate the issue Since a judgment was rendered against Driver for the injuries Son sustained as a result of Driverrsquos negligence resolution of the issue was necessary to dispose of the first action Driver was a party to the first action so she may be bound by the judgment

[NOTE Traditionally issue preclusion required mutualitymdashboth the party asserting issue preclusion and the party against whom issue preclusion was asserted were bound by the prior judgment Under the traditional mutuality rule Mother could not assert issue preclusion against Driver because Mother would not be bound by the judgment if Driver sought to rely on it See Point One There is no mutuality between Mother and Driver with respect to the prior judgment

This traditional mutuality requirement has been abandoned in most jurisdictions The Supreme Court rejected a strict mutuality requirement in Blonder-Tongue Laboratories Inc v University of Illinois Foundation 402 US 313 (1971) (non-mutual defensive collateral estoppel used by a defendant to preclude a plaintiff from relitigating a claim the plaintiff previously litigated) and Parklane Hosiery Co v Shore 439 US 322 (1979) (non-mutual offensive collateral estoppel used by a plaintiff to preclude a defendant from relitigating a claim the defendant previously litigated) In Parklane Hosiery the Court concluded (as a matter of federal preclusion law) that trial courts should have ldquobroad discretionrdquo to determine whether or not to permit a plaintiff to invoke non-mutual issue preclusion ldquoThe general rule should be that in cases where a plaintiff could easily have joined in the earlier action or where the application of offensive estoppel would be unfair to a defendant a trial judge should not allow the use of offensive collateral estoppelrdquo Id at 331

The Parklane Hosiery decision identified a number of circumstances that might make it unfair to allow a plaintiff to invoke non-mutual issue preclusion (non-mutual offensive collateral estoppel in the traditional terminology) against a defendant In particular the Parklane Hosiery court suggested that issue preclusion may not be appropriate if the plaintiff in the second action ldquocould easily have joined in the earlier actionrdquo Id Prohibiting plaintiffs from using non-mutual estoppel under such circumstances would promote judicial efficiency by encouraging plaintiffs to join the prior action It would also discourage plaintiffs from staying out of prior litigation in order to secure in effect two bites at the apple using the prior litigation offensively if the defendant loses and forcing the defendant to litigate a second time if the defendant wins the prior action

An exceptional exam answer might therefore argue that non-mutual issue preclusion should be denied on these facts Son and Mother both reside in State A since they are related they know each other well and Son was driving Motherrsquos car when the accident occurred They could have sued together and Rule 20 of the Federal Rules of Civil Procedure would have authorized joinder of their claims because those claims arose from the same transaction or occurrence and raised a common question of law or fact FED R CIV P 20(a) The facts do not suggest that Mother had any reason not to join Sonrsquos suit other than a desire to see how Sonrsquos action concluded before bringing her own claim Cf Nations v Sun Oil Co (Del) 695 F2d 933 938 (5th Cir 1983) (concluding that plaintiff ldquowas entitled to await the development of his injuries and their predictable consequencesrdquo) Because it appears that Mother may be a ldquowait-and-seerdquo plaintiff who could easily have joined the original action a trial court might disallow as a matter of discretion her use of non-mutual issue preclusion]

30

AGENCY ANALYSIS __________ (Agency I II)

ANALYSIS

Legal Problems

(1) Is the principal or the agent or both liable on contracts with a third party when the principal is an ldquoundisclosed principalrdquo

(2) Is the principal or the agent or both liable on contracts with a third party when the principal is ldquopartially disclosedrdquo or an ldquounidentified principalrdquo

(3) Is the principal or the agent or both liable on contracts with a third party for the purchase of goods when the agent exceeded his authority but the principal nonetheless accepts the goods

DISCUSSION

Summary

The agent but not the owner is liable to the basket manufacturer because the owner is an undisclosed principal and the agent acted without actual or apparent authority Both the agent and the owner however are liable on the burner contract because the owner is an unidentified principal and the agent had apparent authority to enter into that contract With respect to the solar cells contract whether the owner is liable depends upon whether a court would follow the Second or Third Restatement of Agency which take different positions on the effect of the ratification of a contract by an undisclosed principal Under either the agent would also be liable on the contract as he was a party to the contract

[NOTE The contracts that are the subject of this question are contracts for the sale of goods and therefore are governed by Article 2 of the Uniform Commercial Code Article 2 however does not contain agency rules Accordingly common law concepts of agency are applicable UCC sect 1-103(b)]

Point One (35) The agent but not the owner is liable to the basket manufacturer The agent had no actual authority to enter into the contract to buy aluminum baskets and because the owner was an undisclosed principal the manufacturer had no reason to believe that the agent had apparent authority Furthermore the manufacturer had no reason to believe that the agent was not contracting for his own benefit

An agent acting on behalf of a principal can bind the principal to contracts if the agent has either actual or apparent authority An agent has actual authority when contracting on behalf of his principal if he ldquoreasonably believes in accordance with the principalrsquos manifestations to the agent that the principal wishes the agent so to actrdquo RESTATEMENT (THIRD) OF AGENCY sect 201 (2006) Here the agent was told to buy only wicker baskets not aluminum baskets Thus when he contracted with the basket manufacturer to buy aluminum baskets he had no actual authority to do so

31

Agency Analysis

An agent acts with apparent authority ldquowhen a third party [with whom the agent acts] reasonably believes the actor has authority to act on behalf of the principal and that belief is traceable to the principalrsquos manifestationsrdquo Id sect 203 Here the owner notified basket manufacturers that she or her agent might contact them to purchase baskets but that notification did not specifically name the agent or any other person as the ownerrsquos agent Furthermore the basket manufacturer had no prior dealings with the agent or the owner or any reason to think that the agent was acting for the benefit of anyone but himself Thus there is no basis to conclude that the basket manufacturer thought that the agent had apparent authority to act for the owner

Generally when an agent acts on behalf of an undisclosed principal and the agent lacks authority to enter into the contract the agent is liable on the contract as a party to the contract but the principal is not liable This rule is consistent with the third partyrsquos expectations ldquoThe third party expected the agent to be a party to the contract because the agent presented the deal as if he were acting for himself Moreover if the third party is unaware of the principalrsquos existence the third party must be relying on the agentrsquos solvency and reliability when entering into the contractrdquo See ROBERT W HAMILTON JONATHAN R MACEY amp DOUGLAS K MOLL CORPORATIONS INCLUDING PARTNERSHIPS AND LIMITED LIABILITY COMPANIES 34 (11th ed 2010) See also RESTATEMENT (THIRD) OF AGENCY sect 603 cmt c Furthermore because the third party has no idea that the agent is acting or is seemingly acting on behalf of another there is no reason to believe that the third party would be expecting an undisclosed principal to be liable on the contract Id

Point Two (35) Because the owner is an unidentified (as opposed to undisclosed) principal both she and the agent (as a party to the contract) probably are liable on the contract with the burner manufacturer

When the agent contracted with the burner manufacturer he did not have actual authority to do so as the owner had expressly restricted the agentrsquos authority to purchase only burners with ldquowhisper technologyrdquo See Point One However the agent may have had apparent authority to buy burners without whisper technology

An agent acts with apparent authority ldquowhen a third party [with whom the agent acts] reasonably believes the actor has authority to act on behalf of the principal and that belief is traceable to the principalrsquos manifestationsrdquo RESTATEMENT (THIRD) OF AGENCY sect 203 (2006) The owner indicated that an agent might contact the burner manufacturer The notice contained no restriction regarding the type of burners that the agent was authorized to purchase The facts indicate that burner manufacturers regularly receive such notices

Although the agent told the burner manufacturer that he represented a well-known hot-air balloon operator he did not disclose the ownerrsquos name Thus the owner was a partially disclosed or unidentified principal See RESTATEMENT (SECOND) OF AGENCY sect 4(2) (1958) (using term ldquopartially disclosed principalrdquo) RESTATEMENT (THIRD) OF AGENCY sect 104(2)(c) (2006) (using term ldquounidentified principalrdquo) An agent for a partially disclosed principal may have apparent authority RESTATEMENT (SECOND) OF AGENCY sect 159 cmt e (1958) Based upon (1) the notice sent by the owner (2) the agentrsquos revelation that he was acting as an agent and (3) the fact that burner manufacturers regularly receive such notices and sell to agents the manufacturer may argue that it reasonably and actually believed that the agent was authorized to purchase burners without whisper technology The manufacturer may also argue that because the agent revealed that he was an agent his listing of the ownerrsquos address as the delivery address connects the agent to the notice given by the owner Arguably this distinguishes the burner contract from the basket

32

Agency Analysis

contract Here there is a strong case to support the conclusion that the agent had apparent authority if he did then the owner is liable to the burner manufacturer

The agent also is liable as a party to the contract because he did not fully disclose his agency relationship Although he told the burner manufacturer that he represented a well-known hot-air balloon operator he did not disclose the ownerrsquos name Generally even an authorized agent of a partially disclosed or unidentified principal is liable as a party to a contract with a third person RESTATEMENT (SECOND) OF AGENCY sect 321 (1958) (ldquounless otherwise agreedrdquo) RESTATEMENT (THIRD) OF AGENCY sect 602(2) (2006) (ldquounless the agent and the third party agree otherwiserdquo)

Point Three (30) Under the Second Restatement of Agency the owner is not liable on the contract for solar cells because the agent did not have actual or apparent authority and the owner as an undisclosed principal cannot ratify the contract Under the Third Restatement the owner could be liable as she ratified the contract Under either Restatement the agent is liable as a party to the contract

The owner is not liable to the solar cell manufacturer for breach of the contract for the solar cells because the agent had no actual or apparent authority to purchase solar cells on the ownerrsquos behalf and the owner under the Second Restatement of Agency did not ratify the contract with knowledge of the material facts Thus she is not liable as a ratifier of the contract

The facts state that the agent had authority to purchase only propane fuel tanks In addition he had no apparent authority to purchase solar cells The owner made no manifestations to the solar cell manufacturer that would lead a reasonable person in the manufacturerrsquos position to believe that the agent had the authority to bind the owner to a contract to purchase solar cells In fact the agent made no manifestations at all to the solar cell manufacturer Unlike with the basket manufacturer and the burner manufacturer the owner did not notify the manufacturer of solar cells that an agent might contact it to purchase solar cells In addition the solar cells were delivered to the agent and not to the ownerrsquos address In sum the manufacturer was unaware of any relationship between the owner and the agent As to the solar cell manufacturer the owner is an undisclosed principal There can be no apparent authority in the case of an undisclosed principal because there are no manifestations from the principal to the third person See RESTATEMENT (SECOND) OF AGENCY sect 8 cmt a (1958) (ldquothere can be no apparent authority created by an undisclosed principalrdquo) RESTATEMENT (THIRD) OF AGENCY sect 203 cmt f (2006) (ldquoapparent authority is not present when a third party believes that an interaction is with an actor who is a principalrdquo)

The owner also did not ratify the contract Although the owner used the solar cells generally a principal cannot ratify an unauthorized transaction with a third person ldquounless the one acting purported to be acting for the ratifierrdquo RESTATEMENT (SECOND) OF AGENCY sect 85(1) (1958)

The result differs under the Third Restatement which expressly rejects the Second Restatement on this issue The Restatement (Third) of Agency sect 403 (2006) states ldquoA person may ratify an act if the actor acted or purported to act as an agent on the personrsquos behalfrdquo According to comment b ldquoan undisclosed principal may ratify an agentrsquos unauthorized actrdquo Under the Restatement (Third) of Agency rule the owner probably ratified the transaction The agent clearly acted on the ownerrsquos behalf and in addition the ownerrsquos conduct in using the solar cells ldquojustifies a reasonable assumption that [she] is manifesting assent that the act shall affect [her] legal relationsrdquo See id sect 401(2)

33

Agency Analysis

The agent also is liable to the solar cell manufacturer for breach of the contract for the solar cells because he is a party to the contract The facts indicate that the agent never told the solar cell manufacturer that he represented the owner or any other principal Consequently even if the agent were authorized (which as discussed above he is not) he would be liable as a party to the contract See RESTATEMENT (SECOND) OF AGENCY sect 322 (1958) RESTATEMENT (THIRD) OF AGENCY sect 603(2) (2006) Here he has no authority or apparent authority and is liable as a party to the contract

The agent would also be liable under the Third Restatement Under Restatement (Third) of Agency sect 402(1) (2006) ratification generally relates back and the transaction is treated as if it were authorized at the time of the transaction However this does not relieve the agent of an undisclosed principal who ratifies an unauthorized transaction of liability under the ratified contract See id sect 603(2) (authorized agent for undisclosed principal is a party to the contract) and sect 403 cmt b (ldquoAn undisclosed principalrsquos ratification does not eliminate the agentrsquos liability to the third party on the transaction rdquo)

[NOTE An examinee may discuss the concept of inherent agency power This concept is recognized by the Restatement (Second) of Agency sect 8 A (1958) but the concept is not used in the Restatement (Third) of Agency (2006) Here there are no facts to support that the agent had inherent authority

As to contracts with agents for partially disclosed principals (eg the contract for the burners) the basic question is whether the acts done ldquousually accompany or are incidental to transactions which the agent is authorized to conductrdquo RESTATEMENT (SECOND) OF AGENCY

sect 161 (1958) If so the principal is bound if the other party ldquoreasonably believes that the agent is authorized to do them and has no notice that he is not so authorizedrdquo Id The purchase of burners without whisper technology was not authorized nor was it incidental to an authorized transaction Therefore there should not be inherent agency power

As to contracts on behalf of undisclosed principals (eg the other two contracts) the basic question is whether the acts done are usual or necessary in the transactions the agent is authorized to transact RESTATEMENT (SECOND) OF AGENCY sect 194 (1958) The other two contracts seem fundamentally different from the authorized transactions Therefore there should not be inherent agency power

Only minimal credit should be given for discussion of inherent agency power]

34

EVIDENCE ANALYSIS _____ (Evidence IIA VA B E F J K)

ANALYSIS

Legal Problems

(1) Is the authenticated copy of the mechanicrsquos text message relevant and admissible

(2) Is the womanrsquos question ldquoIs my scooter safe to drive for a whilerdquo relevant and admissible

(3) Is the womanrsquos testimony describing the mechanicrsquos thumbs-up relevant and admissible

DISCUSSION

Summary

The mechanicrsquos text message to the woman is relevant to whether (1) the woman lost control of the scooter due to its defective brakes (2) the woman knew that the brakes needed repair and (3) it was negligent for the woman to drive the scooter knowing that its brakes needed repair

The mechanicrsquos text message is hearsay if it is offered by the pedestrian to prove that the scooterrsquos brakes needed repair However it fits the hearsay exception for present sense impressions and probably also fits the exception for business records The mechanicrsquos text message is not hearsay if it is instead offered by the pedestrian to prove the womanrsquos state of mind (ie that she had notice that her brakes needed repair)

The womanrsquos question to the mechanic and his response are also relevant to whether the brakes caused the accident and whether the woman was negligent The question is not hearsay because the woman did not make an assertion

The mechanicrsquos thumbs-up response is nonverbal conduct intended by the mechanic as an assertion and is therefore an out-of-court statement If the woman offers the mechanicrsquos statement to prove that the scooter was actually safe to ride the womanrsquos testimony about the statement is hearsay

However the mechanicrsquos statement is not hearsay if it is offered by the woman to prove her state of mind Therefore the womanrsquos question and the mechanicrsquos response are admissible to prove the womanrsquos state of mind

Point One(a) (20) The mechanicrsquos text message to the woman should be admitted because it is relevant

Evidence is relevant if it has ldquoany tendency to make a fact more or less probable than it would be without the evidencerdquo FED R EVID 401 ldquoRelevant evidence is admissiblerdquo unless it is inadmissible pursuant to some other rule FED R EVID 402

The mechanicrsquos text message to the woman ldquoWhen you pick up your scooter you need to schedule a follow-up brake repair Wersquoll order the partsrdquo is relevant for two reasons First this evidence has some tendency to make it more probable that the brakes malfunctioned and

35

Evidence Analysis

caused the accident Second it has some tendency to make it more probable that the woman was negligent in riding her scooter after being told by the mechanic that it required further repair

Point One(b) (30) The mechanicrsquos text message fits either the hearsay exception for present sense impressions or the exception for business records or it is admissible non-hearsay

The mechanicrsquos text message is a statement under Rule 801(a) because it is ldquoa written assertionrdquo FED R EVID 801(a) The text message is hearsay if the pedestrian offers it to prove the ldquotruth of the matter asserted in the statementrdquo (ie that the scooterrsquos brakes required repair) which resulted in the woman losing control of the scooter and causing the accident FED R EVID 801(c)

However the mechanicrsquos text message fits the hearsay exception for ldquopresent sense impressionsrdquo under Rule 803(1) because it is ldquo[a] statement describing or explaining an event or condition made while or immediately after the declarant perceived itrdquo FED R EVID 803(1) Here the mechanicrsquos text message described the condition of the scooter immediately after he perceived it during the maintenance service

The mechanic is a person with knowledge of the condition of the scooter so if text messages regarding repairs were made and kept by the mechanic in the ordinary course of business this text message also fits the business records exception Under Rule 803(6) a business record is a record of an act ldquomade at or near the time by someone with knowledgerdquo and ldquothe record was kept in the course of a regularly conducted activity of a businessrdquo and ldquomaking the record was a regular practice of that activityrdquo FED R EVID 803(6)

However the text message is not hearsay if it is instead offered to prove that the woman was negligent because she rode her scooter after the mechanic told her it required repair If offered for this purpose it would not be offered for the truth of the matter asserted in the statement but to show the womanrsquos belief about the condition of the scooter (her state of mind)

Point Two (10) The womanrsquos question to the mechanic should be admitted because it is not hearsay

The womanrsquos question to the mechanic is relevant because along with the mechanicrsquos thumbs-up response (see Point Three) it has some tendency to make it more probable that the woman was not negligent andor that the scooter brakes did not malfunction and cause the accident FED R EVID 401 The womanrsquos question does not raise hearsay concerns because it is not an assertion

Hearsay is defined under Rule 801(a) as ldquoan oral assertion written assertion or nonverbal conductrdquo Although ldquoassertionrdquo is not further defined ldquoa favorite [definition] of writers in the [evidence] field for at least a century and a half [is that] the word simply means to say that something is so eg that an event happened or a condition existedrdquo 2 MCCORMICK ON

EVIDENCE sect 246 (6th ed 2006) Under this definition the womanrsquos question is not hearsay because it is not an assertion

Point Three(a) (20) The mechanicrsquos thumbs-up to the woman is a nonverbal assertion that is relevant and the womanrsquos testimony about that response is admissible

36

Evidence Analysis

Hearsay is defined under Rule 801(c) as a ldquostatementrdquo that is ldquoa personrsquos oral assertion written assertion or nonverbal conduct if the person intended it as an assertionrdquo FED R EVID 801(a) Here when the mechanic responded to the womanrsquos question (ldquoIs my scooter safe to ride for a whilerdquo) with a thumbs-up gesture the facts suggest that he intended his nonverbal conduct as an assertion that in his opinion the scooter was safe to ride

The mechanicrsquos assertion is relevant and admissible to prove that the woman was not negligent because the evidence makes it more probable that at the time of the accident she believed that the scooter was safe to ride despite the fact that the brakes required repair FED R EVID 401 Admission of the womanrsquos description of the mechanicrsquos thumbs-up for this purpose does not raise hearsay concerns because the evidence would not be offered for the truth of the matter asserted but to show the womanrsquos belief about the condition of the scooter (her state of mind)

Point Three(b) (20) The mechanicrsquos thumbs-up is relevant to determine whether the scooterrsquos brakes malfunctioned causing the accident but if offered for this purpose it is also hearsay

The mechanicrsquos nonverbal assertion is relevant to the determination of whether the scooterrsquos brakes malfunctioned causing the accident However if offered to prove the ldquotruth of the matter asserted in the statementrdquo (ie that the scooter was safe to ride for a while) it is hearsay that does not fit any hearsay exception

37

TRUSTS AND FUTURE INTERESTS ANALYSIS ____________________ (Trusts and Future Interests IC1 amp 4 G IIF)

ANALYSIS

Legal Problems

(1)(a) Was the revocable trust amendable

(1)(b) If the trust was amendable must the amendment have been executed in accordance with the state Statute of Wills in order to be valid

(2) If the trust amendment was valid does the amendment apply to the probate estate assets passing to the trust pursuant to Settlorrsquos will

(3) If the trust amendment was valid should the trust property be distributed to University

(4) If the trust amendment was not valid should the trust property be distributed to Settlorrsquos grandchild (her only heir) or held in further trust in accordance with the terms of the original trust instrument

DISCUSSION

Summary

A revocable trust is amendable even if the trust instrument does not expressly grant to the trust settlor a power to amend Both inter vivos trusts and amendments thereto are valid even though not executed in accordance with the requirements applicable to wills

Under the Uniform Testamentary Additions to Trusts Act a revocable trust may be amended at any time prior to the settlorrsquos death and the amendment applies to the disposition of assets conveyed to the trust pursuant to a will even if the will was executed prior to the date of the amendment

At Settlorrsquos death trust assets including probate assets passing to the trust under Settlorrsquos will would go to University if as is the case here the trust amendment was valid If the amendment was invalid the trust assets would continue to be held in further trust because there is no violation of the common law Rule Against Perpetuities

Point One(a) (30) Settlor retained the right to amend the inter vivos trust despite her failure to expressly reserve this power

At issue here is whether a retained power of revocation includes the power to amend sometimes referred to as the power to modify The Restatement (Second) of Trusts sect 331 cmt g provides that if a settlor has a power to revoke that retained power ordinarily includes a power to modify (amend) as well Comment g also notes that the power to amend includes both a power to withdraw trust assets and a power to ldquomodify the terms of the trustrdquo The Uniform Trust Code which provides that a power to revoke includes the power to amend is consistent with this view

38

Trusts and Future Interests Analysis

UNIF TRUST CODE sect 602 accord RESTATEMENT (THIRD) OF TRUSTS sect 63 cmt The theory is that even though a power to amend was not expressly retained by a settlor the goal of amendment assuming the power was not included in the power to revoke could easily be achieved by first revoking the trust and then creating a new trust with the same terms contemplated by the amendment To require this would put form over substance

Thus by expressly retaining the power to revoke the trust Settlor retained a power to amend the inter vivos trust despite her failure to expressly reserve this power

[NOTE Under the common law a trust is irrevocable unless the settlor expressly retains a power to revoke the trust Conversely under the Uniform Trust Code a trust is revocable unless the terms of the trust expressly provide otherwise See UNIF TRUST CODE sect 602 The Trust Codersquos position on revocation follows the minority view in the United States and is inconsistent with prior Restatements of Trusts (see Restatement (Second) of Trusts sect 330) Here the trust is revocable because Settlor expressly retained a power of revocation

The Uniform Trust Code has been adopted in 24 jurisdictions Alabama Arizona Arkansas District of Columbia Florida Kansas Maine Michigan Missouri Nebraska New Hampshire New Mexico North Carolina North Dakota Ohio Oregon Pennsylvania South Carolina Tennessee Utah Vermont Virginia West Virginia and Wyoming]

Point One(b) (10) Settlorrsquos amendment of the trust was valid despite her failure to have her signature to the trust amendment witnessed

Neither the common law nor state statutes require a trust instrument or an amendment to a trust instrument to be executed in accordance with the formalities prescribed for execution of a will Indeed an inter vivos trust that does not involve real estate can be created orally Under the Uniform Trust Code the only requirements for creating a valid inter vivos trust are intent the specification of beneficiaries and the designation of a trustee See UNIF TRUST CODE sect 402 accord RESTATEMENT (THIRD) OF TRUSTS sect 13

Here the amendment meets the requirements of both the Uniform Trust Code and the common law Thus the fact that Settlorrsquos signature was not witnessed when she signed the amendment to the trust does not make the amendment invalid

Point Two (20) Under the Uniform Testamentary Additions to Trusts Act a revocable trust may be amended at any time prior to the settlorrsquos death and the amendment applies to probate assets poured into the trust at the settlorrsquos death pursuant to the settlorrsquos will even when the will was executed prior to the date of the amendment

Historically property owned by an individual at her death passed to the individualrsquos heirs or to beneficiaries designated in a will executed with the formalities (writing signing witnessing) prescribed by state law However when a will devises property to the trustee of an inter vivos trust then the provisions of the trustmdashwhich may not have been executed in accordance with the formalities required for willsmdasheffectively determine who will receive the property Because of this possibility some early cases held that if an inter vivos trust was not executed with the same formalities required for a valid will then the trust was ineffective to dispose of probate assets poured into the trust at the settlorrsquos death pursuant to the settlorrsquos will

This line of cases has been overturned by the Uniform Testamentary Additions to Trusts Act (the Act) now Uniform Probate Code sect 2-511 Under the Act adopted in almost all

39

Trusts and Future Interest Analysis

jurisdictions a testamentary bequest to the trustee of an inter vivos trust established by the testator during his or her lifetime is valid if the trust is in writing it is identified in the testatorrsquos will and the trust instrument was executed before concurrently with or after the execution of the will Id The Act further specifies that such a bequest is valid even if the trust is amendable or revocable and that a later amendment applies to assets passing to the trust by a previously executed will

Thus because the trust amendment is valid its terms apply to assets received by Bank from Settlorrsquos estate

Point Three (10) If the trust amendment was valid then the trust assets including assets passing to the trust under Settlorrsquos will should go to University

Under the trust amendment all trust assets (including the assets of Settlorrsquos probate estate poured into the trust) pass to University The facts provide no basis for failing to comply with Settlorrsquos stated intentions

Point Four (30) If the trust amendment was invalid trust assets including assets received pursuant to Settlorrsquos will should be held in accordance with the terms of the original trust instrument because those terms do not violate the Rule Against Perpetuities

Under the dispositive terms of the original trust instrument Settlor created successive income interests in her surviving children and grandchildren with a remainder interest in her great-grandchildren Because the trust was revocable the period during which the common law Rule Against Perpetuities requires that interests vest (ie 21 years plus lives in being) began to run from the date Settlor no longer had a power of revocation (here her death) not the date on which the trust was created See JESSE DUKEMINIER STANLEY J JOHANSON JAMES LINDGREN amp ROBERT SITKOFF WILLS TRUSTS AND ESTATES 678 (7th ed 2005)

Under the common law Rule Against Perpetuities Settlorrsquos trust is thus valid At the time of Settlorrsquos death she was survived by no children one granddaughter and no great-grandchildren Because Settlor cannot have more children after her death the only income beneficiary of the trust is Settlorrsquos surviving granddaughter This granddaughter is the only person who can produce great-grandchildren of Settlor thus all great-grandchildren must of necessity be born during the lifetime of Settlorrsquos only surviving granddaughter who is a life in being The granddaughterrsquos interest vested at Settlorrsquos death and the great-grandchildrenrsquos interest will vest at the death of the granddaughter There is no need to wait the additional 21 years permitted under the Rule Thus under the common law and the statute given in the facts the nonvested interest in the great-grandchildren is valid

[NOTE Both modern wait-and-see statutes and the Uniform Statutory Rule Against Perpetuities upon which the statute in the facts is modeled provide that before using either reform to validate an otherwise invalid nonvested interest one should first determine if the nonvested interest violates the common law Rule If it does not then there is no need to reform This proposition which is applicable in all MEE user jurisdictions that have not simply abrogated the rule is tested by this problem]

40

NEGOTIABLE INSTRUMENTS ANALYSIS (Negotiable Instruments III IV V)

ANALYSIS

Legal Problems

(1)(a) What rights does a person in possession of a note that has been indorsed in blank by the payee have against the maker of the note

(1)(b) Which defenses may the maker of a note raise against a person entitled to enforce it who is not a holder in due course but is a transferee from a holder in due course

(2) What rights does a person entitled to enforce a note have against an indorser who transferred it for consideration with no warranties

(3) What rights does a person entitled to enforce a note have against a previous holder who transferred it as a gift without indorsing it

DISCUSSION

Summary

The niece is a holder of the note and is thus a person entitled to enforce it The chef the issuer of the note is obligated to pay it to the niece as the person entitled to enforce it The niece is not subject to any defense or claim of the chef relating to the improper repair of the oven because the niece has the rights of a holder in due course When the buyer bought the note from the repairman the buyer became a holder in due course of the note and thus took it free of any personal defenses the chef had against the repairman Even though the niece is not herself a holder in due course of the note the niece succeeded to the buyerrsquos rights as holder in due course and thus took free of the chefrsquos personal defenses

Because the chef refused to pay the note the niece can recover from the repairman on the repairmanrsquos obligation as indorser The niece cannot recover on the note against the buyer however because the buyer did not indorse the note (and thus incurred no indorserrsquos obligation) and the buyer did not receive any consideration for transfer of the note to the niece (and therefore made no transfer warranty)

[NOTE Although Article 9 of the Uniform Commercial Code governs the sale of promissory notes (a point that might be correctly noted by examinees) that Article does not determine the answer to any of the questions posed]

Point One(a) (20) The niece is the holder of the note and thus may enforce it against the chef who is the issuer of the note

The chef is the maker of the note and thus its issuer See UCC sectsect 3-103 3-105 The issuer of a note is obligated to pay it in accordance with its terms to a ldquoperson entitled to enforcerdquo it UCC sect 3-412 The niece is a ldquoperson entitled to enforcerdquo the note This is because the niece is the holder of the note and a holder of a note is a person entitled to enforce it UCC sect 3-301 The niece is the holder of the note because (i) the repairmanrsquos signature on the back of the note not

41

Negotiable Instruments Analysis

accompanied by words indicating a person to whom the note was made payable was a ldquoblank indorsementrdquo which had the effect of making the note a bearer instrument (ii) anyone in possession of a bearer instrument is a holder of it and (iii) the niece is in possession of the note See UCC sectsect 1-201(b)(21)(A) 3-204 and 3-205 Accordingly the chef has an obligation to the niece to pay the note in accordance with its terms and the niece may enforce that obligation

Point One(b) (40) The niece is not a holder in due course of the note but because she is a transferee from the buyer who was a holder in due course she has the same enforcement rights as the buyer Because the buyer as a holder in due course would have been able to enforce the note against the chef without being subject to defenses or claims arising from the improper repair the niece has the same rights and will not be subject to the chefrsquos defenses or claims about the repair

As noted in Point One(a) the chef has an obligation to the niece to pay the note in accordance with its terms However except against a person with the rights of a holder in due course the chef can raise any defenses or claims in recoupment that he would have if the claim on the note were an ordinary contract claim UCC sect 3-305 Thus except against a holder in due course the chef would be able to raise the improper repair as a defense or a claim in recoupment (a claim in response to the niecersquos claim)

But claims in recoupment and most defenses cannot be raised against a person with the rights of a holder in due course Against a holder in due course the chef can raise only the four ldquorealrdquo defenses listed in UCC sect 3-305(a)(1) (infancy duress lack of legal capacity or illegality that nullifies the obligation of the obligor under other law fraud in the factum discharge in insolvency proceedings) none of which is present here

The niece is not a holder in due course because she did not take the note for value See UCC sectsect 3-302(a)(2)(i) (criteria for holder in due course status) and 3-303(a) (definition of ldquovaluerdquo) But this does not mean that the niece is subject to the chefrsquos claim arising out of the improper repair The buyer was a holder in due course of the note because he took the note for value ($9500) in good faith and without notice of any facts that would have alerted him to the chefrsquos defense against the repairman UCC sect 3-302(a)(2) As a holder in due course the buyer owned the note free of the chefrsquos claim because that claim did not constitute a ldquorealrdquo defense UCC sect 3-305(b) When the buyer gave the note to the niece this constituted a ldquotransferrdquo of the note See UCC sect 3-203(a) When a note is transferred the transferee receives ldquoany right of the transferor to enforce the instrument including any right as a holder in due courserdquo UCC sect 3-203(b) Under this rule (also known as the ldquoshelter principlerdquo) the buyer transferred his freedom from the chefrsquos defenses to the niece and the niece can enforce the note free of the chefrsquos defenses

Point Two (20) Because the chef dishonored the note the niece can recover from the repairman on the repairmanrsquos obligation as indorser

The chefrsquos refusal to pay the note constituted dishonor See UCC sect 3-502 The repairman as an indorser of the note (see Point One(a)) incurred the obligations of an indorser under UCC sect 3-415(a) When a note has been dishonored one of the obligations of an indorser is to pay the amount of the note to a person entitled to enforce it Therefore the repairman is liable for the amount of the note to the niece a person entitled to enforce the note (so long as the niece gives proper notice of dishonor to the repairman)

42

Negotiable Instruments Analysis

[NOTE Because the repairman indorsed the note without warranties there are no transfer warranties UCC sect 3-416 cmt 5]

Point Three (20) The niece cannot recover on the note against the buyer as either indorser or warrantor because the buyer did not indorse the note and did not receive consideration for transferring the note to the niece

The buyer did not indorse the note and therefore did not incur the obligation of an indorser to pay the note upon dishonor

The niece cannot recover from the buyer under a transfer warranty theory because transfer warranties are made only by a person ldquowho transfers an instrument for considerationrdquo Here the buyer gave the instrument to the niece as a gift So the buyer made no transfer warranty UCC sect 3-416(a) Therefore the niece cannot recover from the buyer on that theory

43

National Conference of Bar Examiners 302 South Bedford Street | Madison WI 53703-3622 Phone 608-280-8550 | Fax 608-280-8552 | TDD 608-661-1275

wwwncbexorg e-mail contactncbexorg

  • Contents
  • Preface
  • Description of the MEE
  • Instructions
  • February 2013 Questions
    • Real Property Question
    • Contracts Question
    • Constitutional Law Question
    • Secured Transactions Question
    • Federal Civil Procedure Question
    • Agency Question
    • Evidence Question
    • Trusts and Future Interests Question
    • Negotiable Instruments Question
      • February 2013 Analyses
        • Real Property Analysis
        • Contracts Analysis
        • Constitutional Law Analysis
        • Secured Transactions Analysis
        • Federal Civil Procedure Analysis
        • Agency Analysis
        • Evidence Analysis
        • Trusts and Future Interests Analysis
        • Negotiable Instruments Analysis
            • ltlt ASCII85EncodePages false AllowTransparency false AutoPositionEPSFiles true AutoRotatePages None Binding Left CalGrayProfile (Dot Gain 20) CalRGBProfile (sRGB IEC61966-21) CalCMYKProfile (US Web Coated 050SWOP051 v2) sRGBProfile (sRGB IEC61966-21) CannotEmbedFontPolicy Error CompatibilityLevel 14 CompressObjects Tags CompressPages true ConvertImagesToIndexed true PassThroughJPEGImages true CreateJobTicket false DefaultRenderingIntent Default DetectBlends true DetectCurves 00000 ColorConversionStrategy CMYK DoThumbnails false EmbedAllFonts true EmbedOpenType false ParseICCProfilesInComments true EmbedJobOptions true DSCReportingLevel 0 EmitDSCWarnings false EndPage -1 ImageMemory 1048576 LockDistillerParams false MaxSubsetPct 100 Optimize true OPM 1 ParseDSCComments true ParseDSCCommentsForDocInfo true PreserveCopyPage true PreserveDICMYKValues true PreserveEPSInfo true PreserveFlatness true PreserveHalftoneInfo false PreserveOPIComments true PreserveOverprintSettings true StartPage 1 SubsetFonts true TransferFunctionInfo Apply UCRandBGInfo Preserve UsePrologue false ColorSettingsFile () AlwaysEmbed [ true ] NeverEmbed [ true ] AntiAliasColorImages false CropColorImages true ColorImageMinResolution 300 ColorImageMinResolutionPolicy OK DownsampleColorImages true ColorImageDownsampleType Bicubic ColorImageResolution 300 ColorImageDepth -1 ColorImageMinDownsampleDepth 1 ColorImageDownsampleThreshold 150000 EncodeColorImages true ColorImageFilter DCTEncode AutoFilterColorImages true ColorImageAutoFilterStrategy JPEG ColorACSImageDict ltlt QFactor 015 HSamples [1 1 1 1] VSamples [1 1 1 1] gtgt ColorImageDict ltlt QFactor 015 HSamples [1 1 1 1] VSamples [1 1 1 1] gtgt JPEG2000ColorACSImageDict ltlt TileWidth 256 TileHeight 256 Quality 30 gtgt JPEG2000ColorImageDict ltlt TileWidth 256 TileHeight 256 Quality 30 gtgt AntiAliasGrayImages false CropGrayImages true GrayImageMinResolution 300 GrayImageMinResolutionPolicy OK DownsampleGrayImages true GrayImageDownsampleType Bicubic GrayImageResolution 300 GrayImageDepth -1 GrayImageMinDownsampleDepth 2 GrayImageDownsampleThreshold 150000 EncodeGrayImages true GrayImageFilter DCTEncode AutoFilterGrayImages true GrayImageAutoFilterStrategy JPEG GrayACSImageDict ltlt QFactor 015 HSamples [1 1 1 1] VSamples [1 1 1 1] gtgt GrayImageDict ltlt QFactor 015 HSamples [1 1 1 1] VSamples [1 1 1 1] gtgt JPEG2000GrayACSImageDict ltlt TileWidth 256 TileHeight 256 Quality 30 gtgt JPEG2000GrayImageDict ltlt TileWidth 256 TileHeight 256 Quality 30 gtgt AntiAliasMonoImages false CropMonoImages true MonoImageMinResolution 1200 MonoImageMinResolutionPolicy OK DownsampleMonoImages true MonoImageDownsampleType Bicubic MonoImageResolution 1200 MonoImageDepth -1 MonoImageDownsampleThreshold 150000 EncodeMonoImages true MonoImageFilter CCITTFaxEncode MonoImageDict ltlt K -1 gtgt AllowPSXObjects false CheckCompliance [ None ] PDFX1aCheck false PDFX3Check false PDFXCompliantPDFOnly false PDFXNoTrimBoxError true PDFXTrimBoxToMediaBoxOffset [ 000000 000000 000000 000000 ] PDFXSetBleedBoxToMediaBox true PDFXBleedBoxToTrimBoxOffset [ 000000 000000 000000 000000 ] PDFXOutputIntentProfile () PDFXOutputConditionIdentifier () PDFXOutputCondition () PDFXRegistryName () PDFXTrapped False CreateJDFFile false Description ltlt ARA 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 BGR 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 CHS ltFEFF4f7f75288fd94e9b8bbe5b9a521b5efa7684002000410064006f006200650020005000440046002065876863900275284e8e9ad88d2891cf76845370524d53705237300260a853ef4ee54f7f75280020004100630072006f0062006100740020548c002000410064006f00620065002000520065006100640065007200200035002e003000204ee553ca66f49ad87248672c676562535f00521b5efa768400200050004400460020658768633002gt CHT ltFEFF4f7f752890194e9b8a2d7f6e5efa7acb7684002000410064006f006200650020005000440046002065874ef69069752865bc9ad854c18cea76845370524d5370523786557406300260a853ef4ee54f7f75280020004100630072006f0062006100740020548c002000410064006f00620065002000520065006100640065007200200035002e003000204ee553ca66f49ad87248672c4f86958b555f5df25efa7acb76840020005000440046002065874ef63002gt CZE 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 DAN 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 DEU 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 ESP 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 ETI 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 FRA 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 GRE ltFEFF03a703c103b703c303b903bc03bf03c003bf03b903ae03c303c403b5002003b103c503c403ad03c2002003c403b903c2002003c103c503b803bc03af03c303b503b903c2002003b303b903b1002003bd03b1002003b403b703bc03b903bf03c503c103b303ae03c303b503c403b5002003ad03b303b303c103b103c603b1002000410064006f006200650020005000440046002003c003bf03c5002003b503af03bd03b103b9002003ba03b103c42019002003b503be03bf03c703ae03bd002003ba03b103c403ac03bb03bb03b703bb03b1002003b303b903b1002003c003c103bf002d03b503ba03c403c503c003c903c403b903ba03ad03c2002003b503c103b303b103c303af03b503c2002003c503c803b703bb03ae03c2002003c003bf03b903cc03c403b703c403b103c2002e0020002003a403b10020005000440046002003ad03b303b303c103b103c603b1002003c003bf03c5002003ad03c703b503c403b5002003b403b703bc03b903bf03c503c103b303ae03c303b503b9002003bc03c003bf03c103bf03cd03bd002003bd03b1002003b103bd03bf03b903c703c403bf03cd03bd002003bc03b5002003c403bf0020004100630072006f006200610074002c002003c403bf002000410064006f00620065002000520065006100640065007200200035002e0030002003ba03b103b9002003bc03b503c403b103b303b503bd03ad03c303c403b503c103b503c2002003b503ba03b403cc03c303b503b903c2002egt HEB 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 HRV (Za stvaranje Adobe PDF dokumenata najpogodnijih za visokokvalitetni ispis prije tiskanja koristite ove postavke Stvoreni PDF dokumenti mogu se otvoriti Acrobat i Adobe Reader 50 i kasnijim verzijama) HUN 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 ITA 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 JPN ltFEFF9ad854c18cea306a30d730ea30d730ec30b951fa529b7528002000410064006f0062006500200050004400460020658766f8306e4f5c6210306b4f7f75283057307e305930023053306e8a2d5b9a30674f5c62103055308c305f0020005000440046002030d530a130a430eb306f3001004100630072006f0062006100740020304a30883073002000410064006f00620065002000520065006100640065007200200035002e003000204ee5964d3067958b304f30533068304c3067304d307e305930023053306e8a2d5b9a306b306f30d530a930f330c8306e57cb30818fbc307f304c5fc59808306730593002gt KOR ltFEFFc7740020c124c815c7440020c0acc6a9d558c5ec0020ace0d488c9c80020c2dcd5d80020c778c1c4c5d00020ac00c7a50020c801d569d55c002000410064006f0062006500200050004400460020bb38c11cb97c0020c791c131d569b2c8b2e4002e0020c774b807ac8c0020c791c131b41c00200050004400460020bb38c11cb2940020004100630072006f0062006100740020bc0f002000410064006f00620065002000520065006100640065007200200035002e00300020c774c0c1c5d0c11c0020c5f40020c2180020c788c2b5b2c8b2e4002egt LTH 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 LVI 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 NLD (Gebruik deze instellingen om Adobe PDF-documenten te maken die zijn geoptimaliseerd voor prepress-afdrukken van hoge kwaliteit De gemaakte PDF-documenten kunnen worden geopend met Acrobat en Adobe Reader 50 en hoger) NOR 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 POL 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 PTB ltFEFF005500740069006c0069007a006500200065007300730061007300200063006f006e00660069006700750072006100e700f50065007300200064006500200066006f0072006d00610020006100200063007200690061007200200064006f00630075006d0065006e0074006f0073002000410064006f0062006500200050004400460020006d00610069007300200061006400650071007500610064006f00730020007000610072006100200070007200e9002d0069006d0070007200650073007300f50065007300200064006500200061006c007400610020007100750061006c00690064006100640065002e0020004f007300200064006f00630075006d0065006e0074006f00730020005000440046002000630072006900610064006f007300200070006f00640065006d0020007300650072002000610062006500720074006f007300200063006f006d0020006f0020004100630072006f006200610074002000650020006f002000410064006f00620065002000520065006100640065007200200035002e0030002000650020007600650072007300f50065007300200070006f00730074006500720069006f007200650073002egt RUM 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 RUS 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 SKY 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 SLV 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 SUO 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 SVE ltFEFF0041006e007600e4006e00640020006400650020006800e4007200200069006e0073007400e4006c006c006e0069006e006700610072006e00610020006f006d002000640075002000760069006c006c00200073006b006100700061002000410064006f006200650020005000440046002d0064006f006b0075006d0065006e007400200073006f006d002000e400720020006c00e4006d0070006c0069006700610020006600f60072002000700072006500700072006500730073002d007500740073006b00720069006600740020006d006500640020006800f600670020006b00760061006c0069007400650074002e002000200053006b006100700061006400650020005000440046002d0064006f006b0075006d0065006e00740020006b0061006e002000f600700070006e00610073002000690020004100630072006f0062006100740020006f00630068002000410064006f00620065002000520065006100640065007200200035002e00300020006f00630068002000730065006e006100720065002egt TUR 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 UKR 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 ENU (Use these settings to create Adobe PDF documents best suited for high-quality prepress printing Created PDF documents can be opened with Acrobat and Adobe Reader 50 and later) gtgt Namespace [ (Adobe) (Common) (10) ] OtherNamespaces [ ltlt AsReaderSpreads false CropImagesToFrames true ErrorControl WarnAndContinue FlattenerIgnoreSpreadOverrides false IncludeGuidesGrids false IncludeNonPrinting false IncludeSlug false Namespace [ (Adobe) (InDesign) (40) ] OmitPlacedBitmaps false OmitPlacedEPS false OmitPlacedPDF false SimulateOverprint Legacy gtgt ltlt AddBleedMarks false AddColorBars false AddCropMarks false AddPageInfo false AddRegMarks false ConvertColors ConvertToCMYK DestinationProfileName () DestinationProfileSelector DocumentCMYK Downsample16BitImages true FlattenerPreset ltlt PresetSelector MediumResolution gtgt FormElements false GenerateStructure false IncludeBookmarks false IncludeHyperlinks false IncludeInteractive false IncludeLayers false IncludeProfiles false MultimediaHandling UseObjectSettings Namespace [ (Adobe) (CreativeSuite) (20) ] PDFXOutputIntentProfileSelector DocumentCMYK PreserveEditing true UntaggedCMYKHandling LeaveUntagged UntaggedRGBHandling UseDocumentProfile UseDocumentBleed false gtgt ]gtgt setdistillerparamsltlt HWResolution [2400 2400] PageSize [612000 792000]gtgt setpagedevice

Page 10: February 2013 MEE Questions and Analyses

SECURED TRANSACTIONS QUESTION

On June 1 a bicycle retailer sold two bicycles to a man for a total purchase price of $1500 The man made a $200 down payment and agreed to pay the balance in one year The man also signed a security agreement that identified the bicycles as collateral for the unpaid purchase price and provided that the man ldquoshall not sell or dispose of the collateral until the balance owed is paid in fullrdquo The retailer never filed a financing statement reflecting this security interest

The man had bought the bicycles for him and his girlfriend to use on vacation However shortly after he bought the bicycles the man and his girlfriend broke up The man has never used the bicycles

On August 1 the man sold one of the bicycles at a garage sale to a buyer who paid the man $400 for the bicycle The buyer bought the bicycle to ride for weekend recreation

On October 1 the man gave the other bicycle to his friend as a birthday present The friend began using the bicycle for morning exercise

Neither the buyer nor the friend had any knowledge of the manrsquos dealings with the retailer

1 Does the buyer own the bicycle free of the retailerrsquos security interest Explain

2 Does the friend own the bicycle free of the retailerrsquos security interest Explain

6

FEDERAL CIVIL PROCEDURE QUESTION _____

Mother and Son who are both adults are citizens and residents of State A Mother owned an expensive luxury car valued in excess of $100000 Son borrowed Motherrsquos car to drive to a store in State A As Son approached a traffic light that had just turned yellow he carefully braked and brought the car to a complete stop Driver who was following immediately behind him failed to stop and rear-ended Motherrsquos car which was damaged beyond repair Son was seriously injured Driver is a citizen of State B

Son sued Driver in the United States District Court for the District of State A alleging that she was negligent in the operation of her vehicle Son sought damages in excess of $75000 for his personal injuries exclusive of costs and interest In her answer Driver alleged that Son was contributorily negligent in the operation of Motherrsquos car She further alleged that the brake lights on Motherrsquos car were burned out and that Motherrsquos negligent failure to properly maintain the car was a contributing cause of the accident

Following a trial on the merits in Sonrsquos case against Driver the jury answered the following special interrogatories

Do you find that Driver was negligent in the operation of her vehicle Yes

Do you find that Son was negligent in the operation of Motherrsquos car No

Do you find that Mother negligently failed to ensure that the brake lights on her car were in proper working order Yes

The judge then entered a judgment in favor of Son against Driver Driver did not appeal

Two months later Mother sued Driver in the United States District Court for the District of State A alleging that Driverrsquos negligence in the operation of her vehicle destroyed Motherrsquos luxury car Mother sought damages in excess of $75000 exclusive of costs and interest

State A follows the same preclusion principles that federal courts follow in federal-question cases

1 Is Motherrsquos claim against Driver barred by the judgment in Son v Driver Explain

2 Does the juryrsquos conclusion in Son v Driver that Mother had negligently failed to maintain the brake lights on her car preclude Mother from litigating that issue in her subsequent suit against Driver Explain

3 Does the juryrsquos conclusion in Son v Driver that Driver was negligent preclude Driver from litigating that issue in the Mother v Driver lawsuit Explain

7

_____

AGENCY QUESTION

Over 5000 individuals in the United States operate hot-air balloon businesses A hot-air balloon has four key components the balloon that holds the heated air the basket that houses the riders the propane burner that heats the air in the balloon and the propane storage tanks

The owner of a hot-air balloon business recently notified several basket and burner manufacturers that she or her agent might be contacting them to purchase baskets or burners The owner did not specifically name any person as her agent Basket and burner manufacturers regularly receive such notices from hot-air balloon operators Such notices typically include no restrictions on the types of baskets or burners agents might purchase for their principals

The owner then retained an agent to acquire baskets burners and fuel tanks from various manufacturers The owner authorized the agent to buy only (a) baskets made of woven wicker (not aluminum) (b) burners that use a unique ldquowhisper technologyrdquo (so as not to scare livestock when the balloon sails over farmland) and (c) propane fuel tanks

The agent then entered into three transactions with manufacturers all of whom had no prior dealings with either the owner or the agent

(1) The agent and a large manufacturer of both wicker and aluminum baskets signed a contract for the purchase of four aluminum baskets for a total cost of $60000 The agent never told the manufacturer that he represented the owner or any other principal The contract listed the agent as the buyer and listed the ownerrsquos address as the delivery address but did not indicate that the address was that of the owner rather than the agent When the baskets were delivered to the owner she learned for the first time that the agent had contracted to buy aluminum not wicker baskets The owner immediately rejected the baskets and returned them to the manufacturer Neither the owner nor the agent has paid the basket manufacturer for them

(2) The agent contacted a burner manufacturer and told him that the agent represented a well-known hot-air balloon operator who wanted to purchase burners The agent did not disclose the ownerrsquos name The agent and the burner manufacturer signed a contract for the purchase of four burners that did not have ldquowhisper technologyrdquo for a total price of $70000 The burner contract like the basket contract listed the ownerrsquos address for delivery but did not disclose whose address it was The burners were delivered to the ownerrsquos business and the owner discovered that the agent had ordered the wrong kind of burners The owner rejected the burners and returned them to the manufacturer Neither the owner nor the agent has paid the burner manufacturer for the burners

(3) The agent contracted with a solar cell manufacturer to make three cells advertised as ldquostrong enough to power all your ballooning needsrdquo The agent did not tell the manufacturer that he was acting on behalf of any other person One week after the cells were delivered to the agent he took them to the owner who installed them and discovered that she could save a lot of money using solar cells instead of propane to power her balloons The owner decided to keep the solar cells but she has not paid the manufacturer for them

8

Agency Question

Assume that the rejection of the baskets and the burners and the failure to pay for the solar cells constitute breach of the relevant contracts

1 Is the owner liable to the basket manufacturer for breach of the contract for the aluminum baskets Is the agent liable Explain

2 Is the owner liable to the burner manufacturer for breach of the contract for the burners Is the agent liable Explain

3 Is the owner liable to the solar cell manufacturer for breach of the contract for the solar cells Is the agent liable Explain (Do not address liability based upon restitution or unjust enrichment)

9

EVIDENCE QUESTION _____

A woman who owns a motorized scooter brought her scooter to a mechanic for routine maintenance service As part of the maintenance service the mechanic inspected the braking system on the scooter As soon as the mechanic finished inspecting and servicing the scooter he sent the woman a text message to her cell phone that read ldquoJust finished your service When you pick up your scooter you need to schedule a follow-up brake repair Wersquoll order the partsrdquo

The woman read the mechanicrsquos text message and returned the next day to pick up her scooter As the woman was wheeling her scooter out of the shop she saw the mechanic working nearby and asked ldquoIs my scooter safe to ride for a whilerdquo The mechanic responded by giving her a thumbs-up The woman waved and rode away on the scooter

One week later while the woman was riding her scooter a pedestrian stepped off the curb into a crosswalk and the woman collided with him causing the pedestrian severe injuries The woman had not had the scooterrsquos brakes repaired before the accident

The pedestrian has sued the woman for damages for his injuries resulting from the accident The pedestrian has alleged that (1) the woman lost control of the scooter due to its defective brakes (2) the woman knew that the brakes needed repair and (3) it was negligent for the woman to ride the scooter knowing that its brakes needed to be repaired

The woman claims that the brakes on the scooter worked perfectly and that the accident happened because the pedestrian stepped into the crosswalk without looking and the woman had no time to stop The woman the pedestrian and the mechanic will testify at the upcoming trial

The pedestrian has proffered an authenticated copy of the mechanicrsquos text message to the woman

The woman plans to testify that she asked the mechanic ldquoIs my scooter safe to ride for a whilerdquo and that he gave her a thumbs-up in response

The evidence rules in this jurisdiction are identical to the Federal Rules of Evidence

Analyze whether each of these items of evidence is relevant and admissible at trial

1 The authenticated copy of the mechanicrsquos text message

2 The womanrsquos testimony that she asked the mechanic ldquoIs my scooter safe to ride for a whilerdquo and

3 The womanrsquos testimony describing the mechanicrsquos thumbs-up

10

TRUSTS AND FUTURE INTERESTS QUESTION

Ten years ago Settlor validly created an inter vivos trust and named Bank as trustee The trust instrument provided that Settlor would receive all of the trust income during her lifetime The trust instrument further provided that

Upon Settlorrsquos death the trust income shall be paid in equal shares to Settlorrsquos surviving children for their lives Upon the death of the last surviving child the trust income shall be paid in equal shares to Settlorrsquos then-living grandchildren for their lives Upon the death of the survivor of Settlorrsquos children and grandchildren the trust corpus shall be distributed in equal shares to Settlorrsquos then-living great-grandchildren

The trust instrument expressly specified that the trust was revocable but it was silent regarding whether Settlor could amend the trust instrument

Immediately after creating the trust Settlor validly executed a will leaving her entire estate to Bank as trustee of her inter vivos trust to ldquohold in accordance with the terms of the trustrdquo

Five years ago Settlor signed an amendment to the inter vivos trust The amendment changed the disposition of the remainder interest specifying that all trust assets ldquoshall be paid upon Settlorrsquos death to Universityrdquo Settlorrsquos signature on this amendment was not witnessed

A state statute provides that any trust interest that violates the common law Rule Against Perpetuities ldquois nonetheless valid if the nonvested interest in the trust actually vests or fails to vest either (a) within 21 years of lives in being at the creation of the nonvested interest or (b) within 90 years of its creationrdquo

Recently Settlor died leaving a probate estate of $200000 She was survived by no children one granddaughter (who would be Settlorrsquos only heir) and no great-grandchildren The granddaughter has consulted your law firm and has raised four questions regarding this trust

1 Was Settlorrsquos amendment of the inter vivos trust valid Explain

2 Assuming that the trust amendment was valid do its provisions apply to Settlorrsquos probate assets Explain

3 Assuming that the trust amendment was valid how should trust assets be distributed Explain

4 Assuming that the trust amendment was invalid how should trust assets be distributed Explain

11

NEGOTIABLE INSTRUMENTS QUESTION

A chef entered into a contract with a repairman pursuant to which the repairman agreed to repair the chefrsquos commercial oven for $10000 The repairman agreed to accept as payment a negotiable promissory note for $10000 payable two months after its issuance

After the repairman worked on the oven the chef gave him a $10000 note as payment for the work As agreed the note was signed by the chef as maker was payable to the order of the repairman was payable in two months and fulfilled all criteria for negotiability

The next day the repairman sold the note to a buyer for $9500 To effectuate the sale the repairman wrote ldquono warrantiesrdquo on the back of the note signed his name immediately below that and handed the note to the buyer The buyer bought the note in good faith and without knowledge of any facts relating to the work that the repairman had performed for the chef

Later the buyer gave the note to his niece as a gift To effectuate the gift the buyer handed the note to the niece but did not indorse it

Shortly thereafter the chef discovered that the repair work had been done improperly and the oven still did not function correctly The chef tried repeatedly to get the repairman to return to correct the repair work but the repairman ignored all the chefrsquos calls

On the notersquos due date the niece contacted the chef and demanded that he pay the amount of the note to her The chef refused and told the niece that he would not pay the note because the repairman did not properly repair the oven

1 What are the niecersquos rights against the chef Explain

2 What are the niecersquos rights against the repairman Explain

3 What are the niecersquos rights against the buyer Explain

12

February 2013 MEE

ANALYSES

Real Property Contracts

Constitutional Law Secured Transactions

Federal Civil Procedure Agency

Evidence Trusts and Future Interests

Negotiable Instruments

REAL PROPERTY ANALYSIS (Real Property ID1a 4 amp 5)

ANALYSIS

Legal Problems

(1) Does the tenant have a defense to the landlordrsquos action for unpaid rent based on constructive eviction

(2) Does the tenant have a defense to the landlordrsquos action for unpaid rent based on the tenantrsquos surrender of the premises

(3) What if anything may the landlord recover from the tenant for the period after the tenant vacated the building

DISCUSSION

Summary

Under the common law the tenant does not have a defense to the landlordrsquos action for unpaid rent based on constructive eviction Constructive eviction is based on the tenant proving that (1) the landlord breached a duty to the tenant (2) the breach caused a loss by the tenant of the substantial use and enjoyment of the premises (3) the tenant gave the landlord adequate notice and opportunity to repair and (4) the tenant vacated the leased premises Here there was no constructive eviction because although the tenant vacated and gave the landlord adequate notice the landlord breached no express or implied duty to the tenant to repair the premises

The tenant does not have a defense based on the landlordrsquos acceptance of his surrender of the premises a landlordrsquos retention of keys does not constitute an acceptance of the tenantrsquos surrender unless the landlord so intended and here the landlordrsquos statements to the tenant at the time of the surrender of the keys do not evidence the intent to accept the tenantrsquos surrender

Under the common law a landlord has no duty to mitigate damages but also cannot sue for rents due in the future Under this approach the landlord can sue only for past-due rents Using this approach on November 1 the landlord could recover all the rent past due (ie rent for September and October) but could not recover for rents due in the future However some courts have authorized recovery for future rent minus the fair market rental value of the premises It is thus possible that the landlord could recover damages equal to the amount of rent due from September 1 to the end of the six-year lease term ($180000) minus the propertyrsquos fair-market rental value over that same period

Point One (45) The tenant was not constructively evicted because the landlord had no duty to repair the commercial premises that were the subject of the lease

The landlord and the tenant entered into a term-of-years lease because the lease specified both a beginning and an ending date HERBERT HOVENKAMP amp SHELDON F KURTZ THE LAW OF

PROPERTY 256 (5th ed 2001) Although a term-of-years lease normally cannot be terminated by the tenant prior to the end of the term a tenant may terminate a term-of-years lease if the tenant

15

Real Property Analysis

is constructively evicted See id at 286ndash88 Typically as here a claim of constructive eviction is made as a defense to a landlordrsquos action for damages or unpaid rent

In order to establish a constructive eviction the tenant must prove that the landlord breached a duty to the tenant such as a duty to repair and that the landlordrsquos breach caused a loss of the substantial use and enjoyment of the premises The tenant must also show that he gave the landlord notice adequate to permit the landlord to meet his duty to the tenant and that the tenant vacated the leased premises Id see also JOHN G SPRANKLING UNDERSTANDING

PROPERTY LAW sect 1704 (2d ed 2007) Under the common law there was no implied duty on the part of a landlord to repair

leased premises such a duty arose only if expressly set forth in the lease SPRANKLING supra sect 1702[B] Here the written lease contained no term requiring the landlord to repair the air-conditioning Even if the conversation created a lease term that the building had air-conditioning that itself should not create a duty for the landlord to repair it

Over the past several decades courts have generally implied a duty to repair in residential leases either as part of a revised constructive eviction doctrine or based on an implied warranty of habitability JOSEPH W SINGER PROPERTY 469ndash70 (3d ed 2010) This shift has been justified based on the economic disparity between the typical landlord and tenant as well as the fact that residential tenants generally lack both the authority to authorize repairs to common areas of a building and the incentive to make repairs that will ultimately benefit the landlord

However courts have been more reluctant to imply a duty to repair in commercial leases a context in which the tenant is often a valuable business and in a better position to assess and make repairs than is the landlord But see eg Davidow v Inwood North Professional Group 747 SW2d 373 (Tex 1988) When courts have implied a duty to repair in a commercial lease it is typically when the repair has been mandated by public authorities and involves work so substantial that it would not ordinarily fall within the tenantrsquos common law repair duty andor the value of the repair would primarily inure to the landlordrsquos reversionary interest See Brown v Green 884 P2d 55 (Cal 1994) Eugene L Grant et al The Tenant as Terminator Constructive Eviction in Modern Commercial Leases 2 THE COMMERCIAL PROPERTY LEASE ch 15 (ABA 1997) Some courts have also permitted constructive eviction claims by commercial tenants of office buildings based on repairs required in common areas of the building See id Echo Consulting Services Inc v North Conway Bank 669 A2d 227 (NH 1995)

Here the tenant is the owner of a valuable manufacturing operation and is the exclusive occupant of the building the repair has not been mandated by public authorities and the repair is not structural To the contrary the repair involves a feature of the building of unusual importance in the tenantrsquos manufacturing operation and the tenant is likely far more knowledgeable than the landlord about the air-conditioning specifications necessary for the manufacture of the tenantrsquos product

Based on these facts it is unlikely that a court will find that the tenant in this case was constructively evicted Although the tenant can show that he gave adequate notice to the landlord of the air-conditioning malfunction and vacated the premises the lease was commercial and it did not contain any promises or covenants by the landlord except a covenant of quiet enjoyment a covenant of quiet enjoyment does not entail any repair obligations

[NOTE An examineersquos conclusion is less important than his or her demonstrated awareness of the elements of constructive eviction and the need to imply a repair duty for such a defense to be viable here Although the implied warranty of habitability is not available to this tenant Texas Minnesota and Massachusetts imply a warranty of suitability in commercial leases in limited circumstances and an examinee might argue that this warranty should apply

16

Real Property Analysis

here If an examinee concludes that this warranty applies he or she should discuss the other requirements for constructive eviction

If the examinee wrongly concludes that the first element for a constructive eviction has been met the examinee will then have to discuss the remaining three elements in order to conclude that the tenant can claim constructive eviction The tenant would have a strong argument that the second elementmdashsubstantial interference with the use and enjoyment of the premisesmdashalso is met As indicated above the landlord was aware that a functioning air-conditioning system was vital to the tenantrsquos manufacturing operations The facts further indicate that the system had failed three times in the past few months The landlord may try to argue that the malfunctions did not substantially interfere with the tenantrsquos use of the premises because the malfunctions caused the temperature to climb above 81 degrees for only a short period of timemdash 3 hours 6 hours and 10 hours respectivelymdashon each occasion The tenant will argue however that the landlord was aware that the tenantrsquos manufacturing operations could tolerate temperatures above 81 degrees for no more than 6 hours The final malfunction exceeded that limit destroying $150000 worth of the tenantrsquos products

The tenant would also have a strong argument that the third element is met notice and opportunity to cure The tenant notified the landlord of the problem immediately upon the systemrsquos first malfunction and did so again when it malfunctioned a second time and then a third time The landlord might argue that there was insufficient time to cure the problem because the system corrected itself within a few hours on the first and second times Although the malfunction lasted more than 10 hours the third time the landlord might argue that the time period was insufficient to get a repair person on the premises A court would be likely to find this argument unpersuasive however because the landlord could have attempted to correct the problem after the first and second malfunctions

Assuming that the landlord was given sufficient notice and opportunity to cure a court would be likely to conclude that the tenant also satisfied the final element of vacating the premises within a reasonable time The landlord might argue that the tenant remained in the premises for almost four months after the air conditioning first failed which would suggest that the problem was not so severe as to have constructively evicted the tenant The tenant will argue however that he gave the landlord three months to cure the problem after the first two malfunctions threatened (but did not actually harm) his operations The tenant then moved out shortly after the final malfunction caused temperatures to exceed the tolerance levels of his manufacturing operations]

Point Two (10) The landlord did not accept the tenantrsquos surrender of the lease

When a tenant wrongfully moves from leased premises with the intent to terminate the lease the landlord may either accept the tenantrsquos surrender of the premises and terminate the lease or hold the tenant to the terms of the lease See HOVENKAMP amp KURTZ supra at 295ndash96 Here the tenantrsquos only basis for the claim that the landlord accepted his surrender is the landlordrsquos retention of the keys Many courts have considered whether a landlordrsquos retention of keys delivered by a tenant constitutes acceptance of surrender The weight of the case law holds that retention of the keys alone does not constitute acceptance of surrender without other evidence showing that the landlord intended to accept the surrender See generally 49 AM JUR 2d Landlord and Tenant sect 213

Here the landlordrsquos note saying ldquoI repeat the air-conditioning is not my problem You have leased the building and you should fix itrdquo strongly suggests that the landlord did not intend

17

Some courts have rejected the no-mitigation-of-damages rule based on efficiency concerns and societyrsquos interest in assuring that resources remain in the stream of commerce rather than lying vacant see id at 464ndash65 and allow landlords to sue tenants who have wrongfully terminated a lease for damages equal to the difference between the unpaid rent due under the lease and the propertyrsquos fair market rental value Other courts have abandoned the no-recovery-for-future-rent rule These courts responding to the fact that a tenant may well disappear or be judgment-proof by the time a lease term is concluded have allowed a landlord to collect damages equal to the value of rent over the entire lease term minus the propertyrsquos fair rental value when a tenant has wrongfully terminated a lease and unequivocally shown an intention not to return to the premises or pay future rent Under this approach a landlord receives approximately the same amount he would have received were there a duty to mitigate damages See Sagamore Corp v Willcutt 180 A 464 (Conn 1935)

Real Property Analysis

to accept the tenantrsquos surrender The tenant might argue that the landlordrsquos failure to make a similar statement when the keys were sent to her a second time and she retained them evidences a change of heart However it is likely that a court would find that the landlordrsquos retention of the keys represented a decision to safeguard the keys not to accept the tenantrsquos surrender

[NOTE An examinee should receive credit for arguing the other way with a well-reasoned argument]

Point Three (45) Under the common law the landlord had no duty to mitigate damages Additionally a landlord was not entitled to recover unpaid rents due in the future but was only entitled to recover rents in arrears at the time of the commencement of the suit Applying the common law here the landlord could recover $5000 the amount of rents due at the commencement of the suit ($2500 for September and the same for October) Today some courts allow the landlord under certain circumstances to sue the tenant for damages (not rent) equal to the difference if any between the unpaid promised rent for the balance of the term (here $175000) and the propertyrsquos fair rental value for the balance of the term

Under the common law because a lease was viewed as a conveyance instead of a contract a landlord had no duty to mitigate damages resulting from a tenantrsquos wrongful termination of a lease A landlord could thus recover the full value of rents that were due and unpaid at the time of the suit However under the common law a landlord could not sue a tenant for rents due in the future because there was always a possibility that the tenant might pay the rent when it was due See SINGER supra at 462 Thus using the common law approach on November 1 the landlord could only recover the full value of the two monthsrsquo rent actually due and unpaid ie $5000 for September and October

Here because the tenant returned the keys to the landlord and said ldquoI will not be returning to the building or making further rent paymentsrdquo the landlord could establish abandonment and an intention not to return It is thus possible that the landlord might recover damages in the amount of $5000 (for the months of September and October) plus the present value of $175000 minus the fair market rental value of the property over the remaining months of the lease

18

CONTRACTS ANALYSIS ____ (Contracts II IVE)

ANALYSIS

Legal Problems

(1) What was the legal effect of the sailorrsquos October 31 letter to the builder

(2)(a) What was the legal effect of the builderrsquos November 25 response to the sailorrsquos October 31 letter

(2)(b) What was the legal effect of the sailorrsquos refusal to take and pay for the boat on December 15

DISCUSSION

Summary

This is a sale of goods governed by the Uniform Commercial Code Because the sailor had reasonable grounds for insecurity about the builderrsquos ability to deliver the boat in a timely manner when the sailor learned about the strike on October 31 the sailor was legally justified in sending the letter to the builder seeking adequate assurance of the builderrsquos performance pursuant to the contract The builderrsquos failure to provide such assurance within a reasonable time operated as a repudiation of the contract However the builder was free to retract the repudiation before the sailor either cancelled the contract or materially changed position in reliance on the builderrsquos repudiation The builder retracted the repudiation when he informed the sailor that the workers were back and that the boat would be delivered by the date stipulated in the partiesrsquo contract Because the sailor had taken no action in response to the original repudiation he no longer had the right to cancel the contract with the builder The sailorrsquos subsequent statement that ldquoour contract is overrdquo may have constituted repudiation by the sailor In any event when the sailor failed to perform on December 15 that constituted breach

Point One (35) Because the sailor had reasonable grounds for insecurity with respect to the builderrsquos performance the sailorrsquos letter to the builder was a justified demand seeking assurance of the builderrsquos performance under the contract failure of the builder to provide such assurance within a reasonable time constituted repudiation of the contract

The sailor was legally justified in sending the letter to the builder on October 31 Contract parties are entitled to expect due performance of contractual obligations and are permitted to take steps to protect that expectation UCC sect 2-609 states that ldquo[w]hen reasonable grounds for insecurity arise with respect to the performance of either party the other may in writing demand adequate assurance of due performance rdquo Here the sailor learned on October 31 that the builderrsquos workers were on strike This gave the sailor reasonable grounds for insecurity about the builderrsquos ability to complete performance on time and thus gave the sailor the right to seek adequate assurance from the builder Because the sailorrsquos demand for assurance was justified the builder was required to provide assurance that was adequate under the circumstances within a reasonable time (not to exceed 30 days) or be held to have repudiated the contract UCC sect 2-609(4)

19

Contracts Analysis

Point Two(a) (30) The builder did not within a reasonable time provide the sailor adequate assurance of due performance this failure to provide assurance constituted a repudiation of the contract

Because the sailor with legal justification (see Point One) demanded from the builder assurance of due performance the builderrsquos failure to provide such assurance within a reasonable time was a repudiation of their contract See UCC sect 2-609(4) (ldquoAfter receipt of a justified demand[] failure to provide within a reasonable time not exceeding thirty days assurance of due performance is a repudiation of the contractrdquo) On October 31 the sailor requested that the builder provide adequate assurance regarding the completion of the boat by December 15 The builder did not respond to the sailorrsquos letter until November 25mdashnearly a month later Even if that response had been given in a reasonable time it nonetheless did not provide assurance of due performance It simply stated ldquoIrsquom sorry about the strike but it is really out of my hands I hope we settle it soon so that we can get back to workrdquo Therefore the builderrsquos November 25 response did not provide adequate assurance in response to the sailorrsquos justified request Thus the builder had repudiated the contract

Point Two(b) (35) Although the builder repudiated the contract with the sailor the builder probably retracted that repudiation on December 3 and the sailor was no longer entitled to cancel their contract Thus the sailorrsquos failure to perform the sailorrsquos obligations under the contract constituted a breach

The builderrsquos failure to provide adequate assurance of performance constituted a repudiation of their contract (see UCC sect 2-609(4)) but the builder was free to retract that repudiation until the sailor cancelled the contract or materially changed his position or indicated by communication or action that the sailor considered the repudiation to be final See UCC sect 2-611(1) (ldquoUntil the repudiating partyrsquos next performance is due he can retract his repudiation unless the aggrieved party has since the repudiation cancelled or materially changed his position or otherwise indicated that he considers the repudiation finalrdquo)

Here the facts state that before the builderrsquos December 3 telephone call to the sailor the sailor did nothing in response to the builderrsquos repudiation such as contracting with a third party for a boat The builderrsquos December 3 call informing the sailor that the boat would be timely delivered probably constituted a retraction of the repudiation because it clearly indicated to the sailor that the builder would be able to perform UCC sect 2-611(2) Thus after being so informed the sailor did not have the right to treat their contract as cancelled UCC sect 2-611(3) Accordingly the sailorrsquos failure to perform the sailorrsquos obligations under the contract by taking the boat and paying for it constituted a breach of the contract

20

CONSTITUTIONAL LAW ANALYSIS (Constitutional Law IVA F2b amp e)

ANALYSIS

Legal Problems

(1) Does AutoCorsquos operation of a ldquocompany townrdquo result in its actions counting as those of the state for purposes of constitutional analysis

(2) Does the expulsion of a schoolchild for failure to recite the Pledge of Allegiance violate the First Amendment as applied through the Fourteenth Amendment

(3) Does the arrest of a pamphleteer in connection with violation of an anti-littering rule where the littering is done by the recipients of leaflets distributed by the pamphleteer violate the First Amendment as applied through the Fourteenth Amendment

DISCUSSION

Summary

The First Amendment as applied through the Fourteenth Amendment applies only to state action It does not typically govern private actors However courts have found state action where the private actor has exercised a ldquopublic functionrdquo such as running a privately owned ldquocompany townrdquo as AutoCo has done here Thus First Amendment protections apply By requiring the son to participate in a mandatory Pledge of Allegiance ceremony AutoCo has compelled the expression of political belief in violation of the First Amendment as applied through the Fourteenth Amendment The fatherrsquos arrest in connection with breaching the anti-litter rule also violated the First Amendment as applied through the Fourteenth Amendment Although state actors can regulate the incidental effects of speech on the public streets on a content-neutral basis this power is limited and cannot extend to punishing a distributor of literature because of littering by third parties

Point One (30) AutoCorsquos operation of a company town (including a school) makes it a state actor under the public function strand of the state action doctrine

The individual rights protections of the Constitution apply only where there is ldquostate actionrdquomdash either direct action by the government or some action by a private party that is fairly attributable to the government As a general rule the actions of a private company like AutoCo or of a private school like the school operated by AutoCo would not constitute state action and the protections of the Constitution (in this case the First Amendment) would not apply

However there are situations in which the actions of a private actor are attributed to the state One such situation is when the private actor undertakes a public function There are not many bright-line rules in the Supreme Courtrsquos state action doctrine but one of them is this Where a private actor undertakes a ldquopublic functionrdquo the Constitution applies to those actions Where a corporation operates a privately owned ldquocompany townrdquo that provides essential services typically provided by a state actor the public function doctrine applies and the Constitution

21

Constitutional Law Analysis

binds agents of the town as if they were agents of the government See eg Marsh v Alabama 326 US 501 (1946) Here AutoCo does more than own the town it provides security services fire protection sanitation services and a school Thus the actions of AutoCo constitute state action and are governed by the Fourteenth Amendment

Point Two (35) The sonrsquos expulsion for failure to recite the Pledge of Allegiance violates the First Amendment as applied through the Fourteenth Amendment as a compelled expression of political belief

As explained in Point One the First Amendment applies to the school as a state actor Although children in public schools (and in schools subject to the First Amendment like

the Oakwood school) have some First Amendment rights Tinker v Des Moines Independent Community School District 393 US 503 506 (1969) schools have greater leeway to regulate the speech of students and teachers than the state would have outside the school context Hazelwood School Dist v Kuhlmeier 484 US 260 (1988) Morse v Frederick 551 US 393 (2007) However the Supreme Court has long held that public schools may not force their students to participate in a flag salute ceremony when it offends the political or religious beliefs of the students or their families West Virginia Board of Educ v Barnette 319 US 624 (1943) (invalidating a mandatory public school flag salute ceremony) see also Wooley v Maynard 430 US 705 (1977) (invalidating compelled expression of political belief on state-issued license plates)

In this case the school requires its students to participate in a flag salute and Pledge of Allegiance ceremony and punishes them when they refuse to participate Pursuant to this policy the school has expelled the son This expulsion violates the First Amendment ban on compelled expression

Point Three (35) Because the father was distributing leaflets in a traditional public forum his trespass arrest violated the First Amendment as applied through the Fourteenth Amendment

As explained in Point One AutoCo is treated as a state actor Thus Oakwoodrsquos commercial district is treated as government-owned property for purposes of the First Amendment Thus the leafleting here is subject to the First Amendment because it is an expressive activity Schneider v State of New Jersey Town of Irvington 308 US 147 (1939) When expression takes place on government-owned property government regulation of the expression is assessed under the public forum doctrine Public streets and sidewalks have long been held to be the classic example of a ldquotraditional public forumrdquo open to the public for expression Hague v CIO 307 US 496 515ndash16 (1939) Because the father was distributing leaflets while standing on a street corner in the commercial district his expressive activity occurred in a traditional public forum

When a state tries to regulate expressive activity in a traditional public forum it is prohibited from doing so based on the expressive activityrsquos content unless its regulation is narrowly tailored to achieve a compelling governmental interest (ldquostrict scrutinyrdquo) In this case however AutoCo is regulating the fatherrsquos expressive activity on the ostensibly neutral ground that his expressive activity has produced litter and made the street unsightly When a state tries to regulate expressive activity without regard to its content intermediate scrutiny applies Under intermediate scrutiny the true purpose of the regulation may not be the suppression of ideas (if so then strict scrutiny applies) the regulation must be narrowly tailored to achieve a significant

22

Constitutional Law Analysis

governmental interest and it must leave open ample alternative channels for expressive activity Ward v Rock Against Racism 491 US 781 791 (1989)

Here the application of the ordinance to the father will fail for two reasons First the Supreme Court has held that the governmentrsquos interest in keeping the streets clean is insufficient to ban leafleting in the public streets as the government power to regulate with incidental effects on public sidewalk speech is very limited See eg Schneider 308 US at 162 (leafletinglittering) Second the regulation (a blanket ban on distribution that results in littering) is not narrowly tailored to protect expression A narrowly tailored alternative would be prosecution only of people who litter Moreover the effect of the littering rule is likely to be a ban on all leafleting thus eliminating an entire class of means of expression This raises the possibility that there are not ldquoample alternative channels of communicationrdquo open to the father as required under the Courtrsquos standard of review for content-neutral regulation of speech

[NOTE Some examinees might argue that this is a ldquotime place and mannerrdquo restriction and that AutoCo might have greater latitude to regulate the public sidewalks under this theory This argument is incorrect for two reasons First the Supreme Court has held that the power to regulate speakers through littering laws is very limited for the reasons given and in the cases cited above But more generally a ldquotime place and mannerrdquo restriction involves the shifting of speech from one time and place to another or to another manner here there is no shifting but a direct punishment for expressive activity (albeit one couched in content-neutral terms) In addition some examinees might read the ordinance to be in effect a total ban on leafleting since most leafleting will produce some litter Those examinees might note that the Court has required total bans on an entire mode of expression to satisfy strict scrutiny and analyze the fatherrsquos prosecution here accordingly See United States v Grace 461 US 171 177 (1983) (invalidating ban on display of signs on public sidewalks surrounding US Supreme Court ldquo[a]dditional restrictions such as an absolute prohibition on a particular type of expression will be upheld only if narrowly drawn to accomplish a compelling governmental interestrdquo)]

23

SECURED TRANSACTIONS ANALYSIS (Secured Transactions IID E IVA B C)

ANALYSIS

Legal Problems

(1) Is a purchase-money security interest in consumer goods perfected even though there has been no filing of a financing statement

(2) Does a person who buys consumer goods for personal use take those goods free of a prior perfected purchase-money security interest in the goods

(3) Does a person who receives consumer goods as a gift take those goods subject to a prior perfected security interest in them

DISCUSSION

Summary

The retailerrsquos security interest in the bicycles was perfected even though no financing statement was filed because it was a purchase-money security interest in consumer goods A purchase-money security interest in consumer goods is automatically perfected upon attachment

The buyer is not subject to the retailerrsquos security interest in the bicycle that the buyer bought from the man Because the bicycle was consumer goods in the hands of the man and the retailer never filed a financing statement covering the bicycle the retailerrsquos security interest is not effective against someone like the buyer who bought the bicycle for value without knowledge of the retailerrsquos security interest and for personal use

On the other hand the retailerrsquos security interest continues in the bicycle given to the friend because the friend did not give value for the bicycle or buy it in the ordinary course of business

Point One (35) The retailerrsquos security interest in the bicycles attached on June 1 Because this interest was a purchase-money security interest in consumer goods it was automatically perfected when it attached

The retailerrsquos security interest in the bicycles attached on June 1 when the man bought the bicycles (acquiring rights in the collateral) signed a security agreement containing a description of the collateral and received value from the retailer (by being given credit with which to purchase the bicycles) UCC sect 9-203(a) amp (b)

Despite the retailerrsquos failure to file a financing statement its security interest was perfected Pursuant to UCC sect 9-309(1) a security interest is automatically perfected upon attachment if the goods are ldquoconsumer goodsrdquo and the security interest is a ldquopurchase-money security interestrdquo

In this case the bicycles sold by the retailer to the man were consumer goods at the time of sale The bicycles were ldquogoodsrdquo because they were ldquomovable when a security interest

24

Secured Transactions Analysis

attachesrdquo UCC sect 9-102(a)(44) They were also consumer goods because they were ldquobought for use primarily for personal family or household purposesrdquo UCC sect 9-102(a)(23) The retailerrsquos security interest in these consumer goods was also a ldquopurchase-money security interestrdquo A purchase-money security interest is an interest that secures a debt that was incurred in order to ldquoenable the debtor to acquire rights in or the use of the collateralrdquo UCC sect 9-103(a) (b)(1) Here the man incurred an obligation to the retailer to purchase the bicycles so the security interest he gave the retailer to secure that obligation was a purchase-money security interest

Because the retailerrsquos security interest was a purchase-money security interest in consumer goods it was automatically perfected on June 1 when the interest attached to the bicycles

Point Two (35) The buyer took the bicycle free of the retailerrsquos security interest because (i) the retailer did not file a financing statement covering the bicycle (ii) the bicycle was ldquoconsumer goodsrdquo and (iii) the buyer bought the bicycle for value without knowledge of the retailerrsquos security interest and for personal use

A security interest continues in collateral even after a sale or other disposition of that collateral unless the creditor authorized the disposition ldquofree of the security interestrdquo or another Article 9 exception applies UCC sectsect 9-201(a) and 9-315(a)(1)

However a buyer of goods like the buyer here can take free of a prior security interest in those goods under certain circumstances See UCC sectsect 9-317(b) (buyers who give value and receive delivery of goods without knowledge of an unperfected security interest in the goods) and 9-320(a) amp (b) (buyer in ordinary course of business buyer of consumer goods in a consumer-to-consumer transaction who gives value) In this case the retailerrsquos security interest was perfected when the buyer purchased the bicycle so UCC sect 9-317(b) does not protect the buyer The buyer also is not a protected ldquobuyer in ordinary course of businessrdquo because he did not purchase from a person who is in the business of selling bicycles See UCC sect 1-201(b)(9)

The buyer can however qualify for the protection of UCC sect 9-320(b) That section provides that a buyer of goods from a person who used them for personal family or household purposes takes free of a perfected security interest in the goods if (1) the buyer had no knowledge of the security interest (2) the buyer gave value for the goods (3) the buyer purchased the goods primarily for personal family or household purposes and (4) the purchase occurred before the filing of a financing statement covering the goods

The buyer met all of these criteria The man used the bicycle for personal purposes The buyer purchased the bicycle from the man and the buyer had no knowledge of the retailerrsquos security interest The buyer gave value ($400) for the bicycle and he bought it ldquoprimarily for personal family or household purposesrdquo as he planned to use it for recreation which is a personal rather than a business use Finally no financing statement had been filed Therefore under UCC sect 9-320(b) the buyer took free of the retailerrsquos security interest

Point Three (30) The retailerrsquos security interest continues in the bicycle that the man gave to the friend Thus the retailer can recover the bicycle from the friend because the friend did not give value for the bicycle or buy it in the ordinary course of business

25

Secured Transactions Analysis

As noted in Point Two the retailer did not authorize the man to dispose of the bicycle Consequently the retailerrsquos security interest continued in the bicycle even after the man transferred ownership of the bicycle to the friend See UCC sectsect 9-201(a) and 9-315(a)(1) The retailerrsquos security interest in the bicycle will be effective against the friend unless some other provision of Article 9 allows the friend to take the bicycle free of that security interest

Unfortunately for the friend there is no Article 9 provision that allows him to take free of the retailerrsquos interest The friendrsquos basic problem is that he is not a buyer of the bicyclemdashhe received the bicycle as a gift and did not give value for it Thus the friend is not protected by any of the applicable exceptions See UCC sectsect 9-317(b) (protecting buyers who give value for goods subject to an unperfected security interest) 9-320(a) (protecting buyers in ordinary course of business) and 9-320(b) (protecting buyers of consumer goods who give value)

In short the retailerrsquos security interest continues in the bicycle that the man gave to the friend The friend took the bicycle subject to that security interest

26

FEDERAL CIVIL PROCEDURE ANALYSIS (Federal Civil Procedure VIE)

ANALYSIS

Legal Problems

(1) Does a judgment in a prior action preclude a nonparty from suing the same defendant on a closely related claim when the nonparty and the original plaintiff are in a family relationship

(2) Does a judgment rendered in an earlier action preclude a nonparty from litigating an issue that was actually decided in the first suit

(3) May a nonparty to an earlier action invoke the judgment in that action to preclude a party to the prior action from relitigating an issue that the party had a full and fair opportunity to litigate in the earlier action

DISCUSSION

Summary

Pursuant to the doctrines of claim preclusion (res judicata) and issue preclusion (collateral estoppel) a judgment is binding on the parties thereto In the absence of privity nonparties to a prior suit cannot be bound by a judgment rendered in their absence Thus in the absence of privity a nonparty to the first suit is not precluded from presenting her claim in a second suit even if it is factually related to the claims and defenses presented in the first suit nor is she bound by determinations of issues made in the first suit A family relationship without more does not support a finding of privity For this reason Mother as a nonparty is not bound by the judgment in the Son-Driver action She may bring her separate claim for damage to her car and she is not precluded from litigating the question of whether she was negligent in the maintenance of her car

Driver on the other hand could be precluded from relitigating the issue of her negligence pursuant to the doctrine of non-mutual issue preclusion (also called non-mutual offensive collateral estoppel) which allows a nonparty to a prior action to invoke issue preclusion to prevent a party to that prior action from relitigating determinations of issues made therein However Mother may be prevented from invoking non-mutual collateral estoppel in this case because she could easily have joined her claim in the prior action but did not do so

[NOTE Federal common law governs the preclusive effect of a judgment rendered by a federal court sitting in diversity See Semtek Intrsquol Inc v Lockheed Martin Corp 531 US 497 508 (2001) But the Semtek Court concluded that federal common law in this context incorporates the preclusion law of the state in which the rendering federal court sits (unless the state law is incompatible with federal interests) id at 508ndash09 Thus State Arsquos preclusion law determines the preclusive effect of the judgment rendered in Sonrsquos suit against Driver The problem says that State A preclusion law is identical to federal preclusion law so the following analysis utilizes general principles of preclusion drawn from Supreme Court case law (announcing federal preclusion rules) and the Restatement (Second) of Judgments]

27

Federal Civil Procedure Analysis

Point One (35) Under the doctrine of claim preclusion the judgment rendered in the first action does not preclude Mother a nonparty from suing Driver for the damage to her car because the judgment binds only parties or those in privity with them and Mother and Son are not in privity

Driver may contend that the doctrine of claim preclusion (res judicata) precludes Mother from presenting a claim arising from the same nucleus of facts that was presented in the first action brought by Son According to the doctrine of claim preclusion ldquowhen a court of competent jurisdiction has entered a final judgment on the merits of a cause of action the parties to the suit and their privies are thereafter bound lsquonot only as to every matter which was offered and received to sustain or defeat the claim or demand but as to any other admissible matter which might have been offered for that purposersquordquo Commissioner of Internal Revenue v Sunnen 333 US 591 597 (1948) (citation omitted)

However the doctrine of claim preclusion does not apply to Mother on the facts of this problem First Mother was not a party to the earlier case ldquoIt is a principle of general application in Anglo-American jurisprudence that one is not bound by a judgment in personam in a litigation in which he is not designated as a party or to which he has not been made a party by service of processrdquo Taylor v Sturgell 553 US 880 884 (2008) (citing Hansberry v Lee 311 US 32 40 (1940)) see also RESTATEMENT (SECOND) OF JUDGMENTS sect 34(3) (1982) This rule reflects our ldquodeep-rooted historic tradition that everyone should have his own day in courtrdquo Martin v Wilks 490 US 755 762 (1989) (citation omitted) (superseded by statute on other grounds) Since Mother was not a party to the first suit she is not bound by the judgment unless an exception to the general rule applies

Mother might be bound by the prior judgment if she were considered to have been sufficiently in privity with Son that Son represented her interests in that action ldquoA person who is not a party to an action but who is represented by a party is bound by and entitled to the benefits of a judgment as though he were a partyrdquo RESTATEMENT (SECOND) OF JUDGMENTS sect 41(1) But there is no suggestion in the facts of the problem that Son who is an adult purported to represent Motherrsquos interests in the first suit ldquo[C]lose family relationships are not sufficient by themselves to establish privity with the original suitrsquos party or to bind a nonparty to that suit by the judgment entered therein rdquo Cuauhtli v Chase Home Finance LLC 308 Fed Appx 772 773 (5th Cir 2009) (citation omitted) accord 18A CHARLES ALAN WRIGHT ET AL FEDERAL

PRACTICE AND PROCEDURE sect 4459 (2d ed 2002) In Taylor v Sturgell supra the Supreme Court identified other special circumstances in

which nonparties may be bound by a prior judgmentmdashwhen a nonparty consents to be bound when a nonparty is in a pre-existing substantive legal relationship with a party (such as preceding and succeeding property owners) when a nonparty assumed control of the prior litigation when a party seeks to relitigate through a proxy or where a special statutory scheme seeks to foreclose successive litigation by nonparties See Taylor 553 US at 893ndash95 None of these circumstances exists here

Because Mother was not a party to the first suit and is not in privity with Son who is an adult the judgment in the first action does not preclude her from bringing her own claim against Driver

Point Two (35) Under the doctrine of issue preclusion the judgment rendered in the first action does not preclude Mother a nonparty from litigating the issue of her negligence in maintaining her carrsquos

28

Federal Civil Procedure Analysis

brake lights because the judgment binds only parties or those in privity with them and Mother and Son are not in privity

By its affirmative response to a special interrogatory the jury in the first action expressly concluded that ldquoMother negligently failed to ensure that the brake lights on her car were in proper working orderrdquo Driver may attempt to invoke the doctrine of issue preclusion to preclude Mother from relitigating this issue in the second action

[I]ssue preclusion arises in a second action on the basis of a prior decision when the same lsquoissuersquo is involved in both actions the issue was lsquoactually litigatedrsquo in the first action after a full and fair opportunity for litigation the issue was lsquoactually decidedrsquo in the first action by a disposition that is sufficiently lsquofinalrsquo lsquoon the meritsrsquo and lsquovalidrsquo it was necessary to decide the issue in disposing of the first action and the later litigation is between the same parties or involves nonparties that are subject to the binding effect or benefit of the first action Once these requirements are met issue preclusion is available not only to defend against a demand for relief but also as offensive support for a demand for relief Issue preclusion moreover is available whether or not the second action involves a new claim or cause of action

18 CHARLES ALAN WRIGHT ET AL FEDERAL PRACTICE AND PROCEDURE sect 4416 at 392ndash93 (2d ed) see also RESTATEMENT (SECOND) OF JUDGMENTS sect 27 (1982)

Here several of the elements necessary for issue preclusion are present The same issue is involved in both actionsmdashthe issue of Motherrsquos negligence in failing to maintain the brake lights on her car That issue was actually litigated in the first action and decided by the jury There is nothing to suggest anything less than a full and fair opportunity to litigate The judgment disposing of the issue was final

Nevertheless the judgment will not preclude Mother from relitigating the issue for two reasons First Mother was not a party to the first action and as explained above Mother and Son are not in privity Therefore she cannot be denied an opportunity to litigate the issue of her negligence Second it does not appear that the juryrsquos decision as to Motherrsquos negligence was necessary to the prior judgment against Driver Nothing suggests that the finding on Motherrsquos negligence had any bearing on the outcome of the first action

Point Three (30) Under the doctrine of non-mutual issue preclusion the judgment rendered in the first action might preclude Driver from relitigating the issue of her negligence However Driver has a strong argument that such a result would be inconsistent with the policy against offensive use of non-mutual estoppel when the non-party plaintiff easily could have joined as a plaintiff in the first action

Because Son already convinced the jury in the first action that ldquoDriver was negligent in the operation of her vehiclerdquo Mother may wish to invoke the doctrine of non-mutual issue preclusion to prevent Driver from relitigating the question of her negligence As noted above ldquoissue preclusion arises in a second action on the basis of a prior decision when the same lsquoissuersquo is involved in both actions the issue was lsquoactually litigatedrsquo in the first action after a full and fair opportunity for litigation the issue was lsquoactually decidedrsquo in the first action by a disposition that is sufficiently lsquofinalrsquo lsquoon the meritsrsquo and lsquovalidrsquo it was necessary to decide the issue in disposing of the first action rdquo 18 CHARLES ALAN WRIGHT ET AL FEDERAL PRACTICE AND

PROCEDURE sect 4416 at 392 (2d ed) see also RESTATEMENT (SECOND) OF JUDGMENTS sect 27

29

Federal Civil Procedure Analysis

Here these basic requirements for issue preclusion are met First the same issue is involved in both suits whether Driver was negligent in the operation of her car Second this issue was actually litigated and decided in the first action the jury answered a special interrogatory raising this very question There is nothing to suggest that Driver lacked a full and fair opportunity to litigate the issue Since a judgment was rendered against Driver for the injuries Son sustained as a result of Driverrsquos negligence resolution of the issue was necessary to dispose of the first action Driver was a party to the first action so she may be bound by the judgment

[NOTE Traditionally issue preclusion required mutualitymdashboth the party asserting issue preclusion and the party against whom issue preclusion was asserted were bound by the prior judgment Under the traditional mutuality rule Mother could not assert issue preclusion against Driver because Mother would not be bound by the judgment if Driver sought to rely on it See Point One There is no mutuality between Mother and Driver with respect to the prior judgment

This traditional mutuality requirement has been abandoned in most jurisdictions The Supreme Court rejected a strict mutuality requirement in Blonder-Tongue Laboratories Inc v University of Illinois Foundation 402 US 313 (1971) (non-mutual defensive collateral estoppel used by a defendant to preclude a plaintiff from relitigating a claim the plaintiff previously litigated) and Parklane Hosiery Co v Shore 439 US 322 (1979) (non-mutual offensive collateral estoppel used by a plaintiff to preclude a defendant from relitigating a claim the defendant previously litigated) In Parklane Hosiery the Court concluded (as a matter of federal preclusion law) that trial courts should have ldquobroad discretionrdquo to determine whether or not to permit a plaintiff to invoke non-mutual issue preclusion ldquoThe general rule should be that in cases where a plaintiff could easily have joined in the earlier action or where the application of offensive estoppel would be unfair to a defendant a trial judge should not allow the use of offensive collateral estoppelrdquo Id at 331

The Parklane Hosiery decision identified a number of circumstances that might make it unfair to allow a plaintiff to invoke non-mutual issue preclusion (non-mutual offensive collateral estoppel in the traditional terminology) against a defendant In particular the Parklane Hosiery court suggested that issue preclusion may not be appropriate if the plaintiff in the second action ldquocould easily have joined in the earlier actionrdquo Id Prohibiting plaintiffs from using non-mutual estoppel under such circumstances would promote judicial efficiency by encouraging plaintiffs to join the prior action It would also discourage plaintiffs from staying out of prior litigation in order to secure in effect two bites at the apple using the prior litigation offensively if the defendant loses and forcing the defendant to litigate a second time if the defendant wins the prior action

An exceptional exam answer might therefore argue that non-mutual issue preclusion should be denied on these facts Son and Mother both reside in State A since they are related they know each other well and Son was driving Motherrsquos car when the accident occurred They could have sued together and Rule 20 of the Federal Rules of Civil Procedure would have authorized joinder of their claims because those claims arose from the same transaction or occurrence and raised a common question of law or fact FED R CIV P 20(a) The facts do not suggest that Mother had any reason not to join Sonrsquos suit other than a desire to see how Sonrsquos action concluded before bringing her own claim Cf Nations v Sun Oil Co (Del) 695 F2d 933 938 (5th Cir 1983) (concluding that plaintiff ldquowas entitled to await the development of his injuries and their predictable consequencesrdquo) Because it appears that Mother may be a ldquowait-and-seerdquo plaintiff who could easily have joined the original action a trial court might disallow as a matter of discretion her use of non-mutual issue preclusion]

30

AGENCY ANALYSIS __________ (Agency I II)

ANALYSIS

Legal Problems

(1) Is the principal or the agent or both liable on contracts with a third party when the principal is an ldquoundisclosed principalrdquo

(2) Is the principal or the agent or both liable on contracts with a third party when the principal is ldquopartially disclosedrdquo or an ldquounidentified principalrdquo

(3) Is the principal or the agent or both liable on contracts with a third party for the purchase of goods when the agent exceeded his authority but the principal nonetheless accepts the goods

DISCUSSION

Summary

The agent but not the owner is liable to the basket manufacturer because the owner is an undisclosed principal and the agent acted without actual or apparent authority Both the agent and the owner however are liable on the burner contract because the owner is an unidentified principal and the agent had apparent authority to enter into that contract With respect to the solar cells contract whether the owner is liable depends upon whether a court would follow the Second or Third Restatement of Agency which take different positions on the effect of the ratification of a contract by an undisclosed principal Under either the agent would also be liable on the contract as he was a party to the contract

[NOTE The contracts that are the subject of this question are contracts for the sale of goods and therefore are governed by Article 2 of the Uniform Commercial Code Article 2 however does not contain agency rules Accordingly common law concepts of agency are applicable UCC sect 1-103(b)]

Point One (35) The agent but not the owner is liable to the basket manufacturer The agent had no actual authority to enter into the contract to buy aluminum baskets and because the owner was an undisclosed principal the manufacturer had no reason to believe that the agent had apparent authority Furthermore the manufacturer had no reason to believe that the agent was not contracting for his own benefit

An agent acting on behalf of a principal can bind the principal to contracts if the agent has either actual or apparent authority An agent has actual authority when contracting on behalf of his principal if he ldquoreasonably believes in accordance with the principalrsquos manifestations to the agent that the principal wishes the agent so to actrdquo RESTATEMENT (THIRD) OF AGENCY sect 201 (2006) Here the agent was told to buy only wicker baskets not aluminum baskets Thus when he contracted with the basket manufacturer to buy aluminum baskets he had no actual authority to do so

31

Agency Analysis

An agent acts with apparent authority ldquowhen a third party [with whom the agent acts] reasonably believes the actor has authority to act on behalf of the principal and that belief is traceable to the principalrsquos manifestationsrdquo Id sect 203 Here the owner notified basket manufacturers that she or her agent might contact them to purchase baskets but that notification did not specifically name the agent or any other person as the ownerrsquos agent Furthermore the basket manufacturer had no prior dealings with the agent or the owner or any reason to think that the agent was acting for the benefit of anyone but himself Thus there is no basis to conclude that the basket manufacturer thought that the agent had apparent authority to act for the owner

Generally when an agent acts on behalf of an undisclosed principal and the agent lacks authority to enter into the contract the agent is liable on the contract as a party to the contract but the principal is not liable This rule is consistent with the third partyrsquos expectations ldquoThe third party expected the agent to be a party to the contract because the agent presented the deal as if he were acting for himself Moreover if the third party is unaware of the principalrsquos existence the third party must be relying on the agentrsquos solvency and reliability when entering into the contractrdquo See ROBERT W HAMILTON JONATHAN R MACEY amp DOUGLAS K MOLL CORPORATIONS INCLUDING PARTNERSHIPS AND LIMITED LIABILITY COMPANIES 34 (11th ed 2010) See also RESTATEMENT (THIRD) OF AGENCY sect 603 cmt c Furthermore because the third party has no idea that the agent is acting or is seemingly acting on behalf of another there is no reason to believe that the third party would be expecting an undisclosed principal to be liable on the contract Id

Point Two (35) Because the owner is an unidentified (as opposed to undisclosed) principal both she and the agent (as a party to the contract) probably are liable on the contract with the burner manufacturer

When the agent contracted with the burner manufacturer he did not have actual authority to do so as the owner had expressly restricted the agentrsquos authority to purchase only burners with ldquowhisper technologyrdquo See Point One However the agent may have had apparent authority to buy burners without whisper technology

An agent acts with apparent authority ldquowhen a third party [with whom the agent acts] reasonably believes the actor has authority to act on behalf of the principal and that belief is traceable to the principalrsquos manifestationsrdquo RESTATEMENT (THIRD) OF AGENCY sect 203 (2006) The owner indicated that an agent might contact the burner manufacturer The notice contained no restriction regarding the type of burners that the agent was authorized to purchase The facts indicate that burner manufacturers regularly receive such notices

Although the agent told the burner manufacturer that he represented a well-known hot-air balloon operator he did not disclose the ownerrsquos name Thus the owner was a partially disclosed or unidentified principal See RESTATEMENT (SECOND) OF AGENCY sect 4(2) (1958) (using term ldquopartially disclosed principalrdquo) RESTATEMENT (THIRD) OF AGENCY sect 104(2)(c) (2006) (using term ldquounidentified principalrdquo) An agent for a partially disclosed principal may have apparent authority RESTATEMENT (SECOND) OF AGENCY sect 159 cmt e (1958) Based upon (1) the notice sent by the owner (2) the agentrsquos revelation that he was acting as an agent and (3) the fact that burner manufacturers regularly receive such notices and sell to agents the manufacturer may argue that it reasonably and actually believed that the agent was authorized to purchase burners without whisper technology The manufacturer may also argue that because the agent revealed that he was an agent his listing of the ownerrsquos address as the delivery address connects the agent to the notice given by the owner Arguably this distinguishes the burner contract from the basket

32

Agency Analysis

contract Here there is a strong case to support the conclusion that the agent had apparent authority if he did then the owner is liable to the burner manufacturer

The agent also is liable as a party to the contract because he did not fully disclose his agency relationship Although he told the burner manufacturer that he represented a well-known hot-air balloon operator he did not disclose the ownerrsquos name Generally even an authorized agent of a partially disclosed or unidentified principal is liable as a party to a contract with a third person RESTATEMENT (SECOND) OF AGENCY sect 321 (1958) (ldquounless otherwise agreedrdquo) RESTATEMENT (THIRD) OF AGENCY sect 602(2) (2006) (ldquounless the agent and the third party agree otherwiserdquo)

Point Three (30) Under the Second Restatement of Agency the owner is not liable on the contract for solar cells because the agent did not have actual or apparent authority and the owner as an undisclosed principal cannot ratify the contract Under the Third Restatement the owner could be liable as she ratified the contract Under either Restatement the agent is liable as a party to the contract

The owner is not liable to the solar cell manufacturer for breach of the contract for the solar cells because the agent had no actual or apparent authority to purchase solar cells on the ownerrsquos behalf and the owner under the Second Restatement of Agency did not ratify the contract with knowledge of the material facts Thus she is not liable as a ratifier of the contract

The facts state that the agent had authority to purchase only propane fuel tanks In addition he had no apparent authority to purchase solar cells The owner made no manifestations to the solar cell manufacturer that would lead a reasonable person in the manufacturerrsquos position to believe that the agent had the authority to bind the owner to a contract to purchase solar cells In fact the agent made no manifestations at all to the solar cell manufacturer Unlike with the basket manufacturer and the burner manufacturer the owner did not notify the manufacturer of solar cells that an agent might contact it to purchase solar cells In addition the solar cells were delivered to the agent and not to the ownerrsquos address In sum the manufacturer was unaware of any relationship between the owner and the agent As to the solar cell manufacturer the owner is an undisclosed principal There can be no apparent authority in the case of an undisclosed principal because there are no manifestations from the principal to the third person See RESTATEMENT (SECOND) OF AGENCY sect 8 cmt a (1958) (ldquothere can be no apparent authority created by an undisclosed principalrdquo) RESTATEMENT (THIRD) OF AGENCY sect 203 cmt f (2006) (ldquoapparent authority is not present when a third party believes that an interaction is with an actor who is a principalrdquo)

The owner also did not ratify the contract Although the owner used the solar cells generally a principal cannot ratify an unauthorized transaction with a third person ldquounless the one acting purported to be acting for the ratifierrdquo RESTATEMENT (SECOND) OF AGENCY sect 85(1) (1958)

The result differs under the Third Restatement which expressly rejects the Second Restatement on this issue The Restatement (Third) of Agency sect 403 (2006) states ldquoA person may ratify an act if the actor acted or purported to act as an agent on the personrsquos behalfrdquo According to comment b ldquoan undisclosed principal may ratify an agentrsquos unauthorized actrdquo Under the Restatement (Third) of Agency rule the owner probably ratified the transaction The agent clearly acted on the ownerrsquos behalf and in addition the ownerrsquos conduct in using the solar cells ldquojustifies a reasonable assumption that [she] is manifesting assent that the act shall affect [her] legal relationsrdquo See id sect 401(2)

33

Agency Analysis

The agent also is liable to the solar cell manufacturer for breach of the contract for the solar cells because he is a party to the contract The facts indicate that the agent never told the solar cell manufacturer that he represented the owner or any other principal Consequently even if the agent were authorized (which as discussed above he is not) he would be liable as a party to the contract See RESTATEMENT (SECOND) OF AGENCY sect 322 (1958) RESTATEMENT (THIRD) OF AGENCY sect 603(2) (2006) Here he has no authority or apparent authority and is liable as a party to the contract

The agent would also be liable under the Third Restatement Under Restatement (Third) of Agency sect 402(1) (2006) ratification generally relates back and the transaction is treated as if it were authorized at the time of the transaction However this does not relieve the agent of an undisclosed principal who ratifies an unauthorized transaction of liability under the ratified contract See id sect 603(2) (authorized agent for undisclosed principal is a party to the contract) and sect 403 cmt b (ldquoAn undisclosed principalrsquos ratification does not eliminate the agentrsquos liability to the third party on the transaction rdquo)

[NOTE An examinee may discuss the concept of inherent agency power This concept is recognized by the Restatement (Second) of Agency sect 8 A (1958) but the concept is not used in the Restatement (Third) of Agency (2006) Here there are no facts to support that the agent had inherent authority

As to contracts with agents for partially disclosed principals (eg the contract for the burners) the basic question is whether the acts done ldquousually accompany or are incidental to transactions which the agent is authorized to conductrdquo RESTATEMENT (SECOND) OF AGENCY

sect 161 (1958) If so the principal is bound if the other party ldquoreasonably believes that the agent is authorized to do them and has no notice that he is not so authorizedrdquo Id The purchase of burners without whisper technology was not authorized nor was it incidental to an authorized transaction Therefore there should not be inherent agency power

As to contracts on behalf of undisclosed principals (eg the other two contracts) the basic question is whether the acts done are usual or necessary in the transactions the agent is authorized to transact RESTATEMENT (SECOND) OF AGENCY sect 194 (1958) The other two contracts seem fundamentally different from the authorized transactions Therefore there should not be inherent agency power

Only minimal credit should be given for discussion of inherent agency power]

34

EVIDENCE ANALYSIS _____ (Evidence IIA VA B E F J K)

ANALYSIS

Legal Problems

(1) Is the authenticated copy of the mechanicrsquos text message relevant and admissible

(2) Is the womanrsquos question ldquoIs my scooter safe to drive for a whilerdquo relevant and admissible

(3) Is the womanrsquos testimony describing the mechanicrsquos thumbs-up relevant and admissible

DISCUSSION

Summary

The mechanicrsquos text message to the woman is relevant to whether (1) the woman lost control of the scooter due to its defective brakes (2) the woman knew that the brakes needed repair and (3) it was negligent for the woman to drive the scooter knowing that its brakes needed repair

The mechanicrsquos text message is hearsay if it is offered by the pedestrian to prove that the scooterrsquos brakes needed repair However it fits the hearsay exception for present sense impressions and probably also fits the exception for business records The mechanicrsquos text message is not hearsay if it is instead offered by the pedestrian to prove the womanrsquos state of mind (ie that she had notice that her brakes needed repair)

The womanrsquos question to the mechanic and his response are also relevant to whether the brakes caused the accident and whether the woman was negligent The question is not hearsay because the woman did not make an assertion

The mechanicrsquos thumbs-up response is nonverbal conduct intended by the mechanic as an assertion and is therefore an out-of-court statement If the woman offers the mechanicrsquos statement to prove that the scooter was actually safe to ride the womanrsquos testimony about the statement is hearsay

However the mechanicrsquos statement is not hearsay if it is offered by the woman to prove her state of mind Therefore the womanrsquos question and the mechanicrsquos response are admissible to prove the womanrsquos state of mind

Point One(a) (20) The mechanicrsquos text message to the woman should be admitted because it is relevant

Evidence is relevant if it has ldquoany tendency to make a fact more or less probable than it would be without the evidencerdquo FED R EVID 401 ldquoRelevant evidence is admissiblerdquo unless it is inadmissible pursuant to some other rule FED R EVID 402

The mechanicrsquos text message to the woman ldquoWhen you pick up your scooter you need to schedule a follow-up brake repair Wersquoll order the partsrdquo is relevant for two reasons First this evidence has some tendency to make it more probable that the brakes malfunctioned and

35

Evidence Analysis

caused the accident Second it has some tendency to make it more probable that the woman was negligent in riding her scooter after being told by the mechanic that it required further repair

Point One(b) (30) The mechanicrsquos text message fits either the hearsay exception for present sense impressions or the exception for business records or it is admissible non-hearsay

The mechanicrsquos text message is a statement under Rule 801(a) because it is ldquoa written assertionrdquo FED R EVID 801(a) The text message is hearsay if the pedestrian offers it to prove the ldquotruth of the matter asserted in the statementrdquo (ie that the scooterrsquos brakes required repair) which resulted in the woman losing control of the scooter and causing the accident FED R EVID 801(c)

However the mechanicrsquos text message fits the hearsay exception for ldquopresent sense impressionsrdquo under Rule 803(1) because it is ldquo[a] statement describing or explaining an event or condition made while or immediately after the declarant perceived itrdquo FED R EVID 803(1) Here the mechanicrsquos text message described the condition of the scooter immediately after he perceived it during the maintenance service

The mechanic is a person with knowledge of the condition of the scooter so if text messages regarding repairs were made and kept by the mechanic in the ordinary course of business this text message also fits the business records exception Under Rule 803(6) a business record is a record of an act ldquomade at or near the time by someone with knowledgerdquo and ldquothe record was kept in the course of a regularly conducted activity of a businessrdquo and ldquomaking the record was a regular practice of that activityrdquo FED R EVID 803(6)

However the text message is not hearsay if it is instead offered to prove that the woman was negligent because she rode her scooter after the mechanic told her it required repair If offered for this purpose it would not be offered for the truth of the matter asserted in the statement but to show the womanrsquos belief about the condition of the scooter (her state of mind)

Point Two (10) The womanrsquos question to the mechanic should be admitted because it is not hearsay

The womanrsquos question to the mechanic is relevant because along with the mechanicrsquos thumbs-up response (see Point Three) it has some tendency to make it more probable that the woman was not negligent andor that the scooter brakes did not malfunction and cause the accident FED R EVID 401 The womanrsquos question does not raise hearsay concerns because it is not an assertion

Hearsay is defined under Rule 801(a) as ldquoan oral assertion written assertion or nonverbal conductrdquo Although ldquoassertionrdquo is not further defined ldquoa favorite [definition] of writers in the [evidence] field for at least a century and a half [is that] the word simply means to say that something is so eg that an event happened or a condition existedrdquo 2 MCCORMICK ON

EVIDENCE sect 246 (6th ed 2006) Under this definition the womanrsquos question is not hearsay because it is not an assertion

Point Three(a) (20) The mechanicrsquos thumbs-up to the woman is a nonverbal assertion that is relevant and the womanrsquos testimony about that response is admissible

36

Evidence Analysis

Hearsay is defined under Rule 801(c) as a ldquostatementrdquo that is ldquoa personrsquos oral assertion written assertion or nonverbal conduct if the person intended it as an assertionrdquo FED R EVID 801(a) Here when the mechanic responded to the womanrsquos question (ldquoIs my scooter safe to ride for a whilerdquo) with a thumbs-up gesture the facts suggest that he intended his nonverbal conduct as an assertion that in his opinion the scooter was safe to ride

The mechanicrsquos assertion is relevant and admissible to prove that the woman was not negligent because the evidence makes it more probable that at the time of the accident she believed that the scooter was safe to ride despite the fact that the brakes required repair FED R EVID 401 Admission of the womanrsquos description of the mechanicrsquos thumbs-up for this purpose does not raise hearsay concerns because the evidence would not be offered for the truth of the matter asserted but to show the womanrsquos belief about the condition of the scooter (her state of mind)

Point Three(b) (20) The mechanicrsquos thumbs-up is relevant to determine whether the scooterrsquos brakes malfunctioned causing the accident but if offered for this purpose it is also hearsay

The mechanicrsquos nonverbal assertion is relevant to the determination of whether the scooterrsquos brakes malfunctioned causing the accident However if offered to prove the ldquotruth of the matter asserted in the statementrdquo (ie that the scooter was safe to ride for a while) it is hearsay that does not fit any hearsay exception

37

TRUSTS AND FUTURE INTERESTS ANALYSIS ____________________ (Trusts and Future Interests IC1 amp 4 G IIF)

ANALYSIS

Legal Problems

(1)(a) Was the revocable trust amendable

(1)(b) If the trust was amendable must the amendment have been executed in accordance with the state Statute of Wills in order to be valid

(2) If the trust amendment was valid does the amendment apply to the probate estate assets passing to the trust pursuant to Settlorrsquos will

(3) If the trust amendment was valid should the trust property be distributed to University

(4) If the trust amendment was not valid should the trust property be distributed to Settlorrsquos grandchild (her only heir) or held in further trust in accordance with the terms of the original trust instrument

DISCUSSION

Summary

A revocable trust is amendable even if the trust instrument does not expressly grant to the trust settlor a power to amend Both inter vivos trusts and amendments thereto are valid even though not executed in accordance with the requirements applicable to wills

Under the Uniform Testamentary Additions to Trusts Act a revocable trust may be amended at any time prior to the settlorrsquos death and the amendment applies to the disposition of assets conveyed to the trust pursuant to a will even if the will was executed prior to the date of the amendment

At Settlorrsquos death trust assets including probate assets passing to the trust under Settlorrsquos will would go to University if as is the case here the trust amendment was valid If the amendment was invalid the trust assets would continue to be held in further trust because there is no violation of the common law Rule Against Perpetuities

Point One(a) (30) Settlor retained the right to amend the inter vivos trust despite her failure to expressly reserve this power

At issue here is whether a retained power of revocation includes the power to amend sometimes referred to as the power to modify The Restatement (Second) of Trusts sect 331 cmt g provides that if a settlor has a power to revoke that retained power ordinarily includes a power to modify (amend) as well Comment g also notes that the power to amend includes both a power to withdraw trust assets and a power to ldquomodify the terms of the trustrdquo The Uniform Trust Code which provides that a power to revoke includes the power to amend is consistent with this view

38

Trusts and Future Interests Analysis

UNIF TRUST CODE sect 602 accord RESTATEMENT (THIRD) OF TRUSTS sect 63 cmt The theory is that even though a power to amend was not expressly retained by a settlor the goal of amendment assuming the power was not included in the power to revoke could easily be achieved by first revoking the trust and then creating a new trust with the same terms contemplated by the amendment To require this would put form over substance

Thus by expressly retaining the power to revoke the trust Settlor retained a power to amend the inter vivos trust despite her failure to expressly reserve this power

[NOTE Under the common law a trust is irrevocable unless the settlor expressly retains a power to revoke the trust Conversely under the Uniform Trust Code a trust is revocable unless the terms of the trust expressly provide otherwise See UNIF TRUST CODE sect 602 The Trust Codersquos position on revocation follows the minority view in the United States and is inconsistent with prior Restatements of Trusts (see Restatement (Second) of Trusts sect 330) Here the trust is revocable because Settlor expressly retained a power of revocation

The Uniform Trust Code has been adopted in 24 jurisdictions Alabama Arizona Arkansas District of Columbia Florida Kansas Maine Michigan Missouri Nebraska New Hampshire New Mexico North Carolina North Dakota Ohio Oregon Pennsylvania South Carolina Tennessee Utah Vermont Virginia West Virginia and Wyoming]

Point One(b) (10) Settlorrsquos amendment of the trust was valid despite her failure to have her signature to the trust amendment witnessed

Neither the common law nor state statutes require a trust instrument or an amendment to a trust instrument to be executed in accordance with the formalities prescribed for execution of a will Indeed an inter vivos trust that does not involve real estate can be created orally Under the Uniform Trust Code the only requirements for creating a valid inter vivos trust are intent the specification of beneficiaries and the designation of a trustee See UNIF TRUST CODE sect 402 accord RESTATEMENT (THIRD) OF TRUSTS sect 13

Here the amendment meets the requirements of both the Uniform Trust Code and the common law Thus the fact that Settlorrsquos signature was not witnessed when she signed the amendment to the trust does not make the amendment invalid

Point Two (20) Under the Uniform Testamentary Additions to Trusts Act a revocable trust may be amended at any time prior to the settlorrsquos death and the amendment applies to probate assets poured into the trust at the settlorrsquos death pursuant to the settlorrsquos will even when the will was executed prior to the date of the amendment

Historically property owned by an individual at her death passed to the individualrsquos heirs or to beneficiaries designated in a will executed with the formalities (writing signing witnessing) prescribed by state law However when a will devises property to the trustee of an inter vivos trust then the provisions of the trustmdashwhich may not have been executed in accordance with the formalities required for willsmdasheffectively determine who will receive the property Because of this possibility some early cases held that if an inter vivos trust was not executed with the same formalities required for a valid will then the trust was ineffective to dispose of probate assets poured into the trust at the settlorrsquos death pursuant to the settlorrsquos will

This line of cases has been overturned by the Uniform Testamentary Additions to Trusts Act (the Act) now Uniform Probate Code sect 2-511 Under the Act adopted in almost all

39

Trusts and Future Interest Analysis

jurisdictions a testamentary bequest to the trustee of an inter vivos trust established by the testator during his or her lifetime is valid if the trust is in writing it is identified in the testatorrsquos will and the trust instrument was executed before concurrently with or after the execution of the will Id The Act further specifies that such a bequest is valid even if the trust is amendable or revocable and that a later amendment applies to assets passing to the trust by a previously executed will

Thus because the trust amendment is valid its terms apply to assets received by Bank from Settlorrsquos estate

Point Three (10) If the trust amendment was valid then the trust assets including assets passing to the trust under Settlorrsquos will should go to University

Under the trust amendment all trust assets (including the assets of Settlorrsquos probate estate poured into the trust) pass to University The facts provide no basis for failing to comply with Settlorrsquos stated intentions

Point Four (30) If the trust amendment was invalid trust assets including assets received pursuant to Settlorrsquos will should be held in accordance with the terms of the original trust instrument because those terms do not violate the Rule Against Perpetuities

Under the dispositive terms of the original trust instrument Settlor created successive income interests in her surviving children and grandchildren with a remainder interest in her great-grandchildren Because the trust was revocable the period during which the common law Rule Against Perpetuities requires that interests vest (ie 21 years plus lives in being) began to run from the date Settlor no longer had a power of revocation (here her death) not the date on which the trust was created See JESSE DUKEMINIER STANLEY J JOHANSON JAMES LINDGREN amp ROBERT SITKOFF WILLS TRUSTS AND ESTATES 678 (7th ed 2005)

Under the common law Rule Against Perpetuities Settlorrsquos trust is thus valid At the time of Settlorrsquos death she was survived by no children one granddaughter and no great-grandchildren Because Settlor cannot have more children after her death the only income beneficiary of the trust is Settlorrsquos surviving granddaughter This granddaughter is the only person who can produce great-grandchildren of Settlor thus all great-grandchildren must of necessity be born during the lifetime of Settlorrsquos only surviving granddaughter who is a life in being The granddaughterrsquos interest vested at Settlorrsquos death and the great-grandchildrenrsquos interest will vest at the death of the granddaughter There is no need to wait the additional 21 years permitted under the Rule Thus under the common law and the statute given in the facts the nonvested interest in the great-grandchildren is valid

[NOTE Both modern wait-and-see statutes and the Uniform Statutory Rule Against Perpetuities upon which the statute in the facts is modeled provide that before using either reform to validate an otherwise invalid nonvested interest one should first determine if the nonvested interest violates the common law Rule If it does not then there is no need to reform This proposition which is applicable in all MEE user jurisdictions that have not simply abrogated the rule is tested by this problem]

40

NEGOTIABLE INSTRUMENTS ANALYSIS (Negotiable Instruments III IV V)

ANALYSIS

Legal Problems

(1)(a) What rights does a person in possession of a note that has been indorsed in blank by the payee have against the maker of the note

(1)(b) Which defenses may the maker of a note raise against a person entitled to enforce it who is not a holder in due course but is a transferee from a holder in due course

(2) What rights does a person entitled to enforce a note have against an indorser who transferred it for consideration with no warranties

(3) What rights does a person entitled to enforce a note have against a previous holder who transferred it as a gift without indorsing it

DISCUSSION

Summary

The niece is a holder of the note and is thus a person entitled to enforce it The chef the issuer of the note is obligated to pay it to the niece as the person entitled to enforce it The niece is not subject to any defense or claim of the chef relating to the improper repair of the oven because the niece has the rights of a holder in due course When the buyer bought the note from the repairman the buyer became a holder in due course of the note and thus took it free of any personal defenses the chef had against the repairman Even though the niece is not herself a holder in due course of the note the niece succeeded to the buyerrsquos rights as holder in due course and thus took free of the chefrsquos personal defenses

Because the chef refused to pay the note the niece can recover from the repairman on the repairmanrsquos obligation as indorser The niece cannot recover on the note against the buyer however because the buyer did not indorse the note (and thus incurred no indorserrsquos obligation) and the buyer did not receive any consideration for transfer of the note to the niece (and therefore made no transfer warranty)

[NOTE Although Article 9 of the Uniform Commercial Code governs the sale of promissory notes (a point that might be correctly noted by examinees) that Article does not determine the answer to any of the questions posed]

Point One(a) (20) The niece is the holder of the note and thus may enforce it against the chef who is the issuer of the note

The chef is the maker of the note and thus its issuer See UCC sectsect 3-103 3-105 The issuer of a note is obligated to pay it in accordance with its terms to a ldquoperson entitled to enforcerdquo it UCC sect 3-412 The niece is a ldquoperson entitled to enforcerdquo the note This is because the niece is the holder of the note and a holder of a note is a person entitled to enforce it UCC sect 3-301 The niece is the holder of the note because (i) the repairmanrsquos signature on the back of the note not

41

Negotiable Instruments Analysis

accompanied by words indicating a person to whom the note was made payable was a ldquoblank indorsementrdquo which had the effect of making the note a bearer instrument (ii) anyone in possession of a bearer instrument is a holder of it and (iii) the niece is in possession of the note See UCC sectsect 1-201(b)(21)(A) 3-204 and 3-205 Accordingly the chef has an obligation to the niece to pay the note in accordance with its terms and the niece may enforce that obligation

Point One(b) (40) The niece is not a holder in due course of the note but because she is a transferee from the buyer who was a holder in due course she has the same enforcement rights as the buyer Because the buyer as a holder in due course would have been able to enforce the note against the chef without being subject to defenses or claims arising from the improper repair the niece has the same rights and will not be subject to the chefrsquos defenses or claims about the repair

As noted in Point One(a) the chef has an obligation to the niece to pay the note in accordance with its terms However except against a person with the rights of a holder in due course the chef can raise any defenses or claims in recoupment that he would have if the claim on the note were an ordinary contract claim UCC sect 3-305 Thus except against a holder in due course the chef would be able to raise the improper repair as a defense or a claim in recoupment (a claim in response to the niecersquos claim)

But claims in recoupment and most defenses cannot be raised against a person with the rights of a holder in due course Against a holder in due course the chef can raise only the four ldquorealrdquo defenses listed in UCC sect 3-305(a)(1) (infancy duress lack of legal capacity or illegality that nullifies the obligation of the obligor under other law fraud in the factum discharge in insolvency proceedings) none of which is present here

The niece is not a holder in due course because she did not take the note for value See UCC sectsect 3-302(a)(2)(i) (criteria for holder in due course status) and 3-303(a) (definition of ldquovaluerdquo) But this does not mean that the niece is subject to the chefrsquos claim arising out of the improper repair The buyer was a holder in due course of the note because he took the note for value ($9500) in good faith and without notice of any facts that would have alerted him to the chefrsquos defense against the repairman UCC sect 3-302(a)(2) As a holder in due course the buyer owned the note free of the chefrsquos claim because that claim did not constitute a ldquorealrdquo defense UCC sect 3-305(b) When the buyer gave the note to the niece this constituted a ldquotransferrdquo of the note See UCC sect 3-203(a) When a note is transferred the transferee receives ldquoany right of the transferor to enforce the instrument including any right as a holder in due courserdquo UCC sect 3-203(b) Under this rule (also known as the ldquoshelter principlerdquo) the buyer transferred his freedom from the chefrsquos defenses to the niece and the niece can enforce the note free of the chefrsquos defenses

Point Two (20) Because the chef dishonored the note the niece can recover from the repairman on the repairmanrsquos obligation as indorser

The chefrsquos refusal to pay the note constituted dishonor See UCC sect 3-502 The repairman as an indorser of the note (see Point One(a)) incurred the obligations of an indorser under UCC sect 3-415(a) When a note has been dishonored one of the obligations of an indorser is to pay the amount of the note to a person entitled to enforce it Therefore the repairman is liable for the amount of the note to the niece a person entitled to enforce the note (so long as the niece gives proper notice of dishonor to the repairman)

42

Negotiable Instruments Analysis

[NOTE Because the repairman indorsed the note without warranties there are no transfer warranties UCC sect 3-416 cmt 5]

Point Three (20) The niece cannot recover on the note against the buyer as either indorser or warrantor because the buyer did not indorse the note and did not receive consideration for transferring the note to the niece

The buyer did not indorse the note and therefore did not incur the obligation of an indorser to pay the note upon dishonor

The niece cannot recover from the buyer under a transfer warranty theory because transfer warranties are made only by a person ldquowho transfers an instrument for considerationrdquo Here the buyer gave the instrument to the niece as a gift So the buyer made no transfer warranty UCC sect 3-416(a) Therefore the niece cannot recover from the buyer on that theory

43

National Conference of Bar Examiners 302 South Bedford Street | Madison WI 53703-3622 Phone 608-280-8550 | Fax 608-280-8552 | TDD 608-661-1275

wwwncbexorg e-mail contactncbexorg

  • Contents
  • Preface
  • Description of the MEE
  • Instructions
  • February 2013 Questions
    • Real Property Question
    • Contracts Question
    • Constitutional Law Question
    • Secured Transactions Question
    • Federal Civil Procedure Question
    • Agency Question
    • Evidence Question
    • Trusts and Future Interests Question
    • Negotiable Instruments Question
      • February 2013 Analyses
        • Real Property Analysis
        • Contracts Analysis
        • Constitutional Law Analysis
        • Secured Transactions Analysis
        • Federal Civil Procedure Analysis
        • Agency Analysis
        • Evidence Analysis
        • Trusts and Future Interests Analysis
        • Negotiable Instruments Analysis
            • ltlt ASCII85EncodePages false AllowTransparency false AutoPositionEPSFiles true AutoRotatePages None Binding Left CalGrayProfile (Dot Gain 20) CalRGBProfile (sRGB IEC61966-21) CalCMYKProfile (US Web Coated 050SWOP051 v2) sRGBProfile (sRGB IEC61966-21) CannotEmbedFontPolicy Error CompatibilityLevel 14 CompressObjects Tags CompressPages true ConvertImagesToIndexed true PassThroughJPEGImages true CreateJobTicket false DefaultRenderingIntent Default DetectBlends true DetectCurves 00000 ColorConversionStrategy CMYK DoThumbnails false EmbedAllFonts true EmbedOpenType false ParseICCProfilesInComments true EmbedJobOptions true DSCReportingLevel 0 EmitDSCWarnings false EndPage -1 ImageMemory 1048576 LockDistillerParams false MaxSubsetPct 100 Optimize true OPM 1 ParseDSCComments true ParseDSCCommentsForDocInfo true PreserveCopyPage true PreserveDICMYKValues true PreserveEPSInfo true PreserveFlatness true PreserveHalftoneInfo false PreserveOPIComments true PreserveOverprintSettings true StartPage 1 SubsetFonts true TransferFunctionInfo Apply UCRandBGInfo Preserve UsePrologue false ColorSettingsFile () AlwaysEmbed [ true ] NeverEmbed [ true ] AntiAliasColorImages false CropColorImages true ColorImageMinResolution 300 ColorImageMinResolutionPolicy OK DownsampleColorImages true ColorImageDownsampleType Bicubic ColorImageResolution 300 ColorImageDepth -1 ColorImageMinDownsampleDepth 1 ColorImageDownsampleThreshold 150000 EncodeColorImages true ColorImageFilter DCTEncode AutoFilterColorImages true ColorImageAutoFilterStrategy JPEG ColorACSImageDict ltlt QFactor 015 HSamples [1 1 1 1] VSamples [1 1 1 1] gtgt ColorImageDict ltlt QFactor 015 HSamples [1 1 1 1] VSamples [1 1 1 1] gtgt JPEG2000ColorACSImageDict ltlt TileWidth 256 TileHeight 256 Quality 30 gtgt JPEG2000ColorImageDict ltlt TileWidth 256 TileHeight 256 Quality 30 gtgt AntiAliasGrayImages false CropGrayImages true GrayImageMinResolution 300 GrayImageMinResolutionPolicy OK DownsampleGrayImages true GrayImageDownsampleType Bicubic GrayImageResolution 300 GrayImageDepth -1 GrayImageMinDownsampleDepth 2 GrayImageDownsampleThreshold 150000 EncodeGrayImages true GrayImageFilter DCTEncode AutoFilterGrayImages true GrayImageAutoFilterStrategy JPEG GrayACSImageDict ltlt QFactor 015 HSamples [1 1 1 1] VSamples [1 1 1 1] gtgt GrayImageDict ltlt QFactor 015 HSamples [1 1 1 1] VSamples [1 1 1 1] gtgt JPEG2000GrayACSImageDict ltlt TileWidth 256 TileHeight 256 Quality 30 gtgt JPEG2000GrayImageDict ltlt TileWidth 256 TileHeight 256 Quality 30 gtgt AntiAliasMonoImages false CropMonoImages true MonoImageMinResolution 1200 MonoImageMinResolutionPolicy OK DownsampleMonoImages true MonoImageDownsampleType Bicubic MonoImageResolution 1200 MonoImageDepth -1 MonoImageDownsampleThreshold 150000 EncodeMonoImages true MonoImageFilter CCITTFaxEncode MonoImageDict ltlt K -1 gtgt AllowPSXObjects false CheckCompliance [ None ] PDFX1aCheck false PDFX3Check false PDFXCompliantPDFOnly false PDFXNoTrimBoxError true PDFXTrimBoxToMediaBoxOffset [ 000000 000000 000000 000000 ] PDFXSetBleedBoxToMediaBox true PDFXBleedBoxToTrimBoxOffset [ 000000 000000 000000 000000 ] PDFXOutputIntentProfile () PDFXOutputConditionIdentifier () PDFXOutputCondition () PDFXRegistryName () PDFXTrapped False CreateJDFFile false Description ltlt ARA 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 BGR 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 CHS ltFEFF4f7f75288fd94e9b8bbe5b9a521b5efa7684002000410064006f006200650020005000440046002065876863900275284e8e9ad88d2891cf76845370524d53705237300260a853ef4ee54f7f75280020004100630072006f0062006100740020548c002000410064006f00620065002000520065006100640065007200200035002e003000204ee553ca66f49ad87248672c676562535f00521b5efa768400200050004400460020658768633002gt CHT ltFEFF4f7f752890194e9b8a2d7f6e5efa7acb7684002000410064006f006200650020005000440046002065874ef69069752865bc9ad854c18cea76845370524d5370523786557406300260a853ef4ee54f7f75280020004100630072006f0062006100740020548c002000410064006f00620065002000520065006100640065007200200035002e003000204ee553ca66f49ad87248672c4f86958b555f5df25efa7acb76840020005000440046002065874ef63002gt CZE ltFEFF005400610074006f0020006e006100730074006100760065006e00ed00200070006f0075017e0069006a007400650020006b0020007600790074007600e101590065006e00ed00200064006f006b0075006d0065006e0074016f002000410064006f006200650020005000440046002c0020006b00740065007200e90020007300650020006e0065006a006c00e90070006500200068006f006400ed002000700072006f0020006b00760061006c00690074006e00ed0020007400690073006b00200061002000700072006500700072006500730073002e002000200056007900740076006f01590065006e00e900200064006f006b0075006d0065006e007400790020005000440046002000620075006400650020006d006f017e006e00e90020006f007400650076015900ed007400200076002000700072006f006700720061006d0065006300680020004100630072006f00620061007400200061002000410064006f00620065002000520065006100640065007200200035002e0030002000610020006e006f0076011b006a016100ed00630068002egt DAN 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 DEU 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 ESP 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 ETI 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 FRA ltFEFF005500740069006c006900730065007a00200063006500730020006f007000740069006f006e00730020006100660069006e00200064006500200063007200e900650072002000640065007300200064006f00630075006d0065006e00740073002000410064006f00620065002000500044004600200070006f0075007200200075006e00650020007100750061006c0069007400e90020006400270069006d007000720065007300730069006f006e00200070007200e9007000720065007300730065002e0020004c0065007300200064006f00630075006d0065006e00740073002000500044004600200063007200e900e90073002000700065007500760065006e0074002000ea0074007200650020006f007500760065007200740073002000640061006e00730020004100630072006f006200610074002c002000610069006e00730069002000710075002700410064006f00620065002000520065006100640065007200200035002e0030002000650074002000760065007200730069006f006e007300200075006c007400e90072006900650075007200650073002egt GRE 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 HEB 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 HRV (Za stvaranje Adobe PDF dokumenata najpogodnijih za visokokvalitetni ispis prije tiskanja koristite ove postavke Stvoreni PDF dokumenti mogu se otvoriti Acrobat i Adobe Reader 50 i kasnijim verzijama) HUN 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 ITA 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 JPN ltFEFF9ad854c18cea306a30d730ea30d730ec30b951fa529b7528002000410064006f0062006500200050004400460020658766f8306e4f5c6210306b4f7f75283057307e305930023053306e8a2d5b9a30674f5c62103055308c305f0020005000440046002030d530a130a430eb306f3001004100630072006f0062006100740020304a30883073002000410064006f00620065002000520065006100640065007200200035002e003000204ee5964d3067958b304f30533068304c3067304d307e305930023053306e8a2d5b9a306b306f30d530a930f330c8306e57cb30818fbc307f304c5fc59808306730593002gt KOR ltFEFFc7740020c124c815c7440020c0acc6a9d558c5ec0020ace0d488c9c80020c2dcd5d80020c778c1c4c5d00020ac00c7a50020c801d569d55c002000410064006f0062006500200050004400460020bb38c11cb97c0020c791c131d569b2c8b2e4002e0020c774b807ac8c0020c791c131b41c00200050004400460020bb38c11cb2940020004100630072006f0062006100740020bc0f002000410064006f00620065002000520065006100640065007200200035002e00300020c774c0c1c5d0c11c0020c5f40020c2180020c788c2b5b2c8b2e4002egt LTH 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 LVI 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 NLD (Gebruik deze instellingen om Adobe PDF-documenten te maken die zijn geoptimaliseerd voor prepress-afdrukken van hoge kwaliteit De gemaakte PDF-documenten kunnen worden geopend met Acrobat en Adobe Reader 50 en hoger) NOR ltFEFF004200720075006b00200064006900730073006500200069006e006e007300740069006c006c0069006e00670065006e0065002000740069006c002000e50020006f0070007000720065007400740065002000410064006f006200650020005000440046002d0064006f006b0075006d0065006e00740065007200200073006f006d00200065007200200062006500730074002000650067006e0065007400200066006f00720020006600f80072007400720079006b006b0073007500740073006b00720069006600740020006100760020006800f800790020006b00760061006c0069007400650074002e0020005000440046002d0064006f006b0075006d0065006e00740065006e00650020006b0061006e002000e50070006e00650073002000690020004100630072006f00620061007400200065006c006c00650072002000410064006f00620065002000520065006100640065007200200035002e003000200065006c006c00650072002000730065006e006500720065002egt POL 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 PTB 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 RUM ltFEFF005500740069006c0069007a00610163006900200061006300650073007400650020007300650074010300720069002000700065006e007400720075002000610020006300720065006100200064006f00630075006d0065006e00740065002000410064006f006200650020005000440046002000610064006500630076006100740065002000700065006e0074007200750020007400690070010300720069007200650061002000700072006500700072006500730073002000640065002000630061006c006900740061007400650020007300750070006500720069006f006100720103002e002000200044006f00630075006d0065006e00740065006c00650020005000440046002000630072006500610074006500200070006f00740020006600690020006400650073006300680069007300650020006300750020004100630072006f006200610074002c002000410064006f00620065002000520065006100640065007200200035002e00300020015f00690020007600650072007300690075006e0069006c006500200075006c0074006500720069006f006100720065002egt RUS ltFEFF04180441043f043e043b044c04370443043904420435002004340430043d043d044b04350020043d0430044104420440043e0439043a043800200434043b044f00200441043e043704340430043d0438044f00200434043e043a0443043c0435043d0442043e0432002000410064006f006200650020005000440046002c0020043c0430043a04410438043c0430043b044c043d043e0020043f043e04340445043e0434044f04490438044500200434043b044f00200432044b0441043e043a043e043a0430044704350441044204320435043d043d043e0433043e00200434043e043f0435044704300442043d043e0433043e00200432044b0432043e04340430002e002000200421043e043704340430043d043d044b04350020005000440046002d0434043e043a0443043c0435043d0442044b0020043c043e0436043d043e0020043e0442043a0440044b043204300442044c002004410020043f043e043c043e0449044c044e0020004100630072006f00620061007400200438002000410064006f00620065002000520065006100640065007200200035002e00300020043800200431043e043b043504350020043f043e04370434043d043804450020043204350440044104380439002egt SKY 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 SLV 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 SUO ltFEFF004b00e40079007400e40020006e00e40069007400e4002000610073006500740075006b007300690061002c0020006b0075006e0020006c0075006f00740020006c00e400680069006e006e00e4002000760061006100740069007600610061006e0020007000610069006e006100740075006b00730065006e002000760061006c006d0069007300740065006c00750074007900f6006800f6006e00200073006f00700069007600690061002000410064006f0062006500200050004400460020002d0064006f006b0075006d0065006e007400740065006a0061002e0020004c0075006f0064007500740020005000440046002d0064006f006b0075006d0065006e00740069007400200076006f0069006400610061006e0020006100760061007400610020004100630072006f0062006100740069006c006c00610020006a0061002000410064006f00620065002000520065006100640065007200200035002e0030003a006c006c00610020006a006100200075007500640065006d006d0069006c006c0061002egt SVE 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 TUR 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 UKR ltFEFF04120438043a043e0440043804410442043e043204430439044204350020044604560020043f043004400430043c043504420440043800200434043b044f0020044104420432043e04400435043d043d044f00200434043e043a0443043c0435043d044204560432002000410064006f006200650020005000440046002c0020044f043a04560020043d04300439043a04400430044904350020043f045604340445043e0434044f0442044c00200434043b044f0020043204380441043e043a043e044f043a04560441043d043e0433043e0020043f0435044004350434043404400443043a043e0432043e0433043e0020043404400443043a0443002e00200020042104420432043e04400435043d045600200434043e043a0443043c0435043d0442043800200050004400460020043c043e0436043d04300020043204560434043a0440043804420438002004430020004100630072006f006200610074002004420430002000410064006f00620065002000520065006100640065007200200035002e0030002004300431043e0020043f04560437043d04560448043e04570020043204350440044104560457002egt ENU (Use these settings to create Adobe PDF documents best suited for high-quality prepress printing Created PDF documents can be opened with Acrobat and Adobe Reader 50 and later) gtgt Namespace [ (Adobe) (Common) (10) ] OtherNamespaces [ ltlt AsReaderSpreads false CropImagesToFrames true ErrorControl WarnAndContinue FlattenerIgnoreSpreadOverrides false IncludeGuidesGrids false IncludeNonPrinting false IncludeSlug false Namespace [ (Adobe) (InDesign) (40) ] OmitPlacedBitmaps false OmitPlacedEPS false OmitPlacedPDF false SimulateOverprint Legacy gtgt ltlt AddBleedMarks false AddColorBars false AddCropMarks false AddPageInfo false AddRegMarks false ConvertColors ConvertToCMYK DestinationProfileName () DestinationProfileSelector DocumentCMYK Downsample16BitImages true FlattenerPreset ltlt PresetSelector MediumResolution gtgt FormElements false GenerateStructure false IncludeBookmarks false IncludeHyperlinks false IncludeInteractive false IncludeLayers false IncludeProfiles false MultimediaHandling UseObjectSettings Namespace [ (Adobe) (CreativeSuite) (20) ] PDFXOutputIntentProfileSelector DocumentCMYK PreserveEditing true UntaggedCMYKHandling LeaveUntagged UntaggedRGBHandling UseDocumentProfile UseDocumentBleed false gtgt ]gtgt setdistillerparamsltlt HWResolution [2400 2400] PageSize [612000 792000]gtgt setpagedevice

Page 11: February 2013 MEE Questions and Analyses

FEDERAL CIVIL PROCEDURE QUESTION _____

Mother and Son who are both adults are citizens and residents of State A Mother owned an expensive luxury car valued in excess of $100000 Son borrowed Motherrsquos car to drive to a store in State A As Son approached a traffic light that had just turned yellow he carefully braked and brought the car to a complete stop Driver who was following immediately behind him failed to stop and rear-ended Motherrsquos car which was damaged beyond repair Son was seriously injured Driver is a citizen of State B

Son sued Driver in the United States District Court for the District of State A alleging that she was negligent in the operation of her vehicle Son sought damages in excess of $75000 for his personal injuries exclusive of costs and interest In her answer Driver alleged that Son was contributorily negligent in the operation of Motherrsquos car She further alleged that the brake lights on Motherrsquos car were burned out and that Motherrsquos negligent failure to properly maintain the car was a contributing cause of the accident

Following a trial on the merits in Sonrsquos case against Driver the jury answered the following special interrogatories

Do you find that Driver was negligent in the operation of her vehicle Yes

Do you find that Son was negligent in the operation of Motherrsquos car No

Do you find that Mother negligently failed to ensure that the brake lights on her car were in proper working order Yes

The judge then entered a judgment in favor of Son against Driver Driver did not appeal

Two months later Mother sued Driver in the United States District Court for the District of State A alleging that Driverrsquos negligence in the operation of her vehicle destroyed Motherrsquos luxury car Mother sought damages in excess of $75000 exclusive of costs and interest

State A follows the same preclusion principles that federal courts follow in federal-question cases

1 Is Motherrsquos claim against Driver barred by the judgment in Son v Driver Explain

2 Does the juryrsquos conclusion in Son v Driver that Mother had negligently failed to maintain the brake lights on her car preclude Mother from litigating that issue in her subsequent suit against Driver Explain

3 Does the juryrsquos conclusion in Son v Driver that Driver was negligent preclude Driver from litigating that issue in the Mother v Driver lawsuit Explain

7

_____

AGENCY QUESTION

Over 5000 individuals in the United States operate hot-air balloon businesses A hot-air balloon has four key components the balloon that holds the heated air the basket that houses the riders the propane burner that heats the air in the balloon and the propane storage tanks

The owner of a hot-air balloon business recently notified several basket and burner manufacturers that she or her agent might be contacting them to purchase baskets or burners The owner did not specifically name any person as her agent Basket and burner manufacturers regularly receive such notices from hot-air balloon operators Such notices typically include no restrictions on the types of baskets or burners agents might purchase for their principals

The owner then retained an agent to acquire baskets burners and fuel tanks from various manufacturers The owner authorized the agent to buy only (a) baskets made of woven wicker (not aluminum) (b) burners that use a unique ldquowhisper technologyrdquo (so as not to scare livestock when the balloon sails over farmland) and (c) propane fuel tanks

The agent then entered into three transactions with manufacturers all of whom had no prior dealings with either the owner or the agent

(1) The agent and a large manufacturer of both wicker and aluminum baskets signed a contract for the purchase of four aluminum baskets for a total cost of $60000 The agent never told the manufacturer that he represented the owner or any other principal The contract listed the agent as the buyer and listed the ownerrsquos address as the delivery address but did not indicate that the address was that of the owner rather than the agent When the baskets were delivered to the owner she learned for the first time that the agent had contracted to buy aluminum not wicker baskets The owner immediately rejected the baskets and returned them to the manufacturer Neither the owner nor the agent has paid the basket manufacturer for them

(2) The agent contacted a burner manufacturer and told him that the agent represented a well-known hot-air balloon operator who wanted to purchase burners The agent did not disclose the ownerrsquos name The agent and the burner manufacturer signed a contract for the purchase of four burners that did not have ldquowhisper technologyrdquo for a total price of $70000 The burner contract like the basket contract listed the ownerrsquos address for delivery but did not disclose whose address it was The burners were delivered to the ownerrsquos business and the owner discovered that the agent had ordered the wrong kind of burners The owner rejected the burners and returned them to the manufacturer Neither the owner nor the agent has paid the burner manufacturer for the burners

(3) The agent contracted with a solar cell manufacturer to make three cells advertised as ldquostrong enough to power all your ballooning needsrdquo The agent did not tell the manufacturer that he was acting on behalf of any other person One week after the cells were delivered to the agent he took them to the owner who installed them and discovered that she could save a lot of money using solar cells instead of propane to power her balloons The owner decided to keep the solar cells but she has not paid the manufacturer for them

8

Agency Question

Assume that the rejection of the baskets and the burners and the failure to pay for the solar cells constitute breach of the relevant contracts

1 Is the owner liable to the basket manufacturer for breach of the contract for the aluminum baskets Is the agent liable Explain

2 Is the owner liable to the burner manufacturer for breach of the contract for the burners Is the agent liable Explain

3 Is the owner liable to the solar cell manufacturer for breach of the contract for the solar cells Is the agent liable Explain (Do not address liability based upon restitution or unjust enrichment)

9

EVIDENCE QUESTION _____

A woman who owns a motorized scooter brought her scooter to a mechanic for routine maintenance service As part of the maintenance service the mechanic inspected the braking system on the scooter As soon as the mechanic finished inspecting and servicing the scooter he sent the woman a text message to her cell phone that read ldquoJust finished your service When you pick up your scooter you need to schedule a follow-up brake repair Wersquoll order the partsrdquo

The woman read the mechanicrsquos text message and returned the next day to pick up her scooter As the woman was wheeling her scooter out of the shop she saw the mechanic working nearby and asked ldquoIs my scooter safe to ride for a whilerdquo The mechanic responded by giving her a thumbs-up The woman waved and rode away on the scooter

One week later while the woman was riding her scooter a pedestrian stepped off the curb into a crosswalk and the woman collided with him causing the pedestrian severe injuries The woman had not had the scooterrsquos brakes repaired before the accident

The pedestrian has sued the woman for damages for his injuries resulting from the accident The pedestrian has alleged that (1) the woman lost control of the scooter due to its defective brakes (2) the woman knew that the brakes needed repair and (3) it was negligent for the woman to ride the scooter knowing that its brakes needed to be repaired

The woman claims that the brakes on the scooter worked perfectly and that the accident happened because the pedestrian stepped into the crosswalk without looking and the woman had no time to stop The woman the pedestrian and the mechanic will testify at the upcoming trial

The pedestrian has proffered an authenticated copy of the mechanicrsquos text message to the woman

The woman plans to testify that she asked the mechanic ldquoIs my scooter safe to ride for a whilerdquo and that he gave her a thumbs-up in response

The evidence rules in this jurisdiction are identical to the Federal Rules of Evidence

Analyze whether each of these items of evidence is relevant and admissible at trial

1 The authenticated copy of the mechanicrsquos text message

2 The womanrsquos testimony that she asked the mechanic ldquoIs my scooter safe to ride for a whilerdquo and

3 The womanrsquos testimony describing the mechanicrsquos thumbs-up

10

TRUSTS AND FUTURE INTERESTS QUESTION

Ten years ago Settlor validly created an inter vivos trust and named Bank as trustee The trust instrument provided that Settlor would receive all of the trust income during her lifetime The trust instrument further provided that

Upon Settlorrsquos death the trust income shall be paid in equal shares to Settlorrsquos surviving children for their lives Upon the death of the last surviving child the trust income shall be paid in equal shares to Settlorrsquos then-living grandchildren for their lives Upon the death of the survivor of Settlorrsquos children and grandchildren the trust corpus shall be distributed in equal shares to Settlorrsquos then-living great-grandchildren

The trust instrument expressly specified that the trust was revocable but it was silent regarding whether Settlor could amend the trust instrument

Immediately after creating the trust Settlor validly executed a will leaving her entire estate to Bank as trustee of her inter vivos trust to ldquohold in accordance with the terms of the trustrdquo

Five years ago Settlor signed an amendment to the inter vivos trust The amendment changed the disposition of the remainder interest specifying that all trust assets ldquoshall be paid upon Settlorrsquos death to Universityrdquo Settlorrsquos signature on this amendment was not witnessed

A state statute provides that any trust interest that violates the common law Rule Against Perpetuities ldquois nonetheless valid if the nonvested interest in the trust actually vests or fails to vest either (a) within 21 years of lives in being at the creation of the nonvested interest or (b) within 90 years of its creationrdquo

Recently Settlor died leaving a probate estate of $200000 She was survived by no children one granddaughter (who would be Settlorrsquos only heir) and no great-grandchildren The granddaughter has consulted your law firm and has raised four questions regarding this trust

1 Was Settlorrsquos amendment of the inter vivos trust valid Explain

2 Assuming that the trust amendment was valid do its provisions apply to Settlorrsquos probate assets Explain

3 Assuming that the trust amendment was valid how should trust assets be distributed Explain

4 Assuming that the trust amendment was invalid how should trust assets be distributed Explain

11

NEGOTIABLE INSTRUMENTS QUESTION

A chef entered into a contract with a repairman pursuant to which the repairman agreed to repair the chefrsquos commercial oven for $10000 The repairman agreed to accept as payment a negotiable promissory note for $10000 payable two months after its issuance

After the repairman worked on the oven the chef gave him a $10000 note as payment for the work As agreed the note was signed by the chef as maker was payable to the order of the repairman was payable in two months and fulfilled all criteria for negotiability

The next day the repairman sold the note to a buyer for $9500 To effectuate the sale the repairman wrote ldquono warrantiesrdquo on the back of the note signed his name immediately below that and handed the note to the buyer The buyer bought the note in good faith and without knowledge of any facts relating to the work that the repairman had performed for the chef

Later the buyer gave the note to his niece as a gift To effectuate the gift the buyer handed the note to the niece but did not indorse it

Shortly thereafter the chef discovered that the repair work had been done improperly and the oven still did not function correctly The chef tried repeatedly to get the repairman to return to correct the repair work but the repairman ignored all the chefrsquos calls

On the notersquos due date the niece contacted the chef and demanded that he pay the amount of the note to her The chef refused and told the niece that he would not pay the note because the repairman did not properly repair the oven

1 What are the niecersquos rights against the chef Explain

2 What are the niecersquos rights against the repairman Explain

3 What are the niecersquos rights against the buyer Explain

12

February 2013 MEE

ANALYSES

Real Property Contracts

Constitutional Law Secured Transactions

Federal Civil Procedure Agency

Evidence Trusts and Future Interests

Negotiable Instruments

REAL PROPERTY ANALYSIS (Real Property ID1a 4 amp 5)

ANALYSIS

Legal Problems

(1) Does the tenant have a defense to the landlordrsquos action for unpaid rent based on constructive eviction

(2) Does the tenant have a defense to the landlordrsquos action for unpaid rent based on the tenantrsquos surrender of the premises

(3) What if anything may the landlord recover from the tenant for the period after the tenant vacated the building

DISCUSSION

Summary

Under the common law the tenant does not have a defense to the landlordrsquos action for unpaid rent based on constructive eviction Constructive eviction is based on the tenant proving that (1) the landlord breached a duty to the tenant (2) the breach caused a loss by the tenant of the substantial use and enjoyment of the premises (3) the tenant gave the landlord adequate notice and opportunity to repair and (4) the tenant vacated the leased premises Here there was no constructive eviction because although the tenant vacated and gave the landlord adequate notice the landlord breached no express or implied duty to the tenant to repair the premises

The tenant does not have a defense based on the landlordrsquos acceptance of his surrender of the premises a landlordrsquos retention of keys does not constitute an acceptance of the tenantrsquos surrender unless the landlord so intended and here the landlordrsquos statements to the tenant at the time of the surrender of the keys do not evidence the intent to accept the tenantrsquos surrender

Under the common law a landlord has no duty to mitigate damages but also cannot sue for rents due in the future Under this approach the landlord can sue only for past-due rents Using this approach on November 1 the landlord could recover all the rent past due (ie rent for September and October) but could not recover for rents due in the future However some courts have authorized recovery for future rent minus the fair market rental value of the premises It is thus possible that the landlord could recover damages equal to the amount of rent due from September 1 to the end of the six-year lease term ($180000) minus the propertyrsquos fair-market rental value over that same period

Point One (45) The tenant was not constructively evicted because the landlord had no duty to repair the commercial premises that were the subject of the lease

The landlord and the tenant entered into a term-of-years lease because the lease specified both a beginning and an ending date HERBERT HOVENKAMP amp SHELDON F KURTZ THE LAW OF

PROPERTY 256 (5th ed 2001) Although a term-of-years lease normally cannot be terminated by the tenant prior to the end of the term a tenant may terminate a term-of-years lease if the tenant

15

Real Property Analysis

is constructively evicted See id at 286ndash88 Typically as here a claim of constructive eviction is made as a defense to a landlordrsquos action for damages or unpaid rent

In order to establish a constructive eviction the tenant must prove that the landlord breached a duty to the tenant such as a duty to repair and that the landlordrsquos breach caused a loss of the substantial use and enjoyment of the premises The tenant must also show that he gave the landlord notice adequate to permit the landlord to meet his duty to the tenant and that the tenant vacated the leased premises Id see also JOHN G SPRANKLING UNDERSTANDING

PROPERTY LAW sect 1704 (2d ed 2007) Under the common law there was no implied duty on the part of a landlord to repair

leased premises such a duty arose only if expressly set forth in the lease SPRANKLING supra sect 1702[B] Here the written lease contained no term requiring the landlord to repair the air-conditioning Even if the conversation created a lease term that the building had air-conditioning that itself should not create a duty for the landlord to repair it

Over the past several decades courts have generally implied a duty to repair in residential leases either as part of a revised constructive eviction doctrine or based on an implied warranty of habitability JOSEPH W SINGER PROPERTY 469ndash70 (3d ed 2010) This shift has been justified based on the economic disparity between the typical landlord and tenant as well as the fact that residential tenants generally lack both the authority to authorize repairs to common areas of a building and the incentive to make repairs that will ultimately benefit the landlord

However courts have been more reluctant to imply a duty to repair in commercial leases a context in which the tenant is often a valuable business and in a better position to assess and make repairs than is the landlord But see eg Davidow v Inwood North Professional Group 747 SW2d 373 (Tex 1988) When courts have implied a duty to repair in a commercial lease it is typically when the repair has been mandated by public authorities and involves work so substantial that it would not ordinarily fall within the tenantrsquos common law repair duty andor the value of the repair would primarily inure to the landlordrsquos reversionary interest See Brown v Green 884 P2d 55 (Cal 1994) Eugene L Grant et al The Tenant as Terminator Constructive Eviction in Modern Commercial Leases 2 THE COMMERCIAL PROPERTY LEASE ch 15 (ABA 1997) Some courts have also permitted constructive eviction claims by commercial tenants of office buildings based on repairs required in common areas of the building See id Echo Consulting Services Inc v North Conway Bank 669 A2d 227 (NH 1995)

Here the tenant is the owner of a valuable manufacturing operation and is the exclusive occupant of the building the repair has not been mandated by public authorities and the repair is not structural To the contrary the repair involves a feature of the building of unusual importance in the tenantrsquos manufacturing operation and the tenant is likely far more knowledgeable than the landlord about the air-conditioning specifications necessary for the manufacture of the tenantrsquos product

Based on these facts it is unlikely that a court will find that the tenant in this case was constructively evicted Although the tenant can show that he gave adequate notice to the landlord of the air-conditioning malfunction and vacated the premises the lease was commercial and it did not contain any promises or covenants by the landlord except a covenant of quiet enjoyment a covenant of quiet enjoyment does not entail any repair obligations

[NOTE An examineersquos conclusion is less important than his or her demonstrated awareness of the elements of constructive eviction and the need to imply a repair duty for such a defense to be viable here Although the implied warranty of habitability is not available to this tenant Texas Minnesota and Massachusetts imply a warranty of suitability in commercial leases in limited circumstances and an examinee might argue that this warranty should apply

16

Real Property Analysis

here If an examinee concludes that this warranty applies he or she should discuss the other requirements for constructive eviction

If the examinee wrongly concludes that the first element for a constructive eviction has been met the examinee will then have to discuss the remaining three elements in order to conclude that the tenant can claim constructive eviction The tenant would have a strong argument that the second elementmdashsubstantial interference with the use and enjoyment of the premisesmdashalso is met As indicated above the landlord was aware that a functioning air-conditioning system was vital to the tenantrsquos manufacturing operations The facts further indicate that the system had failed three times in the past few months The landlord may try to argue that the malfunctions did not substantially interfere with the tenantrsquos use of the premises because the malfunctions caused the temperature to climb above 81 degrees for only a short period of timemdash 3 hours 6 hours and 10 hours respectivelymdashon each occasion The tenant will argue however that the landlord was aware that the tenantrsquos manufacturing operations could tolerate temperatures above 81 degrees for no more than 6 hours The final malfunction exceeded that limit destroying $150000 worth of the tenantrsquos products

The tenant would also have a strong argument that the third element is met notice and opportunity to cure The tenant notified the landlord of the problem immediately upon the systemrsquos first malfunction and did so again when it malfunctioned a second time and then a third time The landlord might argue that there was insufficient time to cure the problem because the system corrected itself within a few hours on the first and second times Although the malfunction lasted more than 10 hours the third time the landlord might argue that the time period was insufficient to get a repair person on the premises A court would be likely to find this argument unpersuasive however because the landlord could have attempted to correct the problem after the first and second malfunctions

Assuming that the landlord was given sufficient notice and opportunity to cure a court would be likely to conclude that the tenant also satisfied the final element of vacating the premises within a reasonable time The landlord might argue that the tenant remained in the premises for almost four months after the air conditioning first failed which would suggest that the problem was not so severe as to have constructively evicted the tenant The tenant will argue however that he gave the landlord three months to cure the problem after the first two malfunctions threatened (but did not actually harm) his operations The tenant then moved out shortly after the final malfunction caused temperatures to exceed the tolerance levels of his manufacturing operations]

Point Two (10) The landlord did not accept the tenantrsquos surrender of the lease

When a tenant wrongfully moves from leased premises with the intent to terminate the lease the landlord may either accept the tenantrsquos surrender of the premises and terminate the lease or hold the tenant to the terms of the lease See HOVENKAMP amp KURTZ supra at 295ndash96 Here the tenantrsquos only basis for the claim that the landlord accepted his surrender is the landlordrsquos retention of the keys Many courts have considered whether a landlordrsquos retention of keys delivered by a tenant constitutes acceptance of surrender The weight of the case law holds that retention of the keys alone does not constitute acceptance of surrender without other evidence showing that the landlord intended to accept the surrender See generally 49 AM JUR 2d Landlord and Tenant sect 213

Here the landlordrsquos note saying ldquoI repeat the air-conditioning is not my problem You have leased the building and you should fix itrdquo strongly suggests that the landlord did not intend

17

Some courts have rejected the no-mitigation-of-damages rule based on efficiency concerns and societyrsquos interest in assuring that resources remain in the stream of commerce rather than lying vacant see id at 464ndash65 and allow landlords to sue tenants who have wrongfully terminated a lease for damages equal to the difference between the unpaid rent due under the lease and the propertyrsquos fair market rental value Other courts have abandoned the no-recovery-for-future-rent rule These courts responding to the fact that a tenant may well disappear or be judgment-proof by the time a lease term is concluded have allowed a landlord to collect damages equal to the value of rent over the entire lease term minus the propertyrsquos fair rental value when a tenant has wrongfully terminated a lease and unequivocally shown an intention not to return to the premises or pay future rent Under this approach a landlord receives approximately the same amount he would have received were there a duty to mitigate damages See Sagamore Corp v Willcutt 180 A 464 (Conn 1935)

Real Property Analysis

to accept the tenantrsquos surrender The tenant might argue that the landlordrsquos failure to make a similar statement when the keys were sent to her a second time and she retained them evidences a change of heart However it is likely that a court would find that the landlordrsquos retention of the keys represented a decision to safeguard the keys not to accept the tenantrsquos surrender

[NOTE An examinee should receive credit for arguing the other way with a well-reasoned argument]

Point Three (45) Under the common law the landlord had no duty to mitigate damages Additionally a landlord was not entitled to recover unpaid rents due in the future but was only entitled to recover rents in arrears at the time of the commencement of the suit Applying the common law here the landlord could recover $5000 the amount of rents due at the commencement of the suit ($2500 for September and the same for October) Today some courts allow the landlord under certain circumstances to sue the tenant for damages (not rent) equal to the difference if any between the unpaid promised rent for the balance of the term (here $175000) and the propertyrsquos fair rental value for the balance of the term

Under the common law because a lease was viewed as a conveyance instead of a contract a landlord had no duty to mitigate damages resulting from a tenantrsquos wrongful termination of a lease A landlord could thus recover the full value of rents that were due and unpaid at the time of the suit However under the common law a landlord could not sue a tenant for rents due in the future because there was always a possibility that the tenant might pay the rent when it was due See SINGER supra at 462 Thus using the common law approach on November 1 the landlord could only recover the full value of the two monthsrsquo rent actually due and unpaid ie $5000 for September and October

Here because the tenant returned the keys to the landlord and said ldquoI will not be returning to the building or making further rent paymentsrdquo the landlord could establish abandonment and an intention not to return It is thus possible that the landlord might recover damages in the amount of $5000 (for the months of September and October) plus the present value of $175000 minus the fair market rental value of the property over the remaining months of the lease

18

CONTRACTS ANALYSIS ____ (Contracts II IVE)

ANALYSIS

Legal Problems

(1) What was the legal effect of the sailorrsquos October 31 letter to the builder

(2)(a) What was the legal effect of the builderrsquos November 25 response to the sailorrsquos October 31 letter

(2)(b) What was the legal effect of the sailorrsquos refusal to take and pay for the boat on December 15

DISCUSSION

Summary

This is a sale of goods governed by the Uniform Commercial Code Because the sailor had reasonable grounds for insecurity about the builderrsquos ability to deliver the boat in a timely manner when the sailor learned about the strike on October 31 the sailor was legally justified in sending the letter to the builder seeking adequate assurance of the builderrsquos performance pursuant to the contract The builderrsquos failure to provide such assurance within a reasonable time operated as a repudiation of the contract However the builder was free to retract the repudiation before the sailor either cancelled the contract or materially changed position in reliance on the builderrsquos repudiation The builder retracted the repudiation when he informed the sailor that the workers were back and that the boat would be delivered by the date stipulated in the partiesrsquo contract Because the sailor had taken no action in response to the original repudiation he no longer had the right to cancel the contract with the builder The sailorrsquos subsequent statement that ldquoour contract is overrdquo may have constituted repudiation by the sailor In any event when the sailor failed to perform on December 15 that constituted breach

Point One (35) Because the sailor had reasonable grounds for insecurity with respect to the builderrsquos performance the sailorrsquos letter to the builder was a justified demand seeking assurance of the builderrsquos performance under the contract failure of the builder to provide such assurance within a reasonable time constituted repudiation of the contract

The sailor was legally justified in sending the letter to the builder on October 31 Contract parties are entitled to expect due performance of contractual obligations and are permitted to take steps to protect that expectation UCC sect 2-609 states that ldquo[w]hen reasonable grounds for insecurity arise with respect to the performance of either party the other may in writing demand adequate assurance of due performance rdquo Here the sailor learned on October 31 that the builderrsquos workers were on strike This gave the sailor reasonable grounds for insecurity about the builderrsquos ability to complete performance on time and thus gave the sailor the right to seek adequate assurance from the builder Because the sailorrsquos demand for assurance was justified the builder was required to provide assurance that was adequate under the circumstances within a reasonable time (not to exceed 30 days) or be held to have repudiated the contract UCC sect 2-609(4)

19

Contracts Analysis

Point Two(a) (30) The builder did not within a reasonable time provide the sailor adequate assurance of due performance this failure to provide assurance constituted a repudiation of the contract

Because the sailor with legal justification (see Point One) demanded from the builder assurance of due performance the builderrsquos failure to provide such assurance within a reasonable time was a repudiation of their contract See UCC sect 2-609(4) (ldquoAfter receipt of a justified demand[] failure to provide within a reasonable time not exceeding thirty days assurance of due performance is a repudiation of the contractrdquo) On October 31 the sailor requested that the builder provide adequate assurance regarding the completion of the boat by December 15 The builder did not respond to the sailorrsquos letter until November 25mdashnearly a month later Even if that response had been given in a reasonable time it nonetheless did not provide assurance of due performance It simply stated ldquoIrsquom sorry about the strike but it is really out of my hands I hope we settle it soon so that we can get back to workrdquo Therefore the builderrsquos November 25 response did not provide adequate assurance in response to the sailorrsquos justified request Thus the builder had repudiated the contract

Point Two(b) (35) Although the builder repudiated the contract with the sailor the builder probably retracted that repudiation on December 3 and the sailor was no longer entitled to cancel their contract Thus the sailorrsquos failure to perform the sailorrsquos obligations under the contract constituted a breach

The builderrsquos failure to provide adequate assurance of performance constituted a repudiation of their contract (see UCC sect 2-609(4)) but the builder was free to retract that repudiation until the sailor cancelled the contract or materially changed his position or indicated by communication or action that the sailor considered the repudiation to be final See UCC sect 2-611(1) (ldquoUntil the repudiating partyrsquos next performance is due he can retract his repudiation unless the aggrieved party has since the repudiation cancelled or materially changed his position or otherwise indicated that he considers the repudiation finalrdquo)

Here the facts state that before the builderrsquos December 3 telephone call to the sailor the sailor did nothing in response to the builderrsquos repudiation such as contracting with a third party for a boat The builderrsquos December 3 call informing the sailor that the boat would be timely delivered probably constituted a retraction of the repudiation because it clearly indicated to the sailor that the builder would be able to perform UCC sect 2-611(2) Thus after being so informed the sailor did not have the right to treat their contract as cancelled UCC sect 2-611(3) Accordingly the sailorrsquos failure to perform the sailorrsquos obligations under the contract by taking the boat and paying for it constituted a breach of the contract

20

CONSTITUTIONAL LAW ANALYSIS (Constitutional Law IVA F2b amp e)

ANALYSIS

Legal Problems

(1) Does AutoCorsquos operation of a ldquocompany townrdquo result in its actions counting as those of the state for purposes of constitutional analysis

(2) Does the expulsion of a schoolchild for failure to recite the Pledge of Allegiance violate the First Amendment as applied through the Fourteenth Amendment

(3) Does the arrest of a pamphleteer in connection with violation of an anti-littering rule where the littering is done by the recipients of leaflets distributed by the pamphleteer violate the First Amendment as applied through the Fourteenth Amendment

DISCUSSION

Summary

The First Amendment as applied through the Fourteenth Amendment applies only to state action It does not typically govern private actors However courts have found state action where the private actor has exercised a ldquopublic functionrdquo such as running a privately owned ldquocompany townrdquo as AutoCo has done here Thus First Amendment protections apply By requiring the son to participate in a mandatory Pledge of Allegiance ceremony AutoCo has compelled the expression of political belief in violation of the First Amendment as applied through the Fourteenth Amendment The fatherrsquos arrest in connection with breaching the anti-litter rule also violated the First Amendment as applied through the Fourteenth Amendment Although state actors can regulate the incidental effects of speech on the public streets on a content-neutral basis this power is limited and cannot extend to punishing a distributor of literature because of littering by third parties

Point One (30) AutoCorsquos operation of a company town (including a school) makes it a state actor under the public function strand of the state action doctrine

The individual rights protections of the Constitution apply only where there is ldquostate actionrdquomdash either direct action by the government or some action by a private party that is fairly attributable to the government As a general rule the actions of a private company like AutoCo or of a private school like the school operated by AutoCo would not constitute state action and the protections of the Constitution (in this case the First Amendment) would not apply

However there are situations in which the actions of a private actor are attributed to the state One such situation is when the private actor undertakes a public function There are not many bright-line rules in the Supreme Courtrsquos state action doctrine but one of them is this Where a private actor undertakes a ldquopublic functionrdquo the Constitution applies to those actions Where a corporation operates a privately owned ldquocompany townrdquo that provides essential services typically provided by a state actor the public function doctrine applies and the Constitution

21

Constitutional Law Analysis

binds agents of the town as if they were agents of the government See eg Marsh v Alabama 326 US 501 (1946) Here AutoCo does more than own the town it provides security services fire protection sanitation services and a school Thus the actions of AutoCo constitute state action and are governed by the Fourteenth Amendment

Point Two (35) The sonrsquos expulsion for failure to recite the Pledge of Allegiance violates the First Amendment as applied through the Fourteenth Amendment as a compelled expression of political belief

As explained in Point One the First Amendment applies to the school as a state actor Although children in public schools (and in schools subject to the First Amendment like

the Oakwood school) have some First Amendment rights Tinker v Des Moines Independent Community School District 393 US 503 506 (1969) schools have greater leeway to regulate the speech of students and teachers than the state would have outside the school context Hazelwood School Dist v Kuhlmeier 484 US 260 (1988) Morse v Frederick 551 US 393 (2007) However the Supreme Court has long held that public schools may not force their students to participate in a flag salute ceremony when it offends the political or religious beliefs of the students or their families West Virginia Board of Educ v Barnette 319 US 624 (1943) (invalidating a mandatory public school flag salute ceremony) see also Wooley v Maynard 430 US 705 (1977) (invalidating compelled expression of political belief on state-issued license plates)

In this case the school requires its students to participate in a flag salute and Pledge of Allegiance ceremony and punishes them when they refuse to participate Pursuant to this policy the school has expelled the son This expulsion violates the First Amendment ban on compelled expression

Point Three (35) Because the father was distributing leaflets in a traditional public forum his trespass arrest violated the First Amendment as applied through the Fourteenth Amendment

As explained in Point One AutoCo is treated as a state actor Thus Oakwoodrsquos commercial district is treated as government-owned property for purposes of the First Amendment Thus the leafleting here is subject to the First Amendment because it is an expressive activity Schneider v State of New Jersey Town of Irvington 308 US 147 (1939) When expression takes place on government-owned property government regulation of the expression is assessed under the public forum doctrine Public streets and sidewalks have long been held to be the classic example of a ldquotraditional public forumrdquo open to the public for expression Hague v CIO 307 US 496 515ndash16 (1939) Because the father was distributing leaflets while standing on a street corner in the commercial district his expressive activity occurred in a traditional public forum

When a state tries to regulate expressive activity in a traditional public forum it is prohibited from doing so based on the expressive activityrsquos content unless its regulation is narrowly tailored to achieve a compelling governmental interest (ldquostrict scrutinyrdquo) In this case however AutoCo is regulating the fatherrsquos expressive activity on the ostensibly neutral ground that his expressive activity has produced litter and made the street unsightly When a state tries to regulate expressive activity without regard to its content intermediate scrutiny applies Under intermediate scrutiny the true purpose of the regulation may not be the suppression of ideas (if so then strict scrutiny applies) the regulation must be narrowly tailored to achieve a significant

22

Constitutional Law Analysis

governmental interest and it must leave open ample alternative channels for expressive activity Ward v Rock Against Racism 491 US 781 791 (1989)

Here the application of the ordinance to the father will fail for two reasons First the Supreme Court has held that the governmentrsquos interest in keeping the streets clean is insufficient to ban leafleting in the public streets as the government power to regulate with incidental effects on public sidewalk speech is very limited See eg Schneider 308 US at 162 (leafletinglittering) Second the regulation (a blanket ban on distribution that results in littering) is not narrowly tailored to protect expression A narrowly tailored alternative would be prosecution only of people who litter Moreover the effect of the littering rule is likely to be a ban on all leafleting thus eliminating an entire class of means of expression This raises the possibility that there are not ldquoample alternative channels of communicationrdquo open to the father as required under the Courtrsquos standard of review for content-neutral regulation of speech

[NOTE Some examinees might argue that this is a ldquotime place and mannerrdquo restriction and that AutoCo might have greater latitude to regulate the public sidewalks under this theory This argument is incorrect for two reasons First the Supreme Court has held that the power to regulate speakers through littering laws is very limited for the reasons given and in the cases cited above But more generally a ldquotime place and mannerrdquo restriction involves the shifting of speech from one time and place to another or to another manner here there is no shifting but a direct punishment for expressive activity (albeit one couched in content-neutral terms) In addition some examinees might read the ordinance to be in effect a total ban on leafleting since most leafleting will produce some litter Those examinees might note that the Court has required total bans on an entire mode of expression to satisfy strict scrutiny and analyze the fatherrsquos prosecution here accordingly See United States v Grace 461 US 171 177 (1983) (invalidating ban on display of signs on public sidewalks surrounding US Supreme Court ldquo[a]dditional restrictions such as an absolute prohibition on a particular type of expression will be upheld only if narrowly drawn to accomplish a compelling governmental interestrdquo)]

23

SECURED TRANSACTIONS ANALYSIS (Secured Transactions IID E IVA B C)

ANALYSIS

Legal Problems

(1) Is a purchase-money security interest in consumer goods perfected even though there has been no filing of a financing statement

(2) Does a person who buys consumer goods for personal use take those goods free of a prior perfected purchase-money security interest in the goods

(3) Does a person who receives consumer goods as a gift take those goods subject to a prior perfected security interest in them

DISCUSSION

Summary

The retailerrsquos security interest in the bicycles was perfected even though no financing statement was filed because it was a purchase-money security interest in consumer goods A purchase-money security interest in consumer goods is automatically perfected upon attachment

The buyer is not subject to the retailerrsquos security interest in the bicycle that the buyer bought from the man Because the bicycle was consumer goods in the hands of the man and the retailer never filed a financing statement covering the bicycle the retailerrsquos security interest is not effective against someone like the buyer who bought the bicycle for value without knowledge of the retailerrsquos security interest and for personal use

On the other hand the retailerrsquos security interest continues in the bicycle given to the friend because the friend did not give value for the bicycle or buy it in the ordinary course of business

Point One (35) The retailerrsquos security interest in the bicycles attached on June 1 Because this interest was a purchase-money security interest in consumer goods it was automatically perfected when it attached

The retailerrsquos security interest in the bicycles attached on June 1 when the man bought the bicycles (acquiring rights in the collateral) signed a security agreement containing a description of the collateral and received value from the retailer (by being given credit with which to purchase the bicycles) UCC sect 9-203(a) amp (b)

Despite the retailerrsquos failure to file a financing statement its security interest was perfected Pursuant to UCC sect 9-309(1) a security interest is automatically perfected upon attachment if the goods are ldquoconsumer goodsrdquo and the security interest is a ldquopurchase-money security interestrdquo

In this case the bicycles sold by the retailer to the man were consumer goods at the time of sale The bicycles were ldquogoodsrdquo because they were ldquomovable when a security interest

24

Secured Transactions Analysis

attachesrdquo UCC sect 9-102(a)(44) They were also consumer goods because they were ldquobought for use primarily for personal family or household purposesrdquo UCC sect 9-102(a)(23) The retailerrsquos security interest in these consumer goods was also a ldquopurchase-money security interestrdquo A purchase-money security interest is an interest that secures a debt that was incurred in order to ldquoenable the debtor to acquire rights in or the use of the collateralrdquo UCC sect 9-103(a) (b)(1) Here the man incurred an obligation to the retailer to purchase the bicycles so the security interest he gave the retailer to secure that obligation was a purchase-money security interest

Because the retailerrsquos security interest was a purchase-money security interest in consumer goods it was automatically perfected on June 1 when the interest attached to the bicycles

Point Two (35) The buyer took the bicycle free of the retailerrsquos security interest because (i) the retailer did not file a financing statement covering the bicycle (ii) the bicycle was ldquoconsumer goodsrdquo and (iii) the buyer bought the bicycle for value without knowledge of the retailerrsquos security interest and for personal use

A security interest continues in collateral even after a sale or other disposition of that collateral unless the creditor authorized the disposition ldquofree of the security interestrdquo or another Article 9 exception applies UCC sectsect 9-201(a) and 9-315(a)(1)

However a buyer of goods like the buyer here can take free of a prior security interest in those goods under certain circumstances See UCC sectsect 9-317(b) (buyers who give value and receive delivery of goods without knowledge of an unperfected security interest in the goods) and 9-320(a) amp (b) (buyer in ordinary course of business buyer of consumer goods in a consumer-to-consumer transaction who gives value) In this case the retailerrsquos security interest was perfected when the buyer purchased the bicycle so UCC sect 9-317(b) does not protect the buyer The buyer also is not a protected ldquobuyer in ordinary course of businessrdquo because he did not purchase from a person who is in the business of selling bicycles See UCC sect 1-201(b)(9)

The buyer can however qualify for the protection of UCC sect 9-320(b) That section provides that a buyer of goods from a person who used them for personal family or household purposes takes free of a perfected security interest in the goods if (1) the buyer had no knowledge of the security interest (2) the buyer gave value for the goods (3) the buyer purchased the goods primarily for personal family or household purposes and (4) the purchase occurred before the filing of a financing statement covering the goods

The buyer met all of these criteria The man used the bicycle for personal purposes The buyer purchased the bicycle from the man and the buyer had no knowledge of the retailerrsquos security interest The buyer gave value ($400) for the bicycle and he bought it ldquoprimarily for personal family or household purposesrdquo as he planned to use it for recreation which is a personal rather than a business use Finally no financing statement had been filed Therefore under UCC sect 9-320(b) the buyer took free of the retailerrsquos security interest

Point Three (30) The retailerrsquos security interest continues in the bicycle that the man gave to the friend Thus the retailer can recover the bicycle from the friend because the friend did not give value for the bicycle or buy it in the ordinary course of business

25

Secured Transactions Analysis

As noted in Point Two the retailer did not authorize the man to dispose of the bicycle Consequently the retailerrsquos security interest continued in the bicycle even after the man transferred ownership of the bicycle to the friend See UCC sectsect 9-201(a) and 9-315(a)(1) The retailerrsquos security interest in the bicycle will be effective against the friend unless some other provision of Article 9 allows the friend to take the bicycle free of that security interest

Unfortunately for the friend there is no Article 9 provision that allows him to take free of the retailerrsquos interest The friendrsquos basic problem is that he is not a buyer of the bicyclemdashhe received the bicycle as a gift and did not give value for it Thus the friend is not protected by any of the applicable exceptions See UCC sectsect 9-317(b) (protecting buyers who give value for goods subject to an unperfected security interest) 9-320(a) (protecting buyers in ordinary course of business) and 9-320(b) (protecting buyers of consumer goods who give value)

In short the retailerrsquos security interest continues in the bicycle that the man gave to the friend The friend took the bicycle subject to that security interest

26

FEDERAL CIVIL PROCEDURE ANALYSIS (Federal Civil Procedure VIE)

ANALYSIS

Legal Problems

(1) Does a judgment in a prior action preclude a nonparty from suing the same defendant on a closely related claim when the nonparty and the original plaintiff are in a family relationship

(2) Does a judgment rendered in an earlier action preclude a nonparty from litigating an issue that was actually decided in the first suit

(3) May a nonparty to an earlier action invoke the judgment in that action to preclude a party to the prior action from relitigating an issue that the party had a full and fair opportunity to litigate in the earlier action

DISCUSSION

Summary

Pursuant to the doctrines of claim preclusion (res judicata) and issue preclusion (collateral estoppel) a judgment is binding on the parties thereto In the absence of privity nonparties to a prior suit cannot be bound by a judgment rendered in their absence Thus in the absence of privity a nonparty to the first suit is not precluded from presenting her claim in a second suit even if it is factually related to the claims and defenses presented in the first suit nor is she bound by determinations of issues made in the first suit A family relationship without more does not support a finding of privity For this reason Mother as a nonparty is not bound by the judgment in the Son-Driver action She may bring her separate claim for damage to her car and she is not precluded from litigating the question of whether she was negligent in the maintenance of her car

Driver on the other hand could be precluded from relitigating the issue of her negligence pursuant to the doctrine of non-mutual issue preclusion (also called non-mutual offensive collateral estoppel) which allows a nonparty to a prior action to invoke issue preclusion to prevent a party to that prior action from relitigating determinations of issues made therein However Mother may be prevented from invoking non-mutual collateral estoppel in this case because she could easily have joined her claim in the prior action but did not do so

[NOTE Federal common law governs the preclusive effect of a judgment rendered by a federal court sitting in diversity See Semtek Intrsquol Inc v Lockheed Martin Corp 531 US 497 508 (2001) But the Semtek Court concluded that federal common law in this context incorporates the preclusion law of the state in which the rendering federal court sits (unless the state law is incompatible with federal interests) id at 508ndash09 Thus State Arsquos preclusion law determines the preclusive effect of the judgment rendered in Sonrsquos suit against Driver The problem says that State A preclusion law is identical to federal preclusion law so the following analysis utilizes general principles of preclusion drawn from Supreme Court case law (announcing federal preclusion rules) and the Restatement (Second) of Judgments]

27

Federal Civil Procedure Analysis

Point One (35) Under the doctrine of claim preclusion the judgment rendered in the first action does not preclude Mother a nonparty from suing Driver for the damage to her car because the judgment binds only parties or those in privity with them and Mother and Son are not in privity

Driver may contend that the doctrine of claim preclusion (res judicata) precludes Mother from presenting a claim arising from the same nucleus of facts that was presented in the first action brought by Son According to the doctrine of claim preclusion ldquowhen a court of competent jurisdiction has entered a final judgment on the merits of a cause of action the parties to the suit and their privies are thereafter bound lsquonot only as to every matter which was offered and received to sustain or defeat the claim or demand but as to any other admissible matter which might have been offered for that purposersquordquo Commissioner of Internal Revenue v Sunnen 333 US 591 597 (1948) (citation omitted)

However the doctrine of claim preclusion does not apply to Mother on the facts of this problem First Mother was not a party to the earlier case ldquoIt is a principle of general application in Anglo-American jurisprudence that one is not bound by a judgment in personam in a litigation in which he is not designated as a party or to which he has not been made a party by service of processrdquo Taylor v Sturgell 553 US 880 884 (2008) (citing Hansberry v Lee 311 US 32 40 (1940)) see also RESTATEMENT (SECOND) OF JUDGMENTS sect 34(3) (1982) This rule reflects our ldquodeep-rooted historic tradition that everyone should have his own day in courtrdquo Martin v Wilks 490 US 755 762 (1989) (citation omitted) (superseded by statute on other grounds) Since Mother was not a party to the first suit she is not bound by the judgment unless an exception to the general rule applies

Mother might be bound by the prior judgment if she were considered to have been sufficiently in privity with Son that Son represented her interests in that action ldquoA person who is not a party to an action but who is represented by a party is bound by and entitled to the benefits of a judgment as though he were a partyrdquo RESTATEMENT (SECOND) OF JUDGMENTS sect 41(1) But there is no suggestion in the facts of the problem that Son who is an adult purported to represent Motherrsquos interests in the first suit ldquo[C]lose family relationships are not sufficient by themselves to establish privity with the original suitrsquos party or to bind a nonparty to that suit by the judgment entered therein rdquo Cuauhtli v Chase Home Finance LLC 308 Fed Appx 772 773 (5th Cir 2009) (citation omitted) accord 18A CHARLES ALAN WRIGHT ET AL FEDERAL

PRACTICE AND PROCEDURE sect 4459 (2d ed 2002) In Taylor v Sturgell supra the Supreme Court identified other special circumstances in

which nonparties may be bound by a prior judgmentmdashwhen a nonparty consents to be bound when a nonparty is in a pre-existing substantive legal relationship with a party (such as preceding and succeeding property owners) when a nonparty assumed control of the prior litigation when a party seeks to relitigate through a proxy or where a special statutory scheme seeks to foreclose successive litigation by nonparties See Taylor 553 US at 893ndash95 None of these circumstances exists here

Because Mother was not a party to the first suit and is not in privity with Son who is an adult the judgment in the first action does not preclude her from bringing her own claim against Driver

Point Two (35) Under the doctrine of issue preclusion the judgment rendered in the first action does not preclude Mother a nonparty from litigating the issue of her negligence in maintaining her carrsquos

28

Federal Civil Procedure Analysis

brake lights because the judgment binds only parties or those in privity with them and Mother and Son are not in privity

By its affirmative response to a special interrogatory the jury in the first action expressly concluded that ldquoMother negligently failed to ensure that the brake lights on her car were in proper working orderrdquo Driver may attempt to invoke the doctrine of issue preclusion to preclude Mother from relitigating this issue in the second action

[I]ssue preclusion arises in a second action on the basis of a prior decision when the same lsquoissuersquo is involved in both actions the issue was lsquoactually litigatedrsquo in the first action after a full and fair opportunity for litigation the issue was lsquoactually decidedrsquo in the first action by a disposition that is sufficiently lsquofinalrsquo lsquoon the meritsrsquo and lsquovalidrsquo it was necessary to decide the issue in disposing of the first action and the later litigation is between the same parties or involves nonparties that are subject to the binding effect or benefit of the first action Once these requirements are met issue preclusion is available not only to defend against a demand for relief but also as offensive support for a demand for relief Issue preclusion moreover is available whether or not the second action involves a new claim or cause of action

18 CHARLES ALAN WRIGHT ET AL FEDERAL PRACTICE AND PROCEDURE sect 4416 at 392ndash93 (2d ed) see also RESTATEMENT (SECOND) OF JUDGMENTS sect 27 (1982)

Here several of the elements necessary for issue preclusion are present The same issue is involved in both actionsmdashthe issue of Motherrsquos negligence in failing to maintain the brake lights on her car That issue was actually litigated in the first action and decided by the jury There is nothing to suggest anything less than a full and fair opportunity to litigate The judgment disposing of the issue was final

Nevertheless the judgment will not preclude Mother from relitigating the issue for two reasons First Mother was not a party to the first action and as explained above Mother and Son are not in privity Therefore she cannot be denied an opportunity to litigate the issue of her negligence Second it does not appear that the juryrsquos decision as to Motherrsquos negligence was necessary to the prior judgment against Driver Nothing suggests that the finding on Motherrsquos negligence had any bearing on the outcome of the first action

Point Three (30) Under the doctrine of non-mutual issue preclusion the judgment rendered in the first action might preclude Driver from relitigating the issue of her negligence However Driver has a strong argument that such a result would be inconsistent with the policy against offensive use of non-mutual estoppel when the non-party plaintiff easily could have joined as a plaintiff in the first action

Because Son already convinced the jury in the first action that ldquoDriver was negligent in the operation of her vehiclerdquo Mother may wish to invoke the doctrine of non-mutual issue preclusion to prevent Driver from relitigating the question of her negligence As noted above ldquoissue preclusion arises in a second action on the basis of a prior decision when the same lsquoissuersquo is involved in both actions the issue was lsquoactually litigatedrsquo in the first action after a full and fair opportunity for litigation the issue was lsquoactually decidedrsquo in the first action by a disposition that is sufficiently lsquofinalrsquo lsquoon the meritsrsquo and lsquovalidrsquo it was necessary to decide the issue in disposing of the first action rdquo 18 CHARLES ALAN WRIGHT ET AL FEDERAL PRACTICE AND

PROCEDURE sect 4416 at 392 (2d ed) see also RESTATEMENT (SECOND) OF JUDGMENTS sect 27

29

Federal Civil Procedure Analysis

Here these basic requirements for issue preclusion are met First the same issue is involved in both suits whether Driver was negligent in the operation of her car Second this issue was actually litigated and decided in the first action the jury answered a special interrogatory raising this very question There is nothing to suggest that Driver lacked a full and fair opportunity to litigate the issue Since a judgment was rendered against Driver for the injuries Son sustained as a result of Driverrsquos negligence resolution of the issue was necessary to dispose of the first action Driver was a party to the first action so she may be bound by the judgment

[NOTE Traditionally issue preclusion required mutualitymdashboth the party asserting issue preclusion and the party against whom issue preclusion was asserted were bound by the prior judgment Under the traditional mutuality rule Mother could not assert issue preclusion against Driver because Mother would not be bound by the judgment if Driver sought to rely on it See Point One There is no mutuality between Mother and Driver with respect to the prior judgment

This traditional mutuality requirement has been abandoned in most jurisdictions The Supreme Court rejected a strict mutuality requirement in Blonder-Tongue Laboratories Inc v University of Illinois Foundation 402 US 313 (1971) (non-mutual defensive collateral estoppel used by a defendant to preclude a plaintiff from relitigating a claim the plaintiff previously litigated) and Parklane Hosiery Co v Shore 439 US 322 (1979) (non-mutual offensive collateral estoppel used by a plaintiff to preclude a defendant from relitigating a claim the defendant previously litigated) In Parklane Hosiery the Court concluded (as a matter of federal preclusion law) that trial courts should have ldquobroad discretionrdquo to determine whether or not to permit a plaintiff to invoke non-mutual issue preclusion ldquoThe general rule should be that in cases where a plaintiff could easily have joined in the earlier action or where the application of offensive estoppel would be unfair to a defendant a trial judge should not allow the use of offensive collateral estoppelrdquo Id at 331

The Parklane Hosiery decision identified a number of circumstances that might make it unfair to allow a plaintiff to invoke non-mutual issue preclusion (non-mutual offensive collateral estoppel in the traditional terminology) against a defendant In particular the Parklane Hosiery court suggested that issue preclusion may not be appropriate if the plaintiff in the second action ldquocould easily have joined in the earlier actionrdquo Id Prohibiting plaintiffs from using non-mutual estoppel under such circumstances would promote judicial efficiency by encouraging plaintiffs to join the prior action It would also discourage plaintiffs from staying out of prior litigation in order to secure in effect two bites at the apple using the prior litigation offensively if the defendant loses and forcing the defendant to litigate a second time if the defendant wins the prior action

An exceptional exam answer might therefore argue that non-mutual issue preclusion should be denied on these facts Son and Mother both reside in State A since they are related they know each other well and Son was driving Motherrsquos car when the accident occurred They could have sued together and Rule 20 of the Federal Rules of Civil Procedure would have authorized joinder of their claims because those claims arose from the same transaction or occurrence and raised a common question of law or fact FED R CIV P 20(a) The facts do not suggest that Mother had any reason not to join Sonrsquos suit other than a desire to see how Sonrsquos action concluded before bringing her own claim Cf Nations v Sun Oil Co (Del) 695 F2d 933 938 (5th Cir 1983) (concluding that plaintiff ldquowas entitled to await the development of his injuries and their predictable consequencesrdquo) Because it appears that Mother may be a ldquowait-and-seerdquo plaintiff who could easily have joined the original action a trial court might disallow as a matter of discretion her use of non-mutual issue preclusion]

30

AGENCY ANALYSIS __________ (Agency I II)

ANALYSIS

Legal Problems

(1) Is the principal or the agent or both liable on contracts with a third party when the principal is an ldquoundisclosed principalrdquo

(2) Is the principal or the agent or both liable on contracts with a third party when the principal is ldquopartially disclosedrdquo or an ldquounidentified principalrdquo

(3) Is the principal or the agent or both liable on contracts with a third party for the purchase of goods when the agent exceeded his authority but the principal nonetheless accepts the goods

DISCUSSION

Summary

The agent but not the owner is liable to the basket manufacturer because the owner is an undisclosed principal and the agent acted without actual or apparent authority Both the agent and the owner however are liable on the burner contract because the owner is an unidentified principal and the agent had apparent authority to enter into that contract With respect to the solar cells contract whether the owner is liable depends upon whether a court would follow the Second or Third Restatement of Agency which take different positions on the effect of the ratification of a contract by an undisclosed principal Under either the agent would also be liable on the contract as he was a party to the contract

[NOTE The contracts that are the subject of this question are contracts for the sale of goods and therefore are governed by Article 2 of the Uniform Commercial Code Article 2 however does not contain agency rules Accordingly common law concepts of agency are applicable UCC sect 1-103(b)]

Point One (35) The agent but not the owner is liable to the basket manufacturer The agent had no actual authority to enter into the contract to buy aluminum baskets and because the owner was an undisclosed principal the manufacturer had no reason to believe that the agent had apparent authority Furthermore the manufacturer had no reason to believe that the agent was not contracting for his own benefit

An agent acting on behalf of a principal can bind the principal to contracts if the agent has either actual or apparent authority An agent has actual authority when contracting on behalf of his principal if he ldquoreasonably believes in accordance with the principalrsquos manifestations to the agent that the principal wishes the agent so to actrdquo RESTATEMENT (THIRD) OF AGENCY sect 201 (2006) Here the agent was told to buy only wicker baskets not aluminum baskets Thus when he contracted with the basket manufacturer to buy aluminum baskets he had no actual authority to do so

31

Agency Analysis

An agent acts with apparent authority ldquowhen a third party [with whom the agent acts] reasonably believes the actor has authority to act on behalf of the principal and that belief is traceable to the principalrsquos manifestationsrdquo Id sect 203 Here the owner notified basket manufacturers that she or her agent might contact them to purchase baskets but that notification did not specifically name the agent or any other person as the ownerrsquos agent Furthermore the basket manufacturer had no prior dealings with the agent or the owner or any reason to think that the agent was acting for the benefit of anyone but himself Thus there is no basis to conclude that the basket manufacturer thought that the agent had apparent authority to act for the owner

Generally when an agent acts on behalf of an undisclosed principal and the agent lacks authority to enter into the contract the agent is liable on the contract as a party to the contract but the principal is not liable This rule is consistent with the third partyrsquos expectations ldquoThe third party expected the agent to be a party to the contract because the agent presented the deal as if he were acting for himself Moreover if the third party is unaware of the principalrsquos existence the third party must be relying on the agentrsquos solvency and reliability when entering into the contractrdquo See ROBERT W HAMILTON JONATHAN R MACEY amp DOUGLAS K MOLL CORPORATIONS INCLUDING PARTNERSHIPS AND LIMITED LIABILITY COMPANIES 34 (11th ed 2010) See also RESTATEMENT (THIRD) OF AGENCY sect 603 cmt c Furthermore because the third party has no idea that the agent is acting or is seemingly acting on behalf of another there is no reason to believe that the third party would be expecting an undisclosed principal to be liable on the contract Id

Point Two (35) Because the owner is an unidentified (as opposed to undisclosed) principal both she and the agent (as a party to the contract) probably are liable on the contract with the burner manufacturer

When the agent contracted with the burner manufacturer he did not have actual authority to do so as the owner had expressly restricted the agentrsquos authority to purchase only burners with ldquowhisper technologyrdquo See Point One However the agent may have had apparent authority to buy burners without whisper technology

An agent acts with apparent authority ldquowhen a third party [with whom the agent acts] reasonably believes the actor has authority to act on behalf of the principal and that belief is traceable to the principalrsquos manifestationsrdquo RESTATEMENT (THIRD) OF AGENCY sect 203 (2006) The owner indicated that an agent might contact the burner manufacturer The notice contained no restriction regarding the type of burners that the agent was authorized to purchase The facts indicate that burner manufacturers regularly receive such notices

Although the agent told the burner manufacturer that he represented a well-known hot-air balloon operator he did not disclose the ownerrsquos name Thus the owner was a partially disclosed or unidentified principal See RESTATEMENT (SECOND) OF AGENCY sect 4(2) (1958) (using term ldquopartially disclosed principalrdquo) RESTATEMENT (THIRD) OF AGENCY sect 104(2)(c) (2006) (using term ldquounidentified principalrdquo) An agent for a partially disclosed principal may have apparent authority RESTATEMENT (SECOND) OF AGENCY sect 159 cmt e (1958) Based upon (1) the notice sent by the owner (2) the agentrsquos revelation that he was acting as an agent and (3) the fact that burner manufacturers regularly receive such notices and sell to agents the manufacturer may argue that it reasonably and actually believed that the agent was authorized to purchase burners without whisper technology The manufacturer may also argue that because the agent revealed that he was an agent his listing of the ownerrsquos address as the delivery address connects the agent to the notice given by the owner Arguably this distinguishes the burner contract from the basket

32

Agency Analysis

contract Here there is a strong case to support the conclusion that the agent had apparent authority if he did then the owner is liable to the burner manufacturer

The agent also is liable as a party to the contract because he did not fully disclose his agency relationship Although he told the burner manufacturer that he represented a well-known hot-air balloon operator he did not disclose the ownerrsquos name Generally even an authorized agent of a partially disclosed or unidentified principal is liable as a party to a contract with a third person RESTATEMENT (SECOND) OF AGENCY sect 321 (1958) (ldquounless otherwise agreedrdquo) RESTATEMENT (THIRD) OF AGENCY sect 602(2) (2006) (ldquounless the agent and the third party agree otherwiserdquo)

Point Three (30) Under the Second Restatement of Agency the owner is not liable on the contract for solar cells because the agent did not have actual or apparent authority and the owner as an undisclosed principal cannot ratify the contract Under the Third Restatement the owner could be liable as she ratified the contract Under either Restatement the agent is liable as a party to the contract

The owner is not liable to the solar cell manufacturer for breach of the contract for the solar cells because the agent had no actual or apparent authority to purchase solar cells on the ownerrsquos behalf and the owner under the Second Restatement of Agency did not ratify the contract with knowledge of the material facts Thus she is not liable as a ratifier of the contract

The facts state that the agent had authority to purchase only propane fuel tanks In addition he had no apparent authority to purchase solar cells The owner made no manifestations to the solar cell manufacturer that would lead a reasonable person in the manufacturerrsquos position to believe that the agent had the authority to bind the owner to a contract to purchase solar cells In fact the agent made no manifestations at all to the solar cell manufacturer Unlike with the basket manufacturer and the burner manufacturer the owner did not notify the manufacturer of solar cells that an agent might contact it to purchase solar cells In addition the solar cells were delivered to the agent and not to the ownerrsquos address In sum the manufacturer was unaware of any relationship between the owner and the agent As to the solar cell manufacturer the owner is an undisclosed principal There can be no apparent authority in the case of an undisclosed principal because there are no manifestations from the principal to the third person See RESTATEMENT (SECOND) OF AGENCY sect 8 cmt a (1958) (ldquothere can be no apparent authority created by an undisclosed principalrdquo) RESTATEMENT (THIRD) OF AGENCY sect 203 cmt f (2006) (ldquoapparent authority is not present when a third party believes that an interaction is with an actor who is a principalrdquo)

The owner also did not ratify the contract Although the owner used the solar cells generally a principal cannot ratify an unauthorized transaction with a third person ldquounless the one acting purported to be acting for the ratifierrdquo RESTATEMENT (SECOND) OF AGENCY sect 85(1) (1958)

The result differs under the Third Restatement which expressly rejects the Second Restatement on this issue The Restatement (Third) of Agency sect 403 (2006) states ldquoA person may ratify an act if the actor acted or purported to act as an agent on the personrsquos behalfrdquo According to comment b ldquoan undisclosed principal may ratify an agentrsquos unauthorized actrdquo Under the Restatement (Third) of Agency rule the owner probably ratified the transaction The agent clearly acted on the ownerrsquos behalf and in addition the ownerrsquos conduct in using the solar cells ldquojustifies a reasonable assumption that [she] is manifesting assent that the act shall affect [her] legal relationsrdquo See id sect 401(2)

33

Agency Analysis

The agent also is liable to the solar cell manufacturer for breach of the contract for the solar cells because he is a party to the contract The facts indicate that the agent never told the solar cell manufacturer that he represented the owner or any other principal Consequently even if the agent were authorized (which as discussed above he is not) he would be liable as a party to the contract See RESTATEMENT (SECOND) OF AGENCY sect 322 (1958) RESTATEMENT (THIRD) OF AGENCY sect 603(2) (2006) Here he has no authority or apparent authority and is liable as a party to the contract

The agent would also be liable under the Third Restatement Under Restatement (Third) of Agency sect 402(1) (2006) ratification generally relates back and the transaction is treated as if it were authorized at the time of the transaction However this does not relieve the agent of an undisclosed principal who ratifies an unauthorized transaction of liability under the ratified contract See id sect 603(2) (authorized agent for undisclosed principal is a party to the contract) and sect 403 cmt b (ldquoAn undisclosed principalrsquos ratification does not eliminate the agentrsquos liability to the third party on the transaction rdquo)

[NOTE An examinee may discuss the concept of inherent agency power This concept is recognized by the Restatement (Second) of Agency sect 8 A (1958) but the concept is not used in the Restatement (Third) of Agency (2006) Here there are no facts to support that the agent had inherent authority

As to contracts with agents for partially disclosed principals (eg the contract for the burners) the basic question is whether the acts done ldquousually accompany or are incidental to transactions which the agent is authorized to conductrdquo RESTATEMENT (SECOND) OF AGENCY

sect 161 (1958) If so the principal is bound if the other party ldquoreasonably believes that the agent is authorized to do them and has no notice that he is not so authorizedrdquo Id The purchase of burners without whisper technology was not authorized nor was it incidental to an authorized transaction Therefore there should not be inherent agency power

As to contracts on behalf of undisclosed principals (eg the other two contracts) the basic question is whether the acts done are usual or necessary in the transactions the agent is authorized to transact RESTATEMENT (SECOND) OF AGENCY sect 194 (1958) The other two contracts seem fundamentally different from the authorized transactions Therefore there should not be inherent agency power

Only minimal credit should be given for discussion of inherent agency power]

34

EVIDENCE ANALYSIS _____ (Evidence IIA VA B E F J K)

ANALYSIS

Legal Problems

(1) Is the authenticated copy of the mechanicrsquos text message relevant and admissible

(2) Is the womanrsquos question ldquoIs my scooter safe to drive for a whilerdquo relevant and admissible

(3) Is the womanrsquos testimony describing the mechanicrsquos thumbs-up relevant and admissible

DISCUSSION

Summary

The mechanicrsquos text message to the woman is relevant to whether (1) the woman lost control of the scooter due to its defective brakes (2) the woman knew that the brakes needed repair and (3) it was negligent for the woman to drive the scooter knowing that its brakes needed repair

The mechanicrsquos text message is hearsay if it is offered by the pedestrian to prove that the scooterrsquos brakes needed repair However it fits the hearsay exception for present sense impressions and probably also fits the exception for business records The mechanicrsquos text message is not hearsay if it is instead offered by the pedestrian to prove the womanrsquos state of mind (ie that she had notice that her brakes needed repair)

The womanrsquos question to the mechanic and his response are also relevant to whether the brakes caused the accident and whether the woman was negligent The question is not hearsay because the woman did not make an assertion

The mechanicrsquos thumbs-up response is nonverbal conduct intended by the mechanic as an assertion and is therefore an out-of-court statement If the woman offers the mechanicrsquos statement to prove that the scooter was actually safe to ride the womanrsquos testimony about the statement is hearsay

However the mechanicrsquos statement is not hearsay if it is offered by the woman to prove her state of mind Therefore the womanrsquos question and the mechanicrsquos response are admissible to prove the womanrsquos state of mind

Point One(a) (20) The mechanicrsquos text message to the woman should be admitted because it is relevant

Evidence is relevant if it has ldquoany tendency to make a fact more or less probable than it would be without the evidencerdquo FED R EVID 401 ldquoRelevant evidence is admissiblerdquo unless it is inadmissible pursuant to some other rule FED R EVID 402

The mechanicrsquos text message to the woman ldquoWhen you pick up your scooter you need to schedule a follow-up brake repair Wersquoll order the partsrdquo is relevant for two reasons First this evidence has some tendency to make it more probable that the brakes malfunctioned and

35

Evidence Analysis

caused the accident Second it has some tendency to make it more probable that the woman was negligent in riding her scooter after being told by the mechanic that it required further repair

Point One(b) (30) The mechanicrsquos text message fits either the hearsay exception for present sense impressions or the exception for business records or it is admissible non-hearsay

The mechanicrsquos text message is a statement under Rule 801(a) because it is ldquoa written assertionrdquo FED R EVID 801(a) The text message is hearsay if the pedestrian offers it to prove the ldquotruth of the matter asserted in the statementrdquo (ie that the scooterrsquos brakes required repair) which resulted in the woman losing control of the scooter and causing the accident FED R EVID 801(c)

However the mechanicrsquos text message fits the hearsay exception for ldquopresent sense impressionsrdquo under Rule 803(1) because it is ldquo[a] statement describing or explaining an event or condition made while or immediately after the declarant perceived itrdquo FED R EVID 803(1) Here the mechanicrsquos text message described the condition of the scooter immediately after he perceived it during the maintenance service

The mechanic is a person with knowledge of the condition of the scooter so if text messages regarding repairs were made and kept by the mechanic in the ordinary course of business this text message also fits the business records exception Under Rule 803(6) a business record is a record of an act ldquomade at or near the time by someone with knowledgerdquo and ldquothe record was kept in the course of a regularly conducted activity of a businessrdquo and ldquomaking the record was a regular practice of that activityrdquo FED R EVID 803(6)

However the text message is not hearsay if it is instead offered to prove that the woman was negligent because she rode her scooter after the mechanic told her it required repair If offered for this purpose it would not be offered for the truth of the matter asserted in the statement but to show the womanrsquos belief about the condition of the scooter (her state of mind)

Point Two (10) The womanrsquos question to the mechanic should be admitted because it is not hearsay

The womanrsquos question to the mechanic is relevant because along with the mechanicrsquos thumbs-up response (see Point Three) it has some tendency to make it more probable that the woman was not negligent andor that the scooter brakes did not malfunction and cause the accident FED R EVID 401 The womanrsquos question does not raise hearsay concerns because it is not an assertion

Hearsay is defined under Rule 801(a) as ldquoan oral assertion written assertion or nonverbal conductrdquo Although ldquoassertionrdquo is not further defined ldquoa favorite [definition] of writers in the [evidence] field for at least a century and a half [is that] the word simply means to say that something is so eg that an event happened or a condition existedrdquo 2 MCCORMICK ON

EVIDENCE sect 246 (6th ed 2006) Under this definition the womanrsquos question is not hearsay because it is not an assertion

Point Three(a) (20) The mechanicrsquos thumbs-up to the woman is a nonverbal assertion that is relevant and the womanrsquos testimony about that response is admissible

36

Evidence Analysis

Hearsay is defined under Rule 801(c) as a ldquostatementrdquo that is ldquoa personrsquos oral assertion written assertion or nonverbal conduct if the person intended it as an assertionrdquo FED R EVID 801(a) Here when the mechanic responded to the womanrsquos question (ldquoIs my scooter safe to ride for a whilerdquo) with a thumbs-up gesture the facts suggest that he intended his nonverbal conduct as an assertion that in his opinion the scooter was safe to ride

The mechanicrsquos assertion is relevant and admissible to prove that the woman was not negligent because the evidence makes it more probable that at the time of the accident she believed that the scooter was safe to ride despite the fact that the brakes required repair FED R EVID 401 Admission of the womanrsquos description of the mechanicrsquos thumbs-up for this purpose does not raise hearsay concerns because the evidence would not be offered for the truth of the matter asserted but to show the womanrsquos belief about the condition of the scooter (her state of mind)

Point Three(b) (20) The mechanicrsquos thumbs-up is relevant to determine whether the scooterrsquos brakes malfunctioned causing the accident but if offered for this purpose it is also hearsay

The mechanicrsquos nonverbal assertion is relevant to the determination of whether the scooterrsquos brakes malfunctioned causing the accident However if offered to prove the ldquotruth of the matter asserted in the statementrdquo (ie that the scooter was safe to ride for a while) it is hearsay that does not fit any hearsay exception

37

TRUSTS AND FUTURE INTERESTS ANALYSIS ____________________ (Trusts and Future Interests IC1 amp 4 G IIF)

ANALYSIS

Legal Problems

(1)(a) Was the revocable trust amendable

(1)(b) If the trust was amendable must the amendment have been executed in accordance with the state Statute of Wills in order to be valid

(2) If the trust amendment was valid does the amendment apply to the probate estate assets passing to the trust pursuant to Settlorrsquos will

(3) If the trust amendment was valid should the trust property be distributed to University

(4) If the trust amendment was not valid should the trust property be distributed to Settlorrsquos grandchild (her only heir) or held in further trust in accordance with the terms of the original trust instrument

DISCUSSION

Summary

A revocable trust is amendable even if the trust instrument does not expressly grant to the trust settlor a power to amend Both inter vivos trusts and amendments thereto are valid even though not executed in accordance with the requirements applicable to wills

Under the Uniform Testamentary Additions to Trusts Act a revocable trust may be amended at any time prior to the settlorrsquos death and the amendment applies to the disposition of assets conveyed to the trust pursuant to a will even if the will was executed prior to the date of the amendment

At Settlorrsquos death trust assets including probate assets passing to the trust under Settlorrsquos will would go to University if as is the case here the trust amendment was valid If the amendment was invalid the trust assets would continue to be held in further trust because there is no violation of the common law Rule Against Perpetuities

Point One(a) (30) Settlor retained the right to amend the inter vivos trust despite her failure to expressly reserve this power

At issue here is whether a retained power of revocation includes the power to amend sometimes referred to as the power to modify The Restatement (Second) of Trusts sect 331 cmt g provides that if a settlor has a power to revoke that retained power ordinarily includes a power to modify (amend) as well Comment g also notes that the power to amend includes both a power to withdraw trust assets and a power to ldquomodify the terms of the trustrdquo The Uniform Trust Code which provides that a power to revoke includes the power to amend is consistent with this view

38

Trusts and Future Interests Analysis

UNIF TRUST CODE sect 602 accord RESTATEMENT (THIRD) OF TRUSTS sect 63 cmt The theory is that even though a power to amend was not expressly retained by a settlor the goal of amendment assuming the power was not included in the power to revoke could easily be achieved by first revoking the trust and then creating a new trust with the same terms contemplated by the amendment To require this would put form over substance

Thus by expressly retaining the power to revoke the trust Settlor retained a power to amend the inter vivos trust despite her failure to expressly reserve this power

[NOTE Under the common law a trust is irrevocable unless the settlor expressly retains a power to revoke the trust Conversely under the Uniform Trust Code a trust is revocable unless the terms of the trust expressly provide otherwise See UNIF TRUST CODE sect 602 The Trust Codersquos position on revocation follows the minority view in the United States and is inconsistent with prior Restatements of Trusts (see Restatement (Second) of Trusts sect 330) Here the trust is revocable because Settlor expressly retained a power of revocation

The Uniform Trust Code has been adopted in 24 jurisdictions Alabama Arizona Arkansas District of Columbia Florida Kansas Maine Michigan Missouri Nebraska New Hampshire New Mexico North Carolina North Dakota Ohio Oregon Pennsylvania South Carolina Tennessee Utah Vermont Virginia West Virginia and Wyoming]

Point One(b) (10) Settlorrsquos amendment of the trust was valid despite her failure to have her signature to the trust amendment witnessed

Neither the common law nor state statutes require a trust instrument or an amendment to a trust instrument to be executed in accordance with the formalities prescribed for execution of a will Indeed an inter vivos trust that does not involve real estate can be created orally Under the Uniform Trust Code the only requirements for creating a valid inter vivos trust are intent the specification of beneficiaries and the designation of a trustee See UNIF TRUST CODE sect 402 accord RESTATEMENT (THIRD) OF TRUSTS sect 13

Here the amendment meets the requirements of both the Uniform Trust Code and the common law Thus the fact that Settlorrsquos signature was not witnessed when she signed the amendment to the trust does not make the amendment invalid

Point Two (20) Under the Uniform Testamentary Additions to Trusts Act a revocable trust may be amended at any time prior to the settlorrsquos death and the amendment applies to probate assets poured into the trust at the settlorrsquos death pursuant to the settlorrsquos will even when the will was executed prior to the date of the amendment

Historically property owned by an individual at her death passed to the individualrsquos heirs or to beneficiaries designated in a will executed with the formalities (writing signing witnessing) prescribed by state law However when a will devises property to the trustee of an inter vivos trust then the provisions of the trustmdashwhich may not have been executed in accordance with the formalities required for willsmdasheffectively determine who will receive the property Because of this possibility some early cases held that if an inter vivos trust was not executed with the same formalities required for a valid will then the trust was ineffective to dispose of probate assets poured into the trust at the settlorrsquos death pursuant to the settlorrsquos will

This line of cases has been overturned by the Uniform Testamentary Additions to Trusts Act (the Act) now Uniform Probate Code sect 2-511 Under the Act adopted in almost all

39

Trusts and Future Interest Analysis

jurisdictions a testamentary bequest to the trustee of an inter vivos trust established by the testator during his or her lifetime is valid if the trust is in writing it is identified in the testatorrsquos will and the trust instrument was executed before concurrently with or after the execution of the will Id The Act further specifies that such a bequest is valid even if the trust is amendable or revocable and that a later amendment applies to assets passing to the trust by a previously executed will

Thus because the trust amendment is valid its terms apply to assets received by Bank from Settlorrsquos estate

Point Three (10) If the trust amendment was valid then the trust assets including assets passing to the trust under Settlorrsquos will should go to University

Under the trust amendment all trust assets (including the assets of Settlorrsquos probate estate poured into the trust) pass to University The facts provide no basis for failing to comply with Settlorrsquos stated intentions

Point Four (30) If the trust amendment was invalid trust assets including assets received pursuant to Settlorrsquos will should be held in accordance with the terms of the original trust instrument because those terms do not violate the Rule Against Perpetuities

Under the dispositive terms of the original trust instrument Settlor created successive income interests in her surviving children and grandchildren with a remainder interest in her great-grandchildren Because the trust was revocable the period during which the common law Rule Against Perpetuities requires that interests vest (ie 21 years plus lives in being) began to run from the date Settlor no longer had a power of revocation (here her death) not the date on which the trust was created See JESSE DUKEMINIER STANLEY J JOHANSON JAMES LINDGREN amp ROBERT SITKOFF WILLS TRUSTS AND ESTATES 678 (7th ed 2005)

Under the common law Rule Against Perpetuities Settlorrsquos trust is thus valid At the time of Settlorrsquos death she was survived by no children one granddaughter and no great-grandchildren Because Settlor cannot have more children after her death the only income beneficiary of the trust is Settlorrsquos surviving granddaughter This granddaughter is the only person who can produce great-grandchildren of Settlor thus all great-grandchildren must of necessity be born during the lifetime of Settlorrsquos only surviving granddaughter who is a life in being The granddaughterrsquos interest vested at Settlorrsquos death and the great-grandchildrenrsquos interest will vest at the death of the granddaughter There is no need to wait the additional 21 years permitted under the Rule Thus under the common law and the statute given in the facts the nonvested interest in the great-grandchildren is valid

[NOTE Both modern wait-and-see statutes and the Uniform Statutory Rule Against Perpetuities upon which the statute in the facts is modeled provide that before using either reform to validate an otherwise invalid nonvested interest one should first determine if the nonvested interest violates the common law Rule If it does not then there is no need to reform This proposition which is applicable in all MEE user jurisdictions that have not simply abrogated the rule is tested by this problem]

40

NEGOTIABLE INSTRUMENTS ANALYSIS (Negotiable Instruments III IV V)

ANALYSIS

Legal Problems

(1)(a) What rights does a person in possession of a note that has been indorsed in blank by the payee have against the maker of the note

(1)(b) Which defenses may the maker of a note raise against a person entitled to enforce it who is not a holder in due course but is a transferee from a holder in due course

(2) What rights does a person entitled to enforce a note have against an indorser who transferred it for consideration with no warranties

(3) What rights does a person entitled to enforce a note have against a previous holder who transferred it as a gift without indorsing it

DISCUSSION

Summary

The niece is a holder of the note and is thus a person entitled to enforce it The chef the issuer of the note is obligated to pay it to the niece as the person entitled to enforce it The niece is not subject to any defense or claim of the chef relating to the improper repair of the oven because the niece has the rights of a holder in due course When the buyer bought the note from the repairman the buyer became a holder in due course of the note and thus took it free of any personal defenses the chef had against the repairman Even though the niece is not herself a holder in due course of the note the niece succeeded to the buyerrsquos rights as holder in due course and thus took free of the chefrsquos personal defenses

Because the chef refused to pay the note the niece can recover from the repairman on the repairmanrsquos obligation as indorser The niece cannot recover on the note against the buyer however because the buyer did not indorse the note (and thus incurred no indorserrsquos obligation) and the buyer did not receive any consideration for transfer of the note to the niece (and therefore made no transfer warranty)

[NOTE Although Article 9 of the Uniform Commercial Code governs the sale of promissory notes (a point that might be correctly noted by examinees) that Article does not determine the answer to any of the questions posed]

Point One(a) (20) The niece is the holder of the note and thus may enforce it against the chef who is the issuer of the note

The chef is the maker of the note and thus its issuer See UCC sectsect 3-103 3-105 The issuer of a note is obligated to pay it in accordance with its terms to a ldquoperson entitled to enforcerdquo it UCC sect 3-412 The niece is a ldquoperson entitled to enforcerdquo the note This is because the niece is the holder of the note and a holder of a note is a person entitled to enforce it UCC sect 3-301 The niece is the holder of the note because (i) the repairmanrsquos signature on the back of the note not

41

Negotiable Instruments Analysis

accompanied by words indicating a person to whom the note was made payable was a ldquoblank indorsementrdquo which had the effect of making the note a bearer instrument (ii) anyone in possession of a bearer instrument is a holder of it and (iii) the niece is in possession of the note See UCC sectsect 1-201(b)(21)(A) 3-204 and 3-205 Accordingly the chef has an obligation to the niece to pay the note in accordance with its terms and the niece may enforce that obligation

Point One(b) (40) The niece is not a holder in due course of the note but because she is a transferee from the buyer who was a holder in due course she has the same enforcement rights as the buyer Because the buyer as a holder in due course would have been able to enforce the note against the chef without being subject to defenses or claims arising from the improper repair the niece has the same rights and will not be subject to the chefrsquos defenses or claims about the repair

As noted in Point One(a) the chef has an obligation to the niece to pay the note in accordance with its terms However except against a person with the rights of a holder in due course the chef can raise any defenses or claims in recoupment that he would have if the claim on the note were an ordinary contract claim UCC sect 3-305 Thus except against a holder in due course the chef would be able to raise the improper repair as a defense or a claim in recoupment (a claim in response to the niecersquos claim)

But claims in recoupment and most defenses cannot be raised against a person with the rights of a holder in due course Against a holder in due course the chef can raise only the four ldquorealrdquo defenses listed in UCC sect 3-305(a)(1) (infancy duress lack of legal capacity or illegality that nullifies the obligation of the obligor under other law fraud in the factum discharge in insolvency proceedings) none of which is present here

The niece is not a holder in due course because she did not take the note for value See UCC sectsect 3-302(a)(2)(i) (criteria for holder in due course status) and 3-303(a) (definition of ldquovaluerdquo) But this does not mean that the niece is subject to the chefrsquos claim arising out of the improper repair The buyer was a holder in due course of the note because he took the note for value ($9500) in good faith and without notice of any facts that would have alerted him to the chefrsquos defense against the repairman UCC sect 3-302(a)(2) As a holder in due course the buyer owned the note free of the chefrsquos claim because that claim did not constitute a ldquorealrdquo defense UCC sect 3-305(b) When the buyer gave the note to the niece this constituted a ldquotransferrdquo of the note See UCC sect 3-203(a) When a note is transferred the transferee receives ldquoany right of the transferor to enforce the instrument including any right as a holder in due courserdquo UCC sect 3-203(b) Under this rule (also known as the ldquoshelter principlerdquo) the buyer transferred his freedom from the chefrsquos defenses to the niece and the niece can enforce the note free of the chefrsquos defenses

Point Two (20) Because the chef dishonored the note the niece can recover from the repairman on the repairmanrsquos obligation as indorser

The chefrsquos refusal to pay the note constituted dishonor See UCC sect 3-502 The repairman as an indorser of the note (see Point One(a)) incurred the obligations of an indorser under UCC sect 3-415(a) When a note has been dishonored one of the obligations of an indorser is to pay the amount of the note to a person entitled to enforce it Therefore the repairman is liable for the amount of the note to the niece a person entitled to enforce the note (so long as the niece gives proper notice of dishonor to the repairman)

42

Negotiable Instruments Analysis

[NOTE Because the repairman indorsed the note without warranties there are no transfer warranties UCC sect 3-416 cmt 5]

Point Three (20) The niece cannot recover on the note against the buyer as either indorser or warrantor because the buyer did not indorse the note and did not receive consideration for transferring the note to the niece

The buyer did not indorse the note and therefore did not incur the obligation of an indorser to pay the note upon dishonor

The niece cannot recover from the buyer under a transfer warranty theory because transfer warranties are made only by a person ldquowho transfers an instrument for considerationrdquo Here the buyer gave the instrument to the niece as a gift So the buyer made no transfer warranty UCC sect 3-416(a) Therefore the niece cannot recover from the buyer on that theory

43

National Conference of Bar Examiners 302 South Bedford Street | Madison WI 53703-3622 Phone 608-280-8550 | Fax 608-280-8552 | TDD 608-661-1275

wwwncbexorg e-mail contactncbexorg

  • Contents
  • Preface
  • Description of the MEE
  • Instructions
  • February 2013 Questions
    • Real Property Question
    • Contracts Question
    • Constitutional Law Question
    • Secured Transactions Question
    • Federal Civil Procedure Question
    • Agency Question
    • Evidence Question
    • Trusts and Future Interests Question
    • Negotiable Instruments Question
      • February 2013 Analyses
        • Real Property Analysis
        • Contracts Analysis
        • Constitutional Law Analysis
        • Secured Transactions Analysis
        • Federal Civil Procedure Analysis
        • Agency Analysis
        • Evidence Analysis
        • Trusts and Future Interests Analysis
        • Negotiable Instruments Analysis
            • ltlt ASCII85EncodePages false AllowTransparency false AutoPositionEPSFiles true AutoRotatePages None Binding Left CalGrayProfile (Dot Gain 20) CalRGBProfile (sRGB IEC61966-21) CalCMYKProfile (US Web Coated 050SWOP051 v2) sRGBProfile (sRGB IEC61966-21) CannotEmbedFontPolicy Error CompatibilityLevel 14 CompressObjects Tags CompressPages true ConvertImagesToIndexed true PassThroughJPEGImages true CreateJobTicket false DefaultRenderingIntent Default DetectBlends true DetectCurves 00000 ColorConversionStrategy CMYK DoThumbnails false EmbedAllFonts true EmbedOpenType false ParseICCProfilesInComments true EmbedJobOptions true DSCReportingLevel 0 EmitDSCWarnings false EndPage -1 ImageMemory 1048576 LockDistillerParams false MaxSubsetPct 100 Optimize true OPM 1 ParseDSCComments true ParseDSCCommentsForDocInfo true PreserveCopyPage true PreserveDICMYKValues true PreserveEPSInfo true PreserveFlatness true PreserveHalftoneInfo false PreserveOPIComments true PreserveOverprintSettings true StartPage 1 SubsetFonts true TransferFunctionInfo Apply UCRandBGInfo Preserve UsePrologue false ColorSettingsFile () AlwaysEmbed [ true ] NeverEmbed [ true ] AntiAliasColorImages false CropColorImages true ColorImageMinResolution 300 ColorImageMinResolutionPolicy OK DownsampleColorImages true ColorImageDownsampleType Bicubic ColorImageResolution 300 ColorImageDepth -1 ColorImageMinDownsampleDepth 1 ColorImageDownsampleThreshold 150000 EncodeColorImages true ColorImageFilter DCTEncode AutoFilterColorImages true ColorImageAutoFilterStrategy JPEG ColorACSImageDict ltlt QFactor 015 HSamples [1 1 1 1] VSamples [1 1 1 1] gtgt ColorImageDict ltlt QFactor 015 HSamples [1 1 1 1] VSamples [1 1 1 1] gtgt JPEG2000ColorACSImageDict ltlt TileWidth 256 TileHeight 256 Quality 30 gtgt JPEG2000ColorImageDict ltlt TileWidth 256 TileHeight 256 Quality 30 gtgt AntiAliasGrayImages false CropGrayImages true GrayImageMinResolution 300 GrayImageMinResolutionPolicy OK DownsampleGrayImages true GrayImageDownsampleType Bicubic GrayImageResolution 300 GrayImageDepth -1 GrayImageMinDownsampleDepth 2 GrayImageDownsampleThreshold 150000 EncodeGrayImages true GrayImageFilter DCTEncode AutoFilterGrayImages true GrayImageAutoFilterStrategy JPEG GrayACSImageDict ltlt QFactor 015 HSamples [1 1 1 1] VSamples [1 1 1 1] gtgt GrayImageDict ltlt QFactor 015 HSamples [1 1 1 1] VSamples [1 1 1 1] gtgt JPEG2000GrayACSImageDict ltlt TileWidth 256 TileHeight 256 Quality 30 gtgt JPEG2000GrayImageDict ltlt TileWidth 256 TileHeight 256 Quality 30 gtgt AntiAliasMonoImages false CropMonoImages true MonoImageMinResolution 1200 MonoImageMinResolutionPolicy OK DownsampleMonoImages true MonoImageDownsampleType Bicubic MonoImageResolution 1200 MonoImageDepth -1 MonoImageDownsampleThreshold 150000 EncodeMonoImages true MonoImageFilter CCITTFaxEncode MonoImageDict ltlt K -1 gtgt AllowPSXObjects false CheckCompliance [ None ] PDFX1aCheck false PDFX3Check false PDFXCompliantPDFOnly false PDFXNoTrimBoxError true PDFXTrimBoxToMediaBoxOffset [ 000000 000000 000000 000000 ] PDFXSetBleedBoxToMediaBox true PDFXBleedBoxToTrimBoxOffset [ 000000 000000 000000 000000 ] PDFXOutputIntentProfile () PDFXOutputConditionIdentifier () PDFXOutputCondition () PDFXRegistryName () PDFXTrapped False CreateJDFFile false Description ltlt ARA ltFEFF06270633062A062E062F0645002006470630064700200627064406250639062F0627062F0627062A002006440625064606340627062100200648062B062706260642002000410064006F00620065002000500044004600200645062A064806270641064206290020064406440637062806270639062900200641064A00200627064406450637062706280639002006300627062A0020062F0631062C0627062A002006270644062C0648062F0629002006270644063906270644064A0629061B0020064A06450643064600200641062A062D00200648062B0627062606420020005000440046002006270644064506460634062306290020062806270633062A062E062F062706450020004100630072006F0062006100740020064800410064006F006200650020005200650061006400650072002006250635062F0627063100200035002E0030002006480627064406250635062F062706310627062A0020062706440623062D062F062B002E0635062F0627063100200035002E0030002006480627064406250635062F062706310627062A0020062706440623062D062F062B002Egt BGR 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 CHS ltFEFF4f7f75288fd94e9b8bbe5b9a521b5efa7684002000410064006f006200650020005000440046002065876863900275284e8e9ad88d2891cf76845370524d53705237300260a853ef4ee54f7f75280020004100630072006f0062006100740020548c002000410064006f00620065002000520065006100640065007200200035002e003000204ee553ca66f49ad87248672c676562535f00521b5efa768400200050004400460020658768633002gt CHT ltFEFF4f7f752890194e9b8a2d7f6e5efa7acb7684002000410064006f006200650020005000440046002065874ef69069752865bc9ad854c18cea76845370524d5370523786557406300260a853ef4ee54f7f75280020004100630072006f0062006100740020548c002000410064006f00620065002000520065006100640065007200200035002e003000204ee553ca66f49ad87248672c4f86958b555f5df25efa7acb76840020005000440046002065874ef63002gt CZE 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 DAN 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 DEU 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 ESP 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 ETI 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 FRA 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 GRE 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 HEB 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 HRV (Za stvaranje Adobe PDF dokumenata najpogodnijih za visokokvalitetni ispis prije tiskanja koristite ove postavke Stvoreni PDF dokumenti mogu se otvoriti Acrobat i Adobe Reader 50 i kasnijim verzijama) HUN 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 ITA 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 JPN ltFEFF9ad854c18cea306a30d730ea30d730ec30b951fa529b7528002000410064006f0062006500200050004400460020658766f8306e4f5c6210306b4f7f75283057307e305930023053306e8a2d5b9a30674f5c62103055308c305f0020005000440046002030d530a130a430eb306f3001004100630072006f0062006100740020304a30883073002000410064006f00620065002000520065006100640065007200200035002e003000204ee5964d3067958b304f30533068304c3067304d307e305930023053306e8a2d5b9a306b306f30d530a930f330c8306e57cb30818fbc307f304c5fc59808306730593002gt KOR ltFEFFc7740020c124c815c7440020c0acc6a9d558c5ec0020ace0d488c9c80020c2dcd5d80020c778c1c4c5d00020ac00c7a50020c801d569d55c002000410064006f0062006500200050004400460020bb38c11cb97c0020c791c131d569b2c8b2e4002e0020c774b807ac8c0020c791c131b41c00200050004400460020bb38c11cb2940020004100630072006f0062006100740020bc0f002000410064006f00620065002000520065006100640065007200200035002e00300020c774c0c1c5d0c11c0020c5f40020c2180020c788c2b5b2c8b2e4002egt LTH 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 LVI 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 NLD (Gebruik deze instellingen om Adobe PDF-documenten te maken die zijn geoptimaliseerd voor prepress-afdrukken van hoge kwaliteit De gemaakte PDF-documenten kunnen worden geopend met Acrobat en Adobe Reader 50 en hoger) NOR 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 POL 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 PTB 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 RUM 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 RUS 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 SKY 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 SLV 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 SUO 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 SVE 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 TUR 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 UKR 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 ENU (Use these settings to create Adobe PDF documents best suited for high-quality prepress printing Created PDF documents can be opened with Acrobat and Adobe Reader 50 and later) gtgt Namespace [ (Adobe) (Common) (10) ] OtherNamespaces [ ltlt AsReaderSpreads false CropImagesToFrames true ErrorControl WarnAndContinue FlattenerIgnoreSpreadOverrides false IncludeGuidesGrids false IncludeNonPrinting false IncludeSlug false Namespace [ (Adobe) (InDesign) (40) ] OmitPlacedBitmaps false OmitPlacedEPS false OmitPlacedPDF false SimulateOverprint Legacy gtgt ltlt AddBleedMarks false AddColorBars false AddCropMarks false AddPageInfo false AddRegMarks false ConvertColors ConvertToCMYK DestinationProfileName () DestinationProfileSelector DocumentCMYK Downsample16BitImages true FlattenerPreset ltlt PresetSelector MediumResolution gtgt FormElements false GenerateStructure false IncludeBookmarks false IncludeHyperlinks false IncludeInteractive false IncludeLayers false IncludeProfiles false MultimediaHandling UseObjectSettings Namespace [ (Adobe) (CreativeSuite) (20) ] PDFXOutputIntentProfileSelector DocumentCMYK PreserveEditing true UntaggedCMYKHandling LeaveUntagged UntaggedRGBHandling UseDocumentProfile UseDocumentBleed false gtgt ]gtgt setdistillerparamsltlt HWResolution [2400 2400] PageSize [612000 792000]gtgt setpagedevice

Page 12: February 2013 MEE Questions and Analyses

_____

AGENCY QUESTION

Over 5000 individuals in the United States operate hot-air balloon businesses A hot-air balloon has four key components the balloon that holds the heated air the basket that houses the riders the propane burner that heats the air in the balloon and the propane storage tanks

The owner of a hot-air balloon business recently notified several basket and burner manufacturers that she or her agent might be contacting them to purchase baskets or burners The owner did not specifically name any person as her agent Basket and burner manufacturers regularly receive such notices from hot-air balloon operators Such notices typically include no restrictions on the types of baskets or burners agents might purchase for their principals

The owner then retained an agent to acquire baskets burners and fuel tanks from various manufacturers The owner authorized the agent to buy only (a) baskets made of woven wicker (not aluminum) (b) burners that use a unique ldquowhisper technologyrdquo (so as not to scare livestock when the balloon sails over farmland) and (c) propane fuel tanks

The agent then entered into three transactions with manufacturers all of whom had no prior dealings with either the owner or the agent

(1) The agent and a large manufacturer of both wicker and aluminum baskets signed a contract for the purchase of four aluminum baskets for a total cost of $60000 The agent never told the manufacturer that he represented the owner or any other principal The contract listed the agent as the buyer and listed the ownerrsquos address as the delivery address but did not indicate that the address was that of the owner rather than the agent When the baskets were delivered to the owner she learned for the first time that the agent had contracted to buy aluminum not wicker baskets The owner immediately rejected the baskets and returned them to the manufacturer Neither the owner nor the agent has paid the basket manufacturer for them

(2) The agent contacted a burner manufacturer and told him that the agent represented a well-known hot-air balloon operator who wanted to purchase burners The agent did not disclose the ownerrsquos name The agent and the burner manufacturer signed a contract for the purchase of four burners that did not have ldquowhisper technologyrdquo for a total price of $70000 The burner contract like the basket contract listed the ownerrsquos address for delivery but did not disclose whose address it was The burners were delivered to the ownerrsquos business and the owner discovered that the agent had ordered the wrong kind of burners The owner rejected the burners and returned them to the manufacturer Neither the owner nor the agent has paid the burner manufacturer for the burners

(3) The agent contracted with a solar cell manufacturer to make three cells advertised as ldquostrong enough to power all your ballooning needsrdquo The agent did not tell the manufacturer that he was acting on behalf of any other person One week after the cells were delivered to the agent he took them to the owner who installed them and discovered that she could save a lot of money using solar cells instead of propane to power her balloons The owner decided to keep the solar cells but she has not paid the manufacturer for them

8

Agency Question

Assume that the rejection of the baskets and the burners and the failure to pay for the solar cells constitute breach of the relevant contracts

1 Is the owner liable to the basket manufacturer for breach of the contract for the aluminum baskets Is the agent liable Explain

2 Is the owner liable to the burner manufacturer for breach of the contract for the burners Is the agent liable Explain

3 Is the owner liable to the solar cell manufacturer for breach of the contract for the solar cells Is the agent liable Explain (Do not address liability based upon restitution or unjust enrichment)

9

EVIDENCE QUESTION _____

A woman who owns a motorized scooter brought her scooter to a mechanic for routine maintenance service As part of the maintenance service the mechanic inspected the braking system on the scooter As soon as the mechanic finished inspecting and servicing the scooter he sent the woman a text message to her cell phone that read ldquoJust finished your service When you pick up your scooter you need to schedule a follow-up brake repair Wersquoll order the partsrdquo

The woman read the mechanicrsquos text message and returned the next day to pick up her scooter As the woman was wheeling her scooter out of the shop she saw the mechanic working nearby and asked ldquoIs my scooter safe to ride for a whilerdquo The mechanic responded by giving her a thumbs-up The woman waved and rode away on the scooter

One week later while the woman was riding her scooter a pedestrian stepped off the curb into a crosswalk and the woman collided with him causing the pedestrian severe injuries The woman had not had the scooterrsquos brakes repaired before the accident

The pedestrian has sued the woman for damages for his injuries resulting from the accident The pedestrian has alleged that (1) the woman lost control of the scooter due to its defective brakes (2) the woman knew that the brakes needed repair and (3) it was negligent for the woman to ride the scooter knowing that its brakes needed to be repaired

The woman claims that the brakes on the scooter worked perfectly and that the accident happened because the pedestrian stepped into the crosswalk without looking and the woman had no time to stop The woman the pedestrian and the mechanic will testify at the upcoming trial

The pedestrian has proffered an authenticated copy of the mechanicrsquos text message to the woman

The woman plans to testify that she asked the mechanic ldquoIs my scooter safe to ride for a whilerdquo and that he gave her a thumbs-up in response

The evidence rules in this jurisdiction are identical to the Federal Rules of Evidence

Analyze whether each of these items of evidence is relevant and admissible at trial

1 The authenticated copy of the mechanicrsquos text message

2 The womanrsquos testimony that she asked the mechanic ldquoIs my scooter safe to ride for a whilerdquo and

3 The womanrsquos testimony describing the mechanicrsquos thumbs-up

10

TRUSTS AND FUTURE INTERESTS QUESTION

Ten years ago Settlor validly created an inter vivos trust and named Bank as trustee The trust instrument provided that Settlor would receive all of the trust income during her lifetime The trust instrument further provided that

Upon Settlorrsquos death the trust income shall be paid in equal shares to Settlorrsquos surviving children for their lives Upon the death of the last surviving child the trust income shall be paid in equal shares to Settlorrsquos then-living grandchildren for their lives Upon the death of the survivor of Settlorrsquos children and grandchildren the trust corpus shall be distributed in equal shares to Settlorrsquos then-living great-grandchildren

The trust instrument expressly specified that the trust was revocable but it was silent regarding whether Settlor could amend the trust instrument

Immediately after creating the trust Settlor validly executed a will leaving her entire estate to Bank as trustee of her inter vivos trust to ldquohold in accordance with the terms of the trustrdquo

Five years ago Settlor signed an amendment to the inter vivos trust The amendment changed the disposition of the remainder interest specifying that all trust assets ldquoshall be paid upon Settlorrsquos death to Universityrdquo Settlorrsquos signature on this amendment was not witnessed

A state statute provides that any trust interest that violates the common law Rule Against Perpetuities ldquois nonetheless valid if the nonvested interest in the trust actually vests or fails to vest either (a) within 21 years of lives in being at the creation of the nonvested interest or (b) within 90 years of its creationrdquo

Recently Settlor died leaving a probate estate of $200000 She was survived by no children one granddaughter (who would be Settlorrsquos only heir) and no great-grandchildren The granddaughter has consulted your law firm and has raised four questions regarding this trust

1 Was Settlorrsquos amendment of the inter vivos trust valid Explain

2 Assuming that the trust amendment was valid do its provisions apply to Settlorrsquos probate assets Explain

3 Assuming that the trust amendment was valid how should trust assets be distributed Explain

4 Assuming that the trust amendment was invalid how should trust assets be distributed Explain

11

NEGOTIABLE INSTRUMENTS QUESTION

A chef entered into a contract with a repairman pursuant to which the repairman agreed to repair the chefrsquos commercial oven for $10000 The repairman agreed to accept as payment a negotiable promissory note for $10000 payable two months after its issuance

After the repairman worked on the oven the chef gave him a $10000 note as payment for the work As agreed the note was signed by the chef as maker was payable to the order of the repairman was payable in two months and fulfilled all criteria for negotiability

The next day the repairman sold the note to a buyer for $9500 To effectuate the sale the repairman wrote ldquono warrantiesrdquo on the back of the note signed his name immediately below that and handed the note to the buyer The buyer bought the note in good faith and without knowledge of any facts relating to the work that the repairman had performed for the chef

Later the buyer gave the note to his niece as a gift To effectuate the gift the buyer handed the note to the niece but did not indorse it

Shortly thereafter the chef discovered that the repair work had been done improperly and the oven still did not function correctly The chef tried repeatedly to get the repairman to return to correct the repair work but the repairman ignored all the chefrsquos calls

On the notersquos due date the niece contacted the chef and demanded that he pay the amount of the note to her The chef refused and told the niece that he would not pay the note because the repairman did not properly repair the oven

1 What are the niecersquos rights against the chef Explain

2 What are the niecersquos rights against the repairman Explain

3 What are the niecersquos rights against the buyer Explain

12

February 2013 MEE

ANALYSES

Real Property Contracts

Constitutional Law Secured Transactions

Federal Civil Procedure Agency

Evidence Trusts and Future Interests

Negotiable Instruments

REAL PROPERTY ANALYSIS (Real Property ID1a 4 amp 5)

ANALYSIS

Legal Problems

(1) Does the tenant have a defense to the landlordrsquos action for unpaid rent based on constructive eviction

(2) Does the tenant have a defense to the landlordrsquos action for unpaid rent based on the tenantrsquos surrender of the premises

(3) What if anything may the landlord recover from the tenant for the period after the tenant vacated the building

DISCUSSION

Summary

Under the common law the tenant does not have a defense to the landlordrsquos action for unpaid rent based on constructive eviction Constructive eviction is based on the tenant proving that (1) the landlord breached a duty to the tenant (2) the breach caused a loss by the tenant of the substantial use and enjoyment of the premises (3) the tenant gave the landlord adequate notice and opportunity to repair and (4) the tenant vacated the leased premises Here there was no constructive eviction because although the tenant vacated and gave the landlord adequate notice the landlord breached no express or implied duty to the tenant to repair the premises

The tenant does not have a defense based on the landlordrsquos acceptance of his surrender of the premises a landlordrsquos retention of keys does not constitute an acceptance of the tenantrsquos surrender unless the landlord so intended and here the landlordrsquos statements to the tenant at the time of the surrender of the keys do not evidence the intent to accept the tenantrsquos surrender

Under the common law a landlord has no duty to mitigate damages but also cannot sue for rents due in the future Under this approach the landlord can sue only for past-due rents Using this approach on November 1 the landlord could recover all the rent past due (ie rent for September and October) but could not recover for rents due in the future However some courts have authorized recovery for future rent minus the fair market rental value of the premises It is thus possible that the landlord could recover damages equal to the amount of rent due from September 1 to the end of the six-year lease term ($180000) minus the propertyrsquos fair-market rental value over that same period

Point One (45) The tenant was not constructively evicted because the landlord had no duty to repair the commercial premises that were the subject of the lease

The landlord and the tenant entered into a term-of-years lease because the lease specified both a beginning and an ending date HERBERT HOVENKAMP amp SHELDON F KURTZ THE LAW OF

PROPERTY 256 (5th ed 2001) Although a term-of-years lease normally cannot be terminated by the tenant prior to the end of the term a tenant may terminate a term-of-years lease if the tenant

15

Real Property Analysis

is constructively evicted See id at 286ndash88 Typically as here a claim of constructive eviction is made as a defense to a landlordrsquos action for damages or unpaid rent

In order to establish a constructive eviction the tenant must prove that the landlord breached a duty to the tenant such as a duty to repair and that the landlordrsquos breach caused a loss of the substantial use and enjoyment of the premises The tenant must also show that he gave the landlord notice adequate to permit the landlord to meet his duty to the tenant and that the tenant vacated the leased premises Id see also JOHN G SPRANKLING UNDERSTANDING

PROPERTY LAW sect 1704 (2d ed 2007) Under the common law there was no implied duty on the part of a landlord to repair

leased premises such a duty arose only if expressly set forth in the lease SPRANKLING supra sect 1702[B] Here the written lease contained no term requiring the landlord to repair the air-conditioning Even if the conversation created a lease term that the building had air-conditioning that itself should not create a duty for the landlord to repair it

Over the past several decades courts have generally implied a duty to repair in residential leases either as part of a revised constructive eviction doctrine or based on an implied warranty of habitability JOSEPH W SINGER PROPERTY 469ndash70 (3d ed 2010) This shift has been justified based on the economic disparity between the typical landlord and tenant as well as the fact that residential tenants generally lack both the authority to authorize repairs to common areas of a building and the incentive to make repairs that will ultimately benefit the landlord

However courts have been more reluctant to imply a duty to repair in commercial leases a context in which the tenant is often a valuable business and in a better position to assess and make repairs than is the landlord But see eg Davidow v Inwood North Professional Group 747 SW2d 373 (Tex 1988) When courts have implied a duty to repair in a commercial lease it is typically when the repair has been mandated by public authorities and involves work so substantial that it would not ordinarily fall within the tenantrsquos common law repair duty andor the value of the repair would primarily inure to the landlordrsquos reversionary interest See Brown v Green 884 P2d 55 (Cal 1994) Eugene L Grant et al The Tenant as Terminator Constructive Eviction in Modern Commercial Leases 2 THE COMMERCIAL PROPERTY LEASE ch 15 (ABA 1997) Some courts have also permitted constructive eviction claims by commercial tenants of office buildings based on repairs required in common areas of the building See id Echo Consulting Services Inc v North Conway Bank 669 A2d 227 (NH 1995)

Here the tenant is the owner of a valuable manufacturing operation and is the exclusive occupant of the building the repair has not been mandated by public authorities and the repair is not structural To the contrary the repair involves a feature of the building of unusual importance in the tenantrsquos manufacturing operation and the tenant is likely far more knowledgeable than the landlord about the air-conditioning specifications necessary for the manufacture of the tenantrsquos product

Based on these facts it is unlikely that a court will find that the tenant in this case was constructively evicted Although the tenant can show that he gave adequate notice to the landlord of the air-conditioning malfunction and vacated the premises the lease was commercial and it did not contain any promises or covenants by the landlord except a covenant of quiet enjoyment a covenant of quiet enjoyment does not entail any repair obligations

[NOTE An examineersquos conclusion is less important than his or her demonstrated awareness of the elements of constructive eviction and the need to imply a repair duty for such a defense to be viable here Although the implied warranty of habitability is not available to this tenant Texas Minnesota and Massachusetts imply a warranty of suitability in commercial leases in limited circumstances and an examinee might argue that this warranty should apply

16

Real Property Analysis

here If an examinee concludes that this warranty applies he or she should discuss the other requirements for constructive eviction

If the examinee wrongly concludes that the first element for a constructive eviction has been met the examinee will then have to discuss the remaining three elements in order to conclude that the tenant can claim constructive eviction The tenant would have a strong argument that the second elementmdashsubstantial interference with the use and enjoyment of the premisesmdashalso is met As indicated above the landlord was aware that a functioning air-conditioning system was vital to the tenantrsquos manufacturing operations The facts further indicate that the system had failed three times in the past few months The landlord may try to argue that the malfunctions did not substantially interfere with the tenantrsquos use of the premises because the malfunctions caused the temperature to climb above 81 degrees for only a short period of timemdash 3 hours 6 hours and 10 hours respectivelymdashon each occasion The tenant will argue however that the landlord was aware that the tenantrsquos manufacturing operations could tolerate temperatures above 81 degrees for no more than 6 hours The final malfunction exceeded that limit destroying $150000 worth of the tenantrsquos products

The tenant would also have a strong argument that the third element is met notice and opportunity to cure The tenant notified the landlord of the problem immediately upon the systemrsquos first malfunction and did so again when it malfunctioned a second time and then a third time The landlord might argue that there was insufficient time to cure the problem because the system corrected itself within a few hours on the first and second times Although the malfunction lasted more than 10 hours the third time the landlord might argue that the time period was insufficient to get a repair person on the premises A court would be likely to find this argument unpersuasive however because the landlord could have attempted to correct the problem after the first and second malfunctions

Assuming that the landlord was given sufficient notice and opportunity to cure a court would be likely to conclude that the tenant also satisfied the final element of vacating the premises within a reasonable time The landlord might argue that the tenant remained in the premises for almost four months after the air conditioning first failed which would suggest that the problem was not so severe as to have constructively evicted the tenant The tenant will argue however that he gave the landlord three months to cure the problem after the first two malfunctions threatened (but did not actually harm) his operations The tenant then moved out shortly after the final malfunction caused temperatures to exceed the tolerance levels of his manufacturing operations]

Point Two (10) The landlord did not accept the tenantrsquos surrender of the lease

When a tenant wrongfully moves from leased premises with the intent to terminate the lease the landlord may either accept the tenantrsquos surrender of the premises and terminate the lease or hold the tenant to the terms of the lease See HOVENKAMP amp KURTZ supra at 295ndash96 Here the tenantrsquos only basis for the claim that the landlord accepted his surrender is the landlordrsquos retention of the keys Many courts have considered whether a landlordrsquos retention of keys delivered by a tenant constitutes acceptance of surrender The weight of the case law holds that retention of the keys alone does not constitute acceptance of surrender without other evidence showing that the landlord intended to accept the surrender See generally 49 AM JUR 2d Landlord and Tenant sect 213

Here the landlordrsquos note saying ldquoI repeat the air-conditioning is not my problem You have leased the building and you should fix itrdquo strongly suggests that the landlord did not intend

17

Some courts have rejected the no-mitigation-of-damages rule based on efficiency concerns and societyrsquos interest in assuring that resources remain in the stream of commerce rather than lying vacant see id at 464ndash65 and allow landlords to sue tenants who have wrongfully terminated a lease for damages equal to the difference between the unpaid rent due under the lease and the propertyrsquos fair market rental value Other courts have abandoned the no-recovery-for-future-rent rule These courts responding to the fact that a tenant may well disappear or be judgment-proof by the time a lease term is concluded have allowed a landlord to collect damages equal to the value of rent over the entire lease term minus the propertyrsquos fair rental value when a tenant has wrongfully terminated a lease and unequivocally shown an intention not to return to the premises or pay future rent Under this approach a landlord receives approximately the same amount he would have received were there a duty to mitigate damages See Sagamore Corp v Willcutt 180 A 464 (Conn 1935)

Real Property Analysis

to accept the tenantrsquos surrender The tenant might argue that the landlordrsquos failure to make a similar statement when the keys were sent to her a second time and she retained them evidences a change of heart However it is likely that a court would find that the landlordrsquos retention of the keys represented a decision to safeguard the keys not to accept the tenantrsquos surrender

[NOTE An examinee should receive credit for arguing the other way with a well-reasoned argument]

Point Three (45) Under the common law the landlord had no duty to mitigate damages Additionally a landlord was not entitled to recover unpaid rents due in the future but was only entitled to recover rents in arrears at the time of the commencement of the suit Applying the common law here the landlord could recover $5000 the amount of rents due at the commencement of the suit ($2500 for September and the same for October) Today some courts allow the landlord under certain circumstances to sue the tenant for damages (not rent) equal to the difference if any between the unpaid promised rent for the balance of the term (here $175000) and the propertyrsquos fair rental value for the balance of the term

Under the common law because a lease was viewed as a conveyance instead of a contract a landlord had no duty to mitigate damages resulting from a tenantrsquos wrongful termination of a lease A landlord could thus recover the full value of rents that were due and unpaid at the time of the suit However under the common law a landlord could not sue a tenant for rents due in the future because there was always a possibility that the tenant might pay the rent when it was due See SINGER supra at 462 Thus using the common law approach on November 1 the landlord could only recover the full value of the two monthsrsquo rent actually due and unpaid ie $5000 for September and October

Here because the tenant returned the keys to the landlord and said ldquoI will not be returning to the building or making further rent paymentsrdquo the landlord could establish abandonment and an intention not to return It is thus possible that the landlord might recover damages in the amount of $5000 (for the months of September and October) plus the present value of $175000 minus the fair market rental value of the property over the remaining months of the lease

18

CONTRACTS ANALYSIS ____ (Contracts II IVE)

ANALYSIS

Legal Problems

(1) What was the legal effect of the sailorrsquos October 31 letter to the builder

(2)(a) What was the legal effect of the builderrsquos November 25 response to the sailorrsquos October 31 letter

(2)(b) What was the legal effect of the sailorrsquos refusal to take and pay for the boat on December 15

DISCUSSION

Summary

This is a sale of goods governed by the Uniform Commercial Code Because the sailor had reasonable grounds for insecurity about the builderrsquos ability to deliver the boat in a timely manner when the sailor learned about the strike on October 31 the sailor was legally justified in sending the letter to the builder seeking adequate assurance of the builderrsquos performance pursuant to the contract The builderrsquos failure to provide such assurance within a reasonable time operated as a repudiation of the contract However the builder was free to retract the repudiation before the sailor either cancelled the contract or materially changed position in reliance on the builderrsquos repudiation The builder retracted the repudiation when he informed the sailor that the workers were back and that the boat would be delivered by the date stipulated in the partiesrsquo contract Because the sailor had taken no action in response to the original repudiation he no longer had the right to cancel the contract with the builder The sailorrsquos subsequent statement that ldquoour contract is overrdquo may have constituted repudiation by the sailor In any event when the sailor failed to perform on December 15 that constituted breach

Point One (35) Because the sailor had reasonable grounds for insecurity with respect to the builderrsquos performance the sailorrsquos letter to the builder was a justified demand seeking assurance of the builderrsquos performance under the contract failure of the builder to provide such assurance within a reasonable time constituted repudiation of the contract

The sailor was legally justified in sending the letter to the builder on October 31 Contract parties are entitled to expect due performance of contractual obligations and are permitted to take steps to protect that expectation UCC sect 2-609 states that ldquo[w]hen reasonable grounds for insecurity arise with respect to the performance of either party the other may in writing demand adequate assurance of due performance rdquo Here the sailor learned on October 31 that the builderrsquos workers were on strike This gave the sailor reasonable grounds for insecurity about the builderrsquos ability to complete performance on time and thus gave the sailor the right to seek adequate assurance from the builder Because the sailorrsquos demand for assurance was justified the builder was required to provide assurance that was adequate under the circumstances within a reasonable time (not to exceed 30 days) or be held to have repudiated the contract UCC sect 2-609(4)

19

Contracts Analysis

Point Two(a) (30) The builder did not within a reasonable time provide the sailor adequate assurance of due performance this failure to provide assurance constituted a repudiation of the contract

Because the sailor with legal justification (see Point One) demanded from the builder assurance of due performance the builderrsquos failure to provide such assurance within a reasonable time was a repudiation of their contract See UCC sect 2-609(4) (ldquoAfter receipt of a justified demand[] failure to provide within a reasonable time not exceeding thirty days assurance of due performance is a repudiation of the contractrdquo) On October 31 the sailor requested that the builder provide adequate assurance regarding the completion of the boat by December 15 The builder did not respond to the sailorrsquos letter until November 25mdashnearly a month later Even if that response had been given in a reasonable time it nonetheless did not provide assurance of due performance It simply stated ldquoIrsquom sorry about the strike but it is really out of my hands I hope we settle it soon so that we can get back to workrdquo Therefore the builderrsquos November 25 response did not provide adequate assurance in response to the sailorrsquos justified request Thus the builder had repudiated the contract

Point Two(b) (35) Although the builder repudiated the contract with the sailor the builder probably retracted that repudiation on December 3 and the sailor was no longer entitled to cancel their contract Thus the sailorrsquos failure to perform the sailorrsquos obligations under the contract constituted a breach

The builderrsquos failure to provide adequate assurance of performance constituted a repudiation of their contract (see UCC sect 2-609(4)) but the builder was free to retract that repudiation until the sailor cancelled the contract or materially changed his position or indicated by communication or action that the sailor considered the repudiation to be final See UCC sect 2-611(1) (ldquoUntil the repudiating partyrsquos next performance is due he can retract his repudiation unless the aggrieved party has since the repudiation cancelled or materially changed his position or otherwise indicated that he considers the repudiation finalrdquo)

Here the facts state that before the builderrsquos December 3 telephone call to the sailor the sailor did nothing in response to the builderrsquos repudiation such as contracting with a third party for a boat The builderrsquos December 3 call informing the sailor that the boat would be timely delivered probably constituted a retraction of the repudiation because it clearly indicated to the sailor that the builder would be able to perform UCC sect 2-611(2) Thus after being so informed the sailor did not have the right to treat their contract as cancelled UCC sect 2-611(3) Accordingly the sailorrsquos failure to perform the sailorrsquos obligations under the contract by taking the boat and paying for it constituted a breach of the contract

20

CONSTITUTIONAL LAW ANALYSIS (Constitutional Law IVA F2b amp e)

ANALYSIS

Legal Problems

(1) Does AutoCorsquos operation of a ldquocompany townrdquo result in its actions counting as those of the state for purposes of constitutional analysis

(2) Does the expulsion of a schoolchild for failure to recite the Pledge of Allegiance violate the First Amendment as applied through the Fourteenth Amendment

(3) Does the arrest of a pamphleteer in connection with violation of an anti-littering rule where the littering is done by the recipients of leaflets distributed by the pamphleteer violate the First Amendment as applied through the Fourteenth Amendment

DISCUSSION

Summary

The First Amendment as applied through the Fourteenth Amendment applies only to state action It does not typically govern private actors However courts have found state action where the private actor has exercised a ldquopublic functionrdquo such as running a privately owned ldquocompany townrdquo as AutoCo has done here Thus First Amendment protections apply By requiring the son to participate in a mandatory Pledge of Allegiance ceremony AutoCo has compelled the expression of political belief in violation of the First Amendment as applied through the Fourteenth Amendment The fatherrsquos arrest in connection with breaching the anti-litter rule also violated the First Amendment as applied through the Fourteenth Amendment Although state actors can regulate the incidental effects of speech on the public streets on a content-neutral basis this power is limited and cannot extend to punishing a distributor of literature because of littering by third parties

Point One (30) AutoCorsquos operation of a company town (including a school) makes it a state actor under the public function strand of the state action doctrine

The individual rights protections of the Constitution apply only where there is ldquostate actionrdquomdash either direct action by the government or some action by a private party that is fairly attributable to the government As a general rule the actions of a private company like AutoCo or of a private school like the school operated by AutoCo would not constitute state action and the protections of the Constitution (in this case the First Amendment) would not apply

However there are situations in which the actions of a private actor are attributed to the state One such situation is when the private actor undertakes a public function There are not many bright-line rules in the Supreme Courtrsquos state action doctrine but one of them is this Where a private actor undertakes a ldquopublic functionrdquo the Constitution applies to those actions Where a corporation operates a privately owned ldquocompany townrdquo that provides essential services typically provided by a state actor the public function doctrine applies and the Constitution

21

Constitutional Law Analysis

binds agents of the town as if they were agents of the government See eg Marsh v Alabama 326 US 501 (1946) Here AutoCo does more than own the town it provides security services fire protection sanitation services and a school Thus the actions of AutoCo constitute state action and are governed by the Fourteenth Amendment

Point Two (35) The sonrsquos expulsion for failure to recite the Pledge of Allegiance violates the First Amendment as applied through the Fourteenth Amendment as a compelled expression of political belief

As explained in Point One the First Amendment applies to the school as a state actor Although children in public schools (and in schools subject to the First Amendment like

the Oakwood school) have some First Amendment rights Tinker v Des Moines Independent Community School District 393 US 503 506 (1969) schools have greater leeway to regulate the speech of students and teachers than the state would have outside the school context Hazelwood School Dist v Kuhlmeier 484 US 260 (1988) Morse v Frederick 551 US 393 (2007) However the Supreme Court has long held that public schools may not force their students to participate in a flag salute ceremony when it offends the political or religious beliefs of the students or their families West Virginia Board of Educ v Barnette 319 US 624 (1943) (invalidating a mandatory public school flag salute ceremony) see also Wooley v Maynard 430 US 705 (1977) (invalidating compelled expression of political belief on state-issued license plates)

In this case the school requires its students to participate in a flag salute and Pledge of Allegiance ceremony and punishes them when they refuse to participate Pursuant to this policy the school has expelled the son This expulsion violates the First Amendment ban on compelled expression

Point Three (35) Because the father was distributing leaflets in a traditional public forum his trespass arrest violated the First Amendment as applied through the Fourteenth Amendment

As explained in Point One AutoCo is treated as a state actor Thus Oakwoodrsquos commercial district is treated as government-owned property for purposes of the First Amendment Thus the leafleting here is subject to the First Amendment because it is an expressive activity Schneider v State of New Jersey Town of Irvington 308 US 147 (1939) When expression takes place on government-owned property government regulation of the expression is assessed under the public forum doctrine Public streets and sidewalks have long been held to be the classic example of a ldquotraditional public forumrdquo open to the public for expression Hague v CIO 307 US 496 515ndash16 (1939) Because the father was distributing leaflets while standing on a street corner in the commercial district his expressive activity occurred in a traditional public forum

When a state tries to regulate expressive activity in a traditional public forum it is prohibited from doing so based on the expressive activityrsquos content unless its regulation is narrowly tailored to achieve a compelling governmental interest (ldquostrict scrutinyrdquo) In this case however AutoCo is regulating the fatherrsquos expressive activity on the ostensibly neutral ground that his expressive activity has produced litter and made the street unsightly When a state tries to regulate expressive activity without regard to its content intermediate scrutiny applies Under intermediate scrutiny the true purpose of the regulation may not be the suppression of ideas (if so then strict scrutiny applies) the regulation must be narrowly tailored to achieve a significant

22

Constitutional Law Analysis

governmental interest and it must leave open ample alternative channels for expressive activity Ward v Rock Against Racism 491 US 781 791 (1989)

Here the application of the ordinance to the father will fail for two reasons First the Supreme Court has held that the governmentrsquos interest in keeping the streets clean is insufficient to ban leafleting in the public streets as the government power to regulate with incidental effects on public sidewalk speech is very limited See eg Schneider 308 US at 162 (leafletinglittering) Second the regulation (a blanket ban on distribution that results in littering) is not narrowly tailored to protect expression A narrowly tailored alternative would be prosecution only of people who litter Moreover the effect of the littering rule is likely to be a ban on all leafleting thus eliminating an entire class of means of expression This raises the possibility that there are not ldquoample alternative channels of communicationrdquo open to the father as required under the Courtrsquos standard of review for content-neutral regulation of speech

[NOTE Some examinees might argue that this is a ldquotime place and mannerrdquo restriction and that AutoCo might have greater latitude to regulate the public sidewalks under this theory This argument is incorrect for two reasons First the Supreme Court has held that the power to regulate speakers through littering laws is very limited for the reasons given and in the cases cited above But more generally a ldquotime place and mannerrdquo restriction involves the shifting of speech from one time and place to another or to another manner here there is no shifting but a direct punishment for expressive activity (albeit one couched in content-neutral terms) In addition some examinees might read the ordinance to be in effect a total ban on leafleting since most leafleting will produce some litter Those examinees might note that the Court has required total bans on an entire mode of expression to satisfy strict scrutiny and analyze the fatherrsquos prosecution here accordingly See United States v Grace 461 US 171 177 (1983) (invalidating ban on display of signs on public sidewalks surrounding US Supreme Court ldquo[a]dditional restrictions such as an absolute prohibition on a particular type of expression will be upheld only if narrowly drawn to accomplish a compelling governmental interestrdquo)]

23

SECURED TRANSACTIONS ANALYSIS (Secured Transactions IID E IVA B C)

ANALYSIS

Legal Problems

(1) Is a purchase-money security interest in consumer goods perfected even though there has been no filing of a financing statement

(2) Does a person who buys consumer goods for personal use take those goods free of a prior perfected purchase-money security interest in the goods

(3) Does a person who receives consumer goods as a gift take those goods subject to a prior perfected security interest in them

DISCUSSION

Summary

The retailerrsquos security interest in the bicycles was perfected even though no financing statement was filed because it was a purchase-money security interest in consumer goods A purchase-money security interest in consumer goods is automatically perfected upon attachment

The buyer is not subject to the retailerrsquos security interest in the bicycle that the buyer bought from the man Because the bicycle was consumer goods in the hands of the man and the retailer never filed a financing statement covering the bicycle the retailerrsquos security interest is not effective against someone like the buyer who bought the bicycle for value without knowledge of the retailerrsquos security interest and for personal use

On the other hand the retailerrsquos security interest continues in the bicycle given to the friend because the friend did not give value for the bicycle or buy it in the ordinary course of business

Point One (35) The retailerrsquos security interest in the bicycles attached on June 1 Because this interest was a purchase-money security interest in consumer goods it was automatically perfected when it attached

The retailerrsquos security interest in the bicycles attached on June 1 when the man bought the bicycles (acquiring rights in the collateral) signed a security agreement containing a description of the collateral and received value from the retailer (by being given credit with which to purchase the bicycles) UCC sect 9-203(a) amp (b)

Despite the retailerrsquos failure to file a financing statement its security interest was perfected Pursuant to UCC sect 9-309(1) a security interest is automatically perfected upon attachment if the goods are ldquoconsumer goodsrdquo and the security interest is a ldquopurchase-money security interestrdquo

In this case the bicycles sold by the retailer to the man were consumer goods at the time of sale The bicycles were ldquogoodsrdquo because they were ldquomovable when a security interest

24

Secured Transactions Analysis

attachesrdquo UCC sect 9-102(a)(44) They were also consumer goods because they were ldquobought for use primarily for personal family or household purposesrdquo UCC sect 9-102(a)(23) The retailerrsquos security interest in these consumer goods was also a ldquopurchase-money security interestrdquo A purchase-money security interest is an interest that secures a debt that was incurred in order to ldquoenable the debtor to acquire rights in or the use of the collateralrdquo UCC sect 9-103(a) (b)(1) Here the man incurred an obligation to the retailer to purchase the bicycles so the security interest he gave the retailer to secure that obligation was a purchase-money security interest

Because the retailerrsquos security interest was a purchase-money security interest in consumer goods it was automatically perfected on June 1 when the interest attached to the bicycles

Point Two (35) The buyer took the bicycle free of the retailerrsquos security interest because (i) the retailer did not file a financing statement covering the bicycle (ii) the bicycle was ldquoconsumer goodsrdquo and (iii) the buyer bought the bicycle for value without knowledge of the retailerrsquos security interest and for personal use

A security interest continues in collateral even after a sale or other disposition of that collateral unless the creditor authorized the disposition ldquofree of the security interestrdquo or another Article 9 exception applies UCC sectsect 9-201(a) and 9-315(a)(1)

However a buyer of goods like the buyer here can take free of a prior security interest in those goods under certain circumstances See UCC sectsect 9-317(b) (buyers who give value and receive delivery of goods without knowledge of an unperfected security interest in the goods) and 9-320(a) amp (b) (buyer in ordinary course of business buyer of consumer goods in a consumer-to-consumer transaction who gives value) In this case the retailerrsquos security interest was perfected when the buyer purchased the bicycle so UCC sect 9-317(b) does not protect the buyer The buyer also is not a protected ldquobuyer in ordinary course of businessrdquo because he did not purchase from a person who is in the business of selling bicycles See UCC sect 1-201(b)(9)

The buyer can however qualify for the protection of UCC sect 9-320(b) That section provides that a buyer of goods from a person who used them for personal family or household purposes takes free of a perfected security interest in the goods if (1) the buyer had no knowledge of the security interest (2) the buyer gave value for the goods (3) the buyer purchased the goods primarily for personal family or household purposes and (4) the purchase occurred before the filing of a financing statement covering the goods

The buyer met all of these criteria The man used the bicycle for personal purposes The buyer purchased the bicycle from the man and the buyer had no knowledge of the retailerrsquos security interest The buyer gave value ($400) for the bicycle and he bought it ldquoprimarily for personal family or household purposesrdquo as he planned to use it for recreation which is a personal rather than a business use Finally no financing statement had been filed Therefore under UCC sect 9-320(b) the buyer took free of the retailerrsquos security interest

Point Three (30) The retailerrsquos security interest continues in the bicycle that the man gave to the friend Thus the retailer can recover the bicycle from the friend because the friend did not give value for the bicycle or buy it in the ordinary course of business

25

Secured Transactions Analysis

As noted in Point Two the retailer did not authorize the man to dispose of the bicycle Consequently the retailerrsquos security interest continued in the bicycle even after the man transferred ownership of the bicycle to the friend See UCC sectsect 9-201(a) and 9-315(a)(1) The retailerrsquos security interest in the bicycle will be effective against the friend unless some other provision of Article 9 allows the friend to take the bicycle free of that security interest

Unfortunately for the friend there is no Article 9 provision that allows him to take free of the retailerrsquos interest The friendrsquos basic problem is that he is not a buyer of the bicyclemdashhe received the bicycle as a gift and did not give value for it Thus the friend is not protected by any of the applicable exceptions See UCC sectsect 9-317(b) (protecting buyers who give value for goods subject to an unperfected security interest) 9-320(a) (protecting buyers in ordinary course of business) and 9-320(b) (protecting buyers of consumer goods who give value)

In short the retailerrsquos security interest continues in the bicycle that the man gave to the friend The friend took the bicycle subject to that security interest

26

FEDERAL CIVIL PROCEDURE ANALYSIS (Federal Civil Procedure VIE)

ANALYSIS

Legal Problems

(1) Does a judgment in a prior action preclude a nonparty from suing the same defendant on a closely related claim when the nonparty and the original plaintiff are in a family relationship

(2) Does a judgment rendered in an earlier action preclude a nonparty from litigating an issue that was actually decided in the first suit

(3) May a nonparty to an earlier action invoke the judgment in that action to preclude a party to the prior action from relitigating an issue that the party had a full and fair opportunity to litigate in the earlier action

DISCUSSION

Summary

Pursuant to the doctrines of claim preclusion (res judicata) and issue preclusion (collateral estoppel) a judgment is binding on the parties thereto In the absence of privity nonparties to a prior suit cannot be bound by a judgment rendered in their absence Thus in the absence of privity a nonparty to the first suit is not precluded from presenting her claim in a second suit even if it is factually related to the claims and defenses presented in the first suit nor is she bound by determinations of issues made in the first suit A family relationship without more does not support a finding of privity For this reason Mother as a nonparty is not bound by the judgment in the Son-Driver action She may bring her separate claim for damage to her car and she is not precluded from litigating the question of whether she was negligent in the maintenance of her car

Driver on the other hand could be precluded from relitigating the issue of her negligence pursuant to the doctrine of non-mutual issue preclusion (also called non-mutual offensive collateral estoppel) which allows a nonparty to a prior action to invoke issue preclusion to prevent a party to that prior action from relitigating determinations of issues made therein However Mother may be prevented from invoking non-mutual collateral estoppel in this case because she could easily have joined her claim in the prior action but did not do so

[NOTE Federal common law governs the preclusive effect of a judgment rendered by a federal court sitting in diversity See Semtek Intrsquol Inc v Lockheed Martin Corp 531 US 497 508 (2001) But the Semtek Court concluded that federal common law in this context incorporates the preclusion law of the state in which the rendering federal court sits (unless the state law is incompatible with federal interests) id at 508ndash09 Thus State Arsquos preclusion law determines the preclusive effect of the judgment rendered in Sonrsquos suit against Driver The problem says that State A preclusion law is identical to federal preclusion law so the following analysis utilizes general principles of preclusion drawn from Supreme Court case law (announcing federal preclusion rules) and the Restatement (Second) of Judgments]

27

Federal Civil Procedure Analysis

Point One (35) Under the doctrine of claim preclusion the judgment rendered in the first action does not preclude Mother a nonparty from suing Driver for the damage to her car because the judgment binds only parties or those in privity with them and Mother and Son are not in privity

Driver may contend that the doctrine of claim preclusion (res judicata) precludes Mother from presenting a claim arising from the same nucleus of facts that was presented in the first action brought by Son According to the doctrine of claim preclusion ldquowhen a court of competent jurisdiction has entered a final judgment on the merits of a cause of action the parties to the suit and their privies are thereafter bound lsquonot only as to every matter which was offered and received to sustain or defeat the claim or demand but as to any other admissible matter which might have been offered for that purposersquordquo Commissioner of Internal Revenue v Sunnen 333 US 591 597 (1948) (citation omitted)

However the doctrine of claim preclusion does not apply to Mother on the facts of this problem First Mother was not a party to the earlier case ldquoIt is a principle of general application in Anglo-American jurisprudence that one is not bound by a judgment in personam in a litigation in which he is not designated as a party or to which he has not been made a party by service of processrdquo Taylor v Sturgell 553 US 880 884 (2008) (citing Hansberry v Lee 311 US 32 40 (1940)) see also RESTATEMENT (SECOND) OF JUDGMENTS sect 34(3) (1982) This rule reflects our ldquodeep-rooted historic tradition that everyone should have his own day in courtrdquo Martin v Wilks 490 US 755 762 (1989) (citation omitted) (superseded by statute on other grounds) Since Mother was not a party to the first suit she is not bound by the judgment unless an exception to the general rule applies

Mother might be bound by the prior judgment if she were considered to have been sufficiently in privity with Son that Son represented her interests in that action ldquoA person who is not a party to an action but who is represented by a party is bound by and entitled to the benefits of a judgment as though he were a partyrdquo RESTATEMENT (SECOND) OF JUDGMENTS sect 41(1) But there is no suggestion in the facts of the problem that Son who is an adult purported to represent Motherrsquos interests in the first suit ldquo[C]lose family relationships are not sufficient by themselves to establish privity with the original suitrsquos party or to bind a nonparty to that suit by the judgment entered therein rdquo Cuauhtli v Chase Home Finance LLC 308 Fed Appx 772 773 (5th Cir 2009) (citation omitted) accord 18A CHARLES ALAN WRIGHT ET AL FEDERAL

PRACTICE AND PROCEDURE sect 4459 (2d ed 2002) In Taylor v Sturgell supra the Supreme Court identified other special circumstances in

which nonparties may be bound by a prior judgmentmdashwhen a nonparty consents to be bound when a nonparty is in a pre-existing substantive legal relationship with a party (such as preceding and succeeding property owners) when a nonparty assumed control of the prior litigation when a party seeks to relitigate through a proxy or where a special statutory scheme seeks to foreclose successive litigation by nonparties See Taylor 553 US at 893ndash95 None of these circumstances exists here

Because Mother was not a party to the first suit and is not in privity with Son who is an adult the judgment in the first action does not preclude her from bringing her own claim against Driver

Point Two (35) Under the doctrine of issue preclusion the judgment rendered in the first action does not preclude Mother a nonparty from litigating the issue of her negligence in maintaining her carrsquos

28

Federal Civil Procedure Analysis

brake lights because the judgment binds only parties or those in privity with them and Mother and Son are not in privity

By its affirmative response to a special interrogatory the jury in the first action expressly concluded that ldquoMother negligently failed to ensure that the brake lights on her car were in proper working orderrdquo Driver may attempt to invoke the doctrine of issue preclusion to preclude Mother from relitigating this issue in the second action

[I]ssue preclusion arises in a second action on the basis of a prior decision when the same lsquoissuersquo is involved in both actions the issue was lsquoactually litigatedrsquo in the first action after a full and fair opportunity for litigation the issue was lsquoactually decidedrsquo in the first action by a disposition that is sufficiently lsquofinalrsquo lsquoon the meritsrsquo and lsquovalidrsquo it was necessary to decide the issue in disposing of the first action and the later litigation is between the same parties or involves nonparties that are subject to the binding effect or benefit of the first action Once these requirements are met issue preclusion is available not only to defend against a demand for relief but also as offensive support for a demand for relief Issue preclusion moreover is available whether or not the second action involves a new claim or cause of action

18 CHARLES ALAN WRIGHT ET AL FEDERAL PRACTICE AND PROCEDURE sect 4416 at 392ndash93 (2d ed) see also RESTATEMENT (SECOND) OF JUDGMENTS sect 27 (1982)

Here several of the elements necessary for issue preclusion are present The same issue is involved in both actionsmdashthe issue of Motherrsquos negligence in failing to maintain the brake lights on her car That issue was actually litigated in the first action and decided by the jury There is nothing to suggest anything less than a full and fair opportunity to litigate The judgment disposing of the issue was final

Nevertheless the judgment will not preclude Mother from relitigating the issue for two reasons First Mother was not a party to the first action and as explained above Mother and Son are not in privity Therefore she cannot be denied an opportunity to litigate the issue of her negligence Second it does not appear that the juryrsquos decision as to Motherrsquos negligence was necessary to the prior judgment against Driver Nothing suggests that the finding on Motherrsquos negligence had any bearing on the outcome of the first action

Point Three (30) Under the doctrine of non-mutual issue preclusion the judgment rendered in the first action might preclude Driver from relitigating the issue of her negligence However Driver has a strong argument that such a result would be inconsistent with the policy against offensive use of non-mutual estoppel when the non-party plaintiff easily could have joined as a plaintiff in the first action

Because Son already convinced the jury in the first action that ldquoDriver was negligent in the operation of her vehiclerdquo Mother may wish to invoke the doctrine of non-mutual issue preclusion to prevent Driver from relitigating the question of her negligence As noted above ldquoissue preclusion arises in a second action on the basis of a prior decision when the same lsquoissuersquo is involved in both actions the issue was lsquoactually litigatedrsquo in the first action after a full and fair opportunity for litigation the issue was lsquoactually decidedrsquo in the first action by a disposition that is sufficiently lsquofinalrsquo lsquoon the meritsrsquo and lsquovalidrsquo it was necessary to decide the issue in disposing of the first action rdquo 18 CHARLES ALAN WRIGHT ET AL FEDERAL PRACTICE AND

PROCEDURE sect 4416 at 392 (2d ed) see also RESTATEMENT (SECOND) OF JUDGMENTS sect 27

29

Federal Civil Procedure Analysis

Here these basic requirements for issue preclusion are met First the same issue is involved in both suits whether Driver was negligent in the operation of her car Second this issue was actually litigated and decided in the first action the jury answered a special interrogatory raising this very question There is nothing to suggest that Driver lacked a full and fair opportunity to litigate the issue Since a judgment was rendered against Driver for the injuries Son sustained as a result of Driverrsquos negligence resolution of the issue was necessary to dispose of the first action Driver was a party to the first action so she may be bound by the judgment

[NOTE Traditionally issue preclusion required mutualitymdashboth the party asserting issue preclusion and the party against whom issue preclusion was asserted were bound by the prior judgment Under the traditional mutuality rule Mother could not assert issue preclusion against Driver because Mother would not be bound by the judgment if Driver sought to rely on it See Point One There is no mutuality between Mother and Driver with respect to the prior judgment

This traditional mutuality requirement has been abandoned in most jurisdictions The Supreme Court rejected a strict mutuality requirement in Blonder-Tongue Laboratories Inc v University of Illinois Foundation 402 US 313 (1971) (non-mutual defensive collateral estoppel used by a defendant to preclude a plaintiff from relitigating a claim the plaintiff previously litigated) and Parklane Hosiery Co v Shore 439 US 322 (1979) (non-mutual offensive collateral estoppel used by a plaintiff to preclude a defendant from relitigating a claim the defendant previously litigated) In Parklane Hosiery the Court concluded (as a matter of federal preclusion law) that trial courts should have ldquobroad discretionrdquo to determine whether or not to permit a plaintiff to invoke non-mutual issue preclusion ldquoThe general rule should be that in cases where a plaintiff could easily have joined in the earlier action or where the application of offensive estoppel would be unfair to a defendant a trial judge should not allow the use of offensive collateral estoppelrdquo Id at 331

The Parklane Hosiery decision identified a number of circumstances that might make it unfair to allow a plaintiff to invoke non-mutual issue preclusion (non-mutual offensive collateral estoppel in the traditional terminology) against a defendant In particular the Parklane Hosiery court suggested that issue preclusion may not be appropriate if the plaintiff in the second action ldquocould easily have joined in the earlier actionrdquo Id Prohibiting plaintiffs from using non-mutual estoppel under such circumstances would promote judicial efficiency by encouraging plaintiffs to join the prior action It would also discourage plaintiffs from staying out of prior litigation in order to secure in effect two bites at the apple using the prior litigation offensively if the defendant loses and forcing the defendant to litigate a second time if the defendant wins the prior action

An exceptional exam answer might therefore argue that non-mutual issue preclusion should be denied on these facts Son and Mother both reside in State A since they are related they know each other well and Son was driving Motherrsquos car when the accident occurred They could have sued together and Rule 20 of the Federal Rules of Civil Procedure would have authorized joinder of their claims because those claims arose from the same transaction or occurrence and raised a common question of law or fact FED R CIV P 20(a) The facts do not suggest that Mother had any reason not to join Sonrsquos suit other than a desire to see how Sonrsquos action concluded before bringing her own claim Cf Nations v Sun Oil Co (Del) 695 F2d 933 938 (5th Cir 1983) (concluding that plaintiff ldquowas entitled to await the development of his injuries and their predictable consequencesrdquo) Because it appears that Mother may be a ldquowait-and-seerdquo plaintiff who could easily have joined the original action a trial court might disallow as a matter of discretion her use of non-mutual issue preclusion]

30

AGENCY ANALYSIS __________ (Agency I II)

ANALYSIS

Legal Problems

(1) Is the principal or the agent or both liable on contracts with a third party when the principal is an ldquoundisclosed principalrdquo

(2) Is the principal or the agent or both liable on contracts with a third party when the principal is ldquopartially disclosedrdquo or an ldquounidentified principalrdquo

(3) Is the principal or the agent or both liable on contracts with a third party for the purchase of goods when the agent exceeded his authority but the principal nonetheless accepts the goods

DISCUSSION

Summary

The agent but not the owner is liable to the basket manufacturer because the owner is an undisclosed principal and the agent acted without actual or apparent authority Both the agent and the owner however are liable on the burner contract because the owner is an unidentified principal and the agent had apparent authority to enter into that contract With respect to the solar cells contract whether the owner is liable depends upon whether a court would follow the Second or Third Restatement of Agency which take different positions on the effect of the ratification of a contract by an undisclosed principal Under either the agent would also be liable on the contract as he was a party to the contract

[NOTE The contracts that are the subject of this question are contracts for the sale of goods and therefore are governed by Article 2 of the Uniform Commercial Code Article 2 however does not contain agency rules Accordingly common law concepts of agency are applicable UCC sect 1-103(b)]

Point One (35) The agent but not the owner is liable to the basket manufacturer The agent had no actual authority to enter into the contract to buy aluminum baskets and because the owner was an undisclosed principal the manufacturer had no reason to believe that the agent had apparent authority Furthermore the manufacturer had no reason to believe that the agent was not contracting for his own benefit

An agent acting on behalf of a principal can bind the principal to contracts if the agent has either actual or apparent authority An agent has actual authority when contracting on behalf of his principal if he ldquoreasonably believes in accordance with the principalrsquos manifestations to the agent that the principal wishes the agent so to actrdquo RESTATEMENT (THIRD) OF AGENCY sect 201 (2006) Here the agent was told to buy only wicker baskets not aluminum baskets Thus when he contracted with the basket manufacturer to buy aluminum baskets he had no actual authority to do so

31

Agency Analysis

An agent acts with apparent authority ldquowhen a third party [with whom the agent acts] reasonably believes the actor has authority to act on behalf of the principal and that belief is traceable to the principalrsquos manifestationsrdquo Id sect 203 Here the owner notified basket manufacturers that she or her agent might contact them to purchase baskets but that notification did not specifically name the agent or any other person as the ownerrsquos agent Furthermore the basket manufacturer had no prior dealings with the agent or the owner or any reason to think that the agent was acting for the benefit of anyone but himself Thus there is no basis to conclude that the basket manufacturer thought that the agent had apparent authority to act for the owner

Generally when an agent acts on behalf of an undisclosed principal and the agent lacks authority to enter into the contract the agent is liable on the contract as a party to the contract but the principal is not liable This rule is consistent with the third partyrsquos expectations ldquoThe third party expected the agent to be a party to the contract because the agent presented the deal as if he were acting for himself Moreover if the third party is unaware of the principalrsquos existence the third party must be relying on the agentrsquos solvency and reliability when entering into the contractrdquo See ROBERT W HAMILTON JONATHAN R MACEY amp DOUGLAS K MOLL CORPORATIONS INCLUDING PARTNERSHIPS AND LIMITED LIABILITY COMPANIES 34 (11th ed 2010) See also RESTATEMENT (THIRD) OF AGENCY sect 603 cmt c Furthermore because the third party has no idea that the agent is acting or is seemingly acting on behalf of another there is no reason to believe that the third party would be expecting an undisclosed principal to be liable on the contract Id

Point Two (35) Because the owner is an unidentified (as opposed to undisclosed) principal both she and the agent (as a party to the contract) probably are liable on the contract with the burner manufacturer

When the agent contracted with the burner manufacturer he did not have actual authority to do so as the owner had expressly restricted the agentrsquos authority to purchase only burners with ldquowhisper technologyrdquo See Point One However the agent may have had apparent authority to buy burners without whisper technology

An agent acts with apparent authority ldquowhen a third party [with whom the agent acts] reasonably believes the actor has authority to act on behalf of the principal and that belief is traceable to the principalrsquos manifestationsrdquo RESTATEMENT (THIRD) OF AGENCY sect 203 (2006) The owner indicated that an agent might contact the burner manufacturer The notice contained no restriction regarding the type of burners that the agent was authorized to purchase The facts indicate that burner manufacturers regularly receive such notices

Although the agent told the burner manufacturer that he represented a well-known hot-air balloon operator he did not disclose the ownerrsquos name Thus the owner was a partially disclosed or unidentified principal See RESTATEMENT (SECOND) OF AGENCY sect 4(2) (1958) (using term ldquopartially disclosed principalrdquo) RESTATEMENT (THIRD) OF AGENCY sect 104(2)(c) (2006) (using term ldquounidentified principalrdquo) An agent for a partially disclosed principal may have apparent authority RESTATEMENT (SECOND) OF AGENCY sect 159 cmt e (1958) Based upon (1) the notice sent by the owner (2) the agentrsquos revelation that he was acting as an agent and (3) the fact that burner manufacturers regularly receive such notices and sell to agents the manufacturer may argue that it reasonably and actually believed that the agent was authorized to purchase burners without whisper technology The manufacturer may also argue that because the agent revealed that he was an agent his listing of the ownerrsquos address as the delivery address connects the agent to the notice given by the owner Arguably this distinguishes the burner contract from the basket

32

Agency Analysis

contract Here there is a strong case to support the conclusion that the agent had apparent authority if he did then the owner is liable to the burner manufacturer

The agent also is liable as a party to the contract because he did not fully disclose his agency relationship Although he told the burner manufacturer that he represented a well-known hot-air balloon operator he did not disclose the ownerrsquos name Generally even an authorized agent of a partially disclosed or unidentified principal is liable as a party to a contract with a third person RESTATEMENT (SECOND) OF AGENCY sect 321 (1958) (ldquounless otherwise agreedrdquo) RESTATEMENT (THIRD) OF AGENCY sect 602(2) (2006) (ldquounless the agent and the third party agree otherwiserdquo)

Point Three (30) Under the Second Restatement of Agency the owner is not liable on the contract for solar cells because the agent did not have actual or apparent authority and the owner as an undisclosed principal cannot ratify the contract Under the Third Restatement the owner could be liable as she ratified the contract Under either Restatement the agent is liable as a party to the contract

The owner is not liable to the solar cell manufacturer for breach of the contract for the solar cells because the agent had no actual or apparent authority to purchase solar cells on the ownerrsquos behalf and the owner under the Second Restatement of Agency did not ratify the contract with knowledge of the material facts Thus she is not liable as a ratifier of the contract

The facts state that the agent had authority to purchase only propane fuel tanks In addition he had no apparent authority to purchase solar cells The owner made no manifestations to the solar cell manufacturer that would lead a reasonable person in the manufacturerrsquos position to believe that the agent had the authority to bind the owner to a contract to purchase solar cells In fact the agent made no manifestations at all to the solar cell manufacturer Unlike with the basket manufacturer and the burner manufacturer the owner did not notify the manufacturer of solar cells that an agent might contact it to purchase solar cells In addition the solar cells were delivered to the agent and not to the ownerrsquos address In sum the manufacturer was unaware of any relationship between the owner and the agent As to the solar cell manufacturer the owner is an undisclosed principal There can be no apparent authority in the case of an undisclosed principal because there are no manifestations from the principal to the third person See RESTATEMENT (SECOND) OF AGENCY sect 8 cmt a (1958) (ldquothere can be no apparent authority created by an undisclosed principalrdquo) RESTATEMENT (THIRD) OF AGENCY sect 203 cmt f (2006) (ldquoapparent authority is not present when a third party believes that an interaction is with an actor who is a principalrdquo)

The owner also did not ratify the contract Although the owner used the solar cells generally a principal cannot ratify an unauthorized transaction with a third person ldquounless the one acting purported to be acting for the ratifierrdquo RESTATEMENT (SECOND) OF AGENCY sect 85(1) (1958)

The result differs under the Third Restatement which expressly rejects the Second Restatement on this issue The Restatement (Third) of Agency sect 403 (2006) states ldquoA person may ratify an act if the actor acted or purported to act as an agent on the personrsquos behalfrdquo According to comment b ldquoan undisclosed principal may ratify an agentrsquos unauthorized actrdquo Under the Restatement (Third) of Agency rule the owner probably ratified the transaction The agent clearly acted on the ownerrsquos behalf and in addition the ownerrsquos conduct in using the solar cells ldquojustifies a reasonable assumption that [she] is manifesting assent that the act shall affect [her] legal relationsrdquo See id sect 401(2)

33

Agency Analysis

The agent also is liable to the solar cell manufacturer for breach of the contract for the solar cells because he is a party to the contract The facts indicate that the agent never told the solar cell manufacturer that he represented the owner or any other principal Consequently even if the agent were authorized (which as discussed above he is not) he would be liable as a party to the contract See RESTATEMENT (SECOND) OF AGENCY sect 322 (1958) RESTATEMENT (THIRD) OF AGENCY sect 603(2) (2006) Here he has no authority or apparent authority and is liable as a party to the contract

The agent would also be liable under the Third Restatement Under Restatement (Third) of Agency sect 402(1) (2006) ratification generally relates back and the transaction is treated as if it were authorized at the time of the transaction However this does not relieve the agent of an undisclosed principal who ratifies an unauthorized transaction of liability under the ratified contract See id sect 603(2) (authorized agent for undisclosed principal is a party to the contract) and sect 403 cmt b (ldquoAn undisclosed principalrsquos ratification does not eliminate the agentrsquos liability to the third party on the transaction rdquo)

[NOTE An examinee may discuss the concept of inherent agency power This concept is recognized by the Restatement (Second) of Agency sect 8 A (1958) but the concept is not used in the Restatement (Third) of Agency (2006) Here there are no facts to support that the agent had inherent authority

As to contracts with agents for partially disclosed principals (eg the contract for the burners) the basic question is whether the acts done ldquousually accompany or are incidental to transactions which the agent is authorized to conductrdquo RESTATEMENT (SECOND) OF AGENCY

sect 161 (1958) If so the principal is bound if the other party ldquoreasonably believes that the agent is authorized to do them and has no notice that he is not so authorizedrdquo Id The purchase of burners without whisper technology was not authorized nor was it incidental to an authorized transaction Therefore there should not be inherent agency power

As to contracts on behalf of undisclosed principals (eg the other two contracts) the basic question is whether the acts done are usual or necessary in the transactions the agent is authorized to transact RESTATEMENT (SECOND) OF AGENCY sect 194 (1958) The other two contracts seem fundamentally different from the authorized transactions Therefore there should not be inherent agency power

Only minimal credit should be given for discussion of inherent agency power]

34

EVIDENCE ANALYSIS _____ (Evidence IIA VA B E F J K)

ANALYSIS

Legal Problems

(1) Is the authenticated copy of the mechanicrsquos text message relevant and admissible

(2) Is the womanrsquos question ldquoIs my scooter safe to drive for a whilerdquo relevant and admissible

(3) Is the womanrsquos testimony describing the mechanicrsquos thumbs-up relevant and admissible

DISCUSSION

Summary

The mechanicrsquos text message to the woman is relevant to whether (1) the woman lost control of the scooter due to its defective brakes (2) the woman knew that the brakes needed repair and (3) it was negligent for the woman to drive the scooter knowing that its brakes needed repair

The mechanicrsquos text message is hearsay if it is offered by the pedestrian to prove that the scooterrsquos brakes needed repair However it fits the hearsay exception for present sense impressions and probably also fits the exception for business records The mechanicrsquos text message is not hearsay if it is instead offered by the pedestrian to prove the womanrsquos state of mind (ie that she had notice that her brakes needed repair)

The womanrsquos question to the mechanic and his response are also relevant to whether the brakes caused the accident and whether the woman was negligent The question is not hearsay because the woman did not make an assertion

The mechanicrsquos thumbs-up response is nonverbal conduct intended by the mechanic as an assertion and is therefore an out-of-court statement If the woman offers the mechanicrsquos statement to prove that the scooter was actually safe to ride the womanrsquos testimony about the statement is hearsay

However the mechanicrsquos statement is not hearsay if it is offered by the woman to prove her state of mind Therefore the womanrsquos question and the mechanicrsquos response are admissible to prove the womanrsquos state of mind

Point One(a) (20) The mechanicrsquos text message to the woman should be admitted because it is relevant

Evidence is relevant if it has ldquoany tendency to make a fact more or less probable than it would be without the evidencerdquo FED R EVID 401 ldquoRelevant evidence is admissiblerdquo unless it is inadmissible pursuant to some other rule FED R EVID 402

The mechanicrsquos text message to the woman ldquoWhen you pick up your scooter you need to schedule a follow-up brake repair Wersquoll order the partsrdquo is relevant for two reasons First this evidence has some tendency to make it more probable that the brakes malfunctioned and

35

Evidence Analysis

caused the accident Second it has some tendency to make it more probable that the woman was negligent in riding her scooter after being told by the mechanic that it required further repair

Point One(b) (30) The mechanicrsquos text message fits either the hearsay exception for present sense impressions or the exception for business records or it is admissible non-hearsay

The mechanicrsquos text message is a statement under Rule 801(a) because it is ldquoa written assertionrdquo FED R EVID 801(a) The text message is hearsay if the pedestrian offers it to prove the ldquotruth of the matter asserted in the statementrdquo (ie that the scooterrsquos brakes required repair) which resulted in the woman losing control of the scooter and causing the accident FED R EVID 801(c)

However the mechanicrsquos text message fits the hearsay exception for ldquopresent sense impressionsrdquo under Rule 803(1) because it is ldquo[a] statement describing or explaining an event or condition made while or immediately after the declarant perceived itrdquo FED R EVID 803(1) Here the mechanicrsquos text message described the condition of the scooter immediately after he perceived it during the maintenance service

The mechanic is a person with knowledge of the condition of the scooter so if text messages regarding repairs were made and kept by the mechanic in the ordinary course of business this text message also fits the business records exception Under Rule 803(6) a business record is a record of an act ldquomade at or near the time by someone with knowledgerdquo and ldquothe record was kept in the course of a regularly conducted activity of a businessrdquo and ldquomaking the record was a regular practice of that activityrdquo FED R EVID 803(6)

However the text message is not hearsay if it is instead offered to prove that the woman was negligent because she rode her scooter after the mechanic told her it required repair If offered for this purpose it would not be offered for the truth of the matter asserted in the statement but to show the womanrsquos belief about the condition of the scooter (her state of mind)

Point Two (10) The womanrsquos question to the mechanic should be admitted because it is not hearsay

The womanrsquos question to the mechanic is relevant because along with the mechanicrsquos thumbs-up response (see Point Three) it has some tendency to make it more probable that the woman was not negligent andor that the scooter brakes did not malfunction and cause the accident FED R EVID 401 The womanrsquos question does not raise hearsay concerns because it is not an assertion

Hearsay is defined under Rule 801(a) as ldquoan oral assertion written assertion or nonverbal conductrdquo Although ldquoassertionrdquo is not further defined ldquoa favorite [definition] of writers in the [evidence] field for at least a century and a half [is that] the word simply means to say that something is so eg that an event happened or a condition existedrdquo 2 MCCORMICK ON

EVIDENCE sect 246 (6th ed 2006) Under this definition the womanrsquos question is not hearsay because it is not an assertion

Point Three(a) (20) The mechanicrsquos thumbs-up to the woman is a nonverbal assertion that is relevant and the womanrsquos testimony about that response is admissible

36

Evidence Analysis

Hearsay is defined under Rule 801(c) as a ldquostatementrdquo that is ldquoa personrsquos oral assertion written assertion or nonverbal conduct if the person intended it as an assertionrdquo FED R EVID 801(a) Here when the mechanic responded to the womanrsquos question (ldquoIs my scooter safe to ride for a whilerdquo) with a thumbs-up gesture the facts suggest that he intended his nonverbal conduct as an assertion that in his opinion the scooter was safe to ride

The mechanicrsquos assertion is relevant and admissible to prove that the woman was not negligent because the evidence makes it more probable that at the time of the accident she believed that the scooter was safe to ride despite the fact that the brakes required repair FED R EVID 401 Admission of the womanrsquos description of the mechanicrsquos thumbs-up for this purpose does not raise hearsay concerns because the evidence would not be offered for the truth of the matter asserted but to show the womanrsquos belief about the condition of the scooter (her state of mind)

Point Three(b) (20) The mechanicrsquos thumbs-up is relevant to determine whether the scooterrsquos brakes malfunctioned causing the accident but if offered for this purpose it is also hearsay

The mechanicrsquos nonverbal assertion is relevant to the determination of whether the scooterrsquos brakes malfunctioned causing the accident However if offered to prove the ldquotruth of the matter asserted in the statementrdquo (ie that the scooter was safe to ride for a while) it is hearsay that does not fit any hearsay exception

37

TRUSTS AND FUTURE INTERESTS ANALYSIS ____________________ (Trusts and Future Interests IC1 amp 4 G IIF)

ANALYSIS

Legal Problems

(1)(a) Was the revocable trust amendable

(1)(b) If the trust was amendable must the amendment have been executed in accordance with the state Statute of Wills in order to be valid

(2) If the trust amendment was valid does the amendment apply to the probate estate assets passing to the trust pursuant to Settlorrsquos will

(3) If the trust amendment was valid should the trust property be distributed to University

(4) If the trust amendment was not valid should the trust property be distributed to Settlorrsquos grandchild (her only heir) or held in further trust in accordance with the terms of the original trust instrument

DISCUSSION

Summary

A revocable trust is amendable even if the trust instrument does not expressly grant to the trust settlor a power to amend Both inter vivos trusts and amendments thereto are valid even though not executed in accordance with the requirements applicable to wills

Under the Uniform Testamentary Additions to Trusts Act a revocable trust may be amended at any time prior to the settlorrsquos death and the amendment applies to the disposition of assets conveyed to the trust pursuant to a will even if the will was executed prior to the date of the amendment

At Settlorrsquos death trust assets including probate assets passing to the trust under Settlorrsquos will would go to University if as is the case here the trust amendment was valid If the amendment was invalid the trust assets would continue to be held in further trust because there is no violation of the common law Rule Against Perpetuities

Point One(a) (30) Settlor retained the right to amend the inter vivos trust despite her failure to expressly reserve this power

At issue here is whether a retained power of revocation includes the power to amend sometimes referred to as the power to modify The Restatement (Second) of Trusts sect 331 cmt g provides that if a settlor has a power to revoke that retained power ordinarily includes a power to modify (amend) as well Comment g also notes that the power to amend includes both a power to withdraw trust assets and a power to ldquomodify the terms of the trustrdquo The Uniform Trust Code which provides that a power to revoke includes the power to amend is consistent with this view

38

Trusts and Future Interests Analysis

UNIF TRUST CODE sect 602 accord RESTATEMENT (THIRD) OF TRUSTS sect 63 cmt The theory is that even though a power to amend was not expressly retained by a settlor the goal of amendment assuming the power was not included in the power to revoke could easily be achieved by first revoking the trust and then creating a new trust with the same terms contemplated by the amendment To require this would put form over substance

Thus by expressly retaining the power to revoke the trust Settlor retained a power to amend the inter vivos trust despite her failure to expressly reserve this power

[NOTE Under the common law a trust is irrevocable unless the settlor expressly retains a power to revoke the trust Conversely under the Uniform Trust Code a trust is revocable unless the terms of the trust expressly provide otherwise See UNIF TRUST CODE sect 602 The Trust Codersquos position on revocation follows the minority view in the United States and is inconsistent with prior Restatements of Trusts (see Restatement (Second) of Trusts sect 330) Here the trust is revocable because Settlor expressly retained a power of revocation

The Uniform Trust Code has been adopted in 24 jurisdictions Alabama Arizona Arkansas District of Columbia Florida Kansas Maine Michigan Missouri Nebraska New Hampshire New Mexico North Carolina North Dakota Ohio Oregon Pennsylvania South Carolina Tennessee Utah Vermont Virginia West Virginia and Wyoming]

Point One(b) (10) Settlorrsquos amendment of the trust was valid despite her failure to have her signature to the trust amendment witnessed

Neither the common law nor state statutes require a trust instrument or an amendment to a trust instrument to be executed in accordance with the formalities prescribed for execution of a will Indeed an inter vivos trust that does not involve real estate can be created orally Under the Uniform Trust Code the only requirements for creating a valid inter vivos trust are intent the specification of beneficiaries and the designation of a trustee See UNIF TRUST CODE sect 402 accord RESTATEMENT (THIRD) OF TRUSTS sect 13

Here the amendment meets the requirements of both the Uniform Trust Code and the common law Thus the fact that Settlorrsquos signature was not witnessed when she signed the amendment to the trust does not make the amendment invalid

Point Two (20) Under the Uniform Testamentary Additions to Trusts Act a revocable trust may be amended at any time prior to the settlorrsquos death and the amendment applies to probate assets poured into the trust at the settlorrsquos death pursuant to the settlorrsquos will even when the will was executed prior to the date of the amendment

Historically property owned by an individual at her death passed to the individualrsquos heirs or to beneficiaries designated in a will executed with the formalities (writing signing witnessing) prescribed by state law However when a will devises property to the trustee of an inter vivos trust then the provisions of the trustmdashwhich may not have been executed in accordance with the formalities required for willsmdasheffectively determine who will receive the property Because of this possibility some early cases held that if an inter vivos trust was not executed with the same formalities required for a valid will then the trust was ineffective to dispose of probate assets poured into the trust at the settlorrsquos death pursuant to the settlorrsquos will

This line of cases has been overturned by the Uniform Testamentary Additions to Trusts Act (the Act) now Uniform Probate Code sect 2-511 Under the Act adopted in almost all

39

Trusts and Future Interest Analysis

jurisdictions a testamentary bequest to the trustee of an inter vivos trust established by the testator during his or her lifetime is valid if the trust is in writing it is identified in the testatorrsquos will and the trust instrument was executed before concurrently with or after the execution of the will Id The Act further specifies that such a bequest is valid even if the trust is amendable or revocable and that a later amendment applies to assets passing to the trust by a previously executed will

Thus because the trust amendment is valid its terms apply to assets received by Bank from Settlorrsquos estate

Point Three (10) If the trust amendment was valid then the trust assets including assets passing to the trust under Settlorrsquos will should go to University

Under the trust amendment all trust assets (including the assets of Settlorrsquos probate estate poured into the trust) pass to University The facts provide no basis for failing to comply with Settlorrsquos stated intentions

Point Four (30) If the trust amendment was invalid trust assets including assets received pursuant to Settlorrsquos will should be held in accordance with the terms of the original trust instrument because those terms do not violate the Rule Against Perpetuities

Under the dispositive terms of the original trust instrument Settlor created successive income interests in her surviving children and grandchildren with a remainder interest in her great-grandchildren Because the trust was revocable the period during which the common law Rule Against Perpetuities requires that interests vest (ie 21 years plus lives in being) began to run from the date Settlor no longer had a power of revocation (here her death) not the date on which the trust was created See JESSE DUKEMINIER STANLEY J JOHANSON JAMES LINDGREN amp ROBERT SITKOFF WILLS TRUSTS AND ESTATES 678 (7th ed 2005)

Under the common law Rule Against Perpetuities Settlorrsquos trust is thus valid At the time of Settlorrsquos death she was survived by no children one granddaughter and no great-grandchildren Because Settlor cannot have more children after her death the only income beneficiary of the trust is Settlorrsquos surviving granddaughter This granddaughter is the only person who can produce great-grandchildren of Settlor thus all great-grandchildren must of necessity be born during the lifetime of Settlorrsquos only surviving granddaughter who is a life in being The granddaughterrsquos interest vested at Settlorrsquos death and the great-grandchildrenrsquos interest will vest at the death of the granddaughter There is no need to wait the additional 21 years permitted under the Rule Thus under the common law and the statute given in the facts the nonvested interest in the great-grandchildren is valid

[NOTE Both modern wait-and-see statutes and the Uniform Statutory Rule Against Perpetuities upon which the statute in the facts is modeled provide that before using either reform to validate an otherwise invalid nonvested interest one should first determine if the nonvested interest violates the common law Rule If it does not then there is no need to reform This proposition which is applicable in all MEE user jurisdictions that have not simply abrogated the rule is tested by this problem]

40

NEGOTIABLE INSTRUMENTS ANALYSIS (Negotiable Instruments III IV V)

ANALYSIS

Legal Problems

(1)(a) What rights does a person in possession of a note that has been indorsed in blank by the payee have against the maker of the note

(1)(b) Which defenses may the maker of a note raise against a person entitled to enforce it who is not a holder in due course but is a transferee from a holder in due course

(2) What rights does a person entitled to enforce a note have against an indorser who transferred it for consideration with no warranties

(3) What rights does a person entitled to enforce a note have against a previous holder who transferred it as a gift without indorsing it

DISCUSSION

Summary

The niece is a holder of the note and is thus a person entitled to enforce it The chef the issuer of the note is obligated to pay it to the niece as the person entitled to enforce it The niece is not subject to any defense or claim of the chef relating to the improper repair of the oven because the niece has the rights of a holder in due course When the buyer bought the note from the repairman the buyer became a holder in due course of the note and thus took it free of any personal defenses the chef had against the repairman Even though the niece is not herself a holder in due course of the note the niece succeeded to the buyerrsquos rights as holder in due course and thus took free of the chefrsquos personal defenses

Because the chef refused to pay the note the niece can recover from the repairman on the repairmanrsquos obligation as indorser The niece cannot recover on the note against the buyer however because the buyer did not indorse the note (and thus incurred no indorserrsquos obligation) and the buyer did not receive any consideration for transfer of the note to the niece (and therefore made no transfer warranty)

[NOTE Although Article 9 of the Uniform Commercial Code governs the sale of promissory notes (a point that might be correctly noted by examinees) that Article does not determine the answer to any of the questions posed]

Point One(a) (20) The niece is the holder of the note and thus may enforce it against the chef who is the issuer of the note

The chef is the maker of the note and thus its issuer See UCC sectsect 3-103 3-105 The issuer of a note is obligated to pay it in accordance with its terms to a ldquoperson entitled to enforcerdquo it UCC sect 3-412 The niece is a ldquoperson entitled to enforcerdquo the note This is because the niece is the holder of the note and a holder of a note is a person entitled to enforce it UCC sect 3-301 The niece is the holder of the note because (i) the repairmanrsquos signature on the back of the note not

41

Negotiable Instruments Analysis

accompanied by words indicating a person to whom the note was made payable was a ldquoblank indorsementrdquo which had the effect of making the note a bearer instrument (ii) anyone in possession of a bearer instrument is a holder of it and (iii) the niece is in possession of the note See UCC sectsect 1-201(b)(21)(A) 3-204 and 3-205 Accordingly the chef has an obligation to the niece to pay the note in accordance with its terms and the niece may enforce that obligation

Point One(b) (40) The niece is not a holder in due course of the note but because she is a transferee from the buyer who was a holder in due course she has the same enforcement rights as the buyer Because the buyer as a holder in due course would have been able to enforce the note against the chef without being subject to defenses or claims arising from the improper repair the niece has the same rights and will not be subject to the chefrsquos defenses or claims about the repair

As noted in Point One(a) the chef has an obligation to the niece to pay the note in accordance with its terms However except against a person with the rights of a holder in due course the chef can raise any defenses or claims in recoupment that he would have if the claim on the note were an ordinary contract claim UCC sect 3-305 Thus except against a holder in due course the chef would be able to raise the improper repair as a defense or a claim in recoupment (a claim in response to the niecersquos claim)

But claims in recoupment and most defenses cannot be raised against a person with the rights of a holder in due course Against a holder in due course the chef can raise only the four ldquorealrdquo defenses listed in UCC sect 3-305(a)(1) (infancy duress lack of legal capacity or illegality that nullifies the obligation of the obligor under other law fraud in the factum discharge in insolvency proceedings) none of which is present here

The niece is not a holder in due course because she did not take the note for value See UCC sectsect 3-302(a)(2)(i) (criteria for holder in due course status) and 3-303(a) (definition of ldquovaluerdquo) But this does not mean that the niece is subject to the chefrsquos claim arising out of the improper repair The buyer was a holder in due course of the note because he took the note for value ($9500) in good faith and without notice of any facts that would have alerted him to the chefrsquos defense against the repairman UCC sect 3-302(a)(2) As a holder in due course the buyer owned the note free of the chefrsquos claim because that claim did not constitute a ldquorealrdquo defense UCC sect 3-305(b) When the buyer gave the note to the niece this constituted a ldquotransferrdquo of the note See UCC sect 3-203(a) When a note is transferred the transferee receives ldquoany right of the transferor to enforce the instrument including any right as a holder in due courserdquo UCC sect 3-203(b) Under this rule (also known as the ldquoshelter principlerdquo) the buyer transferred his freedom from the chefrsquos defenses to the niece and the niece can enforce the note free of the chefrsquos defenses

Point Two (20) Because the chef dishonored the note the niece can recover from the repairman on the repairmanrsquos obligation as indorser

The chefrsquos refusal to pay the note constituted dishonor See UCC sect 3-502 The repairman as an indorser of the note (see Point One(a)) incurred the obligations of an indorser under UCC sect 3-415(a) When a note has been dishonored one of the obligations of an indorser is to pay the amount of the note to a person entitled to enforce it Therefore the repairman is liable for the amount of the note to the niece a person entitled to enforce the note (so long as the niece gives proper notice of dishonor to the repairman)

42

Negotiable Instruments Analysis

[NOTE Because the repairman indorsed the note without warranties there are no transfer warranties UCC sect 3-416 cmt 5]

Point Three (20) The niece cannot recover on the note against the buyer as either indorser or warrantor because the buyer did not indorse the note and did not receive consideration for transferring the note to the niece

The buyer did not indorse the note and therefore did not incur the obligation of an indorser to pay the note upon dishonor

The niece cannot recover from the buyer under a transfer warranty theory because transfer warranties are made only by a person ldquowho transfers an instrument for considerationrdquo Here the buyer gave the instrument to the niece as a gift So the buyer made no transfer warranty UCC sect 3-416(a) Therefore the niece cannot recover from the buyer on that theory

43

National Conference of Bar Examiners 302 South Bedford Street | Madison WI 53703-3622 Phone 608-280-8550 | Fax 608-280-8552 | TDD 608-661-1275

wwwncbexorg e-mail contactncbexorg

  • Contents
  • Preface
  • Description of the MEE
  • Instructions
  • February 2013 Questions
    • Real Property Question
    • Contracts Question
    • Constitutional Law Question
    • Secured Transactions Question
    • Federal Civil Procedure Question
    • Agency Question
    • Evidence Question
    • Trusts and Future Interests Question
    • Negotiable Instruments Question
      • February 2013 Analyses
        • Real Property Analysis
        • Contracts Analysis
        • Constitutional Law Analysis
        • Secured Transactions Analysis
        • Federal Civil Procedure Analysis
        • Agency Analysis
        • Evidence Analysis
        • Trusts and Future Interests Analysis
        • Negotiable Instruments Analysis
            • ltlt ASCII85EncodePages false AllowTransparency false AutoPositionEPSFiles true AutoRotatePages None Binding Left CalGrayProfile (Dot Gain 20) CalRGBProfile (sRGB IEC61966-21) CalCMYKProfile (US Web Coated 050SWOP051 v2) sRGBProfile (sRGB IEC61966-21) CannotEmbedFontPolicy Error CompatibilityLevel 14 CompressObjects Tags CompressPages true ConvertImagesToIndexed true PassThroughJPEGImages true CreateJobTicket false DefaultRenderingIntent Default DetectBlends true DetectCurves 00000 ColorConversionStrategy CMYK DoThumbnails false EmbedAllFonts true EmbedOpenType false ParseICCProfilesInComments true EmbedJobOptions true DSCReportingLevel 0 EmitDSCWarnings false EndPage -1 ImageMemory 1048576 LockDistillerParams false MaxSubsetPct 100 Optimize true OPM 1 ParseDSCComments true ParseDSCCommentsForDocInfo true PreserveCopyPage true PreserveDICMYKValues true PreserveEPSInfo true PreserveFlatness true PreserveHalftoneInfo false PreserveOPIComments true PreserveOverprintSettings true StartPage 1 SubsetFonts true TransferFunctionInfo Apply UCRandBGInfo Preserve UsePrologue false ColorSettingsFile () AlwaysEmbed [ true ] NeverEmbed [ true ] AntiAliasColorImages false CropColorImages true ColorImageMinResolution 300 ColorImageMinResolutionPolicy OK DownsampleColorImages true ColorImageDownsampleType Bicubic ColorImageResolution 300 ColorImageDepth -1 ColorImageMinDownsampleDepth 1 ColorImageDownsampleThreshold 150000 EncodeColorImages true ColorImageFilter DCTEncode AutoFilterColorImages true ColorImageAutoFilterStrategy JPEG ColorACSImageDict ltlt QFactor 015 HSamples [1 1 1 1] VSamples [1 1 1 1] gtgt ColorImageDict ltlt QFactor 015 HSamples [1 1 1 1] VSamples [1 1 1 1] gtgt JPEG2000ColorACSImageDict ltlt TileWidth 256 TileHeight 256 Quality 30 gtgt JPEG2000ColorImageDict ltlt TileWidth 256 TileHeight 256 Quality 30 gtgt AntiAliasGrayImages false CropGrayImages true GrayImageMinResolution 300 GrayImageMinResolutionPolicy OK DownsampleGrayImages true GrayImageDownsampleType Bicubic GrayImageResolution 300 GrayImageDepth -1 GrayImageMinDownsampleDepth 2 GrayImageDownsampleThreshold 150000 EncodeGrayImages true GrayImageFilter DCTEncode AutoFilterGrayImages true GrayImageAutoFilterStrategy JPEG GrayACSImageDict ltlt QFactor 015 HSamples [1 1 1 1] VSamples [1 1 1 1] gtgt GrayImageDict ltlt QFactor 015 HSamples [1 1 1 1] VSamples [1 1 1 1] gtgt JPEG2000GrayACSImageDict ltlt TileWidth 256 TileHeight 256 Quality 30 gtgt JPEG2000GrayImageDict ltlt TileWidth 256 TileHeight 256 Quality 30 gtgt AntiAliasMonoImages false CropMonoImages true MonoImageMinResolution 1200 MonoImageMinResolutionPolicy OK DownsampleMonoImages true MonoImageDownsampleType Bicubic MonoImageResolution 1200 MonoImageDepth -1 MonoImageDownsampleThreshold 150000 EncodeMonoImages true MonoImageFilter CCITTFaxEncode MonoImageDict ltlt K -1 gtgt AllowPSXObjects false CheckCompliance [ None ] PDFX1aCheck false PDFX3Check false PDFXCompliantPDFOnly false PDFXNoTrimBoxError true PDFXTrimBoxToMediaBoxOffset [ 000000 000000 000000 000000 ] PDFXSetBleedBoxToMediaBox true PDFXBleedBoxToTrimBoxOffset [ 000000 000000 000000 000000 ] PDFXOutputIntentProfile () PDFXOutputConditionIdentifier () PDFXOutputCondition () PDFXRegistryName () PDFXTrapped False CreateJDFFile false Description ltlt ARA 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 BGR ltFEFF04180437043f043e043b043704320430043904420435002004420435043704380020043d0430044104420440043e0439043a0438002c00200437043000200434043000200441044a0437043404300432043004420435002000410064006f00620065002000500044004600200434043e043a0443043c0435043d04420438002c0020043c0430043a04410438043c0430043b043d043e0020043f044004380433043e04340435043d04380020043704300020043204380441043e043a043e043a0430044704350441044204320435043d0020043f04350447043004420020043704300020043f044004350434043f0435044704300442043d04300020043f043e04340433043e0442043e0432043a0430002e002000200421044a04370434043004340435043d043804420435002000500044004600200434043e043a0443043c0435043d044204380020043c043e0433043004420020043404300020044104350020043e0442043204300440044f0442002004410020004100630072006f00620061007400200438002000410064006f00620065002000520065006100640065007200200035002e00300020043800200441043b0435043404320430044904380020043204350440044104380438002egt CHS ltFEFF4f7f75288fd94e9b8bbe5b9a521b5efa7684002000410064006f006200650020005000440046002065876863900275284e8e9ad88d2891cf76845370524d53705237300260a853ef4ee54f7f75280020004100630072006f0062006100740020548c002000410064006f00620065002000520065006100640065007200200035002e003000204ee553ca66f49ad87248672c676562535f00521b5efa768400200050004400460020658768633002gt CHT ltFEFF4f7f752890194e9b8a2d7f6e5efa7acb7684002000410064006f006200650020005000440046002065874ef69069752865bc9ad854c18cea76845370524d5370523786557406300260a853ef4ee54f7f75280020004100630072006f0062006100740020548c002000410064006f00620065002000520065006100640065007200200035002e003000204ee553ca66f49ad87248672c4f86958b555f5df25efa7acb76840020005000440046002065874ef63002gt CZE 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 DAN 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 DEU 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 ESP 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 ETI 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 FRA 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 GRE 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 HEB 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 HRV (Za stvaranje Adobe PDF dokumenata najpogodnijih za visokokvalitetni ispis prije tiskanja koristite ove postavke Stvoreni PDF dokumenti mogu se otvoriti Acrobat i Adobe Reader 50 i kasnijim verzijama) HUN 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 ITA 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 JPN ltFEFF9ad854c18cea306a30d730ea30d730ec30b951fa529b7528002000410064006f0062006500200050004400460020658766f8306e4f5c6210306b4f7f75283057307e305930023053306e8a2d5b9a30674f5c62103055308c305f0020005000440046002030d530a130a430eb306f3001004100630072006f0062006100740020304a30883073002000410064006f00620065002000520065006100640065007200200035002e003000204ee5964d3067958b304f30533068304c3067304d307e305930023053306e8a2d5b9a306b306f30d530a930f330c8306e57cb30818fbc307f304c5fc59808306730593002gt KOR ltFEFFc7740020c124c815c7440020c0acc6a9d558c5ec0020ace0d488c9c80020c2dcd5d80020c778c1c4c5d00020ac00c7a50020c801d569d55c002000410064006f0062006500200050004400460020bb38c11cb97c0020c791c131d569b2c8b2e4002e0020c774b807ac8c0020c791c131b41c00200050004400460020bb38c11cb2940020004100630072006f0062006100740020bc0f002000410064006f00620065002000520065006100640065007200200035002e00300020c774c0c1c5d0c11c0020c5f40020c2180020c788c2b5b2c8b2e4002egt LTH 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 LVI 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 NLD (Gebruik deze instellingen om Adobe PDF-documenten te maken die zijn geoptimaliseerd voor prepress-afdrukken van hoge kwaliteit De gemaakte PDF-documenten kunnen worden geopend met Acrobat en Adobe Reader 50 en hoger) NOR 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 POL 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 PTB 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 RUM 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 RUS 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 SKY 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 SLV 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 SUO 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 SVE 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 TUR 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 UKR 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 ENU (Use these settings to create Adobe PDF documents best suited for high-quality prepress printing Created PDF documents can be opened with Acrobat and Adobe Reader 50 and later) gtgt Namespace [ (Adobe) (Common) (10) ] OtherNamespaces [ ltlt AsReaderSpreads false CropImagesToFrames true ErrorControl WarnAndContinue FlattenerIgnoreSpreadOverrides false IncludeGuidesGrids false IncludeNonPrinting false IncludeSlug false Namespace [ (Adobe) (InDesign) (40) ] OmitPlacedBitmaps false OmitPlacedEPS false OmitPlacedPDF false SimulateOverprint Legacy gtgt ltlt AddBleedMarks false AddColorBars false AddCropMarks false AddPageInfo false AddRegMarks false ConvertColors ConvertToCMYK DestinationProfileName () DestinationProfileSelector DocumentCMYK Downsample16BitImages true FlattenerPreset ltlt PresetSelector MediumResolution gtgt FormElements false GenerateStructure false IncludeBookmarks false IncludeHyperlinks false IncludeInteractive false IncludeLayers false IncludeProfiles false MultimediaHandling UseObjectSettings Namespace [ (Adobe) (CreativeSuite) (20) ] PDFXOutputIntentProfileSelector DocumentCMYK PreserveEditing true UntaggedCMYKHandling LeaveUntagged UntaggedRGBHandling UseDocumentProfile UseDocumentBleed false gtgt ]gtgt setdistillerparamsltlt HWResolution [2400 2400] PageSize [612000 792000]gtgt setpagedevice

Page 13: February 2013 MEE Questions and Analyses

Agency Question

Assume that the rejection of the baskets and the burners and the failure to pay for the solar cells constitute breach of the relevant contracts

1 Is the owner liable to the basket manufacturer for breach of the contract for the aluminum baskets Is the agent liable Explain

2 Is the owner liable to the burner manufacturer for breach of the contract for the burners Is the agent liable Explain

3 Is the owner liable to the solar cell manufacturer for breach of the contract for the solar cells Is the agent liable Explain (Do not address liability based upon restitution or unjust enrichment)

9

EVIDENCE QUESTION _____

A woman who owns a motorized scooter brought her scooter to a mechanic for routine maintenance service As part of the maintenance service the mechanic inspected the braking system on the scooter As soon as the mechanic finished inspecting and servicing the scooter he sent the woman a text message to her cell phone that read ldquoJust finished your service When you pick up your scooter you need to schedule a follow-up brake repair Wersquoll order the partsrdquo

The woman read the mechanicrsquos text message and returned the next day to pick up her scooter As the woman was wheeling her scooter out of the shop she saw the mechanic working nearby and asked ldquoIs my scooter safe to ride for a whilerdquo The mechanic responded by giving her a thumbs-up The woman waved and rode away on the scooter

One week later while the woman was riding her scooter a pedestrian stepped off the curb into a crosswalk and the woman collided with him causing the pedestrian severe injuries The woman had not had the scooterrsquos brakes repaired before the accident

The pedestrian has sued the woman for damages for his injuries resulting from the accident The pedestrian has alleged that (1) the woman lost control of the scooter due to its defective brakes (2) the woman knew that the brakes needed repair and (3) it was negligent for the woman to ride the scooter knowing that its brakes needed to be repaired

The woman claims that the brakes on the scooter worked perfectly and that the accident happened because the pedestrian stepped into the crosswalk without looking and the woman had no time to stop The woman the pedestrian and the mechanic will testify at the upcoming trial

The pedestrian has proffered an authenticated copy of the mechanicrsquos text message to the woman

The woman plans to testify that she asked the mechanic ldquoIs my scooter safe to ride for a whilerdquo and that he gave her a thumbs-up in response

The evidence rules in this jurisdiction are identical to the Federal Rules of Evidence

Analyze whether each of these items of evidence is relevant and admissible at trial

1 The authenticated copy of the mechanicrsquos text message

2 The womanrsquos testimony that she asked the mechanic ldquoIs my scooter safe to ride for a whilerdquo and

3 The womanrsquos testimony describing the mechanicrsquos thumbs-up

10

TRUSTS AND FUTURE INTERESTS QUESTION

Ten years ago Settlor validly created an inter vivos trust and named Bank as trustee The trust instrument provided that Settlor would receive all of the trust income during her lifetime The trust instrument further provided that

Upon Settlorrsquos death the trust income shall be paid in equal shares to Settlorrsquos surviving children for their lives Upon the death of the last surviving child the trust income shall be paid in equal shares to Settlorrsquos then-living grandchildren for their lives Upon the death of the survivor of Settlorrsquos children and grandchildren the trust corpus shall be distributed in equal shares to Settlorrsquos then-living great-grandchildren

The trust instrument expressly specified that the trust was revocable but it was silent regarding whether Settlor could amend the trust instrument

Immediately after creating the trust Settlor validly executed a will leaving her entire estate to Bank as trustee of her inter vivos trust to ldquohold in accordance with the terms of the trustrdquo

Five years ago Settlor signed an amendment to the inter vivos trust The amendment changed the disposition of the remainder interest specifying that all trust assets ldquoshall be paid upon Settlorrsquos death to Universityrdquo Settlorrsquos signature on this amendment was not witnessed

A state statute provides that any trust interest that violates the common law Rule Against Perpetuities ldquois nonetheless valid if the nonvested interest in the trust actually vests or fails to vest either (a) within 21 years of lives in being at the creation of the nonvested interest or (b) within 90 years of its creationrdquo

Recently Settlor died leaving a probate estate of $200000 She was survived by no children one granddaughter (who would be Settlorrsquos only heir) and no great-grandchildren The granddaughter has consulted your law firm and has raised four questions regarding this trust

1 Was Settlorrsquos amendment of the inter vivos trust valid Explain

2 Assuming that the trust amendment was valid do its provisions apply to Settlorrsquos probate assets Explain

3 Assuming that the trust amendment was valid how should trust assets be distributed Explain

4 Assuming that the trust amendment was invalid how should trust assets be distributed Explain

11

NEGOTIABLE INSTRUMENTS QUESTION

A chef entered into a contract with a repairman pursuant to which the repairman agreed to repair the chefrsquos commercial oven for $10000 The repairman agreed to accept as payment a negotiable promissory note for $10000 payable two months after its issuance

After the repairman worked on the oven the chef gave him a $10000 note as payment for the work As agreed the note was signed by the chef as maker was payable to the order of the repairman was payable in two months and fulfilled all criteria for negotiability

The next day the repairman sold the note to a buyer for $9500 To effectuate the sale the repairman wrote ldquono warrantiesrdquo on the back of the note signed his name immediately below that and handed the note to the buyer The buyer bought the note in good faith and without knowledge of any facts relating to the work that the repairman had performed for the chef

Later the buyer gave the note to his niece as a gift To effectuate the gift the buyer handed the note to the niece but did not indorse it

Shortly thereafter the chef discovered that the repair work had been done improperly and the oven still did not function correctly The chef tried repeatedly to get the repairman to return to correct the repair work but the repairman ignored all the chefrsquos calls

On the notersquos due date the niece contacted the chef and demanded that he pay the amount of the note to her The chef refused and told the niece that he would not pay the note because the repairman did not properly repair the oven

1 What are the niecersquos rights against the chef Explain

2 What are the niecersquos rights against the repairman Explain

3 What are the niecersquos rights against the buyer Explain

12

February 2013 MEE

ANALYSES

Real Property Contracts

Constitutional Law Secured Transactions

Federal Civil Procedure Agency

Evidence Trusts and Future Interests

Negotiable Instruments

REAL PROPERTY ANALYSIS (Real Property ID1a 4 amp 5)

ANALYSIS

Legal Problems

(1) Does the tenant have a defense to the landlordrsquos action for unpaid rent based on constructive eviction

(2) Does the tenant have a defense to the landlordrsquos action for unpaid rent based on the tenantrsquos surrender of the premises

(3) What if anything may the landlord recover from the tenant for the period after the tenant vacated the building

DISCUSSION

Summary

Under the common law the tenant does not have a defense to the landlordrsquos action for unpaid rent based on constructive eviction Constructive eviction is based on the tenant proving that (1) the landlord breached a duty to the tenant (2) the breach caused a loss by the tenant of the substantial use and enjoyment of the premises (3) the tenant gave the landlord adequate notice and opportunity to repair and (4) the tenant vacated the leased premises Here there was no constructive eviction because although the tenant vacated and gave the landlord adequate notice the landlord breached no express or implied duty to the tenant to repair the premises

The tenant does not have a defense based on the landlordrsquos acceptance of his surrender of the premises a landlordrsquos retention of keys does not constitute an acceptance of the tenantrsquos surrender unless the landlord so intended and here the landlordrsquos statements to the tenant at the time of the surrender of the keys do not evidence the intent to accept the tenantrsquos surrender

Under the common law a landlord has no duty to mitigate damages but also cannot sue for rents due in the future Under this approach the landlord can sue only for past-due rents Using this approach on November 1 the landlord could recover all the rent past due (ie rent for September and October) but could not recover for rents due in the future However some courts have authorized recovery for future rent minus the fair market rental value of the premises It is thus possible that the landlord could recover damages equal to the amount of rent due from September 1 to the end of the six-year lease term ($180000) minus the propertyrsquos fair-market rental value over that same period

Point One (45) The tenant was not constructively evicted because the landlord had no duty to repair the commercial premises that were the subject of the lease

The landlord and the tenant entered into a term-of-years lease because the lease specified both a beginning and an ending date HERBERT HOVENKAMP amp SHELDON F KURTZ THE LAW OF

PROPERTY 256 (5th ed 2001) Although a term-of-years lease normally cannot be terminated by the tenant prior to the end of the term a tenant may terminate a term-of-years lease if the tenant

15

Real Property Analysis

is constructively evicted See id at 286ndash88 Typically as here a claim of constructive eviction is made as a defense to a landlordrsquos action for damages or unpaid rent

In order to establish a constructive eviction the tenant must prove that the landlord breached a duty to the tenant such as a duty to repair and that the landlordrsquos breach caused a loss of the substantial use and enjoyment of the premises The tenant must also show that he gave the landlord notice adequate to permit the landlord to meet his duty to the tenant and that the tenant vacated the leased premises Id see also JOHN G SPRANKLING UNDERSTANDING

PROPERTY LAW sect 1704 (2d ed 2007) Under the common law there was no implied duty on the part of a landlord to repair

leased premises such a duty arose only if expressly set forth in the lease SPRANKLING supra sect 1702[B] Here the written lease contained no term requiring the landlord to repair the air-conditioning Even if the conversation created a lease term that the building had air-conditioning that itself should not create a duty for the landlord to repair it

Over the past several decades courts have generally implied a duty to repair in residential leases either as part of a revised constructive eviction doctrine or based on an implied warranty of habitability JOSEPH W SINGER PROPERTY 469ndash70 (3d ed 2010) This shift has been justified based on the economic disparity between the typical landlord and tenant as well as the fact that residential tenants generally lack both the authority to authorize repairs to common areas of a building and the incentive to make repairs that will ultimately benefit the landlord

However courts have been more reluctant to imply a duty to repair in commercial leases a context in which the tenant is often a valuable business and in a better position to assess and make repairs than is the landlord But see eg Davidow v Inwood North Professional Group 747 SW2d 373 (Tex 1988) When courts have implied a duty to repair in a commercial lease it is typically when the repair has been mandated by public authorities and involves work so substantial that it would not ordinarily fall within the tenantrsquos common law repair duty andor the value of the repair would primarily inure to the landlordrsquos reversionary interest See Brown v Green 884 P2d 55 (Cal 1994) Eugene L Grant et al The Tenant as Terminator Constructive Eviction in Modern Commercial Leases 2 THE COMMERCIAL PROPERTY LEASE ch 15 (ABA 1997) Some courts have also permitted constructive eviction claims by commercial tenants of office buildings based on repairs required in common areas of the building See id Echo Consulting Services Inc v North Conway Bank 669 A2d 227 (NH 1995)

Here the tenant is the owner of a valuable manufacturing operation and is the exclusive occupant of the building the repair has not been mandated by public authorities and the repair is not structural To the contrary the repair involves a feature of the building of unusual importance in the tenantrsquos manufacturing operation and the tenant is likely far more knowledgeable than the landlord about the air-conditioning specifications necessary for the manufacture of the tenantrsquos product

Based on these facts it is unlikely that a court will find that the tenant in this case was constructively evicted Although the tenant can show that he gave adequate notice to the landlord of the air-conditioning malfunction and vacated the premises the lease was commercial and it did not contain any promises or covenants by the landlord except a covenant of quiet enjoyment a covenant of quiet enjoyment does not entail any repair obligations

[NOTE An examineersquos conclusion is less important than his or her demonstrated awareness of the elements of constructive eviction and the need to imply a repair duty for such a defense to be viable here Although the implied warranty of habitability is not available to this tenant Texas Minnesota and Massachusetts imply a warranty of suitability in commercial leases in limited circumstances and an examinee might argue that this warranty should apply

16

Real Property Analysis

here If an examinee concludes that this warranty applies he or she should discuss the other requirements for constructive eviction

If the examinee wrongly concludes that the first element for a constructive eviction has been met the examinee will then have to discuss the remaining three elements in order to conclude that the tenant can claim constructive eviction The tenant would have a strong argument that the second elementmdashsubstantial interference with the use and enjoyment of the premisesmdashalso is met As indicated above the landlord was aware that a functioning air-conditioning system was vital to the tenantrsquos manufacturing operations The facts further indicate that the system had failed three times in the past few months The landlord may try to argue that the malfunctions did not substantially interfere with the tenantrsquos use of the premises because the malfunctions caused the temperature to climb above 81 degrees for only a short period of timemdash 3 hours 6 hours and 10 hours respectivelymdashon each occasion The tenant will argue however that the landlord was aware that the tenantrsquos manufacturing operations could tolerate temperatures above 81 degrees for no more than 6 hours The final malfunction exceeded that limit destroying $150000 worth of the tenantrsquos products

The tenant would also have a strong argument that the third element is met notice and opportunity to cure The tenant notified the landlord of the problem immediately upon the systemrsquos first malfunction and did so again when it malfunctioned a second time and then a third time The landlord might argue that there was insufficient time to cure the problem because the system corrected itself within a few hours on the first and second times Although the malfunction lasted more than 10 hours the third time the landlord might argue that the time period was insufficient to get a repair person on the premises A court would be likely to find this argument unpersuasive however because the landlord could have attempted to correct the problem after the first and second malfunctions

Assuming that the landlord was given sufficient notice and opportunity to cure a court would be likely to conclude that the tenant also satisfied the final element of vacating the premises within a reasonable time The landlord might argue that the tenant remained in the premises for almost four months after the air conditioning first failed which would suggest that the problem was not so severe as to have constructively evicted the tenant The tenant will argue however that he gave the landlord three months to cure the problem after the first two malfunctions threatened (but did not actually harm) his operations The tenant then moved out shortly after the final malfunction caused temperatures to exceed the tolerance levels of his manufacturing operations]

Point Two (10) The landlord did not accept the tenantrsquos surrender of the lease

When a tenant wrongfully moves from leased premises with the intent to terminate the lease the landlord may either accept the tenantrsquos surrender of the premises and terminate the lease or hold the tenant to the terms of the lease See HOVENKAMP amp KURTZ supra at 295ndash96 Here the tenantrsquos only basis for the claim that the landlord accepted his surrender is the landlordrsquos retention of the keys Many courts have considered whether a landlordrsquos retention of keys delivered by a tenant constitutes acceptance of surrender The weight of the case law holds that retention of the keys alone does not constitute acceptance of surrender without other evidence showing that the landlord intended to accept the surrender See generally 49 AM JUR 2d Landlord and Tenant sect 213

Here the landlordrsquos note saying ldquoI repeat the air-conditioning is not my problem You have leased the building and you should fix itrdquo strongly suggests that the landlord did not intend

17

Some courts have rejected the no-mitigation-of-damages rule based on efficiency concerns and societyrsquos interest in assuring that resources remain in the stream of commerce rather than lying vacant see id at 464ndash65 and allow landlords to sue tenants who have wrongfully terminated a lease for damages equal to the difference between the unpaid rent due under the lease and the propertyrsquos fair market rental value Other courts have abandoned the no-recovery-for-future-rent rule These courts responding to the fact that a tenant may well disappear or be judgment-proof by the time a lease term is concluded have allowed a landlord to collect damages equal to the value of rent over the entire lease term minus the propertyrsquos fair rental value when a tenant has wrongfully terminated a lease and unequivocally shown an intention not to return to the premises or pay future rent Under this approach a landlord receives approximately the same amount he would have received were there a duty to mitigate damages See Sagamore Corp v Willcutt 180 A 464 (Conn 1935)

Real Property Analysis

to accept the tenantrsquos surrender The tenant might argue that the landlordrsquos failure to make a similar statement when the keys were sent to her a second time and she retained them evidences a change of heart However it is likely that a court would find that the landlordrsquos retention of the keys represented a decision to safeguard the keys not to accept the tenantrsquos surrender

[NOTE An examinee should receive credit for arguing the other way with a well-reasoned argument]

Point Three (45) Under the common law the landlord had no duty to mitigate damages Additionally a landlord was not entitled to recover unpaid rents due in the future but was only entitled to recover rents in arrears at the time of the commencement of the suit Applying the common law here the landlord could recover $5000 the amount of rents due at the commencement of the suit ($2500 for September and the same for October) Today some courts allow the landlord under certain circumstances to sue the tenant for damages (not rent) equal to the difference if any between the unpaid promised rent for the balance of the term (here $175000) and the propertyrsquos fair rental value for the balance of the term

Under the common law because a lease was viewed as a conveyance instead of a contract a landlord had no duty to mitigate damages resulting from a tenantrsquos wrongful termination of a lease A landlord could thus recover the full value of rents that were due and unpaid at the time of the suit However under the common law a landlord could not sue a tenant for rents due in the future because there was always a possibility that the tenant might pay the rent when it was due See SINGER supra at 462 Thus using the common law approach on November 1 the landlord could only recover the full value of the two monthsrsquo rent actually due and unpaid ie $5000 for September and October

Here because the tenant returned the keys to the landlord and said ldquoI will not be returning to the building or making further rent paymentsrdquo the landlord could establish abandonment and an intention not to return It is thus possible that the landlord might recover damages in the amount of $5000 (for the months of September and October) plus the present value of $175000 minus the fair market rental value of the property over the remaining months of the lease

18

CONTRACTS ANALYSIS ____ (Contracts II IVE)

ANALYSIS

Legal Problems

(1) What was the legal effect of the sailorrsquos October 31 letter to the builder

(2)(a) What was the legal effect of the builderrsquos November 25 response to the sailorrsquos October 31 letter

(2)(b) What was the legal effect of the sailorrsquos refusal to take and pay for the boat on December 15

DISCUSSION

Summary

This is a sale of goods governed by the Uniform Commercial Code Because the sailor had reasonable grounds for insecurity about the builderrsquos ability to deliver the boat in a timely manner when the sailor learned about the strike on October 31 the sailor was legally justified in sending the letter to the builder seeking adequate assurance of the builderrsquos performance pursuant to the contract The builderrsquos failure to provide such assurance within a reasonable time operated as a repudiation of the contract However the builder was free to retract the repudiation before the sailor either cancelled the contract or materially changed position in reliance on the builderrsquos repudiation The builder retracted the repudiation when he informed the sailor that the workers were back and that the boat would be delivered by the date stipulated in the partiesrsquo contract Because the sailor had taken no action in response to the original repudiation he no longer had the right to cancel the contract with the builder The sailorrsquos subsequent statement that ldquoour contract is overrdquo may have constituted repudiation by the sailor In any event when the sailor failed to perform on December 15 that constituted breach

Point One (35) Because the sailor had reasonable grounds for insecurity with respect to the builderrsquos performance the sailorrsquos letter to the builder was a justified demand seeking assurance of the builderrsquos performance under the contract failure of the builder to provide such assurance within a reasonable time constituted repudiation of the contract

The sailor was legally justified in sending the letter to the builder on October 31 Contract parties are entitled to expect due performance of contractual obligations and are permitted to take steps to protect that expectation UCC sect 2-609 states that ldquo[w]hen reasonable grounds for insecurity arise with respect to the performance of either party the other may in writing demand adequate assurance of due performance rdquo Here the sailor learned on October 31 that the builderrsquos workers were on strike This gave the sailor reasonable grounds for insecurity about the builderrsquos ability to complete performance on time and thus gave the sailor the right to seek adequate assurance from the builder Because the sailorrsquos demand for assurance was justified the builder was required to provide assurance that was adequate under the circumstances within a reasonable time (not to exceed 30 days) or be held to have repudiated the contract UCC sect 2-609(4)

19

Contracts Analysis

Point Two(a) (30) The builder did not within a reasonable time provide the sailor adequate assurance of due performance this failure to provide assurance constituted a repudiation of the contract

Because the sailor with legal justification (see Point One) demanded from the builder assurance of due performance the builderrsquos failure to provide such assurance within a reasonable time was a repudiation of their contract See UCC sect 2-609(4) (ldquoAfter receipt of a justified demand[] failure to provide within a reasonable time not exceeding thirty days assurance of due performance is a repudiation of the contractrdquo) On October 31 the sailor requested that the builder provide adequate assurance regarding the completion of the boat by December 15 The builder did not respond to the sailorrsquos letter until November 25mdashnearly a month later Even if that response had been given in a reasonable time it nonetheless did not provide assurance of due performance It simply stated ldquoIrsquom sorry about the strike but it is really out of my hands I hope we settle it soon so that we can get back to workrdquo Therefore the builderrsquos November 25 response did not provide adequate assurance in response to the sailorrsquos justified request Thus the builder had repudiated the contract

Point Two(b) (35) Although the builder repudiated the contract with the sailor the builder probably retracted that repudiation on December 3 and the sailor was no longer entitled to cancel their contract Thus the sailorrsquos failure to perform the sailorrsquos obligations under the contract constituted a breach

The builderrsquos failure to provide adequate assurance of performance constituted a repudiation of their contract (see UCC sect 2-609(4)) but the builder was free to retract that repudiation until the sailor cancelled the contract or materially changed his position or indicated by communication or action that the sailor considered the repudiation to be final See UCC sect 2-611(1) (ldquoUntil the repudiating partyrsquos next performance is due he can retract his repudiation unless the aggrieved party has since the repudiation cancelled or materially changed his position or otherwise indicated that he considers the repudiation finalrdquo)

Here the facts state that before the builderrsquos December 3 telephone call to the sailor the sailor did nothing in response to the builderrsquos repudiation such as contracting with a third party for a boat The builderrsquos December 3 call informing the sailor that the boat would be timely delivered probably constituted a retraction of the repudiation because it clearly indicated to the sailor that the builder would be able to perform UCC sect 2-611(2) Thus after being so informed the sailor did not have the right to treat their contract as cancelled UCC sect 2-611(3) Accordingly the sailorrsquos failure to perform the sailorrsquos obligations under the contract by taking the boat and paying for it constituted a breach of the contract

20

CONSTITUTIONAL LAW ANALYSIS (Constitutional Law IVA F2b amp e)

ANALYSIS

Legal Problems

(1) Does AutoCorsquos operation of a ldquocompany townrdquo result in its actions counting as those of the state for purposes of constitutional analysis

(2) Does the expulsion of a schoolchild for failure to recite the Pledge of Allegiance violate the First Amendment as applied through the Fourteenth Amendment

(3) Does the arrest of a pamphleteer in connection with violation of an anti-littering rule where the littering is done by the recipients of leaflets distributed by the pamphleteer violate the First Amendment as applied through the Fourteenth Amendment

DISCUSSION

Summary

The First Amendment as applied through the Fourteenth Amendment applies only to state action It does not typically govern private actors However courts have found state action where the private actor has exercised a ldquopublic functionrdquo such as running a privately owned ldquocompany townrdquo as AutoCo has done here Thus First Amendment protections apply By requiring the son to participate in a mandatory Pledge of Allegiance ceremony AutoCo has compelled the expression of political belief in violation of the First Amendment as applied through the Fourteenth Amendment The fatherrsquos arrest in connection with breaching the anti-litter rule also violated the First Amendment as applied through the Fourteenth Amendment Although state actors can regulate the incidental effects of speech on the public streets on a content-neutral basis this power is limited and cannot extend to punishing a distributor of literature because of littering by third parties

Point One (30) AutoCorsquos operation of a company town (including a school) makes it a state actor under the public function strand of the state action doctrine

The individual rights protections of the Constitution apply only where there is ldquostate actionrdquomdash either direct action by the government or some action by a private party that is fairly attributable to the government As a general rule the actions of a private company like AutoCo or of a private school like the school operated by AutoCo would not constitute state action and the protections of the Constitution (in this case the First Amendment) would not apply

However there are situations in which the actions of a private actor are attributed to the state One such situation is when the private actor undertakes a public function There are not many bright-line rules in the Supreme Courtrsquos state action doctrine but one of them is this Where a private actor undertakes a ldquopublic functionrdquo the Constitution applies to those actions Where a corporation operates a privately owned ldquocompany townrdquo that provides essential services typically provided by a state actor the public function doctrine applies and the Constitution

21

Constitutional Law Analysis

binds agents of the town as if they were agents of the government See eg Marsh v Alabama 326 US 501 (1946) Here AutoCo does more than own the town it provides security services fire protection sanitation services and a school Thus the actions of AutoCo constitute state action and are governed by the Fourteenth Amendment

Point Two (35) The sonrsquos expulsion for failure to recite the Pledge of Allegiance violates the First Amendment as applied through the Fourteenth Amendment as a compelled expression of political belief

As explained in Point One the First Amendment applies to the school as a state actor Although children in public schools (and in schools subject to the First Amendment like

the Oakwood school) have some First Amendment rights Tinker v Des Moines Independent Community School District 393 US 503 506 (1969) schools have greater leeway to regulate the speech of students and teachers than the state would have outside the school context Hazelwood School Dist v Kuhlmeier 484 US 260 (1988) Morse v Frederick 551 US 393 (2007) However the Supreme Court has long held that public schools may not force their students to participate in a flag salute ceremony when it offends the political or religious beliefs of the students or their families West Virginia Board of Educ v Barnette 319 US 624 (1943) (invalidating a mandatory public school flag salute ceremony) see also Wooley v Maynard 430 US 705 (1977) (invalidating compelled expression of political belief on state-issued license plates)

In this case the school requires its students to participate in a flag salute and Pledge of Allegiance ceremony and punishes them when they refuse to participate Pursuant to this policy the school has expelled the son This expulsion violates the First Amendment ban on compelled expression

Point Three (35) Because the father was distributing leaflets in a traditional public forum his trespass arrest violated the First Amendment as applied through the Fourteenth Amendment

As explained in Point One AutoCo is treated as a state actor Thus Oakwoodrsquos commercial district is treated as government-owned property for purposes of the First Amendment Thus the leafleting here is subject to the First Amendment because it is an expressive activity Schneider v State of New Jersey Town of Irvington 308 US 147 (1939) When expression takes place on government-owned property government regulation of the expression is assessed under the public forum doctrine Public streets and sidewalks have long been held to be the classic example of a ldquotraditional public forumrdquo open to the public for expression Hague v CIO 307 US 496 515ndash16 (1939) Because the father was distributing leaflets while standing on a street corner in the commercial district his expressive activity occurred in a traditional public forum

When a state tries to regulate expressive activity in a traditional public forum it is prohibited from doing so based on the expressive activityrsquos content unless its regulation is narrowly tailored to achieve a compelling governmental interest (ldquostrict scrutinyrdquo) In this case however AutoCo is regulating the fatherrsquos expressive activity on the ostensibly neutral ground that his expressive activity has produced litter and made the street unsightly When a state tries to regulate expressive activity without regard to its content intermediate scrutiny applies Under intermediate scrutiny the true purpose of the regulation may not be the suppression of ideas (if so then strict scrutiny applies) the regulation must be narrowly tailored to achieve a significant

22

Constitutional Law Analysis

governmental interest and it must leave open ample alternative channels for expressive activity Ward v Rock Against Racism 491 US 781 791 (1989)

Here the application of the ordinance to the father will fail for two reasons First the Supreme Court has held that the governmentrsquos interest in keeping the streets clean is insufficient to ban leafleting in the public streets as the government power to regulate with incidental effects on public sidewalk speech is very limited See eg Schneider 308 US at 162 (leafletinglittering) Second the regulation (a blanket ban on distribution that results in littering) is not narrowly tailored to protect expression A narrowly tailored alternative would be prosecution only of people who litter Moreover the effect of the littering rule is likely to be a ban on all leafleting thus eliminating an entire class of means of expression This raises the possibility that there are not ldquoample alternative channels of communicationrdquo open to the father as required under the Courtrsquos standard of review for content-neutral regulation of speech

[NOTE Some examinees might argue that this is a ldquotime place and mannerrdquo restriction and that AutoCo might have greater latitude to regulate the public sidewalks under this theory This argument is incorrect for two reasons First the Supreme Court has held that the power to regulate speakers through littering laws is very limited for the reasons given and in the cases cited above But more generally a ldquotime place and mannerrdquo restriction involves the shifting of speech from one time and place to another or to another manner here there is no shifting but a direct punishment for expressive activity (albeit one couched in content-neutral terms) In addition some examinees might read the ordinance to be in effect a total ban on leafleting since most leafleting will produce some litter Those examinees might note that the Court has required total bans on an entire mode of expression to satisfy strict scrutiny and analyze the fatherrsquos prosecution here accordingly See United States v Grace 461 US 171 177 (1983) (invalidating ban on display of signs on public sidewalks surrounding US Supreme Court ldquo[a]dditional restrictions such as an absolute prohibition on a particular type of expression will be upheld only if narrowly drawn to accomplish a compelling governmental interestrdquo)]

23

SECURED TRANSACTIONS ANALYSIS (Secured Transactions IID E IVA B C)

ANALYSIS

Legal Problems

(1) Is a purchase-money security interest in consumer goods perfected even though there has been no filing of a financing statement

(2) Does a person who buys consumer goods for personal use take those goods free of a prior perfected purchase-money security interest in the goods

(3) Does a person who receives consumer goods as a gift take those goods subject to a prior perfected security interest in them

DISCUSSION

Summary

The retailerrsquos security interest in the bicycles was perfected even though no financing statement was filed because it was a purchase-money security interest in consumer goods A purchase-money security interest in consumer goods is automatically perfected upon attachment

The buyer is not subject to the retailerrsquos security interest in the bicycle that the buyer bought from the man Because the bicycle was consumer goods in the hands of the man and the retailer never filed a financing statement covering the bicycle the retailerrsquos security interest is not effective against someone like the buyer who bought the bicycle for value without knowledge of the retailerrsquos security interest and for personal use

On the other hand the retailerrsquos security interest continues in the bicycle given to the friend because the friend did not give value for the bicycle or buy it in the ordinary course of business

Point One (35) The retailerrsquos security interest in the bicycles attached on June 1 Because this interest was a purchase-money security interest in consumer goods it was automatically perfected when it attached

The retailerrsquos security interest in the bicycles attached on June 1 when the man bought the bicycles (acquiring rights in the collateral) signed a security agreement containing a description of the collateral and received value from the retailer (by being given credit with which to purchase the bicycles) UCC sect 9-203(a) amp (b)

Despite the retailerrsquos failure to file a financing statement its security interest was perfected Pursuant to UCC sect 9-309(1) a security interest is automatically perfected upon attachment if the goods are ldquoconsumer goodsrdquo and the security interest is a ldquopurchase-money security interestrdquo

In this case the bicycles sold by the retailer to the man were consumer goods at the time of sale The bicycles were ldquogoodsrdquo because they were ldquomovable when a security interest

24

Secured Transactions Analysis

attachesrdquo UCC sect 9-102(a)(44) They were also consumer goods because they were ldquobought for use primarily for personal family or household purposesrdquo UCC sect 9-102(a)(23) The retailerrsquos security interest in these consumer goods was also a ldquopurchase-money security interestrdquo A purchase-money security interest is an interest that secures a debt that was incurred in order to ldquoenable the debtor to acquire rights in or the use of the collateralrdquo UCC sect 9-103(a) (b)(1) Here the man incurred an obligation to the retailer to purchase the bicycles so the security interest he gave the retailer to secure that obligation was a purchase-money security interest

Because the retailerrsquos security interest was a purchase-money security interest in consumer goods it was automatically perfected on June 1 when the interest attached to the bicycles

Point Two (35) The buyer took the bicycle free of the retailerrsquos security interest because (i) the retailer did not file a financing statement covering the bicycle (ii) the bicycle was ldquoconsumer goodsrdquo and (iii) the buyer bought the bicycle for value without knowledge of the retailerrsquos security interest and for personal use

A security interest continues in collateral even after a sale or other disposition of that collateral unless the creditor authorized the disposition ldquofree of the security interestrdquo or another Article 9 exception applies UCC sectsect 9-201(a) and 9-315(a)(1)

However a buyer of goods like the buyer here can take free of a prior security interest in those goods under certain circumstances See UCC sectsect 9-317(b) (buyers who give value and receive delivery of goods without knowledge of an unperfected security interest in the goods) and 9-320(a) amp (b) (buyer in ordinary course of business buyer of consumer goods in a consumer-to-consumer transaction who gives value) In this case the retailerrsquos security interest was perfected when the buyer purchased the bicycle so UCC sect 9-317(b) does not protect the buyer The buyer also is not a protected ldquobuyer in ordinary course of businessrdquo because he did not purchase from a person who is in the business of selling bicycles See UCC sect 1-201(b)(9)

The buyer can however qualify for the protection of UCC sect 9-320(b) That section provides that a buyer of goods from a person who used them for personal family or household purposes takes free of a perfected security interest in the goods if (1) the buyer had no knowledge of the security interest (2) the buyer gave value for the goods (3) the buyer purchased the goods primarily for personal family or household purposes and (4) the purchase occurred before the filing of a financing statement covering the goods

The buyer met all of these criteria The man used the bicycle for personal purposes The buyer purchased the bicycle from the man and the buyer had no knowledge of the retailerrsquos security interest The buyer gave value ($400) for the bicycle and he bought it ldquoprimarily for personal family or household purposesrdquo as he planned to use it for recreation which is a personal rather than a business use Finally no financing statement had been filed Therefore under UCC sect 9-320(b) the buyer took free of the retailerrsquos security interest

Point Three (30) The retailerrsquos security interest continues in the bicycle that the man gave to the friend Thus the retailer can recover the bicycle from the friend because the friend did not give value for the bicycle or buy it in the ordinary course of business

25

Secured Transactions Analysis

As noted in Point Two the retailer did not authorize the man to dispose of the bicycle Consequently the retailerrsquos security interest continued in the bicycle even after the man transferred ownership of the bicycle to the friend See UCC sectsect 9-201(a) and 9-315(a)(1) The retailerrsquos security interest in the bicycle will be effective against the friend unless some other provision of Article 9 allows the friend to take the bicycle free of that security interest

Unfortunately for the friend there is no Article 9 provision that allows him to take free of the retailerrsquos interest The friendrsquos basic problem is that he is not a buyer of the bicyclemdashhe received the bicycle as a gift and did not give value for it Thus the friend is not protected by any of the applicable exceptions See UCC sectsect 9-317(b) (protecting buyers who give value for goods subject to an unperfected security interest) 9-320(a) (protecting buyers in ordinary course of business) and 9-320(b) (protecting buyers of consumer goods who give value)

In short the retailerrsquos security interest continues in the bicycle that the man gave to the friend The friend took the bicycle subject to that security interest

26

FEDERAL CIVIL PROCEDURE ANALYSIS (Federal Civil Procedure VIE)

ANALYSIS

Legal Problems

(1) Does a judgment in a prior action preclude a nonparty from suing the same defendant on a closely related claim when the nonparty and the original plaintiff are in a family relationship

(2) Does a judgment rendered in an earlier action preclude a nonparty from litigating an issue that was actually decided in the first suit

(3) May a nonparty to an earlier action invoke the judgment in that action to preclude a party to the prior action from relitigating an issue that the party had a full and fair opportunity to litigate in the earlier action

DISCUSSION

Summary

Pursuant to the doctrines of claim preclusion (res judicata) and issue preclusion (collateral estoppel) a judgment is binding on the parties thereto In the absence of privity nonparties to a prior suit cannot be bound by a judgment rendered in their absence Thus in the absence of privity a nonparty to the first suit is not precluded from presenting her claim in a second suit even if it is factually related to the claims and defenses presented in the first suit nor is she bound by determinations of issues made in the first suit A family relationship without more does not support a finding of privity For this reason Mother as a nonparty is not bound by the judgment in the Son-Driver action She may bring her separate claim for damage to her car and she is not precluded from litigating the question of whether she was negligent in the maintenance of her car

Driver on the other hand could be precluded from relitigating the issue of her negligence pursuant to the doctrine of non-mutual issue preclusion (also called non-mutual offensive collateral estoppel) which allows a nonparty to a prior action to invoke issue preclusion to prevent a party to that prior action from relitigating determinations of issues made therein However Mother may be prevented from invoking non-mutual collateral estoppel in this case because she could easily have joined her claim in the prior action but did not do so

[NOTE Federal common law governs the preclusive effect of a judgment rendered by a federal court sitting in diversity See Semtek Intrsquol Inc v Lockheed Martin Corp 531 US 497 508 (2001) But the Semtek Court concluded that federal common law in this context incorporates the preclusion law of the state in which the rendering federal court sits (unless the state law is incompatible with federal interests) id at 508ndash09 Thus State Arsquos preclusion law determines the preclusive effect of the judgment rendered in Sonrsquos suit against Driver The problem says that State A preclusion law is identical to federal preclusion law so the following analysis utilizes general principles of preclusion drawn from Supreme Court case law (announcing federal preclusion rules) and the Restatement (Second) of Judgments]

27

Federal Civil Procedure Analysis

Point One (35) Under the doctrine of claim preclusion the judgment rendered in the first action does not preclude Mother a nonparty from suing Driver for the damage to her car because the judgment binds only parties or those in privity with them and Mother and Son are not in privity

Driver may contend that the doctrine of claim preclusion (res judicata) precludes Mother from presenting a claim arising from the same nucleus of facts that was presented in the first action brought by Son According to the doctrine of claim preclusion ldquowhen a court of competent jurisdiction has entered a final judgment on the merits of a cause of action the parties to the suit and their privies are thereafter bound lsquonot only as to every matter which was offered and received to sustain or defeat the claim or demand but as to any other admissible matter which might have been offered for that purposersquordquo Commissioner of Internal Revenue v Sunnen 333 US 591 597 (1948) (citation omitted)

However the doctrine of claim preclusion does not apply to Mother on the facts of this problem First Mother was not a party to the earlier case ldquoIt is a principle of general application in Anglo-American jurisprudence that one is not bound by a judgment in personam in a litigation in which he is not designated as a party or to which he has not been made a party by service of processrdquo Taylor v Sturgell 553 US 880 884 (2008) (citing Hansberry v Lee 311 US 32 40 (1940)) see also RESTATEMENT (SECOND) OF JUDGMENTS sect 34(3) (1982) This rule reflects our ldquodeep-rooted historic tradition that everyone should have his own day in courtrdquo Martin v Wilks 490 US 755 762 (1989) (citation omitted) (superseded by statute on other grounds) Since Mother was not a party to the first suit she is not bound by the judgment unless an exception to the general rule applies

Mother might be bound by the prior judgment if she were considered to have been sufficiently in privity with Son that Son represented her interests in that action ldquoA person who is not a party to an action but who is represented by a party is bound by and entitled to the benefits of a judgment as though he were a partyrdquo RESTATEMENT (SECOND) OF JUDGMENTS sect 41(1) But there is no suggestion in the facts of the problem that Son who is an adult purported to represent Motherrsquos interests in the first suit ldquo[C]lose family relationships are not sufficient by themselves to establish privity with the original suitrsquos party or to bind a nonparty to that suit by the judgment entered therein rdquo Cuauhtli v Chase Home Finance LLC 308 Fed Appx 772 773 (5th Cir 2009) (citation omitted) accord 18A CHARLES ALAN WRIGHT ET AL FEDERAL

PRACTICE AND PROCEDURE sect 4459 (2d ed 2002) In Taylor v Sturgell supra the Supreme Court identified other special circumstances in

which nonparties may be bound by a prior judgmentmdashwhen a nonparty consents to be bound when a nonparty is in a pre-existing substantive legal relationship with a party (such as preceding and succeeding property owners) when a nonparty assumed control of the prior litigation when a party seeks to relitigate through a proxy or where a special statutory scheme seeks to foreclose successive litigation by nonparties See Taylor 553 US at 893ndash95 None of these circumstances exists here

Because Mother was not a party to the first suit and is not in privity with Son who is an adult the judgment in the first action does not preclude her from bringing her own claim against Driver

Point Two (35) Under the doctrine of issue preclusion the judgment rendered in the first action does not preclude Mother a nonparty from litigating the issue of her negligence in maintaining her carrsquos

28

Federal Civil Procedure Analysis

brake lights because the judgment binds only parties or those in privity with them and Mother and Son are not in privity

By its affirmative response to a special interrogatory the jury in the first action expressly concluded that ldquoMother negligently failed to ensure that the brake lights on her car were in proper working orderrdquo Driver may attempt to invoke the doctrine of issue preclusion to preclude Mother from relitigating this issue in the second action

[I]ssue preclusion arises in a second action on the basis of a prior decision when the same lsquoissuersquo is involved in both actions the issue was lsquoactually litigatedrsquo in the first action after a full and fair opportunity for litigation the issue was lsquoactually decidedrsquo in the first action by a disposition that is sufficiently lsquofinalrsquo lsquoon the meritsrsquo and lsquovalidrsquo it was necessary to decide the issue in disposing of the first action and the later litigation is between the same parties or involves nonparties that are subject to the binding effect or benefit of the first action Once these requirements are met issue preclusion is available not only to defend against a demand for relief but also as offensive support for a demand for relief Issue preclusion moreover is available whether or not the second action involves a new claim or cause of action

18 CHARLES ALAN WRIGHT ET AL FEDERAL PRACTICE AND PROCEDURE sect 4416 at 392ndash93 (2d ed) see also RESTATEMENT (SECOND) OF JUDGMENTS sect 27 (1982)

Here several of the elements necessary for issue preclusion are present The same issue is involved in both actionsmdashthe issue of Motherrsquos negligence in failing to maintain the brake lights on her car That issue was actually litigated in the first action and decided by the jury There is nothing to suggest anything less than a full and fair opportunity to litigate The judgment disposing of the issue was final

Nevertheless the judgment will not preclude Mother from relitigating the issue for two reasons First Mother was not a party to the first action and as explained above Mother and Son are not in privity Therefore she cannot be denied an opportunity to litigate the issue of her negligence Second it does not appear that the juryrsquos decision as to Motherrsquos negligence was necessary to the prior judgment against Driver Nothing suggests that the finding on Motherrsquos negligence had any bearing on the outcome of the first action

Point Three (30) Under the doctrine of non-mutual issue preclusion the judgment rendered in the first action might preclude Driver from relitigating the issue of her negligence However Driver has a strong argument that such a result would be inconsistent with the policy against offensive use of non-mutual estoppel when the non-party plaintiff easily could have joined as a plaintiff in the first action

Because Son already convinced the jury in the first action that ldquoDriver was negligent in the operation of her vehiclerdquo Mother may wish to invoke the doctrine of non-mutual issue preclusion to prevent Driver from relitigating the question of her negligence As noted above ldquoissue preclusion arises in a second action on the basis of a prior decision when the same lsquoissuersquo is involved in both actions the issue was lsquoactually litigatedrsquo in the first action after a full and fair opportunity for litigation the issue was lsquoactually decidedrsquo in the first action by a disposition that is sufficiently lsquofinalrsquo lsquoon the meritsrsquo and lsquovalidrsquo it was necessary to decide the issue in disposing of the first action rdquo 18 CHARLES ALAN WRIGHT ET AL FEDERAL PRACTICE AND

PROCEDURE sect 4416 at 392 (2d ed) see also RESTATEMENT (SECOND) OF JUDGMENTS sect 27

29

Federal Civil Procedure Analysis

Here these basic requirements for issue preclusion are met First the same issue is involved in both suits whether Driver was negligent in the operation of her car Second this issue was actually litigated and decided in the first action the jury answered a special interrogatory raising this very question There is nothing to suggest that Driver lacked a full and fair opportunity to litigate the issue Since a judgment was rendered against Driver for the injuries Son sustained as a result of Driverrsquos negligence resolution of the issue was necessary to dispose of the first action Driver was a party to the first action so she may be bound by the judgment

[NOTE Traditionally issue preclusion required mutualitymdashboth the party asserting issue preclusion and the party against whom issue preclusion was asserted were bound by the prior judgment Under the traditional mutuality rule Mother could not assert issue preclusion against Driver because Mother would not be bound by the judgment if Driver sought to rely on it See Point One There is no mutuality between Mother and Driver with respect to the prior judgment

This traditional mutuality requirement has been abandoned in most jurisdictions The Supreme Court rejected a strict mutuality requirement in Blonder-Tongue Laboratories Inc v University of Illinois Foundation 402 US 313 (1971) (non-mutual defensive collateral estoppel used by a defendant to preclude a plaintiff from relitigating a claim the plaintiff previously litigated) and Parklane Hosiery Co v Shore 439 US 322 (1979) (non-mutual offensive collateral estoppel used by a plaintiff to preclude a defendant from relitigating a claim the defendant previously litigated) In Parklane Hosiery the Court concluded (as a matter of federal preclusion law) that trial courts should have ldquobroad discretionrdquo to determine whether or not to permit a plaintiff to invoke non-mutual issue preclusion ldquoThe general rule should be that in cases where a plaintiff could easily have joined in the earlier action or where the application of offensive estoppel would be unfair to a defendant a trial judge should not allow the use of offensive collateral estoppelrdquo Id at 331

The Parklane Hosiery decision identified a number of circumstances that might make it unfair to allow a plaintiff to invoke non-mutual issue preclusion (non-mutual offensive collateral estoppel in the traditional terminology) against a defendant In particular the Parklane Hosiery court suggested that issue preclusion may not be appropriate if the plaintiff in the second action ldquocould easily have joined in the earlier actionrdquo Id Prohibiting plaintiffs from using non-mutual estoppel under such circumstances would promote judicial efficiency by encouraging plaintiffs to join the prior action It would also discourage plaintiffs from staying out of prior litigation in order to secure in effect two bites at the apple using the prior litigation offensively if the defendant loses and forcing the defendant to litigate a second time if the defendant wins the prior action

An exceptional exam answer might therefore argue that non-mutual issue preclusion should be denied on these facts Son and Mother both reside in State A since they are related they know each other well and Son was driving Motherrsquos car when the accident occurred They could have sued together and Rule 20 of the Federal Rules of Civil Procedure would have authorized joinder of their claims because those claims arose from the same transaction or occurrence and raised a common question of law or fact FED R CIV P 20(a) The facts do not suggest that Mother had any reason not to join Sonrsquos suit other than a desire to see how Sonrsquos action concluded before bringing her own claim Cf Nations v Sun Oil Co (Del) 695 F2d 933 938 (5th Cir 1983) (concluding that plaintiff ldquowas entitled to await the development of his injuries and their predictable consequencesrdquo) Because it appears that Mother may be a ldquowait-and-seerdquo plaintiff who could easily have joined the original action a trial court might disallow as a matter of discretion her use of non-mutual issue preclusion]

30

AGENCY ANALYSIS __________ (Agency I II)

ANALYSIS

Legal Problems

(1) Is the principal or the agent or both liable on contracts with a third party when the principal is an ldquoundisclosed principalrdquo

(2) Is the principal or the agent or both liable on contracts with a third party when the principal is ldquopartially disclosedrdquo or an ldquounidentified principalrdquo

(3) Is the principal or the agent or both liable on contracts with a third party for the purchase of goods when the agent exceeded his authority but the principal nonetheless accepts the goods

DISCUSSION

Summary

The agent but not the owner is liable to the basket manufacturer because the owner is an undisclosed principal and the agent acted without actual or apparent authority Both the agent and the owner however are liable on the burner contract because the owner is an unidentified principal and the agent had apparent authority to enter into that contract With respect to the solar cells contract whether the owner is liable depends upon whether a court would follow the Second or Third Restatement of Agency which take different positions on the effect of the ratification of a contract by an undisclosed principal Under either the agent would also be liable on the contract as he was a party to the contract

[NOTE The contracts that are the subject of this question are contracts for the sale of goods and therefore are governed by Article 2 of the Uniform Commercial Code Article 2 however does not contain agency rules Accordingly common law concepts of agency are applicable UCC sect 1-103(b)]

Point One (35) The agent but not the owner is liable to the basket manufacturer The agent had no actual authority to enter into the contract to buy aluminum baskets and because the owner was an undisclosed principal the manufacturer had no reason to believe that the agent had apparent authority Furthermore the manufacturer had no reason to believe that the agent was not contracting for his own benefit

An agent acting on behalf of a principal can bind the principal to contracts if the agent has either actual or apparent authority An agent has actual authority when contracting on behalf of his principal if he ldquoreasonably believes in accordance with the principalrsquos manifestations to the agent that the principal wishes the agent so to actrdquo RESTATEMENT (THIRD) OF AGENCY sect 201 (2006) Here the agent was told to buy only wicker baskets not aluminum baskets Thus when he contracted with the basket manufacturer to buy aluminum baskets he had no actual authority to do so

31

Agency Analysis

An agent acts with apparent authority ldquowhen a third party [with whom the agent acts] reasonably believes the actor has authority to act on behalf of the principal and that belief is traceable to the principalrsquos manifestationsrdquo Id sect 203 Here the owner notified basket manufacturers that she or her agent might contact them to purchase baskets but that notification did not specifically name the agent or any other person as the ownerrsquos agent Furthermore the basket manufacturer had no prior dealings with the agent or the owner or any reason to think that the agent was acting for the benefit of anyone but himself Thus there is no basis to conclude that the basket manufacturer thought that the agent had apparent authority to act for the owner

Generally when an agent acts on behalf of an undisclosed principal and the agent lacks authority to enter into the contract the agent is liable on the contract as a party to the contract but the principal is not liable This rule is consistent with the third partyrsquos expectations ldquoThe third party expected the agent to be a party to the contract because the agent presented the deal as if he were acting for himself Moreover if the third party is unaware of the principalrsquos existence the third party must be relying on the agentrsquos solvency and reliability when entering into the contractrdquo See ROBERT W HAMILTON JONATHAN R MACEY amp DOUGLAS K MOLL CORPORATIONS INCLUDING PARTNERSHIPS AND LIMITED LIABILITY COMPANIES 34 (11th ed 2010) See also RESTATEMENT (THIRD) OF AGENCY sect 603 cmt c Furthermore because the third party has no idea that the agent is acting or is seemingly acting on behalf of another there is no reason to believe that the third party would be expecting an undisclosed principal to be liable on the contract Id

Point Two (35) Because the owner is an unidentified (as opposed to undisclosed) principal both she and the agent (as a party to the contract) probably are liable on the contract with the burner manufacturer

When the agent contracted with the burner manufacturer he did not have actual authority to do so as the owner had expressly restricted the agentrsquos authority to purchase only burners with ldquowhisper technologyrdquo See Point One However the agent may have had apparent authority to buy burners without whisper technology

An agent acts with apparent authority ldquowhen a third party [with whom the agent acts] reasonably believes the actor has authority to act on behalf of the principal and that belief is traceable to the principalrsquos manifestationsrdquo RESTATEMENT (THIRD) OF AGENCY sect 203 (2006) The owner indicated that an agent might contact the burner manufacturer The notice contained no restriction regarding the type of burners that the agent was authorized to purchase The facts indicate that burner manufacturers regularly receive such notices

Although the agent told the burner manufacturer that he represented a well-known hot-air balloon operator he did not disclose the ownerrsquos name Thus the owner was a partially disclosed or unidentified principal See RESTATEMENT (SECOND) OF AGENCY sect 4(2) (1958) (using term ldquopartially disclosed principalrdquo) RESTATEMENT (THIRD) OF AGENCY sect 104(2)(c) (2006) (using term ldquounidentified principalrdquo) An agent for a partially disclosed principal may have apparent authority RESTATEMENT (SECOND) OF AGENCY sect 159 cmt e (1958) Based upon (1) the notice sent by the owner (2) the agentrsquos revelation that he was acting as an agent and (3) the fact that burner manufacturers regularly receive such notices and sell to agents the manufacturer may argue that it reasonably and actually believed that the agent was authorized to purchase burners without whisper technology The manufacturer may also argue that because the agent revealed that he was an agent his listing of the ownerrsquos address as the delivery address connects the agent to the notice given by the owner Arguably this distinguishes the burner contract from the basket

32

Agency Analysis

contract Here there is a strong case to support the conclusion that the agent had apparent authority if he did then the owner is liable to the burner manufacturer

The agent also is liable as a party to the contract because he did not fully disclose his agency relationship Although he told the burner manufacturer that he represented a well-known hot-air balloon operator he did not disclose the ownerrsquos name Generally even an authorized agent of a partially disclosed or unidentified principal is liable as a party to a contract with a third person RESTATEMENT (SECOND) OF AGENCY sect 321 (1958) (ldquounless otherwise agreedrdquo) RESTATEMENT (THIRD) OF AGENCY sect 602(2) (2006) (ldquounless the agent and the third party agree otherwiserdquo)

Point Three (30) Under the Second Restatement of Agency the owner is not liable on the contract for solar cells because the agent did not have actual or apparent authority and the owner as an undisclosed principal cannot ratify the contract Under the Third Restatement the owner could be liable as she ratified the contract Under either Restatement the agent is liable as a party to the contract

The owner is not liable to the solar cell manufacturer for breach of the contract for the solar cells because the agent had no actual or apparent authority to purchase solar cells on the ownerrsquos behalf and the owner under the Second Restatement of Agency did not ratify the contract with knowledge of the material facts Thus she is not liable as a ratifier of the contract

The facts state that the agent had authority to purchase only propane fuel tanks In addition he had no apparent authority to purchase solar cells The owner made no manifestations to the solar cell manufacturer that would lead a reasonable person in the manufacturerrsquos position to believe that the agent had the authority to bind the owner to a contract to purchase solar cells In fact the agent made no manifestations at all to the solar cell manufacturer Unlike with the basket manufacturer and the burner manufacturer the owner did not notify the manufacturer of solar cells that an agent might contact it to purchase solar cells In addition the solar cells were delivered to the agent and not to the ownerrsquos address In sum the manufacturer was unaware of any relationship between the owner and the agent As to the solar cell manufacturer the owner is an undisclosed principal There can be no apparent authority in the case of an undisclosed principal because there are no manifestations from the principal to the third person See RESTATEMENT (SECOND) OF AGENCY sect 8 cmt a (1958) (ldquothere can be no apparent authority created by an undisclosed principalrdquo) RESTATEMENT (THIRD) OF AGENCY sect 203 cmt f (2006) (ldquoapparent authority is not present when a third party believes that an interaction is with an actor who is a principalrdquo)

The owner also did not ratify the contract Although the owner used the solar cells generally a principal cannot ratify an unauthorized transaction with a third person ldquounless the one acting purported to be acting for the ratifierrdquo RESTATEMENT (SECOND) OF AGENCY sect 85(1) (1958)

The result differs under the Third Restatement which expressly rejects the Second Restatement on this issue The Restatement (Third) of Agency sect 403 (2006) states ldquoA person may ratify an act if the actor acted or purported to act as an agent on the personrsquos behalfrdquo According to comment b ldquoan undisclosed principal may ratify an agentrsquos unauthorized actrdquo Under the Restatement (Third) of Agency rule the owner probably ratified the transaction The agent clearly acted on the ownerrsquos behalf and in addition the ownerrsquos conduct in using the solar cells ldquojustifies a reasonable assumption that [she] is manifesting assent that the act shall affect [her] legal relationsrdquo See id sect 401(2)

33

Agency Analysis

The agent also is liable to the solar cell manufacturer for breach of the contract for the solar cells because he is a party to the contract The facts indicate that the agent never told the solar cell manufacturer that he represented the owner or any other principal Consequently even if the agent were authorized (which as discussed above he is not) he would be liable as a party to the contract See RESTATEMENT (SECOND) OF AGENCY sect 322 (1958) RESTATEMENT (THIRD) OF AGENCY sect 603(2) (2006) Here he has no authority or apparent authority and is liable as a party to the contract

The agent would also be liable under the Third Restatement Under Restatement (Third) of Agency sect 402(1) (2006) ratification generally relates back and the transaction is treated as if it were authorized at the time of the transaction However this does not relieve the agent of an undisclosed principal who ratifies an unauthorized transaction of liability under the ratified contract See id sect 603(2) (authorized agent for undisclosed principal is a party to the contract) and sect 403 cmt b (ldquoAn undisclosed principalrsquos ratification does not eliminate the agentrsquos liability to the third party on the transaction rdquo)

[NOTE An examinee may discuss the concept of inherent agency power This concept is recognized by the Restatement (Second) of Agency sect 8 A (1958) but the concept is not used in the Restatement (Third) of Agency (2006) Here there are no facts to support that the agent had inherent authority

As to contracts with agents for partially disclosed principals (eg the contract for the burners) the basic question is whether the acts done ldquousually accompany or are incidental to transactions which the agent is authorized to conductrdquo RESTATEMENT (SECOND) OF AGENCY

sect 161 (1958) If so the principal is bound if the other party ldquoreasonably believes that the agent is authorized to do them and has no notice that he is not so authorizedrdquo Id The purchase of burners without whisper technology was not authorized nor was it incidental to an authorized transaction Therefore there should not be inherent agency power

As to contracts on behalf of undisclosed principals (eg the other two contracts) the basic question is whether the acts done are usual or necessary in the transactions the agent is authorized to transact RESTATEMENT (SECOND) OF AGENCY sect 194 (1958) The other two contracts seem fundamentally different from the authorized transactions Therefore there should not be inherent agency power

Only minimal credit should be given for discussion of inherent agency power]

34

EVIDENCE ANALYSIS _____ (Evidence IIA VA B E F J K)

ANALYSIS

Legal Problems

(1) Is the authenticated copy of the mechanicrsquos text message relevant and admissible

(2) Is the womanrsquos question ldquoIs my scooter safe to drive for a whilerdquo relevant and admissible

(3) Is the womanrsquos testimony describing the mechanicrsquos thumbs-up relevant and admissible

DISCUSSION

Summary

The mechanicrsquos text message to the woman is relevant to whether (1) the woman lost control of the scooter due to its defective brakes (2) the woman knew that the brakes needed repair and (3) it was negligent for the woman to drive the scooter knowing that its brakes needed repair

The mechanicrsquos text message is hearsay if it is offered by the pedestrian to prove that the scooterrsquos brakes needed repair However it fits the hearsay exception for present sense impressions and probably also fits the exception for business records The mechanicrsquos text message is not hearsay if it is instead offered by the pedestrian to prove the womanrsquos state of mind (ie that she had notice that her brakes needed repair)

The womanrsquos question to the mechanic and his response are also relevant to whether the brakes caused the accident and whether the woman was negligent The question is not hearsay because the woman did not make an assertion

The mechanicrsquos thumbs-up response is nonverbal conduct intended by the mechanic as an assertion and is therefore an out-of-court statement If the woman offers the mechanicrsquos statement to prove that the scooter was actually safe to ride the womanrsquos testimony about the statement is hearsay

However the mechanicrsquos statement is not hearsay if it is offered by the woman to prove her state of mind Therefore the womanrsquos question and the mechanicrsquos response are admissible to prove the womanrsquos state of mind

Point One(a) (20) The mechanicrsquos text message to the woman should be admitted because it is relevant

Evidence is relevant if it has ldquoany tendency to make a fact more or less probable than it would be without the evidencerdquo FED R EVID 401 ldquoRelevant evidence is admissiblerdquo unless it is inadmissible pursuant to some other rule FED R EVID 402

The mechanicrsquos text message to the woman ldquoWhen you pick up your scooter you need to schedule a follow-up brake repair Wersquoll order the partsrdquo is relevant for two reasons First this evidence has some tendency to make it more probable that the brakes malfunctioned and

35

Evidence Analysis

caused the accident Second it has some tendency to make it more probable that the woman was negligent in riding her scooter after being told by the mechanic that it required further repair

Point One(b) (30) The mechanicrsquos text message fits either the hearsay exception for present sense impressions or the exception for business records or it is admissible non-hearsay

The mechanicrsquos text message is a statement under Rule 801(a) because it is ldquoa written assertionrdquo FED R EVID 801(a) The text message is hearsay if the pedestrian offers it to prove the ldquotruth of the matter asserted in the statementrdquo (ie that the scooterrsquos brakes required repair) which resulted in the woman losing control of the scooter and causing the accident FED R EVID 801(c)

However the mechanicrsquos text message fits the hearsay exception for ldquopresent sense impressionsrdquo under Rule 803(1) because it is ldquo[a] statement describing or explaining an event or condition made while or immediately after the declarant perceived itrdquo FED R EVID 803(1) Here the mechanicrsquos text message described the condition of the scooter immediately after he perceived it during the maintenance service

The mechanic is a person with knowledge of the condition of the scooter so if text messages regarding repairs were made and kept by the mechanic in the ordinary course of business this text message also fits the business records exception Under Rule 803(6) a business record is a record of an act ldquomade at or near the time by someone with knowledgerdquo and ldquothe record was kept in the course of a regularly conducted activity of a businessrdquo and ldquomaking the record was a regular practice of that activityrdquo FED R EVID 803(6)

However the text message is not hearsay if it is instead offered to prove that the woman was negligent because she rode her scooter after the mechanic told her it required repair If offered for this purpose it would not be offered for the truth of the matter asserted in the statement but to show the womanrsquos belief about the condition of the scooter (her state of mind)

Point Two (10) The womanrsquos question to the mechanic should be admitted because it is not hearsay

The womanrsquos question to the mechanic is relevant because along with the mechanicrsquos thumbs-up response (see Point Three) it has some tendency to make it more probable that the woman was not negligent andor that the scooter brakes did not malfunction and cause the accident FED R EVID 401 The womanrsquos question does not raise hearsay concerns because it is not an assertion

Hearsay is defined under Rule 801(a) as ldquoan oral assertion written assertion or nonverbal conductrdquo Although ldquoassertionrdquo is not further defined ldquoa favorite [definition] of writers in the [evidence] field for at least a century and a half [is that] the word simply means to say that something is so eg that an event happened or a condition existedrdquo 2 MCCORMICK ON

EVIDENCE sect 246 (6th ed 2006) Under this definition the womanrsquos question is not hearsay because it is not an assertion

Point Three(a) (20) The mechanicrsquos thumbs-up to the woman is a nonverbal assertion that is relevant and the womanrsquos testimony about that response is admissible

36

Evidence Analysis

Hearsay is defined under Rule 801(c) as a ldquostatementrdquo that is ldquoa personrsquos oral assertion written assertion or nonverbal conduct if the person intended it as an assertionrdquo FED R EVID 801(a) Here when the mechanic responded to the womanrsquos question (ldquoIs my scooter safe to ride for a whilerdquo) with a thumbs-up gesture the facts suggest that he intended his nonverbal conduct as an assertion that in his opinion the scooter was safe to ride

The mechanicrsquos assertion is relevant and admissible to prove that the woman was not negligent because the evidence makes it more probable that at the time of the accident she believed that the scooter was safe to ride despite the fact that the brakes required repair FED R EVID 401 Admission of the womanrsquos description of the mechanicrsquos thumbs-up for this purpose does not raise hearsay concerns because the evidence would not be offered for the truth of the matter asserted but to show the womanrsquos belief about the condition of the scooter (her state of mind)

Point Three(b) (20) The mechanicrsquos thumbs-up is relevant to determine whether the scooterrsquos brakes malfunctioned causing the accident but if offered for this purpose it is also hearsay

The mechanicrsquos nonverbal assertion is relevant to the determination of whether the scooterrsquos brakes malfunctioned causing the accident However if offered to prove the ldquotruth of the matter asserted in the statementrdquo (ie that the scooter was safe to ride for a while) it is hearsay that does not fit any hearsay exception

37

TRUSTS AND FUTURE INTERESTS ANALYSIS ____________________ (Trusts and Future Interests IC1 amp 4 G IIF)

ANALYSIS

Legal Problems

(1)(a) Was the revocable trust amendable

(1)(b) If the trust was amendable must the amendment have been executed in accordance with the state Statute of Wills in order to be valid

(2) If the trust amendment was valid does the amendment apply to the probate estate assets passing to the trust pursuant to Settlorrsquos will

(3) If the trust amendment was valid should the trust property be distributed to University

(4) If the trust amendment was not valid should the trust property be distributed to Settlorrsquos grandchild (her only heir) or held in further trust in accordance with the terms of the original trust instrument

DISCUSSION

Summary

A revocable trust is amendable even if the trust instrument does not expressly grant to the trust settlor a power to amend Both inter vivos trusts and amendments thereto are valid even though not executed in accordance with the requirements applicable to wills

Under the Uniform Testamentary Additions to Trusts Act a revocable trust may be amended at any time prior to the settlorrsquos death and the amendment applies to the disposition of assets conveyed to the trust pursuant to a will even if the will was executed prior to the date of the amendment

At Settlorrsquos death trust assets including probate assets passing to the trust under Settlorrsquos will would go to University if as is the case here the trust amendment was valid If the amendment was invalid the trust assets would continue to be held in further trust because there is no violation of the common law Rule Against Perpetuities

Point One(a) (30) Settlor retained the right to amend the inter vivos trust despite her failure to expressly reserve this power

At issue here is whether a retained power of revocation includes the power to amend sometimes referred to as the power to modify The Restatement (Second) of Trusts sect 331 cmt g provides that if a settlor has a power to revoke that retained power ordinarily includes a power to modify (amend) as well Comment g also notes that the power to amend includes both a power to withdraw trust assets and a power to ldquomodify the terms of the trustrdquo The Uniform Trust Code which provides that a power to revoke includes the power to amend is consistent with this view

38

Trusts and Future Interests Analysis

UNIF TRUST CODE sect 602 accord RESTATEMENT (THIRD) OF TRUSTS sect 63 cmt The theory is that even though a power to amend was not expressly retained by a settlor the goal of amendment assuming the power was not included in the power to revoke could easily be achieved by first revoking the trust and then creating a new trust with the same terms contemplated by the amendment To require this would put form over substance

Thus by expressly retaining the power to revoke the trust Settlor retained a power to amend the inter vivos trust despite her failure to expressly reserve this power

[NOTE Under the common law a trust is irrevocable unless the settlor expressly retains a power to revoke the trust Conversely under the Uniform Trust Code a trust is revocable unless the terms of the trust expressly provide otherwise See UNIF TRUST CODE sect 602 The Trust Codersquos position on revocation follows the minority view in the United States and is inconsistent with prior Restatements of Trusts (see Restatement (Second) of Trusts sect 330) Here the trust is revocable because Settlor expressly retained a power of revocation

The Uniform Trust Code has been adopted in 24 jurisdictions Alabama Arizona Arkansas District of Columbia Florida Kansas Maine Michigan Missouri Nebraska New Hampshire New Mexico North Carolina North Dakota Ohio Oregon Pennsylvania South Carolina Tennessee Utah Vermont Virginia West Virginia and Wyoming]

Point One(b) (10) Settlorrsquos amendment of the trust was valid despite her failure to have her signature to the trust amendment witnessed

Neither the common law nor state statutes require a trust instrument or an amendment to a trust instrument to be executed in accordance with the formalities prescribed for execution of a will Indeed an inter vivos trust that does not involve real estate can be created orally Under the Uniform Trust Code the only requirements for creating a valid inter vivos trust are intent the specification of beneficiaries and the designation of a trustee See UNIF TRUST CODE sect 402 accord RESTATEMENT (THIRD) OF TRUSTS sect 13

Here the amendment meets the requirements of both the Uniform Trust Code and the common law Thus the fact that Settlorrsquos signature was not witnessed when she signed the amendment to the trust does not make the amendment invalid

Point Two (20) Under the Uniform Testamentary Additions to Trusts Act a revocable trust may be amended at any time prior to the settlorrsquos death and the amendment applies to probate assets poured into the trust at the settlorrsquos death pursuant to the settlorrsquos will even when the will was executed prior to the date of the amendment

Historically property owned by an individual at her death passed to the individualrsquos heirs or to beneficiaries designated in a will executed with the formalities (writing signing witnessing) prescribed by state law However when a will devises property to the trustee of an inter vivos trust then the provisions of the trustmdashwhich may not have been executed in accordance with the formalities required for willsmdasheffectively determine who will receive the property Because of this possibility some early cases held that if an inter vivos trust was not executed with the same formalities required for a valid will then the trust was ineffective to dispose of probate assets poured into the trust at the settlorrsquos death pursuant to the settlorrsquos will

This line of cases has been overturned by the Uniform Testamentary Additions to Trusts Act (the Act) now Uniform Probate Code sect 2-511 Under the Act adopted in almost all

39

Trusts and Future Interest Analysis

jurisdictions a testamentary bequest to the trustee of an inter vivos trust established by the testator during his or her lifetime is valid if the trust is in writing it is identified in the testatorrsquos will and the trust instrument was executed before concurrently with or after the execution of the will Id The Act further specifies that such a bequest is valid even if the trust is amendable or revocable and that a later amendment applies to assets passing to the trust by a previously executed will

Thus because the trust amendment is valid its terms apply to assets received by Bank from Settlorrsquos estate

Point Three (10) If the trust amendment was valid then the trust assets including assets passing to the trust under Settlorrsquos will should go to University

Under the trust amendment all trust assets (including the assets of Settlorrsquos probate estate poured into the trust) pass to University The facts provide no basis for failing to comply with Settlorrsquos stated intentions

Point Four (30) If the trust amendment was invalid trust assets including assets received pursuant to Settlorrsquos will should be held in accordance with the terms of the original trust instrument because those terms do not violate the Rule Against Perpetuities

Under the dispositive terms of the original trust instrument Settlor created successive income interests in her surviving children and grandchildren with a remainder interest in her great-grandchildren Because the trust was revocable the period during which the common law Rule Against Perpetuities requires that interests vest (ie 21 years plus lives in being) began to run from the date Settlor no longer had a power of revocation (here her death) not the date on which the trust was created See JESSE DUKEMINIER STANLEY J JOHANSON JAMES LINDGREN amp ROBERT SITKOFF WILLS TRUSTS AND ESTATES 678 (7th ed 2005)

Under the common law Rule Against Perpetuities Settlorrsquos trust is thus valid At the time of Settlorrsquos death she was survived by no children one granddaughter and no great-grandchildren Because Settlor cannot have more children after her death the only income beneficiary of the trust is Settlorrsquos surviving granddaughter This granddaughter is the only person who can produce great-grandchildren of Settlor thus all great-grandchildren must of necessity be born during the lifetime of Settlorrsquos only surviving granddaughter who is a life in being The granddaughterrsquos interest vested at Settlorrsquos death and the great-grandchildrenrsquos interest will vest at the death of the granddaughter There is no need to wait the additional 21 years permitted under the Rule Thus under the common law and the statute given in the facts the nonvested interest in the great-grandchildren is valid

[NOTE Both modern wait-and-see statutes and the Uniform Statutory Rule Against Perpetuities upon which the statute in the facts is modeled provide that before using either reform to validate an otherwise invalid nonvested interest one should first determine if the nonvested interest violates the common law Rule If it does not then there is no need to reform This proposition which is applicable in all MEE user jurisdictions that have not simply abrogated the rule is tested by this problem]

40

NEGOTIABLE INSTRUMENTS ANALYSIS (Negotiable Instruments III IV V)

ANALYSIS

Legal Problems

(1)(a) What rights does a person in possession of a note that has been indorsed in blank by the payee have against the maker of the note

(1)(b) Which defenses may the maker of a note raise against a person entitled to enforce it who is not a holder in due course but is a transferee from a holder in due course

(2) What rights does a person entitled to enforce a note have against an indorser who transferred it for consideration with no warranties

(3) What rights does a person entitled to enforce a note have against a previous holder who transferred it as a gift without indorsing it

DISCUSSION

Summary

The niece is a holder of the note and is thus a person entitled to enforce it The chef the issuer of the note is obligated to pay it to the niece as the person entitled to enforce it The niece is not subject to any defense or claim of the chef relating to the improper repair of the oven because the niece has the rights of a holder in due course When the buyer bought the note from the repairman the buyer became a holder in due course of the note and thus took it free of any personal defenses the chef had against the repairman Even though the niece is not herself a holder in due course of the note the niece succeeded to the buyerrsquos rights as holder in due course and thus took free of the chefrsquos personal defenses

Because the chef refused to pay the note the niece can recover from the repairman on the repairmanrsquos obligation as indorser The niece cannot recover on the note against the buyer however because the buyer did not indorse the note (and thus incurred no indorserrsquos obligation) and the buyer did not receive any consideration for transfer of the note to the niece (and therefore made no transfer warranty)

[NOTE Although Article 9 of the Uniform Commercial Code governs the sale of promissory notes (a point that might be correctly noted by examinees) that Article does not determine the answer to any of the questions posed]

Point One(a) (20) The niece is the holder of the note and thus may enforce it against the chef who is the issuer of the note

The chef is the maker of the note and thus its issuer See UCC sectsect 3-103 3-105 The issuer of a note is obligated to pay it in accordance with its terms to a ldquoperson entitled to enforcerdquo it UCC sect 3-412 The niece is a ldquoperson entitled to enforcerdquo the note This is because the niece is the holder of the note and a holder of a note is a person entitled to enforce it UCC sect 3-301 The niece is the holder of the note because (i) the repairmanrsquos signature on the back of the note not

41

Negotiable Instruments Analysis

accompanied by words indicating a person to whom the note was made payable was a ldquoblank indorsementrdquo which had the effect of making the note a bearer instrument (ii) anyone in possession of a bearer instrument is a holder of it and (iii) the niece is in possession of the note See UCC sectsect 1-201(b)(21)(A) 3-204 and 3-205 Accordingly the chef has an obligation to the niece to pay the note in accordance with its terms and the niece may enforce that obligation

Point One(b) (40) The niece is not a holder in due course of the note but because she is a transferee from the buyer who was a holder in due course she has the same enforcement rights as the buyer Because the buyer as a holder in due course would have been able to enforce the note against the chef without being subject to defenses or claims arising from the improper repair the niece has the same rights and will not be subject to the chefrsquos defenses or claims about the repair

As noted in Point One(a) the chef has an obligation to the niece to pay the note in accordance with its terms However except against a person with the rights of a holder in due course the chef can raise any defenses or claims in recoupment that he would have if the claim on the note were an ordinary contract claim UCC sect 3-305 Thus except against a holder in due course the chef would be able to raise the improper repair as a defense or a claim in recoupment (a claim in response to the niecersquos claim)

But claims in recoupment and most defenses cannot be raised against a person with the rights of a holder in due course Against a holder in due course the chef can raise only the four ldquorealrdquo defenses listed in UCC sect 3-305(a)(1) (infancy duress lack of legal capacity or illegality that nullifies the obligation of the obligor under other law fraud in the factum discharge in insolvency proceedings) none of which is present here

The niece is not a holder in due course because she did not take the note for value See UCC sectsect 3-302(a)(2)(i) (criteria for holder in due course status) and 3-303(a) (definition of ldquovaluerdquo) But this does not mean that the niece is subject to the chefrsquos claim arising out of the improper repair The buyer was a holder in due course of the note because he took the note for value ($9500) in good faith and without notice of any facts that would have alerted him to the chefrsquos defense against the repairman UCC sect 3-302(a)(2) As a holder in due course the buyer owned the note free of the chefrsquos claim because that claim did not constitute a ldquorealrdquo defense UCC sect 3-305(b) When the buyer gave the note to the niece this constituted a ldquotransferrdquo of the note See UCC sect 3-203(a) When a note is transferred the transferee receives ldquoany right of the transferor to enforce the instrument including any right as a holder in due courserdquo UCC sect 3-203(b) Under this rule (also known as the ldquoshelter principlerdquo) the buyer transferred his freedom from the chefrsquos defenses to the niece and the niece can enforce the note free of the chefrsquos defenses

Point Two (20) Because the chef dishonored the note the niece can recover from the repairman on the repairmanrsquos obligation as indorser

The chefrsquos refusal to pay the note constituted dishonor See UCC sect 3-502 The repairman as an indorser of the note (see Point One(a)) incurred the obligations of an indorser under UCC sect 3-415(a) When a note has been dishonored one of the obligations of an indorser is to pay the amount of the note to a person entitled to enforce it Therefore the repairman is liable for the amount of the note to the niece a person entitled to enforce the note (so long as the niece gives proper notice of dishonor to the repairman)

42

Negotiable Instruments Analysis

[NOTE Because the repairman indorsed the note without warranties there are no transfer warranties UCC sect 3-416 cmt 5]

Point Three (20) The niece cannot recover on the note against the buyer as either indorser or warrantor because the buyer did not indorse the note and did not receive consideration for transferring the note to the niece

The buyer did not indorse the note and therefore did not incur the obligation of an indorser to pay the note upon dishonor

The niece cannot recover from the buyer under a transfer warranty theory because transfer warranties are made only by a person ldquowho transfers an instrument for considerationrdquo Here the buyer gave the instrument to the niece as a gift So the buyer made no transfer warranty UCC sect 3-416(a) Therefore the niece cannot recover from the buyer on that theory

43

National Conference of Bar Examiners 302 South Bedford Street | Madison WI 53703-3622 Phone 608-280-8550 | Fax 608-280-8552 | TDD 608-661-1275

wwwncbexorg e-mail contactncbexorg

  • Contents
  • Preface
  • Description of the MEE
  • Instructions
  • February 2013 Questions
    • Real Property Question
    • Contracts Question
    • Constitutional Law Question
    • Secured Transactions Question
    • Federal Civil Procedure Question
    • Agency Question
    • Evidence Question
    • Trusts and Future Interests Question
    • Negotiable Instruments Question
      • February 2013 Analyses
        • Real Property Analysis
        • Contracts Analysis
        • Constitutional Law Analysis
        • Secured Transactions Analysis
        • Federal Civil Procedure Analysis
        • Agency Analysis
        • Evidence Analysis
        • Trusts and Future Interests Analysis
        • Negotiable Instruments Analysis
            • ltlt ASCII85EncodePages false AllowTransparency false AutoPositionEPSFiles true AutoRotatePages None Binding Left CalGrayProfile (Dot Gain 20) CalRGBProfile (sRGB IEC61966-21) CalCMYKProfile (US Web Coated 050SWOP051 v2) sRGBProfile (sRGB IEC61966-21) CannotEmbedFontPolicy Error CompatibilityLevel 14 CompressObjects Tags CompressPages true ConvertImagesToIndexed true PassThroughJPEGImages true CreateJobTicket false DefaultRenderingIntent Default DetectBlends true DetectCurves 00000 ColorConversionStrategy CMYK DoThumbnails false EmbedAllFonts true EmbedOpenType false ParseICCProfilesInComments true EmbedJobOptions true DSCReportingLevel 0 EmitDSCWarnings false EndPage -1 ImageMemory 1048576 LockDistillerParams false MaxSubsetPct 100 Optimize true OPM 1 ParseDSCComments true ParseDSCCommentsForDocInfo true PreserveCopyPage true PreserveDICMYKValues true PreserveEPSInfo true PreserveFlatness true PreserveHalftoneInfo false PreserveOPIComments true PreserveOverprintSettings true StartPage 1 SubsetFonts true TransferFunctionInfo Apply UCRandBGInfo Preserve UsePrologue false ColorSettingsFile () AlwaysEmbed [ true ] NeverEmbed [ true ] AntiAliasColorImages false CropColorImages true ColorImageMinResolution 300 ColorImageMinResolutionPolicy OK DownsampleColorImages true ColorImageDownsampleType Bicubic ColorImageResolution 300 ColorImageDepth -1 ColorImageMinDownsampleDepth 1 ColorImageDownsampleThreshold 150000 EncodeColorImages true ColorImageFilter DCTEncode AutoFilterColorImages true ColorImageAutoFilterStrategy JPEG ColorACSImageDict ltlt QFactor 015 HSamples [1 1 1 1] VSamples [1 1 1 1] gtgt ColorImageDict ltlt QFactor 015 HSamples [1 1 1 1] VSamples [1 1 1 1] gtgt JPEG2000ColorACSImageDict ltlt TileWidth 256 TileHeight 256 Quality 30 gtgt JPEG2000ColorImageDict ltlt TileWidth 256 TileHeight 256 Quality 30 gtgt AntiAliasGrayImages false CropGrayImages true GrayImageMinResolution 300 GrayImageMinResolutionPolicy OK DownsampleGrayImages true GrayImageDownsampleType Bicubic GrayImageResolution 300 GrayImageDepth -1 GrayImageMinDownsampleDepth 2 GrayImageDownsampleThreshold 150000 EncodeGrayImages true GrayImageFilter DCTEncode AutoFilterGrayImages true GrayImageAutoFilterStrategy JPEG GrayACSImageDict ltlt QFactor 015 HSamples [1 1 1 1] VSamples [1 1 1 1] gtgt GrayImageDict ltlt QFactor 015 HSamples [1 1 1 1] VSamples [1 1 1 1] gtgt JPEG2000GrayACSImageDict ltlt TileWidth 256 TileHeight 256 Quality 30 gtgt JPEG2000GrayImageDict ltlt TileWidth 256 TileHeight 256 Quality 30 gtgt AntiAliasMonoImages false CropMonoImages true MonoImageMinResolution 1200 MonoImageMinResolutionPolicy OK DownsampleMonoImages true MonoImageDownsampleType Bicubic MonoImageResolution 1200 MonoImageDepth -1 MonoImageDownsampleThreshold 150000 EncodeMonoImages true MonoImageFilter CCITTFaxEncode MonoImageDict ltlt K -1 gtgt AllowPSXObjects false CheckCompliance [ None ] PDFX1aCheck false PDFX3Check false PDFXCompliantPDFOnly false PDFXNoTrimBoxError true PDFXTrimBoxToMediaBoxOffset [ 000000 000000 000000 000000 ] PDFXSetBleedBoxToMediaBox true PDFXBleedBoxToTrimBoxOffset [ 000000 000000 000000 000000 ] PDFXOutputIntentProfile () PDFXOutputConditionIdentifier () PDFXOutputCondition () PDFXRegistryName () PDFXTrapped False CreateJDFFile false Description ltlt ARA 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 BGR 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 CHS ltFEFF4f7f75288fd94e9b8bbe5b9a521b5efa7684002000410064006f006200650020005000440046002065876863900275284e8e9ad88d2891cf76845370524d53705237300260a853ef4ee54f7f75280020004100630072006f0062006100740020548c002000410064006f00620065002000520065006100640065007200200035002e003000204ee553ca66f49ad87248672c676562535f00521b5efa768400200050004400460020658768633002gt CHT ltFEFF4f7f752890194e9b8a2d7f6e5efa7acb7684002000410064006f006200650020005000440046002065874ef69069752865bc9ad854c18cea76845370524d5370523786557406300260a853ef4ee54f7f75280020004100630072006f0062006100740020548c002000410064006f00620065002000520065006100640065007200200035002e003000204ee553ca66f49ad87248672c4f86958b555f5df25efa7acb76840020005000440046002065874ef63002gt CZE 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 DAN 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 DEU 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 ESP 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 ETI 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 FRA 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 GRE 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 HEB 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 HRV (Za stvaranje Adobe PDF dokumenata najpogodnijih za visokokvalitetni ispis prije tiskanja koristite ove postavke Stvoreni PDF dokumenti mogu se otvoriti Acrobat i Adobe Reader 50 i kasnijim verzijama) HUN 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 ITA 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 JPN ltFEFF9ad854c18cea306a30d730ea30d730ec30b951fa529b7528002000410064006f0062006500200050004400460020658766f8306e4f5c6210306b4f7f75283057307e305930023053306e8a2d5b9a30674f5c62103055308c305f0020005000440046002030d530a130a430eb306f3001004100630072006f0062006100740020304a30883073002000410064006f00620065002000520065006100640065007200200035002e003000204ee5964d3067958b304f30533068304c3067304d307e305930023053306e8a2d5b9a306b306f30d530a930f330c8306e57cb30818fbc307f304c5fc59808306730593002gt KOR ltFEFFc7740020c124c815c7440020c0acc6a9d558c5ec0020ace0d488c9c80020c2dcd5d80020c778c1c4c5d00020ac00c7a50020c801d569d55c002000410064006f0062006500200050004400460020bb38c11cb97c0020c791c131d569b2c8b2e4002e0020c774b807ac8c0020c791c131b41c00200050004400460020bb38c11cb2940020004100630072006f0062006100740020bc0f002000410064006f00620065002000520065006100640065007200200035002e00300020c774c0c1c5d0c11c0020c5f40020c2180020c788c2b5b2c8b2e4002egt LTH 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 LVI 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 NLD (Gebruik deze instellingen om Adobe PDF-documenten te maken die zijn geoptimaliseerd voor prepress-afdrukken van hoge kwaliteit De gemaakte PDF-documenten kunnen worden geopend met Acrobat en Adobe Reader 50 en hoger) NOR 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 POL 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 PTB ltFEFF005500740069006c0069007a006500200065007300730061007300200063006f006e00660069006700750072006100e700f50065007300200064006500200066006f0072006d00610020006100200063007200690061007200200064006f00630075006d0065006e0074006f0073002000410064006f0062006500200050004400460020006d00610069007300200061006400650071007500610064006f00730020007000610072006100200070007200e9002d0069006d0070007200650073007300f50065007300200064006500200061006c007400610020007100750061006c00690064006100640065002e0020004f007300200064006f00630075006d0065006e0074006f00730020005000440046002000630072006900610064006f007300200070006f00640065006d0020007300650072002000610062006500720074006f007300200063006f006d0020006f0020004100630072006f006200610074002000650020006f002000410064006f00620065002000520065006100640065007200200035002e0030002000650020007600650072007300f50065007300200070006f00730074006500720069006f007200650073002egt RUM 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 RUS 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 SKY ltFEFF0054006900650074006f0020006e006100730074006100760065006e0069006100200070006f0075017e0069007400650020006e00610020007600790074007600e100720061006e0069006500200064006f006b0075006d0065006e0074006f0076002000410064006f006200650020005000440046002c0020006b0074006f007200e90020007300610020006e0061006a006c0065007001610069006500200068006f0064006900610020006e00610020006b00760061006c00690074006e00fa00200074006c0061010d00200061002000700072006500700072006500730073002e00200056007900740076006f00720065006e00e900200064006f006b0075006d0065006e007400790020005000440046002000620075006400650020006d006f017e006e00e90020006f00740076006f00720069016500200076002000700072006f006700720061006d006f006300680020004100630072006f00620061007400200061002000410064006f00620065002000520065006100640065007200200035002e0030002000610020006e006f0076016100ed00630068002egt SLV 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 SUO 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 SVE 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 TUR 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 UKR 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 ENU (Use these settings to create Adobe PDF documents best suited for high-quality prepress printing Created PDF documents can be opened with Acrobat and Adobe Reader 50 and later) gtgt Namespace [ (Adobe) (Common) (10) ] OtherNamespaces [ ltlt AsReaderSpreads false CropImagesToFrames true ErrorControl WarnAndContinue FlattenerIgnoreSpreadOverrides false IncludeGuidesGrids false IncludeNonPrinting false IncludeSlug false Namespace [ (Adobe) (InDesign) (40) ] OmitPlacedBitmaps false OmitPlacedEPS false OmitPlacedPDF false SimulateOverprint Legacy gtgt ltlt AddBleedMarks false AddColorBars false AddCropMarks false AddPageInfo false AddRegMarks false ConvertColors ConvertToCMYK DestinationProfileName () DestinationProfileSelector DocumentCMYK Downsample16BitImages true FlattenerPreset ltlt PresetSelector MediumResolution gtgt FormElements false GenerateStructure false IncludeBookmarks false IncludeHyperlinks false IncludeInteractive false IncludeLayers false IncludeProfiles false MultimediaHandling UseObjectSettings Namespace [ (Adobe) (CreativeSuite) (20) ] PDFXOutputIntentProfileSelector DocumentCMYK PreserveEditing true UntaggedCMYKHandling LeaveUntagged UntaggedRGBHandling UseDocumentProfile UseDocumentBleed false gtgt ]gtgt setdistillerparamsltlt HWResolution [2400 2400] PageSize [612000 792000]gtgt setpagedevice

Page 14: February 2013 MEE Questions and Analyses

EVIDENCE QUESTION _____

A woman who owns a motorized scooter brought her scooter to a mechanic for routine maintenance service As part of the maintenance service the mechanic inspected the braking system on the scooter As soon as the mechanic finished inspecting and servicing the scooter he sent the woman a text message to her cell phone that read ldquoJust finished your service When you pick up your scooter you need to schedule a follow-up brake repair Wersquoll order the partsrdquo

The woman read the mechanicrsquos text message and returned the next day to pick up her scooter As the woman was wheeling her scooter out of the shop she saw the mechanic working nearby and asked ldquoIs my scooter safe to ride for a whilerdquo The mechanic responded by giving her a thumbs-up The woman waved and rode away on the scooter

One week later while the woman was riding her scooter a pedestrian stepped off the curb into a crosswalk and the woman collided with him causing the pedestrian severe injuries The woman had not had the scooterrsquos brakes repaired before the accident

The pedestrian has sued the woman for damages for his injuries resulting from the accident The pedestrian has alleged that (1) the woman lost control of the scooter due to its defective brakes (2) the woman knew that the brakes needed repair and (3) it was negligent for the woman to ride the scooter knowing that its brakes needed to be repaired

The woman claims that the brakes on the scooter worked perfectly and that the accident happened because the pedestrian stepped into the crosswalk without looking and the woman had no time to stop The woman the pedestrian and the mechanic will testify at the upcoming trial

The pedestrian has proffered an authenticated copy of the mechanicrsquos text message to the woman

The woman plans to testify that she asked the mechanic ldquoIs my scooter safe to ride for a whilerdquo and that he gave her a thumbs-up in response

The evidence rules in this jurisdiction are identical to the Federal Rules of Evidence

Analyze whether each of these items of evidence is relevant and admissible at trial

1 The authenticated copy of the mechanicrsquos text message

2 The womanrsquos testimony that she asked the mechanic ldquoIs my scooter safe to ride for a whilerdquo and

3 The womanrsquos testimony describing the mechanicrsquos thumbs-up

10

TRUSTS AND FUTURE INTERESTS QUESTION

Ten years ago Settlor validly created an inter vivos trust and named Bank as trustee The trust instrument provided that Settlor would receive all of the trust income during her lifetime The trust instrument further provided that

Upon Settlorrsquos death the trust income shall be paid in equal shares to Settlorrsquos surviving children for their lives Upon the death of the last surviving child the trust income shall be paid in equal shares to Settlorrsquos then-living grandchildren for their lives Upon the death of the survivor of Settlorrsquos children and grandchildren the trust corpus shall be distributed in equal shares to Settlorrsquos then-living great-grandchildren

The trust instrument expressly specified that the trust was revocable but it was silent regarding whether Settlor could amend the trust instrument

Immediately after creating the trust Settlor validly executed a will leaving her entire estate to Bank as trustee of her inter vivos trust to ldquohold in accordance with the terms of the trustrdquo

Five years ago Settlor signed an amendment to the inter vivos trust The amendment changed the disposition of the remainder interest specifying that all trust assets ldquoshall be paid upon Settlorrsquos death to Universityrdquo Settlorrsquos signature on this amendment was not witnessed

A state statute provides that any trust interest that violates the common law Rule Against Perpetuities ldquois nonetheless valid if the nonvested interest in the trust actually vests or fails to vest either (a) within 21 years of lives in being at the creation of the nonvested interest or (b) within 90 years of its creationrdquo

Recently Settlor died leaving a probate estate of $200000 She was survived by no children one granddaughter (who would be Settlorrsquos only heir) and no great-grandchildren The granddaughter has consulted your law firm and has raised four questions regarding this trust

1 Was Settlorrsquos amendment of the inter vivos trust valid Explain

2 Assuming that the trust amendment was valid do its provisions apply to Settlorrsquos probate assets Explain

3 Assuming that the trust amendment was valid how should trust assets be distributed Explain

4 Assuming that the trust amendment was invalid how should trust assets be distributed Explain

11

NEGOTIABLE INSTRUMENTS QUESTION

A chef entered into a contract with a repairman pursuant to which the repairman agreed to repair the chefrsquos commercial oven for $10000 The repairman agreed to accept as payment a negotiable promissory note for $10000 payable two months after its issuance

After the repairman worked on the oven the chef gave him a $10000 note as payment for the work As agreed the note was signed by the chef as maker was payable to the order of the repairman was payable in two months and fulfilled all criteria for negotiability

The next day the repairman sold the note to a buyer for $9500 To effectuate the sale the repairman wrote ldquono warrantiesrdquo on the back of the note signed his name immediately below that and handed the note to the buyer The buyer bought the note in good faith and without knowledge of any facts relating to the work that the repairman had performed for the chef

Later the buyer gave the note to his niece as a gift To effectuate the gift the buyer handed the note to the niece but did not indorse it

Shortly thereafter the chef discovered that the repair work had been done improperly and the oven still did not function correctly The chef tried repeatedly to get the repairman to return to correct the repair work but the repairman ignored all the chefrsquos calls

On the notersquos due date the niece contacted the chef and demanded that he pay the amount of the note to her The chef refused and told the niece that he would not pay the note because the repairman did not properly repair the oven

1 What are the niecersquos rights against the chef Explain

2 What are the niecersquos rights against the repairman Explain

3 What are the niecersquos rights against the buyer Explain

12

February 2013 MEE

ANALYSES

Real Property Contracts

Constitutional Law Secured Transactions

Federal Civil Procedure Agency

Evidence Trusts and Future Interests

Negotiable Instruments

REAL PROPERTY ANALYSIS (Real Property ID1a 4 amp 5)

ANALYSIS

Legal Problems

(1) Does the tenant have a defense to the landlordrsquos action for unpaid rent based on constructive eviction

(2) Does the tenant have a defense to the landlordrsquos action for unpaid rent based on the tenantrsquos surrender of the premises

(3) What if anything may the landlord recover from the tenant for the period after the tenant vacated the building

DISCUSSION

Summary

Under the common law the tenant does not have a defense to the landlordrsquos action for unpaid rent based on constructive eviction Constructive eviction is based on the tenant proving that (1) the landlord breached a duty to the tenant (2) the breach caused a loss by the tenant of the substantial use and enjoyment of the premises (3) the tenant gave the landlord adequate notice and opportunity to repair and (4) the tenant vacated the leased premises Here there was no constructive eviction because although the tenant vacated and gave the landlord adequate notice the landlord breached no express or implied duty to the tenant to repair the premises

The tenant does not have a defense based on the landlordrsquos acceptance of his surrender of the premises a landlordrsquos retention of keys does not constitute an acceptance of the tenantrsquos surrender unless the landlord so intended and here the landlordrsquos statements to the tenant at the time of the surrender of the keys do not evidence the intent to accept the tenantrsquos surrender

Under the common law a landlord has no duty to mitigate damages but also cannot sue for rents due in the future Under this approach the landlord can sue only for past-due rents Using this approach on November 1 the landlord could recover all the rent past due (ie rent for September and October) but could not recover for rents due in the future However some courts have authorized recovery for future rent minus the fair market rental value of the premises It is thus possible that the landlord could recover damages equal to the amount of rent due from September 1 to the end of the six-year lease term ($180000) minus the propertyrsquos fair-market rental value over that same period

Point One (45) The tenant was not constructively evicted because the landlord had no duty to repair the commercial premises that were the subject of the lease

The landlord and the tenant entered into a term-of-years lease because the lease specified both a beginning and an ending date HERBERT HOVENKAMP amp SHELDON F KURTZ THE LAW OF

PROPERTY 256 (5th ed 2001) Although a term-of-years lease normally cannot be terminated by the tenant prior to the end of the term a tenant may terminate a term-of-years lease if the tenant

15

Real Property Analysis

is constructively evicted See id at 286ndash88 Typically as here a claim of constructive eviction is made as a defense to a landlordrsquos action for damages or unpaid rent

In order to establish a constructive eviction the tenant must prove that the landlord breached a duty to the tenant such as a duty to repair and that the landlordrsquos breach caused a loss of the substantial use and enjoyment of the premises The tenant must also show that he gave the landlord notice adequate to permit the landlord to meet his duty to the tenant and that the tenant vacated the leased premises Id see also JOHN G SPRANKLING UNDERSTANDING

PROPERTY LAW sect 1704 (2d ed 2007) Under the common law there was no implied duty on the part of a landlord to repair

leased premises such a duty arose only if expressly set forth in the lease SPRANKLING supra sect 1702[B] Here the written lease contained no term requiring the landlord to repair the air-conditioning Even if the conversation created a lease term that the building had air-conditioning that itself should not create a duty for the landlord to repair it

Over the past several decades courts have generally implied a duty to repair in residential leases either as part of a revised constructive eviction doctrine or based on an implied warranty of habitability JOSEPH W SINGER PROPERTY 469ndash70 (3d ed 2010) This shift has been justified based on the economic disparity between the typical landlord and tenant as well as the fact that residential tenants generally lack both the authority to authorize repairs to common areas of a building and the incentive to make repairs that will ultimately benefit the landlord

However courts have been more reluctant to imply a duty to repair in commercial leases a context in which the tenant is often a valuable business and in a better position to assess and make repairs than is the landlord But see eg Davidow v Inwood North Professional Group 747 SW2d 373 (Tex 1988) When courts have implied a duty to repair in a commercial lease it is typically when the repair has been mandated by public authorities and involves work so substantial that it would not ordinarily fall within the tenantrsquos common law repair duty andor the value of the repair would primarily inure to the landlordrsquos reversionary interest See Brown v Green 884 P2d 55 (Cal 1994) Eugene L Grant et al The Tenant as Terminator Constructive Eviction in Modern Commercial Leases 2 THE COMMERCIAL PROPERTY LEASE ch 15 (ABA 1997) Some courts have also permitted constructive eviction claims by commercial tenants of office buildings based on repairs required in common areas of the building See id Echo Consulting Services Inc v North Conway Bank 669 A2d 227 (NH 1995)

Here the tenant is the owner of a valuable manufacturing operation and is the exclusive occupant of the building the repair has not been mandated by public authorities and the repair is not structural To the contrary the repair involves a feature of the building of unusual importance in the tenantrsquos manufacturing operation and the tenant is likely far more knowledgeable than the landlord about the air-conditioning specifications necessary for the manufacture of the tenantrsquos product

Based on these facts it is unlikely that a court will find that the tenant in this case was constructively evicted Although the tenant can show that he gave adequate notice to the landlord of the air-conditioning malfunction and vacated the premises the lease was commercial and it did not contain any promises or covenants by the landlord except a covenant of quiet enjoyment a covenant of quiet enjoyment does not entail any repair obligations

[NOTE An examineersquos conclusion is less important than his or her demonstrated awareness of the elements of constructive eviction and the need to imply a repair duty for such a defense to be viable here Although the implied warranty of habitability is not available to this tenant Texas Minnesota and Massachusetts imply a warranty of suitability in commercial leases in limited circumstances and an examinee might argue that this warranty should apply

16

Real Property Analysis

here If an examinee concludes that this warranty applies he or she should discuss the other requirements for constructive eviction

If the examinee wrongly concludes that the first element for a constructive eviction has been met the examinee will then have to discuss the remaining three elements in order to conclude that the tenant can claim constructive eviction The tenant would have a strong argument that the second elementmdashsubstantial interference with the use and enjoyment of the premisesmdashalso is met As indicated above the landlord was aware that a functioning air-conditioning system was vital to the tenantrsquos manufacturing operations The facts further indicate that the system had failed three times in the past few months The landlord may try to argue that the malfunctions did not substantially interfere with the tenantrsquos use of the premises because the malfunctions caused the temperature to climb above 81 degrees for only a short period of timemdash 3 hours 6 hours and 10 hours respectivelymdashon each occasion The tenant will argue however that the landlord was aware that the tenantrsquos manufacturing operations could tolerate temperatures above 81 degrees for no more than 6 hours The final malfunction exceeded that limit destroying $150000 worth of the tenantrsquos products

The tenant would also have a strong argument that the third element is met notice and opportunity to cure The tenant notified the landlord of the problem immediately upon the systemrsquos first malfunction and did so again when it malfunctioned a second time and then a third time The landlord might argue that there was insufficient time to cure the problem because the system corrected itself within a few hours on the first and second times Although the malfunction lasted more than 10 hours the third time the landlord might argue that the time period was insufficient to get a repair person on the premises A court would be likely to find this argument unpersuasive however because the landlord could have attempted to correct the problem after the first and second malfunctions

Assuming that the landlord was given sufficient notice and opportunity to cure a court would be likely to conclude that the tenant also satisfied the final element of vacating the premises within a reasonable time The landlord might argue that the tenant remained in the premises for almost four months after the air conditioning first failed which would suggest that the problem was not so severe as to have constructively evicted the tenant The tenant will argue however that he gave the landlord three months to cure the problem after the first two malfunctions threatened (but did not actually harm) his operations The tenant then moved out shortly after the final malfunction caused temperatures to exceed the tolerance levels of his manufacturing operations]

Point Two (10) The landlord did not accept the tenantrsquos surrender of the lease

When a tenant wrongfully moves from leased premises with the intent to terminate the lease the landlord may either accept the tenantrsquos surrender of the premises and terminate the lease or hold the tenant to the terms of the lease See HOVENKAMP amp KURTZ supra at 295ndash96 Here the tenantrsquos only basis for the claim that the landlord accepted his surrender is the landlordrsquos retention of the keys Many courts have considered whether a landlordrsquos retention of keys delivered by a tenant constitutes acceptance of surrender The weight of the case law holds that retention of the keys alone does not constitute acceptance of surrender without other evidence showing that the landlord intended to accept the surrender See generally 49 AM JUR 2d Landlord and Tenant sect 213

Here the landlordrsquos note saying ldquoI repeat the air-conditioning is not my problem You have leased the building and you should fix itrdquo strongly suggests that the landlord did not intend

17

Some courts have rejected the no-mitigation-of-damages rule based on efficiency concerns and societyrsquos interest in assuring that resources remain in the stream of commerce rather than lying vacant see id at 464ndash65 and allow landlords to sue tenants who have wrongfully terminated a lease for damages equal to the difference between the unpaid rent due under the lease and the propertyrsquos fair market rental value Other courts have abandoned the no-recovery-for-future-rent rule These courts responding to the fact that a tenant may well disappear or be judgment-proof by the time a lease term is concluded have allowed a landlord to collect damages equal to the value of rent over the entire lease term minus the propertyrsquos fair rental value when a tenant has wrongfully terminated a lease and unequivocally shown an intention not to return to the premises or pay future rent Under this approach a landlord receives approximately the same amount he would have received were there a duty to mitigate damages See Sagamore Corp v Willcutt 180 A 464 (Conn 1935)

Real Property Analysis

to accept the tenantrsquos surrender The tenant might argue that the landlordrsquos failure to make a similar statement when the keys were sent to her a second time and she retained them evidences a change of heart However it is likely that a court would find that the landlordrsquos retention of the keys represented a decision to safeguard the keys not to accept the tenantrsquos surrender

[NOTE An examinee should receive credit for arguing the other way with a well-reasoned argument]

Point Three (45) Under the common law the landlord had no duty to mitigate damages Additionally a landlord was not entitled to recover unpaid rents due in the future but was only entitled to recover rents in arrears at the time of the commencement of the suit Applying the common law here the landlord could recover $5000 the amount of rents due at the commencement of the suit ($2500 for September and the same for October) Today some courts allow the landlord under certain circumstances to sue the tenant for damages (not rent) equal to the difference if any between the unpaid promised rent for the balance of the term (here $175000) and the propertyrsquos fair rental value for the balance of the term

Under the common law because a lease was viewed as a conveyance instead of a contract a landlord had no duty to mitigate damages resulting from a tenantrsquos wrongful termination of a lease A landlord could thus recover the full value of rents that were due and unpaid at the time of the suit However under the common law a landlord could not sue a tenant for rents due in the future because there was always a possibility that the tenant might pay the rent when it was due See SINGER supra at 462 Thus using the common law approach on November 1 the landlord could only recover the full value of the two monthsrsquo rent actually due and unpaid ie $5000 for September and October

Here because the tenant returned the keys to the landlord and said ldquoI will not be returning to the building or making further rent paymentsrdquo the landlord could establish abandonment and an intention not to return It is thus possible that the landlord might recover damages in the amount of $5000 (for the months of September and October) plus the present value of $175000 minus the fair market rental value of the property over the remaining months of the lease

18

CONTRACTS ANALYSIS ____ (Contracts II IVE)

ANALYSIS

Legal Problems

(1) What was the legal effect of the sailorrsquos October 31 letter to the builder

(2)(a) What was the legal effect of the builderrsquos November 25 response to the sailorrsquos October 31 letter

(2)(b) What was the legal effect of the sailorrsquos refusal to take and pay for the boat on December 15

DISCUSSION

Summary

This is a sale of goods governed by the Uniform Commercial Code Because the sailor had reasonable grounds for insecurity about the builderrsquos ability to deliver the boat in a timely manner when the sailor learned about the strike on October 31 the sailor was legally justified in sending the letter to the builder seeking adequate assurance of the builderrsquos performance pursuant to the contract The builderrsquos failure to provide such assurance within a reasonable time operated as a repudiation of the contract However the builder was free to retract the repudiation before the sailor either cancelled the contract or materially changed position in reliance on the builderrsquos repudiation The builder retracted the repudiation when he informed the sailor that the workers were back and that the boat would be delivered by the date stipulated in the partiesrsquo contract Because the sailor had taken no action in response to the original repudiation he no longer had the right to cancel the contract with the builder The sailorrsquos subsequent statement that ldquoour contract is overrdquo may have constituted repudiation by the sailor In any event when the sailor failed to perform on December 15 that constituted breach

Point One (35) Because the sailor had reasonable grounds for insecurity with respect to the builderrsquos performance the sailorrsquos letter to the builder was a justified demand seeking assurance of the builderrsquos performance under the contract failure of the builder to provide such assurance within a reasonable time constituted repudiation of the contract

The sailor was legally justified in sending the letter to the builder on October 31 Contract parties are entitled to expect due performance of contractual obligations and are permitted to take steps to protect that expectation UCC sect 2-609 states that ldquo[w]hen reasonable grounds for insecurity arise with respect to the performance of either party the other may in writing demand adequate assurance of due performance rdquo Here the sailor learned on October 31 that the builderrsquos workers were on strike This gave the sailor reasonable grounds for insecurity about the builderrsquos ability to complete performance on time and thus gave the sailor the right to seek adequate assurance from the builder Because the sailorrsquos demand for assurance was justified the builder was required to provide assurance that was adequate under the circumstances within a reasonable time (not to exceed 30 days) or be held to have repudiated the contract UCC sect 2-609(4)

19

Contracts Analysis

Point Two(a) (30) The builder did not within a reasonable time provide the sailor adequate assurance of due performance this failure to provide assurance constituted a repudiation of the contract

Because the sailor with legal justification (see Point One) demanded from the builder assurance of due performance the builderrsquos failure to provide such assurance within a reasonable time was a repudiation of their contract See UCC sect 2-609(4) (ldquoAfter receipt of a justified demand[] failure to provide within a reasonable time not exceeding thirty days assurance of due performance is a repudiation of the contractrdquo) On October 31 the sailor requested that the builder provide adequate assurance regarding the completion of the boat by December 15 The builder did not respond to the sailorrsquos letter until November 25mdashnearly a month later Even if that response had been given in a reasonable time it nonetheless did not provide assurance of due performance It simply stated ldquoIrsquom sorry about the strike but it is really out of my hands I hope we settle it soon so that we can get back to workrdquo Therefore the builderrsquos November 25 response did not provide adequate assurance in response to the sailorrsquos justified request Thus the builder had repudiated the contract

Point Two(b) (35) Although the builder repudiated the contract with the sailor the builder probably retracted that repudiation on December 3 and the sailor was no longer entitled to cancel their contract Thus the sailorrsquos failure to perform the sailorrsquos obligations under the contract constituted a breach

The builderrsquos failure to provide adequate assurance of performance constituted a repudiation of their contract (see UCC sect 2-609(4)) but the builder was free to retract that repudiation until the sailor cancelled the contract or materially changed his position or indicated by communication or action that the sailor considered the repudiation to be final See UCC sect 2-611(1) (ldquoUntil the repudiating partyrsquos next performance is due he can retract his repudiation unless the aggrieved party has since the repudiation cancelled or materially changed his position or otherwise indicated that he considers the repudiation finalrdquo)

Here the facts state that before the builderrsquos December 3 telephone call to the sailor the sailor did nothing in response to the builderrsquos repudiation such as contracting with a third party for a boat The builderrsquos December 3 call informing the sailor that the boat would be timely delivered probably constituted a retraction of the repudiation because it clearly indicated to the sailor that the builder would be able to perform UCC sect 2-611(2) Thus after being so informed the sailor did not have the right to treat their contract as cancelled UCC sect 2-611(3) Accordingly the sailorrsquos failure to perform the sailorrsquos obligations under the contract by taking the boat and paying for it constituted a breach of the contract

20

CONSTITUTIONAL LAW ANALYSIS (Constitutional Law IVA F2b amp e)

ANALYSIS

Legal Problems

(1) Does AutoCorsquos operation of a ldquocompany townrdquo result in its actions counting as those of the state for purposes of constitutional analysis

(2) Does the expulsion of a schoolchild for failure to recite the Pledge of Allegiance violate the First Amendment as applied through the Fourteenth Amendment

(3) Does the arrest of a pamphleteer in connection with violation of an anti-littering rule where the littering is done by the recipients of leaflets distributed by the pamphleteer violate the First Amendment as applied through the Fourteenth Amendment

DISCUSSION

Summary

The First Amendment as applied through the Fourteenth Amendment applies only to state action It does not typically govern private actors However courts have found state action where the private actor has exercised a ldquopublic functionrdquo such as running a privately owned ldquocompany townrdquo as AutoCo has done here Thus First Amendment protections apply By requiring the son to participate in a mandatory Pledge of Allegiance ceremony AutoCo has compelled the expression of political belief in violation of the First Amendment as applied through the Fourteenth Amendment The fatherrsquos arrest in connection with breaching the anti-litter rule also violated the First Amendment as applied through the Fourteenth Amendment Although state actors can regulate the incidental effects of speech on the public streets on a content-neutral basis this power is limited and cannot extend to punishing a distributor of literature because of littering by third parties

Point One (30) AutoCorsquos operation of a company town (including a school) makes it a state actor under the public function strand of the state action doctrine

The individual rights protections of the Constitution apply only where there is ldquostate actionrdquomdash either direct action by the government or some action by a private party that is fairly attributable to the government As a general rule the actions of a private company like AutoCo or of a private school like the school operated by AutoCo would not constitute state action and the protections of the Constitution (in this case the First Amendment) would not apply

However there are situations in which the actions of a private actor are attributed to the state One such situation is when the private actor undertakes a public function There are not many bright-line rules in the Supreme Courtrsquos state action doctrine but one of them is this Where a private actor undertakes a ldquopublic functionrdquo the Constitution applies to those actions Where a corporation operates a privately owned ldquocompany townrdquo that provides essential services typically provided by a state actor the public function doctrine applies and the Constitution

21

Constitutional Law Analysis

binds agents of the town as if they were agents of the government See eg Marsh v Alabama 326 US 501 (1946) Here AutoCo does more than own the town it provides security services fire protection sanitation services and a school Thus the actions of AutoCo constitute state action and are governed by the Fourteenth Amendment

Point Two (35) The sonrsquos expulsion for failure to recite the Pledge of Allegiance violates the First Amendment as applied through the Fourteenth Amendment as a compelled expression of political belief

As explained in Point One the First Amendment applies to the school as a state actor Although children in public schools (and in schools subject to the First Amendment like

the Oakwood school) have some First Amendment rights Tinker v Des Moines Independent Community School District 393 US 503 506 (1969) schools have greater leeway to regulate the speech of students and teachers than the state would have outside the school context Hazelwood School Dist v Kuhlmeier 484 US 260 (1988) Morse v Frederick 551 US 393 (2007) However the Supreme Court has long held that public schools may not force their students to participate in a flag salute ceremony when it offends the political or religious beliefs of the students or their families West Virginia Board of Educ v Barnette 319 US 624 (1943) (invalidating a mandatory public school flag salute ceremony) see also Wooley v Maynard 430 US 705 (1977) (invalidating compelled expression of political belief on state-issued license plates)

In this case the school requires its students to participate in a flag salute and Pledge of Allegiance ceremony and punishes them when they refuse to participate Pursuant to this policy the school has expelled the son This expulsion violates the First Amendment ban on compelled expression

Point Three (35) Because the father was distributing leaflets in a traditional public forum his trespass arrest violated the First Amendment as applied through the Fourteenth Amendment

As explained in Point One AutoCo is treated as a state actor Thus Oakwoodrsquos commercial district is treated as government-owned property for purposes of the First Amendment Thus the leafleting here is subject to the First Amendment because it is an expressive activity Schneider v State of New Jersey Town of Irvington 308 US 147 (1939) When expression takes place on government-owned property government regulation of the expression is assessed under the public forum doctrine Public streets and sidewalks have long been held to be the classic example of a ldquotraditional public forumrdquo open to the public for expression Hague v CIO 307 US 496 515ndash16 (1939) Because the father was distributing leaflets while standing on a street corner in the commercial district his expressive activity occurred in a traditional public forum

When a state tries to regulate expressive activity in a traditional public forum it is prohibited from doing so based on the expressive activityrsquos content unless its regulation is narrowly tailored to achieve a compelling governmental interest (ldquostrict scrutinyrdquo) In this case however AutoCo is regulating the fatherrsquos expressive activity on the ostensibly neutral ground that his expressive activity has produced litter and made the street unsightly When a state tries to regulate expressive activity without regard to its content intermediate scrutiny applies Under intermediate scrutiny the true purpose of the regulation may not be the suppression of ideas (if so then strict scrutiny applies) the regulation must be narrowly tailored to achieve a significant

22

Constitutional Law Analysis

governmental interest and it must leave open ample alternative channels for expressive activity Ward v Rock Against Racism 491 US 781 791 (1989)

Here the application of the ordinance to the father will fail for two reasons First the Supreme Court has held that the governmentrsquos interest in keeping the streets clean is insufficient to ban leafleting in the public streets as the government power to regulate with incidental effects on public sidewalk speech is very limited See eg Schneider 308 US at 162 (leafletinglittering) Second the regulation (a blanket ban on distribution that results in littering) is not narrowly tailored to protect expression A narrowly tailored alternative would be prosecution only of people who litter Moreover the effect of the littering rule is likely to be a ban on all leafleting thus eliminating an entire class of means of expression This raises the possibility that there are not ldquoample alternative channels of communicationrdquo open to the father as required under the Courtrsquos standard of review for content-neutral regulation of speech

[NOTE Some examinees might argue that this is a ldquotime place and mannerrdquo restriction and that AutoCo might have greater latitude to regulate the public sidewalks under this theory This argument is incorrect for two reasons First the Supreme Court has held that the power to regulate speakers through littering laws is very limited for the reasons given and in the cases cited above But more generally a ldquotime place and mannerrdquo restriction involves the shifting of speech from one time and place to another or to another manner here there is no shifting but a direct punishment for expressive activity (albeit one couched in content-neutral terms) In addition some examinees might read the ordinance to be in effect a total ban on leafleting since most leafleting will produce some litter Those examinees might note that the Court has required total bans on an entire mode of expression to satisfy strict scrutiny and analyze the fatherrsquos prosecution here accordingly See United States v Grace 461 US 171 177 (1983) (invalidating ban on display of signs on public sidewalks surrounding US Supreme Court ldquo[a]dditional restrictions such as an absolute prohibition on a particular type of expression will be upheld only if narrowly drawn to accomplish a compelling governmental interestrdquo)]

23

SECURED TRANSACTIONS ANALYSIS (Secured Transactions IID E IVA B C)

ANALYSIS

Legal Problems

(1) Is a purchase-money security interest in consumer goods perfected even though there has been no filing of a financing statement

(2) Does a person who buys consumer goods for personal use take those goods free of a prior perfected purchase-money security interest in the goods

(3) Does a person who receives consumer goods as a gift take those goods subject to a prior perfected security interest in them

DISCUSSION

Summary

The retailerrsquos security interest in the bicycles was perfected even though no financing statement was filed because it was a purchase-money security interest in consumer goods A purchase-money security interest in consumer goods is automatically perfected upon attachment

The buyer is not subject to the retailerrsquos security interest in the bicycle that the buyer bought from the man Because the bicycle was consumer goods in the hands of the man and the retailer never filed a financing statement covering the bicycle the retailerrsquos security interest is not effective against someone like the buyer who bought the bicycle for value without knowledge of the retailerrsquos security interest and for personal use

On the other hand the retailerrsquos security interest continues in the bicycle given to the friend because the friend did not give value for the bicycle or buy it in the ordinary course of business

Point One (35) The retailerrsquos security interest in the bicycles attached on June 1 Because this interest was a purchase-money security interest in consumer goods it was automatically perfected when it attached

The retailerrsquos security interest in the bicycles attached on June 1 when the man bought the bicycles (acquiring rights in the collateral) signed a security agreement containing a description of the collateral and received value from the retailer (by being given credit with which to purchase the bicycles) UCC sect 9-203(a) amp (b)

Despite the retailerrsquos failure to file a financing statement its security interest was perfected Pursuant to UCC sect 9-309(1) a security interest is automatically perfected upon attachment if the goods are ldquoconsumer goodsrdquo and the security interest is a ldquopurchase-money security interestrdquo

In this case the bicycles sold by the retailer to the man were consumer goods at the time of sale The bicycles were ldquogoodsrdquo because they were ldquomovable when a security interest

24

Secured Transactions Analysis

attachesrdquo UCC sect 9-102(a)(44) They were also consumer goods because they were ldquobought for use primarily for personal family or household purposesrdquo UCC sect 9-102(a)(23) The retailerrsquos security interest in these consumer goods was also a ldquopurchase-money security interestrdquo A purchase-money security interest is an interest that secures a debt that was incurred in order to ldquoenable the debtor to acquire rights in or the use of the collateralrdquo UCC sect 9-103(a) (b)(1) Here the man incurred an obligation to the retailer to purchase the bicycles so the security interest he gave the retailer to secure that obligation was a purchase-money security interest

Because the retailerrsquos security interest was a purchase-money security interest in consumer goods it was automatically perfected on June 1 when the interest attached to the bicycles

Point Two (35) The buyer took the bicycle free of the retailerrsquos security interest because (i) the retailer did not file a financing statement covering the bicycle (ii) the bicycle was ldquoconsumer goodsrdquo and (iii) the buyer bought the bicycle for value without knowledge of the retailerrsquos security interest and for personal use

A security interest continues in collateral even after a sale or other disposition of that collateral unless the creditor authorized the disposition ldquofree of the security interestrdquo or another Article 9 exception applies UCC sectsect 9-201(a) and 9-315(a)(1)

However a buyer of goods like the buyer here can take free of a prior security interest in those goods under certain circumstances See UCC sectsect 9-317(b) (buyers who give value and receive delivery of goods without knowledge of an unperfected security interest in the goods) and 9-320(a) amp (b) (buyer in ordinary course of business buyer of consumer goods in a consumer-to-consumer transaction who gives value) In this case the retailerrsquos security interest was perfected when the buyer purchased the bicycle so UCC sect 9-317(b) does not protect the buyer The buyer also is not a protected ldquobuyer in ordinary course of businessrdquo because he did not purchase from a person who is in the business of selling bicycles See UCC sect 1-201(b)(9)

The buyer can however qualify for the protection of UCC sect 9-320(b) That section provides that a buyer of goods from a person who used them for personal family or household purposes takes free of a perfected security interest in the goods if (1) the buyer had no knowledge of the security interest (2) the buyer gave value for the goods (3) the buyer purchased the goods primarily for personal family or household purposes and (4) the purchase occurred before the filing of a financing statement covering the goods

The buyer met all of these criteria The man used the bicycle for personal purposes The buyer purchased the bicycle from the man and the buyer had no knowledge of the retailerrsquos security interest The buyer gave value ($400) for the bicycle and he bought it ldquoprimarily for personal family or household purposesrdquo as he planned to use it for recreation which is a personal rather than a business use Finally no financing statement had been filed Therefore under UCC sect 9-320(b) the buyer took free of the retailerrsquos security interest

Point Three (30) The retailerrsquos security interest continues in the bicycle that the man gave to the friend Thus the retailer can recover the bicycle from the friend because the friend did not give value for the bicycle or buy it in the ordinary course of business

25

Secured Transactions Analysis

As noted in Point Two the retailer did not authorize the man to dispose of the bicycle Consequently the retailerrsquos security interest continued in the bicycle even after the man transferred ownership of the bicycle to the friend See UCC sectsect 9-201(a) and 9-315(a)(1) The retailerrsquos security interest in the bicycle will be effective against the friend unless some other provision of Article 9 allows the friend to take the bicycle free of that security interest

Unfortunately for the friend there is no Article 9 provision that allows him to take free of the retailerrsquos interest The friendrsquos basic problem is that he is not a buyer of the bicyclemdashhe received the bicycle as a gift and did not give value for it Thus the friend is not protected by any of the applicable exceptions See UCC sectsect 9-317(b) (protecting buyers who give value for goods subject to an unperfected security interest) 9-320(a) (protecting buyers in ordinary course of business) and 9-320(b) (protecting buyers of consumer goods who give value)

In short the retailerrsquos security interest continues in the bicycle that the man gave to the friend The friend took the bicycle subject to that security interest

26

FEDERAL CIVIL PROCEDURE ANALYSIS (Federal Civil Procedure VIE)

ANALYSIS

Legal Problems

(1) Does a judgment in a prior action preclude a nonparty from suing the same defendant on a closely related claim when the nonparty and the original plaintiff are in a family relationship

(2) Does a judgment rendered in an earlier action preclude a nonparty from litigating an issue that was actually decided in the first suit

(3) May a nonparty to an earlier action invoke the judgment in that action to preclude a party to the prior action from relitigating an issue that the party had a full and fair opportunity to litigate in the earlier action

DISCUSSION

Summary

Pursuant to the doctrines of claim preclusion (res judicata) and issue preclusion (collateral estoppel) a judgment is binding on the parties thereto In the absence of privity nonparties to a prior suit cannot be bound by a judgment rendered in their absence Thus in the absence of privity a nonparty to the first suit is not precluded from presenting her claim in a second suit even if it is factually related to the claims and defenses presented in the first suit nor is she bound by determinations of issues made in the first suit A family relationship without more does not support a finding of privity For this reason Mother as a nonparty is not bound by the judgment in the Son-Driver action She may bring her separate claim for damage to her car and she is not precluded from litigating the question of whether she was negligent in the maintenance of her car

Driver on the other hand could be precluded from relitigating the issue of her negligence pursuant to the doctrine of non-mutual issue preclusion (also called non-mutual offensive collateral estoppel) which allows a nonparty to a prior action to invoke issue preclusion to prevent a party to that prior action from relitigating determinations of issues made therein However Mother may be prevented from invoking non-mutual collateral estoppel in this case because she could easily have joined her claim in the prior action but did not do so

[NOTE Federal common law governs the preclusive effect of a judgment rendered by a federal court sitting in diversity See Semtek Intrsquol Inc v Lockheed Martin Corp 531 US 497 508 (2001) But the Semtek Court concluded that federal common law in this context incorporates the preclusion law of the state in which the rendering federal court sits (unless the state law is incompatible with federal interests) id at 508ndash09 Thus State Arsquos preclusion law determines the preclusive effect of the judgment rendered in Sonrsquos suit against Driver The problem says that State A preclusion law is identical to federal preclusion law so the following analysis utilizes general principles of preclusion drawn from Supreme Court case law (announcing federal preclusion rules) and the Restatement (Second) of Judgments]

27

Federal Civil Procedure Analysis

Point One (35) Under the doctrine of claim preclusion the judgment rendered in the first action does not preclude Mother a nonparty from suing Driver for the damage to her car because the judgment binds only parties or those in privity with them and Mother and Son are not in privity

Driver may contend that the doctrine of claim preclusion (res judicata) precludes Mother from presenting a claim arising from the same nucleus of facts that was presented in the first action brought by Son According to the doctrine of claim preclusion ldquowhen a court of competent jurisdiction has entered a final judgment on the merits of a cause of action the parties to the suit and their privies are thereafter bound lsquonot only as to every matter which was offered and received to sustain or defeat the claim or demand but as to any other admissible matter which might have been offered for that purposersquordquo Commissioner of Internal Revenue v Sunnen 333 US 591 597 (1948) (citation omitted)

However the doctrine of claim preclusion does not apply to Mother on the facts of this problem First Mother was not a party to the earlier case ldquoIt is a principle of general application in Anglo-American jurisprudence that one is not bound by a judgment in personam in a litigation in which he is not designated as a party or to which he has not been made a party by service of processrdquo Taylor v Sturgell 553 US 880 884 (2008) (citing Hansberry v Lee 311 US 32 40 (1940)) see also RESTATEMENT (SECOND) OF JUDGMENTS sect 34(3) (1982) This rule reflects our ldquodeep-rooted historic tradition that everyone should have his own day in courtrdquo Martin v Wilks 490 US 755 762 (1989) (citation omitted) (superseded by statute on other grounds) Since Mother was not a party to the first suit she is not bound by the judgment unless an exception to the general rule applies

Mother might be bound by the prior judgment if she were considered to have been sufficiently in privity with Son that Son represented her interests in that action ldquoA person who is not a party to an action but who is represented by a party is bound by and entitled to the benefits of a judgment as though he were a partyrdquo RESTATEMENT (SECOND) OF JUDGMENTS sect 41(1) But there is no suggestion in the facts of the problem that Son who is an adult purported to represent Motherrsquos interests in the first suit ldquo[C]lose family relationships are not sufficient by themselves to establish privity with the original suitrsquos party or to bind a nonparty to that suit by the judgment entered therein rdquo Cuauhtli v Chase Home Finance LLC 308 Fed Appx 772 773 (5th Cir 2009) (citation omitted) accord 18A CHARLES ALAN WRIGHT ET AL FEDERAL

PRACTICE AND PROCEDURE sect 4459 (2d ed 2002) In Taylor v Sturgell supra the Supreme Court identified other special circumstances in

which nonparties may be bound by a prior judgmentmdashwhen a nonparty consents to be bound when a nonparty is in a pre-existing substantive legal relationship with a party (such as preceding and succeeding property owners) when a nonparty assumed control of the prior litigation when a party seeks to relitigate through a proxy or where a special statutory scheme seeks to foreclose successive litigation by nonparties See Taylor 553 US at 893ndash95 None of these circumstances exists here

Because Mother was not a party to the first suit and is not in privity with Son who is an adult the judgment in the first action does not preclude her from bringing her own claim against Driver

Point Two (35) Under the doctrine of issue preclusion the judgment rendered in the first action does not preclude Mother a nonparty from litigating the issue of her negligence in maintaining her carrsquos

28

Federal Civil Procedure Analysis

brake lights because the judgment binds only parties or those in privity with them and Mother and Son are not in privity

By its affirmative response to a special interrogatory the jury in the first action expressly concluded that ldquoMother negligently failed to ensure that the brake lights on her car were in proper working orderrdquo Driver may attempt to invoke the doctrine of issue preclusion to preclude Mother from relitigating this issue in the second action

[I]ssue preclusion arises in a second action on the basis of a prior decision when the same lsquoissuersquo is involved in both actions the issue was lsquoactually litigatedrsquo in the first action after a full and fair opportunity for litigation the issue was lsquoactually decidedrsquo in the first action by a disposition that is sufficiently lsquofinalrsquo lsquoon the meritsrsquo and lsquovalidrsquo it was necessary to decide the issue in disposing of the first action and the later litigation is between the same parties or involves nonparties that are subject to the binding effect or benefit of the first action Once these requirements are met issue preclusion is available not only to defend against a demand for relief but also as offensive support for a demand for relief Issue preclusion moreover is available whether or not the second action involves a new claim or cause of action

18 CHARLES ALAN WRIGHT ET AL FEDERAL PRACTICE AND PROCEDURE sect 4416 at 392ndash93 (2d ed) see also RESTATEMENT (SECOND) OF JUDGMENTS sect 27 (1982)

Here several of the elements necessary for issue preclusion are present The same issue is involved in both actionsmdashthe issue of Motherrsquos negligence in failing to maintain the brake lights on her car That issue was actually litigated in the first action and decided by the jury There is nothing to suggest anything less than a full and fair opportunity to litigate The judgment disposing of the issue was final

Nevertheless the judgment will not preclude Mother from relitigating the issue for two reasons First Mother was not a party to the first action and as explained above Mother and Son are not in privity Therefore she cannot be denied an opportunity to litigate the issue of her negligence Second it does not appear that the juryrsquos decision as to Motherrsquos negligence was necessary to the prior judgment against Driver Nothing suggests that the finding on Motherrsquos negligence had any bearing on the outcome of the first action

Point Three (30) Under the doctrine of non-mutual issue preclusion the judgment rendered in the first action might preclude Driver from relitigating the issue of her negligence However Driver has a strong argument that such a result would be inconsistent with the policy against offensive use of non-mutual estoppel when the non-party plaintiff easily could have joined as a plaintiff in the first action

Because Son already convinced the jury in the first action that ldquoDriver was negligent in the operation of her vehiclerdquo Mother may wish to invoke the doctrine of non-mutual issue preclusion to prevent Driver from relitigating the question of her negligence As noted above ldquoissue preclusion arises in a second action on the basis of a prior decision when the same lsquoissuersquo is involved in both actions the issue was lsquoactually litigatedrsquo in the first action after a full and fair opportunity for litigation the issue was lsquoactually decidedrsquo in the first action by a disposition that is sufficiently lsquofinalrsquo lsquoon the meritsrsquo and lsquovalidrsquo it was necessary to decide the issue in disposing of the first action rdquo 18 CHARLES ALAN WRIGHT ET AL FEDERAL PRACTICE AND

PROCEDURE sect 4416 at 392 (2d ed) see also RESTATEMENT (SECOND) OF JUDGMENTS sect 27

29

Federal Civil Procedure Analysis

Here these basic requirements for issue preclusion are met First the same issue is involved in both suits whether Driver was negligent in the operation of her car Second this issue was actually litigated and decided in the first action the jury answered a special interrogatory raising this very question There is nothing to suggest that Driver lacked a full and fair opportunity to litigate the issue Since a judgment was rendered against Driver for the injuries Son sustained as a result of Driverrsquos negligence resolution of the issue was necessary to dispose of the first action Driver was a party to the first action so she may be bound by the judgment

[NOTE Traditionally issue preclusion required mutualitymdashboth the party asserting issue preclusion and the party against whom issue preclusion was asserted were bound by the prior judgment Under the traditional mutuality rule Mother could not assert issue preclusion against Driver because Mother would not be bound by the judgment if Driver sought to rely on it See Point One There is no mutuality between Mother and Driver with respect to the prior judgment

This traditional mutuality requirement has been abandoned in most jurisdictions The Supreme Court rejected a strict mutuality requirement in Blonder-Tongue Laboratories Inc v University of Illinois Foundation 402 US 313 (1971) (non-mutual defensive collateral estoppel used by a defendant to preclude a plaintiff from relitigating a claim the plaintiff previously litigated) and Parklane Hosiery Co v Shore 439 US 322 (1979) (non-mutual offensive collateral estoppel used by a plaintiff to preclude a defendant from relitigating a claim the defendant previously litigated) In Parklane Hosiery the Court concluded (as a matter of federal preclusion law) that trial courts should have ldquobroad discretionrdquo to determine whether or not to permit a plaintiff to invoke non-mutual issue preclusion ldquoThe general rule should be that in cases where a plaintiff could easily have joined in the earlier action or where the application of offensive estoppel would be unfair to a defendant a trial judge should not allow the use of offensive collateral estoppelrdquo Id at 331

The Parklane Hosiery decision identified a number of circumstances that might make it unfair to allow a plaintiff to invoke non-mutual issue preclusion (non-mutual offensive collateral estoppel in the traditional terminology) against a defendant In particular the Parklane Hosiery court suggested that issue preclusion may not be appropriate if the plaintiff in the second action ldquocould easily have joined in the earlier actionrdquo Id Prohibiting plaintiffs from using non-mutual estoppel under such circumstances would promote judicial efficiency by encouraging plaintiffs to join the prior action It would also discourage plaintiffs from staying out of prior litigation in order to secure in effect two bites at the apple using the prior litigation offensively if the defendant loses and forcing the defendant to litigate a second time if the defendant wins the prior action

An exceptional exam answer might therefore argue that non-mutual issue preclusion should be denied on these facts Son and Mother both reside in State A since they are related they know each other well and Son was driving Motherrsquos car when the accident occurred They could have sued together and Rule 20 of the Federal Rules of Civil Procedure would have authorized joinder of their claims because those claims arose from the same transaction or occurrence and raised a common question of law or fact FED R CIV P 20(a) The facts do not suggest that Mother had any reason not to join Sonrsquos suit other than a desire to see how Sonrsquos action concluded before bringing her own claim Cf Nations v Sun Oil Co (Del) 695 F2d 933 938 (5th Cir 1983) (concluding that plaintiff ldquowas entitled to await the development of his injuries and their predictable consequencesrdquo) Because it appears that Mother may be a ldquowait-and-seerdquo plaintiff who could easily have joined the original action a trial court might disallow as a matter of discretion her use of non-mutual issue preclusion]

30

AGENCY ANALYSIS __________ (Agency I II)

ANALYSIS

Legal Problems

(1) Is the principal or the agent or both liable on contracts with a third party when the principal is an ldquoundisclosed principalrdquo

(2) Is the principal or the agent or both liable on contracts with a third party when the principal is ldquopartially disclosedrdquo or an ldquounidentified principalrdquo

(3) Is the principal or the agent or both liable on contracts with a third party for the purchase of goods when the agent exceeded his authority but the principal nonetheless accepts the goods

DISCUSSION

Summary

The agent but not the owner is liable to the basket manufacturer because the owner is an undisclosed principal and the agent acted without actual or apparent authority Both the agent and the owner however are liable on the burner contract because the owner is an unidentified principal and the agent had apparent authority to enter into that contract With respect to the solar cells contract whether the owner is liable depends upon whether a court would follow the Second or Third Restatement of Agency which take different positions on the effect of the ratification of a contract by an undisclosed principal Under either the agent would also be liable on the contract as he was a party to the contract

[NOTE The contracts that are the subject of this question are contracts for the sale of goods and therefore are governed by Article 2 of the Uniform Commercial Code Article 2 however does not contain agency rules Accordingly common law concepts of agency are applicable UCC sect 1-103(b)]

Point One (35) The agent but not the owner is liable to the basket manufacturer The agent had no actual authority to enter into the contract to buy aluminum baskets and because the owner was an undisclosed principal the manufacturer had no reason to believe that the agent had apparent authority Furthermore the manufacturer had no reason to believe that the agent was not contracting for his own benefit

An agent acting on behalf of a principal can bind the principal to contracts if the agent has either actual or apparent authority An agent has actual authority when contracting on behalf of his principal if he ldquoreasonably believes in accordance with the principalrsquos manifestations to the agent that the principal wishes the agent so to actrdquo RESTATEMENT (THIRD) OF AGENCY sect 201 (2006) Here the agent was told to buy only wicker baskets not aluminum baskets Thus when he contracted with the basket manufacturer to buy aluminum baskets he had no actual authority to do so

31

Agency Analysis

An agent acts with apparent authority ldquowhen a third party [with whom the agent acts] reasonably believes the actor has authority to act on behalf of the principal and that belief is traceable to the principalrsquos manifestationsrdquo Id sect 203 Here the owner notified basket manufacturers that she or her agent might contact them to purchase baskets but that notification did not specifically name the agent or any other person as the ownerrsquos agent Furthermore the basket manufacturer had no prior dealings with the agent or the owner or any reason to think that the agent was acting for the benefit of anyone but himself Thus there is no basis to conclude that the basket manufacturer thought that the agent had apparent authority to act for the owner

Generally when an agent acts on behalf of an undisclosed principal and the agent lacks authority to enter into the contract the agent is liable on the contract as a party to the contract but the principal is not liable This rule is consistent with the third partyrsquos expectations ldquoThe third party expected the agent to be a party to the contract because the agent presented the deal as if he were acting for himself Moreover if the third party is unaware of the principalrsquos existence the third party must be relying on the agentrsquos solvency and reliability when entering into the contractrdquo See ROBERT W HAMILTON JONATHAN R MACEY amp DOUGLAS K MOLL CORPORATIONS INCLUDING PARTNERSHIPS AND LIMITED LIABILITY COMPANIES 34 (11th ed 2010) See also RESTATEMENT (THIRD) OF AGENCY sect 603 cmt c Furthermore because the third party has no idea that the agent is acting or is seemingly acting on behalf of another there is no reason to believe that the third party would be expecting an undisclosed principal to be liable on the contract Id

Point Two (35) Because the owner is an unidentified (as opposed to undisclosed) principal both she and the agent (as a party to the contract) probably are liable on the contract with the burner manufacturer

When the agent contracted with the burner manufacturer he did not have actual authority to do so as the owner had expressly restricted the agentrsquos authority to purchase only burners with ldquowhisper technologyrdquo See Point One However the agent may have had apparent authority to buy burners without whisper technology

An agent acts with apparent authority ldquowhen a third party [with whom the agent acts] reasonably believes the actor has authority to act on behalf of the principal and that belief is traceable to the principalrsquos manifestationsrdquo RESTATEMENT (THIRD) OF AGENCY sect 203 (2006) The owner indicated that an agent might contact the burner manufacturer The notice contained no restriction regarding the type of burners that the agent was authorized to purchase The facts indicate that burner manufacturers regularly receive such notices

Although the agent told the burner manufacturer that he represented a well-known hot-air balloon operator he did not disclose the ownerrsquos name Thus the owner was a partially disclosed or unidentified principal See RESTATEMENT (SECOND) OF AGENCY sect 4(2) (1958) (using term ldquopartially disclosed principalrdquo) RESTATEMENT (THIRD) OF AGENCY sect 104(2)(c) (2006) (using term ldquounidentified principalrdquo) An agent for a partially disclosed principal may have apparent authority RESTATEMENT (SECOND) OF AGENCY sect 159 cmt e (1958) Based upon (1) the notice sent by the owner (2) the agentrsquos revelation that he was acting as an agent and (3) the fact that burner manufacturers regularly receive such notices and sell to agents the manufacturer may argue that it reasonably and actually believed that the agent was authorized to purchase burners without whisper technology The manufacturer may also argue that because the agent revealed that he was an agent his listing of the ownerrsquos address as the delivery address connects the agent to the notice given by the owner Arguably this distinguishes the burner contract from the basket

32

Agency Analysis

contract Here there is a strong case to support the conclusion that the agent had apparent authority if he did then the owner is liable to the burner manufacturer

The agent also is liable as a party to the contract because he did not fully disclose his agency relationship Although he told the burner manufacturer that he represented a well-known hot-air balloon operator he did not disclose the ownerrsquos name Generally even an authorized agent of a partially disclosed or unidentified principal is liable as a party to a contract with a third person RESTATEMENT (SECOND) OF AGENCY sect 321 (1958) (ldquounless otherwise agreedrdquo) RESTATEMENT (THIRD) OF AGENCY sect 602(2) (2006) (ldquounless the agent and the third party agree otherwiserdquo)

Point Three (30) Under the Second Restatement of Agency the owner is not liable on the contract for solar cells because the agent did not have actual or apparent authority and the owner as an undisclosed principal cannot ratify the contract Under the Third Restatement the owner could be liable as she ratified the contract Under either Restatement the agent is liable as a party to the contract

The owner is not liable to the solar cell manufacturer for breach of the contract for the solar cells because the agent had no actual or apparent authority to purchase solar cells on the ownerrsquos behalf and the owner under the Second Restatement of Agency did not ratify the contract with knowledge of the material facts Thus she is not liable as a ratifier of the contract

The facts state that the agent had authority to purchase only propane fuel tanks In addition he had no apparent authority to purchase solar cells The owner made no manifestations to the solar cell manufacturer that would lead a reasonable person in the manufacturerrsquos position to believe that the agent had the authority to bind the owner to a contract to purchase solar cells In fact the agent made no manifestations at all to the solar cell manufacturer Unlike with the basket manufacturer and the burner manufacturer the owner did not notify the manufacturer of solar cells that an agent might contact it to purchase solar cells In addition the solar cells were delivered to the agent and not to the ownerrsquos address In sum the manufacturer was unaware of any relationship between the owner and the agent As to the solar cell manufacturer the owner is an undisclosed principal There can be no apparent authority in the case of an undisclosed principal because there are no manifestations from the principal to the third person See RESTATEMENT (SECOND) OF AGENCY sect 8 cmt a (1958) (ldquothere can be no apparent authority created by an undisclosed principalrdquo) RESTATEMENT (THIRD) OF AGENCY sect 203 cmt f (2006) (ldquoapparent authority is not present when a third party believes that an interaction is with an actor who is a principalrdquo)

The owner also did not ratify the contract Although the owner used the solar cells generally a principal cannot ratify an unauthorized transaction with a third person ldquounless the one acting purported to be acting for the ratifierrdquo RESTATEMENT (SECOND) OF AGENCY sect 85(1) (1958)

The result differs under the Third Restatement which expressly rejects the Second Restatement on this issue The Restatement (Third) of Agency sect 403 (2006) states ldquoA person may ratify an act if the actor acted or purported to act as an agent on the personrsquos behalfrdquo According to comment b ldquoan undisclosed principal may ratify an agentrsquos unauthorized actrdquo Under the Restatement (Third) of Agency rule the owner probably ratified the transaction The agent clearly acted on the ownerrsquos behalf and in addition the ownerrsquos conduct in using the solar cells ldquojustifies a reasonable assumption that [she] is manifesting assent that the act shall affect [her] legal relationsrdquo See id sect 401(2)

33

Agency Analysis

The agent also is liable to the solar cell manufacturer for breach of the contract for the solar cells because he is a party to the contract The facts indicate that the agent never told the solar cell manufacturer that he represented the owner or any other principal Consequently even if the agent were authorized (which as discussed above he is not) he would be liable as a party to the contract See RESTATEMENT (SECOND) OF AGENCY sect 322 (1958) RESTATEMENT (THIRD) OF AGENCY sect 603(2) (2006) Here he has no authority or apparent authority and is liable as a party to the contract

The agent would also be liable under the Third Restatement Under Restatement (Third) of Agency sect 402(1) (2006) ratification generally relates back and the transaction is treated as if it were authorized at the time of the transaction However this does not relieve the agent of an undisclosed principal who ratifies an unauthorized transaction of liability under the ratified contract See id sect 603(2) (authorized agent for undisclosed principal is a party to the contract) and sect 403 cmt b (ldquoAn undisclosed principalrsquos ratification does not eliminate the agentrsquos liability to the third party on the transaction rdquo)

[NOTE An examinee may discuss the concept of inherent agency power This concept is recognized by the Restatement (Second) of Agency sect 8 A (1958) but the concept is not used in the Restatement (Third) of Agency (2006) Here there are no facts to support that the agent had inherent authority

As to contracts with agents for partially disclosed principals (eg the contract for the burners) the basic question is whether the acts done ldquousually accompany or are incidental to transactions which the agent is authorized to conductrdquo RESTATEMENT (SECOND) OF AGENCY

sect 161 (1958) If so the principal is bound if the other party ldquoreasonably believes that the agent is authorized to do them and has no notice that he is not so authorizedrdquo Id The purchase of burners without whisper technology was not authorized nor was it incidental to an authorized transaction Therefore there should not be inherent agency power

As to contracts on behalf of undisclosed principals (eg the other two contracts) the basic question is whether the acts done are usual or necessary in the transactions the agent is authorized to transact RESTATEMENT (SECOND) OF AGENCY sect 194 (1958) The other two contracts seem fundamentally different from the authorized transactions Therefore there should not be inherent agency power

Only minimal credit should be given for discussion of inherent agency power]

34

EVIDENCE ANALYSIS _____ (Evidence IIA VA B E F J K)

ANALYSIS

Legal Problems

(1) Is the authenticated copy of the mechanicrsquos text message relevant and admissible

(2) Is the womanrsquos question ldquoIs my scooter safe to drive for a whilerdquo relevant and admissible

(3) Is the womanrsquos testimony describing the mechanicrsquos thumbs-up relevant and admissible

DISCUSSION

Summary

The mechanicrsquos text message to the woman is relevant to whether (1) the woman lost control of the scooter due to its defective brakes (2) the woman knew that the brakes needed repair and (3) it was negligent for the woman to drive the scooter knowing that its brakes needed repair

The mechanicrsquos text message is hearsay if it is offered by the pedestrian to prove that the scooterrsquos brakes needed repair However it fits the hearsay exception for present sense impressions and probably also fits the exception for business records The mechanicrsquos text message is not hearsay if it is instead offered by the pedestrian to prove the womanrsquos state of mind (ie that she had notice that her brakes needed repair)

The womanrsquos question to the mechanic and his response are also relevant to whether the brakes caused the accident and whether the woman was negligent The question is not hearsay because the woman did not make an assertion

The mechanicrsquos thumbs-up response is nonverbal conduct intended by the mechanic as an assertion and is therefore an out-of-court statement If the woman offers the mechanicrsquos statement to prove that the scooter was actually safe to ride the womanrsquos testimony about the statement is hearsay

However the mechanicrsquos statement is not hearsay if it is offered by the woman to prove her state of mind Therefore the womanrsquos question and the mechanicrsquos response are admissible to prove the womanrsquos state of mind

Point One(a) (20) The mechanicrsquos text message to the woman should be admitted because it is relevant

Evidence is relevant if it has ldquoany tendency to make a fact more or less probable than it would be without the evidencerdquo FED R EVID 401 ldquoRelevant evidence is admissiblerdquo unless it is inadmissible pursuant to some other rule FED R EVID 402

The mechanicrsquos text message to the woman ldquoWhen you pick up your scooter you need to schedule a follow-up brake repair Wersquoll order the partsrdquo is relevant for two reasons First this evidence has some tendency to make it more probable that the brakes malfunctioned and

35

Evidence Analysis

caused the accident Second it has some tendency to make it more probable that the woman was negligent in riding her scooter after being told by the mechanic that it required further repair

Point One(b) (30) The mechanicrsquos text message fits either the hearsay exception for present sense impressions or the exception for business records or it is admissible non-hearsay

The mechanicrsquos text message is a statement under Rule 801(a) because it is ldquoa written assertionrdquo FED R EVID 801(a) The text message is hearsay if the pedestrian offers it to prove the ldquotruth of the matter asserted in the statementrdquo (ie that the scooterrsquos brakes required repair) which resulted in the woman losing control of the scooter and causing the accident FED R EVID 801(c)

However the mechanicrsquos text message fits the hearsay exception for ldquopresent sense impressionsrdquo under Rule 803(1) because it is ldquo[a] statement describing or explaining an event or condition made while or immediately after the declarant perceived itrdquo FED R EVID 803(1) Here the mechanicrsquos text message described the condition of the scooter immediately after he perceived it during the maintenance service

The mechanic is a person with knowledge of the condition of the scooter so if text messages regarding repairs were made and kept by the mechanic in the ordinary course of business this text message also fits the business records exception Under Rule 803(6) a business record is a record of an act ldquomade at or near the time by someone with knowledgerdquo and ldquothe record was kept in the course of a regularly conducted activity of a businessrdquo and ldquomaking the record was a regular practice of that activityrdquo FED R EVID 803(6)

However the text message is not hearsay if it is instead offered to prove that the woman was negligent because she rode her scooter after the mechanic told her it required repair If offered for this purpose it would not be offered for the truth of the matter asserted in the statement but to show the womanrsquos belief about the condition of the scooter (her state of mind)

Point Two (10) The womanrsquos question to the mechanic should be admitted because it is not hearsay

The womanrsquos question to the mechanic is relevant because along with the mechanicrsquos thumbs-up response (see Point Three) it has some tendency to make it more probable that the woman was not negligent andor that the scooter brakes did not malfunction and cause the accident FED R EVID 401 The womanrsquos question does not raise hearsay concerns because it is not an assertion

Hearsay is defined under Rule 801(a) as ldquoan oral assertion written assertion or nonverbal conductrdquo Although ldquoassertionrdquo is not further defined ldquoa favorite [definition] of writers in the [evidence] field for at least a century and a half [is that] the word simply means to say that something is so eg that an event happened or a condition existedrdquo 2 MCCORMICK ON

EVIDENCE sect 246 (6th ed 2006) Under this definition the womanrsquos question is not hearsay because it is not an assertion

Point Three(a) (20) The mechanicrsquos thumbs-up to the woman is a nonverbal assertion that is relevant and the womanrsquos testimony about that response is admissible

36

Evidence Analysis

Hearsay is defined under Rule 801(c) as a ldquostatementrdquo that is ldquoa personrsquos oral assertion written assertion or nonverbal conduct if the person intended it as an assertionrdquo FED R EVID 801(a) Here when the mechanic responded to the womanrsquos question (ldquoIs my scooter safe to ride for a whilerdquo) with a thumbs-up gesture the facts suggest that he intended his nonverbal conduct as an assertion that in his opinion the scooter was safe to ride

The mechanicrsquos assertion is relevant and admissible to prove that the woman was not negligent because the evidence makes it more probable that at the time of the accident she believed that the scooter was safe to ride despite the fact that the brakes required repair FED R EVID 401 Admission of the womanrsquos description of the mechanicrsquos thumbs-up for this purpose does not raise hearsay concerns because the evidence would not be offered for the truth of the matter asserted but to show the womanrsquos belief about the condition of the scooter (her state of mind)

Point Three(b) (20) The mechanicrsquos thumbs-up is relevant to determine whether the scooterrsquos brakes malfunctioned causing the accident but if offered for this purpose it is also hearsay

The mechanicrsquos nonverbal assertion is relevant to the determination of whether the scooterrsquos brakes malfunctioned causing the accident However if offered to prove the ldquotruth of the matter asserted in the statementrdquo (ie that the scooter was safe to ride for a while) it is hearsay that does not fit any hearsay exception

37

TRUSTS AND FUTURE INTERESTS ANALYSIS ____________________ (Trusts and Future Interests IC1 amp 4 G IIF)

ANALYSIS

Legal Problems

(1)(a) Was the revocable trust amendable

(1)(b) If the trust was amendable must the amendment have been executed in accordance with the state Statute of Wills in order to be valid

(2) If the trust amendment was valid does the amendment apply to the probate estate assets passing to the trust pursuant to Settlorrsquos will

(3) If the trust amendment was valid should the trust property be distributed to University

(4) If the trust amendment was not valid should the trust property be distributed to Settlorrsquos grandchild (her only heir) or held in further trust in accordance with the terms of the original trust instrument

DISCUSSION

Summary

A revocable trust is amendable even if the trust instrument does not expressly grant to the trust settlor a power to amend Both inter vivos trusts and amendments thereto are valid even though not executed in accordance with the requirements applicable to wills

Under the Uniform Testamentary Additions to Trusts Act a revocable trust may be amended at any time prior to the settlorrsquos death and the amendment applies to the disposition of assets conveyed to the trust pursuant to a will even if the will was executed prior to the date of the amendment

At Settlorrsquos death trust assets including probate assets passing to the trust under Settlorrsquos will would go to University if as is the case here the trust amendment was valid If the amendment was invalid the trust assets would continue to be held in further trust because there is no violation of the common law Rule Against Perpetuities

Point One(a) (30) Settlor retained the right to amend the inter vivos trust despite her failure to expressly reserve this power

At issue here is whether a retained power of revocation includes the power to amend sometimes referred to as the power to modify The Restatement (Second) of Trusts sect 331 cmt g provides that if a settlor has a power to revoke that retained power ordinarily includes a power to modify (amend) as well Comment g also notes that the power to amend includes both a power to withdraw trust assets and a power to ldquomodify the terms of the trustrdquo The Uniform Trust Code which provides that a power to revoke includes the power to amend is consistent with this view

38

Trusts and Future Interests Analysis

UNIF TRUST CODE sect 602 accord RESTATEMENT (THIRD) OF TRUSTS sect 63 cmt The theory is that even though a power to amend was not expressly retained by a settlor the goal of amendment assuming the power was not included in the power to revoke could easily be achieved by first revoking the trust and then creating a new trust with the same terms contemplated by the amendment To require this would put form over substance

Thus by expressly retaining the power to revoke the trust Settlor retained a power to amend the inter vivos trust despite her failure to expressly reserve this power

[NOTE Under the common law a trust is irrevocable unless the settlor expressly retains a power to revoke the trust Conversely under the Uniform Trust Code a trust is revocable unless the terms of the trust expressly provide otherwise See UNIF TRUST CODE sect 602 The Trust Codersquos position on revocation follows the minority view in the United States and is inconsistent with prior Restatements of Trusts (see Restatement (Second) of Trusts sect 330) Here the trust is revocable because Settlor expressly retained a power of revocation

The Uniform Trust Code has been adopted in 24 jurisdictions Alabama Arizona Arkansas District of Columbia Florida Kansas Maine Michigan Missouri Nebraska New Hampshire New Mexico North Carolina North Dakota Ohio Oregon Pennsylvania South Carolina Tennessee Utah Vermont Virginia West Virginia and Wyoming]

Point One(b) (10) Settlorrsquos amendment of the trust was valid despite her failure to have her signature to the trust amendment witnessed

Neither the common law nor state statutes require a trust instrument or an amendment to a trust instrument to be executed in accordance with the formalities prescribed for execution of a will Indeed an inter vivos trust that does not involve real estate can be created orally Under the Uniform Trust Code the only requirements for creating a valid inter vivos trust are intent the specification of beneficiaries and the designation of a trustee See UNIF TRUST CODE sect 402 accord RESTATEMENT (THIRD) OF TRUSTS sect 13

Here the amendment meets the requirements of both the Uniform Trust Code and the common law Thus the fact that Settlorrsquos signature was not witnessed when she signed the amendment to the trust does not make the amendment invalid

Point Two (20) Under the Uniform Testamentary Additions to Trusts Act a revocable trust may be amended at any time prior to the settlorrsquos death and the amendment applies to probate assets poured into the trust at the settlorrsquos death pursuant to the settlorrsquos will even when the will was executed prior to the date of the amendment

Historically property owned by an individual at her death passed to the individualrsquos heirs or to beneficiaries designated in a will executed with the formalities (writing signing witnessing) prescribed by state law However when a will devises property to the trustee of an inter vivos trust then the provisions of the trustmdashwhich may not have been executed in accordance with the formalities required for willsmdasheffectively determine who will receive the property Because of this possibility some early cases held that if an inter vivos trust was not executed with the same formalities required for a valid will then the trust was ineffective to dispose of probate assets poured into the trust at the settlorrsquos death pursuant to the settlorrsquos will

This line of cases has been overturned by the Uniform Testamentary Additions to Trusts Act (the Act) now Uniform Probate Code sect 2-511 Under the Act adopted in almost all

39

Trusts and Future Interest Analysis

jurisdictions a testamentary bequest to the trustee of an inter vivos trust established by the testator during his or her lifetime is valid if the trust is in writing it is identified in the testatorrsquos will and the trust instrument was executed before concurrently with or after the execution of the will Id The Act further specifies that such a bequest is valid even if the trust is amendable or revocable and that a later amendment applies to assets passing to the trust by a previously executed will

Thus because the trust amendment is valid its terms apply to assets received by Bank from Settlorrsquos estate

Point Three (10) If the trust amendment was valid then the trust assets including assets passing to the trust under Settlorrsquos will should go to University

Under the trust amendment all trust assets (including the assets of Settlorrsquos probate estate poured into the trust) pass to University The facts provide no basis for failing to comply with Settlorrsquos stated intentions

Point Four (30) If the trust amendment was invalid trust assets including assets received pursuant to Settlorrsquos will should be held in accordance with the terms of the original trust instrument because those terms do not violate the Rule Against Perpetuities

Under the dispositive terms of the original trust instrument Settlor created successive income interests in her surviving children and grandchildren with a remainder interest in her great-grandchildren Because the trust was revocable the period during which the common law Rule Against Perpetuities requires that interests vest (ie 21 years plus lives in being) began to run from the date Settlor no longer had a power of revocation (here her death) not the date on which the trust was created See JESSE DUKEMINIER STANLEY J JOHANSON JAMES LINDGREN amp ROBERT SITKOFF WILLS TRUSTS AND ESTATES 678 (7th ed 2005)

Under the common law Rule Against Perpetuities Settlorrsquos trust is thus valid At the time of Settlorrsquos death she was survived by no children one granddaughter and no great-grandchildren Because Settlor cannot have more children after her death the only income beneficiary of the trust is Settlorrsquos surviving granddaughter This granddaughter is the only person who can produce great-grandchildren of Settlor thus all great-grandchildren must of necessity be born during the lifetime of Settlorrsquos only surviving granddaughter who is a life in being The granddaughterrsquos interest vested at Settlorrsquos death and the great-grandchildrenrsquos interest will vest at the death of the granddaughter There is no need to wait the additional 21 years permitted under the Rule Thus under the common law and the statute given in the facts the nonvested interest in the great-grandchildren is valid

[NOTE Both modern wait-and-see statutes and the Uniform Statutory Rule Against Perpetuities upon which the statute in the facts is modeled provide that before using either reform to validate an otherwise invalid nonvested interest one should first determine if the nonvested interest violates the common law Rule If it does not then there is no need to reform This proposition which is applicable in all MEE user jurisdictions that have not simply abrogated the rule is tested by this problem]

40

NEGOTIABLE INSTRUMENTS ANALYSIS (Negotiable Instruments III IV V)

ANALYSIS

Legal Problems

(1)(a) What rights does a person in possession of a note that has been indorsed in blank by the payee have against the maker of the note

(1)(b) Which defenses may the maker of a note raise against a person entitled to enforce it who is not a holder in due course but is a transferee from a holder in due course

(2) What rights does a person entitled to enforce a note have against an indorser who transferred it for consideration with no warranties

(3) What rights does a person entitled to enforce a note have against a previous holder who transferred it as a gift without indorsing it

DISCUSSION

Summary

The niece is a holder of the note and is thus a person entitled to enforce it The chef the issuer of the note is obligated to pay it to the niece as the person entitled to enforce it The niece is not subject to any defense or claim of the chef relating to the improper repair of the oven because the niece has the rights of a holder in due course When the buyer bought the note from the repairman the buyer became a holder in due course of the note and thus took it free of any personal defenses the chef had against the repairman Even though the niece is not herself a holder in due course of the note the niece succeeded to the buyerrsquos rights as holder in due course and thus took free of the chefrsquos personal defenses

Because the chef refused to pay the note the niece can recover from the repairman on the repairmanrsquos obligation as indorser The niece cannot recover on the note against the buyer however because the buyer did not indorse the note (and thus incurred no indorserrsquos obligation) and the buyer did not receive any consideration for transfer of the note to the niece (and therefore made no transfer warranty)

[NOTE Although Article 9 of the Uniform Commercial Code governs the sale of promissory notes (a point that might be correctly noted by examinees) that Article does not determine the answer to any of the questions posed]

Point One(a) (20) The niece is the holder of the note and thus may enforce it against the chef who is the issuer of the note

The chef is the maker of the note and thus its issuer See UCC sectsect 3-103 3-105 The issuer of a note is obligated to pay it in accordance with its terms to a ldquoperson entitled to enforcerdquo it UCC sect 3-412 The niece is a ldquoperson entitled to enforcerdquo the note This is because the niece is the holder of the note and a holder of a note is a person entitled to enforce it UCC sect 3-301 The niece is the holder of the note because (i) the repairmanrsquos signature on the back of the note not

41

Negotiable Instruments Analysis

accompanied by words indicating a person to whom the note was made payable was a ldquoblank indorsementrdquo which had the effect of making the note a bearer instrument (ii) anyone in possession of a bearer instrument is a holder of it and (iii) the niece is in possession of the note See UCC sectsect 1-201(b)(21)(A) 3-204 and 3-205 Accordingly the chef has an obligation to the niece to pay the note in accordance with its terms and the niece may enforce that obligation

Point One(b) (40) The niece is not a holder in due course of the note but because she is a transferee from the buyer who was a holder in due course she has the same enforcement rights as the buyer Because the buyer as a holder in due course would have been able to enforce the note against the chef without being subject to defenses or claims arising from the improper repair the niece has the same rights and will not be subject to the chefrsquos defenses or claims about the repair

As noted in Point One(a) the chef has an obligation to the niece to pay the note in accordance with its terms However except against a person with the rights of a holder in due course the chef can raise any defenses or claims in recoupment that he would have if the claim on the note were an ordinary contract claim UCC sect 3-305 Thus except against a holder in due course the chef would be able to raise the improper repair as a defense or a claim in recoupment (a claim in response to the niecersquos claim)

But claims in recoupment and most defenses cannot be raised against a person with the rights of a holder in due course Against a holder in due course the chef can raise only the four ldquorealrdquo defenses listed in UCC sect 3-305(a)(1) (infancy duress lack of legal capacity or illegality that nullifies the obligation of the obligor under other law fraud in the factum discharge in insolvency proceedings) none of which is present here

The niece is not a holder in due course because she did not take the note for value See UCC sectsect 3-302(a)(2)(i) (criteria for holder in due course status) and 3-303(a) (definition of ldquovaluerdquo) But this does not mean that the niece is subject to the chefrsquos claim arising out of the improper repair The buyer was a holder in due course of the note because he took the note for value ($9500) in good faith and without notice of any facts that would have alerted him to the chefrsquos defense against the repairman UCC sect 3-302(a)(2) As a holder in due course the buyer owned the note free of the chefrsquos claim because that claim did not constitute a ldquorealrdquo defense UCC sect 3-305(b) When the buyer gave the note to the niece this constituted a ldquotransferrdquo of the note See UCC sect 3-203(a) When a note is transferred the transferee receives ldquoany right of the transferor to enforce the instrument including any right as a holder in due courserdquo UCC sect 3-203(b) Under this rule (also known as the ldquoshelter principlerdquo) the buyer transferred his freedom from the chefrsquos defenses to the niece and the niece can enforce the note free of the chefrsquos defenses

Point Two (20) Because the chef dishonored the note the niece can recover from the repairman on the repairmanrsquos obligation as indorser

The chefrsquos refusal to pay the note constituted dishonor See UCC sect 3-502 The repairman as an indorser of the note (see Point One(a)) incurred the obligations of an indorser under UCC sect 3-415(a) When a note has been dishonored one of the obligations of an indorser is to pay the amount of the note to a person entitled to enforce it Therefore the repairman is liable for the amount of the note to the niece a person entitled to enforce the note (so long as the niece gives proper notice of dishonor to the repairman)

42

Negotiable Instruments Analysis

[NOTE Because the repairman indorsed the note without warranties there are no transfer warranties UCC sect 3-416 cmt 5]

Point Three (20) The niece cannot recover on the note against the buyer as either indorser or warrantor because the buyer did not indorse the note and did not receive consideration for transferring the note to the niece

The buyer did not indorse the note and therefore did not incur the obligation of an indorser to pay the note upon dishonor

The niece cannot recover from the buyer under a transfer warranty theory because transfer warranties are made only by a person ldquowho transfers an instrument for considerationrdquo Here the buyer gave the instrument to the niece as a gift So the buyer made no transfer warranty UCC sect 3-416(a) Therefore the niece cannot recover from the buyer on that theory

43

National Conference of Bar Examiners 302 South Bedford Street | Madison WI 53703-3622 Phone 608-280-8550 | Fax 608-280-8552 | TDD 608-661-1275

wwwncbexorg e-mail contactncbexorg

  • Contents
  • Preface
  • Description of the MEE
  • Instructions
  • February 2013 Questions
    • Real Property Question
    • Contracts Question
    • Constitutional Law Question
    • Secured Transactions Question
    • Federal Civil Procedure Question
    • Agency Question
    • Evidence Question
    • Trusts and Future Interests Question
    • Negotiable Instruments Question
      • February 2013 Analyses
        • Real Property Analysis
        • Contracts Analysis
        • Constitutional Law Analysis
        • Secured Transactions Analysis
        • Federal Civil Procedure Analysis
        • Agency Analysis
        • Evidence Analysis
        • Trusts and Future Interests Analysis
        • Negotiable Instruments Analysis
            • ltlt ASCII85EncodePages false AllowTransparency false AutoPositionEPSFiles true AutoRotatePages None Binding Left CalGrayProfile (Dot Gain 20) CalRGBProfile (sRGB IEC61966-21) CalCMYKProfile (US Web Coated 050SWOP051 v2) sRGBProfile (sRGB IEC61966-21) CannotEmbedFontPolicy Error CompatibilityLevel 14 CompressObjects Tags CompressPages true ConvertImagesToIndexed true PassThroughJPEGImages true CreateJobTicket false DefaultRenderingIntent Default DetectBlends true DetectCurves 00000 ColorConversionStrategy CMYK DoThumbnails false EmbedAllFonts true EmbedOpenType false ParseICCProfilesInComments true EmbedJobOptions true DSCReportingLevel 0 EmitDSCWarnings false EndPage -1 ImageMemory 1048576 LockDistillerParams false MaxSubsetPct 100 Optimize true OPM 1 ParseDSCComments true ParseDSCCommentsForDocInfo true PreserveCopyPage true PreserveDICMYKValues true PreserveEPSInfo true PreserveFlatness true PreserveHalftoneInfo false PreserveOPIComments true PreserveOverprintSettings true StartPage 1 SubsetFonts true TransferFunctionInfo Apply UCRandBGInfo Preserve UsePrologue false ColorSettingsFile () AlwaysEmbed [ true ] NeverEmbed [ true ] AntiAliasColorImages false CropColorImages true ColorImageMinResolution 300 ColorImageMinResolutionPolicy OK DownsampleColorImages true ColorImageDownsampleType Bicubic ColorImageResolution 300 ColorImageDepth -1 ColorImageMinDownsampleDepth 1 ColorImageDownsampleThreshold 150000 EncodeColorImages true ColorImageFilter DCTEncode AutoFilterColorImages true ColorImageAutoFilterStrategy JPEG ColorACSImageDict ltlt QFactor 015 HSamples [1 1 1 1] VSamples [1 1 1 1] gtgt ColorImageDict ltlt QFactor 015 HSamples [1 1 1 1] VSamples [1 1 1 1] gtgt JPEG2000ColorACSImageDict ltlt TileWidth 256 TileHeight 256 Quality 30 gtgt JPEG2000ColorImageDict ltlt TileWidth 256 TileHeight 256 Quality 30 gtgt AntiAliasGrayImages false CropGrayImages true GrayImageMinResolution 300 GrayImageMinResolutionPolicy OK DownsampleGrayImages true GrayImageDownsampleType Bicubic GrayImageResolution 300 GrayImageDepth -1 GrayImageMinDownsampleDepth 2 GrayImageDownsampleThreshold 150000 EncodeGrayImages true GrayImageFilter DCTEncode AutoFilterGrayImages true GrayImageAutoFilterStrategy JPEG GrayACSImageDict ltlt QFactor 015 HSamples [1 1 1 1] VSamples [1 1 1 1] gtgt GrayImageDict ltlt QFactor 015 HSamples [1 1 1 1] VSamples [1 1 1 1] gtgt JPEG2000GrayACSImageDict ltlt TileWidth 256 TileHeight 256 Quality 30 gtgt JPEG2000GrayImageDict ltlt TileWidth 256 TileHeight 256 Quality 30 gtgt AntiAliasMonoImages false CropMonoImages true MonoImageMinResolution 1200 MonoImageMinResolutionPolicy OK DownsampleMonoImages true MonoImageDownsampleType Bicubic MonoImageResolution 1200 MonoImageDepth -1 MonoImageDownsampleThreshold 150000 EncodeMonoImages true MonoImageFilter CCITTFaxEncode MonoImageDict ltlt K -1 gtgt AllowPSXObjects false CheckCompliance [ None ] PDFX1aCheck false PDFX3Check false PDFXCompliantPDFOnly false PDFXNoTrimBoxError true PDFXTrimBoxToMediaBoxOffset [ 000000 000000 000000 000000 ] PDFXSetBleedBoxToMediaBox true PDFXBleedBoxToTrimBoxOffset [ 000000 000000 000000 000000 ] PDFXOutputIntentProfile () PDFXOutputConditionIdentifier () PDFXOutputCondition () PDFXRegistryName () PDFXTrapped False CreateJDFFile false Description ltlt ARA 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 BGR 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 CHS ltFEFF4f7f75288fd94e9b8bbe5b9a521b5efa7684002000410064006f006200650020005000440046002065876863900275284e8e9ad88d2891cf76845370524d53705237300260a853ef4ee54f7f75280020004100630072006f0062006100740020548c002000410064006f00620065002000520065006100640065007200200035002e003000204ee553ca66f49ad87248672c676562535f00521b5efa768400200050004400460020658768633002gt CHT ltFEFF4f7f752890194e9b8a2d7f6e5efa7acb7684002000410064006f006200650020005000440046002065874ef69069752865bc9ad854c18cea76845370524d5370523786557406300260a853ef4ee54f7f75280020004100630072006f0062006100740020548c002000410064006f00620065002000520065006100640065007200200035002e003000204ee553ca66f49ad87248672c4f86958b555f5df25efa7acb76840020005000440046002065874ef63002gt CZE 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 DAN ltFEFF004200720075006700200069006e0064007300740069006c006c0069006e006700650072006e0065002000740069006c0020006100740020006f007000720065007400740065002000410064006f006200650020005000440046002d0064006f006b0075006d0065006e007400650072002c0020006400650072002000620065006400730074002000650067006e006500720020007300690067002000740069006c002000700072006500700072006500730073002d007500640073006b007200690076006e0069006e00670020006100660020006800f8006a0020006b00760061006c0069007400650074002e0020004400650020006f007000720065007400740065006400650020005000440046002d0064006f006b0075006d0065006e0074006500720020006b0061006e002000e50062006e00650073002000690020004100630072006f00620061007400200065006c006c006500720020004100630072006f006200610074002000520065006100640065007200200035002e00300020006f00670020006e0079006500720065002egt DEU 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 ESP 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 ETI 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 FRA 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 GRE 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 HEB ltFEFF05D405E905EA05DE05E905D5002005D105D405D205D305E805D505EA002005D005DC05D4002005DB05D305D9002005DC05D905E605D505E8002005DE05E105DE05DB05D9002000410064006F006200650020005000440046002005D405DE05D505EA05D005DE05D905DD002005DC05D405D305E405E105EA002005E705D305DD002D05D305E405D505E1002005D005D905DB05D505EA05D905EA002E002005DE05E105DE05DB05D90020005000440046002005E905E005D505E605E805D5002005E005D905EA05E005D905DD002005DC05E405EA05D905D705D4002005D105D005DE05E605E205D505EA0020004100630072006F006200610074002005D5002D00410064006F00620065002000520065006100640065007200200035002E0030002005D505D205E805E105D005D505EA002005DE05EA05E705D305DE05D505EA002005D905D505EA05E8002E05D005DE05D905DD002005DC002D005000440046002F0058002D0033002C002005E205D905D905E005D5002005D105DE05D305E805D905DA002005DC05DE05E905EA05DE05E9002005E905DC0020004100630072006F006200610074002E002005DE05E105DE05DB05D90020005000440046002005E905E005D505E605E805D5002005E005D905EA05E005D905DD002005DC05E405EA05D905D705D4002005D105D005DE05E605E205D505EA0020004100630072006F006200610074002005D5002D00410064006F00620065002000520065006100640065007200200035002E0030002005D505D205E805E105D005D505EA002005DE05EA05E705D305DE05D505EA002005D905D505EA05E8002Egt HRV (Za stvaranje Adobe PDF dokumenata najpogodnijih za visokokvalitetni ispis prije tiskanja koristite ove postavke Stvoreni PDF dokumenti mogu se otvoriti Acrobat i Adobe Reader 50 i kasnijim verzijama) HUN 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 ITA 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 JPN ltFEFF9ad854c18cea306a30d730ea30d730ec30b951fa529b7528002000410064006f0062006500200050004400460020658766f8306e4f5c6210306b4f7f75283057307e305930023053306e8a2d5b9a30674f5c62103055308c305f0020005000440046002030d530a130a430eb306f3001004100630072006f0062006100740020304a30883073002000410064006f00620065002000520065006100640065007200200035002e003000204ee5964d3067958b304f30533068304c3067304d307e305930023053306e8a2d5b9a306b306f30d530a930f330c8306e57cb30818fbc307f304c5fc59808306730593002gt KOR ltFEFFc7740020c124c815c7440020c0acc6a9d558c5ec0020ace0d488c9c80020c2dcd5d80020c778c1c4c5d00020ac00c7a50020c801d569d55c002000410064006f0062006500200050004400460020bb38c11cb97c0020c791c131d569b2c8b2e4002e0020c774b807ac8c0020c791c131b41c00200050004400460020bb38c11cb2940020004100630072006f0062006100740020bc0f002000410064006f00620065002000520065006100640065007200200035002e00300020c774c0c1c5d0c11c0020c5f40020c2180020c788c2b5b2c8b2e4002egt LTH 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 LVI 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 NLD (Gebruik deze instellingen om Adobe PDF-documenten te maken die zijn geoptimaliseerd voor prepress-afdrukken van hoge kwaliteit De gemaakte PDF-documenten kunnen worden geopend met Acrobat en Adobe Reader 50 en hoger) NOR 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 POL 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 PTB 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 RUM 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 RUS ltFEFF04180441043f043e043b044c04370443043904420435002004340430043d043d044b04350020043d0430044104420440043e0439043a043800200434043b044f00200441043e043704340430043d0438044f00200434043e043a0443043c0435043d0442043e0432002000410064006f006200650020005000440046002c0020043c0430043a04410438043c0430043b044c043d043e0020043f043e04340445043e0434044f04490438044500200434043b044f00200432044b0441043e043a043e043a0430044704350441044204320435043d043d043e0433043e00200434043e043f0435044704300442043d043e0433043e00200432044b0432043e04340430002e002000200421043e043704340430043d043d044b04350020005000440046002d0434043e043a0443043c0435043d0442044b0020043c043e0436043d043e0020043e0442043a0440044b043204300442044c002004410020043f043e043c043e0449044c044e0020004100630072006f00620061007400200438002000410064006f00620065002000520065006100640065007200200035002e00300020043800200431043e043b043504350020043f043e04370434043d043804450020043204350440044104380439002egt SKY 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 SLV 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 SUO 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 SVE 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 TUR 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 UKR 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 ENU (Use these settings to create Adobe PDF documents best suited for high-quality prepress printing Created PDF documents can be opened with Acrobat and Adobe Reader 50 and later) gtgt Namespace [ (Adobe) (Common) (10) ] OtherNamespaces [ ltlt AsReaderSpreads false CropImagesToFrames true ErrorControl WarnAndContinue FlattenerIgnoreSpreadOverrides false IncludeGuidesGrids false IncludeNonPrinting false IncludeSlug false Namespace [ (Adobe) (InDesign) (40) ] OmitPlacedBitmaps false OmitPlacedEPS false OmitPlacedPDF false SimulateOverprint Legacy gtgt ltlt AddBleedMarks false AddColorBars false AddCropMarks false AddPageInfo false AddRegMarks false ConvertColors ConvertToCMYK DestinationProfileName () DestinationProfileSelector DocumentCMYK Downsample16BitImages true FlattenerPreset ltlt PresetSelector MediumResolution gtgt FormElements false GenerateStructure false IncludeBookmarks false IncludeHyperlinks false IncludeInteractive false IncludeLayers false IncludeProfiles false MultimediaHandling UseObjectSettings Namespace [ (Adobe) (CreativeSuite) (20) ] PDFXOutputIntentProfileSelector DocumentCMYK PreserveEditing true UntaggedCMYKHandling LeaveUntagged UntaggedRGBHandling UseDocumentProfile UseDocumentBleed false gtgt ]gtgt setdistillerparamsltlt HWResolution [2400 2400] PageSize [612000 792000]gtgt setpagedevice

Page 15: February 2013 MEE Questions and Analyses

TRUSTS AND FUTURE INTERESTS QUESTION

Ten years ago Settlor validly created an inter vivos trust and named Bank as trustee The trust instrument provided that Settlor would receive all of the trust income during her lifetime The trust instrument further provided that

Upon Settlorrsquos death the trust income shall be paid in equal shares to Settlorrsquos surviving children for their lives Upon the death of the last surviving child the trust income shall be paid in equal shares to Settlorrsquos then-living grandchildren for their lives Upon the death of the survivor of Settlorrsquos children and grandchildren the trust corpus shall be distributed in equal shares to Settlorrsquos then-living great-grandchildren

The trust instrument expressly specified that the trust was revocable but it was silent regarding whether Settlor could amend the trust instrument

Immediately after creating the trust Settlor validly executed a will leaving her entire estate to Bank as trustee of her inter vivos trust to ldquohold in accordance with the terms of the trustrdquo

Five years ago Settlor signed an amendment to the inter vivos trust The amendment changed the disposition of the remainder interest specifying that all trust assets ldquoshall be paid upon Settlorrsquos death to Universityrdquo Settlorrsquos signature on this amendment was not witnessed

A state statute provides that any trust interest that violates the common law Rule Against Perpetuities ldquois nonetheless valid if the nonvested interest in the trust actually vests or fails to vest either (a) within 21 years of lives in being at the creation of the nonvested interest or (b) within 90 years of its creationrdquo

Recently Settlor died leaving a probate estate of $200000 She was survived by no children one granddaughter (who would be Settlorrsquos only heir) and no great-grandchildren The granddaughter has consulted your law firm and has raised four questions regarding this trust

1 Was Settlorrsquos amendment of the inter vivos trust valid Explain

2 Assuming that the trust amendment was valid do its provisions apply to Settlorrsquos probate assets Explain

3 Assuming that the trust amendment was valid how should trust assets be distributed Explain

4 Assuming that the trust amendment was invalid how should trust assets be distributed Explain

11

NEGOTIABLE INSTRUMENTS QUESTION

A chef entered into a contract with a repairman pursuant to which the repairman agreed to repair the chefrsquos commercial oven for $10000 The repairman agreed to accept as payment a negotiable promissory note for $10000 payable two months after its issuance

After the repairman worked on the oven the chef gave him a $10000 note as payment for the work As agreed the note was signed by the chef as maker was payable to the order of the repairman was payable in two months and fulfilled all criteria for negotiability

The next day the repairman sold the note to a buyer for $9500 To effectuate the sale the repairman wrote ldquono warrantiesrdquo on the back of the note signed his name immediately below that and handed the note to the buyer The buyer bought the note in good faith and without knowledge of any facts relating to the work that the repairman had performed for the chef

Later the buyer gave the note to his niece as a gift To effectuate the gift the buyer handed the note to the niece but did not indorse it

Shortly thereafter the chef discovered that the repair work had been done improperly and the oven still did not function correctly The chef tried repeatedly to get the repairman to return to correct the repair work but the repairman ignored all the chefrsquos calls

On the notersquos due date the niece contacted the chef and demanded that he pay the amount of the note to her The chef refused and told the niece that he would not pay the note because the repairman did not properly repair the oven

1 What are the niecersquos rights against the chef Explain

2 What are the niecersquos rights against the repairman Explain

3 What are the niecersquos rights against the buyer Explain

12

February 2013 MEE

ANALYSES

Real Property Contracts

Constitutional Law Secured Transactions

Federal Civil Procedure Agency

Evidence Trusts and Future Interests

Negotiable Instruments

REAL PROPERTY ANALYSIS (Real Property ID1a 4 amp 5)

ANALYSIS

Legal Problems

(1) Does the tenant have a defense to the landlordrsquos action for unpaid rent based on constructive eviction

(2) Does the tenant have a defense to the landlordrsquos action for unpaid rent based on the tenantrsquos surrender of the premises

(3) What if anything may the landlord recover from the tenant for the period after the tenant vacated the building

DISCUSSION

Summary

Under the common law the tenant does not have a defense to the landlordrsquos action for unpaid rent based on constructive eviction Constructive eviction is based on the tenant proving that (1) the landlord breached a duty to the tenant (2) the breach caused a loss by the tenant of the substantial use and enjoyment of the premises (3) the tenant gave the landlord adequate notice and opportunity to repair and (4) the tenant vacated the leased premises Here there was no constructive eviction because although the tenant vacated and gave the landlord adequate notice the landlord breached no express or implied duty to the tenant to repair the premises

The tenant does not have a defense based on the landlordrsquos acceptance of his surrender of the premises a landlordrsquos retention of keys does not constitute an acceptance of the tenantrsquos surrender unless the landlord so intended and here the landlordrsquos statements to the tenant at the time of the surrender of the keys do not evidence the intent to accept the tenantrsquos surrender

Under the common law a landlord has no duty to mitigate damages but also cannot sue for rents due in the future Under this approach the landlord can sue only for past-due rents Using this approach on November 1 the landlord could recover all the rent past due (ie rent for September and October) but could not recover for rents due in the future However some courts have authorized recovery for future rent minus the fair market rental value of the premises It is thus possible that the landlord could recover damages equal to the amount of rent due from September 1 to the end of the six-year lease term ($180000) minus the propertyrsquos fair-market rental value over that same period

Point One (45) The tenant was not constructively evicted because the landlord had no duty to repair the commercial premises that were the subject of the lease

The landlord and the tenant entered into a term-of-years lease because the lease specified both a beginning and an ending date HERBERT HOVENKAMP amp SHELDON F KURTZ THE LAW OF

PROPERTY 256 (5th ed 2001) Although a term-of-years lease normally cannot be terminated by the tenant prior to the end of the term a tenant may terminate a term-of-years lease if the tenant

15

Real Property Analysis

is constructively evicted See id at 286ndash88 Typically as here a claim of constructive eviction is made as a defense to a landlordrsquos action for damages or unpaid rent

In order to establish a constructive eviction the tenant must prove that the landlord breached a duty to the tenant such as a duty to repair and that the landlordrsquos breach caused a loss of the substantial use and enjoyment of the premises The tenant must also show that he gave the landlord notice adequate to permit the landlord to meet his duty to the tenant and that the tenant vacated the leased premises Id see also JOHN G SPRANKLING UNDERSTANDING

PROPERTY LAW sect 1704 (2d ed 2007) Under the common law there was no implied duty on the part of a landlord to repair

leased premises such a duty arose only if expressly set forth in the lease SPRANKLING supra sect 1702[B] Here the written lease contained no term requiring the landlord to repair the air-conditioning Even if the conversation created a lease term that the building had air-conditioning that itself should not create a duty for the landlord to repair it

Over the past several decades courts have generally implied a duty to repair in residential leases either as part of a revised constructive eviction doctrine or based on an implied warranty of habitability JOSEPH W SINGER PROPERTY 469ndash70 (3d ed 2010) This shift has been justified based on the economic disparity between the typical landlord and tenant as well as the fact that residential tenants generally lack both the authority to authorize repairs to common areas of a building and the incentive to make repairs that will ultimately benefit the landlord

However courts have been more reluctant to imply a duty to repair in commercial leases a context in which the tenant is often a valuable business and in a better position to assess and make repairs than is the landlord But see eg Davidow v Inwood North Professional Group 747 SW2d 373 (Tex 1988) When courts have implied a duty to repair in a commercial lease it is typically when the repair has been mandated by public authorities and involves work so substantial that it would not ordinarily fall within the tenantrsquos common law repair duty andor the value of the repair would primarily inure to the landlordrsquos reversionary interest See Brown v Green 884 P2d 55 (Cal 1994) Eugene L Grant et al The Tenant as Terminator Constructive Eviction in Modern Commercial Leases 2 THE COMMERCIAL PROPERTY LEASE ch 15 (ABA 1997) Some courts have also permitted constructive eviction claims by commercial tenants of office buildings based on repairs required in common areas of the building See id Echo Consulting Services Inc v North Conway Bank 669 A2d 227 (NH 1995)

Here the tenant is the owner of a valuable manufacturing operation and is the exclusive occupant of the building the repair has not been mandated by public authorities and the repair is not structural To the contrary the repair involves a feature of the building of unusual importance in the tenantrsquos manufacturing operation and the tenant is likely far more knowledgeable than the landlord about the air-conditioning specifications necessary for the manufacture of the tenantrsquos product

Based on these facts it is unlikely that a court will find that the tenant in this case was constructively evicted Although the tenant can show that he gave adequate notice to the landlord of the air-conditioning malfunction and vacated the premises the lease was commercial and it did not contain any promises or covenants by the landlord except a covenant of quiet enjoyment a covenant of quiet enjoyment does not entail any repair obligations

[NOTE An examineersquos conclusion is less important than his or her demonstrated awareness of the elements of constructive eviction and the need to imply a repair duty for such a defense to be viable here Although the implied warranty of habitability is not available to this tenant Texas Minnesota and Massachusetts imply a warranty of suitability in commercial leases in limited circumstances and an examinee might argue that this warranty should apply

16

Real Property Analysis

here If an examinee concludes that this warranty applies he or she should discuss the other requirements for constructive eviction

If the examinee wrongly concludes that the first element for a constructive eviction has been met the examinee will then have to discuss the remaining three elements in order to conclude that the tenant can claim constructive eviction The tenant would have a strong argument that the second elementmdashsubstantial interference with the use and enjoyment of the premisesmdashalso is met As indicated above the landlord was aware that a functioning air-conditioning system was vital to the tenantrsquos manufacturing operations The facts further indicate that the system had failed three times in the past few months The landlord may try to argue that the malfunctions did not substantially interfere with the tenantrsquos use of the premises because the malfunctions caused the temperature to climb above 81 degrees for only a short period of timemdash 3 hours 6 hours and 10 hours respectivelymdashon each occasion The tenant will argue however that the landlord was aware that the tenantrsquos manufacturing operations could tolerate temperatures above 81 degrees for no more than 6 hours The final malfunction exceeded that limit destroying $150000 worth of the tenantrsquos products

The tenant would also have a strong argument that the third element is met notice and opportunity to cure The tenant notified the landlord of the problem immediately upon the systemrsquos first malfunction and did so again when it malfunctioned a second time and then a third time The landlord might argue that there was insufficient time to cure the problem because the system corrected itself within a few hours on the first and second times Although the malfunction lasted more than 10 hours the third time the landlord might argue that the time period was insufficient to get a repair person on the premises A court would be likely to find this argument unpersuasive however because the landlord could have attempted to correct the problem after the first and second malfunctions

Assuming that the landlord was given sufficient notice and opportunity to cure a court would be likely to conclude that the tenant also satisfied the final element of vacating the premises within a reasonable time The landlord might argue that the tenant remained in the premises for almost four months after the air conditioning first failed which would suggest that the problem was not so severe as to have constructively evicted the tenant The tenant will argue however that he gave the landlord three months to cure the problem after the first two malfunctions threatened (but did not actually harm) his operations The tenant then moved out shortly after the final malfunction caused temperatures to exceed the tolerance levels of his manufacturing operations]

Point Two (10) The landlord did not accept the tenantrsquos surrender of the lease

When a tenant wrongfully moves from leased premises with the intent to terminate the lease the landlord may either accept the tenantrsquos surrender of the premises and terminate the lease or hold the tenant to the terms of the lease See HOVENKAMP amp KURTZ supra at 295ndash96 Here the tenantrsquos only basis for the claim that the landlord accepted his surrender is the landlordrsquos retention of the keys Many courts have considered whether a landlordrsquos retention of keys delivered by a tenant constitutes acceptance of surrender The weight of the case law holds that retention of the keys alone does not constitute acceptance of surrender without other evidence showing that the landlord intended to accept the surrender See generally 49 AM JUR 2d Landlord and Tenant sect 213

Here the landlordrsquos note saying ldquoI repeat the air-conditioning is not my problem You have leased the building and you should fix itrdquo strongly suggests that the landlord did not intend

17

Some courts have rejected the no-mitigation-of-damages rule based on efficiency concerns and societyrsquos interest in assuring that resources remain in the stream of commerce rather than lying vacant see id at 464ndash65 and allow landlords to sue tenants who have wrongfully terminated a lease for damages equal to the difference between the unpaid rent due under the lease and the propertyrsquos fair market rental value Other courts have abandoned the no-recovery-for-future-rent rule These courts responding to the fact that a tenant may well disappear or be judgment-proof by the time a lease term is concluded have allowed a landlord to collect damages equal to the value of rent over the entire lease term minus the propertyrsquos fair rental value when a tenant has wrongfully terminated a lease and unequivocally shown an intention not to return to the premises or pay future rent Under this approach a landlord receives approximately the same amount he would have received were there a duty to mitigate damages See Sagamore Corp v Willcutt 180 A 464 (Conn 1935)

Real Property Analysis

to accept the tenantrsquos surrender The tenant might argue that the landlordrsquos failure to make a similar statement when the keys were sent to her a second time and she retained them evidences a change of heart However it is likely that a court would find that the landlordrsquos retention of the keys represented a decision to safeguard the keys not to accept the tenantrsquos surrender

[NOTE An examinee should receive credit for arguing the other way with a well-reasoned argument]

Point Three (45) Under the common law the landlord had no duty to mitigate damages Additionally a landlord was not entitled to recover unpaid rents due in the future but was only entitled to recover rents in arrears at the time of the commencement of the suit Applying the common law here the landlord could recover $5000 the amount of rents due at the commencement of the suit ($2500 for September and the same for October) Today some courts allow the landlord under certain circumstances to sue the tenant for damages (not rent) equal to the difference if any between the unpaid promised rent for the balance of the term (here $175000) and the propertyrsquos fair rental value for the balance of the term

Under the common law because a lease was viewed as a conveyance instead of a contract a landlord had no duty to mitigate damages resulting from a tenantrsquos wrongful termination of a lease A landlord could thus recover the full value of rents that were due and unpaid at the time of the suit However under the common law a landlord could not sue a tenant for rents due in the future because there was always a possibility that the tenant might pay the rent when it was due See SINGER supra at 462 Thus using the common law approach on November 1 the landlord could only recover the full value of the two monthsrsquo rent actually due and unpaid ie $5000 for September and October

Here because the tenant returned the keys to the landlord and said ldquoI will not be returning to the building or making further rent paymentsrdquo the landlord could establish abandonment and an intention not to return It is thus possible that the landlord might recover damages in the amount of $5000 (for the months of September and October) plus the present value of $175000 minus the fair market rental value of the property over the remaining months of the lease

18

CONTRACTS ANALYSIS ____ (Contracts II IVE)

ANALYSIS

Legal Problems

(1) What was the legal effect of the sailorrsquos October 31 letter to the builder

(2)(a) What was the legal effect of the builderrsquos November 25 response to the sailorrsquos October 31 letter

(2)(b) What was the legal effect of the sailorrsquos refusal to take and pay for the boat on December 15

DISCUSSION

Summary

This is a sale of goods governed by the Uniform Commercial Code Because the sailor had reasonable grounds for insecurity about the builderrsquos ability to deliver the boat in a timely manner when the sailor learned about the strike on October 31 the sailor was legally justified in sending the letter to the builder seeking adequate assurance of the builderrsquos performance pursuant to the contract The builderrsquos failure to provide such assurance within a reasonable time operated as a repudiation of the contract However the builder was free to retract the repudiation before the sailor either cancelled the contract or materially changed position in reliance on the builderrsquos repudiation The builder retracted the repudiation when he informed the sailor that the workers were back and that the boat would be delivered by the date stipulated in the partiesrsquo contract Because the sailor had taken no action in response to the original repudiation he no longer had the right to cancel the contract with the builder The sailorrsquos subsequent statement that ldquoour contract is overrdquo may have constituted repudiation by the sailor In any event when the sailor failed to perform on December 15 that constituted breach

Point One (35) Because the sailor had reasonable grounds for insecurity with respect to the builderrsquos performance the sailorrsquos letter to the builder was a justified demand seeking assurance of the builderrsquos performance under the contract failure of the builder to provide such assurance within a reasonable time constituted repudiation of the contract

The sailor was legally justified in sending the letter to the builder on October 31 Contract parties are entitled to expect due performance of contractual obligations and are permitted to take steps to protect that expectation UCC sect 2-609 states that ldquo[w]hen reasonable grounds for insecurity arise with respect to the performance of either party the other may in writing demand adequate assurance of due performance rdquo Here the sailor learned on October 31 that the builderrsquos workers were on strike This gave the sailor reasonable grounds for insecurity about the builderrsquos ability to complete performance on time and thus gave the sailor the right to seek adequate assurance from the builder Because the sailorrsquos demand for assurance was justified the builder was required to provide assurance that was adequate under the circumstances within a reasonable time (not to exceed 30 days) or be held to have repudiated the contract UCC sect 2-609(4)

19

Contracts Analysis

Point Two(a) (30) The builder did not within a reasonable time provide the sailor adequate assurance of due performance this failure to provide assurance constituted a repudiation of the contract

Because the sailor with legal justification (see Point One) demanded from the builder assurance of due performance the builderrsquos failure to provide such assurance within a reasonable time was a repudiation of their contract See UCC sect 2-609(4) (ldquoAfter receipt of a justified demand[] failure to provide within a reasonable time not exceeding thirty days assurance of due performance is a repudiation of the contractrdquo) On October 31 the sailor requested that the builder provide adequate assurance regarding the completion of the boat by December 15 The builder did not respond to the sailorrsquos letter until November 25mdashnearly a month later Even if that response had been given in a reasonable time it nonetheless did not provide assurance of due performance It simply stated ldquoIrsquom sorry about the strike but it is really out of my hands I hope we settle it soon so that we can get back to workrdquo Therefore the builderrsquos November 25 response did not provide adequate assurance in response to the sailorrsquos justified request Thus the builder had repudiated the contract

Point Two(b) (35) Although the builder repudiated the contract with the sailor the builder probably retracted that repudiation on December 3 and the sailor was no longer entitled to cancel their contract Thus the sailorrsquos failure to perform the sailorrsquos obligations under the contract constituted a breach

The builderrsquos failure to provide adequate assurance of performance constituted a repudiation of their contract (see UCC sect 2-609(4)) but the builder was free to retract that repudiation until the sailor cancelled the contract or materially changed his position or indicated by communication or action that the sailor considered the repudiation to be final See UCC sect 2-611(1) (ldquoUntil the repudiating partyrsquos next performance is due he can retract his repudiation unless the aggrieved party has since the repudiation cancelled or materially changed his position or otherwise indicated that he considers the repudiation finalrdquo)

Here the facts state that before the builderrsquos December 3 telephone call to the sailor the sailor did nothing in response to the builderrsquos repudiation such as contracting with a third party for a boat The builderrsquos December 3 call informing the sailor that the boat would be timely delivered probably constituted a retraction of the repudiation because it clearly indicated to the sailor that the builder would be able to perform UCC sect 2-611(2) Thus after being so informed the sailor did not have the right to treat their contract as cancelled UCC sect 2-611(3) Accordingly the sailorrsquos failure to perform the sailorrsquos obligations under the contract by taking the boat and paying for it constituted a breach of the contract

20

CONSTITUTIONAL LAW ANALYSIS (Constitutional Law IVA F2b amp e)

ANALYSIS

Legal Problems

(1) Does AutoCorsquos operation of a ldquocompany townrdquo result in its actions counting as those of the state for purposes of constitutional analysis

(2) Does the expulsion of a schoolchild for failure to recite the Pledge of Allegiance violate the First Amendment as applied through the Fourteenth Amendment

(3) Does the arrest of a pamphleteer in connection with violation of an anti-littering rule where the littering is done by the recipients of leaflets distributed by the pamphleteer violate the First Amendment as applied through the Fourteenth Amendment

DISCUSSION

Summary

The First Amendment as applied through the Fourteenth Amendment applies only to state action It does not typically govern private actors However courts have found state action where the private actor has exercised a ldquopublic functionrdquo such as running a privately owned ldquocompany townrdquo as AutoCo has done here Thus First Amendment protections apply By requiring the son to participate in a mandatory Pledge of Allegiance ceremony AutoCo has compelled the expression of political belief in violation of the First Amendment as applied through the Fourteenth Amendment The fatherrsquos arrest in connection with breaching the anti-litter rule also violated the First Amendment as applied through the Fourteenth Amendment Although state actors can regulate the incidental effects of speech on the public streets on a content-neutral basis this power is limited and cannot extend to punishing a distributor of literature because of littering by third parties

Point One (30) AutoCorsquos operation of a company town (including a school) makes it a state actor under the public function strand of the state action doctrine

The individual rights protections of the Constitution apply only where there is ldquostate actionrdquomdash either direct action by the government or some action by a private party that is fairly attributable to the government As a general rule the actions of a private company like AutoCo or of a private school like the school operated by AutoCo would not constitute state action and the protections of the Constitution (in this case the First Amendment) would not apply

However there are situations in which the actions of a private actor are attributed to the state One such situation is when the private actor undertakes a public function There are not many bright-line rules in the Supreme Courtrsquos state action doctrine but one of them is this Where a private actor undertakes a ldquopublic functionrdquo the Constitution applies to those actions Where a corporation operates a privately owned ldquocompany townrdquo that provides essential services typically provided by a state actor the public function doctrine applies and the Constitution

21

Constitutional Law Analysis

binds agents of the town as if they were agents of the government See eg Marsh v Alabama 326 US 501 (1946) Here AutoCo does more than own the town it provides security services fire protection sanitation services and a school Thus the actions of AutoCo constitute state action and are governed by the Fourteenth Amendment

Point Two (35) The sonrsquos expulsion for failure to recite the Pledge of Allegiance violates the First Amendment as applied through the Fourteenth Amendment as a compelled expression of political belief

As explained in Point One the First Amendment applies to the school as a state actor Although children in public schools (and in schools subject to the First Amendment like

the Oakwood school) have some First Amendment rights Tinker v Des Moines Independent Community School District 393 US 503 506 (1969) schools have greater leeway to regulate the speech of students and teachers than the state would have outside the school context Hazelwood School Dist v Kuhlmeier 484 US 260 (1988) Morse v Frederick 551 US 393 (2007) However the Supreme Court has long held that public schools may not force their students to participate in a flag salute ceremony when it offends the political or religious beliefs of the students or their families West Virginia Board of Educ v Barnette 319 US 624 (1943) (invalidating a mandatory public school flag salute ceremony) see also Wooley v Maynard 430 US 705 (1977) (invalidating compelled expression of political belief on state-issued license plates)

In this case the school requires its students to participate in a flag salute and Pledge of Allegiance ceremony and punishes them when they refuse to participate Pursuant to this policy the school has expelled the son This expulsion violates the First Amendment ban on compelled expression

Point Three (35) Because the father was distributing leaflets in a traditional public forum his trespass arrest violated the First Amendment as applied through the Fourteenth Amendment

As explained in Point One AutoCo is treated as a state actor Thus Oakwoodrsquos commercial district is treated as government-owned property for purposes of the First Amendment Thus the leafleting here is subject to the First Amendment because it is an expressive activity Schneider v State of New Jersey Town of Irvington 308 US 147 (1939) When expression takes place on government-owned property government regulation of the expression is assessed under the public forum doctrine Public streets and sidewalks have long been held to be the classic example of a ldquotraditional public forumrdquo open to the public for expression Hague v CIO 307 US 496 515ndash16 (1939) Because the father was distributing leaflets while standing on a street corner in the commercial district his expressive activity occurred in a traditional public forum

When a state tries to regulate expressive activity in a traditional public forum it is prohibited from doing so based on the expressive activityrsquos content unless its regulation is narrowly tailored to achieve a compelling governmental interest (ldquostrict scrutinyrdquo) In this case however AutoCo is regulating the fatherrsquos expressive activity on the ostensibly neutral ground that his expressive activity has produced litter and made the street unsightly When a state tries to regulate expressive activity without regard to its content intermediate scrutiny applies Under intermediate scrutiny the true purpose of the regulation may not be the suppression of ideas (if so then strict scrutiny applies) the regulation must be narrowly tailored to achieve a significant

22

Constitutional Law Analysis

governmental interest and it must leave open ample alternative channels for expressive activity Ward v Rock Against Racism 491 US 781 791 (1989)

Here the application of the ordinance to the father will fail for two reasons First the Supreme Court has held that the governmentrsquos interest in keeping the streets clean is insufficient to ban leafleting in the public streets as the government power to regulate with incidental effects on public sidewalk speech is very limited See eg Schneider 308 US at 162 (leafletinglittering) Second the regulation (a blanket ban on distribution that results in littering) is not narrowly tailored to protect expression A narrowly tailored alternative would be prosecution only of people who litter Moreover the effect of the littering rule is likely to be a ban on all leafleting thus eliminating an entire class of means of expression This raises the possibility that there are not ldquoample alternative channels of communicationrdquo open to the father as required under the Courtrsquos standard of review for content-neutral regulation of speech

[NOTE Some examinees might argue that this is a ldquotime place and mannerrdquo restriction and that AutoCo might have greater latitude to regulate the public sidewalks under this theory This argument is incorrect for two reasons First the Supreme Court has held that the power to regulate speakers through littering laws is very limited for the reasons given and in the cases cited above But more generally a ldquotime place and mannerrdquo restriction involves the shifting of speech from one time and place to another or to another manner here there is no shifting but a direct punishment for expressive activity (albeit one couched in content-neutral terms) In addition some examinees might read the ordinance to be in effect a total ban on leafleting since most leafleting will produce some litter Those examinees might note that the Court has required total bans on an entire mode of expression to satisfy strict scrutiny and analyze the fatherrsquos prosecution here accordingly See United States v Grace 461 US 171 177 (1983) (invalidating ban on display of signs on public sidewalks surrounding US Supreme Court ldquo[a]dditional restrictions such as an absolute prohibition on a particular type of expression will be upheld only if narrowly drawn to accomplish a compelling governmental interestrdquo)]

23

SECURED TRANSACTIONS ANALYSIS (Secured Transactions IID E IVA B C)

ANALYSIS

Legal Problems

(1) Is a purchase-money security interest in consumer goods perfected even though there has been no filing of a financing statement

(2) Does a person who buys consumer goods for personal use take those goods free of a prior perfected purchase-money security interest in the goods

(3) Does a person who receives consumer goods as a gift take those goods subject to a prior perfected security interest in them

DISCUSSION

Summary

The retailerrsquos security interest in the bicycles was perfected even though no financing statement was filed because it was a purchase-money security interest in consumer goods A purchase-money security interest in consumer goods is automatically perfected upon attachment

The buyer is not subject to the retailerrsquos security interest in the bicycle that the buyer bought from the man Because the bicycle was consumer goods in the hands of the man and the retailer never filed a financing statement covering the bicycle the retailerrsquos security interest is not effective against someone like the buyer who bought the bicycle for value without knowledge of the retailerrsquos security interest and for personal use

On the other hand the retailerrsquos security interest continues in the bicycle given to the friend because the friend did not give value for the bicycle or buy it in the ordinary course of business

Point One (35) The retailerrsquos security interest in the bicycles attached on June 1 Because this interest was a purchase-money security interest in consumer goods it was automatically perfected when it attached

The retailerrsquos security interest in the bicycles attached on June 1 when the man bought the bicycles (acquiring rights in the collateral) signed a security agreement containing a description of the collateral and received value from the retailer (by being given credit with which to purchase the bicycles) UCC sect 9-203(a) amp (b)

Despite the retailerrsquos failure to file a financing statement its security interest was perfected Pursuant to UCC sect 9-309(1) a security interest is automatically perfected upon attachment if the goods are ldquoconsumer goodsrdquo and the security interest is a ldquopurchase-money security interestrdquo

In this case the bicycles sold by the retailer to the man were consumer goods at the time of sale The bicycles were ldquogoodsrdquo because they were ldquomovable when a security interest

24

Secured Transactions Analysis

attachesrdquo UCC sect 9-102(a)(44) They were also consumer goods because they were ldquobought for use primarily for personal family or household purposesrdquo UCC sect 9-102(a)(23) The retailerrsquos security interest in these consumer goods was also a ldquopurchase-money security interestrdquo A purchase-money security interest is an interest that secures a debt that was incurred in order to ldquoenable the debtor to acquire rights in or the use of the collateralrdquo UCC sect 9-103(a) (b)(1) Here the man incurred an obligation to the retailer to purchase the bicycles so the security interest he gave the retailer to secure that obligation was a purchase-money security interest

Because the retailerrsquos security interest was a purchase-money security interest in consumer goods it was automatically perfected on June 1 when the interest attached to the bicycles

Point Two (35) The buyer took the bicycle free of the retailerrsquos security interest because (i) the retailer did not file a financing statement covering the bicycle (ii) the bicycle was ldquoconsumer goodsrdquo and (iii) the buyer bought the bicycle for value without knowledge of the retailerrsquos security interest and for personal use

A security interest continues in collateral even after a sale or other disposition of that collateral unless the creditor authorized the disposition ldquofree of the security interestrdquo or another Article 9 exception applies UCC sectsect 9-201(a) and 9-315(a)(1)

However a buyer of goods like the buyer here can take free of a prior security interest in those goods under certain circumstances See UCC sectsect 9-317(b) (buyers who give value and receive delivery of goods without knowledge of an unperfected security interest in the goods) and 9-320(a) amp (b) (buyer in ordinary course of business buyer of consumer goods in a consumer-to-consumer transaction who gives value) In this case the retailerrsquos security interest was perfected when the buyer purchased the bicycle so UCC sect 9-317(b) does not protect the buyer The buyer also is not a protected ldquobuyer in ordinary course of businessrdquo because he did not purchase from a person who is in the business of selling bicycles See UCC sect 1-201(b)(9)

The buyer can however qualify for the protection of UCC sect 9-320(b) That section provides that a buyer of goods from a person who used them for personal family or household purposes takes free of a perfected security interest in the goods if (1) the buyer had no knowledge of the security interest (2) the buyer gave value for the goods (3) the buyer purchased the goods primarily for personal family or household purposes and (4) the purchase occurred before the filing of a financing statement covering the goods

The buyer met all of these criteria The man used the bicycle for personal purposes The buyer purchased the bicycle from the man and the buyer had no knowledge of the retailerrsquos security interest The buyer gave value ($400) for the bicycle and he bought it ldquoprimarily for personal family or household purposesrdquo as he planned to use it for recreation which is a personal rather than a business use Finally no financing statement had been filed Therefore under UCC sect 9-320(b) the buyer took free of the retailerrsquos security interest

Point Three (30) The retailerrsquos security interest continues in the bicycle that the man gave to the friend Thus the retailer can recover the bicycle from the friend because the friend did not give value for the bicycle or buy it in the ordinary course of business

25

Secured Transactions Analysis

As noted in Point Two the retailer did not authorize the man to dispose of the bicycle Consequently the retailerrsquos security interest continued in the bicycle even after the man transferred ownership of the bicycle to the friend See UCC sectsect 9-201(a) and 9-315(a)(1) The retailerrsquos security interest in the bicycle will be effective against the friend unless some other provision of Article 9 allows the friend to take the bicycle free of that security interest

Unfortunately for the friend there is no Article 9 provision that allows him to take free of the retailerrsquos interest The friendrsquos basic problem is that he is not a buyer of the bicyclemdashhe received the bicycle as a gift and did not give value for it Thus the friend is not protected by any of the applicable exceptions See UCC sectsect 9-317(b) (protecting buyers who give value for goods subject to an unperfected security interest) 9-320(a) (protecting buyers in ordinary course of business) and 9-320(b) (protecting buyers of consumer goods who give value)

In short the retailerrsquos security interest continues in the bicycle that the man gave to the friend The friend took the bicycle subject to that security interest

26

FEDERAL CIVIL PROCEDURE ANALYSIS (Federal Civil Procedure VIE)

ANALYSIS

Legal Problems

(1) Does a judgment in a prior action preclude a nonparty from suing the same defendant on a closely related claim when the nonparty and the original plaintiff are in a family relationship

(2) Does a judgment rendered in an earlier action preclude a nonparty from litigating an issue that was actually decided in the first suit

(3) May a nonparty to an earlier action invoke the judgment in that action to preclude a party to the prior action from relitigating an issue that the party had a full and fair opportunity to litigate in the earlier action

DISCUSSION

Summary

Pursuant to the doctrines of claim preclusion (res judicata) and issue preclusion (collateral estoppel) a judgment is binding on the parties thereto In the absence of privity nonparties to a prior suit cannot be bound by a judgment rendered in their absence Thus in the absence of privity a nonparty to the first suit is not precluded from presenting her claim in a second suit even if it is factually related to the claims and defenses presented in the first suit nor is she bound by determinations of issues made in the first suit A family relationship without more does not support a finding of privity For this reason Mother as a nonparty is not bound by the judgment in the Son-Driver action She may bring her separate claim for damage to her car and she is not precluded from litigating the question of whether she was negligent in the maintenance of her car

Driver on the other hand could be precluded from relitigating the issue of her negligence pursuant to the doctrine of non-mutual issue preclusion (also called non-mutual offensive collateral estoppel) which allows a nonparty to a prior action to invoke issue preclusion to prevent a party to that prior action from relitigating determinations of issues made therein However Mother may be prevented from invoking non-mutual collateral estoppel in this case because she could easily have joined her claim in the prior action but did not do so

[NOTE Federal common law governs the preclusive effect of a judgment rendered by a federal court sitting in diversity See Semtek Intrsquol Inc v Lockheed Martin Corp 531 US 497 508 (2001) But the Semtek Court concluded that federal common law in this context incorporates the preclusion law of the state in which the rendering federal court sits (unless the state law is incompatible with federal interests) id at 508ndash09 Thus State Arsquos preclusion law determines the preclusive effect of the judgment rendered in Sonrsquos suit against Driver The problem says that State A preclusion law is identical to federal preclusion law so the following analysis utilizes general principles of preclusion drawn from Supreme Court case law (announcing federal preclusion rules) and the Restatement (Second) of Judgments]

27

Federal Civil Procedure Analysis

Point One (35) Under the doctrine of claim preclusion the judgment rendered in the first action does not preclude Mother a nonparty from suing Driver for the damage to her car because the judgment binds only parties or those in privity with them and Mother and Son are not in privity

Driver may contend that the doctrine of claim preclusion (res judicata) precludes Mother from presenting a claim arising from the same nucleus of facts that was presented in the first action brought by Son According to the doctrine of claim preclusion ldquowhen a court of competent jurisdiction has entered a final judgment on the merits of a cause of action the parties to the suit and their privies are thereafter bound lsquonot only as to every matter which was offered and received to sustain or defeat the claim or demand but as to any other admissible matter which might have been offered for that purposersquordquo Commissioner of Internal Revenue v Sunnen 333 US 591 597 (1948) (citation omitted)

However the doctrine of claim preclusion does not apply to Mother on the facts of this problem First Mother was not a party to the earlier case ldquoIt is a principle of general application in Anglo-American jurisprudence that one is not bound by a judgment in personam in a litigation in which he is not designated as a party or to which he has not been made a party by service of processrdquo Taylor v Sturgell 553 US 880 884 (2008) (citing Hansberry v Lee 311 US 32 40 (1940)) see also RESTATEMENT (SECOND) OF JUDGMENTS sect 34(3) (1982) This rule reflects our ldquodeep-rooted historic tradition that everyone should have his own day in courtrdquo Martin v Wilks 490 US 755 762 (1989) (citation omitted) (superseded by statute on other grounds) Since Mother was not a party to the first suit she is not bound by the judgment unless an exception to the general rule applies

Mother might be bound by the prior judgment if she were considered to have been sufficiently in privity with Son that Son represented her interests in that action ldquoA person who is not a party to an action but who is represented by a party is bound by and entitled to the benefits of a judgment as though he were a partyrdquo RESTATEMENT (SECOND) OF JUDGMENTS sect 41(1) But there is no suggestion in the facts of the problem that Son who is an adult purported to represent Motherrsquos interests in the first suit ldquo[C]lose family relationships are not sufficient by themselves to establish privity with the original suitrsquos party or to bind a nonparty to that suit by the judgment entered therein rdquo Cuauhtli v Chase Home Finance LLC 308 Fed Appx 772 773 (5th Cir 2009) (citation omitted) accord 18A CHARLES ALAN WRIGHT ET AL FEDERAL

PRACTICE AND PROCEDURE sect 4459 (2d ed 2002) In Taylor v Sturgell supra the Supreme Court identified other special circumstances in

which nonparties may be bound by a prior judgmentmdashwhen a nonparty consents to be bound when a nonparty is in a pre-existing substantive legal relationship with a party (such as preceding and succeeding property owners) when a nonparty assumed control of the prior litigation when a party seeks to relitigate through a proxy or where a special statutory scheme seeks to foreclose successive litigation by nonparties See Taylor 553 US at 893ndash95 None of these circumstances exists here

Because Mother was not a party to the first suit and is not in privity with Son who is an adult the judgment in the first action does not preclude her from bringing her own claim against Driver

Point Two (35) Under the doctrine of issue preclusion the judgment rendered in the first action does not preclude Mother a nonparty from litigating the issue of her negligence in maintaining her carrsquos

28

Federal Civil Procedure Analysis

brake lights because the judgment binds only parties or those in privity with them and Mother and Son are not in privity

By its affirmative response to a special interrogatory the jury in the first action expressly concluded that ldquoMother negligently failed to ensure that the brake lights on her car were in proper working orderrdquo Driver may attempt to invoke the doctrine of issue preclusion to preclude Mother from relitigating this issue in the second action

[I]ssue preclusion arises in a second action on the basis of a prior decision when the same lsquoissuersquo is involved in both actions the issue was lsquoactually litigatedrsquo in the first action after a full and fair opportunity for litigation the issue was lsquoactually decidedrsquo in the first action by a disposition that is sufficiently lsquofinalrsquo lsquoon the meritsrsquo and lsquovalidrsquo it was necessary to decide the issue in disposing of the first action and the later litigation is between the same parties or involves nonparties that are subject to the binding effect or benefit of the first action Once these requirements are met issue preclusion is available not only to defend against a demand for relief but also as offensive support for a demand for relief Issue preclusion moreover is available whether or not the second action involves a new claim or cause of action

18 CHARLES ALAN WRIGHT ET AL FEDERAL PRACTICE AND PROCEDURE sect 4416 at 392ndash93 (2d ed) see also RESTATEMENT (SECOND) OF JUDGMENTS sect 27 (1982)

Here several of the elements necessary for issue preclusion are present The same issue is involved in both actionsmdashthe issue of Motherrsquos negligence in failing to maintain the brake lights on her car That issue was actually litigated in the first action and decided by the jury There is nothing to suggest anything less than a full and fair opportunity to litigate The judgment disposing of the issue was final

Nevertheless the judgment will not preclude Mother from relitigating the issue for two reasons First Mother was not a party to the first action and as explained above Mother and Son are not in privity Therefore she cannot be denied an opportunity to litigate the issue of her negligence Second it does not appear that the juryrsquos decision as to Motherrsquos negligence was necessary to the prior judgment against Driver Nothing suggests that the finding on Motherrsquos negligence had any bearing on the outcome of the first action

Point Three (30) Under the doctrine of non-mutual issue preclusion the judgment rendered in the first action might preclude Driver from relitigating the issue of her negligence However Driver has a strong argument that such a result would be inconsistent with the policy against offensive use of non-mutual estoppel when the non-party plaintiff easily could have joined as a plaintiff in the first action

Because Son already convinced the jury in the first action that ldquoDriver was negligent in the operation of her vehiclerdquo Mother may wish to invoke the doctrine of non-mutual issue preclusion to prevent Driver from relitigating the question of her negligence As noted above ldquoissue preclusion arises in a second action on the basis of a prior decision when the same lsquoissuersquo is involved in both actions the issue was lsquoactually litigatedrsquo in the first action after a full and fair opportunity for litigation the issue was lsquoactually decidedrsquo in the first action by a disposition that is sufficiently lsquofinalrsquo lsquoon the meritsrsquo and lsquovalidrsquo it was necessary to decide the issue in disposing of the first action rdquo 18 CHARLES ALAN WRIGHT ET AL FEDERAL PRACTICE AND

PROCEDURE sect 4416 at 392 (2d ed) see also RESTATEMENT (SECOND) OF JUDGMENTS sect 27

29

Federal Civil Procedure Analysis

Here these basic requirements for issue preclusion are met First the same issue is involved in both suits whether Driver was negligent in the operation of her car Second this issue was actually litigated and decided in the first action the jury answered a special interrogatory raising this very question There is nothing to suggest that Driver lacked a full and fair opportunity to litigate the issue Since a judgment was rendered against Driver for the injuries Son sustained as a result of Driverrsquos negligence resolution of the issue was necessary to dispose of the first action Driver was a party to the first action so she may be bound by the judgment

[NOTE Traditionally issue preclusion required mutualitymdashboth the party asserting issue preclusion and the party against whom issue preclusion was asserted were bound by the prior judgment Under the traditional mutuality rule Mother could not assert issue preclusion against Driver because Mother would not be bound by the judgment if Driver sought to rely on it See Point One There is no mutuality between Mother and Driver with respect to the prior judgment

This traditional mutuality requirement has been abandoned in most jurisdictions The Supreme Court rejected a strict mutuality requirement in Blonder-Tongue Laboratories Inc v University of Illinois Foundation 402 US 313 (1971) (non-mutual defensive collateral estoppel used by a defendant to preclude a plaintiff from relitigating a claim the plaintiff previously litigated) and Parklane Hosiery Co v Shore 439 US 322 (1979) (non-mutual offensive collateral estoppel used by a plaintiff to preclude a defendant from relitigating a claim the defendant previously litigated) In Parklane Hosiery the Court concluded (as a matter of federal preclusion law) that trial courts should have ldquobroad discretionrdquo to determine whether or not to permit a plaintiff to invoke non-mutual issue preclusion ldquoThe general rule should be that in cases where a plaintiff could easily have joined in the earlier action or where the application of offensive estoppel would be unfair to a defendant a trial judge should not allow the use of offensive collateral estoppelrdquo Id at 331

The Parklane Hosiery decision identified a number of circumstances that might make it unfair to allow a plaintiff to invoke non-mutual issue preclusion (non-mutual offensive collateral estoppel in the traditional terminology) against a defendant In particular the Parklane Hosiery court suggested that issue preclusion may not be appropriate if the plaintiff in the second action ldquocould easily have joined in the earlier actionrdquo Id Prohibiting plaintiffs from using non-mutual estoppel under such circumstances would promote judicial efficiency by encouraging plaintiffs to join the prior action It would also discourage plaintiffs from staying out of prior litigation in order to secure in effect two bites at the apple using the prior litigation offensively if the defendant loses and forcing the defendant to litigate a second time if the defendant wins the prior action

An exceptional exam answer might therefore argue that non-mutual issue preclusion should be denied on these facts Son and Mother both reside in State A since they are related they know each other well and Son was driving Motherrsquos car when the accident occurred They could have sued together and Rule 20 of the Federal Rules of Civil Procedure would have authorized joinder of their claims because those claims arose from the same transaction or occurrence and raised a common question of law or fact FED R CIV P 20(a) The facts do not suggest that Mother had any reason not to join Sonrsquos suit other than a desire to see how Sonrsquos action concluded before bringing her own claim Cf Nations v Sun Oil Co (Del) 695 F2d 933 938 (5th Cir 1983) (concluding that plaintiff ldquowas entitled to await the development of his injuries and their predictable consequencesrdquo) Because it appears that Mother may be a ldquowait-and-seerdquo plaintiff who could easily have joined the original action a trial court might disallow as a matter of discretion her use of non-mutual issue preclusion]

30

AGENCY ANALYSIS __________ (Agency I II)

ANALYSIS

Legal Problems

(1) Is the principal or the agent or both liable on contracts with a third party when the principal is an ldquoundisclosed principalrdquo

(2) Is the principal or the agent or both liable on contracts with a third party when the principal is ldquopartially disclosedrdquo or an ldquounidentified principalrdquo

(3) Is the principal or the agent or both liable on contracts with a third party for the purchase of goods when the agent exceeded his authority but the principal nonetheless accepts the goods

DISCUSSION

Summary

The agent but not the owner is liable to the basket manufacturer because the owner is an undisclosed principal and the agent acted without actual or apparent authority Both the agent and the owner however are liable on the burner contract because the owner is an unidentified principal and the agent had apparent authority to enter into that contract With respect to the solar cells contract whether the owner is liable depends upon whether a court would follow the Second or Third Restatement of Agency which take different positions on the effect of the ratification of a contract by an undisclosed principal Under either the agent would also be liable on the contract as he was a party to the contract

[NOTE The contracts that are the subject of this question are contracts for the sale of goods and therefore are governed by Article 2 of the Uniform Commercial Code Article 2 however does not contain agency rules Accordingly common law concepts of agency are applicable UCC sect 1-103(b)]

Point One (35) The agent but not the owner is liable to the basket manufacturer The agent had no actual authority to enter into the contract to buy aluminum baskets and because the owner was an undisclosed principal the manufacturer had no reason to believe that the agent had apparent authority Furthermore the manufacturer had no reason to believe that the agent was not contracting for his own benefit

An agent acting on behalf of a principal can bind the principal to contracts if the agent has either actual or apparent authority An agent has actual authority when contracting on behalf of his principal if he ldquoreasonably believes in accordance with the principalrsquos manifestations to the agent that the principal wishes the agent so to actrdquo RESTATEMENT (THIRD) OF AGENCY sect 201 (2006) Here the agent was told to buy only wicker baskets not aluminum baskets Thus when he contracted with the basket manufacturer to buy aluminum baskets he had no actual authority to do so

31

Agency Analysis

An agent acts with apparent authority ldquowhen a third party [with whom the agent acts] reasonably believes the actor has authority to act on behalf of the principal and that belief is traceable to the principalrsquos manifestationsrdquo Id sect 203 Here the owner notified basket manufacturers that she or her agent might contact them to purchase baskets but that notification did not specifically name the agent or any other person as the ownerrsquos agent Furthermore the basket manufacturer had no prior dealings with the agent or the owner or any reason to think that the agent was acting for the benefit of anyone but himself Thus there is no basis to conclude that the basket manufacturer thought that the agent had apparent authority to act for the owner

Generally when an agent acts on behalf of an undisclosed principal and the agent lacks authority to enter into the contract the agent is liable on the contract as a party to the contract but the principal is not liable This rule is consistent with the third partyrsquos expectations ldquoThe third party expected the agent to be a party to the contract because the agent presented the deal as if he were acting for himself Moreover if the third party is unaware of the principalrsquos existence the third party must be relying on the agentrsquos solvency and reliability when entering into the contractrdquo See ROBERT W HAMILTON JONATHAN R MACEY amp DOUGLAS K MOLL CORPORATIONS INCLUDING PARTNERSHIPS AND LIMITED LIABILITY COMPANIES 34 (11th ed 2010) See also RESTATEMENT (THIRD) OF AGENCY sect 603 cmt c Furthermore because the third party has no idea that the agent is acting or is seemingly acting on behalf of another there is no reason to believe that the third party would be expecting an undisclosed principal to be liable on the contract Id

Point Two (35) Because the owner is an unidentified (as opposed to undisclosed) principal both she and the agent (as a party to the contract) probably are liable on the contract with the burner manufacturer

When the agent contracted with the burner manufacturer he did not have actual authority to do so as the owner had expressly restricted the agentrsquos authority to purchase only burners with ldquowhisper technologyrdquo See Point One However the agent may have had apparent authority to buy burners without whisper technology

An agent acts with apparent authority ldquowhen a third party [with whom the agent acts] reasonably believes the actor has authority to act on behalf of the principal and that belief is traceable to the principalrsquos manifestationsrdquo RESTATEMENT (THIRD) OF AGENCY sect 203 (2006) The owner indicated that an agent might contact the burner manufacturer The notice contained no restriction regarding the type of burners that the agent was authorized to purchase The facts indicate that burner manufacturers regularly receive such notices

Although the agent told the burner manufacturer that he represented a well-known hot-air balloon operator he did not disclose the ownerrsquos name Thus the owner was a partially disclosed or unidentified principal See RESTATEMENT (SECOND) OF AGENCY sect 4(2) (1958) (using term ldquopartially disclosed principalrdquo) RESTATEMENT (THIRD) OF AGENCY sect 104(2)(c) (2006) (using term ldquounidentified principalrdquo) An agent for a partially disclosed principal may have apparent authority RESTATEMENT (SECOND) OF AGENCY sect 159 cmt e (1958) Based upon (1) the notice sent by the owner (2) the agentrsquos revelation that he was acting as an agent and (3) the fact that burner manufacturers regularly receive such notices and sell to agents the manufacturer may argue that it reasonably and actually believed that the agent was authorized to purchase burners without whisper technology The manufacturer may also argue that because the agent revealed that he was an agent his listing of the ownerrsquos address as the delivery address connects the agent to the notice given by the owner Arguably this distinguishes the burner contract from the basket

32

Agency Analysis

contract Here there is a strong case to support the conclusion that the agent had apparent authority if he did then the owner is liable to the burner manufacturer

The agent also is liable as a party to the contract because he did not fully disclose his agency relationship Although he told the burner manufacturer that he represented a well-known hot-air balloon operator he did not disclose the ownerrsquos name Generally even an authorized agent of a partially disclosed or unidentified principal is liable as a party to a contract with a third person RESTATEMENT (SECOND) OF AGENCY sect 321 (1958) (ldquounless otherwise agreedrdquo) RESTATEMENT (THIRD) OF AGENCY sect 602(2) (2006) (ldquounless the agent and the third party agree otherwiserdquo)

Point Three (30) Under the Second Restatement of Agency the owner is not liable on the contract for solar cells because the agent did not have actual or apparent authority and the owner as an undisclosed principal cannot ratify the contract Under the Third Restatement the owner could be liable as she ratified the contract Under either Restatement the agent is liable as a party to the contract

The owner is not liable to the solar cell manufacturer for breach of the contract for the solar cells because the agent had no actual or apparent authority to purchase solar cells on the ownerrsquos behalf and the owner under the Second Restatement of Agency did not ratify the contract with knowledge of the material facts Thus she is not liable as a ratifier of the contract

The facts state that the agent had authority to purchase only propane fuel tanks In addition he had no apparent authority to purchase solar cells The owner made no manifestations to the solar cell manufacturer that would lead a reasonable person in the manufacturerrsquos position to believe that the agent had the authority to bind the owner to a contract to purchase solar cells In fact the agent made no manifestations at all to the solar cell manufacturer Unlike with the basket manufacturer and the burner manufacturer the owner did not notify the manufacturer of solar cells that an agent might contact it to purchase solar cells In addition the solar cells were delivered to the agent and not to the ownerrsquos address In sum the manufacturer was unaware of any relationship between the owner and the agent As to the solar cell manufacturer the owner is an undisclosed principal There can be no apparent authority in the case of an undisclosed principal because there are no manifestations from the principal to the third person See RESTATEMENT (SECOND) OF AGENCY sect 8 cmt a (1958) (ldquothere can be no apparent authority created by an undisclosed principalrdquo) RESTATEMENT (THIRD) OF AGENCY sect 203 cmt f (2006) (ldquoapparent authority is not present when a third party believes that an interaction is with an actor who is a principalrdquo)

The owner also did not ratify the contract Although the owner used the solar cells generally a principal cannot ratify an unauthorized transaction with a third person ldquounless the one acting purported to be acting for the ratifierrdquo RESTATEMENT (SECOND) OF AGENCY sect 85(1) (1958)

The result differs under the Third Restatement which expressly rejects the Second Restatement on this issue The Restatement (Third) of Agency sect 403 (2006) states ldquoA person may ratify an act if the actor acted or purported to act as an agent on the personrsquos behalfrdquo According to comment b ldquoan undisclosed principal may ratify an agentrsquos unauthorized actrdquo Under the Restatement (Third) of Agency rule the owner probably ratified the transaction The agent clearly acted on the ownerrsquos behalf and in addition the ownerrsquos conduct in using the solar cells ldquojustifies a reasonable assumption that [she] is manifesting assent that the act shall affect [her] legal relationsrdquo See id sect 401(2)

33

Agency Analysis

The agent also is liable to the solar cell manufacturer for breach of the contract for the solar cells because he is a party to the contract The facts indicate that the agent never told the solar cell manufacturer that he represented the owner or any other principal Consequently even if the agent were authorized (which as discussed above he is not) he would be liable as a party to the contract See RESTATEMENT (SECOND) OF AGENCY sect 322 (1958) RESTATEMENT (THIRD) OF AGENCY sect 603(2) (2006) Here he has no authority or apparent authority and is liable as a party to the contract

The agent would also be liable under the Third Restatement Under Restatement (Third) of Agency sect 402(1) (2006) ratification generally relates back and the transaction is treated as if it were authorized at the time of the transaction However this does not relieve the agent of an undisclosed principal who ratifies an unauthorized transaction of liability under the ratified contract See id sect 603(2) (authorized agent for undisclosed principal is a party to the contract) and sect 403 cmt b (ldquoAn undisclosed principalrsquos ratification does not eliminate the agentrsquos liability to the third party on the transaction rdquo)

[NOTE An examinee may discuss the concept of inherent agency power This concept is recognized by the Restatement (Second) of Agency sect 8 A (1958) but the concept is not used in the Restatement (Third) of Agency (2006) Here there are no facts to support that the agent had inherent authority

As to contracts with agents for partially disclosed principals (eg the contract for the burners) the basic question is whether the acts done ldquousually accompany or are incidental to transactions which the agent is authorized to conductrdquo RESTATEMENT (SECOND) OF AGENCY

sect 161 (1958) If so the principal is bound if the other party ldquoreasonably believes that the agent is authorized to do them and has no notice that he is not so authorizedrdquo Id The purchase of burners without whisper technology was not authorized nor was it incidental to an authorized transaction Therefore there should not be inherent agency power

As to contracts on behalf of undisclosed principals (eg the other two contracts) the basic question is whether the acts done are usual or necessary in the transactions the agent is authorized to transact RESTATEMENT (SECOND) OF AGENCY sect 194 (1958) The other two contracts seem fundamentally different from the authorized transactions Therefore there should not be inherent agency power

Only minimal credit should be given for discussion of inherent agency power]

34

EVIDENCE ANALYSIS _____ (Evidence IIA VA B E F J K)

ANALYSIS

Legal Problems

(1) Is the authenticated copy of the mechanicrsquos text message relevant and admissible

(2) Is the womanrsquos question ldquoIs my scooter safe to drive for a whilerdquo relevant and admissible

(3) Is the womanrsquos testimony describing the mechanicrsquos thumbs-up relevant and admissible

DISCUSSION

Summary

The mechanicrsquos text message to the woman is relevant to whether (1) the woman lost control of the scooter due to its defective brakes (2) the woman knew that the brakes needed repair and (3) it was negligent for the woman to drive the scooter knowing that its brakes needed repair

The mechanicrsquos text message is hearsay if it is offered by the pedestrian to prove that the scooterrsquos brakes needed repair However it fits the hearsay exception for present sense impressions and probably also fits the exception for business records The mechanicrsquos text message is not hearsay if it is instead offered by the pedestrian to prove the womanrsquos state of mind (ie that she had notice that her brakes needed repair)

The womanrsquos question to the mechanic and his response are also relevant to whether the brakes caused the accident and whether the woman was negligent The question is not hearsay because the woman did not make an assertion

The mechanicrsquos thumbs-up response is nonverbal conduct intended by the mechanic as an assertion and is therefore an out-of-court statement If the woman offers the mechanicrsquos statement to prove that the scooter was actually safe to ride the womanrsquos testimony about the statement is hearsay

However the mechanicrsquos statement is not hearsay if it is offered by the woman to prove her state of mind Therefore the womanrsquos question and the mechanicrsquos response are admissible to prove the womanrsquos state of mind

Point One(a) (20) The mechanicrsquos text message to the woman should be admitted because it is relevant

Evidence is relevant if it has ldquoany tendency to make a fact more or less probable than it would be without the evidencerdquo FED R EVID 401 ldquoRelevant evidence is admissiblerdquo unless it is inadmissible pursuant to some other rule FED R EVID 402

The mechanicrsquos text message to the woman ldquoWhen you pick up your scooter you need to schedule a follow-up brake repair Wersquoll order the partsrdquo is relevant for two reasons First this evidence has some tendency to make it more probable that the brakes malfunctioned and

35

Evidence Analysis

caused the accident Second it has some tendency to make it more probable that the woman was negligent in riding her scooter after being told by the mechanic that it required further repair

Point One(b) (30) The mechanicrsquos text message fits either the hearsay exception for present sense impressions or the exception for business records or it is admissible non-hearsay

The mechanicrsquos text message is a statement under Rule 801(a) because it is ldquoa written assertionrdquo FED R EVID 801(a) The text message is hearsay if the pedestrian offers it to prove the ldquotruth of the matter asserted in the statementrdquo (ie that the scooterrsquos brakes required repair) which resulted in the woman losing control of the scooter and causing the accident FED R EVID 801(c)

However the mechanicrsquos text message fits the hearsay exception for ldquopresent sense impressionsrdquo under Rule 803(1) because it is ldquo[a] statement describing or explaining an event or condition made while or immediately after the declarant perceived itrdquo FED R EVID 803(1) Here the mechanicrsquos text message described the condition of the scooter immediately after he perceived it during the maintenance service

The mechanic is a person with knowledge of the condition of the scooter so if text messages regarding repairs were made and kept by the mechanic in the ordinary course of business this text message also fits the business records exception Under Rule 803(6) a business record is a record of an act ldquomade at or near the time by someone with knowledgerdquo and ldquothe record was kept in the course of a regularly conducted activity of a businessrdquo and ldquomaking the record was a regular practice of that activityrdquo FED R EVID 803(6)

However the text message is not hearsay if it is instead offered to prove that the woman was negligent because she rode her scooter after the mechanic told her it required repair If offered for this purpose it would not be offered for the truth of the matter asserted in the statement but to show the womanrsquos belief about the condition of the scooter (her state of mind)

Point Two (10) The womanrsquos question to the mechanic should be admitted because it is not hearsay

The womanrsquos question to the mechanic is relevant because along with the mechanicrsquos thumbs-up response (see Point Three) it has some tendency to make it more probable that the woman was not negligent andor that the scooter brakes did not malfunction and cause the accident FED R EVID 401 The womanrsquos question does not raise hearsay concerns because it is not an assertion

Hearsay is defined under Rule 801(a) as ldquoan oral assertion written assertion or nonverbal conductrdquo Although ldquoassertionrdquo is not further defined ldquoa favorite [definition] of writers in the [evidence] field for at least a century and a half [is that] the word simply means to say that something is so eg that an event happened or a condition existedrdquo 2 MCCORMICK ON

EVIDENCE sect 246 (6th ed 2006) Under this definition the womanrsquos question is not hearsay because it is not an assertion

Point Three(a) (20) The mechanicrsquos thumbs-up to the woman is a nonverbal assertion that is relevant and the womanrsquos testimony about that response is admissible

36

Evidence Analysis

Hearsay is defined under Rule 801(c) as a ldquostatementrdquo that is ldquoa personrsquos oral assertion written assertion or nonverbal conduct if the person intended it as an assertionrdquo FED R EVID 801(a) Here when the mechanic responded to the womanrsquos question (ldquoIs my scooter safe to ride for a whilerdquo) with a thumbs-up gesture the facts suggest that he intended his nonverbal conduct as an assertion that in his opinion the scooter was safe to ride

The mechanicrsquos assertion is relevant and admissible to prove that the woman was not negligent because the evidence makes it more probable that at the time of the accident she believed that the scooter was safe to ride despite the fact that the brakes required repair FED R EVID 401 Admission of the womanrsquos description of the mechanicrsquos thumbs-up for this purpose does not raise hearsay concerns because the evidence would not be offered for the truth of the matter asserted but to show the womanrsquos belief about the condition of the scooter (her state of mind)

Point Three(b) (20) The mechanicrsquos thumbs-up is relevant to determine whether the scooterrsquos brakes malfunctioned causing the accident but if offered for this purpose it is also hearsay

The mechanicrsquos nonverbal assertion is relevant to the determination of whether the scooterrsquos brakes malfunctioned causing the accident However if offered to prove the ldquotruth of the matter asserted in the statementrdquo (ie that the scooter was safe to ride for a while) it is hearsay that does not fit any hearsay exception

37

TRUSTS AND FUTURE INTERESTS ANALYSIS ____________________ (Trusts and Future Interests IC1 amp 4 G IIF)

ANALYSIS

Legal Problems

(1)(a) Was the revocable trust amendable

(1)(b) If the trust was amendable must the amendment have been executed in accordance with the state Statute of Wills in order to be valid

(2) If the trust amendment was valid does the amendment apply to the probate estate assets passing to the trust pursuant to Settlorrsquos will

(3) If the trust amendment was valid should the trust property be distributed to University

(4) If the trust amendment was not valid should the trust property be distributed to Settlorrsquos grandchild (her only heir) or held in further trust in accordance with the terms of the original trust instrument

DISCUSSION

Summary

A revocable trust is amendable even if the trust instrument does not expressly grant to the trust settlor a power to amend Both inter vivos trusts and amendments thereto are valid even though not executed in accordance with the requirements applicable to wills

Under the Uniform Testamentary Additions to Trusts Act a revocable trust may be amended at any time prior to the settlorrsquos death and the amendment applies to the disposition of assets conveyed to the trust pursuant to a will even if the will was executed prior to the date of the amendment

At Settlorrsquos death trust assets including probate assets passing to the trust under Settlorrsquos will would go to University if as is the case here the trust amendment was valid If the amendment was invalid the trust assets would continue to be held in further trust because there is no violation of the common law Rule Against Perpetuities

Point One(a) (30) Settlor retained the right to amend the inter vivos trust despite her failure to expressly reserve this power

At issue here is whether a retained power of revocation includes the power to amend sometimes referred to as the power to modify The Restatement (Second) of Trusts sect 331 cmt g provides that if a settlor has a power to revoke that retained power ordinarily includes a power to modify (amend) as well Comment g also notes that the power to amend includes both a power to withdraw trust assets and a power to ldquomodify the terms of the trustrdquo The Uniform Trust Code which provides that a power to revoke includes the power to amend is consistent with this view

38

Trusts and Future Interests Analysis

UNIF TRUST CODE sect 602 accord RESTATEMENT (THIRD) OF TRUSTS sect 63 cmt The theory is that even though a power to amend was not expressly retained by a settlor the goal of amendment assuming the power was not included in the power to revoke could easily be achieved by first revoking the trust and then creating a new trust with the same terms contemplated by the amendment To require this would put form over substance

Thus by expressly retaining the power to revoke the trust Settlor retained a power to amend the inter vivos trust despite her failure to expressly reserve this power

[NOTE Under the common law a trust is irrevocable unless the settlor expressly retains a power to revoke the trust Conversely under the Uniform Trust Code a trust is revocable unless the terms of the trust expressly provide otherwise See UNIF TRUST CODE sect 602 The Trust Codersquos position on revocation follows the minority view in the United States and is inconsistent with prior Restatements of Trusts (see Restatement (Second) of Trusts sect 330) Here the trust is revocable because Settlor expressly retained a power of revocation

The Uniform Trust Code has been adopted in 24 jurisdictions Alabama Arizona Arkansas District of Columbia Florida Kansas Maine Michigan Missouri Nebraska New Hampshire New Mexico North Carolina North Dakota Ohio Oregon Pennsylvania South Carolina Tennessee Utah Vermont Virginia West Virginia and Wyoming]

Point One(b) (10) Settlorrsquos amendment of the trust was valid despite her failure to have her signature to the trust amendment witnessed

Neither the common law nor state statutes require a trust instrument or an amendment to a trust instrument to be executed in accordance with the formalities prescribed for execution of a will Indeed an inter vivos trust that does not involve real estate can be created orally Under the Uniform Trust Code the only requirements for creating a valid inter vivos trust are intent the specification of beneficiaries and the designation of a trustee See UNIF TRUST CODE sect 402 accord RESTATEMENT (THIRD) OF TRUSTS sect 13

Here the amendment meets the requirements of both the Uniform Trust Code and the common law Thus the fact that Settlorrsquos signature was not witnessed when she signed the amendment to the trust does not make the amendment invalid

Point Two (20) Under the Uniform Testamentary Additions to Trusts Act a revocable trust may be amended at any time prior to the settlorrsquos death and the amendment applies to probate assets poured into the trust at the settlorrsquos death pursuant to the settlorrsquos will even when the will was executed prior to the date of the amendment

Historically property owned by an individual at her death passed to the individualrsquos heirs or to beneficiaries designated in a will executed with the formalities (writing signing witnessing) prescribed by state law However when a will devises property to the trustee of an inter vivos trust then the provisions of the trustmdashwhich may not have been executed in accordance with the formalities required for willsmdasheffectively determine who will receive the property Because of this possibility some early cases held that if an inter vivos trust was not executed with the same formalities required for a valid will then the trust was ineffective to dispose of probate assets poured into the trust at the settlorrsquos death pursuant to the settlorrsquos will

This line of cases has been overturned by the Uniform Testamentary Additions to Trusts Act (the Act) now Uniform Probate Code sect 2-511 Under the Act adopted in almost all

39

Trusts and Future Interest Analysis

jurisdictions a testamentary bequest to the trustee of an inter vivos trust established by the testator during his or her lifetime is valid if the trust is in writing it is identified in the testatorrsquos will and the trust instrument was executed before concurrently with or after the execution of the will Id The Act further specifies that such a bequest is valid even if the trust is amendable or revocable and that a later amendment applies to assets passing to the trust by a previously executed will

Thus because the trust amendment is valid its terms apply to assets received by Bank from Settlorrsquos estate

Point Three (10) If the trust amendment was valid then the trust assets including assets passing to the trust under Settlorrsquos will should go to University

Under the trust amendment all trust assets (including the assets of Settlorrsquos probate estate poured into the trust) pass to University The facts provide no basis for failing to comply with Settlorrsquos stated intentions

Point Four (30) If the trust amendment was invalid trust assets including assets received pursuant to Settlorrsquos will should be held in accordance with the terms of the original trust instrument because those terms do not violate the Rule Against Perpetuities

Under the dispositive terms of the original trust instrument Settlor created successive income interests in her surviving children and grandchildren with a remainder interest in her great-grandchildren Because the trust was revocable the period during which the common law Rule Against Perpetuities requires that interests vest (ie 21 years plus lives in being) began to run from the date Settlor no longer had a power of revocation (here her death) not the date on which the trust was created See JESSE DUKEMINIER STANLEY J JOHANSON JAMES LINDGREN amp ROBERT SITKOFF WILLS TRUSTS AND ESTATES 678 (7th ed 2005)

Under the common law Rule Against Perpetuities Settlorrsquos trust is thus valid At the time of Settlorrsquos death she was survived by no children one granddaughter and no great-grandchildren Because Settlor cannot have more children after her death the only income beneficiary of the trust is Settlorrsquos surviving granddaughter This granddaughter is the only person who can produce great-grandchildren of Settlor thus all great-grandchildren must of necessity be born during the lifetime of Settlorrsquos only surviving granddaughter who is a life in being The granddaughterrsquos interest vested at Settlorrsquos death and the great-grandchildrenrsquos interest will vest at the death of the granddaughter There is no need to wait the additional 21 years permitted under the Rule Thus under the common law and the statute given in the facts the nonvested interest in the great-grandchildren is valid

[NOTE Both modern wait-and-see statutes and the Uniform Statutory Rule Against Perpetuities upon which the statute in the facts is modeled provide that before using either reform to validate an otherwise invalid nonvested interest one should first determine if the nonvested interest violates the common law Rule If it does not then there is no need to reform This proposition which is applicable in all MEE user jurisdictions that have not simply abrogated the rule is tested by this problem]

40

NEGOTIABLE INSTRUMENTS ANALYSIS (Negotiable Instruments III IV V)

ANALYSIS

Legal Problems

(1)(a) What rights does a person in possession of a note that has been indorsed in blank by the payee have against the maker of the note

(1)(b) Which defenses may the maker of a note raise against a person entitled to enforce it who is not a holder in due course but is a transferee from a holder in due course

(2) What rights does a person entitled to enforce a note have against an indorser who transferred it for consideration with no warranties

(3) What rights does a person entitled to enforce a note have against a previous holder who transferred it as a gift without indorsing it

DISCUSSION

Summary

The niece is a holder of the note and is thus a person entitled to enforce it The chef the issuer of the note is obligated to pay it to the niece as the person entitled to enforce it The niece is not subject to any defense or claim of the chef relating to the improper repair of the oven because the niece has the rights of a holder in due course When the buyer bought the note from the repairman the buyer became a holder in due course of the note and thus took it free of any personal defenses the chef had against the repairman Even though the niece is not herself a holder in due course of the note the niece succeeded to the buyerrsquos rights as holder in due course and thus took free of the chefrsquos personal defenses

Because the chef refused to pay the note the niece can recover from the repairman on the repairmanrsquos obligation as indorser The niece cannot recover on the note against the buyer however because the buyer did not indorse the note (and thus incurred no indorserrsquos obligation) and the buyer did not receive any consideration for transfer of the note to the niece (and therefore made no transfer warranty)

[NOTE Although Article 9 of the Uniform Commercial Code governs the sale of promissory notes (a point that might be correctly noted by examinees) that Article does not determine the answer to any of the questions posed]

Point One(a) (20) The niece is the holder of the note and thus may enforce it against the chef who is the issuer of the note

The chef is the maker of the note and thus its issuer See UCC sectsect 3-103 3-105 The issuer of a note is obligated to pay it in accordance with its terms to a ldquoperson entitled to enforcerdquo it UCC sect 3-412 The niece is a ldquoperson entitled to enforcerdquo the note This is because the niece is the holder of the note and a holder of a note is a person entitled to enforce it UCC sect 3-301 The niece is the holder of the note because (i) the repairmanrsquos signature on the back of the note not

41

Negotiable Instruments Analysis

accompanied by words indicating a person to whom the note was made payable was a ldquoblank indorsementrdquo which had the effect of making the note a bearer instrument (ii) anyone in possession of a bearer instrument is a holder of it and (iii) the niece is in possession of the note See UCC sectsect 1-201(b)(21)(A) 3-204 and 3-205 Accordingly the chef has an obligation to the niece to pay the note in accordance with its terms and the niece may enforce that obligation

Point One(b) (40) The niece is not a holder in due course of the note but because she is a transferee from the buyer who was a holder in due course she has the same enforcement rights as the buyer Because the buyer as a holder in due course would have been able to enforce the note against the chef without being subject to defenses or claims arising from the improper repair the niece has the same rights and will not be subject to the chefrsquos defenses or claims about the repair

As noted in Point One(a) the chef has an obligation to the niece to pay the note in accordance with its terms However except against a person with the rights of a holder in due course the chef can raise any defenses or claims in recoupment that he would have if the claim on the note were an ordinary contract claim UCC sect 3-305 Thus except against a holder in due course the chef would be able to raise the improper repair as a defense or a claim in recoupment (a claim in response to the niecersquos claim)

But claims in recoupment and most defenses cannot be raised against a person with the rights of a holder in due course Against a holder in due course the chef can raise only the four ldquorealrdquo defenses listed in UCC sect 3-305(a)(1) (infancy duress lack of legal capacity or illegality that nullifies the obligation of the obligor under other law fraud in the factum discharge in insolvency proceedings) none of which is present here

The niece is not a holder in due course because she did not take the note for value See UCC sectsect 3-302(a)(2)(i) (criteria for holder in due course status) and 3-303(a) (definition of ldquovaluerdquo) But this does not mean that the niece is subject to the chefrsquos claim arising out of the improper repair The buyer was a holder in due course of the note because he took the note for value ($9500) in good faith and without notice of any facts that would have alerted him to the chefrsquos defense against the repairman UCC sect 3-302(a)(2) As a holder in due course the buyer owned the note free of the chefrsquos claim because that claim did not constitute a ldquorealrdquo defense UCC sect 3-305(b) When the buyer gave the note to the niece this constituted a ldquotransferrdquo of the note See UCC sect 3-203(a) When a note is transferred the transferee receives ldquoany right of the transferor to enforce the instrument including any right as a holder in due courserdquo UCC sect 3-203(b) Under this rule (also known as the ldquoshelter principlerdquo) the buyer transferred his freedom from the chefrsquos defenses to the niece and the niece can enforce the note free of the chefrsquos defenses

Point Two (20) Because the chef dishonored the note the niece can recover from the repairman on the repairmanrsquos obligation as indorser

The chefrsquos refusal to pay the note constituted dishonor See UCC sect 3-502 The repairman as an indorser of the note (see Point One(a)) incurred the obligations of an indorser under UCC sect 3-415(a) When a note has been dishonored one of the obligations of an indorser is to pay the amount of the note to a person entitled to enforce it Therefore the repairman is liable for the amount of the note to the niece a person entitled to enforce the note (so long as the niece gives proper notice of dishonor to the repairman)

42

Negotiable Instruments Analysis

[NOTE Because the repairman indorsed the note without warranties there are no transfer warranties UCC sect 3-416 cmt 5]

Point Three (20) The niece cannot recover on the note against the buyer as either indorser or warrantor because the buyer did not indorse the note and did not receive consideration for transferring the note to the niece

The buyer did not indorse the note and therefore did not incur the obligation of an indorser to pay the note upon dishonor

The niece cannot recover from the buyer under a transfer warranty theory because transfer warranties are made only by a person ldquowho transfers an instrument for considerationrdquo Here the buyer gave the instrument to the niece as a gift So the buyer made no transfer warranty UCC sect 3-416(a) Therefore the niece cannot recover from the buyer on that theory

43

National Conference of Bar Examiners 302 South Bedford Street | Madison WI 53703-3622 Phone 608-280-8550 | Fax 608-280-8552 | TDD 608-661-1275

wwwncbexorg e-mail contactncbexorg

  • Contents
  • Preface
  • Description of the MEE
  • Instructions
  • February 2013 Questions
    • Real Property Question
    • Contracts Question
    • Constitutional Law Question
    • Secured Transactions Question
    • Federal Civil Procedure Question
    • Agency Question
    • Evidence Question
    • Trusts and Future Interests Question
    • Negotiable Instruments Question
      • February 2013 Analyses
        • Real Property Analysis
        • Contracts Analysis
        • Constitutional Law Analysis
        • Secured Transactions Analysis
        • Federal Civil Procedure Analysis
        • Agency Analysis
        • Evidence Analysis
        • Trusts and Future Interests Analysis
        • Negotiable Instruments Analysis
            • ltlt ASCII85EncodePages false AllowTransparency false AutoPositionEPSFiles true AutoRotatePages None Binding Left CalGrayProfile (Dot Gain 20) CalRGBProfile (sRGB IEC61966-21) CalCMYKProfile (US Web Coated 050SWOP051 v2) sRGBProfile (sRGB IEC61966-21) CannotEmbedFontPolicy Error CompatibilityLevel 14 CompressObjects Tags CompressPages true ConvertImagesToIndexed true PassThroughJPEGImages true CreateJobTicket false DefaultRenderingIntent Default DetectBlends true DetectCurves 00000 ColorConversionStrategy CMYK DoThumbnails false EmbedAllFonts true EmbedOpenType false ParseICCProfilesInComments true EmbedJobOptions true DSCReportingLevel 0 EmitDSCWarnings false EndPage -1 ImageMemory 1048576 LockDistillerParams false MaxSubsetPct 100 Optimize true OPM 1 ParseDSCComments true ParseDSCCommentsForDocInfo true PreserveCopyPage true PreserveDICMYKValues true PreserveEPSInfo true PreserveFlatness true PreserveHalftoneInfo false PreserveOPIComments true PreserveOverprintSettings true StartPage 1 SubsetFonts true TransferFunctionInfo Apply UCRandBGInfo Preserve UsePrologue false ColorSettingsFile () AlwaysEmbed [ true ] NeverEmbed [ true ] AntiAliasColorImages false CropColorImages true ColorImageMinResolution 300 ColorImageMinResolutionPolicy OK DownsampleColorImages true ColorImageDownsampleType Bicubic ColorImageResolution 300 ColorImageDepth -1 ColorImageMinDownsampleDepth 1 ColorImageDownsampleThreshold 150000 EncodeColorImages true ColorImageFilter DCTEncode AutoFilterColorImages true ColorImageAutoFilterStrategy JPEG ColorACSImageDict ltlt QFactor 015 HSamples [1 1 1 1] VSamples [1 1 1 1] gtgt ColorImageDict ltlt QFactor 015 HSamples [1 1 1 1] VSamples [1 1 1 1] gtgt JPEG2000ColorACSImageDict ltlt TileWidth 256 TileHeight 256 Quality 30 gtgt JPEG2000ColorImageDict ltlt TileWidth 256 TileHeight 256 Quality 30 gtgt AntiAliasGrayImages false CropGrayImages true GrayImageMinResolution 300 GrayImageMinResolutionPolicy OK DownsampleGrayImages true GrayImageDownsampleType Bicubic GrayImageResolution 300 GrayImageDepth -1 GrayImageMinDownsampleDepth 2 GrayImageDownsampleThreshold 150000 EncodeGrayImages true GrayImageFilter DCTEncode AutoFilterGrayImages true GrayImageAutoFilterStrategy JPEG GrayACSImageDict ltlt QFactor 015 HSamples [1 1 1 1] VSamples [1 1 1 1] gtgt GrayImageDict ltlt QFactor 015 HSamples [1 1 1 1] VSamples [1 1 1 1] gtgt JPEG2000GrayACSImageDict ltlt TileWidth 256 TileHeight 256 Quality 30 gtgt JPEG2000GrayImageDict ltlt TileWidth 256 TileHeight 256 Quality 30 gtgt AntiAliasMonoImages false CropMonoImages true MonoImageMinResolution 1200 MonoImageMinResolutionPolicy OK DownsampleMonoImages true MonoImageDownsampleType Bicubic MonoImageResolution 1200 MonoImageDepth -1 MonoImageDownsampleThreshold 150000 EncodeMonoImages true MonoImageFilter CCITTFaxEncode MonoImageDict ltlt K -1 gtgt AllowPSXObjects false CheckCompliance [ None ] PDFX1aCheck false PDFX3Check false PDFXCompliantPDFOnly false PDFXNoTrimBoxError true PDFXTrimBoxToMediaBoxOffset [ 000000 000000 000000 000000 ] PDFXSetBleedBoxToMediaBox true PDFXBleedBoxToTrimBoxOffset [ 000000 000000 000000 000000 ] PDFXOutputIntentProfile () PDFXOutputConditionIdentifier () PDFXOutputCondition () PDFXRegistryName () PDFXTrapped False CreateJDFFile false Description ltlt ARA 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 BGR ltFEFF04180437043f043e043b043704320430043904420435002004420435043704380020043d0430044104420440043e0439043a0438002c00200437043000200434043000200441044a0437043404300432043004420435002000410064006f00620065002000500044004600200434043e043a0443043c0435043d04420438002c0020043c0430043a04410438043c0430043b043d043e0020043f044004380433043e04340435043d04380020043704300020043204380441043e043a043e043a0430044704350441044204320435043d0020043f04350447043004420020043704300020043f044004350434043f0435044704300442043d04300020043f043e04340433043e0442043e0432043a0430002e002000200421044a04370434043004340435043d043804420435002000500044004600200434043e043a0443043c0435043d044204380020043c043e0433043004420020043404300020044104350020043e0442043204300440044f0442002004410020004100630072006f00620061007400200438002000410064006f00620065002000520065006100640065007200200035002e00300020043800200441043b0435043404320430044904380020043204350440044104380438002egt CHS ltFEFF4f7f75288fd94e9b8bbe5b9a521b5efa7684002000410064006f006200650020005000440046002065876863900275284e8e9ad88d2891cf76845370524d53705237300260a853ef4ee54f7f75280020004100630072006f0062006100740020548c002000410064006f00620065002000520065006100640065007200200035002e003000204ee553ca66f49ad87248672c676562535f00521b5efa768400200050004400460020658768633002gt CHT ltFEFF4f7f752890194e9b8a2d7f6e5efa7acb7684002000410064006f006200650020005000440046002065874ef69069752865bc9ad854c18cea76845370524d5370523786557406300260a853ef4ee54f7f75280020004100630072006f0062006100740020548c002000410064006f00620065002000520065006100640065007200200035002e003000204ee553ca66f49ad87248672c4f86958b555f5df25efa7acb76840020005000440046002065874ef63002gt CZE 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 DAN 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 DEU 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 ESP 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 ETI 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 FRA 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 GRE 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 HEB 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 HRV (Za stvaranje Adobe PDF dokumenata najpogodnijih za visokokvalitetni ispis prije tiskanja koristite ove postavke Stvoreni PDF dokumenti mogu se otvoriti Acrobat i Adobe Reader 50 i kasnijim verzijama) HUN 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 ITA 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 JPN ltFEFF9ad854c18cea306a30d730ea30d730ec30b951fa529b7528002000410064006f0062006500200050004400460020658766f8306e4f5c6210306b4f7f75283057307e305930023053306e8a2d5b9a30674f5c62103055308c305f0020005000440046002030d530a130a430eb306f3001004100630072006f0062006100740020304a30883073002000410064006f00620065002000520065006100640065007200200035002e003000204ee5964d3067958b304f30533068304c3067304d307e305930023053306e8a2d5b9a306b306f30d530a930f330c8306e57cb30818fbc307f304c5fc59808306730593002gt KOR ltFEFFc7740020c124c815c7440020c0acc6a9d558c5ec0020ace0d488c9c80020c2dcd5d80020c778c1c4c5d00020ac00c7a50020c801d569d55c002000410064006f0062006500200050004400460020bb38c11cb97c0020c791c131d569b2c8b2e4002e0020c774b807ac8c0020c791c131b41c00200050004400460020bb38c11cb2940020004100630072006f0062006100740020bc0f002000410064006f00620065002000520065006100640065007200200035002e00300020c774c0c1c5d0c11c0020c5f40020c2180020c788c2b5b2c8b2e4002egt LTH 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 LVI 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 NLD (Gebruik deze instellingen om Adobe PDF-documenten te maken die zijn geoptimaliseerd voor prepress-afdrukken van hoge kwaliteit De gemaakte PDF-documenten kunnen worden geopend met Acrobat en Adobe Reader 50 en hoger) NOR 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 POL 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 PTB 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 RUM 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 RUS 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 SKY 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 SLV 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 SUO 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 SVE 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 TUR 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 UKR ltFEFF04120438043a043e0440043804410442043e043204430439044204350020044604560020043f043004400430043c043504420440043800200434043b044f0020044104420432043e04400435043d043d044f00200434043e043a0443043c0435043d044204560432002000410064006f006200650020005000440046002c0020044f043a04560020043d04300439043a04400430044904350020043f045604340445043e0434044f0442044c00200434043b044f0020043204380441043e043a043e044f043a04560441043d043e0433043e0020043f0435044004350434043404400443043a043e0432043e0433043e0020043404400443043a0443002e00200020042104420432043e04400435043d045600200434043e043a0443043c0435043d0442043800200050004400460020043c043e0436043d04300020043204560434043a0440043804420438002004430020004100630072006f006200610074002004420430002000410064006f00620065002000520065006100640065007200200035002e0030002004300431043e0020043f04560437043d04560448043e04570020043204350440044104560457002egt ENU (Use these settings to create Adobe PDF documents best suited for high-quality prepress printing Created PDF documents can be opened with Acrobat and Adobe Reader 50 and later) gtgt Namespace [ (Adobe) (Common) (10) ] OtherNamespaces [ ltlt AsReaderSpreads false CropImagesToFrames true ErrorControl WarnAndContinue FlattenerIgnoreSpreadOverrides false IncludeGuidesGrids false IncludeNonPrinting false IncludeSlug false Namespace [ (Adobe) (InDesign) (40) ] OmitPlacedBitmaps false OmitPlacedEPS false OmitPlacedPDF false SimulateOverprint Legacy gtgt ltlt AddBleedMarks false AddColorBars false AddCropMarks false AddPageInfo false AddRegMarks false ConvertColors ConvertToCMYK DestinationProfileName () DestinationProfileSelector DocumentCMYK Downsample16BitImages true FlattenerPreset ltlt PresetSelector MediumResolution gtgt FormElements false GenerateStructure false IncludeBookmarks false IncludeHyperlinks false IncludeInteractive false IncludeLayers false IncludeProfiles false MultimediaHandling UseObjectSettings Namespace [ (Adobe) (CreativeSuite) (20) ] PDFXOutputIntentProfileSelector DocumentCMYK PreserveEditing true UntaggedCMYKHandling LeaveUntagged UntaggedRGBHandling UseDocumentProfile UseDocumentBleed false gtgt ]gtgt setdistillerparamsltlt HWResolution [2400 2400] PageSize [612000 792000]gtgt setpagedevice

Page 16: February 2013 MEE Questions and Analyses
Page 17: February 2013 MEE Questions and Analyses
Page 18: February 2013 MEE Questions and Analyses
Page 19: February 2013 MEE Questions and Analyses
Page 20: February 2013 MEE Questions and Analyses
Page 21: February 2013 MEE Questions and Analyses
Page 22: February 2013 MEE Questions and Analyses
Page 23: February 2013 MEE Questions and Analyses
Page 24: February 2013 MEE Questions and Analyses
Page 25: February 2013 MEE Questions and Analyses
Page 26: February 2013 MEE Questions and Analyses
Page 27: February 2013 MEE Questions and Analyses
Page 28: February 2013 MEE Questions and Analyses
Page 29: February 2013 MEE Questions and Analyses
Page 30: February 2013 MEE Questions and Analyses
Page 31: February 2013 MEE Questions and Analyses
Page 32: February 2013 MEE Questions and Analyses
Page 33: February 2013 MEE Questions and Analyses
Page 34: February 2013 MEE Questions and Analyses
Page 35: February 2013 MEE Questions and Analyses
Page 36: February 2013 MEE Questions and Analyses
Page 37: February 2013 MEE Questions and Analyses
Page 38: February 2013 MEE Questions and Analyses
Page 39: February 2013 MEE Questions and Analyses
Page 40: February 2013 MEE Questions and Analyses
Page 41: February 2013 MEE Questions and Analyses
Page 42: February 2013 MEE Questions and Analyses
Page 43: February 2013 MEE Questions and Analyses
Page 44: February 2013 MEE Questions and Analyses
Page 45: February 2013 MEE Questions and Analyses
Page 46: February 2013 MEE Questions and Analyses
Page 47: February 2013 MEE Questions and Analyses